enVision Math Common Core Grade K Answer Key Topic 10 Compose and Decompose Numbers 11 to 19

Practice with the help of enVision Math Common Core Kindergarten Answer Key Topic 10 Compose and Decompose Numbers 11 to 19 regularly and improve your accuracy in solving questions.

enVision Math Common Core Grade K Answers Key Topic 10 Compose and Decompose Numbers 11 to 19

Essential Question: How can composing and decomposing numbers from 11 to 19 into ten ones and some further ones help you understand place value?
Envision Math Common Core Grade K Answer Key Topic 10 Compose and Decompose Numbers 11 to 19 1

envision STEM Project: Sunlight and Earth’s Surface
Directions Read the character speech bubbles to students. Find Out! Have students find out how sunlight affects Earth’s surface. Say: Talk to friends and relatives about sunlight and how it affects Earth. Journal: Make a Poster Have students make a poster that shows 3 things sunlight does for Earth. Have them draw a sun with 16 rays. Then have them write an equation for parts of 16.

Review What You Know

Question 1.
Envision Math Common Core Grade K Answer Key Topic 10 Compose and Decompose Numbers 11 to 19 1.1
Answer:

Explanation:
I circled the group that has 16 leafs.

Question 2.
Envision Math Common Core Grade K Answer Key Topic 10 Compose and Decompose Numbers 11 to 19 1.2
Answer:

Explanation:
I circled the group that has 20 leafs.

Question 3.
Envision Math Common Core Grade K Answer Key Topic 10 Compose and Decompose Numbers 11 to 19 1.3
Answer:

Explanation:
I circled the group that has less number of leafs.

Question 4.
Envision Math Common Core Grade K Answer Key Topic 10 Compose and Decompose Numbers 11 to 19 1.4
Answer:

Explanation:
There are 13 leafs in the above picture so, I wrote the number 13.

Question 5.
Envision Math Common Core Grade K Answer Key Topic 10 Compose and Decompose Numbers 11 to 19 1.5
Answer:

Explanation:
There are 17 leafs in the above picture so, I wrote the number 137

Question 6.
Envision Math Common Core Grade K Answer Key Topic 10 Compose and Decompose Numbers 11 to 19 1.6
Answer:

Explanation:
There are 15 leafs in the above picture so, I wrote the number 15.

Directions Have students: 1 draw a circle around the group with 16; 2 draw a circle around the group with 20; 3 draw a circle around the group that is less than the other group; 4-6 count the leaves, and then write the number to tell how many.

Pick a Project

A
Envision Math Common Core Grade K Answer Key Topic 10 Compose and Decompose Numbers 11 to 19 1.7

B
Envision Math Common Core Grade K Answer Key Topic 10 Compose and Decompose Numbers 11 to 19 1.8

Directions Say: You will choose one of these projects. Look at picture A. Think about this question: How great is the great outdoors? If you choose Project A, you will tell a camping story. Look at picture B. Think about this question: What do mice like to eat? If you choose Project B, you will make a mouse poster.

C
Envision Math Common Core Grade K Answer Key Topic 10 Compose and Decompose Numbers 11 to 19 1.9

D
Envision Math Common Core Grade K Answer Key Topic 10 Compose and Decompose Numbers 11 to 19 1.10

Directions Say: You will choose one of these projects. Look at picture C. Think about this question: What do you like to collect? If you choose Project C, you will make a sticker book. Look at picture D. Think about this question: What is in a granola bar? If you choose Project D, you will make a snack-time drawing.

Lesson 10.1 Make 11, 12, and 13

Solve & Share

Envision Math Common Core Grade K Answer Key Topic 10 Compose and Decompose Numbers 11 to 19 2.1
Answer:

Explanation:
I filled the ten-frame with counters and kept 2 counters outside the frame.
The equation that matches with the number of counters is 10+2=12.

Directions Say: Use counters to fill the ten-frame. Put 1, 2, or 3 counters outside of the ten-frame. Draw all of the counters. What equation can you write to tell how many counters there are in all?

Visual Learning Bridge
Envision Math Common Core Grade K Answer Key Topic 10 Compose and Decompose Numbers 11 to 19 2.2

Guided Practice

Question 1.
Envision Math Common Core Grade K Answer Key Topic 10 Compose and Decompose Numbers 11 to 19 2.3
Answer:

Directions 1 Have students write an equation to match the number of blocks shown. Then have them tell how the picture and equation show 10 ones and some more ones.

Question 2.
Envision Math Common Core Grade K Answer Key Topic 10 Compose and Decompose Numbers 11 to 19 2.4
Answer:

Explanation:
The equation that matches the number of blocks is 10+1=11.
The equation tells us that there are 10 ones and 1 more one.

Question 3.
Envision Math Common Core Grade K Answer Key Topic 10 Compose and Decompose Numbers 11 to 19 2.5
Answer:

Explanation:
The equation that matches the number of blocks is 10+3=13.
The equation tells us that there are 10 ones and 3 more ones.

Question 4.
Envision Math Common Core Grade K Answer Key Topic 10 Compose and Decompose Numbers 11 to 19 2.6
Answer:

Explanation:
I drew 12 counterss to match with the equation 10+2=12
The equation tells us that there are 10 ones and 2 more ones.

Question 5.
Envision Math Common Core Grade K Answer Key Topic 10 Compose and Decompose Numbers 11 to 19 2.7
Answer:

Explanation:
I drew 13 counters to match with the equation 10+3=13
The equation tells us that there are 10 ones and 3 more ones.

Directions Have students: 2 and 3 write an equation to match the number of blocks shown. Then have them tell how the picture and equation show 10 ones and some more ones; 4 and 5 draw blocks to match the equation. Then have them tell how the picture and equation show 10 ones and some more ones.

Independent Practice

Question 6.
Envision Math Common Core Grade K Answer Key Topic 10 Compose and Decompose Numbers 11 to 19 2.8
Answer:

Explanation:
I drew 13 counters to match with the equation 10+3=13
The equation tells us that there are 10 ones and 3 more ones.

Question 7.
Envision Math Common Core Grade K Answer Key Topic 10 Compose and Decompose Numbers 11 to 19 2.9
Answer:

Explanation:
I drew 11 counters to match with the equation 10+1=11
The equation tells us that there are 10 ones and 1 more one.

Question 8.
Envision Math Common Core Grade K Answer Key Topic 10 Compose and Decompose Numbers 11 to 19 3.1
Answer:

Explanation:
I drew 12 counters to match with the equation 10+2=12
The missing number is 2
The equation tells us that there are 10 ones and 2 more ones.

Question 9.
Envision Math Common Core Grade K Answer Key Topic 10 Compose and Decompose Numbers 11 to 19 3.2
Answer:

Explanation:
I drew 13 counters to match with the equation 10+3=13
The missing number is 3
The equation tells us that there are 10 ones and 3 more ones.

Directions Have students: 6 draw counters and write an equation to show how to make 13. Then have them tell how the picture and equation show 10 ones and some more ones; 7 draw counters and write an equation to show how to make 11. Then have them tell how the picture and equation show 10 ones and some more ones. 8 Algebra Have students draw counters to find the missing number. Then have them tell how the picture and equation show 10 ones and some more ones. 9 Higher Order Thinking Have students draw counters to find the missing number. Then have them tell how the picture and equation show 10 ones and some more ones.

Lesson 10.2 Make 14, 15 and 16

Solve & Share

Envision Math Common Core Grade K Answer Key Topic 10 Compose and Decompose Numbers 11 to 19 3.3
Answer:

Explanation:
I drew 15 counters to match with the equation 10+5=15
The equation tells us that there are 10 ones and 5 more ones.

Directions Say: Put 15 counters in the double ten-frame to show 10 ones and some more ones. Then complete the equation to match the counters.

Visual Learning Bridge
Envision Math Common Core Grade K Answer Key Topic 10 Compose and Decompose Numbers 11 to 19 3.4

Guided Practice

Question 1.
Envision Math Common Core Grade K Answer Key Topic 10 Compose and Decompose Numbers 11 to 19 3.5
Answer:

Directions 1 Have students write an equation to match the counters. Then have them tell how the picture and equation show 10 ones and some more ones.

Question 2.
Envision Math Common Core Grade K Answer Key Topic 10 Compose and Decompose Numbers 11 to 19 3.6
Answer:

Explanation:
I drew 15 counters to match with the equation 10+5=15
The equation tells us that there are 10 ones and 5 more ones.

Question 3.
Envision Math Common Core Grade K Answer Key Topic 10 Compose and Decompose Numbers 11 to 19 3.7
Answer:

Explanation:
I drew 16 counters to match with the equation 10+6=16
The equation tells us that there are 10 ones and 6 more ones.

Question 4.
Envision Math Common Core Grade K Answer Key Topic 10 Compose and Decompose Numbers 11 to 19 3.8
Answer:

Explanation:
I drew 14 counters to match with the equation 10+4=14
The equation tells us that there are 10 ones and 4 more ones.

Question 5.
Envision Math Common Core Grade K Answer Key Topic 10 Compose and Decompose Numbers 11 to 19 3.9
Answer:

Explanation:
I drew 15 counters to match with the equation 10+5=15
The equation tells us that there are 10 ones and 5 more ones.

Directions Have students: 2-3 write an equation to match the counters. Then have them tell how the picture and equation show 10 ones and some more ones; 4-5 draw counters to match the equation. Then have them tell how the picture and equation show 10 ones and some more ones.

Independent Practice

Question 6.
Envision Math Common Core Grade K Answer Key Topic 10 Compose and Decompose Numbers 11 to 19 4.1
Answer:

Explanation:
I drew 16 counters to match with the equation 10+6=16
The equation tells us that there are 10 ones and 6 more ones.

Question 7.
Envision Math Common Core Grade K Answer Key Topic 10 Compose and Decompose Numbers 11 to 19 4.2
Answer:

Explanation:
I drew 14 counters to match with the equation 10+4=14
The equation tells us that there are 10 ones and 4 more ones.

Question 8.
Envision Math Common Core Grade K Answer Key Topic 10 Compose and Decompose Numbers 11 to 19 4.3
Answer:

Explanation:
The equation 10+5=15 tells us that there are 10 ones and 5 more ones.

Question 9.
Envision Math Common Core Grade K Answer Key Topic 10 Compose and Decompose Numbers 11 to 19 4.4
Answer:

Explanation:
I drew 16 counters to match with the equation 10+6=16
The missing number is 6
The equation tells us that there are 10 ones and 6 more ones.

Directions Have students: 6 draw counters and write an equation to show how to make 16. Then have them tell how the picture and equation show 10 ones and some more ones; 7 draw counters and write an equation to show how to make 14. Then have them tell how the picture and equation show 10 ones and some more ones, 8 Number Sense Have students write an equation to show 15 as 10 ones and some more ones. 9 Higher Order Thinking Have students draw counters to find the missing number in the equation. Then have them tell how the picture and equation show 10 ones and some more ones.

Lesson 10.3 Make 17, 18, and 19

Solve & Share

Envision Math Common Core Grade K Answer Key Topic 10 Compose and Decompose Numbers 11 to 19 4.5
Answer:

Explanation:
I drew 18 counters to match with the equation 10+8=18
The equation tells us that there are 10 ones and 8 more ones.

Directions Say: Jada made 10 prizes for the school carnival. She makes 8 more. Use counters to show how many prizes Jada made in all. Then write an equation to match the counters, and tell how the counters and equation show 10 ones and some more ones.

Visual Learning Bridge
Envision Math Common Core Grade K Answer Key Topic 10 Compose and Decompose Numbers 11 to 19 4.6

Guided Practice

Question 1.
Envision Math Common Core Grade K Answer Key Topic 10 Compose and Decompose Numbers 11 to 19 4.7
Answer:

Directions 1 Have students complete the equation to match the counters. Then have them tell how the picture and equation show 10 ones and some more ones.

Question 2.
Envision Math Common Core Grade K Answer Key Topic 10 Compose and Decompose Numbers 11 to 19 4.8
Answer:

Explanation:
I counted the number of counters, there are 19 counters.
So, i wrote the equation 10+9=19
The equation tells us that there are 10 ones and 9 more ones.

Question 3.
Envision Math Common Core Grade K Answer Key Topic 10 Compose and Decompose Numbers 11 to 19 4.9
Answer:

Explanation:
I counted the number of counters, there are 18 counters.
So, i wrote the equation 10+8=18
The equation tells us that there are 10 ones and 8 more ones.

Question 4.
Envision Math Common Core Grade K Answer Key Topic 10 Compose and Decompose Numbers 11 to 19 4.10
Answer:

Explanation:
I counted the number of cubes, there are 17 cubes
So, i wrote the equation 10+7=17
The equation tells us that there are 10 ones and 7 more ones.

Question 5.
Envision Math Common Core Grade K Answer Key Topic 10 Compose and Decompose Numbers 11 to 19 4.11
Answer:

Explanation:
I counted the number of cubes, there are 19 cubes
So, i wrote the equation 10+9=19
The equation tells us that there are 10 ones and 9 more ones.

Directions Have students 2 and 3 write an equation to match the counters. Then have them tell how the picture and equation show 10 ones and some more ones; 4 and 5 complete the equation to match the cubes. Then have them tell how the picture and equation show 10 ones and some more ones.

Independent Practice

Question 6.
Envision Math Common Core Grade K Answer Key Topic 10 Compose and Decompose Numbers 11 to 19 5.1
Answer:

Explanation:
I drew 18 counters to match with the equation 10+8=18
The equation tells us that there are 10 ones and 8 more ones.

Question 7.
Envision Math Common Core Grade K Answer Key Topic 10 Compose and Decompose Numbers 11 to 19 5.2
Answer:

Explanation:
I drew 17 counters to match with the equation 10+7=17
The equation tells us that there are 10 ones and 7 more ones.

Question 8.
Envision Math Common Core Grade K Answer Key Topic 10 Compose and Decompose Numbers 11 to 19 5.3
Answer:

Explanation:
I drew 19 counters to match with the equation 10+9=19
The equation tells us that there are 10 ones and 9 more ones.

Question 9.
Envision Math Common Core Grade K Answer Key Topic 10 Compose and Decompose Numbers 11 to 19 5.4
Answer:

Explanation:
I drew 19 counters to match with the equation 10+9=19
The missing number is 9
The equation tells us that there are 10 ones and 9 more ones.

Directions Have students: 6 draw counters, and then write an equation to show how to make 18. Then have them tell how the picture and equation show 10 ones and some more ones; 7 draw counters, and then write an equation to show how to make 19. Then have them tell how the picture and equation show 10 ones and some more ones; 8 draw counters, and then write an equation to show how to make 17. Then have them tell how the picture and equation show 10 ones and some more ones. 9 Higher Order Thinking Have students draw counters to find the missing number in the equation. Then have them tell how the picture and equation show 10 ones and some more ones.

Lesson 10.4 Find Parts of 11, 12 and 13

Solve & Share

Envision Math Common Core Grade K Answer Key Topic 10 Compose and Decompose Numbers 11 to 19 5.5
Answer:

Explanation:
13 means 10+3
I drew 13 counters to match with the equation 13=10+3
The equation tells us that there are 10 ones and 3 more ones.

Directions Say: 13 students wait for the train. There are only 10 seats in each train car. How many students will have to ride in a second car? Use counters to show your work. Then tell how the counters and equation show 10 ones and some more ones.

Visual Learning Bridge

Envision Math Common Core Grade K Answer Key Topic 10 Compose and Decompose Numbers 11 to 19 5.6

Guided Practice

Question 1.
Envision Math Common Core Grade K Answer Key Topic 10 Compose and Decompose Numbers 11 to 19 5.7
Answer:

Directions 1 Have students use counters to show 11, draw them in the double ten-frame, and complete the equation to match the picture. Then have them tell how the picture and equation show 10 ones and some more ones.

Question 2.
Envision Math Common Core Grade K Answer Key Topic 10 Compose and Decompose Numbers 11 to 19 5.8
Answer:

Explanation:
13 means 10+3
I drew 13 counters to match with the equation 13=10+3
The equation tells us that there are 10 ones and 3 more ones.

Question 3.
Envision Math Common Core Grade K Answer Key Topic 10 Compose and Decompose Numbers 11 to 19 5.9
Answer:

Explanation:
There are 10 cubes and 2 more cubes.So, the missing numbers in the equation are 10 and 2
The equation tells us that there are 10 ones and 2 more ones.

Question 4.
Envision Math Common Core Grade K Answer Key Topic 10 Compose and Decompose Numbers 11 to 19 6.1
Answer:

Explanation:
I drew 11 counters to match with the equation 1=10+1
The equation tells us that there are 10 ones and 1 more one.

Directions Have students: 2 use counters to show 13, draw them in the double ten-frame, and complete the equation to match the picture. Then have them tell how the picture and equation show 10 ones and some more ones; 3 look at the picture of 12 cubes, and complete the equation to match the picture. Then have them tell how the picture and equation show 10 ones and some more ones; 4 draw counters to match the equation. Then have them tell how the picture and equation show 10 ones and some more ones.

Independent Practice

Question 5.
Envision Math Common Core Grade K Answer Key Topic 10 Compose and Decompose Numbers 11 to 19 6.2
Answer:

Explanation:
12 means 10+2
I drew 12 counters to match with the equation 12=10+2
The equation tells us that there are 10 ones and 2 more ones.

Question 6.
Envision Math Common Core Grade K Answer Key Topic 10 Compose and Decompose Numbers 11 to 19 6.3
Answer:

Explanation:
13 means 10+3
I counted the number of cunes, there are 13 cubes.13=10+3
The equation tells us that there are 10 ones and 3 more ones.

Question 7.
Envision Math Common Core Grade K Answer Key Topic 10 Compose and Decompose Numbers 11 to 19 6.4
Answer:

Explanation:
I drew 11 counters to match with the equation 11=10+1 and 10+1=11.
The equation tells us that there are 10 ones and 1 more ones.

Directions Have students: 5 draw counters to make 12, and complete the equation to match the picture. Then have them tell how the picture and equation show 10 ones and some more ones; 6 color the cubes blue and red to make 13, and complete the equation to match the picture. Then have them tell how the picture and equation show 10 ones and some more ones. 7 Higher Order Thinking Have students draw counters to show 11 and write two equations to match the picture. Then have them tell how the picture and equations show 10 ones and some more ones.

Lesson 10.5 Find Parts of 14, 15, and 16

Solve & Share

Envision Math Common Core Grade K Answer Key Topic 10 Compose and Decompose Numbers 11 to 19 6.5
Answer:

Explanation:
14 means 10+4
I drew 14 counters to match with the equation 14=10+4
The equation tells us that there are 10 ones and 4 more ones.

Directions Say: 14 students go to the zoo. The first bus takes 10 students. The rest of the students go on a second bus. Use counters to describe this situation. Then complete the equation to match the counters and tell how the counters and equation show 10 ones and some more ones.

Visual Learning Bridge
Envision Math Common Core Grade K Answers Topic 10 Compose and Decompose Numbers 11 to 19 6.6

Guided Practice

Question 1.
Envision Math Common Core Grade K Answers Topic 10 Compose and Decompose Numbers 11 to 19 6.7
Answer:

Directions 1 Have students use counters to show 15, draw them in the double ten-frame, and complete the equation to match the picture. Then have them tell how the picture and equation show 10 ones and some more ones.

Question 2.
Envision Math Common Core Grade K Answers Topic 10 Compose and Decompose Numbers 11 to 19 6.8
Answer:

Explanation:
14 means 10+4
I drew 14 counters to match with the equation 14=10+4
The equation tells us that there are 10 ones and 4 more ones.

Question 3.
Envision Math Common Core Grade K Answers Topic 10 Compose and Decompose Numbers 11 to 19 6.9
Answer:

Explanation:
16 means 10+6
I counted the number of cubes, there are 16 cubes.So, the equation is 16=10+6
The equation tells us that there are 10 ones and 6 more ones.

Question 4.
Envision Math Common Core Grade K Answers Topic 10 Compose and Decompose Numbers 11 to 19 6.10
Answer:

Explanation:
I drew 15 counters to match with the equation 15=10+5
The equation tells us that there are 10 ones and 5 more ones.

Directions Have students: 2 use counters to show 14, draw them in the double ten-frame, and complete the equation to match the picture. Then have them tell how the picture and equation show 10 ones and some more ones; 3 look at the picture of 16 cubes, and complete the equation to match the picture. Then have them tell how the picture and equation show 10 ones and some more ones; 4 draw counters to match the equation. Then have them tell how the picture and equation show 10 ones and some more ones.

Independent Practice

Question 5.
Envision Math Common Core Grade K Answers Topic 10 Compose and Decompose Numbers 11 to 19 6.11
Answer:

Explanation:
I drew 16 counters to match with the equation 16=10+6
The equation tells us that there are 10 ones and 6 more ones.

Question 6.
Envision Math Common Core Grade K Answers Topic 10 Compose and Decompose Numbers 11 to 19 6.12
Answer:

Explanation:
14=10+4
I colored 14 cubes to match with the equation 14=10+4
The equation tells us that there are 10 ones and 4 more ones.

Question 7.
Envision Math Common Core Grade K Answers Topic 10 Compose and Decompose Numbers 11 to 19 7.1
Answer:

Explanation:
I drew 16 counters to match with the equation 16=10+6 or 10+6=16
The equation tells us that there are 10 ones and 6 more ones.

Directions Have students 5 draw counters to match the equation. Then have them tell how the picture and equation show 10 ones and some more ones. 6 color the cubes blue and red to show 14, complete the equation to match the picture, and tell how the picture and equation show 10 ones and some more ones. 7 Higher Order Thinking Have students use counters to show 16, draw them in the double ten-frame, and complete two equations to match the picture. Then have them tell how the picture and equations show 10 ones and some more ones.

Lesson 10.6 Find Parts of 17, 18 and 19

Solve & Share

Envision Math Common Core Grade K Answers Topic 10 Compose and Decompose Numbers 11 to 19 7.2
Answer:

Explanation:
I colored 10 boxes blue and the remaining 8 boxes red,
The equation is 10+8=18
The equation tells us that there are 10 ones and 5 more ones.

Directions Say: How can these 18 boxes be split into ten ones and some more ones? Use 2 different color crayons to color the boxes to show your work. Then write an equation to match the picture.

Visual Learning Bridge
Envision Math Common Core Grade K Answers Topic 10 Compose and Decompose Numbers 11 to 19 7.3

Guided Practice

Question 1.
Envision Math Common Core Grade K Answers Topic 10 Compose and Decompose Numbers 11 to 19 7.4
Answer:

Directions 1 Have students color 10 cubes blue to show 10 ones, and then draw 10 blue cubes in the top ten-frame. Have them color the remaining cubes in the train red to show more ones, count them, and then draw red cubes in the bottom ten-frame. Then have them write an equation to match the pictures.

Question 2.
Envision Math Common Core Grade K Answers Topic 10 Compose and Decompose Numbers 11 to 19 7.5
Answer:

Explanation:
I colored 10 squares blue to show 10 ones, and then drew 10 blue squares in the top ten-frame.Then i colored the remaining cubes in the train red to show more ones, counted them, and then drew 9 red squares in the bottom ten-frame. Then i wrote the equation 19=10+9 to match the pictures.

Question 3.
Envision Math Common Core Grade K Answers Topic 10 Compose and Decompose Numbers 11 to 19 7.6
Answer:

Explanation:
I colored 10 squares blue to show 10 ones, and then drew 10 blue squares in the top ten-frame.Then i colored the remaining cubes in the train red to show more ones, counted them, and then drew 7 red squares in the bottom ten-frame. Then i wrote the equation 19=10+7 to match the pictures.

Question 4.
Envision Math Common Core Grade K Answers Topic 10 Compose and Decompose Numbers 11 to 19 7.7
Answer:

Explanation:
I counted the above counters, there are 18 counters.So, the equation is 18=10+8
The equation tells us that there are 10 ones an d8 more ones.

Directions Have students: 2 and 3 color 10 squares blue to show 10 ones, and then draw 10 blue squares in the top ten-frame. Have them color the remaining cubes in the train red to show more ones, count them, and then draw red squares in the bottom ten-frame. Then have them write an equation to match the pictures; 4 complete the equation to match the counters. Then have them tell how the picture and equation show 10 ones and some more ones.

Independent Practice

Question 5.
Envision Math Common Core Grade K Answers Topic 10 Compose and Decompose Numbers 11 to 19 7.8
Answer:

Explanation:
I counted the above counters, there are 17 counters.So, the equation is 17=10+7
The equation tells us that there are 10 ones and 7 more ones.

Question 6.
Envision Math Common Core Grade K Answers Topic 10 Compose and Decompose Numbers 11 to 19 7.9
Answer:

Explanation:
I counted the above counters, there are 19 counters.So, the equation is 19=10+9
The equation tells us that there are 10 ones and 9 more ones.

Question 7.
Envision Math Common Core Grade K Answers Topic 10 Compose and Decompose Numbers 11 to 19 8.1
Answer:

Explanation:
I drew 18 counters to match with the equation 18=10+8 or 10+8=18
The equation tells us that there are 10 ones and 8 more ones.

Directions 5 and 6 Have students complete the equation to match the counters. Then have them tell how the picture and equation show 10 ones and some more ones. 7 Higher Order Thinking Have students use counters to show 18, draw them in the double ten-frame, and write two equations to match the picture. Then have them tell how the picture and equations show 10 ones and some more ones.

Lesson 10.7 Look For and Use Structure

Problem Solving

Solve & Share

Envision Math Common Core Grade K Answers Topic 10 Compose and Decompose Numbers 11 to 19 9.1
Answer:

Explanation:
I kept 2 counters in the red five-frame. Used a red crayon and wrote the number 2 that tells how many counters are in the red frame. I kept the same number of counters in the blue five-frame. Used a blue crayon and write the number that tells how many counters are in the blue frames.
The red nmber is smaller by ten then the blue number,
The pattern is 2,12.

Directions Say: Put some counters in the red five-frame. Use a red crayon and write the number that tells how many counters are in the red frame. Put the same number of counters in the blue five-frame. Use a blue crayon and write the number that tells how many counters are in the blue frames. Show the numbers to a partner. Compare your answers and look for patterns. How is your blue number like your red number? How is it different?

Visual Learning Bridge
Envision Math Common Core Grade K Answers Topic 10 Compose and Decompose Numbers 11 to 19 9.2

Guided Practice

Question 1.
Envision Math Common Core Grade K Answers Topic 10 Compose and Decompose Numbers 11 to 19 9.3
Answer:

Directions 1 Have students find the number with the blue box around it, and then color the number that is 10 greater than the number in the blue box. Have them write an equation to show how the teen number they colored is composed of 10 ones and some more ones. Then have students explain how they decided what parts to add to make the teen number.

Independent Practice

Question 2.
Envision Math Common Core Grade K Answers Topic 10 Compose and Decompose Numbers 11 to 19 9.4
Answer:

Explanation:
I found the number with the blue box around it, it is 7 and then colored the number that is 10 greater than the number in the blue box which is 17.Then wrote an 10+7=17 equation to show how the teen number i colored is composed of 10 ones and 7 more ones.

Question 3.
Envision Math Common Core Grade K Answers Topic 10 Compose and Decompose Numbers 11 to 19 9.5
Answer:

Explanation:
I found the number with the blue box around it, it is 8 and then colored the number that is 10 greater than the number in the blue box which is 18.Then wrote an 10+8=18 equation to show how the teen number i colored is composed of 10 ones and 8 more ones.

Question 4.
Envision Math Common Core Grade K Answers Topic 10 Compose and Decompose Numbers 11 to 19 9.6
Answer:

Explanation:
I found the number with the blue box around it, it is 9 and then colored the number that is 10 greater than the number in the blue box which is 19.Then wrote an 10+9=19 equation to show how the teen number i colored is composed of 10 ones and 9 more ones.

Question 5.
Envision Math Common Core Grade K Answers Topic 10 Compose and Decompose Numbers 11 to 19 9.7
Answer:

Explanation:
The missing numbers in the pattern are 10 and 3.
The equaation is 10+3=13.

Directions Have students: 2-4 find the number with the blue box around it, and color the number that is 10 greater than the number in the blue box. Then have them write an equation to show how the teen number they colored is composed of 10 ones and some more ones; 5 complete the equation to continue the pattern, and then explain the pattern they made.

Problem Solving

Performance Task

Envision Math Common Core Grade K Answers Topic 10 Compose and Decompose Numbers 11 to 19 9.8
Answer:

Directions Read the problem to students. Then have them use multiple problem-solving methods to solve the problem. Say: Mr. Shepard’s class will exchange cards at a holiday party. There are 16 students in the class. The store sells cards in packs of 10. Alex already has 6 cards. Marta already has 7 cards. How many cards will Alex and Marta have after they each buy one pack of cards? 6 Use Structure How can the number chart help you solve the problem? Write the equations for the number of cards Alex and Marto will have. 7 Generalize After you find the number of cards Alex will have, is it easier to find the number of cards Marta will have? 8 Explain Tell a friend why your answers are correct. Then tell the friend about the pattern you see in the number chart and how the equations show 10 ones and some more ones.

Topic 10 Fluency Practice

Find a Match

Activity

Question 1.
Envision Math Common Core Grade K Answers Topic 10 Compose and Decompose Numbers 11 to 19 9.9
Answer:

Explanation:
The clues are 2+3=5=4+1=O, 4-2=2=1+1=G, 5-2=3=4-1=H
I solved the addition and subtraction problems in the above picture and with the help of the clues i found the word HOG.

Question 2.
Envision Math Common Core Grade K Answers Topic 10 Compose and Decompose Numbers 11 to 19 10.1
Answer:

Explanation:
The clues are 2-1=1=5-4=W, 2+2=1+3=4=C, 1-1=0=0+0=O
I solved the addition and subtraction problems in the above picture and with the help of the clues i found the word COW.

Directions 1 and 2 Have students find a partner. Have them point to a clue in the top row, and then solve the addition or subtraction problem. Then have them look at the clues in the bottom row to find a match, and then write the clue letter above the match. Have students find a match for every clue.

Topic 10 Vocabulary Review

Question 1.
Envision Math Common Core Grade K Answers Topic 10 Compose and Decompose Numbers 11 to 19 10.2
Answer:

Question 2.
Envision Math Common Core Grade K Answers Topic 10 Compose and Decompose Numbers 11 to 19 10.3
Answer:

Directions Understand Vocabulary Have students: 1 complete the drawing and the equation to show how many more counters are needed to make 15; 2 complete the drawing and the equation to show how many more counters are needed to make 19.

Topic 10 Reteaching

Set A
Envision Math Common Core Grade K Answers Topic 10 Compose and Decompose Numbers 11 to 19 11.1

Question 1.
Envision Math Common Core Grade K Answers Topic 10 Compose and Decompose Numbers 11 to 19 11.2
Answer:

Explanation:
There are 13 cubres in the above picture.
The equation that matches with the image is 10+3=13
The equation tells us that there are 10 ones and 3 more ones.

Set B
Envision Math Common Core Grade K Answers Topic 10 Compose and Decompose Numbers 11 to 19 11.3

Question 2.
Envision Math Common Core Grade K Answers Topic 10 Compose and Decompose Numbers 11 to 19 11.4
Answer:

Explanation:
I drew 6 more counters to show 16,
The equation 10+6=16 tells us that there are 10 ones and 6 more ones.

Directions Have students: 1 write an equation to match the blocks. Then have them tell how the picture and equation show 10 ones and some more ones; 2 draw counters to show 16, and then write an equation to match the picture. Then tell how the picture and equation show 10 ones and some more ones.

Set C
Envision Math Common Core Grade K Answer Key Topic 10 Compose and Decompose Numbers 11 to 19 100

Question 3.
Envision Math Common Core Grade K Answer Key Topic 10 Compose and Decompose Numbers 11 to 19 101
Answer:

Explanation:
I drew 17 counters to match with the equation 10+7=17
The equation 10+7=17 tells us that there are 10 ones and 7 more ones.

Set D
Envision Math Common Core Grade K Answer Key Topic 10 Compose and Decompose Numbers 11 to 19 102

Question 4.
Envision Math Common Core Grade K Answer Key Topic 10 Compose and Decompose Numbers 11 to 19 103
Answer:

Explanation:
I drew 11 counters to match with the equation 10+1=11
The equation tells us that there are 10 ones and 1 more ones.

Directions Have students: 3 draw counters to match the equation. Then have them tell how the picture and equation show 10 ones and some more ones; 4 draw counters to make 11, and then complete the equation to match the picture. Then have them tell how the picture and equation show 10 ones and some more ones.

Set E
Envision Math Common Core Grade K Answers Topic 10 Compose and Decompose Numbers 11 to 19 11.5

Question 5.
Envision Math Common Core Grade K Answers Topic 10 Compose and Decompose Numbers 11 to 19 11.6
Answer:

Explanation:
I drew 14 counters to match with the equation 10+4=14
The missing numbers in the equation are 10 and 4.
The equation tells us that there are 10 ones and 4 more ones.

Set F
Envision Math Common Core Grade K Answers Topic 10 Compose and Decompose Numbers 11 to 19 11.7

Question 6.
Envision Math Common Core Grade K Answers Topic 10 Compose and Decompose Numbers 11 to 19 11.8
Answer:

Explanation:
I found the number with the blue box around it, it is 8 and then colored the number that is 10 greater than the number in the blue box which is 18.Then wrote an 10+8=18 equation to show how the teen number i colored is composed of 10 ones and 8 more ones.

Directions Have students: 5 use counters to show 14, draw them in the double ten-frame, and complete the equation to match the picture. Then have them tell how the picture and equation show 10 ones and some more ones; 6 find the number with the blue box around it, and color the number that is 10 greater than the number in the blue box. Then have them write an equation to match, and then tell how the equation shows 10 ones and some more ones.

Set G
Envision Math Common Core Grade K Answers Topic 10 Compose and Decompose Numbers 11 to 19 11.9

Question 7.
Envision Math Common Core Grade K Answers Topic 10 Compose and Decompose Numbers 11 to 19 12.1
Answer:

Explanation:
I colored 10 cubes blue in the train to show 10 ones, and then drew 10 blue cubes in the top ten-frame.Then colored the remaining 8 cubes in the train red to show 8 more ones, I counted them, and then draw the same number of red cubes in the bottom ten-frame. Then I wrote an equation 18= 10 + 8  to match the pictures.

Directions Have students: 7 color 10 cubes blue in the train to show 10 ones, and then draw 10 blue cubes in the top ten-frame. Have them color the remaining cubes in the train red to show more ones, count them, and then draw the same number of red cubes in the bottom ten-frame. Then have them write an equation to match the pictures.

Topic 10 Assessment Practice

Question 1.
Envision Math Common Core Grade K Answers Topic 10 Compose and Decompose Numbers 11 to 19 12.20
A. 15 = 10 + 5
B. 14 = 10 + 4
C. 13 = 10 + 3
D. 12 = 10 + 2
Answer:

Explanation:
Option A is correct as there are 15 counters in the above ten frames which tell that there are 10 ones and 5 more ones.

Question 2.
Envision Math Common Core Grade K Answers Topic 10 Compose and Decompose Numbers 11 to 19 12.3
A. 10 and 6
B. 10 and 7
C. 10 and 8
D. 10 and 9
Answer:

Explanation:
Option C is correct as there are 18 counters in the above ten frames which tell that there are 10 ones and 8 more ones.

Question 3.
A 10 and 0
B 10 and 1
C 10 and 2
D 10 and 3
Answer:

Explanation:
Option C is correct as there are 12 counters in the above ten frames which tell that there are 10 ones and 2 more ones.

Directions Have students mark the best answer. 1 Say: Mason uses counters in ten-frames to count his marbles. Which equation matches the picture and shows how many marbles Mason has? 2 Say: Sarah counts the number of counters and gets 18. Which two numbers add to 18? Use the equation and double ten-frame for help. 3 Say: Cole has 12 toy trucks. How can Cole split up his trucks into ten ones and some more ones?

Question 4.
Envision Math Common Core Grade K Answers Topic 10 Compose and Decompose Numbers 11 to 19 20.1
Answer:

Explanation:
I found the number with the blue box around it, it is 4 and then colored the number that is 10 greater than the number in the blue box which is 14.Then wrote an 10+4=14 equation to show how the teen number i colored is composed of 10 ones and 4 more ones.

Question 5.
Envision Math Common Core Grade K Answers Topic 10 Compose and Decompose Numbers 11 to 19 20.2
Answer:

Explanation:
I drew 3 more counters to match with the equation 13 = 10 + 3
The equation tells that there are 10 ones and 3 more ones.

Directions Have students: 4 find the number with the blue box around it, and then color the number that is 10 greater than the number in the blue box. Then have them write an equation that shows how the teen number they colored is composed of ten and some more ones; 5 draw counters to make 13, and then complete the equation to match the picture.

Topic 10 Assessment Practice

Question 6.
Envision Math Common Core Grade K Answers Topic 10 Compose and Decompose Numbers 11 to 19 20.3
Answer:

Explanation:
I drew 6 more counters to match with the equation 16 = 10 + 6
The equation tells that there are 10 ones and 6 more ones.

Question 7.
Envision Math Common Core Grade K Answers Topic 10 Compose and Decompose Numbers 11 to 19 20.4
Answer:

Explanation:
I colored 10 cubes blue in the train to show 10 ones, and then drew 10 blue cubes in the top ten-frame.Then colored the remaining 9 cubes in the train red to show 9 more ones, I counted them, and then draw the same number of red cubes in the bottom ten-frame. Then I wrote an equation 18= 10 + 9  to match the pictures

 

Directions Have students: 6 listen to this story: Gabby has 16 counters. She wants to put her counters into a double ten-frame in order to decompose 16 into tens and ones. Draw counters to match Gabby’s equation. 7 color 10 cubes blue to show 10 ones, and then draw 10 blue cubes in the top ten-frame. Have them color the remaining cubes in the train red to show more ones, count them, and then draw the same number of red cubes in the bottom ten-frame. Then have them write an equation to match the pictures.

Question 8.
Envision Math Common Core Grade K Answers Topic 10 Compose and Decompose Numbers 11 to 19 20.5
Answer:

Explanation:
In the first double ten-frame there are 13 counters, i matched it with the equation 13=10+3
In the Second double ten-frame there are 1 counters, i matched it with the equation 17=10+7
In the third double ten-frame there are 1 counters, i matched it with the equation 11=10+1
In the forth double ten-frame there are 1 counters, i matched it with the equation 14=10+4

Directions 8 Have students choose the equation that matches each double ten-frame.

Question 1.
Envision Math Common Core Grade K Answers Topic 10 Compose and Decompose Numbers 11 to 19 21.1
Answer:

Explanation:
There are 12 marbles in the above ten frame.
I wrote the euqation 10+2=12 to match with the picture.
The equation tll that there are 10 ones and 2 mor eones.

Question 2.
Envision Math Common Core Grade K Answers Topic 10 Compose and Decompose Numbers 11 to 19 21.2
Answer:

Explanation:
I drew 8 more marbles to match with the equation 18 = 10 + 8
The equation tells that there are 10 ones and 8 more ones.

Question 3.
Envision Math Common Core Grade K Answers Topic 10 Compose and Decompose Numbers 11 to 19 21.4
Answer:

Explanation:
I drew 17 yellow marbles to match with the equation 17 = 10 + 7
The equation tells that there are 10 ones and 7 more ones.

Directions Mason’s Marbles Say: Mason collects many different kinds of marbles. He uses ten-frames to help count his marbles. Have students: 1 write the equation to show how many purple marbles Mason has; 2 draw red marbles in the second ten-frame to show 18 red marbles in all, and then complete the equation. Have them tell how the picture and equation show 10 ones and some more ones; 3 draw 17 yellow marbles in the double ten-frame, and then write two equations to match their drawing.

Question 4.
Envision Math Common Core Grade K Answers Topic 10 Compose and Decompose Numbers 11 to 19 23.1
Answer:

Explanation:
I drew 13 green marbles to match with the equation 13 = 10 + 3
The equation tells that there are 10 ones and 3 more ones.

 

Question 5.
Envision Math Common Core Grade K Answers Topic 10 Compose and Decompose Numbers 11 to 19 53.2
Answer:

Explanation:
I found the number with the blue box around it, it is 4 and then colored the number that is 10 greater than the number in the blue box which is 14.Then wrote an 10+4=14 equation to show how the teen number i colored is composed of 10 ones and 4 more ones.

Directions 4 Have students look at the equation Mason wrote to show how many green marbles he has, and then draw the marbles in the double ten-frame to show the number. Have them tell how the picture shows 10 ones and some more ones. 5 Say: Mason put his striped marbles in a five-frame. Then he buys 10 more striped marbles. Have students write the number to tell how many striped marbles Mason had at first, and then color the part of the number chart to show how many striped marbles he has now. Then have them write an equation and ask them to explain how the picture and equation show 10 ones and some more ones.

Envision Math Common Core Grade K Answer Key Topic 7 Understand Subtraction

Practice with the help of enVision Math Common Core Kindergarten Answer Key Topic 7 Understand Subtraction regularly and improve your accuracy in solving questions.

Envision Math Common Core Grade K Answers Key Topic 7 Understand Subtraction

Essential Question:
How can represent taking apart and taking from in different ways help you learn about subtraction?

enVision STEM Project: Animal Needs
Directions Read the character speech bubbles to students. Find Out! Have students find out about how plants, animals, and humans use their environment to meet basic needs such as food, water, nutrients, sunlight, space, and shelter. Say: Different organisms need different things. Talk to friends and relatives about the different needs of plants, animals, and humans, and how different organisms meet those needs. Journal: Make a Poster Have students make a poster. Ask them to draw as many as 5 pictures of a human’s needs and as many as 5 pictures of an animal’s needs. Have them cross out the needs that are the same for humans and animals, and then write how many are left.
Envision Math Common Core Grade K Answer Key Topic 7 Understand Subtraction q 1

Review What You Know

Directions Have students: 1 draw a circle around the plus sign; 2 draw a circle around the equal sign; 3 draw a circle around the sum; 4 – 5 count the objects in each group, and then write the equation to tell how many in all.

Question 1.
3 + 6 = 9
Answer:

Explanation:
I drew a circle around the plus (+) sign.

Question 2.
4 + 1 = 5
Answer:

Explanation:
I drew a circle around the equal (=) sign.

Question 3.
2 + 5 = 7
Answer:

Explanation:
I drew a circle around the sum of the numbers 2 and 5 that is 7.

Question 4.
Envision Math Common Core Grade K Answer Key Topic 7 Understand Subtraction q 2
Answer:

Explanation:
There are 3 counters in first group and 1 counter in second group. Therefore the addition equation is 3+1=4.

Question 5.
Envision Math Common Core Grade K Answer Key Topic 7 Understand Subtraction q 3
Answer:

Explanation:
There are 5 counters in first group and 3 counter in second group. Therefore the addition equation is 5+3=8.

Question 6.
Envision Math Common Core Grade K Answer Key Topic 7 Understand Subtraction q 4
Answer:

Explanation:
There are 6 squares in first group and 4 squares in second group. Therefore the addition equation is 6+4=10.

Pick A Project

Directions Say: You will choose one of these projects. Look at picture A. Think about this question: What jobs can you do? If you choose Project A, you will make a jobs chart. Look at picture B. Think about this question: If you had a garden, how many flowers would you plant? If you choose Project B, you will sing a song about flowers. Look at picture C. Think about this question: Do stars have different colors? If you choose Project C, you will draw a picture of different-colored stars and tell a number story.

Envision Math Common Core Grade K Answer Key Topic 7 Understand Subtraction q 5

Lesson 7.1 Explore Subtraction

Solve & Share

Directions Say: Marta sees 5 goldfish in the pond. I swims away. How many fish are left? Think about the problem in your head. Then act out the story with your fingers. Use counters to show how many are left. Write numbers to explain.

I can ………….. show numbers in many ways.
Envision Math Common Core Grade K Answer Key Topic 7 Understand Subtraction q 6

Visual Learning Bridge

Envision Math Common Core Grade K Answer Key Topic 7 Understand Subtraction q 7

Guided Practice

Directions Have students listen to the story, and then do all of the following to find how many are left: give an explanation of a mental image, use objects to act it out, and hold up fingers. Have them mark Xs on how many birds fly away, and then write the number to tell how many are left.1 8 eagles sit on a branch. 2 fly away. How many eagles are left? 2 5 blue jays hop on the ground. I flies away. How many blue jays are left?

Question 1.
Envision Math Common Core Grade K Answer Key Topic 7 Understand Subtraction q 8
Answer:

Explanation:
8 eagles sit on a branch. 2 fly away.I marked ‘X’ on 2 birds. Therefore 6 birds are left.

Question 2.
Envision Math Common Core Grade K Answer Key Topic 7 Understand Subtraction q 9
Answer:

Explanation:
5 blue jays hop on the ground. I flies away. I marked ‘X’ on 1 bird. Therefore 4 birds are left.

Directions Have students listen to the story, and then do all of the following to find how many are left: give an explanation of a mental image, use objects to act it out, and hold up fingers. Have them mark Xs on how many walk away or are taken out, and then write the number to tell how many are left. 3 9 ladybugs are on a leaf. 4 walk away. How many ladybugs are left? 4 7 caterpillars are on a leaf. 6 walk away. How many caterpillars are left? 5 9 marbles are in a jar. 3 are taken out. How many marbles are left? 6 6 marbles are in a jar. 4 are taken out. How many marbles are left?

Question 3.
Envision Math Common Core Grade K Answer Key Topic 7 Understand Subtraction q 10
Answer:

Explanation:
9 ladybugs are on a leaf. 4 walk away. I marked ‘X’ on 4 bugs. Therefore 5 bugs are left on the leaf.

Question 4.
Envision Math Common Core Grade K Answer Key Topic 7 Understand Subtraction q 11
Answer:

Explanation:
7 caterpillars are on a leaf. 6 walk away. I marked ‘X’ on 6 Caterpillars. Therefore 1 is left on the leaf.

Question 5.
Envision Math Common Core Grade K Answer Key Topic 7 Understand Subtraction q 12
Answer:

Explanation:
9 marbles are in a jar. 3 are taken out. I marked ‘X’ on 3 marbles that are taken out. Therefore 6 marbles are left in the jar.

Question 6.
Envision Math Common Core Grade K Answer Key Topic 7 Understand Subtraction q 13
Answer:

Explanation:
6 marbles are in a jar. 4 are taken out. I marked ‘X’ on 4 marbles that are taken out. Therefore 2 marbles are left in the jar.

Independent Practice

Directions Have students listen to the story, and then do all of the following to find how many are left: give an explanation of a mental image, use objects to act it out, and hold up fingers. Ask them to write the number to tell how many are left. 7 10 fingers are in the air. 2 are put down. How many fingers are left? 8 7 fingers are in the air. 3 are put down. How many fingers are left? 9 Have students listen to the story, and then do all of the following to find how many are left: give an explanation of a mental image, use objects to act it out, and then mark Xs on how many are taken away. Ask them to write the number to tell how many are left. There are 9 marbles. 6 are taken away. How many marbles are left? 10 Higher Order Thinking Have students draw 10 marbles. Have them mark Xs on some of them, and then write the number to tell how many marbles are left.

Question 7.
Envision Math Common Core Grade K Answer Key Topic 7 Understand Subtraction q 14
Answer:

Explanation:
10 fingers are in the air. 2 are put down. So, 8 fingers are left in the air.

Question 8.
Envision Math Common Core Grade K Answer Key Topic 7 Understand Subtraction q 15

Answer:

Explanation:
7 fingers are in the air. 3 are put down. So, 4 fingers are left in the air.

Question 9.
Envision Math Common Core Grade K Answer Key Topic 7 Understand Subtraction q 16
Answer:

Explanation:
There are 9 marbles. 6 are taken away.I marked ‘X’ on 6 marbles that are taken out. Therefore 3 marbles are left.

Question 10.
Envision Math Common Core Grade K Answer Key Topic 7 Understand Subtraction q 17
Answer:

Explanation:
I drew 10 marbles taken away 6 from them.I marked ‘X’ on 6 marbles that are taken out. Therefore 4 marbles are left.

Lesson 7.2 Represent Subtraction as Taking Apart

Solve & Share
Directions Say: Alex picks 7 apples. Some apples are red, and some are yellow. Alex wants to put the red apples in one basket and the yellow in the other. How many red apples and how many yellow apples can there be? Use counters to show the red and yellow apples, and write the numbers to tell how many of each. Draw pictures to show your answer.

I can ………… take apart a number and tell the parts.
Envision Math Common Core Grade K Answer Key Topic 7 Understand Subtraction q 18

Visual Learning Bridge

Envision Math Common Core Grade K Answer Key Topic 7 Understand Subtraction q 19

Guided Practice

Directions Have students: 1 take apart the group of pears. Then have them draw a circle around the parts they made, and then write the numbers to tell the parts; 2 take apart the group of peaches. Then have them draw a circle around the parts they made, and then write the numbers to tell the parts.

Question 1.
Envision Math Common Core Grade K Answer Key Topic 7 Understand Subtraction q 20
Answer:

Explanation:
I took apart the group of pears. Then i drew a circle around the parts i made, and then wrote the numbers 4 and 1 to tell the parts.

Question 2.
Envision Math Common Core Grade K Answer Key Topic 7 Understand Subtraction q 21
Answer:

Explanation:
I took apart the group of peachess. Then i drew a circle around the parts i made, and then wrote the numbers 2 and 4 to tell the parts.

Directions 3 – 6 Have students take apart the group of fruit. Then have them draw a circle around the ports they made, and then write the numbers to tell the parts.

Question 3.
Envision Math Common Core Grade K Answer Key Topic 7 Understand Subtraction q 22
Answer:

Explanation:
I took apart the group of fruits. Then i drew a circle around the parts i made, and then wrote the numbers 1 and 3 to tell the parts.

Question 4.
Envision Math Common Core Grade K Answer Key Topic 7 Understand Subtraction q 23
Answer:

Explanation:
I took apart the group of fruits. Then i drew a circle around the parts i made, and then wrote the numbers 4 and 6 to tell the parts.

Question 5.
Envision Math Common Core Grade K Answer Key Topic 7 Understand Subtraction q 24
Answer:

Explanation:
I took apart the group of fruits. Then i drew a circle around the parts i made, and then wrote the numbers 2 and 1 to tell the parts.

Question 6.
Envision Math Common Core Grade K Answer Key Topic 7 Understand Subtraction q 25
Answer:

Explanation:
I took apart the group of fruits. Then i drew a circle around the parts i made, and then wrote the numbers 3 and 2 to tell the parts.

Independent Practice

Directions 7 and 8 Have students take apart the group of fruit. Then have them draw a circle around the parts they made, and then write the numbers to tell the parts. 9 Higher Order Thinking Have students draw counters to show a group of 5. Then have them take apart the group of counters, draw a circle around the parts they made, and then write the numbers to tell the parts. 10 Higher Order Thinking Have students choose any number between 2 and 10, write that number on the top line, and then draw a group of counters to show that number. Have them take apart the group of counters, draw a circle around the parts they made, and then write the numbers to tell the parts.

Question 7.
Envision Math Common Core Grade K Answer Key Topic 7 Understand Subtraction q 26
Answer:

Explanation:
I took apart the group of fruits. Then i drew a circle around the parts i made, and then wrote the numbers 3 and 3 to tell the parts.

Question 8.
Envision Math Common Core Grade K Answer Key Topic 7 Understand Subtraction q 27
Answer:

Explanation:
I took apart the group of fruits. Then i drew a circle around the parts i made, and then wrote the numbers 1 and 1 to tell the parts.

Question 9.
Envision Math Common Core Grade K Answer Key Topic 7 Understand Subtraction q 28
Answer:

Explanation:
I drew counters to show a group of 5.I took apart the group of counters. Then i drew a circle around the parts i made, and then wrote the numbers 3 and 2 to tell the parts.

Question 10.
Envision Math Common Core Grade K Answer Key Topic 7 Understand Subtraction q 29
Answer:

Explanation:
I choosed a number between 2 and 10, wrote that number 6 on the top line, and then drew a group of 6 counters to show that number.I took apart the group of counters. Then i drew a circle around the parts i made, and then wrote the numbers 2 and 4 to tell the parts.

Lesson 7.3 Represent Subtraction as Taking From

Solve & Share
Directions Say: Marto is watching bugs. She sees q ladybugs together in a group. Then some crawl away. Look at the picture and decide how many are left. Use counters to show what you think happens in the story. Then write numbers to tell how many ladybugs are left ¡n the group.

I can …………. represent subtraction as taking away from a whole.
Envision Math Common Core Grade K Answer Key Topic 7 Understand Subtraction q 30

Visual Learning Bridge

Envision Math Common Core Grade K Answer Key Topic 7 Understand Subtraction q 31

Guided Practice

Directions Have students listen to each story, and then complete the sentence to tell how many bugs are left. 1 Marta sees 6 bumblebees. 3 leave. How many bumblebees are left? 2 Marta sees 7 ladybugs. 2 leave. How many ladybugs are left?

Question 1.
Envision Math Common Core Grade K Answer Key Topic 7 Understand Subtraction q 32
Answer:

Explanation:
Marta sees 6 bumblebees. 3 leave. Therefore 3 bees are left. So, i wrote sentence is 6 takeaway 3 is 3.

Question 2.
Envision Math Common Core Grade K Answer Key Topic 7 Understand Subtraction q 33
Answer:

Explanation:
Marta sees 7 ladybugs. 2 leave. Therefore 5 bugs are left. So, i wrote the sentence 7 takeaway 2 is 5.

Directions Have students listen to each story, and then complete the sentence to tell how many bugs are left. 3 Emily sees 6 grasshoppers on the table. 2 hop away. How many grasshoppers are left? 4 Emily sees 7 dragonflies. 3 fly away. How many dragonflies are left? 5 Emily sees 8 caterpillars resting on a branch. 4 crawl away. How many caterpillars are left? 6 enVision® STEM Say: Ants can move material much bigger than themselves. Emily sees 10 ants on a picnic blanket. 4 walk away. How many ants are left?

Question 3.
Envision Math Common Core Grade K Answer Key Topic 7 Understand Subtraction q 34
Answer:

Explanation:
Emily sees 6 grasshoppers on the table. 2 hop away. Therefore 4 grasshoppers are left. So, i wrote the sentence 6 takeaway 2 is 4.

Question 4.
Envision Math Common Core Grade K Answer Key Topic 7 Understand Subtraction q 35
Answer:

Explanation:
Emily sees 7 dragonflies. 3 fly away. Therefore 4 flies are left. So, i wrote the sentence 7 takeaway 3 is 4.

Question 5.
Envision Math Common Core Grade K Answer Key Topic 7 Understand Subtraction q 36
Answer:

Explanation:
Emily sees 8 caterpillars resting on a branch. 4 crawl away. Therefore 4 Caterpillars are left. So, i wrote the sentence 8 takeaway 4 is 4.

Question 6.
Envision Math Common Core Grade K Answer Key Topic 7 Understand Subtraction q 37
Answer:

Explanation:
Emily sees 10 ants on a picnic blanket. 4 walk away. Therefore 6 ants are left. So, i wrote the sentence 10 takeaway 4 is 6.

Independent Practice

Directions Have students listen to each story, and then complete the sentence to tell how many are left. 7 Jerome sees 8 snails on the sidewalk. 3 slink away. How many snails are left? 8 Jerome sees 6 grasshoppers in the grass. 3 hop away. How many grasshoppers are left? 9 Jerome sees 9 butterflies in the garden. 4 flutter away. How many butterflies are left? 10 Higher Order Thinking Have students listen to the story, draw a picture to show the story, and then complete the sentence to tell how many are left. Jerome sees 7 inchworms on a tree. L crawl away. How many inch worms are left?

Question 7.
Envision Math Common Core Grade K Answer Key Topic 7 Understand Subtraction q 38
Answer:

Explanation:
Jerome sees 8 snails on the sidewalk. 3 slink away. Therefore 5 snails are left. So, i wrote the sentence 8 takeaway 3 is 5.

Question 8.
Envision Math Common Core Grade K Answer Key Topic 7 Understand Subtraction q 39
Answer:

Explanation:
Jerome sees 6 grasshoppers in the grass. 3 hop away. Therefore 3 grasshoppers are left. So, i wrote the sentence 6 takeaway 3 is 3.

Question 9.
Envision Math Common Core Grade K Answer Key Topic 7 Understand Subtraction q 40
Answer:

Explanation:
Jerome sees 9 butterflies in the garden. 4 flutter away. Therefore 5 butterflies are left. So, i wrote the sentence 9 takeaway 4 is 5.

Question 10.
Envision Math Common Core Grade K Answer Key Topic 7 Understand Subtraction q 41
Answer:

Lesson 7.4 Represent and Explain Subtraction with Addition

Solve & Share
Directions Have students listen to the story and use counters to show what happens. Say: There are 6 fire hats. Firefighters take 3 away. What numbers do you subtract to find how many hats are left? How con you show the subtraction?

I can ………. write an equation to show subtraction.
Envision Math Common Core Grade K Answer Key Topic 7 Understand Subtraction q 42

Visual Learning Bridge

Envision Math Common Core Grade K Answer Key Topic 7 Understand Subtraction q 43

Guided Practice

Directions 1 and 2 Have students use counters to model the problem, mark Xs to subtract, and then write a subtraction equation to find
the difference.

Question 1.
Envision Math Common Core Grade K Answer Key Topic 7 Understand Subtraction q 44
Answer:

Explanation:
I marked X to subtract, and then wrote a subtraction equation 5-4=1 to find the difference.

Question 2.
Envision Math Common Core Grade K Answer Key Topic 7 Understand Subtraction q 45
Answer:

Explanation:
I drew 7 counters and marked X on 5 counters to subtract. I wrote a subtraction equation 7-5=2 as the difference is 2.

Directions 3 – 6 Have students use counters to model the problem, mark Xs to subtract, and then write a subtraction equation to find
the difference.

Question 3.
Envision Math Common Core Grade K Answer Key Topic 7 Understand Subtraction q 46
Answer:

Explanation:
I drew 8 counters and marked X on 2 counters to subtract. I wrote a subtraction equation 8-2=6 as the difference is 6.

Question 4.
Envision Math Common Core Grade K Answer Key Topic 7 Understand Subtraction q 47
Answer:

Explanation:
I drew 6 counters and marked X on 5 counters to subtract. I wrote a subtraction equation 6-5=1 as the difference is 1.

Question 5.
Envision Math Common Core Grade K Answer Key Topic 7 Understand Subtraction q 48
Answer:

Explanation:
I drew 9 counters and marked X on 5 counters to subtract. I wrote a subtraction equation 9-5=4 as the difference is 4.

Question 6.
Envision Math Common Core Grade K Answer Key Topic 7 Understand Subtraction q 49
Answer:

Explanation:
I drew 7 counters and marked X on 2 counters to subtract. I wrote a subtraction equation 7-2=5 as the difference is 5.

Independent Practice

Directions 7 – 9 Have students use counters to model the problem, mark Xs to subtract, and then write an equation to find the difference. 10 Higher Order Thinking Have students listen to the story, draw counters and mark Xs to show the problem, and then write an equation to find the difference. There are 7 baseball caps. Some are worn to a game. There are 4 left. How many caps were worn to the game?

Question 7.
Envision Math Common Core Grade K Answer Key Topic 7 Understand Subtraction q 50
Answer:

Explanation:
I drew 8 counters and marked X on 3 counters to subtract. I wrote a subtraction equation 8-3=5 as the difference is 5.

Question 8.
Envision Math Common Core Grade K Answer Key Topic 7 Understand Subtraction q 51
Answer:

Explanation:
I drew 4 counters and marked X on 1 counter to subtract. I wrote a subtraction equation 4-1=3 as the difference is 3.

Question 9.
Envision Math Common Core Grade K Answer Key Topic 7 Understand Subtraction q 52
Answer:

Explanation:
I drew 6 counters and marked X on 4 counters to subtract. I wrote a subtraction equation 6-4=2 as the difference is 2.

Question 10.
Envision Math Common Core Grade K Answer Key Topic 7 Understand Subtraction q 53
Answer:

Explanation:
I drew 7 counters and marked X on 3 counters to subtract. I wrote a subtraction equation 7-3=4 as the difference is 4.

Lesson 7.5 Solve Subtraction Word Problems: Taking From and Apart

Solve & Share
Directions Say: Marta’s dog, Spot, loves to eat doggie biscuits. Moda put 6 biscuits in a bag. One day, Spot ate q biscuits. Now there are only 2 left. How does Marlo know there are 2 biscuits left? Use counters, pictures, or numbers to explain and show your work.

I can … find the difference of two numbers.
Envision Math Common Core Grade K Answer Key Topic 7 Understand Subtraction q 54

Visual Learning Bridge

Envision Math Common Core Grade K Answer Key Topic 7 Understand Subtraction q 55

Guided Practice

Directions 1 Have students listen to the story, draw a picture to show what is happening, and then write a subtraction equation. Then have them explain their work aloud. Say: Marfa has 6 kittens. She gives them a big bowl of water to drink. But there is only room for 4 kittens to drink at the same time. How does Marta know that 2 kittens have to wait?

Question 1.
Envision Math Common Core Grade K Answer Key Topic 7 Understand Subtraction q 56

Directions Have students listen to each story, use or draw a picture to show what is happening, and then write an equation. Then have them explain their work aloud. 2 Emily sees 8 rabbits in a pet store. Someone buys 3 of them. How many rabbits are left? 3 Emily sees 7 birds in a cage. The pet store owner opens the cage door and 3 fly out. How many birds ore left? 4 Emily sees 8 puppies in the store. 6 of them are sold. How many puppies are left? 5 Emily sees 5 hamsters sleeping. I goes away to eat. How many hamsters are left?

Question 2.
Envision Math Common Core Grade K Answers Topic 7 Understand Subtraction q 57
Answer:

Explanation:
Emily sees 8 rabbits in a pet store. Someone buys 3 of them. I used the picture to show what is happening. I marked X on 3 rabbits. Therefore 5 rabbits are left. So, i wrote the equation 8-3=5 as the difference is 5.

Question 3.
Envision Math Common Core Grade K Answers Topic 7 Understand Subtraction q 58
Answer:

Explanation:
Emily sees 7 birds in a cage. The pet store owner opens the cage door and 3 fly out.I used the picture to show what is happening. I marked X on 3 parrots. Therefore 4 rabbits are left. So, i wrote the equation 7-3=4 as the difference is 4.

Question 4.
Envision Math Common Core Grade K Answers Topic 7 Understand Subtraction q 59
Answer:

Explanation:
Emily sees 8 puppies in the store. 6 of them are sold. I drew connecting cubes to show what is happening. I marked X on 3 cubes. Therefore 5 cubes are left. So, i wrote the equation 8-3=5 as the difference is 5.

Question 5.
Envision Math Common Core Grade K Answers Topic 7 Understand Subtraction q 59
Answer:

Explanation:
Emily sees 5 hamsters sleeping. I goes away to eat. I drew connecting cubes to show what is happening. I marked X on 1 cubes. Therefore 4 cubes are left. So, i wrote the equation 5‐1=4 as the difference is 4.

Independent Practice

Directions Have students listen to each story, draw a picture to show what is happening, and then write an equation. 6 There are 6 birds in a birdbath. 4 fly away. How many birds are left? 7 There are 5 acorns under a tree. A squirrel takes 3 of them. How many acorns are left? 8 Higher Order Thinking Have students listen to the story, draw a circle around the picture that shows the story and tell why the other picture does NOT show the story, and then write an equation. There are 4 ducks in a pond, 1 leaves. How many ducks are left?

Question 6.
Envision Math Common Core Grade K Answers Topic 7 Understand Subtraction q 59
Answer:

Explanation:
There are 6 birds in a birdbath. 4 fly away.I drew connecting cubes to show what is happening. I marked X on 2 cubes. Therefore 4 cubes are left. So, i wrote the equation 6‐4=2 as the difference is 2.

Question 7.
Envision Math Common Core Grade K Answers Topic 7 Understand Subtraction q 59
Answer:

Explanation:
There are 5 acorns under a tree. A squirrel takes 3 of them.I drew connecting cubes to show what is happening. I marked X on 3 cubes. Therefore 4 cubes are left. So, i wrote the equation 5‐3=2 as the difference is 2.

Question 8.
Envision Math Common Core Grade K Answers Topic 7 Understand Subtraction q 60
Answer:

Explanation:
There are 4 ducks in a pond, 1 leaves. Therefore the equation is 4‐3=2. The first picture matches  with the equation. So, i circled the first equation.

Lesson 7.6 Use Patterns to Develop Fluency in Subtraction

Solve & Share
Directions Say: Look at the first equation. Write the number from the number that completes the equation on the orange spoce. Repeat for the next equation. Finish the pattern by placing the other number cards on the orange spaces, and then write the numbers to complete the equations. What patterns do you see?

I can….. find patterns in subtraction equations.
Envision Math Common Core Grade K Answers Topic 7 Understand Subtraction q 61

Visual Learning Bridge

Envision Math Common Core Grade K Answers Topic 7 Understand Subtraction q 62

Guided Practice

Directions 1 Have students complete each equation to find the pattern, and then explain the pattern they see.

Question 1.
Envision Math Common Core Grade K Answers Topic 7 Understand Subtraction q 63
Answer:

Explanation:
I completed each equation and found the pattern. The difference is decreasing by 1.

Directions 2 and 3 Have students look for a pattern, explain the pattern they see, and then write an equation for each row of insects.

Question 2.
Envision Math Common Core Grade K Answers Topic 7 Understand Subtraction q 64
Answer:

Explanation:
I observed the pattern in the question. The number of butterflies left are increasing by 1. So, i completed the equations based on the picture. The difference is decreasing by 1.

Question 3.
Envision Math Common Core Grade K Answers Topic 7 Understand Subtraction q 65
Answer:

Explanation:
I observed the pattern in the question. The number of butterflies left are decreasing by 1. So, i completed the equations based on the picture. The difference is increasing  by 1.

Independent Practice

Directions 4 Algebra Have students mark Xs to complete the pattern, explain the pattern they see, and then write an equation for each row of flowers. 5 Higher Order Thinking Have students find the pattern, and then complete the equation. 6 Higher Order Thinking Have students find the pattern, and then write the missing number in the equation.

Question 4.
Envision Math Common Core Grade K Answers Topic 7 Understand Subtraction q 66
Answer:

Explanation:
I marked X on one flower in the second row and left the third row to create the decreasing pattern. Therefore based on the patterns of flowers i completed the equations. The difference is increasing by 1.

Question 5.
Envision Math Common Core Grade K Answers Topic 7 Understand Subtraction q 67
Answer:

Explanation:
I found the pattern and the equation is 10‐4=6.

Question 6.
Envision Math Common Core Grade K Answers Topic 7 Understand Subtraction q 68
Answer:

Explanation:
I found the pattern and the missing number is 4.

Lesson 7.7 Problem Solving

Use Appropriate Tools
Solve & Share

Directions Say: Alex has a food bar with 8 pieces of food for the flamingos at the lake. He takes apart 2 pieces of the bar to feed the flamingos. How many pieces does he have left on his bar? Use one of the tools you have to help solve the problem. Draw a picture of what you did, and then write the equation.

I can ……….. use tools to subtract numbers
Envision Math Common Core Grade K Answers Topic 7 Understand Subtraction q 69

Visual Learning Bridge

Envision Math Common Core Grade K Answers Topic 7 Understand Subtraction q 70

Guided Practice

Directions Have students listen to each story, use a tool to help them solve the problem, and then write the equation. Then have them explain whether or not the tool they chose helped to solve the problem. 1 There is I flamingo standing in the water. 8 more fly over to join it. How many flamingos are there in all? 2 Marta sees 7 seagulls. 4 fly away. How many seagulls are left?

Question 1.
Envision Math Common Core Grade K Answers Topic 7 Understand Subtraction q 71
Answer:

Explanation:
There is I flamingo standing in the water. 8 more fly over to join it. Therefore there are 9 flamigos in all. So, the addition equation is 1+8=9.

Question 2.
Envision Math Common Core Grade K Answers Topic 7 Understand Subtraction q 72
Answer:

Explanation:
Marta sees 7 seagulls. 4 fly away. Therefore 3 seagulls are left. So, the subtraction equation is 7-4=3.

Independent Practice

Directions Have students listen to each story, use a tool to help them solve the problem, and then write the equation. Then have them tell which tool they chose and whether or not it helped to solve the problem. 3 There are 3 raccoons in a tree. 3 more climb the tree to join them. How many raccoons are there in all? 4 Marta sees 9 turtles swimming in a pond. 5 dive under the water. How many turtles are left? 5 There are 7 beavers in the water. 4 swim away. How many beavers are left? 6 Marta see 6 ducks in the lake. 2 more join them. How many ducks are there in all?

Question 3.
Envision Math Common Core Grade K Answers Topic 7 Understand Subtraction q 73
Answer:

Explanation:
There are 3 raccoons in a tree. 3 more climb the tree to join them. Therefore, there are 6 racoons in all. So, the addition equation is 3+3=6.

Question 4.
Envision Math Common Core Grade K Answers Topic 7 Understand Subtraction q 73
Answer:

Explanation:
Marta sees 9 turtles swimming in a pond. 5 dive under the water. Therefore, 4 turtles are swimming. So, the subtraction equation is 9-5=4.

Question 5.
Envision Math Common Core Grade K Answers Topic 7 Understand Subtraction q 73
Answer:

Explanation:
There are 7 beavers in the water. 4 swim away. Therefore, 3 beavers are left. So, the subtraction equation is 7-4=3.

Question 6.
Envision Math Common Core Grade K Answers Topic 7 Understand Subtraction q 73
Answer:

Explanation:
Marta see 6 ducks in the lake. 2 more join them. Therefore, there are 8 ducks in all. So, the addition sentence is 6+2=8.

Problem Solving

Performance Task
Directions Read the problem aloud. Then have students use multiple problem-solving methods to solve the problem. Say: Carlos collects stomps. He has 9 stamps in all. He puts I stamp on the cover. He puts the rest inside the book. How many stamps does Carlos put inside his stamp book? 7 Make Sense What are you trying to find out? Will you use addition or subtraction to solve the problem? 8 Use Tools What tool can you use to help solve the problem? Tell a partner and explain why. 9 Be Precise Did you write the equation correctly? Explain what the numbers and the symbols mean in the equation.
Envision Math Common Core Grade K Answers Topic 7 Understand Subtraction q 74
Answer:

Explanation:
Carlos collects stomps. He has 9 stamps in all. He puts I stamp on the cover. He puts the rest inside the book.
6) I am trying to find how many stamps did carol put in his book. I use subtraction to find the answer.
7) I drew counters as tool to find the difference.
8) yes, my equation is correct. 9-1=8. The symbol (-) means minus and (=) equal.

Topic 7 Vocabulary Review

Directions Understand Vocabulary Have students: 1 write the minus sign to show subtraction; 2 draw a circle around the number that tells how many are left; 3 complete the subtraction sentence; 4 separate the tower into 2 parts, draw each part, and then write the numbers to tell the parts.

Question 1.
Envision Math Common Core Grade K Answers Topic 7 Understand Subtraction q 75
Answer:

Explanation:
I wrote the minus sign in the circle to show the subtraction.

Question 2.
Envision Math Common Core Grade K Answers Topic 7 Understand Subtraction q 76
Answer:

Explanation:
When we subtract 6 from 9 3 will be left. So, i circled the number 3.

Question 3.
Envision Math Common Core Grade K Answers Topic 7 Understand Subtraction q 77
Answer:

Explanation:
The completed the subtraction sentence that is 8 takeaway 2 is 6.

Question 4.
Envision Math Common Core Grade K Answers Topic 7 Understand Subtraction q 78
Answer:

Explanation:
I separated the tower into 2 parts, and drew 3 cubes as a part and 2 cubes as other part. Then wrote the numbers 3 and 2 in the blanks to tell the parts.

Directions Understand Vocabulary Have students: 5 draw a circle around the difference; 6 write an equation to show how to subtract 3 from 7 to find the difference; 7 listen to the story, draw a picture to show how to take away, and then write an equation to match the story. Lorin sees 6 apples on the table. She takes 3 away. How many apples are left?

Question 5.
Envision Math Common Core Grade K Answers Topic 7 Understand Subtraction q 79
Answer:

Explanation:
When we subtract 3 from 8 the difference is 5. So, i circled 5.

Question 6.
Envision Math Common Core Grade K Answers Topic 7 Understand Subtraction q 80
Answer:

Explanation:
The equation that shows the how to subtract 3 from 7 is 7-3=4.

Question 7.
Envision Math Common Core Grade K Answers Topic 7 Understand Subtraction q 80
Answer:

Explanation:
Lorin sees 6 apples on the table. She takes 3 away. I marked X on 3apples as Lorin takes away 3 apples. Therefore 3 apples are left. I wrote the equation 6-3=3 to show what Lorin did.

Topic 7 Reteaching

Directions Have students: 1 count the bees, tell how many are NOT on the flower, and then write the number to tell how many are left on the flower; 2 take apart the group of apples. Have them draw a circle around the parts they made, and then write the numbers to tell the parts.

Set A

Envision Math Common Core Grade K Answers Topic 7 Understand Subtraction q 81

Question 1.
Envision Math Common Core Grade K Answers Topic 7 Understand Subtraction q 82
Answer:

Explanation:
i counted the bees that are not on the flower. There is 1 bee out and there are 4 bees left on the flower. So, i wrote the number 4 in the blank.

Set B

Envision Math Common Core Grade K Answers Topic 7 Understand Subtraction q 83

Question 2.
Envision Math Common Core Grade K Answers Topic 7 Understand Subtraction q 84
Answer:

Explanation:
I took apart the group of apples then drew a circle around the parts i made, and then wrote the numbers 3 and 4 to tell the parts.

Directions Have students: 3 listen to the story, and then complete the sentence to tell how many are left. Javi sees 9 dragonflies. 4 fly away. How many dragonflies are left? 4 use counters to model the problem, mark Xs to subtract, and then write an equation to find the difference.

Set C

Envision Math Common Core Grade K Answers Topic 7 Understand Subtraction q 85

Question 3.
Envision Math Common Core Grade K Answers Topic 7 Understand Subtraction q 86
Answer:

Explanation:
Javi sees 9 dragonflies. 4 fly away. Therefore, 5 flies are left. So, i wrote the sentence 9 takeaway 4 is 5.

Set D

Envision Math Common Core Grade K Answers Topic 7 Understand Subtraction q 87

Question 4.
Envision Math Common Core Grade K Answers Topic 7 Understand Subtraction q 88
Answer:

Explanation:
I used the picture to solve the problem. I marked X on 1 image. Therefore, 4-1=3.

Directions Have students: 5 use counters to model the problem, and then write an equation to tell how many are left; 6 listen to the story, draw a picture to show what is happening, and then write an equation to match the story. Lidia has 5 balloons. 2 balloons pop. How many balloons does she have left?

Set D continued

Envision Math Common Core Grade K Answers Topic 7 Understand Subtraction q 89

Question 5.
Envision Math Common Core Grade K Answers Topic 7 Understand Subtraction q 90
Answer:

Explanation:
There ere 9 birds in the question. I marked X on 6 birds. 3 birds are left. So, i wrote the subtraction equation 9-6=3.

Set E

Envision Math Common Core Grade K Answers Topic 7 Understand Subtraction q 91

Question 6.
Envision Math Common Core Grade K Answers Topic 7 Understand Subtraction q 92
Answer:

Explanation:
Lidia has 5 balloons. 2 balloons pop. So, 3 balloons are left. Therefore, the subtraction equation is 5-2=3.

Directions Have students: 7 complete each equation to find the pattern; 8 listen to the story, use counters to help solve the problem, and then write the equation. Darla sees 3 frogs on the pond. 7 more join them. How many frogs are there in all?

Set F

Envision Math Common Core Grade K Answers Topic 7 Understand Subtraction q 93

Question 7.
Envision Math Common Core Grade K Answers Topic 7 Understand Subtraction q 94
Answer:

Explanation:
I completed the equations to find the pattern. The differences of the equations are increasing by 1.

Set G

Envision Math Common Core Grade K Answers Topic 7 Understand Subtraction q 95

Question 8.
Envision Math Common Core Grade K Answers Topic 7 Understand Subtraction q 96
Answer:

Explanation:
Darla sees 3 frogs on the pond. 7 more join them. I used counters to show what is happening. There are 10 frogs in all. Therefore, i wrote the equation 3+7=10.

Topic 7 Assessment Practice

Directions Have students mark the best answer. 1 Say: Which expression matches the picture and tells the number of cubes in all and a part that is taken away? 2 Say: There are some animals in a group. Then some animals leave. Which sentence and subtraction equation match the picture and tell how many animals are left? 3 Which equation matches the picture and tells how many ducks are left?

Question 1.
Envision Math Common Core Grade K Answers Topic 7 Understand Subtraction q 97
A. 6 take away 2
B. 7 take away 2
C. 4 take away 2
D. 5 take away 3
Answer:
B. 7 takeaway 2.

Explanation:
There are 7 cubes in all and 2 cubes are seperated from the other 5 cubes.

Question 2.
Envision Math Common Core Grade K Answers Topic 7 Understand Subtraction q 98
A. 4 take away 2 is 2. 8 – 2 = 6
B. 4 take away 3 is 1. 4 – 3 = 1
C. 2 take away I is 2. 3 – 1 = 2
D. 5 take away 3 is 2. 5 – 3 = 2
Answer:
B. 4 takeaway 3 is 1. 4-3=1

Explanation:
There 4 mouse in all. 3 are marked X so, 1 is left. So, the equation will be 4-3=1.

Question 3.
Envision Math Common Core Grade K Answers Topic 7 Understand Subtraction q 99
A. 5 – 3 = 2
B. 5 – 2 = 3
C. 7 – 3 = 4
D. 7 – 2 = 5
Answer:
A. 5-3=2.

Explanation:
There are 5 ducks in all. 3 are marked X 2 are left. So, the equation is 5-3=2.

Directions 4 Have students listen to the story, and then complete the sentence to tell how many are left. Kyle sees 10 turtles at the zoo. 2 turtles crawl away. Write a number sentence that tells how many are left. 5 Have students count the fish. Then have them mark Xs on some of the fish that swim away, write the number to tell how many are left, and write a matching equation, 6 Say: Ramona has 7 apples. She puts the apples on 2 plates. Draw apples to show how many Ramona could put on each plate. Then write the numbers to tell the parts.

Question 4.
Envision Math Common Core Grade K Answers Topic 7 Understand Subtraction q 100
Answer:

Explanation:
Kyle sees 10 turtles at the zoo. 2 turtles crawl away. So, 8 are left. Therefore, the sentence is 10 takeaway 2 is 8.

Question 5.
Envision Math Common Core Grade K Answers Topic 7 Understand Subtraction q 101
Answer:

Explanation:
I counted the number of fishes. There are 8 fishes in all. I marked X on 3 fishes that swim away, wrote the number 5 to tell how many are left. The equation is 8-3=5.

Question 6.
Envision Math Common Core Grade K Answers Topic 7 Understand Subtraction q 102
Answer:

Explanation:
Ramona has 7 apples. She puts the apples on 2 plates. I drew 3 apples i one plate and 4 apples in the other plate to show how many Ramona could put on each plate. 3+4=7.

Directions 7 Have students complete each equation to find the pattern, 8 Have students listen to the story, draw a circle around the picture that shows the story, and then write an equation. There were 7 lizards in the sand. I crawls away. How many are left? 9 Which equation matches the picture?

Question 7.
Envision Math Common Core Grade K Answers Topic 7 Understand Subtraction q 103
Answer:

Explanation:
I observed the pattern above, the number to be subtracted is increasing by 1. So, i completed the equations based on the pattern. The difference is decreasing by 1.

Question 8.
Envision Math Common Core Grade K Answers Topic 7 Understand Subtraction q 104
Answer:

Explanation:
There were 7 lizards in the sand. 1 crawls away. Therefore, 6 are left. So, i wrote the equation 7-1=6.

Question 9.
Envision Math Common Core Grade K Answers Topic 7 Understand Subtraction q 105
A. 6 – 2 = 4
B. 5 – 4 = 1
C. 9 – 4 = 5
D. 4 – 4 = 0
Answer:
C. 9 – 4 = 5

Explanation:
There are 9 crabs. 4 crabs are marked X. 5 are left. Therefore the equation 9-4=5 matches with the picture.

Directions Have students: 10 take apart the group of plums. Have them draw a circle around the parts they made, and then write an equation that matches their picture; 11 listen to the story, draw a picture, use counters or other objects to help solve the problem, and then write an equation that matches the story. Kim collects 9 shells. She gives 6 away. How many shells does Kim have left? 12 Have students match each equation with a row of flowers to find the pattern.

Question 10.
Envision Math Common Core Grade K Answers Topic 7 Understand Subtraction q 106
Answer:

Explanation:
I took apart the plums. I marked X on 2 plums. 4 are left. Therefore, the subtraction equation is 6-2=4.

Question 11.
Envision Math Common Core Grade K Answers Topic 7 Understand Subtraction q 107
Answer:

Explanation:
Kim collects 9 shells. She gives 6 away. 3 are left. The subtraction equation is 9-6=3.

Question 12.
Envision Math Common Core Grade K Answers Topic 7 Understand Subtraction q 108
Answer:

Explanation:
I matched the pictures with the equations.

Topic 7 Performance Task

Directions Puppet Show Say: Paco’s class uses many puppets for their puppet show. 1 Have students listen to the story, and then write a subtraction sentence to tell how many duck puppets are left. Paco has 8 duck puppets at school. He takes 3 home. How many duck puppets are left at school? 2 Write an equation to tell how many duck puppets Paco has left at school. 3 Say: The picture shows that Paco put I cat puppet in a drawer. How many cat puppets are left? Have students write an equation for the picture, and then write another equation to complete a pattern.

Question 1.
Envision Math Common Core Grade K Answers Topic 7 Understand Subtraction q 109
Answer:

Explanation:
Paco has 8 duck puppets at school. He takes 3 home. The sentence is 8 takeaway 3 is 5. Therefore, 5 puppets are left at school.

Question 2.
Envision Math Common Core Grade K Answers Topic 7 Understand Subtraction q 110
Answer:

Explanation:
The equation that represents the sentence 8 takeaway 3 is 5 is 8-3=5.

Question 3.
Envision Math Common Core Grade K Answers Topic 7 Understand Subtraction q 111
Answer:

Explanation:
Paco has 5 puppets and he put 1  puppet in drawer. So, the equation is 5-1=4.
Another equation in this pattern is 5-4=1, this tells 1 is in drawer and 4 are with Paco.

Directions 4 Say: Paco’s class puts on a play using 4 puppets. Each scene of the play has I more puppet leave the stage than the scene before. Have students mark Xs to complete the pattern. Then have them write equations to show how many puppets leave each scene. 5 Have students listen to the story, use counters to help solve each part of the problem, and then write an equation. Paco has 4 yellow bird puppets and 3 red bird puppets on his desk. How many bird puppets does Paco have in all? Then Paco moves 2 bird puppets to his friend Owen’s desk. How many bird puppets are left on Paco’s desk?

Question 4.
Envision Math Common Core Grade K Answers Topic 7 Understand Subtraction q 112
Answer:

Explanation:
Paco’s class puts on a play using 4 puppets. Each scene of the play has I more puppet leave the stage than the scene before. So, i marked Xs to complete the pattern. Then wrote equations 4-1=3, 4-2=2 and 4-3=1 to show how many puppets leave each scene.

Question 5.
Envision Math Common Core Grade K Answers Topic 7 Understand Subtraction q 113
Answer:

Envision Math Common Core Grade K Answer Key Topic 4 Compare Numbers 0 to 10

Practice with the help of enVision Math Common Core Kindergarten Answer Key Topic 4 Compare Numbers 0 to 10 regularly and improve your accuracy in solving questions.

Envision Math Common Core Grade K Answers Key Topic 4 Compare Numbers 0 to 10

Essential Question: How can numbers from 0 to 10 be compared and ordered?
Envision Math Common Core Grade K Answer Key Topic 4 Compare Numbers 0 to 10 q1

enVision STEM Project: Weather Changes

Directions Read the character speech bubbles to students. Find Out! Have students find out about weather changes. Say: The weather changes from day to day. Talk to friends and relatives about the weather. Ask them to help you record the number of sunny days and rainy days during the week. Journal: Make a Poster Have students make a poster. Have them draw up to 10 lightning bolts above one house and up to 10 lightning bolts above another house. Ask them to write the number of lightning bolts above each house, and then draw a circle around the number that is greater than the other, or draw a circle around both numbers if they are the same.

Review What You Know

Question 1.
Envision Math Common Core Grade K Answer Key Topic 4 Compare Numbers 0 to 10 q2
Answer:

Explanation:
I circled the group of birds that are lessthan the other group.3 is lessthan 4.

Question 2.
Envision Math Common Core Grade K Answer Key Topic 4 Compare Numbers 0 to 10 q3
Answer:

Explanation:
I circled the group of dogs that is greater than the ther group.5 is greater than 1.

Question 3.
Envision Math Common Core Grade K Answer Key Topic 4 Compare Numbers 0 to 10 q4
Answer:

Explanation:
I circled 2 groups of marbles that are equal in number.3 is equal to 3.

Question 4.
Envision Math Common Core Grade K Answer Key Topic 4 Compare Numbers 0 to 10 q5
Answer:

Explanation:
There are 6 objects in the above picture.So, i counted and wrote 6.

Question 5.
Envision Math Common Core Grade K Answer Key Topic 4 Compare Numbers 0 to 10 q6
Answer:

Explanation:
There are 8 objects in the above picture.So, i counted and wrote 8.

Question 6.
Envision Math Common Core Grade K Answer Key Topic 4 Compare Numbers 0 to 10 q6.1
Answer:

Explanation:
There are 10 objects in the above picture.So, i counted and wrote 10.

Directions Have students: 1 draw a circle around the group of birds that is less than the other group; 2 draw a circle around the group of dogs that is greater than the other group; 3 draw a circle around the two groups that have an equal number of marbles; 4-6 count the number of objects, and then write the number to tell how many.

Pick a Project

Envision Math Common Core Grade K Answer Key Topic 4 Compare Numbers 0 to 10 q7

Envision Math Common Core Grade K Answer Key Topic 4 Compare Numbers 0 to 10 q8

Directions Say: You will choose one of these projects. Look at picture A. Think about this question: How can you train to go into space? If you choose Project A, you will act out an exercise skit. Look at picture B, Think about this question: What kinds of fruit would you put into a fruit salad? If you choose Project B, you will create a fruit salad recipe.

Envision Math Common Core Grade K Answer Key Topic 4 Compare Numbers 0 to 10 q9

Envision Math Common Core Grade K Answer Key Topic 4 Compare Numbers 0 to 10 q10

Directions Say: You will choose one of these projects. Look at picture C. Think about this question: What is the most exciting ride at a theme park? If you choose Project C, you will design a ride. Look at picture D. Think about this question: What do you like to do on a vacation? If you choose Project D, you will make a list.

Lesson 4.1 Compare Groups to 10 by Matching

Solve & Share

Envision Math Common Core Grade K Answer Key Topic 4 Compare Numbers 0 to 10 q11

Directions Say: Work with a partner. Take turns drawing one cube from the bag and placing it on your page in the rectangle of the same color. When the bag is empty, do you have more red or blue cubes? How do you know? Draw a picture of your cubes in The rectangles showing which color is more.

Visual Learning Bridge

Envision Math Common Core Grade K Answer Key Topic 4 Compare Numbers 0 to 10 q12

Guided practice

Question 1.
Envision Math Common Core Grade K Answer Key Topic 4 Compare Numbers 0 to 10 q13
Answer:

Explanation:
I drew a line from each chick in the top group to a chick in the bottom group, and then drew a circle around the group that is greater in number than the other group.7 is greater than 6.

Directions 1 Have students compare the groups, draw a line from each chick in the top group to a chick in the bottom group, and then draw a circle around the group that is greater in number than the other group.

Question 2.
Envision Math Common Core Grade K Answer Key Topic 4 Compare Numbers 0 to 10 q14
Answer:

Explanation:
I compared the groups, drew a line from each chick in the top group to a chick in the bottom group, and then drew a circle around the group that is greater in number than the other group.8 is greater than 3.

Question 3.
Envision Math Common Core Grade K Answer Key Topic 4 Compare Numbers 0 to 10 q15
Answer:

Explanation:
I compared the groups, drew a line from each chick in the top group to a chick in the bottom group, and then drew a circle around the group that is less in number than the other group.4 is less than 5.

Question 4.
Envision Math Common Core Grade K Answer Key Topic 4 Compare Numbers 0 to 10 q16
Answer:

Explanation:
I compared the groups, drew a line from each chick in the top group to a chick in the bottom group, and then drew a circle around the group that is less in number than the other group.7 is less than 9.

Directions 2 envision® STEM Say: Chicks live in coops. Coops protect chicks in different types of weather. Have students compare the groups, draw a line from each chick in the top group to a chick in the bottom group, and then draw a circle around the group that is greater in number than the other group. 3 and 4 Have students compare the groups, draw a line from each chick in the top group to a chick in the bottom group, and then draw a circle around the group that is less in number than the other group.

Independent Practice

Question 5.
Envision Math Common Core Grade K Answer Key Topic 4 Compare Numbers 0 to 10 q17
Answer:

Explanation:
I compared the groups, drew a line from each bucket in the top group to a bucket in the bottom group, and then drew a circle around the group that is greater in number than the other group.8 is greater than 6.

Question 6.
Envision Math Common Core Grade K Answer Key Topic 4 Compare Numbers 0 to 10 q18
Answer:

Explanation:
I compared the groups, drew a line from each bucket in the top group to a bucket in the bottom group, and then drew a circle around the group that is less in number than the other group.2 is less than 3.

Question 7.
Envision Math Common Core Grade K Answer Key Topic 4 Compare Numbers 0 to 10 q19
Answer:

Explanation:
I compared the groups, drew a line from each bucket in the top group to a bucket in the bottom group, and then drew a circle around the group that is less in number than the other group.4 is less than 7.

Question 8.
Envision Math Common Core Grade K Answer Key Topic 4 Compare Numbers 0 to 10 q20
Answer:

Explanation:
I counted the numbe rof buckets, there are 5 buckets in the top group.
I drew a group of 9 buckets that is greater than the buckets in the top group.
9 is greater than 5.

Directions Have students: 5 compare the groups, draw a line from each bucket in the top group to a bucket in the bottom group, and then draw a circle around the group that is greater in number than the other group; 6 and 7 compare the groups, draw a line from each bucket in the top group to a bucket in the bottom group, and then draw a circle around the group that is less in number than the other group, 8 Higher Order Thinking Have students draw a group of buckets that is greater in number than the group shown.

Lesson 4.2 Compare Numbers Using Numerals to 10

Envision Math Common Core Grade K Answer Key Topic 4 Compare Numbers 0 to 10 q21

Answer:

Explanation:
Emily is planting seedlings, or little plants. She plants 5 red pepper seedlings and 7 yellow pepper seedlings.
I drew counters to show groups of seedlings and i wrote the numbers 5 and 7.I circled 7 as it is greater than 5.

Directions Say: Emily is planting seedlings, or little plants. She plants 5 red pepper seedlings and 7 yellow pepper seedlings. Use counters to show the groups of seedlings. Write the numbers, and then circle the number that tells which group has more.

Visual Learning Bridge

Envision Math Common Core Grade K Answer Key Topic 4 Compare Numbers 0 to 10 q22

Guided Practice

Question 1.
Envision Math Common Core Grade K Answer Key Topic 4 Compare Numbers 0 to 10 q23
Answer:

Explanation:
I drew a line from each watering can in the top group to a watering can in the bottom group, and then draw a circle around the number 4 as it is greater than 3.

Directions 1 Have students count the watering cans in each group, write the number to tell how many, draw a line from each watering can in the top group to a watering can in the bottom group, and then draw a circle around the number that is greater than the other number.

Question 2.
Envision Math Common Core Grade K Answer Key Topic 4 Compare Numbers 0 to 10 q24
Answer:

Explanation:
I counted the vegetables in each group, wrote the numbers 4 and 5 to tell how many, drew a line from each vegetable in the top group to a vegetable in the bottom group, and then marked an X on the number 4 as it is less than 4.

Question 3.
Envision Math Common Core Grade K Answer Key Topic 4 Compare Numbers 0 to 10 q25
Answer:

Explanation:
I counted the vegetables in each group, i drew 3 more pea pods to make the groups equal, wrote the numbers 6 to tell how many in each group, and then drew a line from each vegetable in the top group to a vegetable in the bottom group to compare.

Directions 2 Have students count the vegetables in each group, write the number to tell how many, draw a line from each vegetable in the top group to a vegetable in the bottom group, and then mark an X on the number that is less than the other number. 3 Number Sense Have students count the vegetables in each group, draw more pea pods to make the groups equal, write the numbers to tell how many in each group, and then draw a line from each vegetable in the top group to a vegetable in the bottom group to compare.

Independent Practice

Question 4.
Envision Math Common Core Grade K Answer Key Topic 4 Compare Numbers 0 to 10 q26
Answer:

Explanation:
I counted the seed packets in each group, wrote the numbers 10 and 7 to tell how many, drew a line from each seed packet in the top group to a seed packet in the bottom group, and then mark an X on the number 7 as it  is less than 10.

Question 5.
Envision Math Common Core Grade K Answer Key Topic 4 Compare Numbers 0 to 10 q27
Answer:

Explanation:
I counted the flowers in the group, drew a group of 3 flowers that is less than the group shown, and then write the numbers 6 and 3 to tell how many.

Directions 4 Have students count the seed packets in each group, write the number to tell how many, draw a line from each seed packet in the top group to a seed packet in the bottom group, and then mark an X on the number that is less than the other number. 5 Higher Order Thinking Have students count the flowers in the group, draw a group of flowers that is less than the group shown, and then write the numbers to tell how many.

Lesson 4.3 Compare Groups to 10 by Counting

Solve & Share

Envision Math Common Core Grade K Answer Key Topic 4 Compare Numbers 0 to 10 q28
Answer:

Explanation:
I counted and placed counters on goldfish and tetras, there are 6 goldfish and 9 tetras.I drew a circle around the number 9 as it is greater than 6.

Directions Say: The class aquarium has two kinds of fish, goldfish and tetras. Place counters on the fish as you count how many of each kind. Write numbers to tell how many of each kind. Draw a circle around the fish that has a number greater than the other. Tell how you know you are right.

Visual Learning Bridge

Envision Math Common Core Grade K Answer Key Topic 4 Compare Numbers 0 to 10 q29

Guided Practice

Question 1.
Envision Math Common Core Grade K Answer Key Topic 4 Compare Numbers 0 to 10 q30
Answer:

Explanation:
I counted and wrote the number of fishes each color, there are 10 pink fishes and 8 purple fishes.I circled 10 as it is greater than 8.

Directions 1 Have students count the number of each color fish, write the numbers to tell how many, and then draw a circle around the number that is greater than the other number. Use the number sequence to help find the answer.

Question 2.
Envision Math Common Core Grade K Answer Key Topic 4 Compare Numbers 0 to 10 q31
Answer:

Explanation:
I counted and wrote the number of fishes each color, there are 6 green fishes and 7 yellow fishes.I circled 7 as it is greater than 6.

Question 3.
Envision Math Common Core Grade K Answer Key Topic 4 Compare Numbers 0 to 10 q32
Answer:

Explanation:
I counted wrote the number of fishes each color, there are 8 blue fishes and 9 gold fishes.I marked X on both  the numbers as they are NOT equal.

Question 4.
Envision Math Common Core Grade K Answer Key Topic 4 Compare Numbers 0 to 10 q33
Answer:

Explanation:
I counted wrote the number of fishes each color, there are 8 brown fishes and 7 green fishes.I marked X on the number 7 as it is lessthan 8.

Question 5.
Envision Math Common Core Grade K Answer Key Topic 4 Compare Numbers 0 to 10 q34
Answer:

Explanation:
I counted wrote the number of fishes each color, there are 9 purple fishes and 10 gold fishes.I marked X on the number 7 as it is lessthan 10.

Directions Have students count the number of each color fish, write the numbers to tell how many, and then: 2 draw a circle around the number that is greater than the other number; 3 draw a circle around both numbers if they are equal, or mark an X on both numbers if they are NOT equal; 4 and 5 mark an X on the number that is less than the other number. Use the number sequence to help find the answer for each problem.

Independent Practice

Question 6.
Envision Math Common Core Grade K Answer Key Topic 4 Compare Numbers 0 to 10 q35
Answer:

Explanation:
I counted and wrote the number of each critter.I wrote the numbers 6 to tell that there are 6 yellow and 6 green critters.I circled both the numbers as they are equal.

Question 7.
Envision Math Common Core Grade K Answer Key Topic 4 Compare Numbers 0 to 10 q36
Answer:

Explanation:
I counted and wrote the number of each critter.I wrote the numbers 9 and 8 to tell that there are 9 blue and 8 peach critters.I marked X on the numbe 8 as it is lessthan 9.

Question 8.
Envision Math Common Core Grade K Answer Key Topic 4 Compare Numbers 0 to 10 q37
Answer:

Explanation:
There are 7 trantulas in the above picture so, i drew 9 spiders that is two more than the number of trantulas.
Then i wrote the numbers 7 and 9.

Question 9.
Envision Math Common Core Grade K Answer Key Topic 4 Compare Numbers 0 to 10 q38
Answer:

Explanation:
I counted the number of butterflies.There are 6 butterflies.I wrote all the numbers up to 10 that are greater than the number of butterflies shown.The numbers that are greater than 6 are 7, 8, 9 and 10.

Directions Have students count the number of each critter, write the numbers to tell how many, and then: 6 draw a circle around both numbers if they are equal, or mark an X on both numbers if they are NOT equal; 7 mark an X on the number that is less than the other number; 8 draw a group of spiders that is two greater in number than the number of tarantulas shown, and then write the number to tell how many. 9 Higher Order Thinking Have students count the butterflies, and then write all the numbers up to 10 that are greater than the number of butterflies shown. Use the number sequence to help find the answer for each problem.

Lesson 4.4 Compare Numbers to 10

Solve & Share

Envision Math Common Core Grade K Answer Key Topic 4 Compare Numbers 0 to 10 q39

Directions Say: Emily’s mother asked her to bring the towels in off the line. Her basket can hold less than 7 towels. How many towels might Emily bring in? You can give more than one answer. Show how you know your answers are right.
Answer:
Emily’s mother asked her to bring the towels in off the line. Her basket can hold less than 7 towels.
Emily can bring 1, 2, , 4, 5 or 6 towels in her basket.

Visual Learning Bridge

Envision Math Common Core Grade K Answer Key Topic 4 Compare Numbers 0 to 10 q40

Guided Practice

Question 1.
Envision Math Common Core Grade K Answer Key Topic 4 Compare Numbers 0 to 10 q41
Answer:

Explanation:
I counted the numbers 1 to 10. I used the number sequence and drew lines from the numbers to the sequence and i circled 8 a sit is greater than 7.

Question 2.
Envision Math Common Core Grade K Answer Key Topic 4 Compare Numbers 0 to 10 q42
Answer:

Explanation:
I drew counters to show the numbers 6 and 4.I drew a circle around the number 6 as it is greater than the number 4.

Directions Have students: 1 count the numbers 1 to 10 and use the number sequence to show how they know which number is greater than the other, and then draw a circle around the number that is greater; 2 draw counters in the ten-frames to show how they know which number is greater than the other, and then draw a circle around the number that is greater.

Question 3.
Envision Math Common Core Grade K Answer Key Topic 4 Compare Numbers 0 to 10 q43
Answer:

Explanation:
I drew pictures to show the numbers 6 and 9.9 is greater than 6.

Question 4.
Envision Math Common Core Grade K Answer Key Topic 4 Compare Numbers 0 to 10 q44
Answer:

Explanation:
I drew counters to show the numbers 8.
I circled both the numbers as they are equal.

Question 5.
Envision Math Common Core Grade K Answer Key Topic 4 Compare Numbers 0 to 10 q45
Answer:

Explanation:
I used the number sequence to find the greater number.I marked X on the number 9 as it is lessthan 10.

Question 6.
Envision Math Common Core Grade K Answer Key Topic 4 Compare Numbers 0 to 10 q46
Answer:

Explanation:
I drew the pictures to show the numbers 9 and 8.I marked X on the number 8 as it is lessthan 9.

Directions Have students: 3 draw pictures to show how they know which number is greater than the other, and then draw a circle around the number that is greater; 4 draw counters in the ten-frames to show how they know if the numbers are equal, and then draw a circle around both numbers if they are equal, or mark an X on both numbers if they are NOT equal; 5 use the number sequence to show how they know which number is less than the other number, and then mark an X on the number that is less; 6 draw pictures to show how they know which number is less than the other number, and then mark an X on the number that is less.

Independent Practice

Question 7.
Envision Math Common Core Grade K Answer Key Topic 4 Compare Numbers 0 to 10 q47
Answer:

Explanation:
I drew the pictures to show the numbers 6 and 8.I marked X on the number 6 as it is lessthan 8.

Question 8.
Envision Math Common Core Grade K Answer Key Topic 4 Compare Numbers 0 to 10 q48
Answer:

Explanation:
I used the number sequence to find the greater number.I marked X on the number 7 as it is lessthan 9.

Question 9.
Envision Math Common Core Grade K Answer Key Topic 4 Compare Numbers 0 to 10 q49
Answer:

Explanation:
I wrote the next two numbers that are greater than 8. the numbers that are greater than 8 are 9 and 10.

Question 10.
Envision Math Common Core Grade K Answer Key Topic 4 Compare Numbers 0 to 10 q50
Answer:

Explanation:
I wrote the number 7 as it is greater than 5 and lessthan 9.

Directions Have students: 7 draw pictures to show how they know which number is less than the other number, and then mark an X on the number that is less; 8 use the number sequence to show how they know which number is less than the other number, and then mark an X on the number that is less. 9 Higher Order Thinking Have students write the next two numbers that are greater than the number shown, and then tell how they know. 10 Higher Order Thinking Have students write a number that is greater than the number on the left, but less than the number on the right.

Lesson 4.5 Repeated Reasoning

Problem Solving

Solve & Share

Envision Math Common Core Grade K Answer Key Topic 4 Compare Numbers 0 to 10 q51
Answer:

Directions Say: There are 7 fish in a bowl. Emily puts I more fish in the bowl. How many fish are in the bowl now? How can you solve this problem?

Visual Learning Bridge

Envision Math Common Core Grade K Answers Topic 4 Compare Numbers 0 to 10 q52

Guided Practice

Question 1.
Envision Math Common Core Grade K Answers Topic 4 Compare Numbers 0 to 10 q53
Answer:

Explanation:
I drew counters for 4 frogs and i drew one more.The number that is one greater than 4 is 5.

Directions 1 Say: Carlos sees 4 frogs at the pond. Then he sees 1 more. How many frogs are there now? Have students use reasoning to find the number that is 1 greater than the number of frogs shown. Draw counters to show the answer, and then write the number. Have students explain their reasoning.

Independent Practice

Question 2.
Envision Math Common Core Grade K Answers Topic 4 Compare Numbers 0 to 10 q54
Answer:

Explanation:
I drew counters for 7 frogs and i drew one more.The number that is one greater than 7 is 8.

Question 3.
Envision Math Common Core Grade K Answers Topic 4 Compare Numbers 0 to 10 q55
Answer:

Explanation:
I drew counters for 2 frogs and i drew one more.The number that is one greater than 2 is 3.

Question 4.
Envision Math Common Core Grade K Answers Topic 4 Compare Numbers 0 to 10 q56
Answer:

Explanation:
I drew counters for 8 frogs and i drew one more.The number that is one greater than 8 is 9.

Question 5.
Envision Math Common Core Grade K Answers Topic 4 Compare Numbers 0 to 10 q57.1
Answer:

Explanation:
I drew counters for 6 frogs and i drew one more.The number that is one greater than 6 is 7.

Directions Say: Alex sees frogs at the pond. Then he sees 1 more. How many frogs are there now? 2-5 Have students use reasoning to find the number that is 1 greater than the number of frogs shown. Draw counters to show the answer, and then write the number. Have students explain their reasoning.

Problem Solving

Performance Task

Question 6, 7, 8.
Envision Math Common Core Grade K Answers Topic 4 Compare Numbers 0 to 10 q57.2
Answer:

Explanation:
I drew counters for 5 pets of Marta, i drew 1 more counter.The number that is one greater than 5 is 6.Marta will now have 6 pets.

Directions Read the problem aloud. Then have students use multiple problem-solving methods to solve the problem. Say: Marta’s family has 5 pets. Then her family gets 1 more. How many pets do they have now? 6 Generalize Does something repeat in the problem? How does that help? 7 Use Tools What tool can you use to help solve the problem? Use the tool to find the number of pets in Marta’s family now. 8 Make Sense Should the answer be greater than or less than 5?

Topic 4 Vocabulary Review

Question 1.
Envision Math Common Core Grade K Answers Topic 4 Compare Numbers 0 to 10 q58
Answer:

Explanation:
I circled the number 9 as it is greater than 7.

Question 2.
Envision Math Common Core Grade K Answers Topic 4 Compare Numbers 0 to 10 q59
Answer:

Explanation:
I counted the counters given, there are 7 counters in all.so, i wrote the number 7 in the blank.

Question 3.
Envision Math Common Core Grade K Answers Topic 4 Compare Numbers 0 to 10 q60
Answer:

Explanation:
The number that means NONE is 0.

Question 4.
Envision Math Common Core Grade K Answers Topic 4 Compare Numbers 0 to 10 q61
Answer:

Explanation:
There are 2 red cubes and 3 yellow cubes.I circled the red cubes as they are less than yellow cubes.I wote the number 2 as it is lessthan 3.

Directions Understand Vocabulary Have students: 1 draw a circle around the number that is greater than 7; 2 count the counters, and then write the number to tell how many; 3 write the number that means none; 4 count how many of each color cube there is, draw a circle around the group that has a number of cubes that is less than the other group, and then write the number to tell how many there are in that group.

Question 5.
Envision Math Common Core Grade K Answers Topic 4 Compare Numbers 0 to 10 q62
Answer:

Explanation:
I circled the number 8 as it is greater than 3 and marked X on the number 3 as it is lessthan 8.

Question 6.
Envision Math Common Core Grade K Answers Topic 4 Compare Numbers 0 to 10 q63
Answer:

Explanation:
I wrote 4 as it is greater than 3 and lessthan 5.

Question 7.
Envision Math Common Core Grade K Answers Topic 4 Compare Numbers 0 to 10 q64
Answer:

Explanation:
I drew counters for the given number 5.

Question 8.
Envision Math Common Core Grade K Answers Topic 4 Compare Numbers 0 to 10 q65
Answer:

Explanation:
I wrote the missing numbers 3, 4, 6, 7 and 8 in the gien blanks.

Directions Understand Vocabulary Have students: 5 compare the numbers, draw a circle around the number that is greater, and then mark an X on the number that is less; 6 write a number that is greater than 3, but less than 5; 7 draw 5 counters in a row and then write the number to tell how many; 8 write the missing numbers in order.

Topic 4 Reteaching

Set A

Envision Math Common Core Grade K Answers Topic 4 Compare Numbers 0 to 10 q66

Question 1.
Envision Math Common Core Grade K Answers Topic 4 Compare Numbers 0 to 10 q67
Answer:

Explanation:
I counted the number of counters, there are 6 red counters and 10 yellow counters.I circled red counters frame as they are less in number than the yellow counter frame.

Set B

Envision Math Common Core Grade K Answers Topic 4 Compare Numbers 0 to 10 q68

Question 2.
Envision Math Common Core Grade K Answers Topic 4 Compare Numbers 0 to 10 q69
Answer:

Explanation:
I drew lines from the top group to a piece of fruit in the bottom group, and then i drew a circle around the number 7 as it is greater than 5.

Directions Have students: 1 compare the groups, and draw a circle around the group that is less in number than the other group; 2 count the fruit in each group, write the numbers that tell how many, draw a line from each piece of fruit in the top group to a piece of fruit in the bottom group, and then draw a circle around the number that is greater than the other number.

Set C

Envision Math Common Core Grade K Answers Topic 4 Compare Numbers 0 to 10 q70

Question 3.
Envision Math Common Core Grade K Answers Topic 4 Compare Numbers 0 to 10 q71
Answer:

Explanation:
I counted and wrote the number of blue critter and peach critters.They are 6 peach critters and 4 blue critters.
I marked X on the number 4 as it is lessthan 6.

Set D

Envision Math Common Core Grade K Answers Topic 4 Compare Numbers 0 to 10 q72

Question 4.
Envision Math Common Core Grade K Answers Topic 4 Compare Numbers 0 to 10 q73
Answer:

Explanation:
I drew counters for the 6 frogs and drew one more counter.1 greater than 6 is 7.

Directions Have students: 3 count the number of each critter, write the numbers, and then mark an X on the number that is less than the other number; 4 Say: April sees frogs at The pond. Then she sees 1 more. How many frogs does she see now? Have students use reasoning to find the number that is 1 greater than the number of frogs shown. Draw counters to show the answer, and then write the number.

Topic 4 Assessment Practice

Question 1.
Envision Math Common Core Grade K Answers Topic 4 Compare Numbers 0 to 10 q74
Answer:

Explanation:
There are 8 yellow birds.I marked group ‘A’ as it has more number of blue birds than the yellow birds.There are 10 blue bieds in group A.

Question 2.
Envision Math Common Core Grade K Answers Topic 4 Compare Numbers 0 to 10 q75
Answer:

Explanation:
I marked the numbers 6, 5 and 3.The numbers 3, 5 and 6 as they are lessthan the number 7.

Question 3.
Envision Math Common Core Grade K Answers Topic 4 Compare Numbers 0 to 10 q76
Answer:

Explanation:
I counted and wrote the number of lemons and limes, there are 7 lemons and 5 limes.
I circled the number 7 as it is greater than 5.

Directions Have students mark the best answer. 1 Which group of blue birds is greater in number than the group of yellow birds? 2 Look at the number line. Then mark all the numbers that are less than the number on the card. 3 Have students count the number of lemons and limes, write the number that tells how many of each, and then draw a circle around the number that is greater.

Question 4.
Envision Math Common Core Grade K Answers Topic 4 Compare Numbers 0 to 10 q77
Answer:

Explanation:
The number of the first card is 7, the number before 7 is 6.I counted forward from 6 and wrote the numbers 7, 8, 9 and 10 in the blanks.

Question 5.
Envision Math Common Core Grade K Answers Topic 4 Compare Numbers 0 to 10 q78
Answer:

Explanation:
There are 6 sandwiches and I drew the 4 juice boxes as 4 is lessthan 6.

Question 6.
Envision Math Common Core Grade K Answers Topic 4 Compare Numbers 0 to 10 q79
Answer:

Explanation:
I drew counters for 7 beads and drew 1 more counter.one greater than the number 7 is 8.
Kayla will now have 8 beads to make a bracelet.

Directions Have students: 4 write the number that is counted first among the 4 number cards, and then count forward and write the number that is 1 greater than the number before; 5 count the sandwiches in the group, draw a group of juice boxes that is less than the group of sandwiches shown, and then write the numbers to tell how many. 6 Say: Kayla has 7 beads to make a bracelet. Then she buys 1 more. How many beads does she have now? Have students use reasoning to find the number that is 1 greater than the number of beads shown. Draw counters to show the answer, and then write the number to tell how many.

Topic 4 Performance Task

Envision Math Common Core Grade K Answers Topic 4 Compare Numbers 0 to 10 q80

Question 1.
Envision Math Common Core Grade K Answers Topic 4 Compare Numbers 0 to 10 q81
Answer:

Explanation:
There are 9 shunks and 8 raccoons in the forest.
Using number sequence i found that number 8 is lessthan number 9 so, i marked X on the number 8.

Directions Forest Animals Say: The forest is home to woodland animals. One part of the forest has many different animal homes in it. 1 Have students study the picture. Say: How many skunks live in this part of the forest? How many raccoons live in this part of the forest? Count the number of each type of animal and write the numbers. Then have students draw a circle around the number that is greater than the other number and mark an X on the number that is less than the other number. Have them use the number sequence to help find the answers.

Question 2.
Envision Math Common Core Grade K Answers Topic 4 Compare Numbers 0 to 10 q82
Answer:

Explanation:
There are 6 foxes in the forest.I counted and wrote the numbers between 6 and 10.They are 7, 8, 9 and 10.

Question 3.
Envision Math Common Core Grade K Answers Topic 4 Compare Numbers 0 to 10 q83
Answer:

Explanation:
I drew counters for the number 5, I drew circle around both the numbers as they are equal.

Question 4.
Envision Math Common Core Grade K Answers Topic 4 Compare Numbers 0 to 10 q84
Answer:

Explanation:
There are 7 birds in the forest.I drew counters for 7 birds and one more counter as 1 bird flies into the forest.
One greater than the number 7 is 8.

Directions Have students look at the picture on the page before. 2 Say: How many foxes live in this part of the forest? Count how many and write the number. Then have students write all the numbers through 10 that are greater than the number of foxes. 3 Say: 5 chipmunks and 5 frogs move out of this part of the forest. Draw a circle around both numbers if they are equal, or mark an X on both numbers if they are NOT equal. Show how you know you are correct. 4 Say: How many birds live in this part of the forest? Count how many and write the number. I more bird flies into the forest. How many birds are in this part of the forest now? Have students use tools to solve the problem and write the number. Then have them show how they found the answer.

Envision Math Common Core Grade K Answer Key Topic 1 Numbers 0 to 5

Practice with the help of enVision Math Common Core Kindergarten Answer Key Topic 1 Numbers 0 to 5 regularly and improve your accuracy in solving questions.

Envision Math Common Core Grade K Answers Key Topic 1 Numbers 0 to 5

Essential Question:
How can numbers from 0 to 5 be counted, read, and written?
Envision Math Common Core Grade K Answer Key Topic 1 Numbers 0 to 5 q186

Directions Read the character speech bubbles to students. Find Out! Have students pay attention to the daily weather changes. Say: The weather changes from day to day. Talk to friends and relatives about the weather. Ask them to help you record the number of sunny days and rainy days from Monday to Friday. Journal: Make a Poster Have students make a poster of the weather information they collected. Have them draw suns for the number of sunny days and clouds with raindrops for the number of rainy days. Then have students write the numbers to tell how many.

Review What You Know

Question 1.
Envision Math Common Core Grade K Answer Key Topic 1 Numbers 0 to 5 q2
Answer:

Explanation:
I circled the animal to the right.

Question 2.
Envision Math Common Core Grade K Answer Key Topic 1 Numbers 0 to 5 q3
Answer:

Explanation:
I circled the animal to the left.

Question 3.
Envision Math Common Core Grade K Answer Key Topic 1 Numbers 0 to 5 q4
Answer:

Explanation:
I circled the animal that is green in color.

Question 4.
Envision Math Common Core Grade K Answer Key Topic 1 Numbers 0 to 5 q5
Answer:

Explanation:
I drew lines from each object in the top row to bottom row.

Question 5.
Envision Math Common Core Grade K Answer Key Topic 1 Numbers 0 to 5 q6
Answer:

Explanation:
I drew lines from each object in the top row to bottom row.

Question 6.
Envision Math Common Core Grade K Answer Key Topic 1 Numbers 0 to 5 q7
Answer:

Explanation:
I drew lines from each object in the top row to bottom row.

Directions Have students: 1 draw a circle around the animal that is on the right; 2 draw a circle around the animal that is on the left; 3 draw a circle around the animal that is green; 4-6 draw a line from each object in the top row to an object in the bottom row.

Pick a Project

Envision Math Common Core Grade K Answer Key Topic 1 Numbers 0 to 5 q8

Envision Math Common Core Grade K Answer Key Topic 1 Numbers 0 to 5 q9

Envision Math Common Core Grade K Answer Key Topic 1 Numbers 0 to 5 q10

Directions Say: You will choose one of these projects. Look at picture A. Think about this question: What would animals say if they could talk? If you choose Project A, you will create funny animal characters and tell a story. Look at picture B. Think about this question: Does your favorite sport or game use a ball? If you choose Project B, you will make a poster of different balls that are used in sports and games. Look at picture C. Think about this question: What would you want to show on a license plate? If you choose Project C, you will create a license plate for your bike.

3-ACT MATH PREVIEW

Math Modeling

Set the Table

Envision Math Common Core Grade K Answer Key Topic 1 Numbers 0 to 5 q11

Directions Read the robot’s speech bubble to students. Generate Interest Ask students if they have ever set the table. Say: Who sets the table before meals? What chores are you responsible for at home? Have them share stories about responsibilities and rules around the house.

Lesson 1.1 Count 1, 2 and 3

Solve & Share

Envision Math Common Core Grade K Answer Key Topic 1 Numbers 0 to 5 q12
Answer:

Explanation:
I drew 2 counters in the nest and circled the colored box that shows the number 2 as 2 boxes are colored.

Directions Have students place 2 counters in the nest on the workmat. Say: Peeps the bird found these worms for her babies. Draw a circle around the colored box that shows how many worms Peeps found. Tell how you know you are correct.

Visual Learning Bridge

Envision Math Common Core Grade K Answer Key Topic 1 Numbers 0 to 5 q13

Guided Practice

Question 1.
Envision Math Common Core Grade K Answer Key Topic 1 Numbers 0 to 5 q14
Answer:

Explanation:
I colored a box as i counted each worm.There are 2 worms.

Question 2.
Envision Math Common Core Grade K Answer Key Topic 1 Numbers 0 to 5 q15
Answer:

Explanation:
I colored a box as i counted each worm.There is 1 worm.

Directions 1 and 2 Have students color a box as they count each worm to show how many.

Question 3.
Envision Math Common Core Grade K Answer Key Topic 1 Numbers 0 to 5 q16
Answer:

Explanation:
I colored a box as i counted each worm.There are 3 worms.

Question 4.
Envision Math Common Core Grade K Answer Key Topic 1 Numbers 0 to 5 q17
Answer:

Explanation:
I colored a box as i counted each worm.There are 2 worms.

Question 5.
Envision Math Common Core Grade K Answer Key Topic 1 Numbers 0 to 5 q18
Answer:

Explanation:
I colored a box as i counted each worm.There are 3 worms.

Directions 3 and 4 Have students color a box as they count each worm to show how many. 5 Vocabulary Have students count the worms, and color a box as they count each worm aloud.
Envision Math Common Core Grade K Answer Key Topic 1 Numbers 0 to 5 q19

Independent Practice

Question 6.
Envision Math Common Core Grade K Answer Key Topic 1 Numbers 0 to 5 q20
Answer:

Explanation:
I colored a box as i counted each nest.There are 2 nests.

Question 7.
Envision Math Common Core Grade K Answer Key Topic 1 Numbers 0 to 5 q21
Answer:

Explanation:
I colored a box as i counted each nest.There is 1 nest.

Question 8.
Envision Math Common Core Grade K Answer Key Topic 1 Numbers 0 to 5 q22
Answer:

Explanation:
I colored a box as i counted each nest.There are 3 nests.

Question 9.
Envision Math Common Core Grade K Answer Key Topic 1 Numbers 0 to 5 q23
Answer:

Explanation:
I drew 2 nests and colored 2 boxes as i counted the number of nests.

Directions 6-8 Have students color a box as they count each nest to show how many. 9 Higher Order Thinking Have students draw 1, 2, or 3 nests, and then color a box as they draw each nest to show how many.
Envision Math Common Core Grade K Answer Key Topic 1 Numbers 0 to 5 q24

Lesson 1.2 Recognize 1, 2 and 3 in Different Arrangements

Solve & Share

Envision Math Common Core Grade K Answer Key Topic 1 Numbers 0 to 5 q25

Directions Say: Redbird and Bluebird each have 2 babies. Redbird and Blue bird get worms for their babies and put them in their nests. Bluebird’s worms moved around in the nest. Show and count how many worms with your counters. Color the boxes to show the worms Th each nest. Tell how you know you are correct.

Visual Learning Bridge

Envision Math Common Core Grade K Answer Key Topic 1 Numbers 0 to 5 q26

Guided Practice

Question 1.
Envision Math Common Core Grade K Answer Key Topic 1 Numbers 0 to 5 q27
Answer:

Explanation:
I counted each bird, there are 2 birds and i colored 2 boxes.

Question 2.
Envision Math Common Core Grade K Answer Key Topic 1 Numbers 0 to 5 q28
Answer:

Explanation:
I counted each bird, there are 3 birds and i colored 3 boxes.

Directions 1 and 2 Have students count each bird, and then color the boxes to show how many.

Question 3.
Envision Math Common Core Grade K Answer Key Topic 1 Numbers 0 to 5 q29
Answer:

Explanation:
I counted each bird, there are 3 birds and i colored 3 boxes.

Question 4.
Envision Math Common Core Grade K Answer Key Topic 1 Numbers 0 to 5 q30
Answer:

Explanation:
I counted each bird, there are 2 birds and i colored 2 boxes.

Question 5.
Envision Math Common Core Grade K Answer Key Topic 1 Numbers 0 to 5 q31
Answer:

Explanation:
I counted each bird, there is 1 bird and i colored 1 box.

Question 6.
Envision Math Common Core Grade K Answer Key Topic 1 Numbers 0 to 5 q32
Answer:

Explanation:
I counted each bird, there are 3 birds and i colored 3 boxes.

Question 7.
Envision Math Common Core Grade K Answer Key Topic 1 Numbers 0 to 5 q33
Answer:

Explanation:
I counted each bird, there is 1 bird and i colored 1 box.

Question 8.
Envision Math Common Core Grade K Answer Key Topic 1 Numbers 0 to 5 q34
Answer:

Explanation:
I counted each bird, there are 2 birds and i colored 2 boxes.

Directions 3-8 Have students count the dots, and then color the boxes to show how many.
Envision Math Common Core Grade K Answer Key Topic 1 Numbers 0 to 5 q35

Independent Practice

Question 9.
Envision Math Common Core Grade K Answer Key Topic 1 Numbers 0 to 5 q36
Answer:

Explanation:
I counted each dot, there is 1 dot and i colored 1 box.

Question 10.
Envision Math Common Core Grade K Answer Key Topic 1 Numbers 0 to 5 q37
Answer:

Explanation:
I counted each dot, there are 3 dots and i colored 3 boxes.

Question 11.
Envision Math Common Core Grade K Answer Key Topic 1 Numbers 0 to 5 q38
Answer:

Explanation:
I counted each dot, there are 2 dots and i colored 2 boxes.

Question 12.

Envision Math Common Core Grade K Answer Key Topic 1 Numbers 0 to 5 q39
Answer:

Explanation:
I counted each dot, there are 3 dots and i colored 3 boxes.

Question 13.
Envision Math Common Core Grade K Answer Key Topic 1 Numbers 0 to 5 q40
Answer:

Explanation:
I drew 2 counters in two different ways.

Directions 9-12 Have students count the dots, and then color the boxes to show how many. 13 Higher Order Thinking Have students draw 2 counters, and then draw 2 counters in a different way.
Envision Math Common Core Grade K Answer Key Topic 1 Numbers 0 to 5 q41

Lesson 1.3 Read, Make, and Write 1, 2 and 3

Envision Math Common Core Grade K Answer Key Topic 1 Numbers 0 to 5 q42
Answer:

Explanation:
I drew 2 counters in the big cloud.I drew 2 circles to show that Alex saw 2 stars.

Directions Say: Alex sees 2 stars in the sky. He draws 2 stars in a cloud. Place counters in the large cloud on the workmat to show how many stars. How can you show how many stars Alex sees in other ways? Draw or use objects to show other ways in the small, empty cloud.

Visual Learning Bridge

Envision Math Common Core Grade K Answer Key Topic 1 Numbers 0 to 5 q43

Guided Practice

Question 1.
Envision Math Common Core Grade K Answer Key Topic 1 Numbers 0 to 5 q44
Answer:

Explanation:
I counted the stars.There is 1 star.SO, i wrote number 1 in the blank.

Question 2.
Envision Math Common Core Grade K Answer Key Topic 1 Numbers 0 to 5 q45
Answer:

Explanation:
I counted the stars.They are 2 in number.So, i wrote the number 2 in the blank.

Question 3.
Envision Math Common Core Grade K Answer Key Topic 1 Numbers 0 to 5 q46
Answer:

Explanation:
I counted the stars.They are 3 in number.So, i wrote the number 3 in the blank.

Directions 1-3 Have students count the stars, and then write the number to tell how many.

Question 4.
Envision Math Common Core Grade K Answer Key Topic 1 Numbers 0 to 5 q46.1
Answer:

Explanation:
I counted the smilies.There is 1 smilie.So, i practiced writing the number 1..

Question 5.
Envision Math Common Core Grade K Answer Key Topic 1 Numbers 0 to 5 q48
Answer:

Explanation:
I counted the smilies.There are 2 smilies.So, i  practiced writing the number 2.

Question 6.
Envision Math Common Core Grade K Answer Key Topic 1 Numbers 0 to 5 q49
Answer:

Explanation:
I counted the smilies.There are 3 smilies.So, i practiced writing the number 3.

Question 7.
Envision Math Common Core Grade K Answer Key Topic 1 Numbers 0 to 5 q50
Answer:

I counted the smilies.There is 1 smilie.So, i practiced writing the number 1.

Directions 4-7 Have students count the objects, and then practice writing the number that tells how many.
Envision Math Common Core Grade K Answer Key Topic 1 Numbers 0 to 5 q51

Independent Practice

Question 8.
Envision Math Common Core Grade K Answer Key Topic 1 Numbers 0 to 5 q52
Answer:

Explanation:
I used 2 counters to make the number 2, drew circles to represent it.

Question 9.
Envision Math Common Core Grade K Answer Key Topic 1 Numbers 0 to 5 q53
Answer:

Explanation:
I used 2 counters to make the number 3, drew circles to represent it.

Question 10.
Envision Math Common Core Grade K Answer Key Topic 1 Numbers 0 to 5 q54
Answer:

Explanation:
I used 1 counter to make the number 1, drew circles to represent it.

Question 11.
Envision Math Common Core Grade K Answer Key Topic 1 Numbers 0 to 5 q55
Answer:

Explanation:
I drew 3 stars and practiced writing the number 3.

Directions 8-10 Have students use counters to make the number. Then have them draw circles to represent the number. 11 Higher Order Thinking Have students draw 1, 2, or 3 stars, and then practice writing the number that tells how many.

Lesson 1.4 Count 4 and 5

Solve & Share

Envision Math Common Core Grade K Answer Key Topic 1 Numbers 0 to 5 q56
Answer:

Directions Have students place 5 counters in the tree on the workmat. Then say: Chips the chipmunk found these nuts. Draw a circle around the colored box that shows how many nuts Chips found. Tell how you know you are correct.

Visual Learning Bridge

Envision Math Common Core Grade K Answer Key Topic 1 Numbers 0 to 5 q57

Guided Practice

Envision Math Common Core Grade K Answer Key Topic 1 Numbers 0 to 5 q58

Question 1.
Envision Math Common Core Grade K Answer Key Topic 1 Numbers 0 to 5 q59
Answer:

Explanation:
I colored a box as i counted each orange.There are 4 oranges.

Question 2.
Envision Math Common Core Grade K Answer Key Topic 1 Numbers 0 to 5 q60
Answer:

Explanation:
I colored a box as i counted each orange.There are 5 oranges.

Directions 1 and 2 Have students color a box as they count each orange to show how many.

Question 3.
Envision Math Common Core Grade K Answer Key Topic 1 Numbers 0 to 5 q61
Answer:

Explanation:
I colored a box as i counted each orange.There are 4 oranges.

Question 4.
Envision Math Common Core Grade K Answer Key Topic 1 Numbers 0 to 5 q62
Answer:

Explanation:
I colored a box as i counted each fruit.There are 4 fruits.

Question 5.
Envision Math Common Core Grade K Answer Key Topic 1 Numbers 0 to 5 q63
Answer:

Explanation:
I colored a box as i counted each fruit.There are 5 fruits.

Question 6.
Envision Math Common Core Grade K Answer Key Topic 1 Numbers 0 to 5 q64
Answer:

Explanation:
I colored a box as i counted each fruit.There are 4 fruits.

Directions 3-6 Have students color a box as they count each piece of fruit to show how many.

Independent Practice

Question 7.
Envision Math Common Core Grade K Answer Key Topic 1 Numbers 0 to 5 q65
Answer:

Explanation:
I colored a box as i counted each fruit.There are 5 fruits.

Question 8.
Envision Math Common Core Grade K Answer Key Topic 1 Numbers 0 to 5 q66
Answer:

Explanation:
I colored a box as i counted each fruit.There are 4 fruits.

Question 9.
Envision Math Common Core Grade K Answer Key Topic 1 Numbers 0 to 5 q67
Answer:

Explanation:
I colored a box as i counted each fruit.There are 5 fruits.

Question 10.
Envision Math Common Core Grade K Answer Key Topic 1 Numbers 0 to 5 q68
Answer:

Explanation:
I colored a box as i drew each orange.I drew 4 oranges.

Directions 7-9 Have students color a box as they count each piece of fruit to show how many. 10 Higher Order Thinking Have students draw 4 or 5 oranges, and then color a box as they draw each orange to show how many.

Lesson 1.5 Recognize 4 and 5 in Different Arrangements

Envision Math Common Core Grade K Answer Key Topic 1 Numbers 0 to 5 q69

Directions Say: Some bees are flying around before landing on their beehives. Then 4 bees land on one hive and 4 bees land on the other hive. Use cubes to show different ways the bees can land on the hives. Count the bees and color the boxes to show how many landed on each hive. Tell how you know you are correct.

Visual Learning Bridge

Envision Math Common Core Grade K Answer Key Topic 1 Numbers 0 to 5 q70

Guided Practice

Question 1.
Envision Math Common Core Grade K Answer Key Topic 1 Numbers 0 to 5 q71
Answer:

Explanation:
I counted the bees, there are 4 bees.So, i colored 4 boxes.

Question 2.
Envision Math Common Core Grade K Answer Key Topic 1 Numbers 0 to 5 q72
Answer:

Explanation:
I counted the cats.There are 5 cats.SO, i colored 5 boxes.

Directions 1 and 2 Have students count the insects or animals in each group, and then color the boxes to show how many.

Question 3.
Envision Math Common Core Grade K Answer Key Topic 1 Numbers 0 to 5 q73
Answer:

Explanation:
I counted the number of birds.There are 4 birds.So, i colored 4 boxes.

Question 4.
Envision Math Common Core Grade K Answer Key Topic 1 Numbers 0 to 5 q74
Answer:

Explanation:
I counted the number of birds.There are 5 birds.So, i colored 5 boxes.

Question 5.
Envision Math Common Core Grade K Answer Key Topic 1 Numbers 0 to 5 q75
Answer:

Explanation:
I counted the number of birds.There are 5 birds.So, i colored 5 boxes.

Question 6.
Envision Math Common Core Grade K Answer Key Topic 1 Numbers 0 to 5 q76
Answer:

Explanation:
I counted the number of birds.There are 4 birds.So, i colored 4 boxes.

Question 7.
Envision Math Common Core Grade K Answer Key Topic 1 Numbers 0 to 5 q77
Answer:

Explanation:
I counted the number of birds.There are 5 birds.So, i colored 5 boxes.

Question 8.
Envision Math Common Core Grade K Answer Key Topic 1 Numbers 0 to 5 q78
Answer:

Explanation:
I counted the number of birds.There are 4 birds.So, i colored 4 boxes.

Directions 3-8 Have students count the birds, and then color the boxes to show how many.

Independent Practice

Question 9.
Envision Math Common Core Grade K Answer Key Topic 1 Numbers 0 to 5 q79
Answer:

Explanation:
I counted the number of dots.There are 4 dotss.So, i colored 4 boxes.

Question 10.
Envision Math Common Core Grade K Answer Key Topic 1 Numbers 0 to 5 q80
Answer:

Explanation:
I counted the number of dots.There are 5 dots.So, i colored 5 boxes.

Question 11.
Envision Math Common Core Grade K Answer Key Topic 1 Numbers 0 to 5 q81
Answer:

Explanation:
I counted and circled those groups that are 4 in number.

Question 12.
Envision Math Common Core Grade K Answer Key Topic 1 Numbers 0 to 5 q82
Answer:

Explanation:
I counted and circled those groups that are 5 in number.

Question 13.
Envision Math Common Core Grade K Answer Key Topic 1 Numbers 0 to 5 q83
Answer:

Explanation:
I drew 5 counters in the first space and drew 5 counters in two different ways in the next two spaces.

Directions Have students: 9 and 10 count the dots, and then color the boxes to show how many; 11 count the groups, and then draw a circle around the groups that show 4; 12 count the groups, and then draw a circle around the groups that show 5. 13 Higher Order Thinking Have students draw 5 counters in the first space, and then draw 5 counters in two different ways in the other two spaces.

Lesson 1.6 Read, Make, and Write 4 and 5

Envision Math Common Core Grade K Answer Key Topic 1 Numbers 0 to 5 q84
Answer:

Explanation:
I drew 5 counters and 5 circles to show the numbe rof forgs.

Directions Say: Alex sees some frogs sitting on a lily pod. Show how many frogs he sees by placing counters on the large lily pad. Count and tell how many. Then draw or use objects to show how many frogs ¡n other ways on the small, empty lily pod on the workmat.

Visual Learning Bridge

Envision Math Common Core Grade K Answer Key Topic 1 Numbers 0 to 5 q85

Guided Practice

Question 1.
Envision Math Common Core Grade K Answer Key Topic 1 Numbers 0 to 5 q86
Answer:

Explanation:
I counted the butterflies.They are 4 in number.I practiced writing the number 4.

Question 2.
Envision Math Common Core Grade K Answer Key Topic 1 Numbers 0 to 5 q87
Answer:

Explanation:
I counted the butterflies.They are 5 in number.I practiced writing the number 5.

Directions 1 and 2 Have students count the butterflies, and then practice writing the number that tells how many.

Question 3.
Envision Math Common Core Grade K Answer Key Topic 1 Numbers 0 to 5 q88
Answer:

Explanation:
I counted the frogs.They are 4 in number.I practiced writing the number 4.

Question 4.
Envision Math Common Core Grade K Answer Key Topic 1 Numbers 0 to 5 q89
Answer:

Explanation:
I counted the frogs.They are 5 in number.I practiced writing the number 5.

Question 5.
Envision Math Common Core Grade K Answer Key Topic 1 Numbers 0 to 5 q90
Answer:

Explanation:
I counted the frogs.They are 4 in number.I practiced writing the number 4.

Question 6.
Envision Math Common Core Grade K Answer Key Topic 1 Numbers 0 to 5 q91
Answer:

Explanation:
I counted the frogs.They are 5 in number.I practiced writing the number 5.

Directions 3-6 Have students count the frogs, and then practice writing the number that tells how many.

Independent Practice

Question 7.
Envision Math Common Core Grade K Answer Key Topic 1 Numbers 0 to 5 q92
Answer:

Explanation:
I drew 4 counters and 4 squares to represent the number 4.

Question 8.
Envision Math Common Core Grade K Answer Key Topic 1 Numbers 0 to 5 q93
Answer:

Explanation:
I drew 5 counters and 5 squares to represent the number 5.

Question 9.
Envision Math Common Core Grade K Answer Key Topic 1 Numbers 0 to 5 q94
Answer:

Explanation:
I counted the birds in the above picture.There are 5 blue birds and 4 yellow birds.So, i colored 5 for blue bird and 4 boxes for yellow bird and wrote the numbers 5 and 4 in the blanks.

Directions 7 and 8 Have students use counters to make the number. Then have them draw squares to represent the number. 9 Higher Order Thinking Have students count the blue birds and the yellow birds, color a box for each bird, and then write the numbers to tell how many.

Lesson 1.7 Identify the Number 0

Solve & Share

Envision Math Common Core Grade K Answer Key Topic 1 Numbers 0 to 5 q95

Directions Say: Alex has a vegetable garden. Toss a number cube to see how many potatoes he has in his basket. Place counters on the basket to show how many. What does it mean if the cube face you roll has no dots? How can Alex use the boxes to show that there are no potatoes in the basket?

Visual Learning Bridge

Envision Math Common Core Grade K Answer Key Topic 1 Numbers 0 to 5 q96

Guided Practice

Question 1.
Envision Math Common Core Grade K Answer Key Topic 1 Numbers 0 to 5 q97
Answer:

Explanation:
I colored a box as i counted each apple.There are 3apples.

Question 2.
Envision Math Common Core Grade K Answer Key Topic 1 Numbers 0 to 5 q98
Answer:

Explanation:
I dinot color any box as the plate is empty.

Directions 1 and 2 Have students color a box as they count each apple to show how many.

Question 3.
Envision Math Common Core Grade K Answer Key Topic 1 Numbers 0 to 5 q99
Answer:

Explanation:
I colored a box as i counted each banana.There are 2 bananas.

Question 4.
Envision Math Common Core Grade K Answer Key Topic 1 Numbers 0 to 5 q100
Answer:

Explanation:
I colored a box as i counted each strawberry.There are 5 strawberries.

Question 5.
Envision Math Common Core Grade K Answer Key Topic 1 Numbers 0 to 5 q101
Answer:

Explanation:
I dinot color any box as the plate is empty.

Question 6.
Envision Math Common Core Grade K Answer Key Topic 1 Numbers 0 to 5 q102
Answer:

Explanation:
I colored a box as i counted each strawberry.There are 4 strawberries.

Question 7.
Envision Math Common Core Grade K Answer Key Topic 1 Numbers 0 to 5 q103
Answer:

Explanation:
I colored a box as i counted each banana.There are 3 bananas.

Question 8.
Envision Math Common Core Grade K Answer Key Topic 1 Numbers 0 to 5 q104
Answer:

Explanation:
I dinot color any box as the plate is empty.

Directions 3-8 Have students color a box as they count each piece of fruit to show how many.

Independent Practice

Question 9.
Envision Math Common Core Grade K Answer Key Topic 1 Numbers 0 to 5 q105
Answer:

Explanation:
I dinot color any box as the vase is empty.

Question 10.
Envision Math Common Core Grade K Answer Key Topic 1 Numbers 0 to 5 q106
Answer:

Explanation:
I counted and colored 1 box as there is 1 flower in the box.

Question 11.
Envision Math Common Core Grade K Answer Key Topic 1 Numbers 0 to 5 q107
Answer:

Explanation:
I counted and colored 3 boxes as there are 3 flowers in the vase.

Question 12.
Envision Math Common Core Grade K Answer Key Topic 1 Numbers 0 to 5 q108
Answer:

Explanation:
I counted and colored 3 boxes as there are 5 flowers in the vase.

Question 13.
Envision Math Common Core Grade K Answer Key Topic 1 Numbers 0 to 5 q109
Answer:

Explanation:
I dinot color any box as the vase is empty.

Question 14.
Envision Math Common Core Grade K Answer Key Topic 1 Numbers 0 to 5 q110
Answer:

Explanation:
I counted and colored 3 boxes as there are 2 flowers in the vase.

Question 15.
Envision Math Common Core Grade K Answer Key Topic 1 Numbers 0 to 5 q111
Answer:

Explanation:
I drew 3 flowers and colored 3 boxes to show the number of flowers.

Directions 9-14 Have students color a box as they count each flower in the vase to show how many. 15 Higher Order Thinking Have students pick a number between 0 and 5, draw that many flowers, and then color the boxes to show how many.

Lesson 1.8 Read and Write 0

Solve & Share

Envision Math Common Core Grade K Answer Key Topic 1 Numbers 0 to 5 q112
Answer:

Explanation:
I counted the number of colored boxes next to each crayon on the workmat and colored crayons inside the bag.There are 2 blue crayons, 3 red crayons, 1 yellow crayon in the bag.

Directions Say: Count the number of colored boxes next to each crayon on the workmat. Then color the crayons inside the bag to show how many of each type Alex has and tell how you know. How many green crayons does Alex have? Tell how you know.

Visual Learning Bridge

Envision Math Common Core Grade K Answer Key Topic 1 Numbers 0 to 5 q113

Guided Practice

Question 1.
Envision Math Common Core Grade K Answer Key Topic 1 Numbers 0 to 5 q114
Answer:

Explanation:
I counted the pencils in each pencil holder, and then practiced writing the number3.There are 3 pencils.

Question 2.
Envision Math Common Core Grade K Answer Key Topic 1 Numbers 0 to 5 q115
Answer:

Explanation:
I counted the pencils in each pencil holder, and then practiced writing the number 0.There are no pencils.

Directions 1 and 2 Have students count the pencils in each pencil holder, and then practice writing the number that tells how many.

Question 3.
Envision Math Common Core Grade K Answer Key Topic 1 Numbers 0 to 5 q116
Answer:

Explanation:
I counted the pencils in each pencil holder, and then practiced writing the number 2.There are 2 pencils.

Question 4.
Envision Math Common Core Grade K Answer Key Topic 1 Numbers 0 to 5 q117
Answer:

Explanation:
I counted the pencils in each pencil holder, and then practiced writing the number 3.There are 3 pencils.

Question 5.
Envision Math Common Core Grade K Answer Key Topic 1 Numbers 0 to 5 q118
Answer:

Explanation:
I counted the pencils in each pencil holder, and then practiced writing the number 0.There are no pencils.

Question 6.
Envision Math Common Core Grade K Answer Key Topic 1 Numbers 0 to 5 q119
Answer:

Explanation:
I counted the pencils in each pencil holder, and then practiced writing the number 1.There is 1 pencil.

Question 7.
Envision Math Common Core Grade K Answer Key Topic 1 Numbers 0 to 5 q120
Answer:

Explanation:
I counted the pencils in each pencil holder, and then practiced writing the number 2.There are 2 pencils.

Question 8.
Envision Math Common Core Grade K Answer Key Topic 1 Numbers 0 to 5 q121
Answer:

Explanation:
I counted the pencils in each pencil holder, and then practiced writing the number 2.There are 2 pencils.

Directions 3-8 Have students count the balls in each box, and then practice writing the number that tells how many.

Independent Practice

Question 9.
Envision Math Common Core Grade K Answer Key Topic 1 Numbers 0 to 5 q122
Answer:

Explanation:
I counted the balls in each box, and then practiced writing the number 0.There are no balls.

Question 10.
Envision Math Common Core Grade K Answer Key Topic 1 Numbers 0 to 5 q123
Answer:

Explanation:
I counted the balls in each box, and then practiced writing the number 2.There are 2 balls.

Question 11.
Envision Math Common Core Grade K Answer Key Topic 1 Numbers 0 to 5 q124
Answer:

Explanation:
I counted the balls in each box, and then practiced writing the number 1.There is 1 ball.

Question 12.
Envision Math Common Core Grade K Answer Key Topic 1 Numbers 0 to 5 q125
Answer:

Explanation:
I counted the balls in each box, and then practiced writing the number 0.There are no balls.

Question 13.
Envision Math Common Core Grade K Answer Key Topic 1 Numbers 0 to 5 q126
Answer:

Explanation:
I counted and wrote the numbers from 0 to 5.

Question 14.
Envision Math Common Core Grade K Answer Key Topic 1 Numbers 0 to 5 q127
Answer:

Explanation:
I drew 0 counters and wrote the number 0 in the blank and drew 4 counters and wrote number 4 in the box.

Directions Have students: 9-12 count the balls in each box, and then practice writing the number that tells how many; 13 practice writing the numbers 0 to 5. 14 Higher Order Thinking Have students draw zero counters and write the number to tell how many, and then draw 1 to 5 counters and write the number to tell how many.

Lesson 1.9 Numbers to 5

Envision Math Common Core Grade K Answer Key Topic 1 Numbers 0 to 5 q128
Answer:

Explanation:
The numbe before 4 is 3 and after 4 is 5.So, i wrote the numbers 3 and 5 and drew counters to show the numbers3, 4 and 5.

Directions Say: Marta is thinking of two numbers-one is the number that comes just before 4 when counting, and the other is the number that comes just after 4 when counting. Write the two numbers Marta is thinking of. Show how you know you are correct.

Visual Learning Bridge

Envision Math Common Core Grade K Answer Key Topic 1 Numbers 0 to 5 q129

Guided Practice

Question 1.
Envision Math Common Core Grade K Answer Key Topic 1 Numbers 0 to 5 q130
Answer:

Explanation:
The number that comes just before 1 is 0 and after 1 is 3
The number that comes just before 4 is 3 and after 4 is 5
So, i wrote the numbers 0, 2, 3 and 5 in the blanks.

Directions 1 Have students write the number that comes just before 1 and the number that comes just after 1. Then have them write the number that comes just before 4 when counting, and the number that comes just after 4 when counting. Have them say the numbers in order from 0 to 5.

Question 2.
Envision Math Common Core Grade K Answer Key Topic 1 Numbers 0 to 5 q131
Answer:

Explanation:
I colored the cubes as i counted the numbers from 0 to 5 and wrote the numbers 0, 1, 2, 3, 4 and 5.

Question 3.
Envision Math Common Core Grade K Answer Key Topic 1 Numbers 0 to 5 q132
Answer:

Explanation:
I counted the flowers in each vase and wrote the numbers 0, 1, 2, 3, 4 and 5 in the blanks that tell how many.The number that comes just after 1 is 2 so, i circled 2.

Directions Have students: 2 color the cubes to show each number, write the numbers in order, and then draw a circle around the number that comes just after 1 when counting; 3 count the flowers in each vase, write the numbers, and then say the numbers in order from 0 to 5.

Independent Practice

Question 4.
Envision Math Common Core Grade K Answer Key Topic 1 Numbers 0 to 5 q133
Answer:

Explanation:
I counted the balls in each boxes and wrote the numbers 0, 1, 2, 3, 4 and 5 in the blanks that tell how many.The number that comes just after 4 is 5 so, i circled 5.

Question 5.
Envision Math Common Core Grade K Answer Key Topic 1 Numbers 0 to 5 q134
Answer:

Explanation:
I counted backwards from 5 to 0 and colored the cubes to match with the number.I wrote the numbers 5, 4 , 3, 2, 1 and 0.

Directions 4 Have students count the toys in each box, write the numbers, and then draw a circle around the number that comes just after 4 when counting how many. 5 Higher Order Thinking Have students color 5 cubes, and then write the number. Have them color cubes to show the number that comes next when counting backwards from 5, and then write the number of cubes they colored in the tower. Repeat for the remaining towers.

Lesson 1.10 Construct Arguments

Solve & Share

Envision Math Common Core Grade K Answer Key Topic 1 Numbers 0 to 5 q135
Answer:

Directions Say: Alex needs to count the group of shapes. How can you count these shapes? Use objects or words to help. Write the number to tell how many shapes. Tell why your number is correct.

Visual Learning Bridge

Envision Math Common Core Grade K Answer Key Topic 1 Numbers 0 to 5 q136

Guided Practice

Question 1.
Envision Math Common Core Grade K Answer Key Topic 1 Numbers 0 to 5 q137
Answer:

Explanation:
I counted the number of birds and wrote 4 in the blank.There are 4 birds and i drew 4 circles to show the number 4.

Question 2.
Envision Math Common Core Grade K Answer Key Topic 1 Numbers 0 to 5 q138
Answer:

Explanation:
I counted the number of birds and wrote 3 in the blank.There are 3 birds and i wrote the number 1, 2 and 3 circles to show the number 3.

Directions 1 and 2 Have students make a math argument about how many birds are in each row, and then write the number. Have them use objects, words, or a method of their choice to explain their arguments and tell why they are correct.

Independent Practice

Question 3.
Envision Math Common Core Grade K Answer Key Topic 1 Numbers 0 to 5 q139
Answer:

Explanation:
I counted the number of leaves and wrote 5 in the blank.There are 5 leaves and i drew 5 circles to show the number 5.

Question 4.
Envision Math Common Core Grade K Answer Key Topic 1 Numbers 0 to 5 q140
Answer:

Explanation:
I counted the number of leaves and wrote 2 in the blank.There are 2 leaves and i wrote the number 1 and 2 circles to show the number 2.

Question 5.
Envision Math Common Core Grade K Answer Key Topic 1 Numbers 0 to 5 q141
Answer:

Explanation:
I counted the number of leaves and wrote 4 in the blank.There are 4 leaves and i colored 4 boxes to show the number 4.

Question 6.
Envision Math Common Core Grade K Answer Key Topic 1 Numbers 0 to 5 q142
Answer:

Explanation:
I counted the number of leaves and wrote 3 in the blank.There are 4 leaves and i colored 3 boxes to show the number 3.

Directions 3-5 Have students make a math argument about how many leaves are in each row, and then write the number. Have them use objects, words, or a method of their choice to explain their arguments and tell why they are correct. 6 enVision’ STEM Say: Chlorophyll makes leaves green. There is less sunlight in the winter, so trees save their chlorophyll This turns leaves brown, orange, red, and yellow. Have students make a math argument about how many orange leaves are in the row, and then write the number. Have them use objects, words, or a method of their choice to explain their arguments and tell why they are correct.

Problem Solving

Performance Task

Question 7,8,9.
Envision Math Common Core Grade K Answer Key Topic 1 Numbers 0 to 5 q143
Answer:

Explanation:
There are 3 rabbits in the picture.I used the tool of coloring a box as i count each rabbit.So, i colored 3 boxes to show the number of rabbits.

Directions Read the problem to students. Then have them use multiple problem-solving methods to solve the problem. Say: Brooke sees some rabbits. How many rabbits does she see? 7 Reasoning How can you find the number of rabbits Brooke sees? 8 Use Tools What tool can you use to help solve the problem? 9 Explain How can you use math to explain why your work is correct?

Topic 1 Vocabulary Review

Question 1.
Envision Math Common Core Grade K Answer Key Topic 1 Numbers 0 to 5 q144
Answer:

Explanation:
I circled the number 2.

Question 2.
Envision Math Common Core Grade K Answer Key Topic 1 Numbers 0 to 5 q145
Answer:

Explanation:
I wrote the number 0 as it means none.

Question 3.
Envision Math Common Core Grade K Answer Key Topic 1 Numbers 0 to 5 q146
Answer:

Explanation:
I circled the number 4.

Question 4.
Envision Math Common Core Grade K Answer Key Topic 1 Numbers 0 to 5 q147
Answer:

Explanation:
I marked a X on red cube and circled all the 5 cubes.

Directions Understand Vocabulary Have students: 1 draw a circle around the number; 2 write the number that means none; 3 draw a circle around the number four; 4 mark an X on the one red cube, and draw a circle around all five cubes in the group.

Question 5.
Envision Math Common Core Grade K Answer Key Topic 1 Numbers 0 to 5 q148
Answer:

Explanation:
I circled the number 1.

Question 6.
Envision Math Common Core Grade K Answer Key Topic 1 Numbers 0 to 5 q149
Answer:

Explanation:
I wrote the number 3.

Question 7.
Envision Math Common Core Grade K Answer Key Topic 1 Numbers 0 to 5 q150
Answer:

Explanation:
I counted the number of cubes and wrote the number 2 as there are 2 cubes.

Question 8.
Envision Math Common Core Grade K Answer Key Topic 1 Numbers 0 to 5 q151
Answer:

Explanation:
I wrote the numbers from 0 to 5 in order, and then drew counters to show the the numbers 0, 1, 2, 3, 4 and 5.

Directions Understand Vocabulary Have students: 5 draw a circle around the number one; 6 write the number three; 7 count the number of cubes, and then write the number to tell how many; 8 write the numbers 0 to 5 in order, and then draw counters to show that many of each number.

Topic 1 Reteaching

Set A

Envision Math Common Core Grade K Answer Key Topic 1 Numbers 0 to 5 q152

Question 1.
Envision Math Common Core Grade K Answer Key Topic 1 Numbers 0 to 5 q153
Answer:

Explanation:
I colored a box as i count each ball.There are 3 balls in the picture.

Question 2.
Envision Math Common Core Grade K Answer Key Topic 1 Numbers 0 to 5 q154
Answer:

Explanation:
I colored a box as i counted each ball.There ia 1 ball in the picture.

Set B

Envision Math Common Core Grade K Answer Key Topic 1 Numbers 0 to 5 q155

Question 3.
Envision Math Common Core Grade K Answer Key Topic 1 Numbers 0 to 5 q156
Answer:

Explanation:
I counted and practiced writing the number 2.There are 2 flowers in the vase.

Question 4.
Envision Math Common Core Grade K Answer Key Topic 1 Numbers 0 to 5 q157
Answer:

Explanation:
I counted and practiced writing the number 1.There are 1 flower in the vase.

Directions Have students: 1 and 2 color a box as they count each ball to show how many; 3 and 4 count the flowers in the vase, and then practice writing the number that tells how many.

Set C

Envision Math Common Core Grade K Answer Key Topic 1 Numbers 0 to 5 q158

Question 5.
Envision Math Common Core Grade K Answer Key Topic 1 Numbers 0 to 5 q159
Answer:

Explanation:
I counted the number of snakes and colored 5 boxes as there are 5 snakes.

Question 6.
Envision Math Common Core Grade K Answer Key Topic 1 Numbers 0 to 5 q160
Answer:

Explanation:
I counted the number of birds and colored 4 boxes as there are 4 birds

Directions 5 and 6 Have students color a box as they count each animal to show how many.

Set D

Envision Math Common Core Grade K Answer Key Topic 1 Numbers 0 to 5 q161

Question 7.
Envision Math Common Core Grade K Answer Key Topic 1 Numbers 0 to 5 q162
Answer:

Explanation:
The number that comes just before 3 is 2 and just after 3 is 4.So, i wrote the number 2 and 4.

Set E

Envision Math Common Core Grade K Answer Key Topic 1 Numbers 0 to 5 q163

Question 8.
Envision Math Common Core Grade K Answer Key Topic 1 Numbers 0 to 5 q164
Answer:

Explanation:
I counted the number of flowers in the vase and practiced writing the number 3.There are 3 flowers in the vase.

Question 9.
Envision Math Common Core Grade K Answer Key Topic 1 Numbers 0 to 5 q165
Answer:

Explanation:
I counted the number of flowers in the vase and practiced writing the number 0.There are no flowers in the vase.

Directions Have students: 7 write the numbers that are 1 less than 3 and 1 more than 3 when counting; 8 and 9 Count the flowers in the vase, and then practice writing the number that tells how many.

Set F

Envision Math Common Core Grade K Answer Key Topic 1 Numbers 0 to 5 q166

Question 10.
Envision Math Common Core Grade K Answer Key Topic 1 Numbers 0 to 5 q167
Answer:

Explanation:
I counted the number of octopuses and practiced writing number 3 as there are 3 octopuses.

Question 11.
Envision Math Common Core Grade K Answer Key Topic 1 Numbers 0 to 5 q168
Answer:

Explanation:
I drew 5 circles to show the number 5.

Directions 10 and 11 Have students count the octopuses or read the number, and then practice writing the number or draw circles to tell how many.

Topic 1 Assessment Practice

Question 1.
Envision Math Common Core Grade K Answer Key Topic 1 Numbers 0 to 5 q169
Answer:

Explanation:
I marked option c as it shows 3 flowers.

Question 2.
Envision Math Common Core Grade K Answer Key Topic 1 Numbers 0 to 5 q170
Answer:

Explanation:
I marked option D as it shows the number of counters 4.

Question 3.
Envision Math Common Core Grade K Answer Key Topic 1 Numbers 0 to 5 q171
Answer:

Explanation:
I marked the option D as it shows the number of pears in the picture.There are 5 pears.

Question 4.
Envision Math Common Core Grade K Answer Key Topic 1 Numbers 0 to 5 q172
Answer:

Explanation:
I marked the option A as it shows that the box has no toys in it.

Directions Have students mark the best answer. 1 Which shows 3 flowers? 2 How many counters are there? 3 How many pears are there? 4 Which box has 0 toys in it?

Question 5.
Envision Math Common Core Grade K Answer Key Topic 1 Numbers 0 to 5 q173
Answer:

Explanation:
I marked the option C as it shows 4 leaves.One more than the counters shown.

Question 6.
Envision Math Common Core Grade K Answer Key Topic 1 Numbers 0 to 5 q174
Answer:

Explanation:
I counted the butterflies and colored a box as i counted each buttetrfly.There are 2 butterflies.

Question 7.
Envision Math Common Core Grade K Answer Key Topic 1 Numbers 0 to 5 q175
Answer:

Explanation:
I drew counters to show the numbe rof dots.

Directions Have students: 5 listen to the story: Tina is counting leaves. She uses these counters to show how many leaves she has counted so far. Which of these shows the next number of leaves she will count? 6 count the butterflies, and then color the boxes to show how many; 7 count the number of dots and then draw counters in the box to show the same number of dots.

Question 8.
Envision Math Common Core Grade K Answer Key Topic 1 Numbers 0 to 5 q176
Answer:

Explanation:
I wrote the number 0, 1, 2, 3 and 4 as i counted the number of snowflakes given.

Question 9.
Envision Math Common Core Grade K Answer Key Topic 1 Numbers 0 to 5 q177
Answer:

Explanation:
I countec and wrote the number of red  and yellow counters.

Directions Have students: 8 count the snowflakes, and then write the number to tell how many; 9 listen to the story: Jack has some red and some yellow counters. He uses the counters to show one way to make 4. Write numbers to tell how many of each color he used.

Question 10.
Envision Math Common Core Grade K Answer Key Topic 1 Numbers 0 to 5 q178
Answer:

Explanation:
I counted the plates and wrote the numbe 3 as there are 3 plates.

Question 11.
Envision Math Common Core Grade K Answer Key Topic 1 Numbers 0 to 5 q179
Answer:

Explanation:
I didnot color any apple as there are no apples in the plate.

Question 12.
Envision Math Common Core Grade K Answer Key Topic 1 Numbers 0 to 5 q180
Answer:

Explanation:
I drew 5 marbles to show the number 5.

Directions Have students: 10 count the plates, and then write the number to tell how many; 11 count the number of apples on the plate, and then color the apples to show how many; 12 draw 5 marbles, and then write the number to tell how many.

Topic 1 Performance Task

Envision Math Common Core Grade K Answer Key Topic 1 Numbers 0 to 5 q181

Question 1.
Envision Math Common Core Grade K Answer Key Topic 1 Numbers 0 to 5 q182
Answer:

Explanation:
I counted and wrote the number of flowers in the vases.

Directions Flower Cart Say: Michael’s family sells flowers from a flower cart, 1 Have students count how many of each kind of flower, and then write the number to tell how many.

Question 2.
Envision Math Common Core Grade K Answer Key Topic 1 Numbers 0 to 5 q183
Answer:

Explanation:
As there are no flowers in the vase so, i wrote the number 0.

Question 3.
Envision Math Common Core Grade K Answer Key Topic 1 Numbers 0 to 5 q184
Answer:

Explanation:
I counted the flowers and colored a box for each flower i count.There are 4 flowers and i wrote the number 4 to show it.

Question 4.
Envision Math Common Core Grade K Answer Key Topic 1 Numbers 0 to 5 q185
Answer:

Explanation:
Michael can give his mother sunflowers.I drew 5 flowers andwrote the number 5 to show the number of flowers.

Directions 2 Say: How can Michael show the number of flowers in the green vase? Have students show two different ways. 3 Have students look at the 4 flowers, and then draw another way to show 4. 4 Say: Michael wants to give his mother 5 flowers from the cart. What kinds of flowers can he give her? Have students show one way to make an arrangement of 5 flowers, and write the number to tell how many.

enVision Math Common Core Grade 8 Answer Key Topic 6 Congruence And Similarity

Practice with the help of enVision Math Common Core Grade 8 Answer Key Topic 6 Congruence and Similarity regularly and improve your accuracy in solving questions.

enVision Math Common Core 8th Grade Answers Key Topic 6 Congruence And Similarity

Topic Essential Question
How can you show that two figures are either congruent or similar to one another?
Answer:
When two line segments have the same length, we can say that they are congruent. When two figures have the same shape and size, we can say that the two figures are congruent. These two triangles are congruent. We can also say that their side lengths are the same and that their angle measures are the same

3-ACT MATH

Envision Math Common Core 8th Grade Answer Key Topic 6 Congruence And Similarity 1

Tricks of the Trade
All kinds of objects in nature have symmetry: beehives, pine cones, butterflies, and snowflakes, to name a few. If you look closely, you will start to see patterns left and right. Think about this during the 3-Act Mathematical Modeling lesson.
Envision Math Common Core 8th Grade Answer Key Topic 6 Congruence And Similarity 2

Topic 6 ënVision STEM Project

Did You Know?
Trees provide wood for cooking and heating for half of the world’s population.
Envision Math Common Core 8th Grade Answer Key Topic 6 Congruence And Similarity 3
As trees grow, carbon dioxide is removed from the atmosphere for photosynthesis. Forests are called “carbon sinks” because one acre of forest absorbs six tons of carbon dioxide and puts out four tons of oxygen.
Envision Math Common Core 8th Grade Answer Key Topic 6 Congruence And Similarity 4
Envision Math Common Core 8th Grade Answer Key Topic 6 Congruence And Similarity 5

Trees provide lumber for buildings, tools, and furniture. Other products include rubber, sponges, cork, paper, chocolate, nuts, and fruit.
About 30% of the land is covered by forests.
Envision Math Common Core 8th Grade Answer Key Topic 6 Congruence And Similarity 6
Forests are now being managed to preserve wildlife and old-growth forests, protect biodiversity, safeguard watersheds, and develop recreation, as well as extract timber.

Forests also need to be managed to prevent raging wildfires, invasive species, overgrazing, and disease.
Envision Math Common Core 8th Grade Answer Key Topic 6 Congruence And Similarity 7

Your Task: Forest Health
The proper management of forests is a growing science. You and your classmates will learn about forest health indicators and use what you know about similar triangles and ratios to gather and interpret data in order to assess the health of a forest.
Envision Math Common Core 8th Grade Answer Key Topic 6 Congruence And Similarity 8

Topic 6 GET READY!

Review What You Know!

Vocabulary
Choose the best term from the box to complete each definition.
adjacent angles
complementary angles
supplementary angles
vertical angles

Question 1.
_________ have a sum of 90°.
Answer:
We know that,
The “Complementary angles” have a sum of 90°
Hence, from the above,
We can conclude that the best term to complete the given definition is: Complementary angles

Question 2.
_________ share the same ray.
Answer:
We know that,
The “Adjacent Angles” share the same ray
Hence, from the above,
We can conclude that the best term to complete the given definition is: Adjacent angles

Question 3.
_________ are pairs of opposite angles made by intersecting lines.
Answer:
We know that,
The “Vertical Angles” are pairs of opposite angles made by intersecting lines
Hence, from the above,
We can conclude that the best term to complete the given definition is: Vertical angles

Question 4.
__________ have a sum of 180°
Answer:
We know that,
The “Supplementary Angles” have a sum of 180°
Hence, from the above,
We can conclude that the best term to complete the given definition is: Supplementary angles

Multiplying Real Numbers

Simplify the expression.
Question 5.
5 × 2 = ________
Answer:
The given expression is:
5 × 2
Hence, from the above,
We can conclude that
5 × 2 = 10

Question 6.
6 × \(\frac{1}{2}\) = ________
Answer:
The given expression is:
6 × \(\frac{1}{2}\)
So,
6 × \(\frac{1}{2}\)
= \(\frac{6}{2}\)
= 3
Hence, from the above,
We can conclude that
6 × \(\frac{1}{2}\) = 3

Question 7.
12 × \(\frac{1}{3}\) = ________
Answer:
The given expression is:
12 × \(\frac{1}{3}\)
So,
12 × \(\frac{1}{3}\)
= \(\frac{12}{3}\)
= 4
Hence, from the above,
We can conclude that
12 × \(\frac{1}{3}\) = 4

Identifying Points on a Coordinate Plane

Name the location of the point.
Envision Math Common Core 8th Grade Answer Key Topic 6 Congruence And Similarity 9
Answer:
The given coordinate plane is:
Envision Math Common Core 8th Grade Answer Key Topic 6 Congruence And Similarity 9
From the given coordinate plane,
We can observe that

Question 8.
point W
Answer:
From the given coordinate plane,
We can observe that point W lies where both the x-axis and y-axis are negative
Hence, from the above,
We can conclude that point W lies in the third quadrant

Question 9.
point X
Answer:
From the given coordinate plane,
We can observe that point X lies where the x-axis is negative and the y-axis is positive
Hence, from the above,
We can conclude that point X lies in the second quadrant

Question 10.
point Y
Answer:
From the given coordinate plane,
We can observe that point Y lies where both the x-axis and y-axis are positive
Hence, from the above,
We can conclude that point Y lies in the first quadrant

Question 11.
point Z
Answer:
From the given coordinate plane,
We can observe that point Z lies where the x-axis is positive and the y-axis is negative
Hence, from the above,
We can conclude that point Z lies in the fourth quadrant

Supplementary Angles

The angles are supplementary. Find the missing angle measure.
Question 12.
Envision Math Common Core 8th Grade Answer Key Topic 6 Congruence And Similarity 10
Answer:
The given figure is:
Envision Math Common Core 8th Grade Answer Key Topic 6 Congruence And Similarity 10
We know that,
The sum of the supplementary angles is: 180°
Now,
Let the missing angle measure be: x°
So,
x° + 130° = 180°
x° = 180° – 130°
x° = 50°
Hence, from the above,
We can conclude that the missing angle measure is: 50°

Question 13.
Envision Math Common Core 8th Grade Answer Key Topic 6 Congruence And Similarity 11
Answer:
The given figure is:
Envision Math Common Core 8th Grade Answer Key Topic 6 Congruence And Similarity 11
We know that,
The sum of the supplementary angles is: 180°
Now,
Let the missing angle measure be: x°
So,
x° + 139° = 180°
x° = 180° – 139°
x° = 41°
Hence, from the above,
We can conclude that the missing angle measure is: 41°

Language Development
Complete the graphic organizer with an illustration for each transformation. Write either congruent or similar to make the given statement true.
Envision Math Common Core 8th Grade Answer Key Topic 6 Congruence And Similarity 12
Envision Math Common Core 8th Grade Answer Key Topic 6 Congruence And Similarity 13
Answer:

Topic 6 PICK A PROJECT

PROJECT 6A
How might an artist use mathematics?
PROJECT: WRITE A BIOGRAPHY
Envision Math Common Core 8th Grade Answer Key Topic 6 Congruence And Similarity 14

PROJECT 6B
What geometric shapes do you see around you?
PROJECT: RECORD A VIDEO ABOUT SIMILAR FIGURES
Envision Math Common Core 8th Grade Answer Key Topic 6 Congruence And Similarity 15

PROJECT 6C
What different types of bridges have you crossed?
PROJECT: BUILD A MODEL OF A TRUSS BRIDGE
Envision Math Common Core 8th Grade Answer Key Topic 6 Congruence And Similarity 16

PROJECT 6D
What shapes tessellate?
PROJECT: DESIGN A TESSELLATION
Envision Math Common Core 8th Grade Answer Key Topic 6 Congruence And Similarity 17

Lesson 6.1 Analyze Translations

Solve & Discuss It!
Ashanti draws a trapezoid on the coordinate plane and labels it in Figure 1. Then she draws Figure 2. How can she determine whether the figures have the same side lengths and the same angle measures?
Envision Math Common Core 8th Grade Answer Key Topic 6 Congruence And Similarity 18
I can… translate two-dimensional figures.
Answer:
It is given that
Ashanti draws a trapezoid on the coordinate plane and labels it in Figure 1. Then she draws Figure 2.
The given figure is:
Envision Math Common Core 8th Grade Answer Key Topic 6 Congruence And Similarity 18
From the figure,
We can observe that the first figure was translated or moved to the right by some units and became a second figure
We know that,
A “Translation” is a transformation that moves every point in a figure the same distance in the same direction
We know that,
The pre-image and image in the “Translation” are similar
We know that,
Two figures are said to be similar if they are the same shape. In more mathematical language, two figures are similar if their corresponding angles are congruent, and the ratios of the lengths of their corresponding sides are equal.
Hence, from the above,
We can conclude that by using the “Translation” property, she can determine whether the figures have the same side lengths and the same angle measures

Focus on math practices
Be Precise How do you know that the method you described shows whether the side lengths and angle measures are equal? Explain.
Answer:
We know that,
A “Translation” is a transformation that moves every point in a figure the same distance in the same direction
We know that,
The pre-image and image in the “Translation” are similar
We know that,
Two figures are said to be similar if they are the same shape. In more mathematical language, two figures are similar if their corresponding angles are congruent, and the ratios of the lengths of their corresponding sides are equal.

Essential Question
How does a translation affect the properties of a two-dimensional figure?
Answer:
When you translate something in geometry, you’re simply moving it around. You don’t distort it in any way. If you translate a segment, it remains a segment, and its length doesn’t change. Similarly, if you translate an angle, the measure of the angle doesn’t change.

Try It!

The clients also want the small table below the window moved 5 feet to the right. Where should the architect place the small table? Draw the new location of the table on the plan.
Answer:
It is given that
The clients also want the small table below the window moved 5 feet to the right
Now,
The original plan is:

From the above plan,
We can observe that the window is at most right
So,
When we move the table,
The new location of the table will be below the window
Hence,
The location of the table on the new plan is:

Convince Me!
An equilateral triangle with side lengths 5 inches is translated 3 units down and 2 units right. Describe the shape and dimensions of the translated figure.
Answer:
It is given that
An equilateral triangle with side lengths 5 inches is translated 3 units down and 2 units right.
We know that,
In an equilateral triangle, all the side lengths are equal
Now,
Let the dimensions of an equilateral triangle be (x1, y1), (x2, y2), and (x3, y3)
So,
The dimensions of the translated figure are: (x1 + 2, y1 – 3), (x2 + 2, y2 – 3), and (x3 + 2, y3 – 3)
Now,
Let us suppose the dimensions of an equilateral to be:
(1, 1), (1, 6), and (5, 3)
So,
The dimensions of the translated equilateral triangle are:
(3, -2), (3, 3), and (7, 0)
We know that,
When you translate something in geometry, you’re simply moving it around. You don’t distort it in any way
So,
The shape of the translated figure remains the same as the original figure
Hence,
The representation and shape of the original and translated equilateral triangles are:

Try It!

Triangle ABC is translated 5 units right and 1 unit down. Graph and label the image A’ B’C’. If m∠A = 30° in ΔABC, then what is m∠A in ΔA’B’C’?
Envision Math Common Core 8th Grade Answer Key Topic 6 Congruence And Similarity 19
Answer:
The given figure is:
Envision Math Common Core 8th Grade Answer Key Topic 6 Congruence And Similarity 19
From the given figure,
The dimensions of ΔABC are:
A (-2, 3), B (-3, -4), and C (-5, -1)
Now,
Let the dimensions of an equilateral triangle be (x1, y1), (x2, y2), and (x3, y3)
So,
The dimensions of the translated figure are: (x1 + 5, y1 – 1), (x2 + 5, y2 – 1), and (x3 + 5, y3 – 1)
So,
The dimensions of the translated triangle (A’B’C’) are:
A (3, 2), B (2, -5), and C (0, -2)
We know that,
A translated figure has the same shape as the original shape i.e, the lengths and the angles of the translated figure are also the same as the original figure
So,
m∠A in Triangle ABC = m∠A in Triangle A’B’C’ = 30°
Hence,
The representation of the original and translated triangles are:

KEY CONCEPT

A translation, or slide, is a transformation that moves Preimage to every point of a figure the same distance and the same direction.
Envision Math Common Core 8th Grade Answer Key Topic 6 Congruence And Similarity 20

Do You Understand?
Question 1.
Essential Question How does a translation affect the properties of a two-dimensional figure?
Answer:
When you translate something in geometry, you’re simply moving it around. You don’t distort it in any way. If you translate a segment, it remains a segment, and its length doesn’t change. Similarly, if you translate an angle, the measure of the angle doesn’t change.

Question 2.
Construct Arguments Triangle L’M’N’ is the image of triangle LMN after a translation. How are the side lengths and angle measures of the triangles related? Explain.
Answer:
We know that,
When you translate something in geometry, you’re simply moving it around. You don’t distort it in any way. If you translate a segment, it remains a segment, and its length doesn’t change. Similarly, if you translate an angle, the measure of the angle doesn’t change.
Hence, from the above,
We can conclude that
The side lengths and the angle measures of Triangle L’M’N’ is the same as The side lengths and the angle measures of Triangle LMN

Question 3.
Generalize Sanjay determined that one vertex of a figure was mapped to its image by translating the point 2 units left and 7 units down. What is the rule that maps the other vertices of the figure to their images?
Answer:
It is given that
Sanjay determined that one vertex of a figure was mapped to its image by translating the point 2 units left and 7 units down
Now,
Now,
Let the dimensions of any vertex of the given be (x, y)
So,
The dimensions of any vertex of the translated figure are: (x – 2, y + 7)
Hence, from the above,
We can conclude that
The rule that maps the other vertices of the figure to their images is: (x – 2, y – 7)

Do You Know How?
In 4-6, use the coordinate plane.
Envision Math Common Core 8th Grade Answer Key Topic 6 Congruence And Similarity 21

Question 4.
Which figure is a translation of Figure A? Explain
Answer:
We know that,
When you translate something in geometry, you’re simply moving it around. You don’t distort it in any way. If you translate a segment, it remains a segment, and its length doesn’t change. Similarly, if you translate an angle, the measure of the angle doesn’t change.
So,
When we observe the coordinate plane,
We can see that A and C have the same shape i.e., have the same length and the same angle
Hence, from the above,
We can conclude that
Figure C is a translation of Figure A

Question 5.
Graph the translation of Figure A 3 units right and 4 units up.
Answer:
From the given coordinate plane,
The dimensions of Figure A are:
(-2, -1), (-4, -1), (-4, -2), (-3, -2), (-3, -3), (-4, -3), (-4, -4), and (-2, -4)
So,
The rule that maps the vertices of the given figure to their images is: (x + 3, y + 4)
So,
The vertices of the image are:
(1, 3), (-1, 3), (-1, 2), (0, 2), (0, 1), (-1, 1), (-1, 0), and (1, 0)
Hence,
The representation of Figure A and its translated figure is:

Question 6.
Describe the translation needed to move Figure B to the same position as the image from Item 5.
Answer:
From the given coordinate plane,
The dimensions of figure B are:
(-2, 4), (-4, 4), (-4, 3), (-3, 3), (-3, 2), (-4, 2), (-4, 1), and (-2, 1)
Hence,
The translation needed to move figure B to the same position as the image from Item 5 is:
The X-axis: Translate 3 units right
The Y-axis: Translate 1 unit down

Practice & Problem Solving

Question 7.
Graph G’R’A’M’, the image of GRAM after a translation 11 units right and 2 units up.
Envision Math Common Core 8th Grade Answer Key Topic 6 Congruence And Similarity 22
Answer:
The given coordinate plane is:
Envision Math Common Core 8th Grade Answer Key Topic 6 Congruence And Similarity 22
From the given coordinate plane,
The dimensions of GRAM are:
G (-8, 2), R (-8, 6), A (-4, 6), and M (-5, 2)
So,
After a translation of 11 units right and 2 units up,
The dimensions of G’R’A’M’ are:
G’ (-8 + 11, 2 + 2), R’ (-8 + 11, 6 + 2), A’ (-4 + 11, 6 + 2), and M’ (-5 + 11, 2 + 2)
So,
G’ (3, 4), R’ (3, 8), A’ (7, 8), and M’ (6, 4)
Hence,
The representation of GRAM and its image G’R’A’M’ is:

Question 8.
∆A’ B’ C’ is a translation of ∆ABC. Describe the translation.
Envision Math Common Core 8th Grade Answer Key Topic 6 Congruence And Similarity 23
Answer:
The given coordinate plane is:
Envision Math Common Core 8th Grade Answer Key Topic 6 Congruence And Similarity 23
From the coordinate plane,
The dimensions of ΔABC are:
A (1, -5), B (-2, -2), and C (0, 0)
Now,
The dimensions of ΔA’B’C’ are:
A’ (0, -1), B’ (-3, 2), and C’ (-1, 4)
So,
By observing the dimensions of ΔABC and ΔA’B’C’,
The translation between ΔABC and ΔA’B’C is:
The x-axis: Translate 1 unit left
The y-axis: Translate 4 units up

Question 9.
Which triangle is the image of ∆DEF after a translation? Describe the translation.
Envision Math Common Core 8th Grade Answer Key Topic 6 Congruence And Similarity 24
Answer:
The given figure is:
Envision Math Common Core 8th Grade Answer Key Topic 6 Congruence And Similarity 24
We know that,
When you translate something in geometry, you’re simply moving it around. You don’t distort it in any way. If you translate a segment, it remains a segment, and its length doesn’t change. Similarly, if you translate an angle, the measure of the angle doesn’t change.
So,
From the given figure,
We can observe that
ΔMNO has the same shape as ΔDEF
So,
The image of ΔDEF is: ΔMNO
Now,
The dimensions of ΔDEF are:
D (-8, -10), F (-6, -10), and E (-8, -4)
Now,
The dimensions of ΔMNO are:
M (2, -10), O (4, -10), and N (2, -4)
So,
By observing the dimensions of ΔDEF and ΔMNo,
The translation between ΔDEF and ΔMNO is:
The x-axis: Translate 10 units right
The y-axis: No Translation required

Question 10.
The vertices of figure QRST are translated 3 units left and 11 units down to form figure Q’R’S’T’. Explain the similarities and differences between the two figures.
Answer:
It is given that
The vertices of figure QRST are translated 3 units left and 11 units down to form figure Q’R’S’T’.
Hence,
The similarities between figure QRST and figure Q’R’S’T’ are:
a. The two figures have the same side length
b. The two figures have the same angle measure
c. The two figures have the same shape
The differences between figure QRST and figure Q’R’S’T’ are:
a. Different dimensions of the vertices of QRST and Q’R’S’T’
b. Different positions of figure QRST and figure Q’R’S’T’

Question 11.
Graph the image of the given triangle after a translation of 3 units right and 2 units up.
Envision Math Common Core 8th Grade Answer Key Topic 6 Congruence And Similarity 25
Answer:
The given coordinate plane is:
Envision Math Common Core 8th Grade Answer Key Topic 6 Congruence And Similarity 25
From the given coordinate plane,
The dimensions of the given triangle are:
(-3, 4), (-5, -1), and (-8, 3)
Now,
After a translation 3 units right and 2 units up,
The dimensions of the given triangle are:
(-3 + 3, 4 + 2), (-5 + 3, -1 + 2), and (-8 + 3, 3 + 2)
(0, 6), (-2, 1), and (-5, 5)
Hence,
The representation of the given triangle and its image is:

Question 12.
Quadrilateral P’Q’R’ S’ is the image of quadrilateral PQRS after a translation.
Envision Math Common Core 8th Grade Answer Key Topic 6 Congruence And Similarity 26
Answer:
It is given that
Quadrilateral P’Q’R’ S’ is the image of quadrilateral PQRS after a translation.
We know that,
When you translate something in geometry, you’re simply moving it around. You don’t distort it in any way. If you translate a segment, it remains a segment, and its length doesn’t change. Similarly, if you translate an angle, the measure of the angle doesn’t change.

a. If the length of side PQ is about 2.8 units, what is the length of side P’ Q’?
Answer:
It is given that
The length of the side PQ is about 2.8 units
Hence, from the above,
We can conclude that the length of side P’Q’ is also about 2.8 units

b. If m∠R = 75°, what is m∠R’?
Answer:
It is given that
m∠R = 75°
Hence, from the above,
We can conclude that
m∠R’ = 75°

Question 13.
Higher-Order Thinking A farmer has a plot of land shaped like the figure in the graph. There is another identical plot of land 120 yards east and 100 yards north of the original plot.
Envision Math Common Core 8th Grade Answer Key Topic 6 Congruence And Similarity 27
a. Draw the image after the given translation.
Answer:
It is given that
A farmer has a plot of land shaped like the figure in the graph. There is another identical plot of land 120 yards east and 100 yards north of the original plot.
Now,
The given plot of land is:
Envision Math Common Core 8th Grade Answer Key Topic 6 Congruence And Similarity 27
Now,
From the given plot of land,
The dimensions of land are:
(0, 0), (0, 300), (300, 0), and (300, 300)
Now,
After the translation of 120 units right and 100 units up,
The dimensions of the identical plot are:
(0 + 120, 0 + 100), (0 + 120, 300 + 100), (300 + 120, 0 + 100), and (300 + 120, 300 + 100)
(120, 100), (120, 400), (420, 100), and (420, 400)
Hence,
The representation of the plot and its image after the translation is:

b. Find the combined area of the 2 plots in square yards.
Answer:
From part (a),
We can observe that the shape of the plot of land and its identical is like a square
We know that,
The square has the equal side lengths
Now,
From part (a),
We can observe that the side length of the plot of land and its identical is: 300 yards
Now,
We know that,
The area of square = Side²
So,
The area of the plot of land = 300²
= 90,000 yard²
The area of the identical plot of land = 300²
= 90,000 yard²
So,
The combined area of the 2 plots = 90,000 + 90,000
= 1,80,000 yard²
Hence, from the above,
We can conclude that the combined area of the 2 plots is: 1,80,000 yard²

Assessment Practice
Question 14.
What is true about the preimage of a figure and its image created by a translation? Select all that apply.
☐ Each point in the image moves the same distance and direction from the preimage.
☐ Each point in the image has the same x-coordinate as the corresponding point in the preimage.
☐ Each point in the image has the same y-coordinate as the corresponding point in the preimage.
☐ The preimage and the image are the same size.
☐ The preimage and the image are the same shape.
Answer:
We know that,
When you translate something in geometry, you’re simply moving it around. You don’t distort it in any way. If you translate a segment, it remains a segment, and its length doesn’t change. Similarly, if you translate an angle, the measure of the angle doesn’t change.
Hence,
The statements that are true about the preimage of a figure and its image created by a translation are:

Question 15.
The vertices of parallelogram QUAD are Q(-7, -7), U(-6, -4), A(-2,-4), and D(-3, -7).
Envision Math Common Core 8th Grade Answer Key Topic 6 Congruence And Similarity 28
Answer:
The given coordinate plane is:
Envision Math Common Core 8th Grade Answer Key Topic 6 Congruence And Similarity 28
Now,
From the given coordinate plane,
The dimensions of the parallelogram QUAD are:
Q (-7, -7), U (-6, -4), A (-2, -4), and D (-3, -7)
Now,
After the translation of 11 units right and 9 units up,
The dimensions of the parallelogram QUAD are:
Q’ (-7 + 11, -7 + 9), U’ (-6 + 11, -4 + 9), A’ (-2 + 11, -4 + 9), and D’ (-3 + 11, -7 + 9)
Q’ (4, 2), U’ (5, 5), A’ (9, 5), and D’ (8, 2)

PART A
Graph and label the image of QUAD after a translation 11 units right and 9 units up.
Answer:
The representation of the parallelogram QUAd and its image is:

PART B
If m∠U = 110°, what is m∠ U’?
Answer:
We know that,
When you translate something in geometry, you’re simply moving it around. You don’t distort it in any way. If you translate a segment, it remains a segment, and its length doesn’t change. Similarly, if you translate an angle, the measure of the angle doesn’t change.
Hence, from the above,
We can conclude that
m∠U’ = 110°

PART C
If the length of side UA is 4 units, what is the length of side U’ A’?
Answer:
We know that,
When you translate something in geometry, you’re simply moving it around. You don’t distort it in any way. If you translate a segment, it remains a segment, and its length doesn’t change. Similarly, if you translate an angle, the measure of the angle doesn’t change.
Hence, from the above,
We can conclude that
The length of the side U’ A’ is: 4 units

Lesson 6.2 Analyze Reflections

Solve & Discuss It!
Dale draws a triangle on grid paper and labels it in Figure 1. Then using his pencil as a guide, he draws another triangle directly on the opposite side of the pencil so that the vertical side is now one square to the right of the pencil instead of one square to the left of the pencil. He labels this triangle in Figure 2. How are the figures the same? How are they different?
Envision Math Common Core 8th Grade Answer Key Topic 6 Congruence And Similarity 29
I can… reflect two-dimensional figures.
Answer:
Dale draws a triangle on grid paper and labels it in Figure 1. Then using his pencil as a guide, he draws another triangle directly on the opposite side of the pencil so that the vertical side is now one square to the right of the pencil instead of one square to the left of the pencil. He labels this triangle in Figure 2.
Hence,
The representation of Figure 1 and Figure 2 is:

So,
From the above representation,
The similarities between Figure 1 and Figure 2 are:
a. Both the figures have the same side lengths
b. Both the figures have the same angle measures
c. Both the figures have the same size
d. Both the figures have the same distance
The differences between Figure 1 and Figure 2 are:
a. The positions of both figures are different
b. The orientations of both figures are different
c. The directions of both the figures are different

Look for Relationships
What do you notice about the size, shape, and direction of the two figures?
Answer:
The similarities between Figure 1 and Figure 2 are:
a. Both the figures have the same side lengths
b. Both the figures have the same angle measures
c. Both the figures have the same size
d. Both the figures have the same distance
The differences between Figure 1 and Figure 2 are:
a. The positions of both figures are different
b. The orientations of both figures are different
c. The directions of both the figures are different

Focus on math practices
Reasoning Dale draws a line in place of his pencil and folds the grid paper along the line. How do the triangles align when the grid paper is folded? Explain.
Answer:
It is given that
Dale draws a line in place of his pencil and folds the grid paper along the line
Now,
The representation of Figure 1 and Figure 2 are:

Now,
From the given figure,
We can observe that the two triangles will stack on each other when the grid paper is folded i.e.,
a. The vertical side of figure 2 is on top of the vertical side of figure 1
b. The base of figure 2 is on top of the base of figure 1
c. The hypotenuse of figure 2 is on top of the hypotenuse of figure 1

Essential Question
How does a reflection affect the properties of a two-dimensional figure?
Answer:
When the reflection takes place along the x-axis,
The values of x will remain constant and the values of y will have a sign change
Ex:
When (x, y) and (-x, y) reflects along the x-axis,
The reflection of (x, y) will become (x, -y)
The reflection of (-x, y) will become (-x, -y)
When the reflection takes place along the y-axis,
The values of y will remain constant and the values of x will have a sign change
Ex:
When (x, y) and (x, -y) reflects along the y-axis,
The reflection of (x, y) will become (-x, y)
The reflection of (x, -y) will become (-x, -y)

Try It!

While updating the design, the architect accidentally clicked on the chair and reflected it across the centerline. Draw the new location of the chair on the plan.

Answer:
It is given that
While updating the design, the architect accidentally clicked on the chair and reflected it across the centerline.
Hence,
The representation of the new location of the chair is:

Convince Me!
How do the preimage and image compare after a reflection?
Answer:
A reflection is a transformation that turns a figure into its mirror image by flipping it over a line. The line of reflection is the line that a figure is reflected over. If a point is on the line of reflection then the image is the same as the preimage. Otherwise,
the image is not the same as the preimage. Images are always congruent to preimages

Try It!

Quadrilateral KLMN has vertices at K(2, 6), L(3, 8), M(5, 4), and N(3, 2). It is reflected across the y-axis, resulting in quadrilateral K’L’M’N’. What are the coordinates of point N’?
Answer:
It is given that
Quadrilateral KLMN has vertices at K(2, 6), L(3, 8), M(5, 4), and N(3, 2). It is reflected across the y-axis, resulting in quadrilateral K’L’M’N’
Now,
We know that,
When (x, y) and (x, -y) reflects along the y-axis,
The reflection of (x, y) will become (-x, y)
The reflection of (x, -y) will become (-x, -y)
So,
The reflection of N (3, 2) i.e., the coordinates of N’ is: (-3, 2)
Hence, from the above,
We can conclude that the coordinates of N’ are: (-3, 2)

Try It!

Polygon ABCDE is reflected across the line x = -2. Graph and label the image A’B’C’D’E’. Is m∠A= M∠A? Explain.
Envision Math Common Core 8th Grade Answer Key Topic 6 Congruence And Similarity 30
Answer:
The given coordinate plane is:
Envision Math Common Core 8th Grade Answer Key Topic 6 Congruence And Similarity 30
Now,
From the given coordinate plane,
The vertices of polygon ABCDE are:
A (-4, 4), B (-3, 3), C (-3, 2), D (-5, 1), and E (-5, 3)
It is given that polygon ABCDE is reflected along x = -2 i.e., alone the x-axis
We know that,
When (x, y) and (-x, y) reflects along the y-axis,
The reflection of (x, y) will become (x, -y)
The reflection of (-x, y) will become (-x, -y)
So,
The vertices of the reflection of polygon ABCDE i..e, A’B’C’D’E’ are:
A’ (-4, -4), B’ (-3, -3), C’ (-3, -2), D’ (-5, -1), and E’ (-5, -3)
We know that,
In reflection,
The side lengths and the angle measures in the image and the preimage are the same
Hence,
The representation of polygon ABCDE and its reflection polygon A’B’C’D’E’ is:

Hence,
m ∠A = M ∠A

KEY CONCEPT

A reflection, or flip, is a transformation that flips a figure across a line of reflection. The preimage and image are the same distance from the line of reflection but on opposite sides. They have the same size and shape but different orientations.
Envision Math Common Core 8th Grade Answer Key Topic 6 Congruence And Similarity 31

Do You Understand?
Question 1.
Essential Question How does a reflection affect the properties of a two-dimensional figure?
Answer:
When the reflection takes place along the x-axis,
The values of x will remain constant and the values of y will have a sign change
Ex:
When (x, y) and (-x, y) reflects along the x-axis,
The reflection of (x, y) will become (x, -y)
The reflection of (-x, y) will become (-x, -y)
When the reflection takes place along the y-axis,
The values of y will remain constant and the values of x will have a sign change
Ex:
When (x, y) and (x, -y) reflects along the y-axis,
The reflection of (x, y) will become (-x, y)
The reflection of (x, -y) will become (-x, -y)

Question 2.
Generalize What do you notice about the corresponding coordinates of the preimage and image after a reflection across the x-axis?
Answer:
When the reflection takes place along the x-axis,
The values of x will remain constant and the values of y will have a sign change
Ex:
When (x, y) and (-x, y) reflects along the x-axis,
The reflection of (x, y) will become (x, -y)
The reflection of (-x, y) will become (-x, -y)

Question 3.
Construct Arguments Jorge said the y-values would stay the same when you reflect a preimage across the line y = 5 since the y-values stay the same when you reflect a preimage across the y-axis. Is Jorge correct? Explain.
Answer:
It is given that
Jorge said the y-values would stay the same when you reflect a preimage across the line y = 5 since the y-values stay the same when you reflect a preimage across the y-axis.
Now,
We know that,
When the reflection takes place along the y-axis,
The values of y will remain constant and the values of x will have a sign change
Ex:
When (x, y) and (x, -y) reflects along the y-axis,
The reflection of (x, y) will become (-x, y)
The reflection of (x, -y) will become (-x, -y)
Hence, from the above,
We can conclude that Jorge is correct

Do You Know How?
Question 4.
Is AX’ Y’ Z’ a reflection of AXYZ across line g?
Envision Math Common Core 8th Grade Answer Key Topic 6 Congruence And Similarity 32
Answer:
The given figure is:
Envision Math Common Core 8th Grade Answer Key Topic 6 Congruence And Similarity 32
Now,
From the given figure,
We can observe that ΔXYZ is reflected across the line g i.e., y-axis
We know that,
When the reflection takes place along the y-axis,
The values of y will remain constant and the values of x will have a sign change
Ex:
When (x, y) and (x, -y) reflects along the y-axis,
The reflection of (x, y) will become (-x, y)
The reflection of (x, -y) will become (-x, -y)
So,
From the figure,
We can observe that
The negative x-coordinates of the vertices of ΔXYZ became the positive x -coordinates for Δ X’Y’Z’
Hence, from the above,
We can conclude that ΔX’Y’Z’ is the reflection of ΔXYZ across the line g

Use the coordinate grid below for 5 and 6.
Envision Math Common Core 8th Grade Answer Key Topic 6 Congruence And Similarity 33

Question 5.
Describe the reflection of figure EFGH.
Answer:
The given coordinate plane is:
Envision Math Common Core 8th Grade Answer Key Topic 6 Congruence And Similarity 33
From the given coordinate plane,
We can observe that
The reflection of figure EFGH is: Figure E’F’G’H’
Now,
We can observe that
The reflection of the figure EFGH takes place across the x-axis
So,
The x-coordinates of the vertices of the reflection of the figure EFGH will be constant and only y-coordinates will have a change in value
Hence,
The figure EFGH will flip i.e., top becomes down and vice-versa to form a reflection i.e., figure E’F’G’H’

Question 6.
Draw the image that would result from a reflection of figure E’F’G’H across the line x = -1.
Answer:
The given coordinate plane is:
Envision Math Common Core 8th Grade Answer Key Topic 6 Congruence And Similarity 33
Now,
From the reflection of EFGH i.e., figure E’F’G’H’,
We can observe that the vertices of the figure E’F’G’H’ are:
E’ (-8, 7), F’ (-5, 5), G’ (-4, 6), and H’ (-2, 6)
Now,
To form the image of the figure E’F’G’H’,
We need to reflect the figure E’F’G’H’ across the y-axis
When the reflection takes place along the x-axis,
The values of x will remain constant and the values of y will have a sign change
Ex:
When (x, y) and (-x, y) reflects along the x-axis,
The reflection of (x, y) will become (x, -y)
The reflection of (-x, y) will become (-x, -y)
So,
The vertices for the image of the figure E’F’G’H’ are:
E’ (-8, -7), F’ (-5, -5), G’ (-4, -6), and H’ (-2, -6)
Hence,
The representation of the figure E’F’G’H’ and its image is:

Practice & Problem Solving

Question 7.
Leveled Practice Trapezoid ABCD is shown. Draw the reflection of trapezoid ABCD across the y-axis.
Envision Math Common Core 8th Grade Answer Key Topic 6 Congruence And Similarity 34
Plot the points and draw trapezoid A’ B’C’D’.
Answer:
From the given trapezoid ABCD,
The vertices are:
A (2, 8), B (6, 8), C (8, 3), and D (1, 3)
It is given that
Draw the reflection of trapezoid ABCD across the y-axis i.e., y is constant
Now,
We know that,
When the reflection takes place along the y-axis,
The values of y will remain constant and the values of x will have a sign change
Ex:
When (x, y) and (x, -y) reflects along the y-axis,
The reflection of (x, y) will become (-x, y)
The reflection of (x, -y) will become (-x, -y)
So,
The vertices for the reflection of trapezoid ABCD are:
A’ (-2, 8), B (-6, 8), C’ (-8, 3), and D (-1, 3)
So,
The points of the preimage and image are:

Hence,
The representation of trapezoid A’B’C’D’ is:

Question 8.
Reasoning is triangle A’ B’C’a reflection of triangle ABC across the line? Explain.
Envision Math Common Core 8th Grade Answer Key Topic 6 Congruence And Similarity 35
Answer:
The given figure is:
Envision Math Common Core 8th Grade Answer Key Topic 6 Congruence And Similarity 35
Now,
From the given figure,
We can observe that the reflection of ΔABC takes place across the y-axis
Now,
When the reflection takes place along the y-axis,
The values of y will remain constant and the values of x will have a sign change
Ex:
When (x, y) and (x, -y) reflects along the y-axis,
The reflection of (x, y) will become (-x, y)
The reflection of (x, -y) will become (-x, -y)
So,
From the vertices of ΔABC,
The x-coordinates are negative and the y-coordinates are positive
So,
For the reflection of ΔABC along the y-axis,
The x-coordinates will have to become positive and the y-coordinates will be positive as in the vertices of ΔABC
Hence, from the above,
We can conclude that ΔA’B’C’ is the reflection of ΔABC across the given line

Question 9.
Your friend gives you the graph of quadrilateral ABCD and its image, quadrilateral A’B’C’D’. What reflection produces this image?
Envision Math Common Core 8th Grade Answer Key Topic 6 Congruence And Similarity 36
Answer:
It is given that
Your friend gives you the graph of quadrilateral ABCD and its image, quadrilateral A’B’C’D’
Now,
The quadrilateral ABCD and its image quadrilateral A’B’C’D’ is:
Envision Math Common Core 8th Grade Answer Key Topic 6 Congruence And Similarity 36
Now,
From the given figure,
We can observe that
The quadrilateral ABCD and its image will have the same y-coordinates but the x-coordinates are different
We know that,
When the reflection takes place across the y-axis,
The y-coordinates are the same for the image and the preimage
The x-coordiantes have sign changes with the same values for the image and the preimage
Hence, from the above,
We can conclude that the reflection across the y-axis produces the image quadrilateral A’B’C’D’

Question 10.
Construct Arguments Your friend incorrectly says that the reflection of ∆EFG to its image ∆E’ F’G’ is a reflection across the y-axis.
Envision Math Common Core 8th Grade Answer Key Topic 6 Congruence And Similarity 37
a. What is your friend’s mistake?
Answer:
The given coordinate plane is:
Envision Math Common Core 8th Grade Answer Key Topic 6 Congruence And Similarity 37
From the above figure,
We can observe that
For the vertices of the given figure,
The x-coordinates remain the same whereas the y-coordinates are different
We know that,
When the reflection takes place across the x-axis,
The x-coordinates of the preimage and the image are the same
The y-coordinates of the preimage and the image will have sign change with the same values
Hence, from the above,
We can conclude that the reflection of ΔEFG takes pace across the x-axis instead of across the y-axis

b. What is the correct description of the reflection?
Answer:
From the given figure,
We can observe that
For the vertices of the given figure,
The x-coordinates remain the same whereas the y-coordinates are different
We know that,
When the reflection takes place across the x-axis,
The x-coordinates of the preimage and the image are the same
The y-coordinates of the preimage and the image will have sign change with the same values
Hence, from the above,
We can conclude that the reflection of ΔEFG to its image ΔE’F’G’ takes pace across the x-axis

Question 11.
Make Sense and Persevere The vertices of ∆ABC are A(-5, 5), B(-2,5), and C(-2, 3). If ∆ ABC is reflected across the line y = -1, find the coordinates of the vertex C’
Answer:
It is given that
The vertices of ∆ABC are A(-5, 5), B(-2,5), and C(-2, 3) and ∆ ABC is reflected across the line y = -1
We know that,
When the reflection takes place across the y-axis,
The y-coordinates of the preimage and the image are the same
The x-coordinates of the preimage and the image will have sign change with the same values
So,
The reflection of the vertex C (-2, 3) is: (2, 3)
Hence, from the above,
We can conclude that the coordinates of the vertex C’ i.e., the reflection of the vertex C is: (2, 3)

Question 12.
Higher Order Thinking What reflection of the parallelogram ABCD results in image A’B’C’D?
Envision Math Common Core 8th Grade Answer Key Topic 6 Congruence And Similarity 38
Answer:
The given figure is:
Envision Math Common Core 8th Grade Answer Key Topic 6 Congruence And Similarity 38
From the given figure,
We can observe that the parallelogram ABCD and its reflection A’B’C’D’ are parallel to the x-axis
Now,
We can observe that,
For both the parallelogram ABCD and its reflection A’B’C’D’,
The x-coordinates are changing but the y-coordinates remain the same
Now,
We know that,
When the reflection takes place across the y-axis,
The y-coordinates of the preimage and the image are the same
The x-coordinates of the preimage and the image will have sign change with the same values
Hence, from the above,
We can conclude that the reflection across the y-axis of the parallelogram ABCD results in image A’B’C’D

Assessment Practice
Question 13.
∆JAR has vertices J(4, 5), A6, 4), and R(5,2). What graph shows ∆JAR and its image after a reflection across the line y = 1?
PART A
Envision Math Common Core 8th Grade Answer Key Topic 6 Congruence And Similarity 39
Answer:
It is given that
∆JAR has vertices J(4, 5), A6, 4), and R(5,2)
Now,
We know that,
The reflection across the y-axis is:
The y -coordinates are the same
The x-coordinates will have sign changes with the same value
So,
The reflection of the vertices of ΔJAR is:
J’ (-4, 5), A’ (-6, 4), and R (-5, 2)
Hence,f rom the above,
We can conclude that option D matches with the given vertices of the reflection i.e., ΔJ’A’R’

PART B
The measure of ∠A = 90°. What is m∠A’?
Answer:
We know that,
In Reflection,
The side lengths and the angle measures of the preimage and the image are the same
Hence, from the above,
We can conclude that
∠A = ∠A’ = 90°

Lesson 6.3 Analyze Rotations

Explain It!
Maria boards a car at the bottom of the Ferris wheel. She rides to the top, where the car stops. Maria tells her friend that she completed \(\frac{1}{4}\) of the turn before the car stopped.
Envision Math Common Core 8th Grade Answer Key Topic 6 Congruence And Similarity 40
I can… rotate a two-dimensional figure.

A. Do you agree with Maria? Explain.
Answer:
It is given that
Maria boards a car at the bottom of the Ferris wheel. She rides to the top, where the car stops. Maria tells her friend that she completed \(\frac{1}{4}\) of the turn before the car stopped.
Now,
We know that,
From the given figure,
The Ferris wheel looks like a circle
We know that
The total angle measure of a circle (Ferris wheel) is: 360°
But,
It is given that
Maria tells her friend that she completed \(\frac{1}{4}\) of the turn before the car stopped
But, from the figure,
We can observe that she completed the \(\frac{1}{2}\) of the ride
Hence, from the above,
We can conclude that we can’t agree with Maria

B. How could you use angle measures to describe the change in position of the car?
Answer:
We know that,
From the given figure,
The Ferris wheel looks like a circle
We know that
The total angle measure of a circle (Ferris wheel) is: 360°
So,
The starting position of the car in terms of angle measure is given as 0°
The \(\frac{1}{2}\) of the position of the car in terms of angle measure = \(\frac{360°}{2}\)
= 180°
Hence, from the above,
We can conclude that
In terms of angle measures,
The starting position of the car is: 0°
The \(\frac{1}{2}\) of the car is: 180°

Focus on math practices
Construct Arguments How can you describe Maria’s change in position when her car returns to the position at which she began the ride?
Answer:
We know that,
The starting position of the car is: 0°
The \(\frac{1}{2}\) of the car is: 180°
Now,
If Maria’s car returns to the position at which she began the ride, then
Maria has completed the Ferris wheel (or) Maria returned back from her previous \(\frac{1}{2}\) of the position of the car
Hence,
In terms of angle measures,
The change in Maria’s position when her car returns to the position at which she began the ride is: 360° (or) -180°
Here,
-180° represents that Maria returned back from her previous \(\frac{1}{2}\) of the position of the car

Essential Question
How does rotation affect the properties of a two-dimensional figure?
Answer:
When you rotate a two-dimensional figure, you are just moving it.
Ex:
If you rotate a rectangle, then it will remain a rectangle, just moved wherever you move it. This is similarly the same with an angle and aside length, the measure of the angle and the side length won’t change.

Try It!

The architect continues to rotate the umbrella in a counterclockwise direction until it is in its original position. What is the angle of this rotation?

Answer:
It is given that
The architect continues to rotate the umbrella in a counterclockwise direction until it is in its original position.
Now,
We know that,
If any figure rotates until it comes to its original position again, then that means the figure completed a full cycle (or) revolution and the angle of the full cycle is: 360°
Hence, from the above,
We can conclude that the architect continues to rotate the umbrella for 360° in a counterclockwise direction until it is in its original position

Convince Me!
How does an image compare to its preimage after a -45° rotation?
Answer:

Try It!

The coordinates of the vertices of quadrilateral HIJK are H(1,4), I(3, 2), J(-1,-4), and K(-3, -2). If quadrilateral HIJK is rotated 270° about the origin, what are the vertices of the resulting image, quadrilateral H’I’I’K?
Answer:
It is given that
The coordinates of the vertices of quadrilateral HIJK are H(1,4), I(3, 2), J(-1,-4), and K(-3, -2)
Now,.
We know that,
The change in the x and y-coordinates for the given angle of rotation is:

Now,
It is given that to rotate the quadrilateral HIJK 270° about the origin to form the quadrilateral H’I’J’K’
So,
The vertices of the quadrilateral H’I’J’K’ are:
H’ (4, -1), I’ (2, -3), J’ (-4, 1), K’ (-2, 3)
Hence,
The representation of the quadrilateral HIJK and its image H’I’J’K’ is:

Try It!

Describe the rotation that maps ∆FGH to ∆FG’H’.
Envision Math Common Core 8th Grade Answer Key Topic 6 Congruence And Similarity 41
Answer:
The given coordinate plane is:
Envision Math Common Core 8th Grade Answer Key Topic 6 Congruence And Similarity 41
Now,
From the given coordinate plane,
The vertices of ΔFGH are:
F (-5, -2), G (-1, -2), and H (-1, 1)
The vertices of ΔF’G’H’ are:
F’ (5, 2), G’ (1, 2), and H’ (1, -1)
Now,
To find the rotation that maps ΔFGH through ΔF’G’H’, the following steps are:
Step 1:
Draw rays from the origin through point G and point G’
Step 2:
Measure the angle formed by the rays
So,
The representation of ΔFGH and its image ΔF’G’H’ with its angle of rotation is:

Hence, from the above,
We can conclude that the angle of rotation that maps ΔFGH through ΔF’G’H’ is: 180°

KEY CONCEPT

A rotation is a transformation that turns a figure about a fixed point called the center of rotation. The angle of rotation is the number of degrees the figure is rotated. The x- and y-coordinates change in predictable ways when rotated.
Envision Math Common Core 8th Grade Answer Key Topic 6 Congruence And Similarity 42

Do You Understand?
Question 1.
Essential Question How does rotation affect the properties of a two-dimensional figure?
Answer:
When you rotate a two-dimensional figure, you are just moving it.
Ex:
If you rotate a rectangle, then it will remain a rectangle, just moved wherever you move it. This is similarly the same with an angle and aside length, the measure of the angle and the side length won’t change.

Question 2.
Reasoning If a preimage is rotated 360 degrees about the origin how can you describe its image?
Answer:
We know that,
To complete the full cycle i.e., to a preimage to return to its original position, the angle measure is: 360°
Hence, from the above,
We can conclude that
If a preimage is rotated 360° about the origin, then the image and the preimage are the same

Question 3.
Construct Arguments In Example 3, side AB is parallel to side DC. How are side A’ B’ and side D’ C’ related? Explain.

Answer:
In Example 3,
The given two-dimensional figure is a parallelogram
We know that,
In a parallelogram, the opposite sides have the same side lengths
So,
Now,
It is given that
In parallelogram ABCD,
AB is parallel to CD
So,
In parallelogram A’B’C’D’,
A’B’ is parallel to C’D’ since these two sides are just the images of the sides AB and CD
Hence, from the above,
We can conclude that A’B’ is parallel to C’D’

Do You Know How?
Question 4.
The coordinates of the vertices of rectangle ABCD are A(3,-2), B(3, 2), C(-3, 2), and D(-3,-2).
a. Rectangle ABCD is rotated 90° about the origin. What are the coordinates of the vertices of rectangle A’B’C’D’?
Answer:
It is given that
The coordinates of the vertices of rectangle ABCD are A(3,-2), B(3, 2), C(-3, 2), and D(-3,-2).
Now,
We know that,
The change in the x and y-coordinates for the given angle of rotation is:

Now,
It is given that to rotate the rectangle ABCD 90° about the origin to form the rectangle A’B’C’D’
So,
The vertices of the rectangle A’B’C’D’ are:
A’ (2, 3), B’ (-2, 3), C’ (-2, -3), and D’ (2, -3)
Hence,
The representation of the rectangle ABCD and its image rectangle A’B’C’D’ is:

b. What are the measures of the angles of A’B’C’D’?
Answer:
From part (a),
We can observe that all the angles of the rectangle A’B’C’D’ are the same
Hence, from the above,
We can conclude that the angle measures of the rectangle A’B’C’D’ are:
∠A = 90°, ∠B = 90°, ∠C = 90°,and ∠D = 90°

Question 5.
Describe the counterclockwise rotation that maps ∆QRS to ∆Q’R’S.
Envision Math Common Core 8th Grade Answer Key Topic 6 Congruence And Similarity 43
Answer:
The given coordinate plane is:
Envision Math Common Core 8th Grade Answer Key Topic 6 Congruence And Similarity 43
Now,
From the given coordinate plane,
The vertices of ΔQRS are:
Q (6, 2), R (4, 9), and S (2, 7)
The vertices of ΔQ’R’S’ are:
Q’ (2, -6), R’ (9, -4), and S’ (7, -2)
Now,
To find the rotation that maps ΔQRS through ΔQ’R’S’, the following steps are:
Step 1:
Draw rays from the origin through point Q and point Q’
Step 2:
Measure the angle formed by the rays
So,
The representation of ΔQRS and its image ΔQ’R’S’ with its angle of rotation is:

Hence, from the above,
We can conclude that the angle of rotation that maps ΔQRS through ΔQ’R’S’ is: 90°

Practice & Problem Solving

Question 6.
What is the angle of rotation about the origin that maps △ΡQR to △Ρ’ Ο’ R’?
Envision Math Common Core 8th Grade Answer Key Topic 6 Congruence And Similarity 44
Answer:
The given coordinate plane is:
Envision Math Common Core 8th Grade Answer Key Topic 6 Congruence And Similarity 44
Now,
From the given coordinate plane,
The vertices of ΔPQR are:
P (3, 3), Q (5, 3), and R (5, 7)
The vertices of ΔP’Q’R’ are:
P’ (-3, 3), Q’ (-3, 5), and S’ (-7, 5)
Now,
To find the rotation that maps ΔPQR through ΔP’Q’R’, the following steps are:
Step 1:
Draw rays from the origin through point P and point P’
Step 2:
Measure the angle formed by the rays
So,
The representation of ΔPQR and its image ΔP’Q’R” with its angle of rotation is:

Hence, from the above,
We can conclude that the angle of rotation that maps ΔPQR through ΔP’Q’R’ is: 90°

Question 7.
Is △X’ Y’Z’ a rotation of △XYZ? Explain.
Envision Math Common Core 8th Grade Answer Key Topic 6 Congruence And Similarity 45
Answer:
The given coordinate plane is:
Envision Math Common Core 8th Grade Answer Key Topic 6 Congruence And Similarity 45
From the given coordinate plane,
The vertices of ΔXYZ are:
X (-2, 3), Y (-5, 4), and Z (-2, 7)
The vertices of ΔX’Y’Z’ are:
X’ (3, 2), Y’ (4, 5), and Z’ (7, 2)
Now,
To find the angle of rotation,
Compare the x and y-coordinates of ΔXYZ and ΔX’Y’Z’
So,
(x, y) before rotation ——-> (y, -x) after rotation
We know that,

Hence, from the above,
We can conclude that ΔX’Y’Z’ is a rotation of ΔXYZ

Question 8.
△PQR is rotated 270° about the origin. Graph and label the coordinates of P’, Q’, and R’.
Envision Math Common Core 8th Grade Answer Key Topic 6 Congruence And Similarity 46
Answer:
The given coordinate plane is:
Envision Math Common Core 8th Grade Answer Key Topic 6 Congruence And Similarity 46
Now,
From the given coordinate plane,
The vertices of ΔPQR are:
P (2, 3), Q (4, 6), and R (2, 7)
It is given that
Rotate ΔPQR 270° at the origin
We know that,

So,
The vertices of ΔP’Q’R’ when the angle of rotation is 270° are:
P’ (3, -2), Q’ (6, -4), and R’ (7, -2)
Hence,
The representation of ΔPQR and its image ΔP’Q’R’ is:

Question 9.
Is △P’ Q’R’a 270° rotation of △PQR about the origin? Explain.
Envision Math Common Core 8th Grade Answer Key Topic 6 Congruence And Similarity 47
Answer:
The given coordinate plane is:
Envision Math Common Core 8th Grade Answer Key Topic 6 Congruence And Similarity 47
From the given coordinate plane,
The vertices of ΔPQR are:
P (3, 3), Q (7, 4), and R (3, 5)
The vertices of ΔP’Q’R’ are:
P’ (-3, 3), Q’ (-4, 7), and R’ (-5, 3)
Now,
To find the angle of rotation,
Compare the x and y-coordinates of ΔPQR and ΔP’Q’R’
So,
(x, y) before rotation ——-> (-y, x) after rotation
We know that,

Hence, from the above,
We can conclude that ΔP’Q’R’ is a rotation of ΔPQR

Question 10.
Reasoning Explain why any rotation can be described by an angle between 0° and 360°.
Answer:
If you rotate an object 360°, it’s like the object never moved because the object would still be in the same spot as if you didn’t move it.
Hence,
Any rotation can be described by an angle of 0° to 360°

Question 11.
Rotate rectangle KLMN 270° about the origin.
Envision Math Common Core 8th Grade Answer Key Topic 6 Congruence And Similarity 48
Answer:
The given coordinate plane is:
Envision Math Common Core 8th Grade Answer Key Topic 6 Congruence And Similarity 48
Now,
From the given coordinate plane,
The vertices of rectangle KLMN are:
K (-3, 2), L (-5, 2), M (-5, 4), and N (-3, 4)
It is given that
Rotate rectangle KLMN 270° at the origin
We know that,

So,
The vertices of rectangle K’L’M’N’ when the angle of rotation is 270° are:
K’ (2, 3), L’ (2, 5), M’ (4, 5), and N’ (4, 3)
Hence,
The representation of rectangle KLMN and its image rectangle K’L’M’N’ is:

 

Question 12.
Higher-Order Thinking An architect is designing a new windmill with four sails. In her sketch, the sails’ center of rotation is the origin, (0, 0), and the tip of one of the sails, point Q, has coordinates (2, -3). She wants to make another sketch that shows the windmill after the sails have rotated 270° about their center of rotation. What would be the coordinates of?
Answer:
It is given that
An architect is designing a new windmill with four sails. In her sketch, the sails’ center of rotation is the origin, (0, 0), and the tip of one of the sails, point Q, has coordinates (2, -3). She wants to make another sketch that shows the windmill after the sails have rotated 270° about their center of rotation
So,
We have to rotate point Q 270° about the origin
We know that,

So,
When we rotate any point 270° about the origin,
(x, y) before rotation ——–> (y, -x)
So,
The representation of point Q after representation is: (-3, -2)
Hence,
The representation of point Q and its image Q’ is:

Assessment Practice
Question 13.
A rotation about the origin maps △TRI to △T’ R’I’.
PART A
Which graph shows an angle you could measure to find the angle of rotation about the origin?
Envision Math Common Core 8th Grade Answer Key Topic 6 Congruence And Similarity 49
Answer:
It is given that
A rotation about the origin maps △TRI to △T’ R’I’
Now,
We know that,
We can find the angle of rotation only when the vertex maps with its image but not with any other images
Hence, from the above,
We can conclude that option A matches the given situation

PART B
What is the angle of rotation about the origin?
A. 90°
B. 180°
C. 270°
D. 360°
Answer:
From part (a),
When we observe option A,
The vertices of ΔTRI are:
T (-3, 3), R (-5, 3), and I (-4, 5)
The vertices of ΔT’R’I’ are:
T’ (3, -3), R’ (5, -3), and I’ (4, -5)
We know that,

So,
When we compare the vertices of ΔTRI and ΔT’R’I’,
We can observe that
(x, y) before rotation ———–> (-x, -y) after rotation
Hence, from the above,
We can conclude that the angle of rotation about the origin is: 180°

Lesson 6.4 Compose Transformations

Solve & Discuss It!
How can you map Figure A onto Figure B?
Envision Math Common Core 8th Grade Answer Key Topic 6 Congruence And Similarity 50
I can… describe and perform a sequence of transformations.
Answer:
The given figure is:
Envision Math Common Core 8th Grade Answer Key Topic 6 Congruence And Similarity 50
Now,
The steps to obtain Figure B from Figure A are:
Step 1:
Reflect Figure A across the x-axis
Step 2:
Reflect the figure that we obtained in Step 1 across the y-axis
Hence,
The representation of the mapping of Figure A and Figure B using the above steps is:

Focus on math practices
Look for Relationships Is there another transformation or sequence of transformations that will map Figure A to Figure B?
Answer:
Yes, there are another sequence of transformations that will map Figure A to figure B
Now,
The steps to obtain Figure B from Figure A are:
Step 1:
Reflect Figure A across the y-axis
Step 2:
Reflect the figure that we obtained in Step 1 across the x-axis
Hence,
The representation of the mapping of Figure A and Figure B using the above steps is:

Essential Question
How can you use a sequence of transformations to map a preimage to its image?
Answer:
Mathematical transformations involve changing an image in some prescribed manner. There are four main types of transformations They are:
A) Translation B) Rotation C) Reflection D) Dilation

Try It!

Ava decided to move the cabinet to the opposite wall. What sequence of transformations moves the cabinet to its new position?

Answer:
It is given that
Ava decided to move the cabinet to the opposite wall
So,
From the figure,
We can observe that
To move the cabinet wall to the opposite wall, the following sequences of transformations have to be followed:
Step 1:
Translate the cabinet 8 units down
Step 2:
Rotate the cabinet 360° counterclockwise
Hence, from the above,
We can conclude that
The new position of the cabinet is:

Convince Me!
Ava decides that she would like the chairs to be placed directly across from the couch. What is a sequence of transformations that she can use to move the chairs to their new positions?

Try It!

What is another sequence of transformations that maps △ABC onto △A” B” C”?

Answer:
The given coordinate plane is:

From the given coordinate plane,
The vertices of ΔABC are:
A (-5, 5), B (-3, 3), and C (-6, 1)
Now,
Another sequence of transformations that maps ΔABC onto ΔA”B” C” is:
Step 1:
Draw ΔABC and make its reflection across the y-axis and name it ΔA’B’C’
Step 2:
Translate ΔA’B’C’ 2 units to the right and 6 units down
Step 3:
Reflect the figure we obtained in step 2 across the y-axis
Hence, from the above,
We can conclude that
The representation of another sequence of transformations that maps ΔABC onto ΔA”B” C” is:

KEY CONCEPT
You can use a sequence of two or more transformations to map a preimage to its image.
You can map △ABC onto △Α” Β” C” by translation of 3 units right followed by a 90° clockwise rotation about the origin.
Envision Math Common Core 8th Grade Answers Topic 6 Congruence And Similarity 51

Do You Understand?
Question 1.
Essential Question How can you use a sequence of transformations to map a preimage to its image?
Answer:
Mathematical transformations involve changing an image in some prescribed manner. There are four main types of transformations They are:
A) Translation B) Rotation C) Reflection D) Dilation

Question 2.
Make Sense and Persevere A preimage is rotated 180° about the origin and then rotated 180° about the origin again. Compare the preimage and image.
Answer:
It is given that
A preimage is rotated 180° about the origin and then rotated 180° about the origin again
Now,
When the preimage is rotated 180° and again 180°
The image will be the same as the preimage
Hence, from the above,
We can conclude that
When a preimage is rotated 180° about the origin and then rotated 180° about the origin again, the preimage and the image will be the same

Question 3.
Reasoning A figure ABC, with vertices A(2, 1), B(7, 4), and C(2, 7), is rotated 90° clockwise about the origin, and then reflected across the x-axis. Describe another sequence that would result in the same image.
Answer:
It is given that
A figure ABC, with vertices A(2, 1), B(7, 4), and C(2, 7), is rotated 90° clockwise about the origin, and then reflected across the x-axis
So,
The steps for another sequence of transformations that would result in the same image as the given situation is:
Step 1:
Draw the given vertices of Triangle ABC
Step 2:
Rotate Triangle ABC 90° counterclockwise
Step 3:
Reflect the image that we obtained in step 2 across the x-axis so that we will get the same image as in the given situation
Hence,
The representation of another sequence of transformations is:

Do You Know How?
In 4-6, use the diagram below.
Envision Math Common Core 8th Grade Answers Topic 6 Congruence And Similarity 52
Answer:
The given coordinate plane is:
Envision Math Common Core 8th Grade Answers Topic 6 Congruence And Similarity 52
From the given coordinate plane,
The vertices of Figure WXYZ are:
W (2, 4), X (5, 4), Y (5, 2), and Z (2, 2)
The vertices of Figure W’X’Y’Z’ are:
W’ (-4, -4), X’ (-4, -1), Y’ (-2, -1), and Z’ (-2, -4)

Question 4.
Describe a sequence of transformations that maps rectangle WXYZ onto rectangle W’X’Y’Z’.
Answer:
The steps that we have to follow to obtain the given sequence of transformations that maps rectangle WXYZ onto rectangle W’X’Y’Z’ are:
Step 1:
Draw rectangle WXYZ
Step 2:
Reflect rectangle WXYZ across the x-axis
Step 3:
Rotate the image we obtained in step 2 90° counterclockwise
Step 4:
Translate the image we obtained in step-3 6 units left so that we can obtain rectangle W’X’Y’Z’
Hence,
The representation of the sequence of transformations for the given situation is:

Question 5.
Describe another way that you could map rectangle WXYZ onto W’X’Y’Z’.
Answer:
The steps for another way of transformations that maps rectangle WXYZ onto W’X’Y’Z’ are:
Step 1:
Draw Rectangle WXYZ
Step 2:
Rotate rectangle WXYZ 90° counterclockwise
Step 3:
Rotate the image we obtained in step-2 180° counterclockwise
Step 4:
Translate the image we obtained in step 3 6 units down so that we can obtain rectangle W’X’Y’Z’
Hence,
The representation of another sequence of transformations is:

Question 6.
Draw the image of rectangle WXYZ after a reflection across the line y = 1 and a translation 1 unit right. Label the image W” X” Y” Z”.
Answer:
The vertices of rectangle WXYZ are:
W (2, 4), X (5, 4), Y (5, 2), and Z (2, 2)
After the reflection across the line y = 1,
The vertices of rectangle WXYZ are:
W’ (2, -5), X’ (5, -5), Y’ (5, -3), and Z’ (2, -3)
After the translation of 1 unit right,
The vertices of rectangle WXYZ are:
W” (3, -5), X” (6, -5), Y” (6, -3), and Z” (3, -3)
Hence,
The representation of rectangle WXYZ and its image W”X” Y”Z” after the above sequence of transformations is:

Practice & Problem Solving

Question 7.
Leveled Practice Describe a sequence of transformations that maps △QRS onto △TUV.
Envision Math Common Core 8th Grade Answers Topic 6 Congruence And Similarity 53
A translation ________ units left and ________ units down, followed by a _________ across the ________.
Answer:
The given coordinate plane is:
Envision Math Common Core 8th Grade Answers Topic 6 Congruence And Similarity 53
From the given coordinate plane,
The vertices of ΔQRS are:
Q (3, 4), R (7, 4), and S (6, 9)
The vertices of ΔTUV are:
T (0, 0), U (-4, 0), and V (-3, 5)
Now,
In order to obtain the vertices of ΔTUV, the following transformations we have to follow are:
Step 1:
Translate the vertices of ΔQRS 3 units left and 4 units down
Step 2:
Reflect the image we obtained in step 1 across the x-axis in order to get the vertices of ΔTUV
Hence, from the above,
We can conclude that
A translation of 3 units left and 4 units down, followed by a reflection across the x-axis

Question 8.
Model with Math
A family moves a table, shown as rectangle EFGH, by translating it 3 units left and 3 units down followed by a 90° rotation about the origin. Graph E’ F’G’H’ to show the new location of the table.
Envision Math Common Core 8th Grade Answers Topic 6 Congruence And Similarity 54
Answer:
It is given that
A family moves a table, shown as rectangle EFGH, by translating it 3 units left and 3 units down followed by a 90° rotation about the origin
Now,
The given coordinate plane is:
Envision Math Common Core 8th Grade Answers Topic 6 Congruence And Similarity 54
From the given coordinate plane,
The vertices of rectangle EFGH are:
E (3, 3), F (8, 3), G (8, 7), and H (3, 7)
Now,
To obtain rectangle E’F’G’H’,
The following series of transformations are:
Step 1:
By translating 3 units left and 3 units down,
The vertices of rectangle EFGH will become:
E (3 – 3, 3 – 3), F (8 – 3, 3 – 3), G (8 – 3, 7 – 3), and H (3 – 3, 7 – 3)
E (0, 0), F (5, 0), G (5, 4), and H (0, 4)
Step 2:
Rotate the vertices we obtain in step 1 90° counterclockwise about the origin
We know that,
When we rotate a point 90° counterclockwise about the origin,
(x, y) before rotation ——> (-y, x) after rotation
So,
The vertices of rectangle E’F’G’H’ are:
E’ (0, 0), F’ (0, 5), G’ (-4, 5), and H’ (-4, 0)
Hence,
The representation of the new location of the table is:

Question 9.
Describe a sequence of transformations that maps quadrilateral ABCD to quadrilateral HIJK.
Envision Math Common Core 8th Grade Answers Topic 6 Congruence And Similarity 55
Answer:
The given coordinate plane is:
Envision Math Common Core 8th Grade Answers Topic 6 Congruence And Similarity 55
From the given coordinate plane,
The vertices of quadrilateral ABCD are:
A (3, 1), B (4, 1), C (4, 3), and D (3, 3)
The vertices of quadrilateral HIJK are:
H (-3, 0), I (-2, 0), J (-2, -2), and K (-3, -2)
Now,
The series of transformations that maps quadrilateral ABCD onto quadrilateral HIJK are:
Step 1:
Draw quadrilateral ABCD
Step 2:
Reflect quadrilateral ABCD across the x-axis
So,
The vertices of quadrilateral ABCD are:
A (3, -1), B (4, -1), C (4, -3), and D (3, -3)
Step 3:
Translate 6 units left and 1 unit up
So,
The vertices that we obtained in step 2 will become (The vertices of quadrilateral HIJK):
H (-3, 0), I (-2, 0), J (-2, -2), and K (-3, -2)
Hence,
The representation of the series of transformations that map quadrilateral ABCD onto quadrilateral HIJK are:

Question 10.
Map △QRS to △Q’R’ S’with a reflection across the y-axis followed by a translation 6 units down.
Envision Math Common Core 8th Grade Answers Topic 6 Congruence And Similarity 56
Answer:
The given coordinate plane is:
Envision Math Common Core 8th Grade Answers Topic 6 Congruence And Similarity 56
From the given coordinate plane,
The vertices of ΔQRS are:
Q (-3, 5), R (-2, 4), and S (-5, 3)
Now,
The series of transformations that maps ΔQRS to ΔQ’R’S’ as given above are:
Step 1:
Reflect the vertices of ΔQRS along the y-axis
So,
Q (3, 5), R (2, 4), and S (5, 3)
Step 2:
Translate the vertices that we obtain in step-1 6 units down so that we can obtain the vertices of ΔQ’R’S’
So,
Q’ (3, 5 – 6), R’ (2, 4 – 6), and S’ (5, 3 – 6)
Q’ (3, -1), R’ (2, -2), and S’ (5, -3)
Hence,
The representation of the series of transformations for the given situation is:

 

Question 11.
Higher-Order Thinking A student says that he was rearranging furniture at home and he used a glide reflection to move a table with legs from one side of the room to the other. Will a glide reflection result in a functioning table? Explain.
Answer:
It is given that
A student says that he was rearranging furniture at home and he used a glide reflection to move a table with legs from one side of the room to the other.
We know that,
A glide reflection is a  sequence of translation and reflection
Now,
From the given situation,
We can observe that the table is moving from one room to the other
So,
The “Translation” occurs
Now,
After moving he will rearrange the table in the room
So,
A “Reflection” may take place
Hence, from the above,
We can conclude that a glide reflection result in a functioning table

Assessment Practice
Question 12.
PART A
Which sequence of transformations maps rectangle ABCD onto rectangle A’ B’C’D?
Envision Math Common Core 8th Grade Answers Topic 6 Congruence And Similarity 57
A. translation 6 units down, reflection across the x-axis
B. reflection across the x-axis, translation 6 units right
C. reflection across the x-axis, translation 6 units left
D. translation 6 units left, reflection across the y-axis
Answer:
The given coordinate plane is:
Envision Math Common Core 8th Grade Answers Topic 6 Congruence And Similarity 57
From the given coordinate plane,
The vertices of rectangle ABCD are:
A (2, -2), B (4, -2), C (4, -3), and D (2, -3)
The vertices of rectangle A’B’C’D’ are:
A’ (-4, 2), B’ (-2, 2), C’ (-2, 3), and D’ (-4, 3)
So,
The sequence of transformations that maps rectangle ABCD to rectangle A’B’C’D’ is:

Hence, from the above,
We can conclude that option C matches the given situation

PART B
Describe a sequence of transformations that maps A’B’C’ D’ onto ABCD.
Answer:
The sequence of transformations that maps A’B’C’D’ onto ABCD is:
Step 1:
Draw the rectangle ABCD
So,
The vertices of rectangle ABCD are:
A (2, -2), B (4, -2), C (4, -3), and D (2, -3)
Step 2:
Reflect rectangle ABCD across the x-axis
So,
The vertices of rectangle ABCD are:
A (2, 2), B (4, 2), C (4, 3), and D (2, 3)
Step 3:
Translate the vertices that we obtained in step 2 6 units left so that we can obtain rectangle A’B’C’D’
So,
A’ (2 – 6, 2), B’ (4 – 6, 2), C’ (4 – 6, 3), and D’ (2 – 6, 3)
So,
A’ (-4, 2), B’ (-2, 2), C’ (-2, 3), and D’ (-4, 3)

Question 13.
PART A
Which figures are the image of Figure A after a reflection across the x-axis and a translation of 4 units right?
Envision Math Common Core 8th Grade Answers Topic 6 Congruence And Similarity 58
A. Figure B
B. Figure C
C. Figure D
D. Figure E
Answer:
We know that,
A “Reflection” is called a “Flip” but the reflection does not affect the shape and length of the figure
So,
From the given figures,
When we observe Figures A and B,
We can say that Figure B is a reflection of A because the shape and length is the same
But,
when we observe the other figures,
We have different shapes and lengths between the preimage and the image
Hence, from the above,
We can conclude that
Figure B is the image of Figure A after a reflection across the x-axis and a translation of 4 units right

PART B
Which figure can be transformed into Figure G after a rotation 90° about the origin, then a translation 13 units right and 4 units down?
A. Figure B
B. Figure D
C. Figure E
D. Figure F
Answer:
From the given figures,
We have to obtain
The coordinates of Figure G are:
(6, -6), (9, -6), (9, -9), (7, -9), and (6, -5)
We know that,
When a point rotates 90° about the origin,
(x, y) before rotation ——-> (-y, x) after rotation
Now,
For the translation of 13 units right and 4 units down,
The vertices will be like: (x + 13, y – 4)
Hence, from the above,
We can conclude that option A matches the given situation

3-ACT MATH

3-Act Mathematical Modeling: Tricks of the Trade
Envision Math Common Core 8th Grade Answers Topic 6 Congruence And Similarity 59

ACT 1
Question 1.
After watching the video, what is the first question that comes to mind?
Answer:

Question 2.
Write the Main Question you will answer.
Envision Math Common Core 8th Grade Answers Topic 6 Congruence And Similarity 60
Answer:

Question 3.
Make a prediction to answer this Main Question.
Answer:

Question 4.
Construct Arguments Explain how you arrived at your prediction.
Answer:

ACT 2
Question 5.
What information in this situation would be helpful to know? How would you use that information?
Envision Math Common Core 8th Grade Answers Topic 6 Congruence And Similarity 61
Answer:

Question 6.
Use Appropriate Tools What tools can you use to solve the problem? Explain how you would use them strategically.
Answer:

Question 7.
Model with Math Represent the situation using mathematics. Use your representation to answer the Main Question.
Answer:

Question 8.
What is your answer to the Main Question? Does it differ from your prediction? Explain.
Envision Math Common Core 8th Grade Answers Topic 6 Congruence And Similarity 62
Answer:

ACT 3
Question 9.
Write the answer you saw in the video.
Envision Math Common Core 8th Grade Answers Topic 6 Congruence And Similarity 63
Answer:

Question 10.
Reasoning Does your answer match the answer in the video? If not, what are some reasons that would explain the difference?
Answer:

Question 11.
Make Sense and Persevere Would you change your model now that you know the answer? Explain.
Answer:

ACT 3 Extension
Reflect
Question 12.
Model with Math Explain how you used a mathematical model to represent the situation. How did the model help you answer the Main Question?
Envision Math Common Core 8th Grade Answers Topic 6 Congruence And Similarity 64
Answer:

Question 13.
Make Sense and Persevere When did you struggle most while solving the problem? How did you overcome that obstacle?
Answer:

SEQUEL
Question 14.
Be Precise Find another optical illusion online involving shapes that look different but are the same. Explain how you know the shapes are the same.
Envision Math Common Core 8th Grade Answers Topic 6 Congruence And Similarity 65
Answer:

Lesson 6.5 Understand Congruent Figures

Solve & Discuss It!
Simone plays a video game in which she moves shapes into empty spaces. After several rounds, her next move must fit the blue piece into the dashed space. How can Simone move the blue piece to fit in the space?
Envision Math Common Core 8th Grade Answers Topic 6 Congruence And Similarity 66
I can… use a sequence of translations, reflections, and rotations to show that figures are congruent.
Answer:
It is given that
Simone plays a video game in which she moves shapes into empty spaces. After several rounds, her next move must fit the blue piece into the dashed space.
Now,
From the given figure,
We can observe that
The blue piece and the dashed  piece are the reflections of each other
Hence,
Simone move the blue piece to fit in the space by the translation followed by reflection

Reasoning
How can you use what you know about sequences of transformations to move the piece?
Answer:
From the given figure,
We can observe that
The blue piece should be moved to the place of the dashed space and it will be possible only due to the “Translation”
But,
We can observe that it does not fit into the dashed space.
So,
Reflect the blue piece that it can fit into the dashed piece
Hence, from the above,
We can conclude that the sequence of transformations we can use to move the transformation are:
a. Translation b. Reflection

Focus on math practices
Construct Arguments How do you know that the piece that fits into the space is the same as the original blue shape? Explain.
Answer:
We know that,
In the reflection,
a. The shape of the preimage and the image are the same
b. The length of the preimage and the image are the same
c. The orientation of the image and the preimage are different
Hence, from the above,
We can conclude that
Due to the properties of the reflection,
We know that the piece that fits into space is the same as the original shape

Essential Question
How does a sequence of translations, reflections, and rotations result in congruent figures?
Answer:
If we copy one figure on tracing paper and move the paper so the copy covers the other figure exactly, then that suggests they are congruent. We can prove that two figures are congruent by describing a sequence of translations, rotations, and reflections that move one figure onto the other so they match up exactly.

Try It!

How can you determine whether the orange and blue rectangles are congruent?
Envision Math Common Core 8th Grade Answers Topic 6 Congruence And Similarity 67
Answer:
The given coordinate plane is:

From the given coordinate plane,
We can observe that
The blue rectangle has a length of 6 units and a width of 5 units
The orange rectangle has a length of 5 units and a width of 6 units
Now,
We can say that
We can obtain the orange rectangle by rotating the blue rectangle
We know that,
Rotations, reflections, and translations are isometric. That means that these transformations do not change the size of the figure. If the size and shape of the figure is not changed, then the figures are congruent
Hence, from the above,
We can conclude that the orange and blue rectangles are congruent

Convince Me!
Quadrilateral PQRS is congruent to quadrilateral P’ Q’R’S. What do you know about how these figures relate?
Answer:
It is given that
Quadrilateral PQRS is congruent to quadrilateral P’ Q’R’S
We know that,
When the two figures are congruent,
a. The shapes of the two figures are the same
b. The lengths of the two figures are the same
c. The angle measures of the two figures are the same
Hence,
In Quadrilateral PQRS and Quadrilateral P’Q’R’S’, the two figures are said to be congruent when
a. PQ = P’Q’ and RS = R’S’
b. ∠P = ∠P’, ∠Q = ∠Q’, ∠R = ∠R’, and ∠S = ∠S’

Try It!

Are the figures congruent? Explain.
Envision Math Common Core 8th Grade Answers Topic 6 Congruence And Similarity 68
Answer:
The given coordinate plane is:
Envision Math Common Core 8th Grade Answers Topic 6 Congruence And Similarity 68
From the given coordinate plane,
The vertices of Figure 1 are:
(1, 5), (3, 7), (3, 5), and (2, 3)
The vertices of Figure 2 are:
(6, 3), (8, 3), (10, 2), and (8, 1)
Now,
Find out whether Translation, Reflection, and Rotation is possible between 2 figures or not
So,
The representation of Figure 1 and Figure 2 are:

Now,
From the given figures,
We can observe that none of the transformations is possible
Hence, from the above,
We can conclude that the two figures are not congruent

KEY CONCEPT

Two-dimensional figures are congruent if there is a sequence of translations, reflections, and rotations that maps one figure onto the other.
Envision Math Common Core 8th Grade Answers Topic 6 Congruence And Similarity 69

Do You Understand?
Question 1.
Essential Question How does a sequence of translations, reflections, and rotations result in congruent figures?
Answer:
If we copy one figure on tracing paper and move the paper so the copy covers the other figure exactly, then that suggests they are congruent. We can prove that two figures are congruent by describing a sequence of translations, rotations, and reflections that move one figure onto the other so they match up exactly.

Question 2.
Reasoning Does a sequence of transformations have to include a translation, a reflection, and a rotation to result in congruent figures? Explain.
Answer:
Rotations, reflections, and translations are isometric. That means that these transformations do not change the size of the figure. If the size and shape of the figure is not changed, then the figures are congruent

Question 3.
Construct Arguments Is there a sequence of reflections, rotations, and translations that makes the preimage and image not only congruent, but identical in orientation? Explain.
Answer:
We know that,
In the reflection,
The orientation of the preimage and the image will differ
Hence, from the above,
We can conclude that
There is a sequence of reflections, rotations, and translations that makes the preimage and image only congruent but not identical in orientation

Do You Know How?
Question 4.
A rectangle with an area of 25 square centimeters is rotated and reflected in the coordinate plane. What will be the area of the resulting image? Explain.
Answer:
It is given that
A rectangle with an area of 25 square centimeters is rotated and reflected in the coordinate plane.
We know that,
In a sequence of transformations like Translation, Rotation, and Reflection,
The shapes and side lengths of the image and the preimage are the same
Since the side lengths are the same, the area will also be the same
Hence, from the above,
We can conclude that the area of the image will also be 25 square centimeters

In 5 and 6, use the coordinate grid below.
Envision Math Common Core 8th Grade Answers Topic 6 Congruence And Similarity 70
Answer:
The given coordinate plane is:
Envision Math Common Core 8th Grade Answers Topic 6 Congruence And Similarity 70
From the given coordinate plane,
The vertices of ΔABC are:
A (1, 4), B (2, 2), and C (5, 2)
The vertices of ΔDEF are:
D (9, 9), E (8, 7), and F (5, 7)
The vertices of ΔGHI are:
G (6, 6), H (8, 5), and I (8, 1)

Question 5.
Is △ABC ≅ △DEF? Explain.
Answer:
To find whether ΔABC is congruent to ΔDEF or not,
Step 1:
Reflect Triangle ABC across the x-axis
Step 2:
Translate the image that we obtained in step-1 10 units right and 5 units up
So,
The representation of step 1 and step 2 is:

Hence, from the above,
We can observe that the vertices we obtained in step 2 are the same as ΔDEF
Hence,
ΔABC is congruent to ΔDEF

Question 6.
Is △ABC ≅ △GHI? Explain.
Answer:
The representation of △ABC and △GHI is:

From the above,
We can observe that △ABC and △GHI do not have the same size
We know that,
In order to be 2 figures congruent,
a. The sizes of the figures would be the same
b. The shapes of the figures would be the same
c. The side lengths of the figures should be the same
Hence, from the above,
We can conclude that
△ABC is not congruent to △GHI

Practice & Problem Solving

Question 7.
△Q’R’ S’ is the image of △QRS after a reflection across the y-axis and a translation 6 units down. Is the image the same size and shape as the preimage?
Envision Math Common Core Grade 8 Answer Key Topic 6 Congruence And Similarity 71
△QRS and △Q’R’S’ _________ the same size and shape.
Answer:
It is given that
△Q’R’ S’ is the image of △QRS after a reflection across the y-axis and a translation 6 units down
Now,
The given coordinate plane is:
Envision Math Common Core Grade 8 Answer Key Topic 6 Congruence And Similarity 71
From the given coordinate plane,
The vertices of ΔQRS are:
Q (-3, 5), R (-2, 4), and S (-5, 3)
The vertices of ΔQ’R’S’ are:
Q’ (3, -1), R’ (2, -2), and S’ (5, -3)
Now,
Step 1:
After a reflection of ΔQRS across the y-axis,
The vertices of ΔQRS will be:
Q (3, 5), R (2, 4), and S (5, 3)
Step 2:
After a translation of 6 units down,
The vertices that we obtained in Step 1 are:
Q’ (3, 5 – 6), R’ (2, 4 – 6), and S’ (5, 3 – 6)
Q’ (3, -1), R’ (2, -2), and S’ (5, -3)
So,
The vertices of ΔQ’R’S’ we obtained from the coordinate plane and the vertices of ΔQ’R’S’ we obtained after the sequence of transformations are the same
Hence, from the above,
We can conclude that △QRS and △Q’R’S’ have the same size and shape and the image (ΔQ’R’S’) is the same size and shape as the preimage (ΔQRS)

Question 8.
Is △DEF ≅ △D’ E’F’? Explain.
Envision Math Common Core Grade 8 Answer Key Topic 6 Congruence And Similarity 72
Answer:
The given coordinate plane is:
Envision Math Common Core Grade 8 Answer Key Topic 6 Congruence And Similarity 72
From the given coordinate plane,
The vertices of ΔDEF are:
D (5, 5), E (6, 3), and F (2, 4)
The vertices of ΔD’E’F’ are:
D’ (-2, -1), E’ (-3, 1), and F’ (1, 0)
So,
The representtaion of the sequence of transformations of ΔDEF to show it is congruent to ΔD’E’F’ is:

Hence, from the above,
We can conclude that ΔDEF is congruent to ΔD’E’F’

Question 9.
Construct Arguments Describe a way to show that quadrilateral ABCD is congruent to quadrilateral A’B’C’D’.
Envision Math Common Core Grade 8 Answer Key Topic 6 Congruence And Similarity 73
Answer:
The given coordinate plane is:
Envision Math Common Core Grade 8 Answer Key Topic 6 Congruence And Similarity 73
From the given coordinate plane,
The vertices of quadrilateral ABCD are:
A (3, 5), B (5, 5), C (5, 4), and D (3, 4)
The vertices of quadriateral A’B’C’D’ are:
A’ (-3, 0), B’ (-5, 0), C’ (-5, -1), and D’ (-3, -1)
So,
The representtaion of the transformation of sequences that shows quadrilateral ABCd is congruent to quadrilateral A’B’C’D’ is:

Hence, from the above,
We can conclude that quadrilateral ABCD is congruent to quadrilateral A’B’C’D’

Question 10.
You are making two triangular flags for a project and need the flags to be the same shape and size. △XYZ and △X’Y’Z’ are the flags you have drawn. Are the flags the same shape and size? Explain.
Envision Math Common Core Grade 8 Answer Key Topic 6 Congruence And Similarity 74
Answer:
It is given that
You are making two triangular flags for a project and need the flags to be the same shape and size. △XYZ and △X’Y’Z’ are the flags you have drawn.
Now,
The given coordinate plane is:
Envision Math Common Core Grade 8 Answer Key Topic 6 Congruence And Similarity 74
From the given coordiate plane,
The vertices of ΔXYZ are:
X (5, 6), Y (6, 1), and Z (2, 2)
he vertices of ΔX’Y’Z’ are:
X’ (-2, 0), Y’ (-3, -5), and Z’ (1, -4)
So,
The representtaion of the sequence of transformation to find whether two flags have the same size and the same shape or not is:

Question 11.
Which two triangles are congruent? Describe the sequence of transformations that maps one figure onto the other.
Envision Math Common Core Grade 8 Answer Key Topic 6 Congruence And Similarity 75
Answer:
The given coordinate plane is:
Envision Math Common Core Grade 8 Answer Key Topic 6 Congruence And Similarity 75
From the given coordinate plane,
The vertices of ΔABC are:
A (-3, 2), B (-2, 7), and C (-7, 4)
The vertices of ΔQRS are:
Q (4, 7), R (2, 2), and S (7, 2)
The vertices of ΔXYZ are:
X (-2, -7), Y (-2, -3), and Z (-7, -5)
The vertices of ΔDEF are:
D (7, -5), E (2, -2), and F (2, -7)
Now,
We know that,
The two figures are said to be congruent when
a. The 2 figures have the same size i.e., the same length and the same angle measure
b. The 2 figures have the same shape
So,
The figures that are congruent to each other are represented as:

So,
From the above,
We can say that ΔQRS and ΔDEf are congruent
Hence,
The sequence of Transformations to show ΔQRS and ΔDEF are congruent is:

Question 12.
Is △LMN ≅ △XYZ? Explain.
Envision Math Common Core Grade 8 Answer Key Topic 6 Congruence And Similarity 76
Answer:
The given coordinate plane is:
Envision Math Common Core Grade 8 Answer Key Topic 6 Congruence And Similarity 76
From the given coordinate plane,
The vertices of ΔLMN are:
L (7, 9), M (9, 5), and N (6, 5)
The vertices of ΔXYZ are:
X (2, 2), Y (5, 4), and Z (5, 1)
Now,
We know that,
The two figures are said to be congruent when
a. The 2 figures have the same size i.e., the same length and the same angle measure
b. The 2 figures have the same shape
So,
The sequence of transformations to find whether ΔLMN is congruent to ΔXYZ is:

Hence, from the above,
We can conclude that ΔLMN is not congruent to ΔXYZ since they don’t have the same side lengths

Question 13.
Higher-Order Thinking A student was asked to describe a sequence of transformations that maps △DEF onto △D’ E’F’, given that △DEF ≅ △D’ E’F’. She incorrectly said the sequence of transformations that maps △DEF onto △D’ E’F’ is a reflection across the x-axis, followed by a translation of 6 units right and 4 units up. What mistake did the student likely make?
Envision Math Common Core Grade 8 Answer Key Topic 6 Congruence And Similarity 77
Answer:
It is given that
A student was asked to describe a sequence of transformations that maps △DEF onto △D’ E’F’, given that △DEF ≅ △D’ E’F’. She incorrectly said the sequence of transformations that maps △DEF onto △D’ E’F’ is a reflection across the x-axis, followed by a translation of 6 units right and 4 units up
Now,
The given coordinate plane is:
Envision Math Common Core Grade 8 Answer Key Topic 6 Congruence And Similarity 77
From the given coordinate plane,
The vertices of ΔDEF are:
D (4, 5), E (5, 1), and F (1, 2)
The vertices of ΔD’E’F’ are:
D’ (-2, -1), E’ (-1, 3), and F’ (-5, 2)
Now,
The correct sequence of Transformations to show ΔDEF and ΔD’E’F’ are congruent is:

Hence, from the above,
We can conclude that the mistake done by a student is the interchange of the sequence of Transformations
Hence,
The correct sequence of Transformations is:
Translation of 6 units left and 4 units down followed by the reflection across the x-axis

Assessment Practice
Question 14.
PART A
How can you determine whether △DEF ≅ △D’ E’ F?
Envision Math Common Core Grade 8 Answer Key Topic 6 Congruence And Similarity 78
A. Determine whether a sequence of rotations maps △DEF onto △D’E’F’.
B. Determine whether a sequence of transformations maps △DEF onto △D’ E’F’.
C. Determine whether a sequence of translations maps △DEF onto △D’ E’F’.
D. Determine whether a sequence of reflections maps △DEF onto △D’ E’F’.
Answer:
We know that,
If we want to find whether the given figures are congruent or not, then
We have to determine whether a sequence of transformations maps ΔDEF onto ΔD’E’F’
Hence, from the above,
We can conclude that option A matches the given situation

PART B
Is △DEF ≅ △D’ E’ F? Explain.
Answer:
The given coordinate plane is:
Envision Math Common Core Grade 8 Answer Key Topic 6 Congruence And Similarity 78
From the given coordinate plane,
The vertices of ΔDEF are:
D (5, 5), E (6, 3), and F (2, 4)
The vertices of ΔD’E’F’ are:
D’ (-2, -1), E’ (-3, 1), and F’ (1, 0)
So,
The representation of the sequence of transformations to find out whether ΔDEf and ΔD’E’F’ are congruent or not is:

Hence, from the above,
We can conclude that ΔDEF is congruent to ΔD’E’F’

Topic 6 MID-TOPIC CHECKPOINT

Question 1.
Vocabulary Describe three transformations where the image and preimage have the same size and shape. Lesson 6-1, Lesson 6-2, and Lesson 6-3
Answer:
We know that,
There are four types of transformations. They are:
a. Translation
b. Reflection
c. Rotation
d. Translation
Now,
Some transformations keep the pre-image and image congruent. Congruent means that they are the same size and shape or that they have the same measurements. They make not have the same orientation
Hence,
Three of the four transformations that preserve the size and shape of the pre-image are: Translation, Rotation, and Reflections

For 2-6, use the figures below.
Envision Math Common Core Grade 8 Answer Key Topic 6 Congruence And Similarity 79
Answer:
The given coordinate plane is:
Envision Math Common Core Grade 8 Answer Key Topic 6 Congruence And Similarity 79
From the given coordinate plane,
The vertices of the quadrilateral MNPQ are:
M (1, 2), N (2, 4), P (4, 5), and Q (3, 3)
The vertices of the quadrilateral RSTU are:
R (1, -2), S (2, -4), T (4, -5), and U (3, -3)

Question 2.
What are the coordinates of each point after quadrilateral RSTU is rotated 90° about the origin? Lesson 6-3
Answer:
We know that,
When any point is rotated 90° about the origin,
(x, y) before rotation ——–> (-y, x) after rotation
Hence,
After 90° rotation about the origin,
The coordinates of each point of the quadrilateral RSTU will become:
R (2, 1), S (4, 2), T (5, 4), and U (3, 3)

Question 3.
What are the coordinates of each point after quadrilateral MNPQ is translated 2 units right and 5 units down? Lesson 6-1
Answer:
We know that,
When the translation occurs,
(x, y) before Translation ——-> (x + h, y + k) after Translation
Where,
h is the translation on the x-axis
k is the translation on the y-axis
We will take the positive value of h when the translation occurs on the right side of the x-axis
We will take the negative value of h when the translation occurs on the left side of the x-axis
We will take the positive value of k when the translation occurs on the top side of the y-axis
We will take the negative value of k when the translation occurs on the down side of the y-axis
Hence,
After the Translation of 2 units right and 5 units down,
The coordinates of each point of the quadrilateral MNPQ will become:
M (1 + 2, 2 – 5), N (2 + 2, 4 – 5), P (4 + 2, 5 – 5), and Q (3 + 2, 3 – 5)
M (3, -3), N (4, -1), P (6, 0), and Q (5, -2)

Question 4.
What are the coordinates of each point after quadrilateral MNPQ is reflected across the x-axis and then translated into 3 units left? Lessons 6-2 and 6-4
Answer:
We know that,
The vertices of the quadrilateral MNPQ are:
M (1, 2), N (2, 4), P (4, 5), and Q (3, 3)
So,
After the vertices of the quadrilateral, MNPQ reflected across the x-axis,
The vertices of the quadrilateral MNPQ will become:
M (1, -2), N (2, -4), P (4, -5), and Q (3, -3)
Now,
After the Translation of 3 units left,
The vertices of the quadrilateral MNPQ will become:
M (1 – 3, -2), N (2 – 3, -4), P (4 – 3, -5), and Q (3 – 3, -3)
M (-2, -2), N (-1, -4), P (1, -5), and Q (0, -3)
Hence, from the above,
We can conclude that the coordinates of each point after quadrilateral MNPQ is reflected across the x-axis and then translated into 3 units left are:
M (-2, -2), N (-1, -4), P (1, -5), and Q (0, -3)

Question 5.
Which series of transformations maps quadrilateral MNPQ onto quadrilateral RSTU? Lesson 6-4
A. reflection across the x-axis, translation 4 units down
B. reflection across the y-axis, translation 4 units down
C. rotation 180° about the origin, and then reflection across the x-axis
D. rotation 180° about the origin, and then reflection across the y-axis
Answer:
The representation of the series of transformations that maps the quadrilateral MNPQ onto the quadrilateral RSTU is:

Hence, from the above,
We can conclude that option A matches the given situation

Question 6.
Is quadrilateral MNPQ congruent to quadrilateral RSTU? Explain. Lesson 6-5
Answer:
We know that,
The 2 figures are said to be congruent only when:
a. The shapes of the 2 figures are the same
b. The sizes of the 2 figures are the same (Side lengths, and Angle measures)
Now,

From the above,
We can observe that the side lengths are not the same
Hence, from the above,
We can conclude that the quadrilateral MNPQ is not congruent to the quadrilateral RSTU

Topic 6 MID-TOPIC PERFORMANCE TASK

A tessellation is a design in a plane that uses one or more congruent figures, with no overlaps and no gaps, to cover the entire plane. A tessellation of an equilateral triangle is shown.
Envision Math Common Core Grade 8 Answer Key Topic 6 Congruence And Similarity 80

PART A
Explain how the tessellation of an equilateral triangle is formed using reflections.
Answer:
We know that,
When you cut a shape out of paper, then flip it over, the flipped shape looks like a mirror image of the original shape. So a tessellation made with this technique is called a “Reflection tessellation”
Now,
See if the figure will fit together with no gaps. The answer is yes, the figures will tessellate because it is made up of two shapes that do tessellate

PART B
Explain how the tessellation of an equilateral triangle is formed using rotations.
Answer:
We know that
A “Rotational tessellation” is a pattern where the repeating shapes fit together by rotating 90 degrees
Now,
See if the figure will fit together with no gaps. The answer is yes, the figures will tessellate because it is made up of two shapes that do tessellate

PART C
Which of the regular polygon(s) below can be tessellated using a series of transformations?
Envision Math Common Core Grade 8 Answer Key Topic 6 Congruence And Similarity 81
Answer:
We know that,
Equilateral triangles, squares, and regular hexagons are the only regular polygons that will tessellate. Therefore, there are only three regular tessellations.
Hence, from the above,
We can conclude that square and pentagon can be tessellated by using a series of transformations

Lesson 6.6 Describe Dilations

Solve & Discuss It!
A landscape architect designs a small splash pad represented by △ABC. Then she decides to make the splash pad larger as shown by △ADE. How are the splash pad designs alike? How are they different?
Envision Math Common Core Grade 8 Answer Key Topic 6 Congruence And Similarity 82
I can… dilate two-dimensional figures
Answer:
It is given that
A landscape architect designs a small splash pad represented by △ABC. Then she decides to make the splash pad larger as shown by △ADE.
Now,
The given landscape architect is:
Envision Math Common Core Grade 8 Answer Key Topic 6 Congruence And Similarity 82
Now,
From the given landscape architect,
We can observe that the difference in the splash designs
Hence, from the above landscape architect,
We can conclude that
The splash designs are alike in:
a. Shape b. Angle measures c. Orientation
The splash designs are different in:
a. Side lengths

Look for Relationships
How can you use what you know about scale drawings to compare and contrast the designs?
Answer:
A dilation is a transformation that results in an image with the same shape, angle measures, and orientation as the preimage, but different side lengths.
We know that,
When the scale factor is greater than 1, the dilation is a reduction.
When the scale factor is between 0 and 1, the dilation is an enlargement.

Focus on math practices
Reasoning Paul wants to make two square picnic tables. One table will have side lengths that are \(\frac{1}{2}\) of the lengths of the second table. How do the tablets compare? Explain.
Answer:
It is given that
Paul wants to make two square picnic tables. One table will have side lengths that are \(\frac{1}{2}\) of the lengths of the second table
So,
The size of the tables are in the ratio of \(\frac{1}{2}\) : 1
So,
The size of the tables are in the ratio of 1:2
Hence, from the above,
We can conclude that the side lengths of one table are 2 times the side lengths of the second table

Essential Question
What is the relationship between a preimage and its image after a dilation?
Answer:
After dilation, the pre-image and image have the same shape but not the same size.
In terms of Sides:
In dilation, the sides of the pre-image and the corresponding sides of the image are proportional.

Try It!

F’G’ H’I’ is the image of FGHI after a dilation with center at the origin. What is the scale factor?
Envision Math Common Core Grade 8 Answer Key Topic 6 Congruence And Similarity 83
The ratio of a side length in FGHI to a corresponding side length in F’GH’I’is: \(\frac{}{}\)
The scale factor is __________.
Answer:
It is given that
F’G’ H’I’ is the image of FGHI after a dilation with center at the origin.
Now,
The given coordinate plane is:
Envision Math Common Core Grade 8 Answer Key Topic 6 Congruence And Similarity 83
From the given coordinate plane,
The vertices of FGHI are:
F (1, 1), G (1, 2), H (2, 2), and I (2, 1)
The vertices of F’G’H’I’ are:
F’ (5, 5), G’ (5, 10), H’ (10, 10), and I’ (10, 5)
We know that,
The “Scale factor” is the ratio of a length in the image to the corresponding length in the preimage
So,
The ratio of a side length in FGHI to a corresponding side length in F’GH’I’is:
\(\frac{Side length of F’G’}{Side length of FG}\) (or) \(\frac{Side length of H’I’}{Side length of HI}\)
= \(\frac{0 + 5}{0 + 1}\) (or) \(\frac{0 + 5}{0 + 1}\)
= 5
So,
The scale factor is: 5
Hence, from the above,
We can conclude that
The ratio of a side length in FGHI to a corresponding side length in F’GH’I’is: \(\frac{5}{1}\)
The scale factor is 5.

Convince Me!
Quadrilateral WXYZ is the image of quadrilateral FGHI after a dilation with center at the origin and a scale factor of 3.5. What are the coordinates of the vertices of quadrilateral WXYZ?
Answer:
It is given that
Quadrilateral WXYZ is the image of quadrilateral FGHI after a dilation with center at the origin and a scale factor of 3.5
Now,
The given coordinate plane is:
Envision Math Common Core Grade 8 Answer Key Topic 6 Congruence And Similarity 83
From the given coordinate plane,
The vertices of FGHI are:
F (1, 1), G (1, 2), H (2, 2), and I (2, 1)
It is given that
The scale factor is: 3.5
So,
The vertices of the quadrilateral WXYZ are:
W ( 1 × 3.5, 1 ×3.5), X (1 × 3.5, 2 × 3.5), Y (2 × 3.5, 2 × 3.5), and Z (2 × 3.5, 1 × 3.5)
So,
W (3.5, 3.5), X (3.5, 7), Y (7, 7), and Z (7, 3.5)
Hence, from the above,
We can conclude that
The coordinates of the vertices of quadrilateral WXYZ are:
W (3.5, 3.5), X (3.5, 7), Y (7, 7), and Z (7, 3.5)

Try It!

A dilation maps point L(3, 6) to its image L’ (2, 4). Complete the dilation of figure LMN and label the image L’M’N’. What is the scale factor? What is the length of side M’N’?
Envision Math Common Core Grade 8 Answer Key Topic 6 Congruence And Similarity 84
Answer:
It is given that
A dilation maps point L(3, 6) to its image L’ (2, 4). Complete the dilation of figure LMN and label the image L’M’N’
Now,
The given coordinate plane is:
Envision Math Common Core Grade 8 Answer Key Topic 6 Congruence And Similarity 84
From the coordinate plane,
The vertices of ΔLMN are:
L (3, 6), M (3, 3), and N (6, 3)
It is given that the image of L is: L’ (2, 4)
Now,
We know that,
The “Scale factor” is the ratio of a length in the image to the corresponding length in the preimage
So,
The scale factor = \(\frac{Side length of L’}{Side length of L}\)
= \(\frac{4 – 2}{6 – 3}\)
= \(\frac{2}{3}\)
So,
The scale factor is: \(\frac{2}{3}\)
Now,
The coordinates of M’ = The coordinates of M × \(\frac{2}{3}\)
= (3, 3) × \(\frac{2}{3}\)
= (3 × \(\frac{2}{3}\), 3 × \(\frac{2}{3}\))
= (2, 2)
The coordinates of N’ = The coordinates of N × \(\frac{2}{3}\)
= (6, 3) × \(\frac{2}{3}\)
= (6 × \(\frac{2}{3}\), 3 × \(\frac{2}{3}\))
= (4, 2)
Now,
Compare M’ and N’ with (x 1, y 1), (x 2, y 2 )
We know that,
The distance between 2 points = √(x 2 – x 1 ) 2 + (y 2 – y 1 ) 2
So,
The side length of M’N’ = √(4 – 2 ) 2 + (2 – 2 )
= 2 units
Hence, from the above,
We can conclude that
The scale factor is: \(\frac{2}{3}\)
The side length of M’N’ is: 2 units

KEY CONCEPT

A dilation is a transformation that results in an image with the same shape, angle measures, and orientation as the preimage, but different side lengths.
When the scale factor is greater than 1, the dilation is an enlargement.
Envision Math Common Core Grade 8 Answer Key Topic 6 Congruence And Similarity 85

When the scale factor is between 0 and 1, the dilation is a reduction.
Envision Math Common Core Grade 8 Answer Key Topic 6 Congruence And Similarity 86

Do You Understand?
Question 1.
Essential Question What is the relationship between a preimage and its image after a dilation?
Answer:
After dilation, the pre-image and image have the same shape but not the same size.
In terms of Sides:
In dilation, the sides of the pre-image and the corresponding sides of the image are proportional.

Question 2.
Generalize When will a dilation be a reduction? When will it be an enlargement?
Answer:
When the scale factor is greater than 1, the dilation is an enlargement.
Envision Math Common Core Grade 8 Answer Key Topic 6 Congruence And Similarity 85
When the scale factor is between 0 and 1, the dilation is a reduction.
Envision Math Common Core Grade 8 Answer Key Topic 6 Congruence And Similarity 86

Question 3.
Reasoning Flora draws a rectangle with points at (12, 12), (15, 12), (15,9), and (12, 9). She dilates the figure with center at the origin and a scale factor of \(\frac{3}{4}\). what is the measure of each angle in the image? Explain.
Answer:
It is given that
Flora draws a rectangle with points at (12, 12), (15, 12), (15,9), and (12, 9). She dilates the figure with center at the origin and a scale factor of \(\frac{3}{4}\)
So,
The vertices for the image of the given rectangle = (x × \(\frac{3}{4}\), y × \(\frac{3}{4}\))
So,
The vertices for the image of the given rectangle wil be:
( 12 × \(\frac{3}{4}\), 12 × \(\frac{3}{4}\)), (15 × \(\frac{3}{4}\), 12 × \(\frac{3}{4}\)), (15 × \(\frac{3}{4}\), 9 × \(\frac{3}{4}\)), and (12 × \(\frac{3}{4}\), 9 × \(\frac{3}{4}\))
= (9, 9), (11.25, 9), (11.25, 6.75), and (9, 6.75)
Now,
We know that,
In dilation,
a. The preimage and image are the same in shape, orientation, and angle measures
b. the preimage and the image are different in size and the side lengths
So,
The representation of the angle measures in the preimage and image of the given rectangle is:

Hence, from the above,
We can conclude that the measure of each angle in the image is: 90°

Do You Know How?
In 4-6, use the coordinate grid below.
Envision Math Common Core Grade 8 Answer Key Topic 6 Congruence And Similarity 87
Answer:
The given coordinate plane is:
Envision Math Common Core Grade 8 Answer Key Topic 6 Congruence And Similarity 87
From the coordinate plane,
The vertices of Figure 1 are:
(4, 4), (6, 8), and (8, 4)
The vertices of Figure 2 are:
(2, 2), (3, 4), and (4, 2)
The vertices of Figure 3 are:
(1, 1), (1.5, 2), and (2, 1)

Question 4.
Figure 3 is the image of Figure 1 after a dilation with a center at the origin. What is the scale factor? Explain.
Answer:
It is given that
Figure 3 is the image of Figure 1 after a dilation with a center at the origin
Now,
We know that,
The “Scale factor” is the ratio of a length in the image to the corresponding length in the preimage
So,
Scale factor = \(\frac{4}{1}\) (or) \(\frac{6}{1.5}\) (or) \(\frac{8}{2}\)
= 4
Hence, from the above,
We can conclude that the scale factor is “4” so that Figure 3 is the image of Figure 1

Question 5.
What are the coordinates of the image of Figure 2 after a dilation with center at the origin and a scale factor of 3?
Answer:
We know that,
The vertices of Figure 2 are:
(2, 2), (3, 4), and (4, 2)
So,
With a scale factor of 3,
The vertices of Figure 2 will become:
(2 × 3, 2 × 3), (3 × 3, 4 × 3), and (4 × 3, 2 × 3)
= (6, 6), (9, 12), and (12, 6)
Hence, from the above,
We can conclude that the coordinates of the image of Figure 2 after a dilation at the origin and a scale factor of 3 are:
(6, 6), (9, 12), and (12, 6)

Question 6.
Which figures represent a dilation with a scale factor of \(\frac{1}{2}\)?
Answer:
We know that,
The vertices of Figure 1 are:
(4, 4), (6, 8), and (8, 4)
The vertices of Figure 2 are:
(2, 2), (3, 4), and (4, 2)
The vertices of Figure 3 are:
(1, 1), (1.5, 2), and (2, 1)
Now,
If we consider Figure 2 as the preimage and Figure 1 as the image, then
Scale factor = \(\frac{2}{4}\) (or) \(\frac{3}{6}\) (or) \(\frac{4}{8}\)
= \(\frac{1}{2}\)
If we consider Figure 3 as the preimage and Figure 2 as the image, then
Scale factor = \(\frac{1}{2}\) (or) \(\frac{1.5}{3}\) (or) \(\frac{2}{4}\)
= \(\frac{1}{2}\)
Hence, from the above,
We can conclude that (Figure 2, Figure 1) and (Figure 3, Figure 2) represent a dilation with a scale factor of \(\frac{1}{2}\)

Practice & Problem Solving

Question 7.
Leveled Practice Draw the image of △DEF after a dilation with center (0, 0) and scale factor of 2.
Envision Math Common Core Grade 8 Answer Key Topic 6 Congruence And Similarity 88
Find the coordinates of each point in the original figure.
D(____, (____) E(____, (____) F(____, (____)
Multiply each coordinate by _______.
Find the coordinates of each point in the image:
D'(____, (____) E'(____, (____) F'(____, (____)
Answer:
The given coordinate plane is:
Envision Math Common Core Grade 8 Answer Key Topic 6 Congruence And Similarity 88
Now,
From the given coordinate plane,
The vertices of ΔDEF are:
D (0, 0), E (2, 0), and F (0, 2)
Now,
With a scale factor of 2,
Multiply each coordinate with 2
The coordinates of ΔDEF will become:
D (0 × 2, 0 × 2), E (2 × 2, 0 × 2), and F (0 × 2, 2 × 2)
D (0, 0), E (4, 0), and F (0, 4)
So,
The coordinates of the points for the image of ΔDEF are:
D’ (0, 0), E’ (4, 0), and F’ (0, 4)
Hence,
The representation of ΔDEF and its image ΔD’E’F’ is:

Question 8.
Find the scale factor for the dilation shown.
Envision Math Common Core Grade 8 Answer Key Topic 6 Congruence And Similarity 89
Answer:
The given coordinate plane is:
Envision Math Common Core Grade 8 Answer Key Topic 6 Congruence And Similarity 89
Now,
From the given coordinate plane,
The vertices of Figure DEFG are:
D (0, 0), E (5, 0), F (5, 6), and G (0, 6)
The vertices of Figure D’E’F’G’ are:
D’ (0, 0), E’ (15, 0), G’ (15, 18), and H’ (0, 18)
Now,
We know that,
The “Scale factor” is the ratio of a length in the image to the corresponding length in the preimage
So,
Scale factor = \(\frac{15}{5}\) (or) \(\frac{18}{6}\)
= 3
Hence, from the above,
We can conclude that
The scale factor for the given dilation is: 3

Question 9.
Critique Reasoning For the dilation with center (0, 0) shown on the graph, your friend says the scale factor is \(\frac{5}{2}\). What is the correct scale factor? What mistake did your friend likely make?
Envision Math Common Core Grade 8 Answer Key Topic 6 Congruence And Similarity 90
Answer:
It is given that
For the dilation with center (0, 0) shown on the graph, your friend says the scale factor is \(\frac{5}{2}\)
Now,
The given coordinate plane is:
Envision Math Common Core Grade 8 Answer Key Topic 6 Congruence And Similarity 90
From the given coordinate plane,
The vertices of ΔABC are:
A (0, 5), B (-5, 0), and C (5, -5)
The vertices of ΔA’B’C’ are:
A’ (0, 2), B’ (-2, 0), C’ (2, -2)
Now,
We know that,
The “Scale factor” is the ratio of a length in the image to the corresponding length in the preimage
So,
Scale factor = \(\frac{2}{5}\) (or) \(\frac{-2}{-5}\)
= \(\frac{2}{5}\)
Hence, from the above,
We can conclude that
The correct scale factor is: \(\frac{2}{5}\)
The mistake done by your friend is:
The consideration of the scale factor as \(\frac{Length of the preimage}{length of the image}\) instead of \(\frac{Length of the image}{Length of the preimage}\)

Question 10.
The smaller figure is the image of dilation of the larger figure. The origin is the center of dilation. Tell whether the dilation is an enlargement or a reduction. Then find the scale factor of the dilation.
Envision Math Common Core Grade 8 Answer Key Topic 6 Congruence And Similarity 91
Answer:
It is given that
The smaller figure is the image of dilation of the larger figure. The origin is the center of dilation
Now,
The given coordinate plane is:
Envision Math Common Core Grade 8 Answer Key Topic 6 Congruence And Similarity 91
From the given coordinate plane,
The coordinates of each point of the preimage are:
(3, 6), (15, 6), (15, 18), and (3, 18)
The coordinates of each point of the image are:
(1, 2), (5, 2), (5, 6), and (1, 6)
Now,
We know that,
The “Scale factor” is the ratio of a length in the image to the corresponding length in the preimage
So,
Scale factor = \(\frac{1}{3}\) (or) \(\frac{2}{6}\) (or) \(\frac{5}{15}\) (or) \(\frac{6}{18}\)
= \(\frac{1}{3}\)
Now,
Since the scale factor is less than 1, the dilation is a reduction
Hence, from the above,
We can conclude that
The given dilation is a reduction
The scale factor is: \(\frac{1}{3}\)

Question 11.
Higher-Order Thinking Q’R’S’T’ is the image of QRST after a dilation with center at the origin.
Envision Math Common Core Grade 8 Answer Key Topic 6 Congruence And Similarity 92
Answer:
It is given that
Q’R’S’T’ is the image of QRST after a dilation with center at the origin
Now,
The given coordinate plane is:
Envision Math Common Core Grade 8 Answer Key Topic 6 Congruence And Similarity 92
From the given coordinate plane,
The vertices of the parallelogram QRST are:
Q (4, 4), R (16, 4), S (20, 16), and T (8, 16)
The vertices of the parallelogram Q’R’S’T’ are:
Q’ (1, 1), R’ (4, 1), S’ (5, 4), and T’ (2, 4)

a. Find the scale factor.
Answer:
We know that,
The “Scale factor” is the ratio of a length in the image to the corresponding length in the preimage
So,
Scale factor = \(\frac{4}{1}\) (or) \(\frac{16}{4}\) (or) \(\frac{20}{5}\) (or) \(\frac{8}{2}\)
= 4
Hence, from the above,
We can conclude that the scale factor is: 4

b. Find the area of each parallelogram. What is the relationship between the areas?
Answer:
We know that,
Area of the parallelogram = Base × Height
Now,
the representation of the side lengths of the parallelogram QRST and its dilation Q’R’S’T’ is:

So,
The area of the parallelogram QRST = 12.6 × 12
= 151.2 square units
The area of the parallelogram Q’R’S’T’ = \(\frac{The area of the parallelogram QRST}{4}\)
= 37.8 square units
Hence, from the above,
We can conclude that
The area of the parallelogram QRST is: 151.2 sq. units
The area of the parallelogram Q’R’S’T’ is: 37.8 sq. units
The relationship between the two areas is:
The area of the parallelogram Q’R’S’T’ = \(\frac{The area of the parallelogram QRST}{4}\)

Assessment Practice
Question 12.
Triangle PQR is the image of △JKL after a dilation. Is the dilation an enlargement or a reduction? Explain.
Envision Math Common Core Grade 8 Answer Key Topic 6 Congruence And Similarity 93
A. An enlargement, because the image is larger than the original figure
B. An enlargement, because the image is smaller than the original figure
C. A reduction, because the image is smaller than the original figure
D. A reduction, because the image is larger than the original figure
Answer:
It is given that
Triangle PQR is the image of △JKL after a dilation
Now,
From the given figure,
We can observe that ΔPQR is larger than ΔJKL
So,
We can say that the dilation is an enlargement
Hence, from the above,
We can conclude that option A matches with the given situation

Question 13.
Rectangle QUAD has coordinates Q(0, 0), U(0, 3), A(6, 3), and D(6, 0). Q’U’ A’D’ is the image of QUAD after a dilation with center (0, 0) and a scale factor of 6. What are the coordinates of point D’? Explain.
Answer:
It is given that
Rectangle QUAD has coordinates Q(0, 0), 4(0, 3), A6, 3), and D(6, 0). Q’U’ A’D’ is the image of QUAD after a dilation with center (0, 0) and a scale factor of 6.
So,
With a scale factor of 6,
The vertices of the rectangle QUAD will become:
Q (0 × 6, 0 × 6), U (0 × 6, 3 × 6), A (6 × 6, 3 × 6) and D (6 × 6, 0 × 6)
= Q (0, 0), U (0, 18), A (36, 18), and D (6, 0)
So,
After a dilation,
The vertices of the rectangle QUAD will become the vertices of the rectangle Q’U’A’D’
So,
The vertices of the rectangle Q’U’A’D’ are:
Q’ (0, 0), U’ (0, 18), A’ (36, 18), and D’ (36, 0)
Hence, from the above,
We can conclude that the coordinates of point D’ are: (36, 0)

Lesson 6.7 Understand Similar Figures

Solve & Discuss It!
Andrew draws the two figures shown on a coordinate plane. How are the two figures alike? How are they different? How do you know?
Envision Math Common Core Grade 8 Answer Key Topic 6 Congruence And Similarity 94
I can… use a sequence of transformations, including dilations, to show that figures are similar.
Answer:
It is given that
Andrew draws the two figures shown on a coordinate plane
Now,
The given figures are:
Envision Math Common Core Grade 8 Answer Key Topic 6 Congruence And Similarity 94
From the above,
We can observe that
ΔABC is the preimage
ΔA’B’C’ is the image
We can also observe that
The image is smaller than the image since the transformation we used when drawing the image is a “Dilation”
Hence,
Since the transformation we used is a “Dilation”,
The two figures are similar in terms of:
a. Shape b. Size c. Orientation d. Angle measures
The two figures are different in terms of:
a. Side lengths

Look for Relationships
Is △ABC a preimage of △A’B’C’? How do you know?
Answer:
The given figures are:
Envision Math Common Core Grade 8 Answer Key Topic 6 Congruence And Similarity 94
From the given figures,
We can clearly observe that the transformation called “Dilation” takes place
Since the dilation takes place,
We can conclude that ΔABC is a preimage of ΔA’B’C’

Focus on math practices
Reasoning How can you use the coordinates of the vertices of the triangles to identify the transformation that maps △ABC to △A’B’C’? Explain.
Answer:
The given figures are:
Envision Math Common Core Grade 8 Answer Key Topic 6 Congruence And Similarity 94
Now,
Fro the given figures,
We know that a transformation called a “Dilation” takes place
So,
We know that,
The “Scale factor” is the ratio of a length in the image to the corresponding length in the preimage
So,
Scale factor = [altex]\frac{The x-coordinate (or) y-coordinate that maps A’ or B’ or C’}{The x-coordinate (or) y-coordinate that maps A or B or C}[/latex}
Hence, from the above,
We can conclude that
By using the “Scale factor”, the coordinates of the vertices of the triangles can be used to identify the transformation that maps △ABC to △A’B’C’

Essential Question
How are similar figures related by a sequence of transformations?
Answer:
Two figures are similar if and only if one figure can be obtained from the other by a single transformation, or a sequence of transformations, including translations, reflections, rotations, and/or dilations. Similarity transformations preserve shape, but not necessarily size, making the figures “similar”

Try It!

Is △ΑΒC similar to △A’ B’ C?
The triangles _________ similar.
Envision Math Common Core Grade 8 Answer Key Topic 6 Congruence And Similarity 95
Answer:
The given figures are:
Envision Math Common Core Grade 8 Answer Key Topic 6 Congruence And Similarity 95
Now,
If we want to find the 2 triangles are similar or not, find the scale factor and find whether they have the same shape or not
If the scale factor for 2 triangles is the same, then those triangles are similar
Now,
We know that,
The scale factor is defined as the distance from this center point to a point on the preimage and also the distance from the center point to a point on the image
Now,
For AB and A’B’,
Scale factor = \(\frac{27}{12}\)
= \(\frac{9}{4}\)
For BC and B’C’,
Scale factor = \(\frac{27}{12}\)
= \(\frac{9}{4}\)
For AC and A’C’,
Scale factor = \(\frac{18}{8}\)
= \(\frac{9}{4}\)
Since the scale factor is equal for all the corresponding sides
We can conclude that ΔABC is similar to ΔA’B’C’

Convince Me!
What sequence of transformations shows that △ABC is similar to △A’ B’C’?
Answer:
The sequence of Transformations that shows ΔABC is similar to ΔA’B’C’ is:
a. Rotation b. Dilation c. Translation

Try It!

a. Graph the image of JKL after a reflection across the line x = 1 followed by dilation with a scale factor of \(\frac{1}{2}\) and center of dilation point J’.
Envision Math Common Core Grade 8 Answer Key Topic 6 Congruence And Similarity 96
Answer:
The given coordinate plane is:
Envision Math Common Core Grade 8 Answer Key Topic 6 Congruence And Similarity 96
From the given coordinate plane,
The vertices of ΔJKL are:
J (-2, -4), K (-4, 0), and L (-2, 1)
The vertices of ΔPQR are:
P (-4, 2), Q (4, 6), and R (6, 2)
Now,
After the reflection of ΔJKL across the line x = 1,
J (-1, 4), K (-3, 0), and L (-1, -1)
So,
With a scale factor of \(\frac{1}{2}\),
The vertices of ΔJKL will become:
J (-1, 4) × \(\frac{1}{2}\), K (-3, 0) × \(\frac{1}{2}\), and L (-1, -1) × \(\frac{1}{2}\)
So,
J’ (-0.5, 2), K’ (-1.5, 0), and L’ (-0.5, -0.5)
Hence,
The representation of the image of JKL after a reflection across the line x = 1 followed by dilation with a scale factor of \(\frac{1}{2}\) and center of dilation point J’ is:

b. Is △JKL similar to △PQR?
Answer:
We know that,
The vertices of ΔJKL are:
J (-2, -4), K (-4, 0), and L (-2, 1)
The vertices of ΔPQR are:
P (-4, 2), Q (4, 6), and R (6, 2)
Now,
Step 1:
Rotate the vertices of ΔJKL 270° about the origin
We know that,
(x, y) before 270° rotation —-> (y, -x) after 270° rotation
So,
The vertices of ΔJKL will be:
J (-4, 2), K (0, 4), and L (1, 2)
Step 2:
Dilate the vertices of ΔJKL we obtained in step 1 by a scale factor of 2
So,
The vertices of ΔJKL will be:
J (-8, 4), K (0, 8), and L (2, 4)
Step 3:
Translate the vertices of ΔJKL we obtained in step 2 by 4 units right and 2 units down
So,
The vertices of ΔJKL will become:
P (-4, 2), Q (4, 6), and R (6, 2)
So,
The representation of the sequence of transformations is:

Hence, from the above,
We can conclude that ΔJKL is similar to ΔPQR

KEY CONCEPT

Two-dimensional figures are similar if there is a sequence of rotations, reflections, translations, and dilations that maps one figure onto the other.

Do You Understand?
Question 1.
Essential Question How are similar figures related by a sequence of transformations?
Answer:
Two figures are similar if and only if one figure can be obtained from the other by a single transformation, or a sequence of transformations, including translations, reflections, rotations, and/or dilations. Similarity transformations preserve the shape, but not necessarily size, making the figures “similar”

Question 2.
Be Precise How do the angle measures and side lengths compare in similar figures?
Answer:
Matching sides of two or more polygons are called corresponding sides, and matching angles are called corresponding angles. If two figures are similar, then the measures of the corresponding angles are equal and the ratios of the lengths of the corresponding sides are proportional.

Question 3.
Generalize Does a given translation, reflection, or rotation, followed by a given dilation, always map a figure to the same image as that same dilation followed by that same translation, reflection, or rotation? Explain.
Answer:
We know that,
The images formed by reflection, translation, and rotation have the same shape and size whereas dilation has the same shape but a different size
In the same way,
The image formed by dilation has the same shape but a different size whereas the image formed by reflection, translation, and rotation has the same shape and size as the image formed in dilation
Hence, from the above,
We can conclude that
The translation, reflection, or rotation, followed by a given dilation, always map a figure to the same image as that same dilation followed by that same translation, reflection, or rotation

Do You Know How?
Question 4.
Is trapezoid ABCD – trapezoid EFGH? Explain.
Envision Math Common Core Grade 8 Answer Key Topic 6 Congruence And Similarity 97
Answer:
The given figures are:
Envision Math Common Core Grade 8 Answer Key Topic 6 Congruence And Similarity 97
From the given figures,
We can observe that
The corresponding angles i.e., the opposite angles are equal
Now,
The ratio of the side lengths of the corresponding figures = \(\frac{32}{16}\) (or) \(\frac{28}{14}\) (or) \(\frac{20}{10}\)
= 2
So,
The ratio of the side lengths of the corresponding figures are the same
Now,
We know that,
Two figures are said to be similar if they are the same shape. In more mathematical language, two figures are similar if their corresponding angles are congruent, and the ratios of the lengths of their corresponding sides are equal. This common ratio is called the scale factor
Hence, from the above,
We can conclude that
Trapezoid ABCD – Trapezoid EFGH

Use the graph for 5 and 6.
Envision Math Common Core Grade 8 Answer Key Topic 6 Congruence And Similarity 98
Answer:
The given coordinate plane is:
Envision Math Common Core Grade 8 Answer Key Topic 6 Congruence And Similarity 98
From the given coordinate plane,
The vertices of ΔABC are:
A (-4, -2), B (-1, -2), and C (-3, -3)
The vertices of ΔDEF are:
D (9, -5), E (6, -8), and F (1, -5)

Question 5.
△ABC is dilated by a factor of 2 with a center of dilation at point C, reflected across the x-axis, and translated 3 units up. Graph the resulting similar figure.
Answer:
We know that,
The vertices of ΔABC are:
A (-4, -2), B (-1, -2), and C (-3, -3)
So,
Step 1:
After dilation by a factor of 2:
Multiply all the coordinates of each point by 2
So,
The vertices of ΔABC are:
A (-8, -4), B (-2, -4), and C (-6, -6)
Now,
With a dilation at point C,
Add all the coordinates of each point with -3
So,
The vertices of ΔABC are:
A (-11, -7), B (-5, -7), and C (-9, -9)
Step 2:
When the vertices of ΔABC that we obtained in step 1 reflects across the x-axis,
The vertices of ΔABC will be:
A (-11, 7), B (-5, 7), and C (-9, 9)
Step 3:
When the vertices of ΔABC we obtained in step 2 translated by 3 units up,
Add all the y-coordinates of each point with 3
So,
The vertices of ΔABC will be:
A (-11, 10), B (-5, 10), and C (-9, 12)
Hence,
The representation of all the steps is:

Question 6.
Is △ABC similar to △DEF? Explain.
Answer:
We know that,
The vertices of ΔABC are:
A (-4, -2), B (-1, -2), and C (-3, -3)
The vertices of ΔDEF are:
D (9, -5), E (6, -8), and F (1, -5)
We know that,
Two figures are said to be similar if they are the same shape. In more mathematical language, two figures are similar if their corresponding angles are congruent, and the ratios of the lengths of their corresponding sides are equal
So,
The representation of ΔABC and ΔDEF are:

From the above,
We can observe that the ratio of the corresponding side lengths are not in the same proportion
Hence, from the above,
We can conclude that ΔABC is not similar to ΔDEF

Practice & Problem Solving

Question 7.
Leveled Practice RSTU and VXYZ are quadrilaterals. Given RSTU ~ VXYZ, describe a sequence of transformations that maps RSTU to VXYZ.
Envision Math Common Core Grade 8 Answer Key Topic 6 Congruence And Similarity 99
• reflection across the ________
• translation ______ unit(s) left and _______ unit(s) up
• dilation with center (0,0) and a scale factor of ________
Answer:
The given coordinate plane is:
Envision Math Common Core Grade 8 Answer Key Topic 6 Congruence And Similarity 99
From the coordinate plane,
The vertices of the quadrilateral VXYZ are:
V (-2, -1), X (-3, 0), Y (-2, 2), and Z (-1, 0)
The vertices of the quadrilateral RSTU are:
R (3, 0), S (6, 3), T (3, 9), and U (0, 3)
Now,
Step 1:
Reflect the vertices of the quadrilateral VXYZ across the y-axis
So,
The vertices of the quadrilateral VXYZ will be:
V (2, -1), X (3, 0), Y (2, 2), and Z (1, 0)
Step 2:
Dilate the vertices of the quadrilateral VXYZ with a scale factor of 3
So,
Multiply all the coordinates of all the vertices of the quadrilateral VXYZ by 3
So,
The vertices of the quadrilateral VXYZ will be:
V (6, -3), X (9, 0), Y (6, 6), and Z (3, 0)
Step 3:
Translate the vertices of the quadrilateral VXYZ 3 units left and 3 units up
So,
The vertices of the quadrilateral VXYZ will become:
R (3, 0), S (6, 3), T (3, 9), and U (0, 3)
Hence,
The sequence of Transformations that map the quadrilateral VXYZ onto the quadrilateral RSTU is:

Question 8.
Reasoning Is △MNO similar to △PQO? Explain.
Envision Math Common Core Grade 8 Answer Key Topic 6 Congruence And Similarity 100
Answer:
The given coordinate plane is:
Envision Math Common Core Grade 8 Answer Key Topic 6 Congruence And Similarity 100
From the coordinate plane,
The vertices of ΔMNO are:
M (6, 6 ), N (0, 6), and O (0, 0)
The vertices of ΔPQO are:
P (-12, -9), Q (0, -9), and O (0, 0)
We know that,
Two figures are said to be similar if they are the same shape. In more mathematical language, two figures are similar if their corresponding angles are congruent, and the ratios of the lengths of their corresponding sides are equal
So,
The representation of ΔMNO and ΔPQO are:

From the above,
We can observe that
a. The ratio of the corresponding side lengths are not in the same proportion
b. The angle measures are not the same
Hence, from the above,
We can conclude that ΔMNO is not similar to ΔPQO

Question 9.
△PQR is dilated by a scale factor of 2 with a center of dilation (0, 0) and rotated 180° about the origin. Graph the resulting similar △XYZ.
Envision Math Common Core Grade 8 Answer Key Topic 6 Congruence And Similarity 101
Answer:
It is given that
△PQR is dilated by a scale factor of 2 with a center of dilation (0, 0) and rotated 180° about the origin.
Now,
The given coordinate plane is:
Envision Math Common Core Grade 8 Answer Key Topic 6 Congruence And Similarity 101
From the given coordinate plane,
The vertices of ΔPQR are:
P (2, 2), Q (4, 2), and R (3, 4)
Now,
Step 1:
Dilation of ΔPQR by a scale factor of 2:
Multiply all the coordinates of each point of ΔPQR by 2
So,
The vertices of ΔPQR will be:
P (4, 4), Q (8, 4), and R (6, 8)
Step 2:
Rotation of 180° counterclockwise about the origin:
We know that,
(x, y) before rotating 180° ——- > (-x, -y) after rotating 180°
So,
The vertices of ΔPQR that we obtained in step 1 will become:
X (-4, -4), Y (-8, -4), and Z (-6, -8)
Hence,
The graph of the resulting ΔXYZ is:

Question 10.
Describe a sequence of transformations that shows that quadrilateral RSTU is similar to quadrilateral VXYZ.
Envision Math Common Core Grade 8 Answer Key Topic 6 Congruence And Similarity 102
Answer:
The given coordinate plane is:
Envision Math Common Core Grade 8 Answer Key Topic 6 Congruence And Similarity 102
From the given coordinate plane,
The vertices of the quadrilateral VXYZ are:
V (-5, -5), X (-5, -1), Y (-1, -1), and Z (-1, -5)
The vertices of the quadrilateral RSTU are:
R (-16, -14), S (-16, -6), T (-8, -6), and U (-8, -14)
Now,
Step 1:
Translate the quadrilateral VXYZ 3 units left and 2 units down
So,
The vertices of the quadrilateral VXYZ will be:
V (-8, -7), X (-8, -3), Y (-4, -3), and Z (-4, -7)
Step 2:
Dilate the vertices of the quadrilateral VXYZ we obtained in step 1 by a scale factor of 2
So,
The vertices of the quadrilateral VXYZ will be:
R (-16, -14), S (-16, -6), T (-8, -6), and U (-8, -14)
Hence,
The sequence of Transformations that shows the quadrilateral VXYZ is similar to the quadrilateral RSTU is:

Question 11.
Construct Arguments Is △PQR similar to △XYZ? Explain.
Envision Math Common Core Grade 8 Answer Key Topic 6 Congruence And Similarity 103
Answer:
The given coordinate plane is:
Envision Math Common Core Grade 8 Answer Key Topic 6 Congruence And Similarity 103
From the given coordinate plane,
The vertices of ΔXYZ are:
X (4, 4), Y (4, 8), and Z (8, 8)
The vertices of ΔPQR are:
P (2, -2), Q (4, -2), and R (4, -4)
Now,
Step 1:
Rotate ΔXYZ 270° about the origin
We know that,
(x, Y) before 270° rotation —— > (y, -x) after 270° rotation
So,
The vertices of ΔXYZ will become:
X ( 4, -4), Y (8, -4), and Z (8, -8)
Step 2:
Dilate ΔXYZ with a scale factor of \(\frac{1}{2}\)
So,
The vertices of ΔXYZ that we obtained in step 1 will become:
P (2, -2), Q (4, -2), and R (4, -4)
So,
After the sequence of Rotation and Dilation of ΔXYZ,
We obtained the vertices of ΔPQR
Hence, from the above,
We can conclude that ΔPQR is similar to ΔXYZ

Question 12.
Higher-Order Thinking Given △JKL ~ △XYZ, find two possible coordinates for missing point Y. For each coordinate chosen, describe a sequence of transformations, including a dilation, that will map △JKL to △XYZ.
Envision Math Common Core Grade 8 Answer Key Topic 6 Congruence And Similarity 104
Answer:
It is given that
Given △JKL ~ △XYZ
Now,
The given coordinate plane is:
Envision Math Common Core Grade 8 Answer Key Topic 6 Congruence And Similarity 104
From the coordinate plane,
The vertices of ΔJKL are:
J (2, 8), K (6, 2), and L (2, 2)
The vertices of ΔXYZ are:
X (-2, 5), Y (x, y), and Z (-2, 2)
Now,
Step 1:
Reflect ΔJKL across the y-axis
So,
The vertices of ΔJKL will be:
J (-2, 8), K (-6, 2), and L (-2, 2)
Step 2:
Dilate the vertices of ΔJKL that we obtained in step by a scale factor of \(\frac{1}{2}\)
So,
The vertices of ΔJKL will be:
(-1, 4), K (-3, 1), and L (-1, 1)
Step 3:
Translate the vertices of ΔJKL that we obtained in step 2 by 1 unit left and 1 unit up
So,
The vertices of ΔJKL will become:
X (-2, 5), Y (-4, 2), and Z (-2, 2)
Hence, from the above,
We can conclude that the coordinates for the missing point Y are: (-4, 2)

Assessment Practice
Question 13.
Rajesh is making pennants in preparation for a school soccer game. He wants the pennants to be similar triangles. Which of these triangles could he use for the pennants?
Envision Math Common Core Grade 8 Answer Key Topic 6 Congruence And Similarity 105
A. △QRS and △TVW
B. △QRS and △XYZ
C. △TVW and △JKL
D. △TVW and △XYZ
Answer:
It is given that
Rajesh is making pennants in preparation for a school soccer game. He wants the pennants to be similar triangles.
Now,
If we want to find which of the triangles are similar, find the scale factor and find whether they have the same shape or not
If the scale factor for 2 triangles is the same, then those triangles are similar
Now,
We know that,
The scale factor is defined as the distance from this center point to a point on the preimage and also the distance from the center point to a point on the image
Now,
For ΔQRS and ΔTVW,
Scale factor = \(\frac{3}{1}\) (or) \(\frac{3}{1.5}\)
= 3 (or) 2
For ΔQRS and ΔXYZ,
Scale factor = \(\frac{3}{1.5}\) (or) \(\frac{3}{1.5}\)
= 2
Hence from the above,
We can conclude that ΔQRS and ΔXYZ are similar triangles and he can use these triangles for the pennants

Question 14.
Determine whether the following pairs of triangles are similar or not similar.
Envision Math Common Core Grade 8 Answer Key Topic 6 Congruence And Similarity 106
Answer:
If we want to find which of the triangles are similar, find the scale factor and find whether they have the same shape or not
If the scale factor for 2 triangles is the same, then those triangles are similar
Now,
We know that,
The scale factor is defined as the distance from this center point to a point on the preimage and also the distance from the center point to a point on the image
Now,
For ΔABC and ΔDEF,
Scale factor = \(\frac{2}{1}\) (or) \(\frac{2}{1}\)
= 2
For ΔABC and ΔLMN,
Scale factor = \(\frac{2}{1.5}\) (or) \(\frac{2}{1}\)
= 1.33 (or) 2
For ΔDEF and ΔLMN,
Scale factor = \(\frac{1}{1}\) (or) \(\frac{1.5}{1}\)
= 1 (or) 1.5
Hence from the above,
We can conclude that ΔABC and ΔDEF are similar triangles

Lesson 6.8 Angles, Lines, and Transversals

Solve & Discuss It!
Draw two parallel lines. Then draw a line that intersects both lines. Which angles have equal measures?
Envision Math Common Core Grade 8 Answer Key Topic 6 Congruence And Similarity 107
I can… identify and find the measures of angles formed by parallel lines and a transversal.
Answer:
Let a and b be the two parallel lines and the line that is intersecting the parallel lines is a “Transversal”
So,
The representation of the parallel lines along with the transversal is:

Now,
From the above,
The angles that have equal measures are:
Corresponding angles:
∠1 = ∠5, ∠2 = ∠6, ∠3 = ∠7, and ∠4 = ∠8
Alternate Interior angles:
∠4 = ∠6, ∠3 = ∠5
Alternate Exterior angles:
∠1 = ∠7, ∠2 = ∠8
Vertically Opposite Angles:
∠1 = ∠7, ∠2 = ∠8

Use Appropriate Tools
What tools can you use to determine which angles have equal measures?
Answer:
The tools that can be used to determine which angles have equal measures are:
a. Pencil b. Scale c. Protractor d. Setsquare

Focus on math practices
Reasoning What properties or definitions can you use to describe which angles have equal measures?
Answer:
The definitions that you can use to describe which angles have equal measures are:
Corresponding angles:
Corresponding angles are angles that are in the same position relative to lines intersected by a transversal
Alternate Interior angles:
When two parallel lines are crossed by a transversal, the pair of angles formed on the inner side of the parallel lines, but on the opposite sides of the transversal are called alternate interior angles
Alternate Exterior angles:
Alternate exterior angles are the pair of angles that lie on the outer side of the two parallel lines but on either side of the transversal line

Essential Question
What are the relationships among angles that are created when a line intersects two parallel lines?
Answer:
If two parallel lines are cut by a third line, the third line is called the transversal.
So,
The relationships among angles that are created when a line intersects two parallel lines are :
(1) Corresponding angles
(2) Vertically Opposite angles
(3) Alternate interior angles
(4) Alternate exterior angles

Try It!

Which angles are congruent to ∠8?
Envision Math Common Core Grade 8 Answer Key Topic 6 Congruence And Similarity 108
Envision Math Common Core Grade 8 Answer Key Topic 6 Congruence And Similarity 109 Envision Math Common Core Grade 8 Answer Key Topic 6 Congruence And Similarity 109 Envision Math Common Core Grade 8 Answer Key Topic 6 Congruence And Similarity 109
Which angles are supplementary to ∠8?
Envision Math Common Core Grade 8 Answer Key Topic 6 Congruence And Similarity 109 Envision Math Common Core Grade 8 Answer Key Topic 6 Congruence And Similarity 109 Envision Math Common Core Grade 8 Answer Key Topic 6 Congruence And Similarity 109 Envision Math Common Core Grade 8 Answer Key Topic 6 Congruence And Similarity 109
Answer:
The given transversal is:
Envision Math Common Core Grade 8 Answer Key Topic 6 Congruence And Similarity 108
We know that,
The angle relationships that have equal angle measures are:
a. Corresponding angles b. Vertically opposite angles c. Alternate interior angles d. Alternate exterior angles
We know that,
The angle relationships that have supplementary angle measures are:
a. Adjacent angles
Hence, from the above,
We can conclude that
The angles that are congruent to ∠8 are: ∠4, ∠6 and ∠2
The angles that are supplementary to ∠8 are: ∠7, ∠1, ∠2, and ∠6

Convince Me!
Use what you know about other angle relationships to explain why ∠4 and ∠5 are supplementary angles.
Answer:
The given transversal is:
Envision Math Common Core Grade 8 Answer Key Topic 6 Congruence And Similarity 108
Now,
From the given Transversal,
We can observe that
∠4 and ∠5 are consecutive interior angles
Now,
We know that,
The sum of the consecutive interior angles is always equal to 180°
The angles that have the sum 180° are also called “Supplementary angles”
Hence, from the above,
We can conclude that ∠4 and ∠5 are supplementary angles

Try It!

What are the measures of ∠7 and ∠2? Explain.

Answer:
The given transversal is:

Now,
From the given transversal,
We can observe that
∠4 and 99° are the corresponding angles
∠4 and ∠2 are alternate interior angles
∠5 and ∠7 are vertical angles
∠4 and ∠5 are supplementary angles
So,
Now,
∠4 = 99°
Now,
∠4 + ∠5 = 180°
∠5 = 180° – 99°
∠5 = 81°
Now,
∠2 = 99°
∠7 = 81°
Hence, from the above,
We can conclude that
The values of ∠7 and ∠2 are: 81° and 99° respectively

Try It!

In the figure, a || b. What is the value of x?
Envision Math Common Core Grade 8 Answer Key Topic 6 Congruence And Similarity 110
Answer:
It is given that a || b
Now,
The given transversal is:
Envision Math Common Core Grade 8 Answer Key Topic 6 Congruence And Similarity 110
From the given transversal,
We can observe that
59° and (x + 12)° are consecutive exterior angles
Now,
We know that,
The sum of the angles of consecutive exterior angles is: 180°
So,
59°+ (x + 12)° = 180°
x + 71° = 180°
x = 180° – 71°
x = 109°
Hence, from the above,
We can conclude that the value of x is: 109°

KEY CONCEPT

If parallel lines are intersected by a transversal, then
• Corresponding angles are congruent.
• Alternate interior angles are congruent.
• Same-side interior angles are supplementary.
Envision Math Common Core Grade 8 Answer Key Topic 6 Congruence And Similarity 111

Do You Understand?
Question 1.
Essential Question What are the relationships among angles that are created when a line intersects two parallel lines?
Answer:
If two parallel lines are cut by a third line, the third line is called the transversal.
So,
The relationships among angles that are created when a line intersects two parallel lines are :
(1) Corresponding angles
(2) Vertically Opposite angles
(3) Alternate interior angles
(4) Alternate exterior angles

Question 2.
When parallel lines are cut by a transversal, how can you use a translation to describe how angles are related?
Answer:
Let a and b be the parallel lines
Now,
The given transversal is:

Now,
When the parallel lines are cut by a transversal,
The relation between angles is:
Corresponding angles:
∠1 = ∠5, ∠2 = ∠6, ∠3 = ∠7, and ∠4 = ∠8
Alternate Interior angles:
∠4 = ∠6, ∠3 = ∠5
Alternate Exterior angles:
∠1 = ∠7, ∠2 = ∠8
Vertically Opposite Angles:
∠1 = ∠7, ∠2 = ∠8
Consecutive Interior angles:
∠4 + ∠5 = 180°, ∠3 + ∠6 = 180°
Consecutive Exterior angles:
∠1 + ∠8 = 180°, ∠7 + ∠2 = 180°

Question 3.
How many angles are created when two parallel lines are cut by a transversal? How many different angle measures are there?
Answer:
When two parallel lines are cut by a transversal, the number of angles created are: 8
Now,
The different angle measures are:
a. Corresponding angles
b. Alternate Interior angles
c. Alternate Exterior angles
d. Vertical angles
e. Consecutive interior angles
f. Consecutive exterior angles
g. Adjacent angles

Question 4.
Use Structure How can you use angle measures to tell whether two lines are parallel?
Answer:
The ways to prove two lines are parallel using angle measures are:
a. If corresponding angles are congruent.
b. If alternate interior angles are congruent.
c. If consecutive, or same side, interior angles are supplementary.
d. If alternate exterior angles are congruent.

Do You Know How?
In 5-7, use the figure below.
Envision Math Common Core Grade 8 Answer Key Topic 6 Congruence And Similarity 112

Question 5.
Which angles are congruent to ∠8?
Answer:
The given transversal is:
Envision Math Common Core Grade 8 Answer Key Topic 6 Congruence And Similarity 112
Now,
We know that,
The angle relationships that have equal angle measures are:
a. Corresponding angles b. Vertically opposite angles c. Alternate interior angles d. Alternate exterior angles
Hence, from the above,
We can conclude that
The angles that are congruent to ∠8 are: ∠2, ∠4, and ∠6

Question 6.
If m∠4 = 70°, what is m∠6? Explain.
Answer:
The given transversal is:
Envision Math Common Core Grade 8 Answer Key Topic 6 Congruence And Similarity 112
Now,
From the given transversal,
We can observe that
∠4 and ∠6 are alternate interior angles
We know that,
The alternate interior angles are congruent
Hence, from the above,
We can conclude that
∠4 = ∠6 = 70°

Question 7.
If m∠1 = 95°, write an equation that could be used to find the measure of ∠8. Find m∠8.
Answer:
The given transversal is:
Envision Math Common Core Grade 8 Answer Key Topic 6 Congruence And Similarity 112
Now,
From the given transversal,
We can observe that
∠1 and ∠8 are consecutive exterior angles
Now,
We know that,
The sum of the consecutive exterior angle measures is 180°
So,
∠1 + ∠8 = 180°
∠8 = 180° – ∠1
∠8 = 180° – 95°
∠8 = 85°
Hence, from the above,
We can conclude that the measure of ∠8 is: 85°

Question 8.
What must x equal if line a is parallel to line b?
Envision Math Common Core Grade 8 Answer Key Topic 6 Congruence And Similarity 113
Answer:
It is given that line a is parallel to line b
Now,
The given transversal is:
Envision Math Common Core Grade 8 Answer Key Topic 6 Congruence And Similarity 113
Now,
From the given transversal,
We can observe that
(2x + 35)° and 103° are the corresponding angles
We know that,
The corresponding angles are congruent
So,
(2x + 35)° = 103°
2x° = 103° – 35°
2x° = 68°
x° = \(\frac{68°}{2}\)
x° = 34°
Hence, from the above,
We can conclude that the value of x is: 34°

Practice & Problem Solving

Question 9.
If p || q, what is the value of u?
Envision Math Common Core Grade 8 Answer Key Topic 6 Congruence And Similarity 114
Answer:
The given transversal is:
Envision Math Common Core Grade 8 Answer Key Topic 6 Congruence And Similarity 114
From the given transversal,
We can observe that
u and 148° are vertically opposite angles
Now,
We know that.
The vertically opposite angles are always congruent
Hence, from the above,
We can conclude that the value of u is: 148°

Question 10.
Are ∠K and ∠B corresponding angles? Explain.
Envision Math Common Core Grade 8 Answer Key Topic 6 Congruence And Similarity 115
Answer:
The given transversal is:
Envision Math Common Core Grade 8 Answer Key Topic 6 Congruence And Similarity 115
Now,
We know that,
Corresponding angles are angles that are in the same position relative to lines intersected by a transversal
So,
From the given transversal,
We can observe that
∠B is internal and ∠K is external. They are equals if the two intersected lines by the transversal are parallel
Hence, from the above,
We can conclude that
∠K and ∠B are called “Corresponding angles” only when m || n

Question 11.
Streets A and B run parallel to each other. The measure of ∠6 is 155°. What is the measure of ∠4?
Envision Math Common Core Grade 8 Answer Key Topic 6 Congruence And Similarity 116
Answer:
It is given that
Streets A and B run parallel to each other. The measure of ∠6 is 155°
Now,
The given representation of streets A and B are:
Envision Math Common Core Grade 8 Answer Key Topic 6 Congruence And Similarity 116
Now,
From the given representation,
We can observe that
∠4 and ∠6 are consecutive interior angles
We know that,
The sum of consecutive interior angles is always equal to 180°
So,
∠4 + ∠6 = 180°
∠4 = 180° – ∠6
∠4 = 180° – 155°
∠4 = 25°
Hence, from the above,
We can conclude that the measure of ∠4 is: 25°

Question 12.
Reasoning The figure shows the design of a rectangular windowpane. The four horizontal lines are parallel. The measure of ∠6 is 53°. What is the measure of ∠12? Write and solve an equation to find the answer.
Envision Math Common Core Grade 8 Answer Key Topic 6 Congruence And Similarity 117
Answer:
It is given that
The figure shows the design of a rectangular windowpane. The four horizontal lines are parallel. The measure of ∠6 is 53°
Now,
The representation of the rectangular windowpane is:
Envision Math Common Core Grade 8 Answer Key Topic 6 Congruence And Similarity 117
Now,
From the given representation,
We can observe that
∠6 and ∠12 are consecutive interior angles
We know that,
The sum of consecutive interior angles is: 180°
so,
∠6 + ∠12 = 180°
∠12 = 180° – ∠6
∠12 = 180° – 53°
∠12 = 127°
Hence, from the above,
We can conclude that the angle measure of ∠12 is: 127°

Question 13.
In the figure, m || n. If m∠8 is (4x + 7)° and m∠2 is 107°, what is the value of x? Explain.
Envision Math Common Core Grade 8 Answer Key Topic 6 Congruence And Similarity 118
Answer:
It is given that
m || n and m∠8 is (4x + 7)° and m∠2 is 107°
Now,
The given transversal is:
Envision Math Common Core Grade 8 Answer Key Topic 6 Congruence And Similarity 118
From the given transversal,
We can observe that
∠2 and ∠8 are vertically opposite angles
We know that,
The vertically opposite angles are always congruent
So,
∠2 = ∠8
107° = (4x + 7)°
4x° = 107°- 7°
4x° = 100°
x° = \(\frac{100°}{4}\)
x° = 25°
Hence, from the above,
We can conclude that the value of x is: 25°

Question 14.
For the given figure, can you conclude m || n? Explain.
Envision Math Common Core Grade 8 Answer Key Topic 6 Congruence And Similarity 119
Answer:
The given figure is:
Envision Math Common Core Grade 8 Answer Key Topic 6 Congruence And Similarity 119
Now,
From the given figure,
We can observe that
74° and 74° are alternate interior angles
Now,
We know that,
The Converse of the Alternate Interior Angles Theorem states that if two lines are cut by a transversal and the alternate interior angles are congruent, then the lines are parallel
Hence, from the above,
We can conclude that m || n since the alternate interior angles are congruent

Question 15.
Line m is parallel to line n. Find the value of x and each missing angle measure.
Envision Math Common Core Grade 8 Answer Key Topic 6 Congruence And Similarity 120
Answer:
It is given that line m is parallel to n
Now,
The given transversal is:
Envision Math Common Core Grade 8 Answer Key Topic 6 Congruence And Similarity 120
From the given transversal,
We can observe that
(2x + 25)° and 86° are vertically opposite angles
We know that,
The vertically opposite angles are always congruent
So,
(2x + 25)° = 86°
2x° = 86°- 25°
2x°= 61°
x° = \(\frac{61°}{2}\)
x° = 30.5°
So,
The value of x is: 30.5°
Now,
We can observe that
∠1 and 86° are supplementary angles
∠1 and ∠3 are vertical angles
∠2 and ∠3 are supplementary angles
(2x + 25)° and ∠6 are supplementary angles
∠6 and ∠4 are vertical angles
∠4 and ∠5 are supplementary angles
So,
∠1 + 86° = 180°
∠1 = 180° – 86°
∠1 = 94°
∠1 = ∠3 = 94°
∠2 + ∠3 = 180°
∠2 = 180° – ∠3
∠2 = 180° – 94°
∠2 = 86°
Now,
(2x + 25)° + ∠6 = 180°
2 (30.5)° + ∠6 = 180° – 25°
∠6 = 155° – 61°
∠6 = 94°
∠ 6 = ∠4 = 94°
∠4 + ∠5 = 180°
∠5 = 180° – ∠4
∠5 = 180° – 94°
∠5 = 96°
Hence, from the above,
We can conclude that
The value of x is: 30.5°
The missing angle measures are:
∠1 = ∠3 = 94°; ∠4 = ∠6 = 94°; ∠2 = 86°; ∠5 = 96°

Question 16.
Higher Order Thinking
a. Find the value of x given that r || s.
Envision Math Common Core Grade 8 Answer Key Topic 6 Congruence And Similarity 121
m∠1 = (63 – x)°
m∠2 = (72 – 2x)°
Answer:
It is given that r || s
Now,
The given transversal is:
Envision Math Common Core Grade 8 Answer Key Topic 6 Congruence And Similarity 121
From the given transversal,
We can observe that
∠1 and ∠2 are corresponding angles
So,
∠1 = ∠2
So,
(63 – x)° = (72 – 2x)°
2x° – x°= 72° – 63°
x° = 9°
Hence, from the above,
We can conclude that the value of x is: 9°

b. Find m∠1 and m∠2.
Answer:
From part (a),
We can observe that the value of x is: 9°
So,
∠1 = (63 – x)°
∠1 = 63° – 9°
∠1 = 56°
∠2 = (72 – 2x)°
∠2 = 72° – 2 (9)°
∠2 = 72° – 18°
∠2 = 54°
Hence, from the above,
We can conclude that
The values of ∠1 and ∠2 are: 56° and 54° respectively

Question 17.
Find the measures of ∠b and ∠d given that m || n.
Envision Math Common Core Grade 8 Answer Key Topic 6 Congruence And Similarity 122
Answer:
It is given that m || n
Now,
The given transversal is:
Envision Math Common Core Grade 8 Answer Key Topic 6 Congruence And Similarity 122
From the given transversal,
We can observe that
∠b and 119.3° are supplementary angles
So,
∠b + 119.3° = 180°
∠b = 180° – 119.3°
∠b = 70.7°
Now,
∠d and 136.9° are supplementary angles
So,
∠d + 136.9° = 180°
∠d = 180° – 136.9°
∠d = 43.1°
Hence, from the above,
We can conclude that
The values of ∠b and ∠d are 70.7° and 43.1° respectively

Assessment Practice
Question 18.
In the figure, g || p. Which angles are alternate interior angles? Select all that apply.
Envision Math Common Core Grade 8 Answer Key Topic 6 Congruence And Similarity 123
☐ ∠q and ∠r
☐ ∠q and ∠t
☐ ∠q and ∠k
☐ ∠r and ∠t
☐ ∠r and ∠k
☐ ∠u and ∠9
Answer:
It is given that g || p
Now,
The given transversal is:
Envision Math Common Core Grade 8 Answer Key Topic 6 Congruence And Similarity 123
From the given transversal,
We can observe that
∠r and ∠k are alternate interior angles
∠q and ∠t are alternate interior angles
Hence, from the above,
We can conclude that
The alternate interior angles are:

Question 19.
In the figure, p || q. On a recent math test, Jacob incorrectly listed the value of w as 101.
Envision Math Common Core Grade 8 Answer Key Topic 6 Congruence And Similarity 124

PART A
Find the value of w.
Answer:
It is given that p || q
Now,
The given transversal is:
Envision Math Common Core Grade 8 Answer Key Topic 6 Congruence And Similarity 124
From the given transversal,
We can observe that
79° and ∠w are the corresponding angles
Hence, from the above,
We can conclude that the value of ∠w is: 79°

PART B
What mistake did Jacob likely make?
Answer:
We know that,
The corresponding angles are always congruent
Hence,
The mistake that Jacob likely made is:
Jacob considers ∠w  and 101° the corresponding angles

Lesson 6.9 Interior and Exterior Angles of Triangles

Solve & Discuss It!
Nell cuts tile to make a decorative strip for a kitchen backsplash. She must cut the tiles precisely to be congruent triangles. She plans to place the tiles between two pieces of molding, as shown. What is m∠2? Explain.
Envision Math Common Core Grade 8 Answer Key Topic 6 Congruence And Similarity 125
I can… find the interior and exterior angle measures of a triangle.
Answer:
It is given that
Nell cuts tile to make a decorative strip for a kitchen backsplash. She must cut the tiles precisely to be congruent triangles. She plans to place the tiles between two pieces of molding, as shown
Now,
The representation of the decorative strip for a kitchen backsplash is:

Now,
From the given representation,
We can observe that
∠1 of second congruent triangle = ∠1 of the first congruent triangle
∠3 of second congruent triangle = ∠3 of the third congruent triangle
Now,
We know that,
The sum of all the adjacent angles in a transversal is 180°
So,
∠1 + ∠2 + ∠3 = 180°
From the given representation,
∠1 = ∠3 = 65°
So,
∠2 = 180° – (65° + 65°)
∠2 = 180° – 130°
∠2 = 50°
Hence, from the above,
We can conclude that the value of ∠2 is: 50°

Model with Math
How can you use your knowledge of parallel lines and transversals to solve the problem?
Answer:
The representation of the above problem in the form of parallel lines and transversals is:

Now,
From the above figure,
We can observe that
According to the alternate interior angles theorem,
∠1 of second congruent triangle = ∠1 of the first congruent triangle
∠3 of second congruent triangle = ∠3 of the third congruent triangle
Now,
We know that,
The sum of all the adjacent angles in a transversal is 180°

Focus on math practices
Reasoning What assumption(s) did you need to make to find m∠2? Explain why your assumption(s) is reasonable.
Answer:
The assumptions you need to find ∠2 are:
a. ∠1 of the second congruent triangle = ∠1 of the first congruent triangle
b. ∠3 of the second congruent triangle = ∠3 of the third congruent triangle
c. The sum of all the adjacent angles in a transversal is 180°

Essential Question
How are the interior and exterior angles of a triangle related?
Answer:
An exterior angle of a triangle is equal to the sum of the two opposite interior angles. The sum of exterior angle and interior angle is equal to 180 degrees.

Try It!

Find the unknown angle measure in the triangle at the right.
Envision Math Common Core Grade 8 Answer Key Topic 6 Congruence And Similarity 126
Answer:
The given triangle is:
Envision Math Common Core Grade 8 Answer Key Topic 6 Congruence And Similarity 126
Now,
We know that,
The sum of all the interior angles in a triangle is: 180°
Now,
Let x° be the unknown angle measure
So,
68° + 40° + x° = 180°
x° = 180° – 108°
x° = 72°
Hence, from the above,
We can conclude that the value of the unknown angle measure is: 72°

Convince Me!
Could a triangle have interior angle measures of 23°, 71°, and 96°? Explain.
Answer:
The given interior angle measures of a triangle are 23°, 71°, and 96°
Now,
We know that,
The sum of all the interior angles in a triangle is: 180°
So,
23° + 71° + 96° = 180°
94° + 96° = 180°
190° ≠ 180°
Hence, from the above,
We can conclude that a triangle could not have given interior angle measures

Try It!

What is the measure of the exterior angle shown?
Envision Math Common Core Grade 8 Answer Key Topic 6 Congruence And Similarity 127
Answer:
The given triangle is:
Envision Math Common Core Grade 8 Answer Key Topic 6 Congruence And Similarity 127
Now,
From the given figure,
We can observe that
(7x – 1)° and (8x + 8)° are interior angle measures
16x° is an exterior angle measure
Now,
We know that,
An exterior angle of a triangle is equal to the sum of the two opposite interior angles. The sum of exterior angle and interior angle is equal to 180 degrees.
So,
16x° = (7x – 1)° + (8x + 8)°
16x° = 15x° + 7°
16x° – 15x° = 7°
x° = 7°
So,
16x° = 16 (7)°
= 112°
Hence, from the above,
We can conclude that the exterior angle measure of the given triangle is: 112°

KEY CONCEPT

The sum of the measures of the interior angles of a triangle is 180°.
m∠1 + m∠2 + m∠3 = 180°
Envision Math Common Core Grade 8 Answer Key Topic 6 Congruence And Similarity 128
The measure of an exterior angle of a triangle is equal to the sum of the measures of its remote interior angles.
m∠2 + m∠3 = m∠4

Do You Understand?
Question 1.
Essential Question How are the interior and exterior angles of a triangle related?
Answer:
An exterior angle of a triangle is equal to the sum of the two opposite interior angles. The sum of exterior angle and interior angle is equal to 180 degrees.

Question 2.
Reasoning Maggie draws a triangle with a right angle. The other two angles have equal measures. What are the possible values of the exterior angles for Maggie’s triangle? Explain.
Answer:
It is given that
Maggie draws a triangle with a right angle. The other two angles have equal measures
Now,
We know that,
The sum of the interior angles in a triangle is: 180°
Now,
Let the unknown two angle measures be x°
So,
90° + x° + x° = 180°
2x° = 180° – 90°
2x°= 90°
x° = \(\frac{90°}{2}\)
x° = 45°
So,
The two unknown angle measures are 45° and 45°
Now,
We know that,
An exterior angle of a triangle is equal to the sum of the two opposite interior angles. The sum of exterior angle and interior angle is equal to 180 degrees.
Now,
Let the exterior angle measure be y°
So,
y° = 45° + 45°
y° = 90°
Hence, from the above,
We can conclude that the possible value for the exterior angle measure of Maggie’s triangle is: 90°

Question 3.
Brian draws a triangle with interior angles of 32° and 87°, and one exterior angle of 93°. Draw the triangle. Label all of the interior angles and the exterior angle.
Answer:
It is given that
Brian draws a triangle with interior angles of 32° and 87°, and one exterior angle of 93°
Now,
We know that,
The sum of all the interior angles in a triangle is 180°
Now,
Let the unknown interior angle measure be: x°
So,
x°+ 32° + 87° = 180°
x° = 180° – 119°
x° = 61°
Now,
We know that,
An exterior angle of a triangle is equal to the sum of the two opposite interior angles. The sum of exterior angle and interior angle is equal to 180 degrees.
Now,
Let the unknown exterior angle measure be y°
So,
From the given interior angle measures,
y° = 61° + 32°
So,
In a triangle,
The interior angle measures are 61°, 32°, and 87°
The exterior angle measure is: 93°
Hence,
The representation of the interior angles and the exterior angle of a triangle is:

Do You Know How?
Use the diagram below for 4 and 5. Assume that a || b.
Envision Math Common Core Grade 8 Answer Key Topic 6 Congruence And Similarity 129

Question 4.
What are the measures of ∠1 and ∠2? Explain.
Answer:
It is given that a || b
Now,
The given transversal is:
Envision Math Common Core Grade 8 Answer Key Topic 6 Congruence And Similarity 129
From the given transversal,
We can observe that
∠1 and 37.3° are vertically opposite angles
So,
∠1 = 37.3°
Now,
We know that,
The sum of all the interior angles in a triangle is 180°
So,
∠1 + 79.4° + ∠2 = 180°
37.3° + 79.4° + ∠2 = 180°
∠2 = 180° – 116.7
∠2 = 63.3°
Hence, from the above,
We can conclude that the measures of ∠1 and ∠2 are 37.3° and 63.3° respectively

Question 5.
What are the measures of ∠3 and ∠4? Explain.
Answer:
It is given that a || b
Now,
The given transversal is:
Envision Math Common Core Grade 8 Answer Key Topic 6 Congruence And Similarity 129
Now,
From the given transversal,
We can observe that
∠3 and ∠4 are the exterior angles
Now,
∠3 = ∠2 + 79.4°
∠3 = 63.3° + 79.4°
∠3 = 132.7°
Now,
∠4 = ∠1 + 79.4°
∠4 = 37.3° + 79.4°
∠4 = 116.7°
Hence, from the above,
We can conclude that the values of ∠3 and ∠4 are: 132.7° and 116.7°

Question 6.
In △ABC, M∠A = x°, m∠B = (2x)°, and m∠C = (6x + 18)°. What is the measure of each angle?
Answer:
It is given that
In ΔABC,
The angle measures of all the interior angles are:
∠A = x°, ∠B = 2x°, and ∠C = (6x + 18)°
Now,
We know that,
The sum of all the interior angles in a triangle is 180°
So,
x° + 2x° + 6x° + 18° = 180°
9x°= 180° – 18°
9x° = 162°
x° = \(\frac{162°}{9}\)
x° = 18°
So,
∠A = 18°
∠B = 2 (18°) = 36°
∠C = (6x + 18)° = 6 (18°) + 18° = 126°
Hence, from the above,
We can conclude that the measure of each angle is: 18°, 36°, and 126°

Practice & Problem Solving

Question 7.
Leveled Practice For the figure shown, find m∠1.
Envision Math Common Core Grade 8 Answer Key Topic 6 Congruence And Similarity 130
Angle 1 is an _________ angle of the triangle.
m∠1 is equal to the sum of its ___________.
m∠1 = _______° + _______°
m∠1 = _______°
Answer:
The given triangle is:
Envision Math Common Core Grade 8 Answer Key Topic 6 Congruence And Similarity 130
Now,
From the given triangle,
We can observe that
∠1 is the exterior angle
59° and 56° are the interior angle measures
Now,
We know that,
An exterior angle of a triangle is equal to the sum of the two opposite interior angles. The sum of exterior angle and interior angle is equal to 180 degrees.
So,
∠1 = 59° + 56°
∠1 = 115°
Hence, from the above,
We can conclude that the value of ∠1 is: 115°

Question 8.
Find m∠1 and m∠2.
Envision Math Common Core Grade 8 Answer Key Topic 6 Congruence And Similarity 131
Answer:
The given triangle is:
Envision Math Common Core Grade 8 Answer Key Topic 6 Congruence And Similarity 131
From the given triangle,
We can observe that
There are 2 triangles and ∠1 is the exterior angle for the second triangle
Now,
We know that,
An exterior angle of a triangle is equal to the sum of the two opposite interior angles. The sum of exterior angle and interior angle is equal to 180 degrees.
So,
For the first triangle,
138° = ∠1 + 18°
∠1 = 138° – 18°
∠1 = 120°
For the second triangle,
∠1 = ∠2 + 85°
120° = ∠2 + 85°
∠2 = 120° – 85°
∠2 = 35°
Hence, from the above,
We can conclude that the values of ∠1 and ∠2 are: 120° and 35°

Question 9.
In △ABC, what is m∠c?
Envision Math Common Core Grade 8 Answer Key Topic 6 Congruence And Similarity 132
Answer:
The given triangle is:
Envision Math Common Core Grade 8 Answer Key Topic 6 Congruence And Similarity 132
Now,
We know that,
The sum of all the interior angles in a triangle is: 180°
So,
x° + (4x)° + (5x – 13)° = 180°
(10x – 13)° = 180°
10x° = 180° + 13°
10x° = 193°
x° = \(\frac{193°}{10}\)
x° = 19.3°
So,
∠C = (5x – 13)°
∠C = 5 (19.3)° – 13°
∠C = 96.5° – 13°
∠C = 83.5°
Hence, from the above,
We can conclude that the value of ∠C is: 83.5°

Question 10.
In the figure, m∠1 = (8x + 7)°, m∠2 = (4x + 14)°, and m∠4 = (13x + 12)°. Your friend incorrectly says that m∠4 = 51°. What is m∠4? What mistake might your friend have made?
Envision Math Common Core Grade 8 Answer Key Topic 6 Congruence And Similarity 133
Answer:
It is given that
In the given figure, m∠1 = (8x + 7)°, m∠2 = (4x + 14)°, and m∠4 = (13x + 12)°. Your friend incorrectly says that m∠4 = 51°
Now,
The given figure is:
Envision Math Common Core Grade 8 Answer Key Topic 6 Congruence And Similarity 133
From the given figure,
We can observe that
∠1, ∠2, and ∠3 are the interior angles
∠4 is an exterior angle
Now,
We know that,
An exterior angle of a triangle is equal to the sum of the two opposite interior angles. The sum of exterior angle and interior angle is equal to 180 degrees.
The sum of all the interior angles in a triangle is 180°
So,
∠1 + ∠2 + ∠3 = 180°
∠4 = ∠1 + ∠2
So,
(13x + 12)° = (8x + 7)° + (4x + 14)°
13x° + 12° = 12x° + 21°
13x° – 12x° = 21°- 12°
x°= 9°
So,
∠4 = (13x + 12)°
∠4 = 13 (9)° + 12°
∠4 = 117° + 12°
∠4 = 129°
Hence, from the above,
We can conclude that
The value of ∠4 is: 120°
The mistake mightyour friend has made is:
He considered ∠4 as an interior angle and he subtracted the value of ∠4 from 180°

Question 11.
What is m∠1?
Envision Math Common Core Grade 8 Answer Key Topic 6 Congruence And Similarity 134
Answer:
The given triangle is:
Envision Math Common Core Grade 8 Answer Key Topic 6 Congruence And Similarity 134
Now,
From the given triangle,
We can observe that
∠1 is an exterior angle
26° and 90° are interior angles
Now,
We know that,
An exterior angle of a triangle is equal to the sum of the two opposite interior angles. The sum of exterior angle and interior angle is equal to 180 degrees.
So,
∠1 = 26° + 90°
∠1 = 116°
Hence, from the above,
We can conclude that the value of ∠1 is: 116°

Question 12.
Higher Order Thinking Given that m∠1 = (16x)°, m∠2 = (8x + 21)°, and m∠4 = (25x + 19)°, what is an expression for m∠3? What is m∠3?
Envision Math Common Core Grade 8 Answer Key Topic 6 Congruence And Similarity 135
Answer:
It is given that
Given that m∠1 = (16x)°, m∠2 = (8x + 21)°, and m∠4 = (25x + 19)°, what is an expression for m∠3
Now,
The given triangle is:
Envision Math Common Core Grade 8 Answer Key Topic 6 Congruence And Similarity 135
From the given figure,
We can observe that
∠1, ∠2, and ∠3 are the interior angles
∠4 is an exterior angle
Now,
We know that,
An exterior angle of a triangle is equal to the sum of the two opposite interior angles. The sum of exterior angle and interior angle is equal to 180 degrees.
The sum of all the interior angles in a triangle is 180°
So,
∠1 + ∠2 + ∠3 = 180°
∠4 = ∠1 + ∠2
So,
(25x + 19)° = (16x)° + (8x + 21)°
25x° + 19° = 24x° + 21°
25x° – 24x° = 21°- 19°
x°= 2°
So,
∠3 = 180° – (16 (2)° + 8 (2)° + 21°)
∠3 = 180° – 69°
∠3 = 111°
Hence, from the above,
We can conclude that
The value of ∠3 is: 111°

Question 13.
A ramp attached to a building is being built to help with deliveries. The angle that the bottom of the ramp makes with the ground is 37.2°. Find the measure of the other acute angle.
Envision Math Common Core Grade 8 Answer Key Topic 6 Congruence And Similarity 136
Answer:
It is given that
A ramp attached to a building is being built to help with deliveries. The angle that the bottom of the ramp makes with the ground is 37.2°
Now,
The representation of the ramp is:
Envision Math Common Core Grade 8 Answer Key Topic 6 Congruence And Similarity 136
From the above,
We can observe that the ramp is in the form of a right triangle
Now,
We know that,
The sum of all the interior angles in a triangle is: 180°
So,
37.2° + 90° + x° = 180°
127.2° + x° = 180°
x°= 180° – 127.2°
x°= 62.8°
Hence, from the above,
We can conclude that the measure of the other acute angle is: 62.8°

Assessment Practice
Question 14.
The measure of ∠F is 110°. The measure of ∠E is 100°. What is the measure of ∠D?
Envision Math Common Core Grade 8 Answer Key Topic 6 Congruence And Similarity 137
A. 150°
B. 80°
C. 70°
D. 30°
Answer:
It is given that
The measure of ∠F is 110°. The measure of ∠E is 100°
Now,
The given transversal is:
Envision Math Common Core Grade 8 Answer Key Topic 6 Congruence And Similarity 137
From the given transversal,
We can observe that
∠A and ∠D are the vertical angles
Now,
We know that,
The sum of the adjacent angles is: 180°
So,
∠F + ∠C = 180°
∠B + ∠E = 180°
So,
∠C = 180° – ∠F
∠C = 180°- 110°
∠C = 70°
Now,
∠B = 180° – ∠E
∠B = 180° – 100°
∠B = 80°
Now,
We know that,
The sum of all the interior angles in a triangle is: 180°
So,
∠A + ∠B + ∠C = 180°
70° + 80°+ ∠A = 180°
∠A = 180° – 150°
∠A = 30°
So,
∠D = 30°
Hence, from the above,
We can conclude that the measure of ∠D is: 30°

Question 15.
In the figure, m∠1 = (3x + 12)°, m∠2 = (3x + 18)° and m∠3 = (7x + 10)°. What is m∠3 in degrees?
Envision Math Common Core Grade 8 Answer Key Topic 6 Congruence And Similarity 138
Answer:
It is given that
m∠1 = (3x + 12)°, m∠2 = (3x + 18)° and m∠3 = (7x + 10)°
Now,
The given triangle is:
Envision Math Common Core Grade 8 Answer Key Topic 6 Congruence And Similarity 138
From the given triangle,
We can observe that
∠1 and ∠2 are the interior angles
∠3 is an exterior angle
Now,
We know that,
An exterior angle of a triangle is equal to the sum of the two opposite interior angles. The sum of exterior angle and interior angle is equal to 180 degrees.
So,
∠3 = ∠1 + ∠2
(7x + 10)° = (3x + 12)° + (3x + 18)°
(7x + 10)° = (6x + 30)°
7x° – 6x° = 30° – 10°
x° = 20°
So,
∠3 = 7 (20)° + 10°
∠3 = 140° + 10°
∠3 = 150°
Hence, from the above,
We can conclude that the measure of ∠3 is 150°

Lesson 6.10 Angle-Angle Triangle Similarity

Explore It!
Justin made two flags for his model sailboat.
Envision Math Common Core Grade 8 Answer Key Topic 6 Congruence And Similarity 139
I can… use angle measures to determine whether two triangles are similar.

A. Draw and label triangles to represent each flag.
Answer:
It is given that
Justin made two flags for his model sailboat
Hence,
The representation of triangles that represent each flag is:

B. How are the side lengths of the triangles related?
Answer:
The representation of triangles that represent each flag is:

From the given triangles,
We can observe that,
After dilation,
Flag A and Flag B have different side lengths that are divided (or) multiplied by some value of scale factor and the sizes of Flag A and Flag B are different
Hence, from the above,
We can conclude that
After dilation transformation,
The side lengths of Flag A is greater than the side lengths of Flag B

C. How are the angle measurements of the triangles related?
Answer:
The representation of triangles that represent each flag is:

From the given triangles,
We can observe that the apex vertex of Flag A is 46° and the base vertices of Flag B are 67° and 67°
Now,
We know that,
The sum of all the angles in a triangle is 180°
So,
For Flag B,
Let the unknown angle measure be x°
So,
67° + 67° + x° = 180°
x° = 180° – 134°
x° = 46°
So,
For Flag B,
The apex vertex is: 46°
Now,
When we observe two triangles,
The apex vertex of Flag A and Flag B is the same
Hence, from the above,
We can conclude that the corresponding angle measurements for Flag A and Flag B are the same

Focus on math practices
Reasoning Justin makes a third flag that has sides that are shorter than the sides of the small flag. Two of the angles for each flag measure the same. Are the third angles for each flag the same measure? Explain.
Answer:
It is given that
Justin makes a third flag that has sides that are shorter than the sides of the small flag. Two of the angles for each flag measure the same
So,
Two of the angles for each flag are 67° and 67°
Since two angle measures are the same for all the flags, the remaining third angle measure will also be the same
Hence, from the above,
We can conclude that the third angles for each flag have the same measure

Essential Question
How can you use angle measures to determine whether two triangles are similar?
Answer:
The Angle-Angle (AA) Criterion states that if two angles in one triangle are congruent to two angles in another triangle, the two triangles are similar triangles.
Envision Math Common Core Grade 8 Answer Key Topic 6 Congruence And Similarity 143
∠A ≅ ∠D and B ≅ ∠E,
So,
△ABC – △DEF.

Try It!

Is △ΧΥΖ ~ △LMN?
Envision Math Common Core Grade 8 Answer Key Topic 6 Congruence And Similarity 140
m∠X = _______
m∠N = ________
The triangles ________ similar.
Answer:
The given triangles are:
Envision Math Common Core Grade 8 Answer Key Topic 6 Congruence And Similarity 140
Now,
For ΔXYZ:
We know that,
The sum of all the angles in a triangle is: 180°
So,
∠X + ∠Y + ∠Z = 180°
∠X = 180° – 92° – 42°
∠X = 180° – 134°
∠X = 46°
For ΔLMN:
We know that,
The sum of all the angles in a triangle is: 180°
So,
∠L + ∠M + ∠N = 180°
∠N = 180° – 92° – 53°
∠N = 180° – 148°
∠N = 32°
Now,
To find whether the given triangles are similar or not,
Find out the angle measures of the corresponding sides
So,
For ΔXYZ and ΔLMN,
∠X = ∠L, ∠Y = ∠M,  ∠Z = ∠N
But,
We can observe that
∠X ≠ ∠L
Hence, from the above,
We can conclude that
Since the angle measure of the corresponding sides are not the same,
ΔXYZ is not similar to ΔLMN

Convince Me!
Use what you know about transformations and parallel lines to explain why the Angle-Angle Criterion is true for all triangles.

Try It!

If QR || YZ, is △XYZ ~ △XRQ? Explain.
Envision Math Common Core Grade 8 Answer Key Topic 6 Congruence And Similarity 141
Answer:
It is given that QR || YZ
Now,
The given figure is:
Envision Math Common Core Grade 8 Answer Key Topic 6 Congruence And Similarity 141
Now,
We know that,
Since the lines are parallel, the angles that corresponding to the sides will also be the same
Hence,
According to the Side – Side criterion,
ΔXYZ is similar to ΔXRQ

Try It!

Find the value of x if the two triangles are similar. Explain.
Envision Math Common Core Grade 8 Answer Key Topic 6 Congruence And Similarity 142
Answer:
The given triangles are:
Envision Math Common Core Grade 8 Answer Key Topic 6 Congruence And Similarity 142
It is given that two triangles are similar
So,
According to Angle – Angle criterion,
(15x)° = 90°
So,
x° = \(\frac{90°}{15}\)
x° = 6°
Hence, from the above,
We can conclude that the value of x is: 6°

KEY CONCEPT

The Angle-Angle (AA) Criterion states that if two angles in one triangle are congruent to two angles in another triangle, the two triangles are similar triangles.
Envision Math Common Core Grade 8 Answer Key Topic 6 Congruence And Similarity 143
∠A ≅ ∠D and B ≅ ∠E, so △ABC – △DEF.

Do You Understand?
Question 1.
Essential Question How can you use angle measures to determine whether two triangles are similar?
Answer:
The Angle-Angle (AA) Criterion states that if two angles in one triangle are congruent to two angles in another triangle, the two triangles are similar triangles.
Envision Math Common Core Grade 8 Answer Key Topic 6 Congruence And Similarity 143
∠A ≅ ∠D and B ≅ ∠E,
So,
△ABC – △DEF.

Question 2.
Construct Arguments Claire says that the AA Criterion should be called the AAA Criterion. Explain why Claire might say this. Do you agree? Explain.
Answer:
It is given that
Claire says that the AA Criterion should be called the AAA Criterion.
Now,
We know that
AA criterion tells us that two triangles are similar if two corresponding angles are equal to each other
Now,
AAA criterion may be reformulated as the AAA (angle-angle-angle) similarity theorem
According to the AAA criterion,
Two triangles have their corresponding angles equal if and only if their corresponding sides are proportional.
So,
From the above two criterions’ statements,
We can observe that the definition of both the criteria are the same
Hence, from the above,
We can conclude that you can agree with Clara

Question 3.
Reasoning Which triangle pairs below are always similar? Explain.
Two right triangles
Two isosceles right triangles
Two equilateral triangles
Answer:
The given pairs of triangles are:
a. Two right triangles
b. Two isosceles right triangles
c. Two equilateral triangles
Now,
We know that,
Two triangles are said to be similar if their corresponding angles are congruent and the corresponding sides are in proportion
Hence, from the above,
We can conclude that
The triangle pairs that are similar are:
a. Two isosceles right triangles
b. Two equilateral triangles

Do You Know How?
Question 4.
Are the two triangles similar? Explain.
Envision Math Common Core Grade 8 Answer Key Topic 6 Congruence And Similarity 144
Answer:
The given triangles are:
Envision Math Common Core Grade 8 Answer Key Topic 6 Congruence And Similarity 144
From the given triangles,
We can observe that the apex angle measure in the first triangle is 44° and the apex angle measure in the second triangle is 46°
Now,
We know that
AA criterion tells us that two triangles are similar if two corresponding angles are equal to each other
So,
44° ≠ 46°
Hence, from the above,
We can conclude that the two triangles are not similar

Question 5.
Is △QRS ~ △QLM? Explain.
Envision Math Common Core Grade 8 Answer Key Topic 6 Congruence And Similarity 145
Answer:
The given triangles are:
Envision Math Common Core Grade 8 Answer Key Topic 6 Congruence And Similarity 145
Now,
From the given triangles,
We can observe that
∠R from ΔQRS = ∠L from ΔQLM
Now,
We know that
AA criterion tells us that two triangles are similar if two corresponding angles are equal to each other
So,
90° = 90°
Hence, from the above,
We can conclude that ΔQRS is similar to ΔQLM

Question 6.
Are the triangles similar? What is the value of x?
Envision Math Common Core Grade 8 Answer Key Topic 6 Congruence And Similarity 146
Answer:
The given triangles are:
Envision Math Common Core Grade 8 Answer Key Topic 6 Congruence And Similarity 146
Now,
From the given triangles,
We can observe that
The apex angle of the smaller triangle and the apex angle of the larger triangle is the same
Now,
We know that,
AA criterion tells us that two triangles are similar if two corresponding angles are equal to each other
So,
The given two triangles are similar
Now,
According to AAA criterion,
(4x)° and 76° are alternate angles
Now,
We know that,
The alternate angles are always congruent
So,
(4x)° = 76°
x° = \(\frac{76°}{4}\)
x° = 19°
Hence, from the above,
We can conclude that the value of x is: 19°

Practice & Problem Solving

Question 7.
Is △XYZ ~ △XTU?
Envision Math Common Core Grade 8 Answer Key Topic 6 Congruence And Similarity 147
Answer:
The given triangles are:
Envision Math Common Core Grade 8 Answer Key Topic 6 Congruence And Similarity 147
Now,
We know that,
The sum of all the angles in a triangle is: 180°
So,
In ΔXTU,
103° + ∠T + 48° = 180°
∠T = 180° – 151°
∠T = 29°
Now,
∠T in ΔXTU and ∠Y in ΔXYZ are not the same
Now,
We know that,
AA criterion tells us that two triangles are similar if two corresponding angles are equal to each other
Hence, from the above,
We can conclude that
ΔXTU and ΔXYZ are not similar

Question 8.
For what value of x are △RST and △NSP similar? Explain.
Envision Math Common Core Grade 8 Answer Key Topic 6 Congruence And Similarity 148
Answer:
The given triangles are:
Envision Math Common Core Grade 8 Answer Key Topic 6 Congruence And Similarity 148
Now,
For the given triangles to be equal,
We know that,
AA criterion tells us that two triangles are similar if two corresponding angles are equal to each other
So,
∠T in ΔRST = ∠P in ΔNSP
So,
(x + 19)° = (2x)°
2x° – x°= 19°
x° = 19°
Hence, from the above,
We can conclude that the value of x so that ΔRSt and ΔNSP are similar is: 19°

Question 9.
Is △FGH ~ △JIH? Explain.
Envision Math Common Core Grade 8 Answer Key Topic 6 Congruence And Similarity 149
Answer:
The given triangles are:
Envision Math Common Core Grade 8 Answer Key Topic 6 Congruence And Similarity 149
Now,
We know that,
The sum of all the angles in a triangle is equal to 180°
So,
In ΔJIH,
∠J + ∠I + ∠H = 180°
∠J = 180° – (43° + 35°)
∠J = 180° – 78°
∠J = 102°
So,
From the given triangles,
We can observe that
∠J in ΔJIH and ∠F in ΔFGH are not the same
Now,
We know that,
AA criterion tells us that two triangles are similar if two corresponding angles are equal to each other
Hence, from the above,
We can conclude that ΔRST and ΔJIH are not similar

Question 10.
Are △RST and △NSP similar? Explain.
Envision Math Common Core Grade 8 Answer Key Topic 6 Congruence And Similarity 150
Answer:
The given triangles are:
Envision Math Common Core Grade 8 Answer Key Topic 6 Congruence And Similarity 150
Now,
For the given triangles to be equal,
We know that,
AA criterion tells us that two triangles are similar if two corresponding angles are equal to each other
So,
∠T in ΔRST = ∠P in ΔNSP
So,
(x + 15)° = (2x)°
2x° – x°= 15°
x° = 15°
Hence, from the above,
We can conclude that the value of x so that ΔRSt and ΔNSP are similar is: 15°

Question 11.
Contruct Arguments Describe how to use angle relationships to decide whether any two triangles are similar.
Answer:
According to AA criterion,
If two pairs of corresponding angles in a pair of triangles are congruent, then the triangles are similar. We know this because if two angle pairs are the same, then the third pair must also be equal. When the three angle pairs are all equal, the three pairs of sides must also be in proportion.

Question 12.
Higher Order Thinking Are the triangles shown below similar? Explain.
Envision Math Common Core Grade 8 Answer Key Topic 6 Congruence And Similarity 151
Answer:
The given triangles are:
Envision Math Common Core Grade 8 Answer Key Topic 6 Congruence And Similarity 151
Now,
From the given triangles,
We can observe that
The angle measures for the smaller triangle can be obtained by dividing the angle measures of the larger triangle by 3
But,
We know that,
The angle measures must be equal for both the triangles with the proportionate side lengths
Now,
We know that,
AA criterion tells us that two triangles are similar if two corresponding angles are equal to each other
Hence, from the above,
We can conclude that the given triangles are not similar

Assessment Practice
Question 13.
Which of the following statements are true? Select all that apply.
Envision Math Common Core Grade 8 Answer Key Topic 6 Congruence And Similarity 152
☐ △XYZ ~ △SQR
☐ △XYZ ~ △QSR
☐ △XYZ ~ △GHI
☐ △GIH ~ △SRQ
☐ △ZXY ~ △GIH
☐ △GHI ~ △SRQ
Answer:
The given triangles are:
Envision Math Common Core Grade 8 Answer Key Topic 6 Congruence And Similarity 152
Now,
From the given triangles,
We can observe that
∠I in ΔGIH or ΔHIG is equal to ∠R in ΔQRS or ΔSRQ
∠G in ΔGIH or ΔGHI is equal to ∠X in ΔXYZ or ΔXZY
Now,
We know that,
AA criterion tells us that two triangles are similar if two corresponding angles are equal to each other
Hence, from the above,
We can conclude that
a. △GIH ~ △SRQ
b. △XYZ ~ △GHI

Question 14.
Is △GHI ~ △QRS? Explain your reasoning.
Envision Math Common Core Grade 8 Answer Key Topic 6 Congruence And Similarity 153
Answer:
The given triangles are:
Envision Math Common Core Grade 8 Answer Key Topic 6 Congruence And Similarity 153
Now,
From the given triangles,
We can observe that
∠G in ΔGHi and ∠Q in ΔQRS are the same
Now,
We know that,
AA criterion tells us that two triangles are similar if two corresponding angles are equal to each other
Hence, from the above,
We can conclude that ΔGHI and ΔQRS are similar

Topic 6 REVIEW

Topic Essential Question
How can you show that two figures are either congruent or similar to one another?
Answer:
When the two figures are congruent,
a. The shapes and sizes of the two figures are the same
b. The side lengths and the angle measures are the same
c. The orientation is not the same
When the two figures are similar,
a. The shapes and orientations are the same
b. The sizes may are not the same
c. The angle measures are the same but the side lengths are different

Vocabulary Review
Complete each sentence by matching each vocabulary word to its definition. Assume pairs of lines are parallel.
Envision Math Common Core Grade 8 Answer Key Topic 6 Congruence And Similarity 154
Answer:

Use Vocabulary in Writing
Describe a way to show that △ABC is congruent to △DEF Use vocabulary terms from this Topic in your description.
Envision Math Common Core Grade 8 Answer Key Topic 6 Congruence And Similarity 155
Answer:
The given coordinate plane is:
Envision Math Common Core Grade 8 Answer Key Topic 6 Congruence And Similarity 155
From the given coordinate plane,
The vertices of ΔABC are:
A (3, 5), B (6, 2), and C (2, 2)
The vertices of ΔDEF are:
D (-2, -1), E (-5, -4), and F (-1, -4)
Now,
The sequence of transformations to show ΔABC is congruent to ΔDEF is:
Step 1:
Reflect the vertices of ΔABC across the y-axis
Step 2:
Translate the image we obtained in step 1 by 1 unit right and 6 units down so that we can obtain the vertices of ΔDEF
Hence,
The representation of the sequence of transformations to show ΔABC is congruent to ΔDEF is:

Concepts and Skills Review

Lesson 6.1 Analyze Translations

Quick Review
A translation is a transformation that maps each point of the preimage the same distance and in the same direction.

Example
Translate △XYZ 5 units right and 3 units up.
Answer:
Envision Math Common Core Grade 8 Answer Key Topic 6 Congruence And Similarity 156

Practice
Question 1.
Draw the image after a translation 3 units left and 2 units up.
Envision Math Common Core Grade 8 Answer Key Topic 6 Congruence And Similarity 157
Answer:
The given coordinate plane is:
Envision Math Common Core Grade 8 Answer Key Topic 6 Congruence And Similarity 157
Now,
From the given coordinate plane,
The vertices of the preimage are:
(-1, -3), (0.5, 2), (3.5, 2), and (5, -3)
Now,
After the translation of 3 units left and 2 units up,
The vertices of the image will become:
(-1 – 3, -3 + 2), (0.5 – 3, 2 + 2), (3.5 – 3, 2 + 2), and (5 – 3, -3 + 2)
(-4, -1), (-2.5, 4), (0.5, 4), and (2, -1)
Hence,
The representation of the preimage and the image is:

Lesson 6.2 Analyze Reflections

Quick Review
Reflected figures are the same distance from the line of reflection but on opposite sides.

Example
What are the coordinates of the image of △ABC after a reflection across the y-axis?
Envision Math Common Core Grade 8 Answer Key Topic 6 Congruence And Similarity 158
Answer:
Use the rule (x, y) → (-x, y).
A (-4, 1) → A'(4, 1)
B (-1, 1) → B'(1, 1)
C (-1, 5) → C'(1, 5)

Practice
Use the figure.
Envision Math Common Core Grade 8 Answer Key Topic 6 Congruence And Similarity 159
Question 1.
What are the coordinates of the image of rectangle WXYZ after a reflection across the X-axis?
Answer:
The given coordinate plane is:
Envision Math Common Core Grade 8 Answer Key Topic 6 Congruence And Similarity 159
From the given coordinate plane,
The vertices of the rectangle WXYZ are:
W (-4, -2), X (-1, -2), Y (-1, -4), and Z (-4, -4)
Now,
We know that,
If a point reflects across the x-axis,
The x-coordinate will be the same but the y-coordinate will change the sign
So,
(x, y) before reflection —– > (x, -y) after reflection
Hence,
The vertices of the rectangle WXYZ after reflection across the x-axis will become:
W’ (-4, 2), X’ (-1, 2), Y’ (-1, 4), and Z’ (-4, 4)
Hence,
The representation of the rectangle WXYZ (Preimage) and the rectangle W’X’Y’Z’ (Image) is:

Question 2.
What are the coordinates of the image of WXYZ after a reflection across the y-axis?
Answer:
We know that,
The vertices of the rectangle WXYZ are:
W (-4, -2), X (-1, -2), Y (-1, -4), and Z (-4, -4)
Now,
We know that,
If a point reflects across the y-axis,
The y-coordinate will be the same but the x-coordinate will change the sign
So,
(x, y) before reflection —– > (-x, y) after reflection
Hence,
The vertices of the rectangle WXYZ after reflection across the y-axis will become:
W’ (4, -2), X’ (1, -2), Y’ (1, -4), and Z’ (4, -4)
Hence,
The representation of the rectangle WXYZ (Preimage) and the rectangle W’X’Y’Z’ (Image) is:

Lesson 6.3 Analyze Rotations

Quick Review
A rotation turns a figure about a fixed point, called the center of rotation. The angle of rotation is the number of degrees the figure is rotated.

Example
What are the coordinates of the image of △ABC after a 90° rotation about the origin?
Envision Math Common Core Grade 8 Answer Key Topic 6 Congruence And Similarity 160
Answer:
Use the rule (x, y) → (-y, x).
A(1, 4) → A'(-4, 1)
B(4, 4) → B'(-4, 4)
C(4, 1) → C'(-1, 4)

Practice
Use the figure.
Envision Math Common Core Grade 8 Answer Key Topic 6 Congruence And Similarity 161
Question 1.
What are the coordinates of the image of quadrilateral STUV after a 180° rotation about the origin?
Answer:
The given coordinate plane is:
Envision Math Common Core Grade 8 Answer Key Topic 6 Congruence And Similarity 161
Now,
From the given coordinate plane,
The vertices of the quadrilateral STUV are:
S (-4, -2), T (-2, -2), U (-2, -4), and V (-4, -4)
Now,
We know that,
When any point rotates 180° counterclockwise about the origin,
(x, y) before rotating 180° —– > (-x, -y) after rotating 180°
So,
The vertices of the quadrilateral STUV after the rotation of 180° are:
S’ (4, 2), T (2, 2), U (2, 4), and V (4, 4)
Hence,
The representation of the quadrilateral STUV after the rotation of 180° is:

Question 2.
What are the coordinates of the image of quadrilateral STUV after a 270° rotation about the origin?
Answer:
The given coordinate plane is:
Envision Math Common Core Grade 8 Answer Key Topic 6 Congruence And Similarity 161
Now,
From the given coordinate plane,
The vertices of the quadrilateral STUV are:
S (-4, -2), T (-2, -2), U (-2, -4), and V (-4, -4)
Now,
We know that,
When any point rotates 270° counterclockwise about the origin,
(x, y) before rotating 270° —– > (y, -x) after rotating 270°
So,
The vertices of the quadrilateral STUV after the rotation of 270° are:
S’ (-2, 4), T (-2, 2), U (-4, 2), and V (-4, 4)
Hence,
The representation of the quadrilateral STUV after the rotation of 180° is:

Lesson 6.4 Compose Transformations

Quick Review
To compose a sequence of transformations, perform one transformation, and then use the resulting image to perform the next transformation.

Example
How can you use a sequence of transformations to map Figure A onto Figure B?
Envision Math Common Core Grade 8 Answer Key Topic 6 Congruence And Similarity 162
Answer:
Translate Figure A 3 units up, and then reflect Figure A across the y-axis.

Practice
Question 1.
Translate rectangle ABCD 5 units down, and then reflect it across the y-axis.
Envision Math Common Core Grade 8 Answer Key Topic 6 Congruence And Similarity 163
Answer:
The given coordinate plane is:
Envision Math Common Core Grade 8 Answer Key Topic 6 Congruence And Similarity 163
Now,
From the given coordinate plane,
The vertices of the rectangle ABCD are:
A (-5, 4), B (-1, 4), C (-1, 2), and D (-5, 2)
Now,
The sequence of transformations to draw the image of the rectangle ABCD is:
Step 1:
Translate the rectangle ABCD 5 units down
So,
The vertices of the rectangle ABCD are:
A (-5, 4 – 5), B (-1, 4 – 5), C (-1, 2 – 5), and D (-5, 2 -5)
A (-5, -1), B (-1, -1), C (-1, -3), and D (-5, -3)
Step 2:
Reflect the vertices that we obtained in step 1 across the y-axis
Now,
We know that,
If a point reflects across the y-axis,
The y-coordinate will be the same but the x-coordinate will change the sign
So,
(x, y) before reflection —– > (-x, y) after reflection
So,
The vertices of the rectangle ABCD are:
A (5, -1), B (1, -1), C (1, -3), and D (5, -3)
Hence,
The representation of the sequence of transformations to represent the rectangle ABCD and its image is:

Lesson 6.5 Understand Congruent Figures

Quick Review
Two figures are congruent if a sequence of transformations maps one figure onto the other.

Example
How can you determine if Figure A is congruent to Figure B?
Envision Math Common Core Grade 8 Answer Key Topic 6 Congruence And Similarity 164
Answer:
Reflect Figure A across the y-axis, and then translate Figure A 6 units up and 1 unit left.

Practice
Question 1.
Is quadrilateral A congruent to quadrilateral B? Explain.
Envision Math Common Core Grade 8 Answer Key Topic 6 Congruence And Similarity 165
Answer:
The given coordinate plane is:
Envision Math Common Core Grade 8 Answer Key Topic 6 Congruence And Similarity 165
Now,
From the given coordinate plane,
The vertices of quadrilateral A are:
(1, 1), (2, 4), (4, 4), and (5, 1)
The vertices of quadrilateral B are:
(-5, -4), (-4, -1), (-3, -1), and (-2, -4)
Now,
We know that,
The two figures are said to be congruent when
a. The shapes and sizes of the figures must be the same
b. The side lengths of the 2 figures must be the same
c. The angle measures must be the same
So,
The representation of the quadrilateral A and the quadrilateral B is:

Hence, from the above,
We can conclude that quadrilateral A and quadrilateral B are not congruent side their side lengths are not the same

Lesson 6.6 Describe Dilations

Quick Review
A dilation results in an image that is the same shape but not the same size as the preimage.

Example
What dilation maps WXYZ to W’X’Y’Z?
Envision Math Common Core Grade 8 Answer Key Topic 6 Congruence And Similarity 166
Answer:
A dilation with center at the origin and a scale factor of 2 maps WXYZ to W’X’Y’Z’.

Practice
Use the figure.
Envision Math Common Core Grade 8 Answer Key Topic 6 Congruence And Similarity 167
Question 1.
What are the coordinates of the image of parallelogram ABCD after a dilation with center (0, 0) and a scale factor of 3?
Answer:
The given coordinate plane is:
Envision Math Common Core Grade 8 Answer Key Topic 6 Congruence And Similarity 167
Now,
From the given coordinate plane,
The vertices of the parallelogram ABCD are:
A (-4, 1), B (0, 1), C (1, -1), and D (-3, -1)
Now,
After a dilation with center (0, 0) and a scale factor of 3,
The vertices of the parallelogram must be multiplied with 3
So,
A (-4, 1) × 3, B (0, 1) × 3, C (1, -1) × 3, and D (-3, -1) × 3
A’ (-12, 3), B’ (0, 3), C’ (3, -3), and D’ (-9, -3)
Hence, from the above,
We can conclude that the coordinates for the image of the parallelogram ABCD after a dilation with center (0, 0) and a scale factor of 3 is:
A’ (-12, 3), B’ (0, 3), C’ (3, -3), and D’ (-9, -3)

Question 2.
What are the coordinates of the image of parallelogram ABCD after a dilation with center (0, 0) and a scale factor of \(\frac{1}{2}\)?
Answer:
The given coordinate plane is:
Envision Math Common Core Grade 8 Answer Key Topic 6 Congruence And Similarity 167
Now,
From the given coordinate plane,
The vertices of the parallelogram ABCD are:
A (-4, 1), B (0, 1), C (1, -1), and D (-3, -1)
Now,
After a dilation with center (0, 0) and a scale factor of \(\frac{1}{2}\),
The vertices of the parallelogram must be multiplied with \(\frac{1}{2}\)
So,
A (-4, 1) × \(\frac{1}{2}\), B (0, 1) × \(\frac{1}{2}\), C (1, -1) × \(\frac{1}{2}\), and D (-3, -1) × \(\frac{1}{2}\)
A’ (-2, 0.5), B’ (0, 0.5), C’ (0.5, -0.5), and D’ (-1.5, -0.5)
Hence, from the above,
We can conclude that the coordinates for the image of the parallelogram ABCD after a dilation with center (0, 0) and a scale factor of \(\frac{1}{2}\) is:
A’ (-2, 0.5), B’ (0, 0.5), C’ (0.5, -0.5), and D’ (-1.5, -0.5)

Lesson 6.7 Understand Similar Figures

Quick Review
Two-dimensional figures are similar if there is a sequence of translations, reflections, rotations, and dilations that maps one figure onto the other figure. Similar figures have the same shape, congruent angles, and proportional side lengths.

Example
Is rectangle ABCD ~ rectangle A’B’C’D?
Envision Math Common Core Grade 8 Answer Key Topic 6 Congruence And Similarity 168
Answer:
All the angles are right angles.
\(\frac{A B}{A^{\prime} B^{\prime}}=\frac{B C}{B^{\prime} C^{\prime}}=\frac{C D}{C^{\prime} D^{\prime}}=\frac{A D}{A^{\prime} D^{\prime}}=\frac{2}{1}\) = 2
The figures have congruent angle measures and proportional side lengths, so they are similar.

Practice
Use the figure.
Envision Math Common Core Grade 8 Answer Key Topic 6 Congruence And Similarity 169
Question 1.
Is △ABC similar to △A’B’C’ ? Explain.
Answer:
The given coordinate plane is:
Envision Math Common Core Grade 8 Answer Key Topic 6 Congruence And Similarity 169
Now,
From the given coordinate plane,
The vertices of ΔABC are:
A (-4, 1) B (-4, 5), and C (-1, 1)
The vertices of ΔA’B’C’ are:
A’ (8, 2), B’ (8, 10), and C’ (2, 2)
Now,
The sequence of transformations to find whether △ABC is similar to △A’B’C’ or not is:
Step 1:
Reflect the vertices of △ABC across the y-axis
Step 2:
Dilate the vertices of △ABC we obtained in step 1 with center (0, 0) and a scale factor of 2
Hence,
The representation of the sequence of transformations to represent that △ABC is similar to △A’B’C’ is:

Question 2.
What sequence of transformations shows that △ABC is similar to △A’B’C’?
Answer:
The sequence of transformations that shows △ABC is similar to △A’B’C’ is:
Step 1:
Reflect the vertices of △ABC across the y-axis
Step 2:
Dilate the vertices of △ABC we obtained in step 1 with center (0, 0) and a scale factor of 2

Lesson 6.8 Angles, Lines, and Transversals

Quick Review
When parallel lines are intersected by a transversal, corresponding angles are congruent, alternate interior angles are congruent, and same-side interior angles are supplementary.

Example
If m || n, what is the value of x?
Envision Math Common Core Grade 8 Answer Key Topic 6 Congruence And Similarity 170
Answer:
m∠3 = 45° (5x + 25)°
45+ (5x + 25) = 180 – n
x = 22

Practice
In the figure, a || b. What is the value of x?
Envision Math Common Core Grade 8 Answer Key Topic 6 Congruence And Similarity 171
Answer:
It is given that a || b
Now,
The given transversal is:
Envision Math Common Core Grade 8 Answer Key Topic 6 Congruence And Similarity 171
Now,
From the given transversal,
We can observe that
(3x + 9)° and 129° are alternate angles
Now,
We know that,
The alternate angles are always congruent
So,
(3x + 9)° = 129°
3x° = 129° – 9°
3x° = 120°
x° = \(\frac{120°}{3}\)
x°= 40°
Hence, from the above,
We can conclude that the value of x is: 40°

Lesson 6.9 Interior and Exterior Angles of Triangles

Quick Review
The sum of the measures of the interior angles of a triangle is 180°. The measure of an exterior angle of a triangle is equal to the sum of the measures of its remote interior angles.

Example
Find the missing angle measure.
Envision Math Common Core Grade 8 Answer Key Topic 6 Congruence And Similarity 172
Answer:
x + 40 = 100, so x = 60

Practice
Question 1.
Find the missing angle measure.
Envision Math Common Core Grade 8 Answer Key Topic 6 Congruence And Similarity 173
Answer:
The given triangle is:
Envision Math Common Core Grade 8 Answer Key Topic 6 Congruence And Similarity 173
Now,
From the given triangle,
The angle measures are 48°, 102°
Let the unknown angle measure be x°
Now,
We know that,
The sum of all the angles in a given triangle is equal to 180°
So,
48°+ 102° + x° = 180°
x° = 180° – 150°
x° = 30°
Hence, from the above,
We can conclude that the missing angle measure is: 30°

Question 2.
Find the value of x.
Envision Math Common Core Grade 8 Answer Key Topic 6 Congruence And Similarity 174
Answer:
The given triangle is:
Envision Math Common Core Grade 8 Answer Key Topic 6 Congruence And Similarity 174
Now,
We know that,
The angle measure of an exterior angle is equal to the sum of the remote interior angles that is opposite to the given exterior angle
So,
3x° + 2x° = 115°
5x° = 115°
x° = \(\frac{115°}{3}\)
x° = 23°
Hence, from the above,
We can conclude that the value of x is: 23°

Lesson 6-10 Angle-Angle Triangle Similarity

Quick Review
By the AA Criterion, if two angles in one triangle are congruent to two angles in another triangle, then the triangles are similar.

Example
Is △ABC ~ △DEF? Explain.
Envision Math Common Core Grade 8 Answer Key Topic 6 Congruence And Similarity 175
Answer:
m∠B = 180° – 90° – 37° = 53°
m∠A = m∠D = 90° and m∠B = m∠E = 53°
Because two angles of the triangles are congruent, the triangles are similar by the AA Criterion.

Practice
Question 1.
AB || XY. Is △ABC ~ △XYC? Explain.
Envision Math Common Core Grade 8 Answer Key Topic 6 Congruence And Similarity 176
Answer:
It is given that AB || XY
Now,
The given triangles are:
Envision Math Common Core Grade 8 Answer Key Topic 6 Congruence And Similarity 176
Now,
From the given triangles,
We can observe that
∠A in ΔABC = ∠X in ΔXYC
Now,
We know that,
AA criterion tells us that two triangles are similar if two corresponding angles are equal to each other
Hence, from the above,
We can conclude that ΔABC is similar to ΔXYC

Question 2.
Find the values of x and y given that △ABC is similar to △MNC.
Envision Math Common Core Grade 8 Answer Key Topic 6 Congruence And Similarity 177
Answer:
It is given that ΔABC is similar to ΔMNC
Now,
The given triangles are:
Envision Math Common Core Grade 8 Answer Key Topic 6 Congruence And Similarity 177
Now,
We know that,
The sum of adjacent angles is equal to 180°
So,
4x° + 2x° = 180°
6x° = 180°
x° = \(\frac{180°}{6}\)
x° = 30°
Now,
Since ∠N = 60°,
∠A = 60° (By AA criterion)
Now,
We know that,
The sum of all the interior angles in a triangle is equal to 180°
So,
5y° + 60° + 30° = 180°
5y° = 180° – 90°
5y° = 90°
y° = \(\frac{90°}{5}\)
y° = 18°
Hence, from the above,
We can conclude that the values of x and y are: 30° and 18° respectively

Topic 6 Fluency Practice

Crisscrossed
Solve each equation. Write your answers in the cross-number puzzle below. Each digit, negative sign, and decimal point of your answer goes in its own box.
Envision Math Common Core Grade 8 Answer Key Topic 6 Congruence And Similarity 178
I can… solve multistep equations. © 8.EE.C.7b
Envision Math Common Core Grade 8 Answer Key Topic 6 Congruence And Similarity 179

enVision Math Common Core Grade 8 Answer Key Topic 3 Use Functions To Model Relationships

Practice with the help of enVision Math Common Core Grade 8 Answer Key Topic 3 Use Functions to Model Relationships regularly and improve your accuracy in solving questions.

enVision Math Common Core 8th Grade Answers Key Topic 3 Use Functions To Model Relationships

Topic 3 GET READY!

Review What You Know!

Vocabulary
Choose the best term from the box to complete each definition.

Question 1.
The ____ is the ratio of the vertical change to the horizontal change of a line.
Answer:
We know that,
The “Slope” is the ratio of the vertical change to the horizontal change of a line
Hence, from the above,
We can conclude that the best term from the box to complete the given definition is “Slope”

Question 2.
A relationship that can be modeled by the equation y = mx is a ___
Answer:
We know that,
A relationship that can be modeled by the equation y = mx is a “Proportional relationship”
Hence, from the above,
We can conclude that the best term from the box to complete the given definition is “Proportional relationship”

Question 3.
y-value at which a line of a graph crosses the y-axis is called the ___
Answer:
We know that,
y-value at which a line of a graph crosses the y-axis is called the “y-intercept”
Hence, from the above,
We can conclude that the best term from the box to complete the given definition is “y-intercept”

Question 4.
An equation written in the form y = mx + b is called the ___
Answer:
We know that,
An equation written in the form y = mx + b is called the “Linear equation” or the “Slope-intercept form”
Hence, from the above,
We can conclude that the best term from the box to complete the given definition is “Linear equation”(or) the “Slope-intercept form”

Slope and y-Intercept

Find the slope and y-intercept of a line that passes through these points.

Question 5.
(2, 2) and (3, 0)
Answer:
The given points are:
(2, 2), and (3, 0)
Compare the given points with (x1, y1), (x2,y2)
We know that,
Slope(m) = y2 – y1 / x2 – x1
So,
m = \(\frac{0 – 2}{3 – 2}\)
= \(\frac{-2}{1}\)
= -2
We know that,
The linear equation in the slope-intercept form is:
y = mx + c
Where,
m is the slope
c is the y-intercept
We know that,
We can obtain the y-intercept by putting the value of x equal to 0
So,
y = -2x + c
Substitute (3, 0) or (2, 2) in the above equation
So,
0 = -6 + c
So,
c = 6
Hence, from the above,
We can conclude that
The slope of a line that passes through the given points is: -2
The y-intercept of a line is: 6

Question 6.
(1, 5) and (4, 10)
Answer:
The given points are:
(1, 5), and (4, 10)
Compare the given points with (x1, y1), (x2,y2)
We know that,
Slope(m) = y2 – y1 / x2 – x1
So,
m = \(\frac{10 – 5}{4 – 1}\)
= \(\frac{5}{3}\)
We know that,
The linear equation in the slope-intercept form is:
y = mx + c
Where,
m is the slope
c is the y-intercept
We know that,
We can obtain the y-intercept by putting the value of x equal to 0
So,
y = \(\frac{5}{3}\)x + c
Substitute (4, 10) or (1, 5) in the above equation
So,
5 = \(\frac{5}{3}\) + c
So,
c = \(\frac{10}{3}\)
Hence, from the above,
We can conclude that
The slope of a line that passes through the given points is: \(\frac{5}{3}\)
The y-intercept of a line is: \(\frac{10}{3}\)

Question 7.
(8, 2) and (-8,6)
Answer:
The given points are:
(8, 2), and (-8, 6)
Compare the given points with (x1, y1), (x2,y2)
We know that,
Slope(m) = y2 – y1 / x2 – x1
So,
m = \(\frac{6 – 2}{-8 – 8}\)
= \(\frac{4}{-16}\)
= –\(\frac{1}{4}\)
We know that,
The linear equation in the slope-intercept form is:
y = mx + c
Where,
m is the slope
c is the y-intercept
We know that,
We can obtain the y-intercept by putting the value of x equal to 0
So,
y = –\(\frac{1}{4}\)x + c
Substitute (-8, 6) or (8, 2) in the above equation
So,
2 = –\(\frac{1}{4}\) (8) + c
So,
c = 4
Hence, from the above,
We can conclude that
The slope of a line that passes through the given points is: –\(\frac{1}{4}\)
The y-intercept of a line is: 4

Compare Proportional Relationships

Jenna’s mother is shopping for energy drinks in 12-ounce bottles for Jenna’s soccer team. Store A sells a case of 18 bottles for $10. Store B sells a case of 12 bottles for $6. Which store sells the drinks for less? Use the graph to compare the unit costs of the drinks.

Question 8.
Envision Math Common Core Grade 8 Answer Key Topic 3 Use Functions To Model Relationships 1
Answer:
Jenna’s mother is shopping for energy drinks in 12-ounce bottles for Jenna’s soccer team. Store A sells a case of 18 bottles for $10. Store B sells a case of 12 bottles for $6
Now,
The unit cost rate of a bottle in store A = \(\frac{The cost of 18 bottles}{18}\)
= \(\frac{$10}{18}\)
= $0.55
The unit cost rate of a bottle in store B = \(\frac{The cost of 12 bottles}{12}\)
= \(\frac{$6}{12}\)
= $0.5
So,
The representation of the unit cost rate of a bottle in both stores is:

So,
From the above graph,
We can observe that
The unit cost rate of a bottle in store B < The unit cost rate of a bottle in store A
Hence, from the above,
We can conclude that store B sells the drinks for less cost

Linear Equations

Question 9.
Write the equation for the graph of the line shown.
Envision Math Common Core Grade 8 Answer Key Topic 3 Use Functions To Model Relationships 3.2
Answer:
The given graph is:
Envision Math Common Core Grade 8 Answer Key Topic 3 Use Functions To Model Relationships 3.2
We know that,
The y-intercept is the value of the point that passes through the y-axis
So,
From the graph,
The point that passes through the y-axis is: (0, -6)
So,
The y-intercept is: -6
Now,
To find the slope,
The points are: (2, 2), and (0, -6)
Now,
Compare the given points with (x1, y1), (x2,y2)
We know that,
Slope(m) = y2 – y1 / x2 – x1
So,
m = \(\frac{-6 – 2}{0 – 2}\)
= \(\frac{-8}{-2}\)
= 4
We know that,
The linear equation in the slope-intercept form is:
y = mx + c
So,
y = 4x – 6
Hence, from the above,
We can conclude that the equation of the line for the given graph is:
y = 4x – 6

Language Development

Write key words or phrases associated with each representation. Then write function or not a function on the given lines.
Envision Math Common Core Grade 8 Answer Key Topic 3 Use Functions To Model Relationships 3.3
Answer:

Topic 3 PICK A PROJECT

PROJECT 3A
Envision Math Common Core Grade 8 Answer Key Topic 3 Use Functions To Model Relationships 3.10
What machine could be invented to make your life better?
PROJECT: BUILD A RUBE GOLDBERG MACHINE

PROJECT 3B
Envision Math Common Core Grade 8 Answer Key Topic 3 Use Functions To Model Relationships 3.11
What games can you play indoors?
PROJECT: MAKE A MATH
CARD GAME

PROJECT 3C
Envision Math Common Core Grade 8 Answer Key Topic 3 Use Functions To Model Relationships 101
What are the steps for fixing a leaky pipe?
PROJECT: PLAN A MAINTENANCE ROUTE

PROJECT 3D
Envision Math Common Core Grade 8 Answer Key Topic 3 Use Functions To Model Relationships 102
If you were to make a video game, what kind of game would it be?
PROJECT: DESIGN A VIDEO GAME ELEMENT

Lesson 3.1 Understand Relations and Functions

Solve & Discuss It!

The 10 members of Photography Club want to raise $500, so they will hold a raffle with donated prizes. Jesse proposes that to reach their goal, each member should sell 50 raffle tickets. Alexis proposes that each member should raise $50.
Whose plan would you recommend? Explain.
Envision Math Common Core Grade 8 Answer Key Topic 3 Use Functions To Model Relationships 3.15
RAFFLE TICKETS
$1 1 ticket
$5 6 tickets
$20 25 tickets
Answer:
It is given that
The 10 members of the Photography Club want to raise $500, so they will hold a raffle with donated prizes. Jesse proposes that to reach their goal, each member should sell 50 raffle tickets. Alexis proposes that each member should raise $50.
It is also given that
$1              –       1 ticket
$5              –        6 tickets
$20            –        25 tickets
Now,
According to Jesse’s goal,
Each member should sell 50 raffle tickets to make the total amount of $500
The possible combinations may be:
50 $1 tickets will be sold by each member
Any other combination is not possible
According to Alexis’s plan,
Each member should raise $50 to make the total amount of $500
It is possible and very easy because $50 by each member can be raised in many ways
Hence, from the above,
We can conclude that Alexis’s plan would be recommended

Focus on math practices
Reasoning How are the two plans different? How are they similar?
Answer:
According to Jesse’s goal,
Each member should sell 50 raffle tickets to make the total amount of $500
The possible combinations may be:
50 $1 tickets will be sold by each member
Any other combination is not possible
Now,
According to Alexis’s plan,
Each member should raise $50 to make the total amount of $500
It is possible and very easy because $50 by each member can be raised in many ways

? Essential Question
when is a relation a function?
Answer:
A “Relation” from a set X to a set Y is called a “Function” if and only if each element of X is related to exactly one element in Y.

Try It!
Joe needs to advertise his company. He considers several different brochures of different side lengths and areas. He presents the data as ordered pairs (side length, area).
(4, 24), (5, 35), (8, 24), (2, 20), (9, 27)
Envision Math Common Core Grade 8 Answer Key Topic 3 Use Functions To Model Relationships 4.1
Complete the arrow diagram. Is the area of a brochure a function of the side length? Explain.
Answer:
Joe needs to advertise his company. He considers several different brochures of different side lengths and areas. He presents the data as ordered pairs (side length, area).
(4, 24), (5, 35), (8, 24), (2, 20), (9, 27)
We know that,
The ordered pairs can be represented in the form of (x, y)
Where,
x is the input
y is the output
Now,
From the given ordered pairs,
We can observe that for different values of the input, there are different values of output
Note:
If there are the same outputs for the different inputs, then also a relationship is considered a function
So,
The complete arrow diagram for the given ordered pairs are:

Hence, from the above,
We can conclude that the area of the brochure is a function of the side length

Convince Me!
There are two outputs of 24. Does this help you determine whether the relation is a function? Explain.
Answer:
We know that,
If there are the same outputs for the different inputs, then also a relationship is considered a function
Hence, from the above
We can conclude that even for the two outputs of 24, the given relationship is considered a function

Try It!

Frank reverses the ordered pairs to show the heights and ages of the same six students. Is age a function of height? Explain.
Envision Math Common Core Grade 8 Answer Key Topic 3 Use Functions To Model Relationships 4.2
Answer:
It is given that
Frank reverses the ordered pairs to show the heights and ages of the same six students.
Now,
The given table is:
Envision Math Common Core Grade 8 Answer Key Topic 3 Use Functions To Model Relationships 4.2
Now,
From the given table,
We can observe that
For different values of age, there are different values of height
Where,
Age —-> Input
Height —> Output
Hence, from the above,
We can conclude that age is a function of height

Try It!
Heather claims that she can tell exactly how long a family was at the museum by how much the family pays for parking. Is Heather correct? Explain.
Answer:
It is given that
Heather claims that she can tell exactly how long a family was at the museum by how much the family pays for parking.
Now,
The table for the given situation is: (Example 3)

Now,
From the table,
We can observe that
There are different costs for the different times in hours
So,
We can say that cost is a function of time
Hence, from the above,
We can conclude that Heather’s claim is correct

KEY CONCEPT
A relation is a function if each input corresponds to exactly one output. You can use an arrow diagram or a table to determine whether a relation is a function.
This relation is a function.
Envision Math Common Core Grade 8 Answer Key Topic 3 Use Functions To Model Relationships 4.3
This relation is not a function.
Envision Math Common Core Grade 8 Answer Key Topic 3 Use Functions To Model Relationships 4.4

Do You Understand?

Question 1.
? Essential Question
when is a relation a function?
Answer:
A relation from a set X to a set Y is called a function if and only if each element of X is related to exactly one element in Y

Question 2.
Model with Math
How can you use different representations of a relation to determine whether the relation is a function?
Answer:
Relations can be displayed as a table, a mapping, or a graph. In a table, the x-values and y-values are listed in separate columns. Each row represents an ordered pair: Displaying a relation as a table

Question 3.
Generalize
Is a relation always a function? Is a function always a relation? Explain.
Answer:
All functions are relations, but not all relations are functions. A function is a relation that for each input, there is only one output. Here are mappings of functions. The domain is the input or the x-value, and the range is the output or the y-value.

Question 4.
Is the relation shown below a function? Explain.
Envision Math Common Core Grade 8 Answer Key Topic 3 Use Functions To Model Relationships 4.5
Answer:
The given relation is:
Envision Math Common Core Grade 8 Answer Key Topic 3 Use Functions To Model Relationships 4.5
From the given relation,
We can observe that there is the same input for the different outputs,
We know that,
A relation can be considered as a function when the different inputs have different outputs
Hence, from the above,
We can conclude that the given relation is not a function

Question 5.
Is the relation shown below a function? Explain.
Envision Math Common Core Grade 8 Answer Key Topic 3 Use Functions To Model Relationships 4.6
Answer:
The given relation is:
Envision Math Common Core Grade 8 Answer Key Topic 3 Use Functions To Model Relationships 4.6
From the given relation,
We can observe that there are different outputs for different inputs
We know that,
A relation can be considered as a function when the different inputs have different outputs
Hence, from the above,
We can conclude that the given relation is a function

Question 6.
Is the relation shown below a function? Explain.
(4,16), (5, 25), (3,9), (6, 36), (2, 4), (1, 1)
Answer:
The given relation is:
(4,16), (5, 25), (3,9), (6, 36), (2, 4), (1, 1)
From the given relation,
We can observe that there are different outputs for different inputs
We know that,
A relation can be considered as a function when the different inputs have different outputs
Hence, from the above,
We can conclude that the given relation is a function

Practice & Problem Solving

Question 7.
The set of ordered pairs (1, 19), (2, 23), (3, 23), (4, 29), (5, 31) represents the number of tickets sold for a fundraiser. The input values represent the day and the output values represent the number of tickets sold on that day.
a. Make an arrow diagram that represents
Answer:
The arrow diagram for the given relation is:

b. is the relation a function? Explain.
Answer:
It is given that
The set of ordered pairs (1, 19), (2, 23), (3, 23), (4, 29), (5, 31) represents the number of tickets sold for a fundraiser. The input values represent the day and the output values represent the number of tickets sold on that day.
Now,
The given relation is:
(1, 19), (2, 23), (3, 23), (4, 29), (5, 31)
From the given relation,
We can observe that there are different outputs for different inputs
We know that,
A relation can be considered as a function when the different inputs have different outputs
Hence, from the above,
We can conclude that the given relation is a function

Question 8.
Does the relation shown below represent a function? Explain.
(-2, 2), (-7, 1), (-3, 9), (3, 4), (-9,5), (-6, 8)
Answer:
The given relation is:
(-2, 2), (-7, 1), (-3, 9), (3, 4), (-9,5), (-6, 8)
From the given relation,
We can observe that there are different outputs for different inputs
We know that,
A relation can be considered as a function when the different inputs have different outputs
Hence, from the above,
We can conclude that the given relation is a function

Question 9.
Is the relation shown in the table a function? Explain.
Envision Math Common Core Grade 8 Answer Key Topic 3 Use Functions To Model Relationships 4.10
Answer:
The given relation is:
Envision Math Common Core Grade 8 Answer Key Topic 3 Use Functions To Model Relationships 4.10

From the given relation,
We can observe that there are different outputs for the same inputs
We know that,
A relation can be considered as a function when the different inputs have different outputs
Hence, from the above,
We can conclude that the given relation is not a function

Question 10.
Construct Arguments
During a chemistry experiment, Sam records how the temperature changes over time using ordered pairs (time in minutes, temperature in °C).
(0, 15), (5, 20), (10,50) (15, 80). (20, 100), (25, 100) Is the relation a function? Explain.
Envision Math Common Core Grade 8 Answer Key Topic 3 Use Functions To Model Relationships 5.1
Answer:
It is given that
During a chemistry experiment, Sam records how the temperature changes over time using ordered pairs (time in minutes, temperature in °C).
(0, 15), (5, 20), (10,50) (15, 80). (20, 100), (25, 100)
We know that,
An ordered pair can be represented as (x, y)
Where,
x is the time
y is the temperature in °C
Now,
The given relation is:
(0, 15), (5, 20), (10,50) (15, 80). (20, 100), (25, 100)
From the given relation,
We can observe that there are different outputs for the different inputs
We know that,
A relation can be considered as a function when the different inputs have different outputs
Hence, from the above,
We can conclude that the given relation is a function

Question 11.
Reasoning
Taylor has tracked the number of students in his grade since third grade. He records his data in the table below. Is the relation a function? Explain.
Envision Math Common Core Grade 8 Answer Key Topic 3 Use Functions To Model Relationships 5.12
Answer:
It is given that
Taylor has tracked the number of students in his grade since third grade. He records his data in the table
Now,
The given table is:
Envision Math Common Core Grade 8 Answer Key Topic 3 Use Functions To Model Relationships 5.12
From the given table,
We can observe that there are different outputs (People) for the different inputs (Grade)
We know that,
A relation can be considered as a function when the different inputs have different outputs
Hence, from the above,
We can conclude that the given table is a function

Question 12.
James raises chickens. He tracks the number of eggs his chickens lay at the end of each week. Is this relation a function? Explain.
Envision Math Common Core Grade 8 Answer Key Topic 3 Use Functions To Model Relationships 5.3
Answer:
It is given that
James raises chickens. He tracks the number of eggs his chickens lay at the end of each week
Now,
The given relation is:
Envision Math Common Core Grade 8 Answer Key Topic 3 Use Functions To Model Relationships 5.3
From the given relation,
We can observe that there are different outputs (Eggs) for the different inputs (Weeks)
We know that,
A relation can be considered as a function when the different inputs have different outputs
Hence, from the above,
We can conclude that the given relation is a function

Question 13.
Relations P and Q are shown below.
Envision Math Common Core Grade 8 Answer Key Topic 3 Use Functions To Model Relationships 5.4
a. Make an arrow diagram to represent Relation P.
Answer:
The given relation is:
Envision Math Common Core Grade 8 Answer Key Topic 3 Use Functions To Model Relationships 5.4
Hence,
The arrow diagram to represent the relation P is:

Envision Math Common Core Grade 8 Answer Key Topic 3 Use Functions To Model Relationships 5.5
b. Make an arrow diagram to represent Relation Q.
Answer:
The given relation is:
Envision Math Common Core Grade 8 Answer Key Topic 3 Use Functions To Model Relationships 5.5
Hence,
The arrow diagram to represent the relation Q is:

c. Which relation is a function? Explain.
Answer:
From relation P,
We can observe that there are different outputs for the different inputs
From relation Q,
We can observe that there are different outputs for the same inputs
Hence, from the above,
We can conclude that relation P is a function

Question 14.
Higher-Order Thinking
On a recent test, students had to determine whether the relation represented by the ordered pairs (1, 2), (6, 12), (12, 24), (18, 36) is a function. Bobby drew the arrow diagram on the right and said the relationship was not a function. What error did Bobby most likely make?
Envision Math Common Core Grade 8 Answer Key Topic 3 Use Functions To Model Relationships 5.6
Answer:
It is given that
On a recent test, students had to determine whether the relation represented by the ordered pairs (1, 2), (6, 12), (12, 24), (18, 36) is a function. Bobby drew the arrow diagram on the right and said the relationship was not a function.
Now,
From the given arrow diagram and ordered pairs,
We can observe that
In the arrow diagram, inputs and outputs are reversely represented
Hence, from the above,
We can conclude that the error Bobby most likely made is the reversal of inputs and outputs

Assessment Practice

Question 15.
Write the set of ordered pairs that is represented by the arrow diagram at the right. Is the relation a function? Explain.
Envision Math Common Core Grade 8 Answer Key Topic 3 Use Functions To Model Relationships 5.7
Answer:
The given arrow diagram is:
Envision Math Common Core Grade 8 Answer Key Topic 3 Use Functions To Model Relationships 5.7
In the arrow diagram,
The left side represented the inputs and the right side represented the outputs
So,
The representation of the arrow diagram in the form of the ordered pairs (Input, Output) are:
(49, 13), (61, 36), (10, 27), (76, 52), (23, 52)
From the above relation,
We can observe that there are different outputs for the different inputs
We know that,
A relation can be considered as a function when the different inputs have different outputs
Hence, from the above,
We can conclude that the given relation is a function

Question 16.
Which of these relations are functions? Select all that apply.
Envision Math Common Core Grade 8 Answer Key Topic 3 Use Functions To Model Relationships 5.8
Answer:
We know that,
A relation can be considered as a function when the different inputs have different outputs
So,
From the given relations,
Relation 2, Relation 3 are the functions
Hence, from the above,
We can conclude that Relation 2 and Relation 3 are the functions

Lesson 3.2 Connect Representations of Functions

Solve & Discuss It!

Eliza volunteers at a nearby aquarium, where she tracks the migratory patterns of humpback whales from their feeding grounds to their breeding grounds. She recorded the distance, in miles, traveled by the whales each day for the first 7-day period of their migration. Based on Eliza’s data, how long will it take the humpback whales to travel the 3,100 miles to their breeding grounds?
Envision Math Common Core Grade 8 Answer Key Topic 3 Use Functions To Model Relationships 5.9

Focus on math practices
Construct Arguments How does finding an average distance the whales travel in miles help with finding a solution to this problem?

? Essential Question
What are different representations of a function?
Answer:
Relationships and functions can be represented as graphs, tables, equations, or verbal descriptions. Each representation gives us certain information. A table of values, mapping diagram, or set of ordered pairs gives us a list of input values and their corresponding output values.

Try It!
As the pump is pumping water, the amount of water in the pool decreases at a constant rate. Complete the statements below. Then graph the function.
Envision Math Common Core Grade 8 Answer Key Topic 3 Use Functions To Model Relationships 5.10
The amount of water remaining in the pool is Envision Math Common Core Grade 8 Answer Key Topic 3 Use Functions To Model Relationships 5.11 gallons.
The amount of water pumped each hour is Envision Math Common Core Grade 8 Answer Key Topic 3 Use Functions To Model Relationships 5.11 gallons.
The equation is Envision Math Common Core Grade 8 Answer Key Topic 3 Use Functions To Model Relationships 5.11
Answer:
It is given that
As the pump is pumping water, the amount of water in the pool decreases at a constant rate
Now,
Let the initial amount of water present in the pool is: 9,000 gallons
So,
The rate of the amount of water that pumped each hour = \(\frac{The initial amount of water present in the pool}{The time that is present where the initial amount of water present}\)
= \(\frac{9,000}{12}\)
= 750 gallons per hour
So,
The amount of water remaining in the pool = The initial amount of water present in the pool – The amount of water that pumped each hour
= 9,000 – 1,500
= 7,500 gallons
Let the number of hours be x
We know that,
The linear equation is in the form of
y = mx + c
So,
The total amount of water present in the pool = The rate at which the water pumps out + The amount of water that pumped each hour
9,000 = 750x + 7,500
Hence, from the above,
We can conclude that
The amount of water remaining in the pool is 7,500 gallons.
The amount of water pumped each hour is 1,500 gallons.
The equation is:
9,000 = 750x + 7,500

Convince Me!
How is the rate of change of this function different from that in Example 1? Explain.
Answer:
The rate of change of the function present in Example 1 is increasing at a constant rate whereas the rate of change of the function in this situation is decreasing at a constant rate

Try It!
Draw a graph that represents a linear function?
Envision Math Common Core Grade 8 Answer Key Topic 3 Use Functions To Model Relationships 5.111
Answer:
We know that,
The representation of the linear equation is:
y = mx —–> Slope form
y = mx + c —-> Slope-intercept form
Now,
Let the linear equation in the slope-intercept form be:
y = x + 3
Hence,
The graph of the above linear equation in the coordinate plane is:

KEY CONCEPT

You can represent a function in different ways: in a table, in a graph, or as an equation.
A day at the amusement park costs $10 for an entrance fee and $2.50 for each ride ticket.
Envision Math Common Core Grade 8 Answer Key Topic 3 Use Functions To Model Relationships 5.122

Do You Understand?

Question 1.
?Essential Question What are different representations of a function?
Answer:
Relationships and functions can be represented as graphs, tables, equations, or verbal descriptions. Each representation gives us certain information. A table of values, mapping diagram, or set of ordered pairs gives us a list of input values and their corresponding output values.

Question 2.
Use Appropriate Tools How can you use a graph to determine that a relationship is NOT a function?
Answer:
Use the vertical line test to determine whether or not a graph represents a function. If a vertical line is moved across the graph and, at any time, touches the graph at only one point, then the graph is a function. If the vertical line touches the graph at more than one point, then the graph is not a function.

Question 3.
Construct Arguments Must the ordered pairs of a function be connected by a straight line or a curve on a graph? Explain.
Answer:
The points can be connected by a straight line. Thus, the ordered pairs represent a linear function.

Do You Know How?

Question 4.
Each week, Darlene tracks the number of party hats her company has in stock. The table shows the weekly stock. Is the relationship a linear function? Use the graph below to support your answer.
Envision Math Common Core Grade 8 Answer Key Topic 3 Use Functions To Model Relationships 4.12
Answer:
It is given that
Each week, Darlene tracks the number of party hats her company has in stock. The table shows the weekly stock.
We know that,
A relation is defined as a function only when there are different outputs for different inputs
Now,
From the given table,
We can observe that the outputs (party hats) are different for different inputs (Weeks)
Now,
The given function can be called “Linear function” if the rate of change is constant
The given function can be called a “Non-linear function” if the rate of change is not constant
Now,
The rate of change = Initial value – Next value
Hence, from the above,
We can conclude that the given relationship is a linear function

Question 5.
How can Darlene use the graph above to know when to order more party hats?
Answer:
From the graph,
Darlene know when to order more hats when there is no stack when observing the track sheet

Practice & Problem Solving

Leveled Practice In 6-7, explain whether each graph represents a function.

Question 6.
Envision Math Common Core Grade 8 Answer Key Topic 3 Use Functions To Model Relationships 6.2
Answer:
The given graph is:
Envision Math Common Core Grade 8 Answer Key Topic 3 Use Functions To Model Relationships 6.2
From the above graph,
We can observe that each input has a different output and the rate of change is constant
Hence, from the above,
We can conclude that the given graph represents a function

Question 7.
Envision Math Common Core Grade 8 Answer Key Topic 3 Use Functions To Model Relationships 6.3
Answer:
The given graph is:
Envision Math Common Core Grade 8 Answer Key Topic 3 Use Functions To Model Relationships 6.3
From the given graph,
We can observe that each input has a different output but the rate of change is not constant
Hence, from the above,
We can conclude that the given graph represents a function but a non-linear function

Question 8.
Hannah approximates the areas of circles using the equation A = 3r2 and records areas of circles with different radius lengths in a table.
Envision Math Common Core Grade 8 Answer Key Topic 3 Use Functions To Model Relationships 6.4
a. Graph the ordered pairs from the table.
Answer:
The given table is:
Envision Math Common Core Grade 8 Answer Key Topic 3 Use Functions To Model Relationships 6.4
So,
From the table,
The representation of the ordered pairs (in, in²) are:
(1, 3), (2, 12), (3, 27), (4, 48), (5, 75)
Hence,
The representation of the ordered pairs in the coordinate plane is:

b. Is the relation a function? Explain.
Answer:
From part (a),
We can observe that there are different outputs for different inputs
Hence, from the above,
We can conclude that the given relation is a function

Question 9.
Model with Math
The relationship between the number of hexagons, x, and the perimeter of the figure they form, y, is shown in the graph. is the perimeter of the figure a function of the number of hexagons? Explain.
Envision Math Common Core Grade 8 Answer Key Topic 3 Use Functions To Model Relationships 6.5
Answer:
It is given that
The relationship between the number of hexagons, x, and the perimeter of the figure they form, y, is shown in the graph
Now,
From the graph,
We can observe that for different values of perimeters, the number of hexagons is also different
Hence, from the above,
We can conclude that the perimeter of a figure is a function of the number of hexagons

Question 10.
Construct Arguments Do the ordered pairs plotted in the graph below represent a function? Explain.
Envision Math Common Core Grade 8 Answer Key Topic 3 Use Functions To Model Relationships 6.6
Answer:
The given graph is:
Envision Math Common Core Grade 8 Answer Key Topic 3 Use Functions To Model Relationships 6.6
From the given graph,
We can observe that for the different values of x, there are different values of y
Hence, from the above,
We can conclude that the given ordered pairs in the graph represents a function

Question 11.
A train leaves the station at time t = 0. Traveling at a constant speed, the train travels 360 kilometers in 3 hours.
a. Write a function that relates the distance traveled, d, to the time, t.
Envision Math Common Core Grade 8 Answer Key Topic 3 Use Functions To Model Relationships 6.7
Answer:
It is given that
A train leaves the station at time t = 0. Traveling at a constant speed, the train travels 360 kilometers in 3 hours.
We know that,
Speed = \(\frac{Distance}{Time}\)
Here,
Speed is constant
So,
Distance = m (Time)
d = mt
Where,
m is the rate of change or proportionality constant
Now,
m = \(\frac{Distance}{Time}\)
= \(\frac{360}{3}\)
= 120 km / hour
Hence, from the above,
We can conclude that the function that relates to the distance d, and time t is:
d = 120t

b. Graph the function and tell whether it is a linear function or a nonlinear function.
The function is a Envision Math Common Core Grade 8 Answer Key Topic 3 Use Functions To Model Relationships 6.8 function.
Answer:
From part (a),
The function that relates the distance d, and time t is:
d = 120t
Now,
Compare the above equation with y = mx
Hence,
The graph of the above function is:

Hence, from the above,
We can conclude that the given function is a linear function from the above graph

Question 12.
Higher-Order Thinking Tell whether each graph is a function and justify your answer. Which graph is not a good representation of a real-world situation? Explain.
Envision Math Common Core Grade 8 Answer Key Topic 3 Use Functions To Model Relationships 6.88
Answer:
The given graphs are:
Envision Math Common Core Grade 8 Answer Key Topic 3 Use Functions To Model Relationships 6.88
From graph A,
We can observe that there are different outputs for the same input
So,
Graph A does not represent the function
From graph B,
We can observe that there are different outputs for the different inputs
So,
Graph B does represent a function
Hence, from the above,
We can say that graph A does not represent the real-world situation

Assessment Practice

Question 13.
You have an ant farm with 22 ants. The population of ants on your farm doubles every 3 months.
PART A
Complete the table.
Envision Math Common Core Grade 8 Answer Key Topic 3 Use Functions To Model Relationships 7.1
Answer:
It is given that
You have an ant farm with 22 ants. The population of ants on your farm doubles every 3 months.
Hence,
The completed table for the given situation is:

PART B
Is the relation a function? If so, is it a linear function or a nonlinear function? Explain.
Envision Math Common Core Grade 8 Answer Key Topic 3 Use Functions To Model Relationships 7.2
Answer:
From part (a),
We can observe that the ant population is different for the different number of months
Now,
Rate of change = \(\frac{44}{22}\)
= 2
So,
The rate of change is also constant for all the table
Hence, from the above,
We can conclude that the given relation is a linear function

Question 14.
Use the function y = \(\frac{3}{2}\)x + 3 to complete the table of values.
Envision Math Common Core Grade 8 Answer Key Topic 3 Use Functions To Model Relationships 7.3
Answer:
The given function is:
y = \(\frac{3}{2}\)x + 3
Hence,
The completed table for the given values of y is:

Lesson 3.3 Compare Linear and Nonlinear Functions

Solve & Discuss It!

Two streaming video subscription services offer family plans with different monthly costs, as shown in the ads below. What do the two plans have in common? How are they different? When is Movies4You a better deal than Family Stream?
Envision Math Common Core Grade 8 Answer Key Topic 3 Use Functions To Model Relationships 7.4
Answer:
It is given that
Two streaming video subscription services offer family plans with different monthly costs, as shown in the ads
Now,
Let the number of devices be x
Let the total subscription cost be y
So,
For Movies 4 U,
The total subscription cost = The subscription cost of the first device + The subscription fee for additional devices
y = $10 + $2x
For Family Stream,
The total subscription cost = The subscription cost of the first device + The subscription fee for additional devices
y = \(\frac{$12}{4}\) + $1x
y = $3 + $1x
Now,
The above equations are in the form of slope-intercept form
We know that,
The slope-intercept form of the equation is:
y = mx + c
Now,
The common points in the two plans are:
A) The subscription cost of the first device plan
B) The additional fee plan
The different points in the two plans are:
A) The first plan consists of the additional fee of each device
B) The second plan consists of the additional fee for greater than 5 devices
Now,
Movies 4 U is better than Family Stream when the subscription cost of the first device will be less

Model with Math
How can you represent the relationship between cost and number of devices?
Answer:
The representation of the relationship between the cost and the number of devices is:
The total subscription cost = The subscription cost of the first device + The subscription fee for additional devices

Focus on math practices
Look for Relationships Describe the relationship between the cost and the number of devices for each service. What do you notice about each relationship?
Answer:
The relationship between the cost and the number of devices for each service is:
The total subscription cost = The subscription cost of the first device + The subscription fee for additional devices
In the service of Family Stream,
The subscription fee is given for up to 4 devices and the additional subscription fee is for greater than 5 devices

? Essential Question
How can you compare two functions?
Answer:
The two functions can be compared by:
A) Identify the rate of change for the first function
B) Identify the rate of change for the second function
C) Identify the y-intercept of the first function
D) Identify the y-intercept of the second function
E) Compare the properties of each function

Try It!
The welding rate of a third robot is represented by the equation t = 10.8w, where t represents the time in minutes and w represents the number of welding tasks. How does it compare to the other two?
Answer:
It is given that
The welding rate of a third robot is represented by the equation t = 10.8w, where t represents the time in minutes and w represents the number of welding tasks (Refer to Example 1)
Now,
For a third robot,
The wielding rate = \(\frac{The number of wielding tasks (w)}{Time in minutes (t)}\)
= 10.8 (From the equation t = 10.8w)
Now,
When we compare the wielding rates of the three robots,
The wielding rate of the first robot (10.4) < The wielding rate of the third robot (10.8) < The wielding rate of the second robot (11.2)
Hence, from the above,
We can conclude that the comparison of the wielding rates of the three robots is:
The first robot’s wielding rate < The third robot’s wielding rate < The second robot’s wielding rate

Convince Me!
How can linear equations help you compare linear functions?
Answer:
While all linear equations produce straight lines when graphed, not all linear equations produce linear functions. In order to be a linear function, a graph must be both linear (a straight line) and a function (matching each x-value to only one y-value).

Try It!
Compare the properties of these two linear functions.
Envision Math Common Core Grade 8 Answer Key Topic 3 Use Functions To Model Relationships 8.1
Answer:
The given functions are:
Envision Math Common Core Grade 8 Answer Key Topic 3 Use Functions To Model Relationships 8.1
Now,
For function 1,
Find out the rate of change and the y-intercept i.e., the initial value
We know that,
Rate of change = \(\frac{y}{x}\)
Now,
For y = 1 and x = 2.
Rate of change = 0.5
For y = 5.5 and x = 5,
Rate of change = 1.1
SO,
From the above values,
We can say that the rate of change is not constant
So,
The given function is a non-linear function and it does not have any initial value i.e., the y-intercept is 0
Now,
For function 2,
Compare the given equation with
y = mx + c
Where,
m is the slope or the rate of change
c is the initial value or the y-intercept
So,
From the given equation,
Rate of change (m): 2
The y-intercept is: -4
Hence, from the above 2 functions,
We can conclude that
The y-intercept of function1 > The y-intercept of function 2

KEY CONCEPT
You can compare functions in different representations by using the properties of functions.
Compare the constant rate of change and the initial value.
Envision Math Common Core Grade 8 Answer Key Topic 3 Use Functions To Model Relationships 8.2

Do You Understand?

Question 1.
? Essential Question
How can you compare two functions?
Answer:
The two functions can be compared by:
A) Identify the rate of change for the first function
B) Identify the rate of change for the second function
C) Identify the y-intercept of the first function
D) Identify the y-intercept of the second function
E) Compare the properties of each function

Question 2.
Reasoning Anne is running on a trail at an average speed of 6 miles per hour beginning at mile marker 4. John is running on the same trail at a constant speed, shown in the table. How can you determine who is running faster?
Envision Math Common Core Grade 8 Answer Key Topic 3 Use Functions To Model Relationships 9.1
Answer:
It is given that
Anne is running on a trail at an average speed of 6 miles per hour beginning at mile marker 4. John is running on the same trail at a constant speed, shown in the table.
So,
For Anne,
The rate of change is defined as the average speed
The y-intercept is defined as the beginning point
Hence,
For Anne,
The rate of change is: 6 miles per hour
The y-intercept is: 4
Now,
The given table is:
Envision Math Common Core Grade 8 Answer Key Topic 3 Use Functions To Model Relationships 9.1
From the given table,
For John,
Rate of change = \(\frac{y}{x}\) = \(\frac{Mile marker}{Time (hours)}\)
Now,
The total distance traveled by John (y) = Final value – Initial value
= 11.5 – 1
= 10.5 miles
The total time took by John (x) = 1.5 hours
So,
Rate of change = \(\frac{y}{x}\)
= \(\frac{10.5}{1.5}\)
= 7 miles per hour
Now,
We know that,
The y-intercept is the value of y when x = 0
So,
The y-intercept is: 1
Hence, from the above,
By comparing the values of the rate of change,
We can conclude that John is running faster

Question 3.
Reasoning In Item 2, how do Anne and John’s starting positions compare? Explain.
Answer:
We know that,
The starting positions are nothing but the initial positions of both Anne and John i.e., the y-intercepts of both Anne and John
So,
The y-intercept of Anne is: 4
The y-intercept of John is: 1
Hence, from the above,
By comparing the y-intercepts,
We can conclude that
The starting position of Anne > The starting position of John

Do You Know How?

Felipe and Samantha use a payment plan to buy musical instruments. Felipe writes the equation y = -30x + 290 to represent the amount owed, y, after x payments. The graph shows how much Samantha owes after each payment.
Envision Math Common Core 8th Grade Answers Topic 3 Use Functions To Model Relationships 9.2
Answer:
It is given that
Felipe and Samantha use a payment plan to buy musical instruments. Felipe writes the equation y = -30x + 290 to represent the amount owed, y, after x payments. The graph shows how much Samantha owes after each payment.
Now,
Compare the given equation with
y = mx + c
Where,
m is the rate of change
c is the initial value or the y-intercept
So,
From the given equation
For Felipe,
The rate of change is: -30
The initial value is: 290
Now,
The given graph is:
Envision Math Common Core 8th Grade Answers Topic 3 Use Functions To Model Relationships 9.2
For Samantha,
From the given graph,
The initial value is: 240
The rate of change = \(\frac{Any value of y}{The value of x that corresponds to the value of y}\)
= \(\frac{120}{6}\)
= 20

Question 4.
Whose musical instrument costs more, Felipe’s or Samantha’s? Explain.
Answer:
We know that,
If the amount owed (y) is less i.e, the rate of change is negative, then the cost of the instrument will also be less
So,
When we compare the rate of change of Felip and Samantha,
The rate of change of Felip < The rate of change of Samantha
Hence, from the above comparison,
We can conclude that the instrument of Samantha costs more

Question 5.
Who will pay more each month? Explain.
Answer:
When we compare the rate of change of Felip and Samantha,
The rate of change of Felip < The rate of change of Samantha
Hence, from the above comparison,
We can conclude that Samantha will pay more each month

Practice & Problem Solving

Question 6.
Two linear functions are shown below. Which function has the greater rate of change?
Envision Math Common Core 8th Grade Answers Topic 3 Use Functions To Model Relationships 9.3
Answer:
The given linear functions are:
Envision Math Common Core 8th Grade Answers Topic 3 Use Functions To Model Relationships 9.3
We know that,
The rate of change = \(\frac{y}{x}\)
So,
For Function A,
The rate of change = \(\frac{Any value of y}{The value of x that corresponds to y}\)
= \(\frac{4}{2}\)
= 2
For Function B,
The rate of change = \(\frac{y}{x}\)
=  \(\frac{3}{2}\)
= 1.5
Hence, from the above,
We can conclude that Function A has a greater rate of change

Question 7.
Two linear functions are shown below. Which function has the greater initial value?
Envision Math Common Core 8th Grade Answers Topic 3 Use Functions To Model Relationships 9.4
Answer:
The given functions are:
Envision Math Common Core 8th Grade Answers Topic 3 Use Functions To Model Relationships 9.4
We know that,
The initial value is also known as the y-intercept
The y-intercept is the value of y when x = 0
So,
For Function A,
The initial value (y-intercept) is: 4
For function B,
Compare the given equation with
y = mx + c
Where,
m is the rate of change
c is the initial value or the y-intercept
So.
The initial value (y-intercept) is: 3
Hence, from the above,
We can conclude that Function A has the greater initial value

Question 8.
Tell whether each function is linear or nonlinear.
Envision Math Common Core 8th Grade Answers Topic 3 Use Functions To Model Relationships 9.5
Answer:
The given functions are:
Envision Math Common Core 8th Grade Answers Topic 3 Use Functions To Model Relationships 9.5
We know that,
To find whether the given function is linear or non-linear, we have to verify whether the rate of change is constant or not
If the rate of change is constant, then the function is linear
If the rate of change is not constant, then the function is non-linear
Now,
For Function A,
The rate of change = \(\frac{y}{x}\)
For x = 1 and y = 2,
The rate of change = 2
For x = 2 and y = 5,
The rate of change = 2.5
Hence,
Function A is a non-linear function
Now,
For Function B,
The rate of change = \(\frac{y}{x}\)
For x = 1 and y = 4,
The rate of change = 4
For x = 1.5 and y = 3,
The rate of change = 2
Hence,
Function B is a non-linear function

Question 9.
Tell whether each function is linear or nonlinear.
Envision Math Common Core 8th Grade Answers Topic 3 Use Functions To Model Relationships 9.6
Answer:
The given functions are:
Envision Math Common Core 8th Grade Answers Topic 3 Use Functions To Model Relationships 9.6
We know that,
For a relation to be a graph, each input has a different output but the same input will not have different outputs
Now,
From Function A,
We can observe that there are different inputs for different outputs i.e., the values of x and y are different
So,
The rate of change is not constant since the graph is non-linear
Hence,
Function A is a non-linear function
Now,
From Function B,
Compare the given equation with
y = mx + c
Where,
m = 1
c = 0
So,
The rate of change is constant for Function B
Hence,
Function B is a linear function

Question 10.

Determine whether each function is linear or nonlinear from its graph.
Envision Math Common Core 8th Grade Answers Topic 3 Use Functions To Model Relationships 9.7
Answer:
The given graphs are:
Envision Math Common Core 8th Grade Answers Topic 3 Use Functions To Model Relationships 9.7
We know that,
If the graph is a straight line, then the function is a linear function
If the graph is not a straight line, then the function is not a linear function
Hence, from the above,
We can conclude that
The function 1 is a linear function
The function 2 is a non-linear function

Question 11.
Look for Relationships Justin opens a savings account with $4. He saves $2 each week. Does a linear function or a nonlinear function represent this situation? Explain.
Envision Math Common Core 8th Grade Answers Topic 3 Use Functions To Model Relationships 9.8
Answer:
It is given that
Justin opens a savings account with $4. He saves $2 each week.
Now,
The given table is:
Envision Math Common Core 8th Grade Answers Topic 3 Use Functions To Model Relationships 9.8
From the given table,
We can observe that there is a constant rate of change
Now,
For weeks,
We can obtain the number of weeks by adding 1 i.e., 0 + 1, 1 + 1, etc
So,
The rate of change is constant i.e., 1
For money in account,
We can obtain the money by adding 2 to the initial amount of money i.e., 4 + 2, 6 + 2, etc
So,
The rate of change is constant i.e., 2
Hence, from the above,
We can conclude that since the rate of change is constant, the given situation represents a linear function

Question 12.
Reasoning The function y = 4x + 3 describes Player A’s scores in a game of trivia, where x is the number of questions answered correctly and y is the score. The function represented in the table shows Player B’s scores. What do the rates of change tell you about how each player earns points?
Envision Math Common Core 8th Grade Answers Topic 3 Use Functions To Model Relationships 9.9
Answer:
It is given that
The function y = 4x + 3 describes Player A’s scores in a game of trivia, where x is the number of questions answered correctly and y is the score. The function represented in the table shows Player B’s scores.
Now,
For Player A,
The given equation is:
y = 4x + 3
Compare the givene quation with
y = mx + c
Where,
m is the rate of change
c is the y-intercept
So,
The rate of change of Player A is: 4
Now,
The given table is:
Envision Math Common Core 8th Grade Answers Topic 3 Use Functions To Model Relationships 9.9
For Player B,
From the given table,
We can observe that the rate of change is constant for both the values of x and y
The rate of change for both the values of x and y is: 1
So,
The rate of change of Player B is: 1
So,
The rate of change of Player A > The rate of change of Player B
Hence, from the above,
We can conclude that Player A earns more points

Question 13.
Two athletes are training over a two-week period to increase the number of push-ups each can do consecutively. Athlete A can do 16 push-ups to start, and increases his total by 2 each day. Athlete B’s progress is charted in the table. Compare the initial values for each. What does the initial value mean in this situation?
Envision Math Common Core 8th Grade Answers Topic 3 Use Functions To Model Relationships 9.10
Answer:
It is given that
Two athletes are training over a two-week period to increase the number of push-ups each can do consecutively. Athlete A can do 16 push-ups to start, and increases his total by 2 each day. Athlete B’s progress is charted in the table
Now,
For Athlete A,
The starting point is: 16
So,
The initial point for Athlete A is: 16
Now,
For Athlete B,
The given table is:
Envision Math Common Core 8th Grade Answers Topic 3 Use Functions To Model Relationships 9.10
We know that,
The initial point or the y-intercept is the value of y when x = 0
So,
The initial point for Athlete B is: 12
Hence, from the above,
We can conclude that
The initial points in the given situation describe the number of pushups one can do at a time without stop
The initial point for Athlete A > The initial point for Athlete B

Question 14.
Higher-Order Thinking The equation y = 4x – 2 and the table and graph shown at the right describe three different linear functions. Which function has the greatest rate of change? Which has the least? Explain.
Envision Math Common Core 8th Grade Answers Topic 3 Use Functions To Model Relationships 9.11
Answer:
It is given that
The equation y = 4x – 2 and the table and graph shown at the right describe three different linear functions
Now,
a)
The given equation is:
y = 4x – 2
Compare the given equation with
y = mx + c
Where,
m is the constant rate of change
So,
For the given equation,
The rate of change is: 4
b)
The given table and graph are:
Envision Math Common Core 8th Grade Answers Topic 3 Use Functions To Model Relationships 9.11
Now,
From the given table,
The rate of change = \(\frac{y}{x}\)
For x = 1 and y = 5,
The rate of change is: 5
For x = 2 and y = 10
The rate of change is: 5
Now,
Since the rate of change is constant for all the cases,
The rate of change for the given table is: 5
Now,
From the given graph,
The given points to find the slope are: (0, 4), and (2, 0)
So,
Slope (or) The rate of change = \(\frac{0 – 4}{2 – 0}\)
= \(\frac{-4}{2}\)
= -2
Now,
When we compare the rate of change for all the three linear functions,
The rate of change of the table > The rate of change of the equation < The rate of change of the graph
Hence, from the above,
We can conclude that
The function that has the greatest rate of change is: Table
The function that has the least rate of change is: Graph

Assessment Practice

Question 15.
The students in the After-School Club ate 12 grapes per minute. After 9 minutes, there were 32 grapes remaining. The table shows the number of carrots remaining after different amounts of time. Which snack did the students eat at a faster rate? Explain.
Envision Math Common Core 8th Grade Answers Topic 3 Use Functions To Model Relationships 9.12
Answer:
It is given that
The students in the After-School Club ate 12 grapes per minute. After 9 minutes, there were 32 grapes remaining. The table shows the number of carrots remaining after different amounts of time.
Now,
The rate of change of grapes consumption is: 12 grapes per minute
Now,
The given table is:
Envision Math Common Core 8th Grade Answers Topic 3 Use Functions To Model Relationships 9.12
Now,
The rate of change of carrots consumption = \(\frac{The difference between any 2 values of carrots remaining}{The values of the tie elapsed corresponding to the carrots remaining}\)
= \(\frac{118 – 136}{8 – 6}\)
= –\(\frac{18}{2}\)
= -9 carrots per minute
So,
The consumption rate of grapes > The consumption rate of carrots
Hence, from the baove,
We can conclude that grapes can be eaten at a faster rate

Question 16.
The height of a burning candle can be modeled by a linear function. Candle A has an initial height of 201 millimeters, and its height decreases to 177 millimeters after 4 hours of burning. The height, h, in millimeters, of Candle B, can be modeled by the function h = 290 – 5t, where t is the time in hours. Which of the following statements are true?
Envision Math Common Core 8th Grade Answers Topic 3 Use Functions To Model Relationships 9.13 The initial height of Candle A is greater than the initial height of Candle B.
Envision Math Common Core 8th Grade Answers Topic 3 Use Functions To Model Relationships 9.13 The height of Candle A decreases at a faster rate than the height of Candle B.
Envision Math Common Core 8th Grade Answers Topic 3 Use Functions To Model Relationships 9.13 Candle B will burn out in 58 hours.
Envision Math Common Core 8th Grade Answers Topic 3 Use Functions To Model Relationships 9.13 After 10 hours, the height of Candle A is 110 millimeters.
Envision Math Common Core 8th Grade Answers Topic 3 Use Functions To Model Relationships 9.13 Candle A will burn out before Candle B.
Answer:
Let the given options be named as A, B, C, D, and E
It is given that
The height of a burning candle can be modeled by a linear function. Candle A has an initial height of 201 millimeters, and its height decreases to 177 millimeters after 4 hours of burning. The height, h, in millimeters, of Candle B, can be modeled by the function h = 290 – 5t, where t is the time in hours
Now,
The rate of change of Candle A = \(\frac{201 – 177}{4}\)
= \(\frac{24}{4}\)
= 6 millimeters per hour
Now,
For Candle B,
The time to burn the Candle B = \(\frac{290}{5}\)
= 58 hours
Hence, from the above,
We can conclude that B, C, and E match with the given situation

Topic 3 MID-TOPIC CHECKPOINT

Question 1.
Vocabulary How can you determine whether a relation is a function? Lesson 3.1
Answer:
Identify the output values. If each input value leads to only one output value, classify the relationship as a function. If any input value leads to two or more outputs, do not classify the relationship as a function.

Question 2.
Can an arrow or arrows be drawn from 10.3 so the relation in the diagram is a function? Explain your answer. Lesson 3.1
Envision Math Common Core 8th Grade Answers Topic 3 Use Functions To Model Relationships 102
Answer:
The given diagram is:
Envision Math Common Core 8th Grade Answers Topic 3 Use Functions To Model Relationships 102
For the relation in the diagram to be a function,
The arrows from the right side to the left side can be many but the arrows from the left side to the right side can only be one

Question 3.
Two linear functions are shown below. Which function has the greater rate of change? Justify your response. Lesson 3.3
Envision Math Common Core 8th Grade Answers Topic 3 Use Functions To Model Relationships 10.1
Answer:
The given linear functions are:
Envision Math Common Core 8th Grade Answers Topic 3 Use Functions To Model Relationships 10.1
We know that,
For a linear function,
The rate of change is constant
So,
For Function A,
The rate of change = \(\frac{y}{x}\)
= \(\frac{3}{6}\)
= \(\frac{1}{2}\)
= 0.5
For Function B,
The given equation is:
y = \(\frac{1}{2}\)x – 1
y = 0.5x – 1
Compare the above equation with
y = mx + c
Where,
m is the rate of change
So,
The rate of change is: 0.5
Hence, from the above,
We can conclude that the two linear functions have the same rate of change

Question 4.
Neil took 3 math tests this year. The number of hours he spent studying for each test and the corresponding grades he earned is shown in the table. Is the relation of hours of study time to the grade earned on a test a function? Explain why. Use the graph to justify your answer. Lesson 3.2
Envision Math Common Core 8th Grade Answers Topic 3 Use Functions To Model Relationships 10.2
Answer:
It is given that
Neil took 3 math tests this year. The number of hours he spent studying for each test and the corresponding grades he earned is shown in the table.
Now,
For the number of hours as input and the Grades as the output,
We can observe that
For each value of the hours, there are the same values of the Grade
So,
The relation of hours of study time to the grade earned on a test is not considered a function
Now,
The representation of the relation in the form of the ordered pairs is:
(4, 75), (6, 75), and (6, 82)
Hence,
The representation of the ordered pairs in the coordinate plane is:

Question 5.
Is the function shown linear or nonlinear? Explain your answer. Lesson 3.3
Envision Math Common Core 8th Grade Answers Topic 3 Use Functions To Model Relationships 10.3
Answer:
The given graph is:
Envision Math Common Core 8th Grade Answers Topic 3 Use Functions To Model Relationships 10.3
From the given graph,
We can observe that
For each value of x, there is only 1 value of y
So,
The given graph is a function
We know that,
A function is called a linear if the graph is a straight line
A function is called non-linear if the graph is in any shape other than the straight line
Hence, from the above,
We can conclude that the given graph is a non-linear function

How well did you do on the mid-topic checkpoint? Fill in the stars.Envision Math Common Core 8th Grade Answers Topic 3 Use Functions To Model Relationships 10.5

Topic 3 MID-TOPIC PERFORMANCE TASK

Sarah, Gene, and Paul are proposing plans for a class fundraiser. Each presents his or her proposal for the amount of money raised, y, for x number of hours worked, in different ways.
Envision Math Common Core 8th Grade Answers Topic 3 Use Functions To Model Relationships 10.4
Answer:
The given graphs are:
Envision Math Common Core 8th Grade Answers Topic 3 Use Functions To Model Relationships 10.4
Now,
a)
From the given graph,
We can observe that
For each value of x, there is only 1 value of y
The given graph is a straight line
Now,
The rate of change of the given graph = \(\frac{y}{x}\)
= \(\frac{0 – 10}{1 – 0}\)
= -10
Hence,
The given graph is considered a linear function
b)
From the given table,
To consider a function linear, verify whether the rate of change is constant or not
Now,
For all the values of x,
The rate of change is: 5
For all the values of y,
The rate of change is: 35
So,
The rate of change of the given table = \(\frac{y}{x}\)
= \(\frac{5}{5}\)
= 7
Hence,
The given table is considered a linear function
c)
The given equation is:
y = 10x + 7
Compare the given equation with
y = mx + c
Where,
m is the rate of change
So,
The rate of change for the given equation is: 10
Hence,
The given equation is considered a linear function

PART A
Is each of the proposals represented by linear functions? Explain.
Answer:
Yes, all the proposals are represented by linear functions

PART B
Does the class have any money in the account now? How can you tell?
Answer:
From the given graph,
We can observe that the straight line does not start from 0 but from 10
So,
The initial value of the graph will be: 10
We know that,
The initial value is considered the y-intercept
Hence, from the above,
We can conclude that the class have the money in the account now i.e., $10

PART C
Which fundraising proposal raises money at the fastest rate? Explain.
Answer:
Since the rate of change is the highest for Paul’s proposal,
Paul’s Proposal raises money at the fastest rate

PART D
If Sarah and her classmates are hoping to raise $200, which proposal do you recommend that Sarah and her classmates choose? Explain why you recommend that proposal.
Answer:
It is given that Sarah and her classmates are hoping to raise $200
So,
To raise the money,
We have to choose the plan which has the highest rate of change
Hence, from the above,
We can conclude that Sarah and her classmates choose Paul’s proposal

3-Act Mathematical Modeling: Every Drop Counts

3-ACT MATH

Envision Math Common Core 8th Grade Answers Topic 3 Use Functions To Model Relationships 10.55

АСТ 1

Question 1.
After watching the video, what is the first question that comes to mind?
Answer:
After watching the video,
The first question that comes to mind is:
How much amount of water people waste brushing their teeth?

Question 2.
Write the Main Question you will answer.
Answer:
The main question you will answer is:
How much amount of water people waste brushing their teeth?

Question 3.
Construct Arguments Predict an answer to this Main Question. Explain your prediction.
Envision Math Common Core 8th Grade Answers Topic 3 Use Functions To Model Relationships 10.6
Answer:
The answer to the main question is: 4 gallons
The prediction of the answer for the main question is according to the surveys done by International Organisations

Question 4.
On the number line below, write a number that is too small to be the answer. Write a number that is too large.
Envision Math Common Core 8th Grade Answers Topic 3 Use Functions To Model Relationships 10.7
Answer:
From the above,
We can observe that
The maximum amount (Too large) of water used to brush teeth is: 4 gallons
The minimum amount (Too small) of water used to brush the teeth is: 2 gallons
Hence,
The representation of the amounts of water used to brush teeth in this situation is:

Question 5.
Plot your prediction on the same number line.
Answer:
From the above,
We can observe that there are minimum and maximum amounts of water used to brush the teeth

Now,
Let x be the amount of water used to brush the teeth
So,
The prediction will be: 2 < x < 4
Hence,
The representation of the prediction on the number line is:

ACT 2

Question 6.
What information in this situation would be helpful to know? How would you use that information?
Envision Math Common Core 8th Grade Answers Topic 3 Use Functions To Model Relationships 10.8
Answer:
The information in this situation that would be helpful to know is:
How much time did it take to completely brush your teeth?
From the above information,
We can estimate the amount of water used to brush your teeth

Question 7.
Use Appropriate Tools What tools can you use to solve the problem? Explain how you would use them strategically.
Answer:

Question 8.
Model with Math
Represent the situation using mathematics.
Use your representation to answer the Main Question.
Answer:

Question 9.
What is your answer to the Main Question? Is it higher or lower than your prediction? Explain why.
Envision Math Common Core 8th Grade Answers Topic 3 Use Functions To Model Relationships 11.1
Answer:

ACT 3

Question 10.
Write the answer you saw in the video.
Answer:

Question 11.
Reasoning Does your answer match the answer in the video? If not, what are some reasons that would explain the difference?
Envision Math Common Core 8th Grade Answers Topic 3 Use Functions To Model Relationships 11.3
Answer:

Question 12.
Make Sense and Persevere Would you change your model now that you know the answer? Explain.
Answer:

ACT 3

Reflect

Question 13.
Model with Math
Explain how you used a mathematical model to represent the situation. How did the model help you answer the Main Question?
Envision Math Common Core 8th Grade Answers Topic 3 Use Functions To Model Relationships 11.4
Answer:

Question 14.
Be Precise How do the units you chose and the method you used help you communicate your answer?
Answer;

SEQUEL

Question 15.
Use Structure How much water will he save in a year?
Envision Math Common Core 8th Grade Answers Topic 3 Use Functions To Model Relationships 11.5
Answer:

Lesson 3.4 Construct Functions to Model Linear Relationships

ACTIVITY

Explore It!
Erick wants to buy a new mountain bike that costs $250. He has already saved $120 and plans to save $20 each week from the money he earns for mowing lawns. He thinks he will have saved enough money after seven weeks.
Envision Math Common Core 8th Grade Answers Topic 3 Use Functions To Model Relationships 11.6
Answer:
It is given that
Erick wants to buy a new mountain bike that costs $250. He has already saved $120 and plans to save $20 each week from the money he earns for mowing lawns. He thinks he will have saved enough money after seven weeks.
So,
The total amount he saved = The amount he saved already + The amount he planned to save each week
Let,
The number of weeks —-> x
The total amount he saved —-> y
So,
y = $20x + $120
Compare the above equation with y = mx + c
Where,
m s the rate of change (or) slope
c is the y-intercept
So,
For the given equation,
The rate of change (m) is: 20
The y-intercept is: 120

A. Complete the table. Then graph the data.
Envision Math Common Core 8th Grade Answers Topic 3 Use Functions To Model Relationships 11.7
Answer:
The equation is:
y = $20x + $120
So,
The completed table for the above equation is:

So,
The representation of the given equation in the coordinate plane is:

B. How can you tell that the relationship is a linear function from the table? How can you tell from the graph?
Answer:
We know that,
A relation is said to be a function when an input value matches only with an output value
A function is said to be a linear function when the graph of that function is a straight line
So,
From part (a),
From the table,
We can observe that for each value of the week, there is a different amount saved
So,
From the table,
The given relation is said to be a function
From the graph of the equation,
We can observe that the graph is a straight line
So,
We can say that the function is a linear function

Focus on math practices
Generalize How can the different representations help you determine the properties of functions?
Answer:
The different representations of the functions are:
A) Symbolic or Algebraic representation – The properties can be found out by comparing with the standard form
B) Numerical (Tables) representation – The properties can be found out by the rate of change and the initial values
C) Graphical representation – The properties can be found out by the values of x and y
D) Verbal representation – The properties can be found out by the keywords

? Essential Question
How can you use a function to represent a linear relationship?
Answer:
Another approach to representing linear functions is by using function notation. One example of function notation is an equation written in the form known as the slope-intercept form of a line, where x is the input value, m is the rate of change, and c is the initial value of the dependent variable.

Try It!
How will the height of the ramp change if the plan shows that for every 3 inches of height, the triangle should have a base that is 15 inches long?
Envision Math Common Core 8th Grade Answers Topic 3 Use Functions To Model Relationships 11.8
Graph the function. The slope of the function shown in the graph is Envision Math Common Core 8th Grade Answers Topic 3 Use Functions To Model Relationships 11.9. The equation of the function is y = Envision Math Common Core 8th Grade Answers Topic 3 Use Functions To Model Relationships 11.9
x. If the base length is 110 inches, then the height of the ramp will be Envision Math Common Core 8th Grade Answers Topic 3 Use Functions To Model Relationships 11.9 inches.
Answer:
It is given that
The plan shows that for every 3 inches of height, the triangle should have a base that is 15 inches long
So,
The rate of change (m) = \(\frac{Rise}{Run}\)
m = \(\frac{3}{15}\)
m = \(\frac{1}{5}\)
So,
The slope of the function shown in the graph is: \(\frac{1}{5}\)
Now,
We know that,
The representation of the linear equation is:
y = mx
So,
y = \(\frac{1}{5}\)x
Where,
y is the height of the ramp
x is the base length of the triangle
Now,
For the base length of 110 inches,
y = \(\frac{1}{5}\) (110)
y = 22 inches
Hence, from the above,
We can conclude that the height of the ramp will be 22 inches for the base length of 110 inches

Convince Me!
Explain why the initial value and the y-intercept are equivalent.
Answer:
An equation in slope-intercept form of a line includes the slope and the initial value of the function. The initial value, or y-intercept, is the output value when the input of a linear function is zero. It is the y-value of the point where the line crosses the y-axis.

Try It!
Jin is tracking how much food he feeds his dogs each week. After 2 weeks, he has used 8\(\frac{1}{2}\) cups of dog food. After 5 weeks, he has used 21\(\frac{1}{4}\) cups. Construct a function in the form y = mx + b to represent the amount of dog food used, y, after x weeks.
Answer:
It is given that
Jin is tracking how much food he feeds his dogs each week. After 2 weeks, he has used 8\(\frac{1}{2}\) cups of dog food. After 5 weeks, he has used 21\(\frac{1}{4}\) cups.
Now,
Let x be the number of weeks
Let y be the number of cups of dog food used
So,
For x = 2, y = 8.5 (The value of 8\(\frac{1}{2}\))
For x = 5, y = 21.25 (The value of 21\(\frac{1}{4}\))
We know that,
The equation in the slope-intercept form is:
y = mx + c
So,
8.5 = 2m + c —–> (1)
21.25 = 5m + c ——> (2)
Solve eq (1) and eq (2)
So,
8.5 = 2m + 21.25 – 5m
8.5 – 21.25 = 2m – 5m
-12.75 = -3m
3m = 12.75
m = \(\frac{12.75}{3}\)
m = 4.25
Now,
For the value of c,
Substitute the value of m either in eq (1) or in eq (2)
So,
8.5 = 2m + c
8.5 = 2 (4.25) + c
8.5 – 8.5 = c
c = 0
Hence, from the above,
We can conclude that the representation of the amount of dog used y, after x weeks in the equation form is:
y = 4.25x

Try It!
The graph shows the relationship between the number of pages printed by a printer and the warm-up time before each printing. What function in the form y = mx + b represents this relationship?
Envision Math Common Core 8th Grade Answers Topic 3 Use Functions To Model Relationships 13.1
Answer:
It is given that
The graph shows the relationship between the number of pages printed by a printer and the warm-up time before each printing
Now,
The given graph is:
Envision Math Common Core 8th Grade Answers Topic 3 Use Functions To Model Relationships 13.1
From the given graph,
We can observe that there is an initial value or y-intercept
So,
The y-intercept from the given graph is: 1
Now,
To find the slope from the given graph,
The given points are: (30, 4), and (10, 2)
So,
Slope (m) = \(\frac{2 – 4}{10 – 30}\)
m = \(\frac{-2}{-20}\)
m = \(\frac{1}{10}\)
We know that,
The representation of the equation in the slope-intercept form is:
y = mx + c
So,
y = \(\frac{1}{10}\)x + 1
Hence, from the above,
We can conclude that the equation that represents the given situation is:
y = \(\frac{1}{10}\)x + 1

KEY CONCEPT

A function in the form y= mx + b represents a linear relationship between two quantities, x, and y.
Envision Math Common Core 8th Grade Answers Topic 3 Use Functions To Model Relationships 13.2

Do You Understand?

Question 1.
?Essential Question How can you use a function to represent a linear relationship?
Answer:
Another approach to representing linear functions is by using function notation. One example of function notation is an equation written in the form known as the slope-intercept form of a line, where x is the input value, m is the rate of change, and c is the initial value of the dependent variable.

Question 2.
Make Sense and Persevere Tonya is looking at a graph that shows a line drawn between two points with a slope of -5. One of the points is smudged and she cannot read it. The points as far as she can tell are (3, 5) and (x, 10). What must the value of x be? Explain.
Answer:
It is given that
Tonya is looking at a graph that shows a line drawn between two points with a slope of -5. One of the points is smudged and she cannot read it. The points as far as she can tell are (3, 5) and (x, 10)
Now,
We know that,
Slope (m) = y2 – y1 / x2 – x1
Compare the given points with (x1, y1), (x2, y2)
So,
–5 = \(\frac{10 – 5}{x – 3}\)
-5 = \(\frac{5}{x – 3}\)
-5 (x – 3) = 5
-5 (x) + 5 (3) = 5
-5x + 15 = 5
-5x = 5 – 15
-5x = -10
5x = 10
x = \(\frac{10}{5}\)
x = 2
Hence, from the above,
We can conclude that the value of x is: 2

Question 3.
Reasoning What is the initial value of all linear functions that show a proportional relationship?
Answer:
We know that,
The representation of the proportional relationship is:
y = mx + 0
Where,
m is the slope (or) rate of change
So,
From the above equation,
We can say that the initial value (or) the y-intercept is: 0
Hence, from the above,
We can conclude that the initial value of all linear functions that show a proportional relationship is: 0

Do You Know How?

Question 4.
Write a function in the form y = mx + b for the line that contains the points (-8.3, -5.2) and (6.4, 9.5).
Answer:
The given points are:
(-8.3, -5.2) and (6.4, 9.5)
Now,
We know that,
Slope (m) = y2 – y1 / x2 – x1
So,
Compare the given points with (x1, y1), (x2, y2)
So,
m = \(\frac{9.5 + 5.2}{6.4 + 8.3}\)
m = \(\frac{14.7}{14.7}\)
m = 1
We know that,
The representation of the equation in the slope-intercept form is:
y = mx + c
So,
y = x + c
Now,
To find the value of c,
Substitute any one of the points in the above equation
So,
-5.2 = -8.3 + c
c = 8.3 – 5.2
c = 3.1
Hence, from the above,
We can conclude that the representation of the linear equation for the given points is:
y = x + 3.1

Question 5.
The data in the table below represent a linear relationship. Fill in the missing data.
Envision Math Common Core 8th Grade Answers Topic 3 Use Functions To Model Relationships 14.1
Answer:
It is given that the data in the table represent a linear relationship
So,
For a linear relationship, for each value of x, there is only 1 value of y
Now,
To find the missing data,
Find the rate of change for x and y
For all the values of x,
The rate of change = 20 – 10
= 10
For all the values of y,
The rate of change = 15 – 10
= 5
Hence,
The completed table with the missing data is:

Question 6.
What is an equation that represents the linear function described by the data in Item 5?
Answer:
From the data in Item 5,
Slope (m) = \(\frac{The rate of change of y}{The rate of change of x}\)
m = \(\frac{5}{10}\)
m = \(\frac{1}{2}\)
We know that,
The equation that represents a linear relationship is:
y = mx
So,
y = \(\frac{1}{2}\)x
Hence, from the above,
The equation that represents the linear function described by the data in Item 5 is:
y = \(\frac{1}{2}\)x

Practice & Problem Solving

Question 7.
A line passes through the points (4, 19) and (9, 24). Write a linear function in the form y = mx + b for this line.
Answer:
The given points are:
(4, 19) and (9, 24)
Now,
We know that,
Slope (m) = y2 – y1 / x2 – x1
So,
Compare the given points with (x1, y1), (x2, y2)
So,
m = \(\frac{24 – 19}{9 – 4}\)
m = \(\frac{5}{5}\)
m = 1
We know that,
The representation of the equation in the slope-intercept form is:
y = mx + c
So,
y = x + c
Now,
To find the value of c,
Substitute any one of the points in the above equation
So,
19 = 4 + c
c = 19 – 4
c = 15
Hence, from the above,
We can conclude that the representation of the linear equation for the given points is:
y = x + 15

Question 8.
What is a linear function in the form y = mx + b for the line passing through (4.5, -4.25) with y-intercept 2.5?
Answer:
It is given that
A line passing through (4.5, -4.25) with y-intercept 2.5
We know that,
The y-intercept is the value of y when x = 0
Now,
The given points are:
(4.5, -4.25) and (0, 2.5)
Now,
We know that,
Slope (m) = y2 – y1 / x2 – x1
So,
Compare the given points with (x1, y1), (x2, y2)
So,
m = \(\frac{2.5 + 4.25}{0 – 4.5}\)
m = \(\frac{6.75}{-4.5}\)
m = -1.5
We know that,
The representation of the equation in the slope-intercept form is:
y = mx + c
So,
y = -1.5x + 2.5
Hence, from the above,
We can conclude that the representation of the linear equation for the given points is:
y = -1.5x + 2.5

Question 9.
A car moving at a constant speed passes a timing device at t = 0. After 8 seconds, the car has traveled 840 feet. What linear function in the form y = mx + b represents the distance in feet, d, the car has traveled any number of seconds, t, after passing the timing device?
Answer:
It is given that
A car moving at a constant speed passes a timing device at t = 0. After 8 seconds, the car has traveled 840 feet
We know that,
Speed = \(\frac{Distance}{Time}\)
So,
For a constant speed,
Time is considered as input and distance is considered as the output
It is also given that time will start from (0, 0) i..e, at t = 0
So,
The representation of the equation that passes through the origin is:
y = mx
Where,
m is the rate of change (or) slope
In this situation,
The rate of change = \(\frac{Distance}{Time}\)
So,
y = \(\frac{840}{8}\)x
y = 105x
Hence, from the above,
We can conclude that the linear equation that represents the distance traveled by a car after passing the timing device is:
y = 105x

Question 10.
At time t = 0, water begins to drip out of a pipe into an empty bucket. After 56 minutes, 8 inches of water are in the bucket. What linear function in the form y = mx + b represents the amount of water in inches, w, in the bucket after t minutes?
Answer:
It is given that
At time t = 0, water begins to drip out of a pipe into an empty bucket
Now,
In this situation,
Time is considered as output and the amount of water in the bucket is considered as the input
It is also given that time will start from (0, 0) i..e, at t = 0
So,
The representation of the equation that passes through the origin is:
y = mx
Where,
m is the rate of change (or) slope
In this situation,
The rate of change = \(\frac{Time}{The amount of water}\)
So,
y = \(\frac{56}{8}\)x
y = 7x
Hence, from the above,
We can conclude that the linear equation that represents the amount of water in inches, w, in the bucket after t minutes
y = 7x

Question 11.
The graph of the line represents the cost of renting a kayak. Write a linear function in the form y = mx + b to represent the relationship of the total cost, c, of renting a kayak for t hours.
Envision Math Common Core 8th Grade Answers Topic 3 Use Functions To Model Relationships 16.1
Answer:
It is given that
The graph of the line represents the cost of renting a kayak
Now,
The given graph is:
Envision Math Common Core 8th Grade Answers Topic 3 Use Functions To Model Relationships 16.1
From the given graph,
We can observe that the line does not pass through the origin and it has the initial value i..e, the y-intercept
So,
From the graph,
The y-intercept is: 8
Now,
To find the slope,
The points from the graph are: (2, 12), and (4, 16)
We know that,
Slope (m) = y2 – y1 / x2 – x1
So,
Compare the given points with (x1, y1), (x2, y2)
So,
m = \(\frac{16 – 12}{4 – 2}\)
m = \(\frac{4}{2}\)
m = 2
We know that,
The representation of the equation in the slope-intercept form is:
y = mx + c
So,
y = 2x + 8
Hence, from the above,
We can conclude that the relationship to represent the total cost, c, of renting a kayak for t hours is:
y = 2x + 8

Question 12.
An online clothing company sells custom sweatshirts. The company charges $6.50 for each sweatshirt and a flat fee of $3.99 for shipping.
a. Write a linear function in the form y = mx + b that represents the total cost, y, in dollars, for a single order of x sweatshirts.
Answer:
It is given that
An online clothing company sells custom sweatshirts. The company charges $6.50 for each sweatshirt and a flat fee of $3.99 for shipping.
So,
The total cost of the sweatshirts = The cost of each sweatshirt + The flat fee of the sweatshirt for shipping
Let the number of sweatshirts be x
let the total cost of the sweatshirts be y
So,
y = $3.99 + $6.50x
Now,
We know that,
The linear equation representation in the slope-intercept form is:
y = mx + c
Hence, from the above,
We can conclude that the equation that represents the total cost, y, in dollars, for a single order of x sweatshirts is:
y = $3.99 + 6.50x

b. Describe how the linear function would change if the shipping charge applied to each sweatshirt.
Answer:
From part (a),
We know that,
The equation that represents the total cost, y, in dollars, for a single order of x sweatshirts, is:
y = $3.99 + $6.50x
Where,
$3.99 is the shipping charge for x sweatshirts,
Now,
If the shipping charge applied to each sweatshirt, then
The representation of the linear equation which we obtained in part (a) is:
y = $6.50x + \(\frac{$3.99}{x}\)
Hence, from the above,
We can conclude that the linear function that represents if the shipping charge applied to each sweatshirt is:
y = $6.50x + \(\frac{$3.99}{x}\)

Question 13.
A store sells packages of comic books with a poster.
a. Model with Math Write a linear function in the form y = mx + b that represents the cost, y, of a package containing any number of comic books, x.
b. Construct Arguments Suppose another store sells a similar package, modeled by a linear function with initial value $7.99. Which store has the better deal? Explain.
Envision Math Common Core 8th Grade Answers Topic 3 Use Functions To Model Relationships 16.3
Answer:
a)
It is given that a store seller sells packages of comic books with a poster
Now,
Let x be the number of comics
Let y be the amount obtained by selling comics & poster
So,
For x = 6, y = $12.75
For x = 13, y = $19.75
We know that,
The representation of the linear equation in the slope-intercept form is:
y = mx + c
Where,
m is the slope
c is the y-intercept
Now,
To find the slope,
The points are: (6, 12.75), and (13, 19.75)
Now,
We know that,
Slope (m) = y2 – y1 / x2 – x1
So,
Compare the given points with (x1, y1), (x2, y2)
So,
m = \(\frac{19.75 – 12.75}{13 – 6}\)
m = \(\frac{7}{7}\)
m = 1
So,
y = x + c
Now,
To find the value of c,
Substitute any one of the points in the above equation
So,
12.75 = 6 + c
c = 12.75 – 6
c = 6.75
Hence, from the above,
We can conclude that the representation of the linear equation that represents the cost, y, of a package containing any number of comic books, x is:
y = x + 6.75
b)
It is given that another store sells a similar package, modeled by a linear function with an initial value of $7.99
Now,
From part (a),
The initial value is: $6.75
From the above initial values,
We can observe that
$6.75 < $7.99
Hence, from the above,
We can conclude that another store has the better deal

Question 14.
Higher-Order Thinking Recommendations for safely thawing a frozen turkey are provided on the packaging.
a. What is the thaw rate of the turkey in hours per pound for refrigerator thawing? For cold water thawing?
b. Write a linear function in the form y = mx + b to represent the time, t, in hours it takes to thaw a turkey in the refrigerator as a function of the weight, w, in pounds of the turkey.
Envision Math Common Core 8th Grade Answers Topic 3 Use Functions To Model Relationships 16.4
Answer:
a)
We know that,
The thaw rate is nothing but the rate of change
So,
For refrigerator thawing,
Rate of change = \(\frac{The number of pounds}{Time}\)
= 4 pounds per day
We know that
1 day = 24 hours
So,
Rate of change = \(\frac{4}{24}\)
= \(\frac{1}{6}\) pound per hour
For Cold water thawing,
Rate of change = \(\frac{The number of pounds}{Time}\)
= 1 pound per 30 minutes
We know that
1 hour = 60minutes
So,
Rate of change = \(\frac{1 (2)}{1}\)
= 2 pounds per hour
Hence, from the above,
We can conclude that
The rate of change for refrigerator thawing is: \(\frac{1}{6}\) pounds per hour
The rate of change for cold water thawing is: 2 pounds per hour
b)
We know that,
The representation of the linear equation in the slope-intercept form is:
y = mx + c
So,
For refrigerator thawing,
The representation of the linear equation as a function of weight w in pounds is:
y = \(\frac{1}{6}\)x + c
For the value of c,
Substitute (24, 4) in the above equation [ The time as x in hours and the weight as y]
So,
4 = \(\frac{24}{6}\) + c
4 = 4 + c
c = 0
Hence, from the above,
We can conclude that the representation of the linear equation as a function of weight w in pounds is:
y = \(\frac{1}{6}\)x

Question 15.
Reasoning The graph shows the relationship between the number of cubic yards of mulch ordered and the total cost of the mulch delivered.
a. What is the constant rate of change? What does it represent?
Envision Math Common Core 8th Grade Answers Topic 3 Use Functions To Model Relationships 16.5
Answer:
It is given that
The graph shows the relationship between the number of cubic yards of mulch ordered and the total cost of the mulch delivered.
Now,
The given graph is:
Envision Math Common Core 8th Grade Answers Topic 3 Use Functions To Model Relationships 16.5
From the given graph,
To find the rate of change,
The points are: (20, 450), and (10, 300)
Now,
We know that,
Slope (m) = y2 – y1 / x2 – x1
So,
Compare the given points with (x1, y1), (x2, y2)
So,
m = \(\frac{300 – 450}{10 – 20}\)
m = \(\frac{150}{10}\)
m = 15
Hence, from the above,
We can conclude that the constant rate of change for the given graph is: 15

b. What is the initial value? What might that represent?
Answer:
We know that,
The initial value is nothing but the y-intercept
Hence, from the above,
We can conclude that the initial value is 50 and this value represents the initial cost of the mulch

Assessment Practice

Question 16.
An international food festival charges for admission and for each sample of food. Admission and 3 samples cost $5.75. Admission and 6 samples cost $8.75. Which linear function represents the cost, y, for any number of samples, x?
A. y = x + 2.75
B. y = 3x + 2.75
C. y = x + 3
D. y = 3x + 3
Answer:
It is given that
An international food festival charges for admission and for each sample of food. Admission and 3 samples cost $5.75. Admission and 6 samples cost $8.75
Now,
Let x be the number of samples
Let y be the cost of samples &Admission
So,
For x = 3, y = $5.75
For x = 6, y = $8.75
We know that,
The representation of the linear equation in the slope-intercept form is:
y = mx + c
Where,
m is the slope
c is the y-intercept
Now,
To find the slope,
The points are: (3, 5.75), and (6, 8.75)
Now,
We know that,
Slope (m) = y2 – y1 / x2 – x1
So,
Compare the given points with (x1, y1), (x2, y2)
So,
m = \(\frac{8.75 – 5.75}{6 – 3}\)
m = \(\frac{3}{3}\)
m = 1
So,
y = x + c
Now,
To find the value of c,
Substitute any one of the points in the above equation
So,
5.75 = 3 + c
c = 5.75 – 3
c = 2.75
So,
y = x + 2.75
Hence, from the above,
We can conclude that option A matches the given situation

Question 17.
Some eighth-graders are making muffins for a fundraiser. They have already made 200 muffins and figure they can make 40 muffins in an hour.

PART A
Write a linear function in the form y = mx + b that represents the total number of muffins the students will make, y, and the number of additional hours spent making the muffins, x.
Answer:
It is given that
Some eighth-graders are making muffins for a fundraiser. They have already made 200 muffins and figure they can make 40 muffins in an hour.
Now,
Let x be the number of hours
So,
The total number of muffins students will make = The number of muffins that the students had already made + The number of muffins the students will make in x hours if they made 40 muffins in an hour
y = 40x + 200
We know that,
The representation of the linear equation in the slope-intercept form is:
y = mx + c
Hence, from the above,
We can conclude that
A linear function that represents the total number of muffins the students will make, y, and the number of additional hours spent making the muffins, x is:
y = 40x + 200

PART B
How many additional hours would the students spend to make 640 muffins?
Answer:
From part (a),
A linear function that represents the total number of muffins the students will make, y, and the number of additional hours spent making the muffins, x is:
y = 40x + 200
It is given that the number of muffins students made is: 640
So,
y = 640
So,
640 = 40x + 200
40x = 640 – 200
40x = 440
4x = 44
x = \(\frac{44}{4}\)
x = 11 hours
Hence, from the above,
We can conclude that the additional hours the students would spend to make 640 muffins is: 11 hours

Lesson 3.5 Intervals of Increase and Decrease

Solve & Discuss It!

Martin will ride his bike from his house to his aunt’s house. He has two different routes he can take. One route goes up and down a hill. The other route avoids the hill by going around the edge of the hill. How do you think the routes will differ? What do you think about the relationship between speed and time?
Envision Math Common Core Grade 8 Answers Topic 3 Use Functions To Model Relationships 20.1
Answer:
It is given that
Martin will ride his bike from his house to his aunt’s house. He has two different routes he can take. One route goes up and down a hill. The other route avoids the hill by going around the edge of the hill.
We know that,
Speed = \(\frac{Distance}{Time}\)
In this situation,
The distance is constant for the 2 routes
So,
As speed increases, the time decreases
Now,
For route 1:
Route 1 is divided into 2 parts
For the first part (Going up),
As there is friction,
The speed decreases as time increases
For the second part (Going down),
The speed increases as time decreases
For route 2:
Route 2 is divided into 2 parts
For the first part (Going down),
The speed increases as time decreases
For the second part,
The speed is constant as time increases

Focus on math practices
Reasoning How do the characteristics of each route affect Martin’s travel time and speed?
Answer:
We know that,
For the constant distance,
Speed ∝ \(\frac{1}{Time}\)
Now,
Route 1 is in the shape of a parabola
In route 1,
For the first part,
The speed increases as time decreases
For the second part,
The speed decreases as time increases
Now,
Route 2 is increasing for some time and later becomes constant
In route 2,
For the first part,
The speed increases as time decreases
For the second part,
The speed is constant as time increases

? Essential Question
How does a qualitative graph describe the relationship between quantities?
Answer:
The formal term to describe a straight-line graph is linear, whether or not it goes through the origin, and the relationship between the two variables is called a linear relationship. Similarly, the relationship shown by a curved graph is called non-linear.

Try It!
The graph at the right shows another interval in the train’s travel. Which best describes the behavior of the train in the interval shown?
As time Envision Math Common Core Grade 8 Answers Topic 3 Use Functions To Model Relationships 20.2, the speed of the train
Envision Math Common Core Grade 8 Answers Topic 3 Use Functions To Model Relationships 20.3
The function is Envision Math Common Core Grade 8 Answers Topic 3 Use Functions To Model Relationships 20.2
Answer:
It is given that the graph shows another interval in the train’s travel
Now,
The given graph is:
Envision Math Common Core Grade 8 Answers Topic 3 Use Functions To Model Relationships 20.3
From the given graph,
As speed decreases, the time increases
Hence,
The function of the given graph is decreasing in nature

Convince Me!
How would the graph of the function change if the speed of the train was increasing?
Answer:
We know that,
If speed increases, then the time decreases for a constant distance
So,
For a constant distance,
Speed ∝ \(\frac{1}{Time}\)
Hence,
The graph of the function is decreasing in nature as the speed of the train is increased

Try It!
Write a scenario that the graph above could represent. (Example 3)
Answer:
The scenario that the given graph could represent is:
The traveling of a vehicle on a hill

KEY CONCEPT
You can describe the relationship between two quantities by analyzing the behavior of the function relating the quantities in different intervals on a graph.
Envision Math Common Core Grade 8 Answers Topic 3 Use Functions To Model Relationships 20.4

Do You Understand?

Question 1.
Essential Question How does a qualitative graph describe the relationship between quantities?
Answer:
The formal term to describe a straight-line graph is linear, whether or not it goes through the origin, and the relationship between the two variables is called a linear relationship. Similarly, the relationship shown by a curved graph is called non-linear.

Question 2.
Look for Relationships How would knowing the slope of a linear function help determine whether a function is increasing or decreasing?
Answer:
The graph of an increasing function has a positive slope. A line with a positive slope slants upward from left to right. For a decreasing function, the slope is negative. The output values decrease as the input values increase.

Question 3.
Use Structure What kind of graph of a function shows the same output values, or y-values, for each input value, or x-value?
Answer:
The vertical line test can be used to determine whether a graph represents a function. A vertical line includes all points with a particular x value. The y value of a point where a vertical line intersects a graph represents an output for that input x value.

Do You Know How?

Question 4.
What does the graph of the function at each interval represent?
Envision Math Common Core Grade 8 Answers Topic 3 Use Functions To Model Relationships 30.1
Answer:
The given graph is:
Envision Math Common Core Grade 8 Answers Topic 3 Use Functions To Model Relationships 30.1
Now,
The given graph is divided into 2 intervals
In the first interval,
The height increases with the increase of time
In the second interval,
The height decreases with the increase of time

Question 5.
In which intervals is the function increasing, decreasing, or constant?
Envision Math Common Core Grade 8 Answers Topic 3 Use Functions To Model Relationships 30.2
Answer:
The given graph is:
Envision Math Common Core Grade 8 Answers Topic 3 Use Functions To Model Relationships 30.2
We know that,
The graph is said to be increasing in nature when the line moves from left to right or the slope is positive
The graph is said to be decreasing in nature when the line moves from right to left or the slope is negative
The graph is said to be constant in nature when the line is parallel to any one of the coordinate axes
Hence, from the above,
We can conclude that
The intervals that the function is constant – 1, 5
The intervals that the function is increasing – 3, 4
The intervals that the function is decreasing – 2, 6

Practice & Problem Solving

Question 6.
Use the graph to complete the statements.
Envision Math Common Core Grade 8 Answers Topic 3 Use Functions To Model Relationships 30.3
The function is Envision Math Common Core Grade 8 Answers Topic 3 Use Functions To Model Relationships 30.4 in intervals 1, 3, and 6.
The function is Envision Math Common Core Grade 8 Answers Topic 3 Use Functions To Model Relationships 30.4 in intervals 2 and 5.
The function is constant in interval Envision Math Common Core Grade 8 Answers Topic 3 Use Functions To Model Relationships 30.5
Answer:
The given graph is:
Envision Math Common Core Grade 8 Answers Topic 3 Use Functions To Model Relationships 30.3
We know that,
The graph is said to be increasing in nature when the line moves from left to right or the slope is positive
The graph is said to be decreasing in nature when the line moves from right to left or the slope is negative
The graph is said to be constant in nature when the line is parallel to any one of the coordinate axes
Hence, from the above,
We can conclude that
The intervals that the function is constant – 4
The intervals that the function is increasing – 1, 3, 6
The intervals that the function is decreasing – 2, 5

Question 7.
The graph below shows the temperature in Paula’s house over time after her mother turned on the air conditioner. Describe the relationship between the two quantities.
Envision Math Common Core Grade 8 Answers Topic 3 Use Functions To Model Relationships 30.44
Answer:
It is given that
The graph below shows the temperature in Paula’s house over time after her mother turned on the air conditioner
Now,
The given graph is:
Envision Math Common Core Grade 8 Answers Topic 3 Use Functions To Model Relationships 30.44
From the given graph,
We can observe that
As time increases, the temperature decreases
Hence, from the above,
We can conclude that
Temperature (°F) ∝ \(\frac{1}{Time}\)

Question 8.
You have a device that monitors the voltage across a lamp over time. The results are shown in the graph. Describe the behavior of the function in each interval.
In interval (a), the function is Envision Math Common Core Grade 8 Answers Topic 3 Use Functions To Model Relationships 30.66
Envision Math Common Core Grade 8 Answers Topic 3 Use Functions To Model Relationships 30.55
In the interval (b), the function is Envision Math Common Core Grade 8 Answers Topic 3 Use Functions To Model Relationships 30.66
In the interval (c), the function is Envision Math Common Core Grade 8 Answers Topic 3 Use Functions To Model Relationships 30.66
In interval (d), the function is Envision Math Common Core Grade 8 Answers Topic 3 Use Functions To Model Relationships 30.66
Answer:
It is given that
You have a device that monitors the voltage across a lamp over time. The results are shown in the graph
Now,
The given graph is:
Envision Math Common Core Grade 8 Answers Topic 3 Use Functions To Model Relationships 30.55
We know that,
The graph is said to be increasing in nature when the line moves from left to right or the slope is positive
The graph is said to be decreasing in nature when the line moves from right to left or the slope is negative
The graph is said to be constant in nature when the line is parallel to any one of the coordinate axes
Hence, from the above,
We can conclude that
In interval (a), the function is increasing
In the Interval (b), the function is constant
In the interval (c), the function is decreasing
In the interval (d), the function is constant

Question 9.
The graph below shows the height of a roller coaster over time during a single ride. Circle the intervals in which the function is increasing. In which interval is the increase the greatest?
Envision Math Common Core Grade 8 Answers Topic 3 Use Functions To Model Relationships 30.6
Answer:
It is given that
The graph below shows the height of a roller coaster over time during a single ride.
Now,
The given graph is:
Envision Math Common Core Grade 8 Answers Topic 3 Use Functions To Model Relationships 30.6
We know that,
The graph is said to be increasing in nature when the line moves from left to right or the slope is positive
The graph is said to be decreasing in nature when the line moves from right to left or the slope is negative
The graph is said to be constant in nature when the line is parallel to any one of the coordinate axes
Now,
The given graph with the intervals is:

So,
From the given graph,
The intervals which are increasing in nature are: 1, 3, 5
Hence, from the above,
We can conclude that the increase is the greatest in the 1st interval

Question 10.
Reasoning The graph shows the speed of a car over time. What might the constant intervals in the function represent?
Envision Math Common Core Grade 8 Answers Topic 3 Use Functions To Model Relationships 30.7
Answer:
It is given that
The graph shows the speed of a car over time
Now,
The given graph is:
Envision Math Common Core Grade 8 Answers Topic 3 Use Functions To Model Relationships 30.7
We know that,
The graph is said to be increasing in nature when the line moves from left to right or the slope is positive
The graph is said to be decreasing in nature when the line moves from right to left or the slope is negative
The graph is said to be constant in nature when the line is parallel to any one of the coordinate axes
We know that,
Speed = \(\frac{Distance}{Time}\)
Now,
From the given graph,
We can observe that
The constant lines represent that the speed becomes constant even though the time is increasing
Hence, from the above,
We can conclude that the constant lines in the given graph represent the constant speed

Question 11.
Higher-Order Thinking A signal generator is used to generate signals for a lab experiment over time. The graph shows the frequency of the signal generated.
a. In how many intervals is the function decreasing?
b. How are the decreasing intervals alike?
Envision Math Common Core Grade 8 Answers Topic 3 Use Functions To Model Relationships 30.8
Answer:
It is given that
A signal generator is used to generate signals for a lab experiment over time. The graph shows the frequency of the signal generated.
Now,
The given graph is:
Envision Math Common Core Grade 8 Answers Topic 3 Use Functions To Model Relationships 30.8
We know that,
The graph is said to be increasing in nature when the line moves from left to right or the slope is positive
The graph is said to be decreasing in nature when the line moves from right to left or the slope is negative
The graph is said to be constant in nature when the line is parallel to any one of the coordinate axes
Now,
The given graph with the marked intervals are:

a)
The intervals that the function is decreasing are: 3, 7, 11
b)
The decreasing intervals are all decreasing in nature and have a negative slope

c. How are the decreasing intervals different?
Answer:
The sizes of the decreasing intervals are different
So,
The values of the negative slopes for decreasing intervals will also be different

Question 12.
Critique Reasoning The graph shows the speed of a person riding his stationary exercise bicycle over time. a. A student claims that the function is constant in two intervals. Do you agree? Explain.
Envision Math Common Core Grade 8 Answers Topic 3 Use Functions To Model Relationships 30.9
Answer:
It is given that
The graph shows the speed of a person riding his stationary exercise bicycle over time. a. A student claims that the function is constant in two intervals.
We know that,
The graph is said to be increasing in nature when the line moves from left to right or the slope is positive
The graph is said to be decreasing in nature when the line moves from right to left or the slope is negative
The graph is said to be constant in nature when the line is parallel to any one of the coordinate axes
So,
From the given graph,
We can observe that
The number of intervals that are constant in nature is: 4
Hence, from the above,
We can conclude that the claim of the student is not correct

b. What error might the student have made?
Answer:
The student considered only the highest constant intervals but not the lowest intervals
The student also did not consider the last constant interval due to the misconception that it is decreasing in nature but it is constant after that decreasing in nature
So,
The above are the errors the student has made

Question 13.
Look for Relationships The graph shows the speed of a roller coaster over time. Describe the relationship of speed as a function of time.
Envision Math Common Core Grade 8 Answers Topic 3 Use Functions To Model Relationships 40.1
Answer:
It is given that
The graph shows the speed of a roller coaster over time. Describe the relationship of speed as a function of time.
Now,
The given graph is:
Envision Math Common Core Grade 8 Answers Topic 3 Use Functions To Model Relationships 40.1
From the given graph,
We can observe that
First, the speed of a roller coaster increases with time, and then it fluctuates between increasing and decreasing as time increases further, and at last, the speed of the roller coaster decreases

Assessment Practice

Question 14.
Which statements about the graph are true? Select all that apply.
Envision Math Common Core Grade 8 Answers Topic 3 Use Functions To Model Relationships 40.2 The graph is decreasing in intervals (1) and (4).
Envision Math Common Core Grade 8 Answers Topic 3 Use Functions To Model Relationships 40.2 The graph shows a constant function in interval (2).
Envision Math Common Core Grade 8 Answers Topic 3 Use Functions To Model Relationships 40.2 The graph is increasing in intervals (2) and (4).
Envision Math Common Core Grade 8 Answers Topic 3 Use Functions To Model Relationships 40.2 The graph has a constant rate of change.
Envision Math Common Core Grade 8 Answers Topic 3 Use Functions To Model Relationships 40.2 The graph shows a constant function in interval (3).
Envision Math Common Core Grade 8 Answers Topic 3 Use Functions To Model Relationships 40.3
Answer:
Let the given options be named as A, B, C, D, and E respectively
Now,
The given graph is:
Envision Math Common Core Grade 8 Answers Topic 3 Use Functions To Model Relationships 40.3
We know that,
The graph is said to be increasing in nature when the line moves from left to right or the slope is positive
The graph is said to be decreasing in nature when the line moves from right to left or the slope is negative
The graph is said to be constant in nature when the line is parallel to any one of the coordinate axes
Hence, from the above,
We can conclude that the options that match with the given situation are: A and E

Lesson 3.6 Sketch Functions from Verbal Descriptions

ACTIVITY

Explain It!

The Environmental Club is learning about oil consumption and energy conservation around the world. Jack says oil consumption in the United States has dropped a lot. Ashley says oil consumption in China is the biggest problem facing the world environment.

Envision Math Common Core Grade 8 Answers Topic 3 Use Functions To Model Relationships 40.4

A. Do you agree or disagree with Jack’s statement? Construct an argument based on the graph to support your position.
Answer:
The given graph is:
Envision Math Common Core Grade 8 Answers Topic 3 Use Functions To Model Relationships 40.4
Now,
The statement of Jack is:
Oil consumption in the United States has dropped a lot
Now,
From the given graph,
We can observe that the oil consumption of the United States (The first line in the graph) is constant from 2000 to 2003 and the consumption is constant from 2004 to 2008 and the consumption decreased abruptly from 2008 to 2011
Hence, from the above,
We can conclude that we can agree with Jack’s statement

B. Do you agree or disagree with Ashley’s statement? Construct an argument based on the graph to support your position.
Answer:
The given graph is:
Envision Math Common Core Grade 8 Answers Topic 3 Use Functions To Model Relationships 40.4
Now,
The statement of Ashley is:
Oil consumption in China is the biggest problem facing the world environment.
Now,
From the given graph,
We can observe that the oil consumption of China (The third line in the graph) is constant from 2000 to 2004 and the consumption is increased abruptly from 2004 to 2008 and the consumption increased minimally from 2008 to 2011
Hence, from the above,
We can conclude that we can agree with Ashley’s statement since China’s consumption of oil will make the other countries suffer

Focus on math practices
Look for Relationships What trend do you see in oil consumption in the United States and in Europe?
Answer:
The given graph is:
Envision Math Common Core Grade 8 Answers Topic 3 Use Functions To Model Relationships 40.4
From the given graph,
We can observe that
The trend in the oil consumption of the United States and Europe (The first line and the second line in the graph) is:
a) The consumption increases gradually for both the United States and Europe from 2000 to 2003
b) From 2004 to 2008, the oil consumption of the United States becomes constant but the consumption of Europe increases gradually
c) From 2009 to 2011, the oil consumption of the United States and Europe decreases gradually

? Essential Question
How does the sketch of a graph of a function help describe its behavior?
Envision Math Common Core Grade 8 Answers Topic 3 Use Functions To Model Relationships 40.5

Answer:
From the graph,
We can observe that,
At t = 0,
The level of oxygen is full
At t = 22 minutes,,
The level of oxygen decreases
Hence, from the above,
We can conclude that
The behavior of the time and the oxygen level is:
Time ∝ \(\frac{1}{Oxygen level}\)

Try It!

The weight of the water exerts pressure on a diver. At a depth of 10 feet, the water pressure is 19.1 pounds per square inch (psi) and at a depth of 14 feet, the water pressure is 20.9 psi. Complete the statements, and then sketch the qualitative graph of this function.
Envision Math Common Core Grade 8 Answers Topic 3 Use Functions To Model Relationships 40.6
The input, or x-variable, is Envision Math Common Core Grade 8 Answers Topic 3 Use Functions To Model Relationships 40.7
The output, or y-variable, is Envision Math Common Core Grade 8 Answers Topic 3 Use Functions To Model Relationships 40.7
Answer:
It is given that
The weight of the water exerts pressure on a diver. At a depth of 10 feet, the water pressure is 19.1 pounds per square inch (psi) and at a depth of 14 feet, the water pressure is 20.9 psi.
So,
From the above statements,
We can observe that
As the depth of the water increases, the water pressure also increases
So,
The input or x-variable for the given situation is: Depth
The output or y-variable for the given situation is: Water pressure
Now,
To draw the graph,
The required points are: (10, 19.1), and (14, 20.9)
We know that,
the equation of the straight line in the slope-intercept form is:
y = mx + c
Now,
We know that,
Slope (m) = y2 – y1 / x2 – x1
So,
Compare the given points with (x1, y1), (x2, y2)
So,
m = \(\frac{20.9 – 19.1}{14 – 10}\)
m = \(\frac{1.8}{4}\)
m = \(\frac{9}{20}\)
So,
y = \(\frac{9}{20}\)x + c
Now,
To find the value of c,
Substitute any one of the points in the above equation
So,
19.1 = \(\frac{9}{20}\) (10) + c
c = 19.1 – 4.5
c = 14.6
So,
y = \(\frac{9}{20}\)x + 14.6
Hence,
The representation of the above equation in the coordinate plane is:

Convince Me!
Generalize How are the sketches of the two functions similar? How are they different?
Answer:
From Example 1 and the above item’s graph,
We can observe that both graphs have the equation in the slope-intercept form
But,
The graph in Example 1 is decreasing in nature
The graph in the above item is increasing in nature

Try It!
Haru rides his bike from his home for 30 minutes at a fast pace. He stops to rest for 20 minutes and then continues in the same direction at a slower pace for 30 more minutes. Sketch a graph of the relationship of Haru’s distance from home over time.
Envision Math Common Core Grade 8 Answers Topic 3 Use Functions To Model Relationships 40.77
Answer:
It is given that
Haru rides his bike from his home for 30 minutes at a fast pace. He stops to rest for 20 minutes and then continues in the same direction at a slower pace for 30 more minutes.
Hence,
The graph of the relationship of Haru’s distance from the house over time is:

KEY CONCEPT

You can sketch a graph of a function to describe its behavior. When sketching a function, follow these steps:
1. Identify the two variables (input, output) that have a relationship.
2. Analyze the situation. Look for keywords that indicate that the function is increasing, decreasing, or constant.
3. Sketch the graph.
Envision Math Common Core Grade 8 Answers Topic 3 Use Functions To Model Relationships 40.8

Do You Understand?

Question 1.
? Essential Question How does the sketch of a graph of a function help describe its behavior?

Envision Math Common Core Grade 8 Answers Topic 3 Use Functions To Model Relationships 40.5

Answer:
From the graph,
We can observe that,
At t = 0,
The level of oxygen is full
At t = 22 minutes,,
The level of oxygen decreases
Hence, from the above,
We can conclude that
The behavior of the time and the oxygen level is:
Time ∝ \(\frac{1}{Oxygen level}\)

Question 2.
Make Sense and Persevere How do you know which variable goes with which axis when you graph?
Answer:
The independent variable belongs on the x-axis (horizontal line) of the graph and the dependent variable belongs on the y-axis (vertical line).

Question 3.
Reasoning How can you determine the shape of a graph?
Answer:
The four ways to describe shape are whether it is symmetric, how many peaks it has if it is skewed to the left or right, and whether it is uniform. A graph with a single peak is called unimodal. A single peak over the center is called bell-shaped. And, a graph with two peaks is called bimodal.

Do You Know How?

Question 4.
A class plants a tree. Sketch the graph of the height of the tree over time.
Envision Math Common Core Grade 8 Answers Topic 3 Use Functions To Model Relationships 70.1
a. Identify the two variables.
Answer:
It is given that a class plants a tree
Now,
From the given figure,
We can observe that
As time increases, the height of the tree increases
Hence, from the above,
We can conclude that
Input variable (or) x-coordinate: Time
Output variable (or) y-coordinate: Height of the tree

b. How can you describe the relationship between the two variables?
Answer:
From part (a),
We can observe that
As time increases, the height of the tree increases
Hence, from the above,
We can conclude that the relationship between the variables is:
Time ∝ Height of the tree

c. Sketch the graph.
Envision Math Common Core Grade 8 Answers Topic 3 Use Functions To Model Relationships 70.2
Answer:
To draw the graph,
The required points are: (0, 3), and (3, 7)
We know that,
the equation of the straight line in the slope-intercept form is:
y = mx + c
Now,
We know that,
Slope (m) = y2 – y1 / x2 – x1
So,
Compare the given points with (x1, y1), (x2, y2)
So,
m = \(\frac{7 – 3}{3 – 0}\)
m = \(\frac{4}{3}\)
So,
y = \(\frac{4}{3}\)x + c
Now,
To find the value of c,
Substitute any one of the points in the above equation
So,
3 = \(\frac{4}{3}\) (0) + c
c = 3 – 0
c = 3
So,
y = \(\frac{4}{3}\)x + 3
Hence,
The representation of the above equation in the coordinate plane is:

Question 5.
An airplane takes 15 minutes to reach its cruising altitude. The plane cruises at that altitude for 90 minutes, and then descend for 20 minutes before it lands. Sketch the graph of the height of the plane over time.
Envision Math Common Core Grade 8 Answers Topic 3 Use Functions To Model Relationships 70.3
Answer:
It is given that
An airplane takes 15 minutes to reach its cruising altitude. The plane cruises at that altitude for 90 minutes, and then descend for 20 minutes before it lands.
Hence,
The graph of the height of the plane over time is (Height of the plane is any value) is:

Practice & Problem Solving

Question 6.
What relationship between money (in dollars) and time (in months) does this graph show? Write a description of the given graph.
Envision Math Common Core Grade 8 Answers Topic 3 Use Functions To Model Relationships 70.4
Answer:
The given graph is:
Envision Math Common Core Grade 8 Answers Topic 3 Use Functions To Model Relationships 70.4
From the given graph,
We can observe that
The amount of money is in an increasing trend for the first half of the month
The amount of money is in a decreasing trend for the second half of the month

Question 7.
When a new laptop became available in a store, the number sold in the first week was high. Sales decreased over the next two weeks and then they remained steady over the next two weeks. The following week, the total number sold by the store increased slightly. Sketch the graph that represents this function over the six weeks.
Envision Math Common Core Grade 8 Answers Topic 3 Use Functions To Model Relationships 70.5
Answer:
It is given that
When a new laptop became available in a store, the number sold in the first week was high. Sales decreased over the next two weeks and then they remained steady over the next two weeks. The following week, the total number sold by the store increased slightly
Hence,
The graph which represents the given function over the six weeks is:

Question 8.
Aaron’s mother drives to the gas station and fills up her tank. Then she drives to the market. Sketch the graph that shows the relationship between the amount of fuel in the gas tank of her car and time.
Envision Math Common Core Grade 8 Answers Topic 3 Use Functions To Model Relationships 70.6
Answer:
It is given that
Aaron’s mother drives to the gas station and fills up her tank. Then she drives to the market.
So,
At first, the amount of fuel in the gas tank will be of some initial value and it is in an increasing trend after filling up the tank
After that, when she drives to the market, the amount of fuel will be in a decreasing trend over time
Hence,
The graph that shows the relationship between the amount of fuel in the gas tank of her car and time is:

Question 9.
Melody starts at her house and rides her bike for 10 minutes to a friend’s house. She stays at her friend’s house for 60 minutes. Sketch a graph that represents this description.
Envision Math Common Core Grade 8 Answers Topic 3 Use Functions To Model Relationships 70.7
Answer:
It is given that
Melody starts at her house and rides her bike for 10 minutes to a friend’s house. She stays at her friend’s house for 60 minutes
Hence,
The graph that represents the above description is:

Question 10.
Which description best represents the graph shown?
A. People are waiting for a train. A train comes and some people get on. The other people wait for the next train. As time goes by, people gradually leave the station.
B. One train arrives and some people get off the train and wait in the station.
C. People are waiting for a train. Everyone gets on the first train that comes.
D. People are waiting for a train. A train comes and some people get on the train. The other people wait for the next train. Another train arrives and all of the remaining people get on.
Envision Math Common Core Grade 8 Answers Topic 3 Use Functions To Model Relationships 70.8
Answer:
The given graph is:
Envision Math Common Core Grade 8 Answers Topic 3 Use Functions To Model Relationships 70.8
From the given graph,
We can observe that
At first, there are some people and after some time, the number of people decreased
After some time, there are some people other than the people that decreased and after some further time, the total number of people becomes zero
Hence, from the above,
We can conclude that option D matches the above description

Question 11.
A baker has already made 10 cakes. She can make the same number of cakes each hour, which she does for 5 hours. Sketch the graph of the relationship between the number of cakes made and time.
Envision Math Common Core 8th Grade Answer Key Topic 3 Use Functions To Model Relationships 70.9
Answer:
It is given that
A baker has already made 10 cakes. She can make the same number of cakes each hour, which she does for 5 hours.
So,
The initial value for the given graph is: 10
Hence,
The graph of the relationship between the number of cakes made and time is:

Question 12.
Model with Math An air cannon launches a T-shirt upward toward basketball fans. It reaches a maximum height and then descends for a couple seconds until a fan grabs it. Sketch the graph that represents this situation.
Envision Math Common Core 8th Grade Answer Key Topic 3 Use Functions To Model Relationships 70.10
Answer:
It is given that
An air cannon launches a T-shirt upward toward basketball fans. It reaches a maximum height and then descends for a couple of seconds until a fan grabs it
We know that,
The projectile (Graph) of an air cannon is always like a “Parabola”
Hence,
The graph that represents the given situation is:

Question 13.
Higher-Order Thinking Write a verbal description of how these two variables are related. The description must suggest at least two intervals. Sketch the graph that represents the verbal description.
Envision Math Common Core 8th Grade Answer Key Topic 3 Use Functions To Model Relationships 70.11
Answer:
The verbal description of the relationship between the total people and time in hours is:
People are waiting for a train. A train comes and some people get on the train. The other people wait for the next train. Another train arrives and all of the remaining people get on.
Hence,
The graph that describes the above verbal description is:

Assessment Practice

Question 14.
A baseball team scores the same number of runs in each of the first 4 innings. After that, the team did not score a run for the rest of the game, which lasts 9 innings. Let x represent the innings of the game, and y represent the total number of runs.
PART A
Sketch the graph of this situation below.
Envision Math Common Core 8th Grade Answer Key Topic 3 Use Functions To Model Relationships 70.12
Answer:
It is given that
A baseball team scores the same number of runs in each of the first 4 innings. After that, the team did not score a run for the rest of the game, which lasts 9 innings.
Hence,
The graph for the above situation is:

PART B
How would the graph change if the innings in which the team scores runs changes?
Answer:
If the innings in which the team scores runs change, then
The graph will be either in an increasing trend or in a decreasing trend

Topic 3 REVIEW

? Topic Essential Question

How can you use functions to model linear relationships?
Answer:
Another approach to representing linear functions is by using function notation. One example of function notation is an equation written in the form known as the slope-intercept form of a line, where x is the input value, m is the rate of change, and b is the initial value of the dependent variable.

Vocabulary Review

Match each vocabulary term with its definition.
Envision Math Common Core 8th Grade Answer Key Topic 3 Use Functions To Model Relationships 82.1
Answer:
Each vocabulary term with its definition is:

Use Vocabulary in Writing

Explain how to write a linear function in the form y = mx + b by using the two points given below. Use vocabulary words in your explanation. (0, -2), (2, 6)
Answer:
The given points are:
(0, -2) and (2, 6)
Now,
We know that,
Slope (m) = y2 – y1 / x2 – x1
So,
Compare the given points with (x1, y1), (x2, y2)
So,
m = \(\frac{6 + 2}{2 – 0}\)
m = \(\frac{8}{2}\)
m = 4
We know that,
The representation of the equation in the slope-intercept form is:
y = mx + c
So,
y = 4x + c
Now,
To find the value of c,
Substitute any one of the points in the above equation
So,
-2 = 0 + c
c = -2 – 0
c = -2
Hence, from the above,
We can conclude that the representation of the linear equation for the given points is:
y = 4x – 2

Concepts and Skills Review

LESSON 3.1 Understand Relations and Functions

Quick Review
A relation is a set of ordered pairs. A relation is a function if each input, or x-value, has exactly one unique output, or y-value.

Practice

Question 1.
Is the relation shown in the table a function? Explain.
Envision Math Common Core 8th Grade Answer Key Topic 3 Use Functions To Model Relationships 85.1
Answer:
The given relation is:
Envision Math Common Core 8th Grade Answer Key Topic 3 Use Functions To Model Relationships 85.1
We know that,
A relation is said to be a function if each input has only 1 output
So,
From the given relation,
We can observe that each input has only 1 output
Hence, from the above,
We can conclude that the given relation is a function

Question 2.
Does the relation {(-5, -3), (7, 2), (3,8), (3, -8), (5, 10)} represent a function? Use the arrow diagram. Then explain your answer.
Envision Math Common Core 8th Grade Answer Key Topic 3 Use Functions To Model Relationships 85.2
Answer:
The given ordered pairs are:
(-5, -3), (7, 2), (3,8), (3, -8), (5, 10)
We know that,
The ordered pairs are in the form of (input (x), output (y))
In the arrow diagram,
The left side represents the values of the input
The right side represents the values of the output
So,
The representation of the ordered pairs in the arrow diagram is:

We know that,
A relation is said to be a function only when each input corresponds with only 1 output
Now,
From the arrow diagram,
We can observe that
The same input corresponds with 2 outputs
Hence, from the above,
We can conclude that the given relation is not a function

LESSON 3.2 Connect Representations of Functions

Quick Review
You can represent a function in a table, in a graph, or as an equation. The graph of a linear function is a straight line.

Practice
Mark has a $100 gift card to buy apps for his smartphone. Each week, he buys one new app for $4.99.
1. Write an equation that relates the amount left on the card, y, over time, x.
Answer:
It is given that
Mark has a $100 gift card to buy apps for his smartphone. Each week, he buys one new app for $4.99.
Now,
Let y be the amount left on the card
Let x be the number of weeks
So,
The amount left on the card = The total amount of the card – The amount of money he used to buy a new app for x weeks
y = $100 – $4.99x
Hence, from the above,
We can conclude that
The equation that relates the amount left on the card, y, over time, x is:
y = $100 – $4.99x

2. Make a graph of the function.
Envision Math Common Core 8th Grade Answer Key Topic 3 Use Functions To Model Relationships 85.3
Answer:
From part (a),
The equation that relates the amount left on the card, y, over time, x is:
y = $100 – $4.99x
Hence,
The representation of the above equation in the coordinate plane is:

LESSON 3.3 Compare Linear and Nonlinear Functions

Quick Review
You can compare functions in different representations by looking at the properties of functions: the constant rate of change and the initial value.

Practice
Two linear functions are shown.
Envision Math Common Core 8th Grade Answer Key Topic 3 Use Functions To Model Relationships 90.1
Answer:
The given linear functions are:
Envision Math Common Core 8th Grade Answer Key Topic 3 Use Functions To Model Relationships 90.1
Function A:
Compare the given equation with
y = mx + c
Where,
m is the rate of change (or) slope
c is the initial value (or) y-intercept
So,
From the above equation,
The rate of change is: -3
The initial value is: 2
Function B:
From the given table,
We can observe that
The rate of change for all the values of x is: 1
The rate of change for all the values of y is: 2
So,
The rate of change for the table = \(\frac{y}{x}\)
= \(\frac{2}{1}\)
= 2
We know that,
The initial value is the value of y when x = 0
So,
For the given table,
The initial value is: 0
The rate of change is: 2

Question 1.
Which function has the greater initial value? Explain.
Answer:
Function A has the greater initial value

Question 2.
Which function has the greater rate of change?
Answer:
Function B has the greater rate of change

LESSON 3.4 Construct Functions to Model Linear Relationships

Quick Review
A function in the form y = mx + b represents a linear relationship between two quantities, x and y, where m represents the constant rate of change and b represents the initial value.

Practice

Question 1.
What is the equation of a line that passes through (0.5, 4.25) and (2, 18.5) and has a y-intercept of -0.5?
Answer:
It is given that
A line passing through (0.5, 4.25), and (2, 18.5) and has a y-intercept of  -0.5
We know that,
The y-intercept is the value of y when x = 0
Now,
The given points are:
(0.5, 4.25) and (2, 18.5)
Now,
We know that,
Slope (m) = y2 – y1 / x2 – x1
So,
Compare the given points with (x1, y1), (x2, y2)
So,
m = \(\frac{18.5 – 4.25}{2 – 0.5}\)
m = \(\frac{14.25}{1.5}\)
m = 9.5
We know that,
The representation of the equation in the slope-intercept form is:
y = mx + c
So,
y = 9.5x – 0.5
Hence, from the above,
We can conclude that the representation of the linear equation for the given points is:
y = 9.5x – 0.5

Question 2.
The graph shows the relationship of the number of gallons being drained from an aquarium over time. What function models the relationship?
Envision Math Common Core 8th Grade Answer Key Topic 3 Use Functions To Model Relationships 95.1
Answer:
It is given that
The graph shows the relationship of the number of gallons being drained from an aquarium over time.
Now,
The given graph is:
Envision Math Common Core 8th Grade Answer Key Topic 3 Use Functions To Model Relationships 95.1
From the given graph,
We can observe that there is an initial value or y-intercept
So,
The y-intercept from the given graph is: 90
Now,
To find the slope from the given graph,
The given points are: (0, 90), and (9, 0)
So,
Slope (m) = \(\frac{0 – 90}{9 – 0}\)
m = \(\frac{-90}{9}\)
m = -10
We know that,
The representation of the equation in the slope-intercept form is:
y = mx + c
So,
y = -10x + 90
Hence, from the above,
We can conclude that the equation that represents the given relationship is:
y = -10x + 90

LESSON 3-5 Intervals of Increase and Decrease

Quick Review
You can describe the relationship between two quantities by looking at the behavior of the line at different intervals on a qualitative graph. The function is increasing if both x- and y-values increase. The function is decreasing if the y-values decrease as the x-values increase.

Practice

The graph shows the altitude of an airplane over time.
Envision Math Common Core 8th Grade Answer Key Topic 3 Use Functions To Model Relationships 95.3
Answer:
We know that,
The graph is said to be increasing in nature when the line moves from left to right or the slope is positive
The graph is said to be decreasing in nature when the line moves from right to left or the slope is negative
The graph is said to be constant in nature when the line is parallel to any one of the coordinate axes

Question 1.
In which intervals is the graph of the function constant? Explain.
Answer:
The intervals in which the graph of the function is constant are: 2, 4, 6

Question 2.
In which intervals is the graph of the function decreasing? Explain.
Answer:
The intervals in which the graph of the function is decreasing in nature are: 5, 7

LESSON 3.6 Sketch Functions from Verbal Descriptions

Quick Review
You can sketch a graph of a function to describe its behavior. When sketching a function, identify the variables (input, output) that have a relationship, analyze the situation, and then sketch the graph.

Practice

Question 1.
Jack’s mother brings him a bowl of carrots as a snack. At first he does not eat any; then he eats one at a time until half of the carrots are gone. Then he does not eat any more. Sketch a graph that shows the number of carrots in the bowl over time.
Envision Math Common Core 8th Grade Answer Key Topic 3 Use Functions To Model Relationships 96.1
Answer:
It is given that
Jack’s mother brings him a bowl of carrots as a snack. At first, he does not eat any; then he eats one at a time until half of the carrots are gone. Then he does not eat anymore
So,
At first,
The number of carrots present in a bowl is: An arbitrary number
The number of carrots Jack consumed is: 0
Hence,
The graph that shows the number of carrots in the bowl over time is:

Topic 3 Fluency Practice

What’s the Message?

In each row, determine which equation has the greater solution. Circle the letter next to the equation with the greater solution in each row.

Envision Math Common Core 8th Grade Answer Key Topic 3 Use Functions To Model Relationships 96.3
Answer:
The row of equations that has great solutions are:

What does the zero say to the eight?
Envision Math Common Core 8th Grade Answer Key Topic 3 Use Functions To Model Relationships 96.4
Answer:
The statement said by zero to eight is:
You are greater than me

enVision Math Common Core Grade 7 Answer Key Topic 8 Solve Problems Involving Geometry

Practice with the help of enVision Math Common Core Grade 7 Answer Key Topic 8 Solve Problems Involving Geometry regularly and improve your accuracy in solving questions.

enVision Math Common Core 7th Grade Answers Key Topic 8 Solve Problems Involving Geometry

GET READY!
Review What You Know!

Vocabulary
Choose the best term from the box to complete each definition.
Envision Math Common Core 7th Grade Answers Topic 8 Solve Problems Involving Geometry 1

Question 1.
The number of square units that a figure covers is its _________.
Answer:
We know that,
The number of square units that a figure covers is its “Area”
Hence, from the above,
We can conclude that the best term to complete the given definition is the “Area”

Question 2.
The _________ of a triangle is the length of the perpendicular line segment from a vertex to the opposite side.
Answer:
We know that,
The “Height” of a triangle is the length of the perpendicular line segment from a vertex to the opposite side.
Hence, from the above,
We can conclude that the best term to complete the given definition is the “Height”

Question 3.
The _________ of a solid figure is the number of cubic units needed to fill it.
Answer:
We know that,
The “Volume” of a solid figure is the number of cubic units needed to fill it
Hence, from the above,
We can conclude that the best term to complete the given definition is the “Volume”

Question 4.
Any line segment that connects the center of a circle to a point on the circle is called a __________.
Answer:
We know that,
Any line segment that connects the center of a circle to a point on the circle is called a “Radius”
Hence, from the above,
We can conclude that the best term to complete the given definition is the “Radius”

Area and Volume
Find each measure.

Question 5.
Area of a triangle with a base 6 feet and height 9 feet
Answer:
It is given that
The base of the triangle is: 6 feet
The height of the triangle is: 9 feet
Now,
We know that,
The area of a triangle (A) = \(\frac{1}{2}\) × Base of the triangle × Height of the triangle
So,
A = \(\frac{1}{2}\) × 6 × 9
= \(\frac{54}{2}\)
= 27 feet²
Hence, from the above,
We can conclude that the area of the given triangle is: 27 feet²

Question 6.
The volume of a rectangular prism with length 4 inches, width 2 inches, and height 2 inches
Answer:
It is given that
The length of a rectangular prism is: 4 inches
The width of a rectangular prism is: 2 inches
The height of a rectangular prism is: 2 inches
Now,
We know that,
The volume of a rectangular prism (V) = Length × Width × Height
So,
V = 4 × 2 × 2
= 16 inches³
Hence, from the above,
We can conclude that the volume of the given rectangular prism is: 16 inches³

Measure Angles
Use a protractor to find the measure of each angle.
Question 7.
Envision Math Common Core 7th Grade Answers Topic 8 Solve Problems Involving Geometry 2
Answer:
To measure the angle, use “https://www.geogebra.org/calculator”
So,
The measured angle is:

Hence, from the above,
We can conclude that the measured angle using the protractor is: 54.5°

Question 8.
Envision Math Common Core 7th Grade Answers Topic 8 Solve Problems Involving Geometry 3
Answer:
To measure the angle, use “https://www.geogebra.org/calculator”
So,
The measured angle is:

Hence, from the above,
We can conclude that the measured angle using the protractor is: 104.3°

Describe Characteristics of Shapes
Describe this figure using as many geometry terms as you can.
Question 9.
Envision Math Common Core 7th Grade Answers Topic 8 Solve Problems Involving Geometry 4
Answer:
The given figure is:
Envision Math Common Core 7th Grade Answers Topic 8 Solve Problems Involving Geometry 4
Now,
From the given figure,
We can observe that
a. All the angles are 90°
b. The parallel sides are the same
c. The diagonals bisect each other
d. The lengths of the diagonals are equal
So,

Hence, from the above,
We can conclude that the given figure is a “Rectangle”

Language Development
Fill in the word web to connect keywords you learn in this topic. A sample keyword and its connections are given.
Envision Math Common Core 7th Grade Answers Topic 8 Solve Problems Involving Geometry 5
Envision Math Common Core 7th Grade Answers Topic 8 Solve Problems Involving Geometry 6
PICK A PROJECT

PROJECT 8A
If you built a sculpture, what materials would you use?
PROJECT: CONSTRUCT A THREE-DIMENSIONAL SCULPTURE

PROJECT 8B
If you made a pizza, what kind of pizza would it be?
PROJECT: ANALYZE A PEPPERONI PIZZA

PROJECT 8C
What places have you visited where being a tour guide would be fun?
PROJECT: PLAN A GUIDED TOUR

PROJECT 8D
How could you determine which is a larger-a tall building or a wide building?
PROJECT: BUILD A SCALE MODEL

Lesson 8.1 Solve Problems Involving Scale Drawings

Explore It!

Calvin made a scale model of the plane shown.
Envision Math Common Core 7th Grade Answers Topic 8 Solve Problems Involving Geometry 7

I can… use the key in a scale drawing to find missing measures.

A. How can you represent the relationship between the model of the plane and the actual plane?
Answer:
The given figure is:
Envision Math Common Core 7th Grade Answers Topic 8 Solve Problems Involving Geometry 7
Now,
The representation of the relationship between the model of the plane and the actual plane is:
The actual diagram is an enlarged diagram of the scale diagram
So,
The scale factor = \(\frac{15 in.}{240 ft}\)
= \(\frac{1}{16}\)
Hence, from the above,
We can conclude that the actual diagram is 16 times larger than the scale diagram

B. What do you notice about the relationship between the model of the plane and the actual plane?
Answer:
We know that,
The relationship between the model of the plane and the actual plane can be given by a “Scale factor”
So,
The scale factor = \(\frac{15 in.}{240 ft}\)
= \(\frac{1}{16}\)
Hence, from the above,
We can conclude that the actual diagram is 16 times larger than the scale diagram

Focus on math practices
Look for Relationships If the model and the actual plane are to scale, what do you know about the relationship between all the other parts of the model and the actual plane, aside from the total length?
Answer:
If the model and the actual plane are to scale, then
All the other parts of the model and the actual plane, aside from the total length are also scaled

Essential Question
How do scale drawings and actual measurements represent proportional relationships?
Answer:
The “Scale factor” is the amount that an image is enlarged or shrunk. It is also called ‘k’ because k represents the multiplier between two proportional ratios and scale drawings represent proportional relationships

Try It!

What is the actual width, w, of the island if the width in the drawing is 2.5 inches?
Envision Math Common Core 7th Grade Answers Topic 8 Solve Problems Involving Geometry 11
The actual width of the island is _______ feet.
Answer:
From Example 1,
It is given that,
Scale: 1 inch = 1.5 ft
Now,
\(\frac{1.5}{1}\) = \(\frac{w}{2.5}\)
w = \(\frac{1.5 × 2.5}{1}\)
w = 3.75 feet
Hence, from the above,
We can conclude that the actual width of the island is: 3.75 feet

Convince Me! How would the proportion for Example 1 change if the scale changed?
Answer:
For Example 1,
If the scale changed, then
The proportion would increase or decrease
Case 1:
If the scale changed to:
1 in = 2 ft
Then, the proportion will change like
\(\frac{2}{1}\) = \(\frac{x}{6}\)
x = \(\frac{6 × 2}{1}\)
x = 12 feet
So,
The actual width of the island is: 12 feet
Case 2:
If the scale changed to:
1 in = 1.25 ft
Then, the proportion will change like
\(\frac{1.25}{1}\) = \(\frac{x}{6}\)
x = \(\frac{6 × 1.25}{1}\)
x = 7.5 feet
So,
The actual width of the island is: 7.5 feet
Hence, from the above,
We can conclude that
If the scale increases, then the proportion also increases
If the scale decreases, then the proportion also decreases

Try It!

The scale drawing shown represents an existing barn. The shortest side of the barn measures 150 meters. If a new barn that is \(\frac{2}{3}\) its size replaces the existing barn, what will be the scale of this drawing to the new barn?
Envision Math Common Core 7th Grade Answers Topic 8 Solve Problems Involving Geometry 15
Answer:
It is given that
The scale drawing shown represents an existing barn. The shortest side of the barn measures 150 meters and a new barn that is \(\frac{2}{3}\) its size replaces the existing barn
Now,
The representation of the existing barn is:
Envision Math Common Core 7th Grade Answers Topic 8 Solve Problems Involving Geometry 15
Now,
The length of the shortest side for a new barn = \(\frac{2}{3}\) × 150
= 100 in.
So,
The scale factor for the shortest side for a new barn = \(\frac{150}{100}\)
= 1.5
Hence, from the above,
We can conclude that
The scale of the drawing to the new barn is 1: 1.5

KEY CONCEPT
The scale factor of a scale drawing is the ratio of an actual length, y, to the corresponding length, x, in the drawing. The ratio is the constant of proportionality, k, that relates the actual figure to the scale drawing. You can use a proportion or use an equation of the form y = kx to solve problems involving scale drawings.
Envision Math Common Core 7th Grade Answers Topic 8 Solve Problems Involving Geometry 16

Do You Understand?

Question 1.
Essential Question How do scale drawings and actual measurements represent proportional relationships?
Answer:
The “Scale factor” is the amount that an image is enlarged or shrunk. It is also called “k” because k represents the multiplier between two proportional ratios and scale drawings represent proportional relationships

Question 2.
Generalize Describe the ratio of corresponding measures in scale drawings and the actual measures they represent?
Answer:
A scale drawing is a drawing that is to scale an original image. When an object is “to scale,” that means it is the same shape, but not the same size. It is also called k because k represents the multiplier between two proportional ratios and scale drawings represent proportional relationships.

Question 3.
Reasoning Mikayla is determining the actual distance between Harrisville and Lake Town using a map. The scale on her map reads. 1 inch = 50 miles. She measures the distance to be 4.5 inches and writes the following proportion:
\(\frac{1 \mathrm{in.}}{4.5 \mathrm{in.}}=\frac{50 \mathrm{mi}}{x \mathrm{mi}}\)
Explain why her proportion is equivalent to
\(\frac{50 \mathrm{mi}}{1 \text { in. }}=\frac{x \mathrm{mi}}{4.5 \mathrm{in} .}\)
Answer:
It is given that
Reasoning Mikayla is determining the actual distance between Harrisville and Lake Town using a map. The scale on her map reads. 1 inch = 50 miles. She measures the distance to be 4.5 inches
So,
The scale for the distance between Harrisville and Lake Town using a map is: 50: 1
Now,
Let the value that is equivalent to 4.5 inches be x miles
Now,
To find the value in miles that is equivalent to 4.5 inches,
We have to write
\(\frac{1 in.}{4.5 in.}\) = \(\frac{50 mi}{x mi}\)
Now,
Cross-Multiply the given equation
So,
\(\frac{50 mi}{1 in.}\) = \(\frac{x miles}{4.5 in.}\)
Hence, from the above,
We can conclude that the proportion is in the above form because of the cross-multiplication

Do You Know How?

Question 4.
What is the actual base length of the triangle depicted in the scale drawing?
Envision Math Common Core 7th Grade Answers Topic 8 Solve Problems Involving Geometry 17
Answer:
The given figure is:
Envision Math Common Core 7th Grade Answers Topic 8 Solve Problems Involving Geometry 17
Now,
From the given figure,
We can observe that
The scale factor = \(\frac{2}{1}\)
So,
The actual base length of the given triangle = (The base length as depicted in the given triangle) × (Scale factor)
= 2.5 × \(\frac{2}{1}\)
= 2.5 × 2
= 5 m.
Hence, from the above,
We can conclude that
The actual base length of the triangle as depicted in the given scale drawing is: 5 m

Question 5.
What is the area of the actual square window shown in the scale drawing?
Envision Math Common Core 7th Grade Answers Topic 8 Solve Problems Involving Geometry 18
Answer:
The given figure is:
Envision Math Common Core 7th Grade Answers Topic 8 Solve Problems Involving Geometry 18
Now,
From the given figure,
We can observe that
Scale factor = \(\frac{2}{1}\)
So,
The actual side length of the given square window = (The side length of the given square window) × (Scale factor)
= 0.75 × 2
= 15 m
Now,
We know that,
The area of a square = Side²
So,
The area of the actual square window = (Actual Side length of the given square window)²
= 15 m × 15 m
= 225 m²
Hence, from the above,
We can conclude that the area of the actual square window is: 225 m²

Question 6.
A distance of 30 miles on a map is represented by a 2-inch line. If the map is enlarged to 3 times its size, what will be the scale of the enlarged map?
Answer:
It is given that
A distance of 30 miles on a map is represented by a 2-inch line and the map is enlarged to 3 times its size
Now,
The scale distance on a map = \(\frac{Actual distance on a map}{Scale of a map}\)
= \(\frac{30}{2}\)
= 15 inches
Now,
The scale distance on the enlarged map = (The scale distance on a normal map) × 3
= 15 × 3
= 45 inches
So,
The scale of the enlarged map = \(\frac{45}{3}\)
= 15
Hence, from the above,
We can conclude that the scale of the enlarged map is: 1inch = 15 miles

Practice & Problem Solving

Leveled Practice For 7 and 8, fill in the boxes to find the actual measures.
Question 7.
On a map, 1 inch equals 5 miles. Two cities are 8 inches apart on the map.
What is the actual distance between the cities?
Envision Math Common Core 7th Grade Answers Topic 8 Solve Problems Involving Geometry 19
Answer:
It is given that
On a map, 1 inch equals 5 miles. Two cities are 8 inches apart on the map.
Now,
The scale factor from the given information is:
1 inch = 5 miles
So,

Hence, from the above,
We can conclude that
The actual distance between the cities is: 40 miles

Question 8.
Ryan makes a scale drawing of a banner for a school dance. He uses a scale of 1 inch = 3 feet, and the width of the drawing is 5 inches. What is the actual width, w, of the banner?
Envision Math Common Core 7th Grade Answers Topic 8 Solve Problems Involving Geometry 20
Answer:
It is given that
Ryan makes a scale drawing of a banner for a school dance. He uses a scale of 1 inch = 3 feet, and the width of the drawing is 5 inches
Now,
From the given information,
We can observe that
The actual width of the banner (w) = (The width of the drawing) × (Scale factor)
So,

Hence, from the above,
We can conclude that
The actual width (w) of the banner is: 15 feet

Question 9.
On a map, 1 inch equals 7.2 miles. Two houses are 1.5 inches apart on the map. What is the actual distance between the houses?
Answer:
It is given that
On a map, 1 inch equals 7.2 miles. Two houses are 1.5 inches apart on the map
So,
From the given information,
The scale factor = \(\frac{7.2}{1}\)
Now,
Let the actual distance between the houses be d
So,
\(\frac{7.2}{1}\) = \(\frac{d}{1.5}\)
d = 7.2 × 1.5
d = 10.80 miles
Hence, from the above,
We can conclude that
The actual distance between the houses is: 10.8 miles

Question 10.
The original blueprint for the Morenos’ living room has a scale of 2 inches = 5 feet. The family wants to use a new blueprint that shows the length of the living room to be 15 inches. If the width of the living room on the original blueprint is 6 inches and the length is 9.6 inches, what are the scale and the width of the new blueprint?
Answer:
It is given that
The original blueprint for the Morenos’ living room has a scale of 2 inches = 5 feet. The family wants to use a new blueprint that shows the length of the living room to be 15 inches and the width of the living room on the original blueprint is 6 inches and the length is 9.6 inches
Now,
The length of the new blueprint = (Scale factor) × (The length of the original blueprint)
= \(\frac{5}{2}\) × 9.6
= 5 × 4.8
= 24.0 inches
= 24 inches
The width of the new blueprint = (Scale factor) × (The width of the original blueprint)
= \(\frac{5}{2}\) × 6
= 5 × 3
= 15 inches
Now,
\(\frac{24}{15}\) = \(\frac{x}{1}\)
x = \(\frac{8}{3}\)
Hence, from the above,
We can conclude that
The scale factor for the new blueprint is:
3 inches = 8 feet

Question 11.
The scale for a drawing of the tennis court is 1 centimeter = 2 meters. What is the area of the actual tennis court?
Envision Math Common Core 7th Grade Answers Topic 8 Solve Problems Involving Geometry 21
Answer:
It is given that
The scale for a drawing of the tennis court is 1 centimeter = 2 meters.
Now,
The given figure is:
Envision Math Common Core 7th Grade Answers Topic 8 Solve Problems Involving Geometry 21
So,
The length of the actual tennis court = (Scale factor) × (The length of the scaled tennis court)
= 2 × 10
= 20 meters
So,
The width of the actual tennis court = (Scale factor) × (The width of the scaled tennis court)
= 2 × 5.5
= 11 meters
Now,
We know that,
The area of a rectangle = Length × Width
So,
The area of the actual tennis court = (The length of the actual tennis court) × (The width of the actual tennis court)
= 20 × 11
= 220 meters²
Hence, from the above,
We can conclude that the area of the actual tennis court is: 220 meters²

Question 12.
The scale for the drawing of a rectangular playing field is 2 inches = 5 feet.
a. Write an equation you can use to find the dimensions of the actual field, where x is a dimension of the scale drawing (in inches) and y is the corresponding dimension of the actual field (in feet).
Envision Math Common Core 7th Grade Answers Topic 8 Solve Problems Involving Geometry 22
Answer:
It is given that
The scale for the drawing of a rectangular playing field is 2 inches = 5 feet.
Now,
The given figure is:
Envision Math Common Core 7th Grade Answers Topic 8 Solve Problems Involving Geometry 22
Now,
From the given information,
The scale factor = \(\frac{5}{2}\)
So,
The actual length of the rectangular playing field = (Scale factor) × (The length of the scaled rectangular playing field)
= \(\frac{5}{2}\) × 20
= 5 × 10
= 50 feet
The actual width of the rectangular playing field = (Scale factor) × (The width of the scaled rectangular playing field)
= \(\frac{5}{2}\) × 10
= 5 × 5
= 25 feet
So,
The scale factor for the actual rectangular playing field can be found out by:
\(\frac{50}{20}\) = \(\frac{x}{1}\)   (or)   \(\frac{25}{10}\) = \(\frac{x}{1}\)
x = \(\frac{5}{2}\)
Hence, from the above,
We can conclude that
The scale for the actual rectangular playing field is:
2 inches = 5 feet

b. What is the area of the field?
Answer:
From part (a),
We can observe that
The actual length of the rectangular playing field = (Scale factor) × (The length of the scaled rectangular playing field)
= \(\frac{5}{2}\) × 20
= 5 × 10
= 50 feet
The actual width of the rectangular playing field = (Scale factor) × (The width of the scaled rectangular playing field)
= \(\frac{5}{2}\) × 10
= 5 × 5
= 25 feet
So,
The area of the rectangular playing field = (Actual length) × (Actual width)
= 50 × 25
= 1,250 feet²
Hence, from the above,
We can conclude that
The area of the field is: 1,250 feet²

Question 13.
How many square feet of flooring are needed to cover the entire floor of Bedroom 1?
Envision Math Common Core 7th Grade Answers Topic 8 Solve Problems Involving Geometry 23
Answer:

Question 14.
The actual distance between Point A and Point B is 200 meters. A length of 1.9 feet represents this distance on a certain wall map. Point C and Point D are 3.8 feet apart on this map. What is the actual distance between Point C and Point D?
Envision Math Common Core 7th Grade Answers Topic 8 Solve Problems Involving Geometry 24
Answer:
It is given that
The actual distance between Point A and Point B is 200 meters. A length of 1.9 feet represents this distance on a certain wall map. Point C and Point D are 3.8 feet apart on this map
Now,
Let the actual distance between points C and D be x meters
So,
\(\frac{200}{x}\) = \(\frac{1.9}{3.8}\)
\(\frac{200}{x}\) = \(\frac{1}{2}\)
x = 200 × 2
x = 400 meters
Hence, from the above,
We can conclude that
The actual distance between points C and D is: 400 meters

Question 15.
Higher-Order Thinking A map of a highway has a scale of 2 inches equals 33 miles. The length of the highway on the map is 6 inches. There are 11 rest stops equally spaced on the highway, including one at each end. You are making a new map with a scale of 1 inch equals 30 miles. How far apart are the rest stops on the new map?
Answer:
It is given that
A map of a highway has a scale of 2 inches equals 33 miles. The length of the highway on the map is 6 inches. There are 11 rest stops equally spaced on the highway, including one at each end. You are making a new map with a scale of 1 inch equals 30 miles
So,
The actual length of the highway according to the scale of 2 in = 33 miles is:
The actual length of the highway = (Scale factor) × (The length of the highway on the map)
= \(\frac{33}{2}\) × 6
= 33 ×3
= 99 miles
The actual length of the highway according to the scale of 1 in = 30 miles is:
The actual length of the highway = (Scale factor) × (The length of the highway on the map)
= \(\frac{30}{1}\) × 6
= 30 ×6
= 180 miles
Now,
Let the number of stops on the highway with a scale of 1 in = 30 miles be x
So,
\(\frac{99}{11}\) = \(\frac{180}{x}\)
\(\frac{180}{x}\) = 9
x = \(\frac{180}{9}\)
x = 20 stops
Now,
The distance between the rest stops = \(\frac{30}{20}\)
= 1.5 feet
Hence, from the above,
We can conclude that
The distance between the rest stops on the new map are 1.5 feet

Assessment Practice
Question 16.
The original blueprint of a concrete patio has a scale of 2 inches = 3 feet.
Envision Math Common Core 7th Grade Answers Topic 8 Solve Problems Involving Geometry 25
Victoria wants to make a new blueprint of the patio with a length of 16.8 inches.
PART A
What is the scale for the new blueprint?
1 inch = _______ feet
Answer:
It is given that
The original blueprint of a concrete patio has a scale of 2 inches = 3 feet.
Now,
The given figure is:
Envision Math Common Core 7th Grade Answers Topic 8 Solve Problems Involving Geometry 25
Now,
From the given information,
Scale factor = \(\frac{3}{2}\)
So,
The length of the new blueprint = (Scale factor) × (The length of the original blueprint)
= \(\frac{3}{2}\) × 14
= 3 × 7
= 21 inches
Now,
\(\frac{14}{16.8}\) = \(\frac{21}{x}\)
x = 8.4 × 3
x = 25.2
Hence, from the above,
We can conclude that
The scale factor for the new blueprint is:
1 inch = 25.2 feet

PART B
What is the width, in inches, of the blueprint with the new scale?
Answer:
The given figure is:
Envision Math Common Core 7th Grade Answers Topic 8 Solve Problems Involving Geometry 25
So,
The width of the new blueprint = (Scale factor) × (The length of the original blueprint)
= \(\frac{3}{2}\) × 12
= 3 × 6
= 18 inches
Hence, from the above,
We can conclude that
The width (w) of the blueprint with the new scale is: 18 inches

Lesson 8.2 Draw Geometric Figures

Solve & Discuss It!

Students in the Art Club are designing a flag with the school’s mascot and emblem. The flag has four sides, with two sides that are twice as long as the other two sides. What shape could the flag be, and what dimensions could it have? Make and label a scale drawing as part of your answer.
Envision Math Common Core 7th Grade Answers Topic 8 Solve Problems Involving Geometry 26
I can… draw figures with given conditions.
Answer:
It is given that
Students in the Art Club are designing a flag with the school’s mascot and emblem. The flag has four sides, with two sides that are twice as long as the other two sides.
Now,
We know that,
A “Quadrilateral” is only a geometrical figure which has 4 sides
Now,
Let the length of the two sides be x cm
Let the length of the other two sides be 2x cm
Where,
x = 1, 2, 3,……….., n
Now,
Let the value of x be 2
Hence,
The representations of the flag are:

Make Sense and Persevere: Is there more than one shape that could represent the flag?
Answer:
Yes, there is more than one shape that could represent the flag
Now,
The representations of the flag are:

Hence,
From the above representation,
The different shapes of the flags are: Rectangle, Parallelogram, and Trapezium, and a Quadrilateral

Focus on math practices
Reasoning How did you decide what lengths to use for the four sides of the flag you drew? What lengths could the actual flag be, based on your drawing?
Answer:
It is given that
The flag has four sides, with two sides that are twice as long as the other two sides
Now,
Let the length of the two sides be x cm
Let the length of the other two sides be 2x cm
Where,
x = 1, 2, 3,……….., n
Now,
Based on the drawing that is present in part (a),
The actual lengths of the drawing are: 2 inches, 2 inches, 4 inches, and 4 inches

Essential Question
How can a shape that meets given conditions be drawn?
Answer:
Use the conditions that are given to draw a shape and based on that shape, determine the name of the shape and its dimensions

Try It!
Use a ruler and protractor to draw a quadrilateral with two equal sides that meet at a right angle, and two nonadjacent angles of the same measure. What is the name of the quadrilateral you drew?
The quadrilateral I drew is a ___________
Answer:
It is given that
Use a ruler and protractor to draw a quadrilateral with two equal sides that meet at a right angle, and two nonadjacent angles of the same measure.
Now,
The steps to draw the quadrilateral are:
Step 1:
Draw the line segment AB with a certain length i.e., AB = 4 cm
Step 2:
From B, make an angle of 90° with the same length as AB and draw a line segment BC
Step 3:
From C, make another angle of 90° and draw a line segment CD
Hence, from the above steps,
We can conclude that the quadrilateral drawn is: Square

Convince Me! Could you have drawn more than one shape that fits the given conditions? Explain.
Answer:
No, we can’t draw any shape other than the square that fits the above conditions

Try It!
a. Make a sketch to show another way Mr. Miller can arrange the desks to seat 10 people for lunch.
Answer:
The representation of the arrangement of desks to seating 10 people for lunch is:

Hence, from the above arrangement,
We can conclude that Mr.Millercan arrange 6 desks in 2 rows to seat 10 people for lunch

b. Use geometry software to make a rhombus with a side length of 6 units and two angles that measure 45°.
Answer:
To draw a rhombus,
The geometry software we can use is: Geogebra
Now,
To draw a rhombus, the following steps have to be followed:
Step 1:
Draw a line segment AB with a length of 6 cm
Step 2:
From A, make an angle of 45° and join that segment and name it as AD
Step 3:
From D, draw a line segment DC = 6 cm
Step 4:
From C, make an angle of 45° and join A and C
Hence, from the above steps,
The representation of the rhombus is:

KEY CONCEPT
You can draw shapes that meet given conditions freehand, with a ruler and protractor, or with technology, The given conditions may include properties of geometric figures and relationships between parts of the figures.

Use Appropriate Tools Deciding how precise the drawing of the shape should be will help you choose the method you use to draw the shape.

Do You Understand?
Question 1.
Essential Question How can a shape that meets given conditions be drawn?
Answer:
Use the conditions that are given to draw a shape and based on that shape, determine the name of the shape and its dimensions

Question 2.
Use Appropriate Tools How can you decide whether to draw a shape freehand, with a ruler and protractor, or using technology?
Answer:
a. We will draw a shape freehand when there are not any measurements to draw a particular shape but only the name of the shape
b. We will draw a shape with a ruler when the measurements are specified for a particular shape
c. We will draw a shape with a protractor and a ruler when the measurements and an angle for a particular shape are specified

Question 3.
Construct Arguments Why can you draw more than one quadrilateral using four right angles?
Answer:
We know that,
A parallelogram is a quadrilateral with 2 pairs of opposite sides parallel. A rectangle is a special parallelogram that has 4 right angles. A square is a special rectangle that has all four sides congruent. The angle between these two sides could be a right angle
Hence, from the above,
We can conclude that we can draw more than one quadrilateral using four right angles

Do You Know How?
Question 4.
Draw, freehand, a quadrilateral with exactly one pair of parallel sides and at least one angle measuring 45°.
Answer:
The steps to draw a freehand quadrilateral with exactly one pair of parallel sides and at least one angle measuring 45° are:
Step 1:
Draw a line segment AB with a certain length
Step 2:
Mark an angle of 45° from point A using the protractor and join the segment and name it as BC
Step 3:
From B, draw a line segment such that the length of AB must not be equal to the line segment that is drawn from B and name it BC
Step 4 :
From C, draw a line segment named CDand the length of CD must be equal to the length of AB since they are 2 parallel sides
Step 5:
Join D and A and so we will get the desired quadrilateral

Question 5.
Use a ruler and protractor to draw a quadrilateral with four right angles, two side lengths each measuring 3 inches, and two side lengths each measuring 4 inches. What is the most descriptive name of the figure you drew?
Answer:
It is given that
Use a ruler and protractor to draw a quadrilateral with four right angles, two side lengths each measuring 3 inches, and two side lengths each measuring 4 inches
So,
By using the Geometry software,
The representation of the quadrilateral with the given specifications is:

Hence, from the above,
We can conclude that the suitable name of the quadrilateral that was drawn using the given specifications is: Rectangle

Question 6.
Use geometry software to draw a quadrilateral with two angles measuring 80° and two angles measuring 100°. What is the name of the figure you drew?
Answer:
It is given that
Use geometry software to draw a quadrilateral with two angles measuring 80° and two angles measuring 100°.
So,
The representation of the quadrilateral with the given specifications is:

Now,
From the above figure,
We can observe that
a. All the angles are not equal to 90°
b. All the side lengths are equal
Hence, from the above,
We can conclude that the quadrilateral we were drawn using the given specifications is: Rhombus

Practice & Problem Solving

Question 7.
What quadrilaterals can you draw that have exactly four right angles?
Answer:
The quadrilaterals that we can draw that have exactly four right angles are:
a. Rectangle b. Square

Question 8.
A four-sided sandbox has more than two right angles, two side lengths of 2 feet, and two side lengths of 5 feet. What geometric shape best describes the shape of the sandbox?
Answer:
It is given that
A four-sided sandbox has more than two right angles, two side lengths of 2 feet, and two side lengths of 5 feet
So,
The representation of the sandbox with the given specifications is:

Now,
From the above figure,
We can observe that
a. All the angle measures are 90°
b. The parallel sides are of the same length
Hence, from the above,
We can conclude that the shape of the given sandbox is: Rectangle

Question 9.
What quadrilateral can you draw that has exactly one pair of parallel sides?
Answer:
The quadrilateral you can draw that has exactly one pair of parallel sides is: Trapezoid

Question 10.
A friend is building a 4-sided garden with two side lengths of 19 feet and exactly one right angle. What quadrilaterals could describe the garden?
Answer:
It is given that
A friend is building a 4-sided garden with two side lengths of 19 feet and exactly one right angle.
So,
From the above specifications,
The shape of the garden must be a rectangle
So,
The representation of the 4-sided garden with the given specifications is:

Hence, from the above,
We can conclude that the quadrilateral that could describe the garden is: Rectangle

Question 11.
What quadrilaterals can you draw that have two side lengths of 9 centimeters and two side lengths of 4 centimeters?
Answer:
The quadrilaterals that you can draw that have two side lengths of 9 cm and two side lengths of 4 cm are:
a. Rectangle (With all the angle measures 90°)
b. Parallelogram ( WIth all the angle measures not equal to 90°)
Hence,
The representation of the two quadrilateral with the given specifications is:

Question 12.
A park has a pond shaped like a quadrilateral with side lengths of 17 feet and no right angles. What other geometric shapes could describe the shape of the pond?
Answer:
It is given that
A park has a pond shaped like a quadrilateral with side lengths of 17 feet and no right angles
Now,
We know that,
A quadrilateral that has all the side lengths equal and no right angles is: Rhombus
So,
The shape of the pond is: Rhombus
Now,
We know that,
Any other geometric shape does not have all the equal side lengths other than the square but in the square, all the angle measures are 90°
Hence, from the above,
We can conclude that the shape of the pond is: Rhombus
  Scale: 1 cm = 2 feet

Question 13.
Draw a quadrilateral that has one angle measure of 20° and exactly one side length of 4 units.
Answer:
It is given that to draw a quadrilateral that has one angle measure of 20° and exactly one side length of 4 units.
Hence,
The representation of the quadrilateral with the given specifications by using the geometry software is:

Question 14.
Which of the following shapes are trapezoids that have side lengths of 7 inches and 5 inches and a right angle? Select all that apply.
Envision Math Common Core 7th Grade Answers Topic 8 Solve Problems Involving Geometry 27
Answer:
We know that,
A right-angled Trapezoid has 2 adjacent right angles and a pair of parallel sides
Hence,
The Trapezoids that have represented by the given specifications are:

Question 15.
Using computer software, draw a quadrilateral with two sets of parallel sides and two angles measuring 135 degrees.
Answer:
It is given that
Draw a quadrilateral with two sets of parallel sides and two angles measuring 135 degrees.
Hence,
The representation of the quadrilateral with the given specifications by using the geometry software is:

Question 16.
Higher-Order Thinking Draw a rhombus with side lengths of 6 units and angle measures of 100°, 80°, 100°, and 80°.
Answer:
It is given that
Draw a rhombus with side lengths of 6 units and angle measures of 100°, 80°, 100°, and 80°.
Hence,
The representation of a rhombus with the given specifications by using the geometry software is:

Assessment Practice
Question 17.
Thomas is painting a geometry mural. He is painting quadrilaterals that have at least 1 line of symmetry.
PART A
Which could be a quadrilateral that Thomas painted? Select all that apply.
Envision Math Common Core 7th Grade Answers Topic 8 Solve Problems Involving Geometry 28
Answer:
It is given that
Thomas is painting a geometry mural. He is painting quadrilaterals that have at least 1 line of symmetry.
Now,
We know that,
The “Symmetry lines” are the lines that have equal side lengths
Hence, from the above,
The quadrilaterals that Thomas painted are:

PART B
Which of the following figures can he also include in his painting?
A. quadrilateral with no equal sides
B. A quadrilateral with only 2 equal sides that are perpendicular to each other
C. A quadrilateral with 2 pairs of equal sides and 1 right angle
D. A quadrilateral with 2 pairs of parallel equal sides, with no right angles
Answer:
From part (a),
We can observe that the quadrilaterals that are included in Thomas painting have:
a. The equal sides and 1 right angle (or) more than one right angle
b. The equal pair of sides and no right angles
Hence, from the above,
We can conclude that the nature of figures that are included in Thomas paintings are: Option C and Option  D

Lesson 8.3 Draw Triangles with Given Conditions

Solve & Discuss It!

Kane has 4 pieces of wood available to build a triangle-shaped garden. Which pieces of wood can he use?
Envision Math Common Core 7th Grade Answers Topic 8 Solve Problems Involving Geometry 29
I can… draw triangles when given information about their side lengths and angle measures.
Answer:
It is given that
Kane has 4 pieces of wood available to build a triangle-shaped garden.
Now,
We know that,
To build a Triangle,
The sum of the lengths of the two shortest sides must be greater than the length of the third side
Now,
The given pieces of wood are:
a. 2 feet b, 3 feet c. 4 feet d. 5 feet
So,
The possible combination of pieces of wood is:
a. (2, 3, 5) b. (2, 3, 4) c. (3, 4, 5) d. (2, 4, 5)
Now,
The possible combination of pieces of wood that allow to form a triangle-shaped garden is:
a. 2 + 3 !>[Not greater thhan] 5
b. 2 + 3 > 4
c. 3 + 4 > 5
d. 2 + 4 > 5
Hence, from the above,
We can conclude that the possible combinations of the pieces of wood that form a Triangle-shaped garden are:
a. (2 feet, 3 feet, 4 feet)
b. (3 feet, 4 feet, 5 feet)
c. (2 feet, 4 feet, 5 feet)

Make Sense and Persevere Try all possible combinations of three pieces of wood.
Answer:
We know that,
To build a Triangle,
The sum of the lengths of the two shortest sides must be greater than the length of the third side
Now,
The given pieces of wood from part (a) are:
a. 2 feet b, 3 feet c. 4 feet d. 5 feet
So,
The possible combination of pieces of wood to form a triangle-shaped garden is:
a. (2, 3, 5) b. (2, 3, 4) c. (3, 4, 5) d. (2, 4, 5)
Now,
The possible combination of pieces of wood that allow to form a triangle-shaped garden is:
a. 2 + 3 !>[Not greater thhan] 5
b. 2 + 3 > 4
c. 3 + 4 > 5
d. 2 + 4 > 5
Hence, from the above,
We can conclude that the possible combinations of the pieces of wood that form a Triangle-shaped garden are:
a. (2 feet, 3 feet, 4 feet)
b. (3 feet, 4 feet, 5 feet)
c. (2 feet, 4 feet, 5 feet)

Focus on math practices
Use Structure Are there any combinations of three pieces of wood that will not create a triangle? Explain.
Answer:
Yes, there is a combination of three pieces of wood that will not create a triangle
Now,
From part (b),
We can observe that the combination of three pieces of wood that will not create a triangle is: (2, 3, 5)
Hence, from the above,
We can conclude that
The combination of three pieces of wood that will not create a triangle is: (2 feet, 3 feet, 5 feet)

Essential Question
How can you determine when it is possible to draw a triangle given certain conditions?
Answer:
The condition to draw a Triangle under the given conditions is:
The sum of the lengths of the two shortest sides must be greater than the length of the third side

Try It!
How many unique triangles can be drawn with given side lengths of 8 inches, 10.3 inches, and 13 inches?
_________ unique triangle(s) can be drawn with the given side lengths.
Answer:
The given side lengths are: 8 inches, 10.3 inches, and 13 inches
Now,
The representations of the given side lengths of the triangle in different positions are:

So,
From the above,
We can observe that the triangles with the same side lengths have the same shape and the same size, no matter how they are positioned
Hence, from the above,
We can conclude that “1 unique Triangle” can be drawn with the given side lengths

Convince Me! When two sides of a triangle are switched, why is it still considered the same triangle?
Answer:
When two sides of a triangle are switched, we are only switching their positions with their original side lengths.
So,
It is still considered the same triangle

Try It!
a. Write three side lengths that will form a triangle. Write three side lengths that will NOT form a triangle.
Answer:
We know that,
In order to construct a Triangle,
The sum of the lengths of the two shortest sides must be greater than the length of the third side
Hence, from the above,
We can conclude that
The three side lengths that form a triangle are: 3 units, 4 units, and 5 units
The three side lengths that will not form a triangle are: 2 units, 3 units, and 5 units

b. Can a triangle be drawn with a side length of 3 inches and angles at each end measuring 90° and 89°? Explain.
Answer:
It is given that
The side length of a triangle is: 3 inches
The angle measures of a triangle are 90° and 89°
Now,
We know that,
A triangle can only have 1 right angle
But,
The given triangle has 1 right angle and another almost right angle with a third angle measure 1°
Now,
The representation of the triangle with the given specifications will be like:

Hence, from the above,
We can conclude that
It is not possible to draw a triangle with a side length of 3 inches and angles at each end measuring 90° and 89°

Try It!
Can more than one triangle be drawn with two side lengths of 6 inches and a non-included angle of 60°? Explain.
Answer:
It is given that
The two side lengths of the triangle are: 6 inches and 6 inches
The non-included angle is: 60°
Now,
We know that,
As the two side lengths are the same, the triangle will be an “Isosceles Triangle”
So,
Besides the given angle measure, the other two angle measures must be the same
So,
The other two angle measures will be: 60° and 60° [ Since the first angle measure is 60° and the remaining angle measure is 120°]
So,
The representation of the triangle with the given specifications is:

Now,
From the above triangles,
We can observe that there is no change in shape irrespective of the same side lengths and the same non-included angle
Hence, from the above,
We can conclude that more than one triangle can not be drawn with two side lengths of 6 inches and a non-included angle of 60°

KEY CONCEPT
You can analyze given conditions of side lengths and angle measures to determine whether one unique triangle, more than one unique triangle, or no triangle can be drawn.
There is more than one possible triangle given these cases: all three angles, or two sides and a nonincluded angle.
Envision Math Common Core 7th Grade Answers Topic 8 Solve Problems Involving Geometry 30
There is one unique triangle given these cases: all three sides, two sides and an included angle, or two angles and an included side.
Envision Math Common Core 7th Grade Answers Topic 8 Solve Problems Involving Geometry 31

Do You Understand?
Question 1.
Essential Question How can you determine when it is possible to draw a triangle given certain conditions?
Answer:
The condition to draw a Triangle under the given conditions is:
The sum of the lengths of the two shortest sides must be greater than the length of the third side

Question 2.
Look for Relationships What is the relationship between all triangles that can be drawn given the same three angle measures?
Answer:
If we have all the same three angle measures, then that triangle is known as an “Equilateral triangle”
So,
Whatever the turns, reflection, etc., we did, the shape of the equilateral triangle will not change
Hence, from the above,
We can conclude that a “Unique Triangle” can be formed if we draw a triangle will the same three angle measures

Question 3.
Why can there be only one way to draw a triangle if two sides and an included angle are given?
Answer:
It is possible to draw more than one triangle has the side lengths and angle measure as given. It depends on which line you start with, which end of the line you draw the angles, and whether they are above or below the line or not.
Hence, from the above,
We can conclude that because of the above, there is only one way to draw a triangle if two sides and an included angle are given

Do You Know How?
Question 4.
How many triangles can be drawn with side lengths 4 centimeters, 4.5 centimeters, and 9 centimeters? Explain.
Answer:
The given side lengths of a triangle are: 4 cm, 4.5 cm, and 9 cm
Now,
We know that,
In order to construct a triangle,
The sum of the length of the two shortest sides must be greater than the length of the third side
So,
4 + 4.5 > 9
8.5 >! 9
Hence, from the above,
We can conclude that the triangles can not be drawn for the given side lengths since the condition to form a triangle is not true for the given side lengths

Question 5.
Can more than one triangle be drawn with side lengths of 5 inches and 7 inches and an included angle with a measure of 50°? Explain.
Answer:
The given side lengths of a triangle are: 5 inches and 7 inches
The included angle of a triangle is: 50°
Now,
The representations of the triangles with the given specifications are:

Now,
From the above,
We can observe that the triangle with the given specifications is the same even we turn the triangle to any degree
Hence, from the above,
We can conclude that
We can not draw more than one triangle with side lengths of 5 inches and 7 inches and an included angle with a measure of 50°

Question 6.
Sketch two different triangles that have angle measures of 45°, 45°, and 90°.
Answer:
The representation of the different triangles that have angle measures of 45°, 45°, and 90° are:

Practice & Problem Solving

Question 7.
Draw two different triangles with angle measurements of 90°, 35°, and 55°.
Answer:
The representation of the two different triangles with angle measurements of 90°, 35°, and 55° is:

Question 8.
If you form a triangle from three given side lengths, will you always get one triangle or more than one triangle?
Answer:
If you form a triangle from three given side lengths, then you will not always get only one triangle
Ex:
The given side lengths of a triangle are: 3 cm, 4 cm, and 5 cm
Now,
We know that,
In order to construct a triangle,
The sum of the lengths of the two short sides must be greater than the length of the third side
So,
The possible combinations for forming a triangle are: (3, 4, 5), (3, 5, 4), (4, 3, 5), (5, 3, 4)

Question 9.
How can you make different-looking triangles given two of the angle measures and the included side lengths?
Answer:
Since the two angles are already given, therefore the value of the third angle is already fixed. That is, the third angle is the difference between 180° and the sum of the two angles. If the angles and the side lengths are given, then we can form any number of angles as long as the ratio of angles are consistent

Question 10.
If you form a triangle from two given angle measures and the length of the included side, will you always get one triangle or will you get more than one triangle?
Answer:
It is not always possible to construct a triangle from a given side length and two angles. If the two given angles add to more than 180°, then the sides of the triangle will diverge and never meet.

Question 11.
How can you make different triangles with the same angle measures?
Answer:
We can make different triangles with the same angle measures by making the sum of the angles consistent
Example:
The given angle measures are: 30°, 70°, 80°
Now,
The combination of different angle measures to make different triangles are:
a. 30°, 60° 90°
b. 40°, 50°, 90°
c. 40°, 60°, 80°

Question 12.
Given two side lengths of 15 units and 9.5 units, with a nonincluded angle of 75°, can you draw no triangles, only one triangle, or more than one triangle?
Answer:
The given side lengths are: 15 units and 9.5 units
The non-included angle is: 75°
Now,
The representation of the triangle with the given specifications are:

Now,
From the above,
We can observe that there is more than one triangle with the given specifications
Hence, from the above,
We can conclude that we can draw more than one triangle with the given specifications

Question 13.
A student was asked to form different triangles with angle measures of 90°, 30°, and 60°. She incorrectly said this triangle is the only triangle with angle measures of 90°, 30°, and 60°. What mistake might she have made?
Envision Math Common Core 7th Grade Answers Topic 8 Solve Problems Involving Geometry 32
Answer:
It is given that
A student was asked to form different triangles with angle measures of 90°, 30°, and 60°. She incorrectly said this triangle is the only triangle with angle measures of 90°, 30°, and 60°
Now,
We know that,
We can form any number of triangles as long as the sum of the angles is 180°
So,
The other combinations of the given angle measures are:
a. 30°, 60°, 90°
b. 40°, 60°, 80°
c. 30°, 50°, 100°
Hence, from the above,
We can conclude that the mistake done by her is:
She did not consider the other shapes of the triangles by adjusting the given angle measures

Question 14.
In triangle QRS, m∠QSR = 100°, m∠SQR = 45°, and QR = 4 units. In triangle XYZ, m∠XYZ = 100°, m∠ZXY = 45°, and XY = 4 units. Are triangles QRS and XYZ the same? Explain.
Answer:

Question 15.
You are asked to make a triangular sign using the given information about triangle WXY. In triangle WXY, m∠WXY = 45°, m∠YWX = 90°, and WX = 5 feet.
a. Which triangle is correct? Each square on the grid is equal to 1 square foot.
Envision Math Common Core 7th Grade Answers Topic 8 Solve Problems Involving Geometry 33
Answer:
It is given that
You are asked to make a triangular sign using the given information about triangle WXY. In triangle WXY, m∠WXY = 45°, m∠YWX = 90°, and WX = 5 feet.
So,
The representation of the triangle with the given specifications is:

Hence, from the above,
We can conclude that Option B matches the given above figure

b. Explain why only one triangle can be formed with these three pieces of information.
Answer:
We know that,
It is not always possible to construct a triangle from a given side length and two angles. If the two given angles add to more than 180°, then the sides of the triangle will diverge and never meet.
Hence, from the above,
We can conclude that only one triangle can be formed by the given information in part (a)

Question 16.
Look for Relationships Two different triangles have side lengths of 13 and 16 units and a non-included angle of 50°. Explain how the triangles are different.
Answer:
It is given that
Two different triangles have side lengths of 13 and 16 units and a non-included angle of 50°
Now,
The representations of the triangle with the given specifications are: (Scale: 1 cm = 2 units)

Now,
From the above,
We can observe that the angles are different for the two representations
Hence, from the above,
We can conclude that the representations of the triangles are different due to the remaining angle measures other than the non-included angle measure

Question 17.
Higher-Order Thinking Two triangles have side lengths of 12 units and 15 units and a non-included angle of 45°. Draw two different triangles with these conditions.
Answer:
It is given that
Two triangles have side lengths of 12 units and 15 units and a non-included angle of 45°
Hence,
The representation of the triangle with the given specifications are:

Question 18.
For triangle RST, RS is 12 centimeters, ST is 16 centimeters, and RT is 19 centimeters. How many triangles can be drawn with the given side lengths?
Answer:
It is given that
For triangle RST, RS is 12 centimeters, ST is 16 centimeters, and RT is 19 centimeters.
So,
The representation of the triangle with the given specifications is:

Hence, from the above,
We can conclude that only one triangle can be drawn with the given side lengths

Question 19.
A triangle has two side lengths of 8.5 centimeters and 9.5 centimeters. What is a possible length for the third side? Explain why this is a possible length.
Answer:
It is given that
A triangle has two side lengths of 8.5 centimeters and 9.5 centimeters.
Now,
We know that,
In order to construct a triangle,
The sum of the lengths of the two shortest sides must be greater than the length of the longer side
Now,
Let the third side be x
Case 1:
8.5 + 9.5 > x
18 > x
x < 18
Case 2:
8.5 + x > 9.5
x > 9.5 – 8.5
x > 1
Hence, from the above,
We can conclude that the possible length of the third side must be: 1 < x < 18

Question 20.
Can a triangle be formed with side lengths of 4, 5, and 7 units?
Answer:
The given side lengths are: 4 units, 5 units, and 7 units
Now,
We know that,
In order to construct a triangle,
The sum of the lengths of the two shortest sides must be greater than the length of the longer side
So,
4 + 5 > 7
9 > 7
Hence, from the above,
We can conclude that the triangle can be formed wit the given side lengths 4, 5, and 7 units

Assessment Practice
Question 21.
Which of the following combinations of side lengths would form a triangle? Select all that apply.
☐ 7 in., 10 in., 2.5 in.
☐ 4.5 ft, 8 ft, 5 ft
☐ 5 yd, 11 yds, 5 yd
☐ 12 in., 5 in., 9.5 in.
☐ 7 m, 7 m, 9 m
☐ 6 ft, 16 ft, 9 ft
Answer:
We know that,
In order to construct a triangle,
The sum of the lengths of the two shortest sides must be greater than the length of the longer side
Hence,
The combinations of side lengths that would form a triangle are:

Question 22.
Which of the following combinations of side lengths would NOT form a triangle?
A. 7 cm, 10 cm, 13 cm
B. 10 ft, 13 ft, 15 ft
C. 10 yd, 11 yd, 13 yd
D. 10 in., 13 in., 23 in.
Answer:
We know that,
In order to construct a triangle,
The sum of the lengths of the two shortest sides must be greater than the length of the longer side
Hence,
The combinations of side lengths that would not form a triangle are:

Question 23.
Draw a triangle that has exactly one line of symmetry.
Envision Math Common Core 7th Grade Answers Topic 8 Solve Problems Involving Geometry 34
Answer:
We know that,
The triangle, which has two equal sides, two equal angles and exactly one line of symmetry is known as the Isosceles Triangle. So, ΔABC is an Isosceles Triangle. Therefore, the triangle which has only one line of symmetry has been known as the Isosceles Triangle.
Hence,
The representation of the triangle that has exactly on the line of symmetry is:

Lesson 8.4 Solve Problems Using Angle Relationships

Explore It!

The intersecting skis form four angles.
Envision Math Common Core 7th Grade Answers Topic 8 Solve Problems Involving Geometry 35

I can… solve problems involving angle relationships.

A. List all the pairs of angles that share a ray.
Answer:
It is given that the intersecting skis form four angles.
Now,
The representation of the intersecting skies and the angles made by intersection are:
Envision Math Common Core 7th Grade Answers Topic 8 Solve Problems Involving Geometry 35
Hence, from the above figure,
We can conclude that the angles that share a ray are:
∠1, ∠2, ∠3, and ∠4

B. Suppose the measure of Z1 increases. What happens to the size of ∠2? ∠3?
Answer:
From the given figure,
We can observe that ∠1 and ∠2 are on the same side and they are known as “Adjacent angles”
We can observe that ∠1 and ∠3 are on the opposite sides of the ray and they are known as “Vertical angles”
Now,
We know that,
The sum of the adjacent angles is 180°
The angle measures of the vertical angles are the same
Hence, from the above,
We can conclude that
When the value of ∠1 increases, the value of ∠2 automatically decreases
When the value of ∠1 increases, the value of ∠3 also increases

C. How does the sum of the measures of ∠1 and ∠2 change when one ski moves? Explain.
Answer:
From the given figure,
We can observe that ∠1 and ∠2 lies on the same side of the ski and they are known as “Adjacent angles” (or) “Supplementary angles”
Now,
We know that,
The sum of the angle measures of adjacent angles is 180°
Hence, from the above,
We can conclude that the sum of the measures of ∠1 and ∠2 always remains constant even when one ski moves

Focus on math practices
Construct Arguments Why does the sum of all four angle measures stay the same when one of the skis moves?
Answer:
From the given figure,
We can observe that the intersecting skies form 4 angles
Now,
We know that,
The sum of the adjacent angles’ angle measures is 180°
The angle measures of the vertical angles are the same
So,
∠1 + ∠2 = 180° and ∠3 + ∠4 = 180°
∠1 = ∠3 and ∠2 = ∠4
So,
∠1 + ∠2 + ∠3 + ∠4 = 360°
So,
Even when we change one angle measure, the remaining angle measures will adjust their angle measures accordingly so that the sum of all the four angles remain the same i.e, 360°
Hence, from the above,
We can conclude that the sum of all four angle measures stay the same even when one of the skis moves

Essential Question
How are angles formed by intersecting lines related?
Answer:
When two lines intersect, the angles that are opposite each other are vertical angles. Recall that a linear pair is a pair of adjacent angles whose non-common sides are opposite rays. So, when two lines intersect, the angles that are on the same side of a line form a linear pair.

Try It!
∠MNQ and ∠PNR are vertical angles. What is the value of x?
Envision Math Common Core 7th Grade Answers Topic 8 Solve Problems Involving Geometry 36
Vertical angles are __________, so the equation ___________ can be used to find x. The value of x is __________.
Answer:
It is given that ∠MNQ and ∠PNR are vertical angles
Now,
The given figure is:
Envision Math Common Core 7th Grade Answers Topic 8 Solve Problems Involving Geometry 36
Now,
We know that,
The angle measures of the vertical angles are the same
So,
(3x – 6)° = 114°
3x° = 114° + 6°
3x° = 120°
x° = \(\frac{120°}{3}\)
x° = 40°
Hence, from the above,
We can conclude that
Since the vertical angles are congruent, the equation (3x – 6)° = 114° can be used to find the value of x
Hence,
The value of x is: 40°

Convince Me! Why can you use an equation when solving for x in the diagram?
Answer:
The given figure is:
Envision Math Common Core 7th Grade Answers Topic 8 Solve Problems Involving Geometry 36
Now,
We know that,
The vertical angles are always congruent
So,
The angle measures of the vertical angles are also congruent
Now,
From the given figure,
We can observe that ∠MNQ and ∠PNR are vertical angles
So,
∠MNQ = ∠PNR
So,
(3x – 6)° = 114°
Hence, from the above,
We can conclude that we will use the equation to solve for x since the given angles are the vertical angles

Try It!
m∠1 is 4 times m∠2. ∠1 and ∠2 are complementary. ∠1 and ∠3 are vertical angles. ∠3 and ∠4 are supplementary. What are the measures of the four angles?
Answer:
It is given that
m∠1 is 4 times m∠2. ∠1 and ∠2 are complementary. ∠1 and ∠3 are vertical angles. ∠3 and ∠4 are supplementary
Now,
The given relation between ∠1 and ∠2 is:
∠2 = 4∠1
Now,
From the given information,
∠1 + ∠2 = 90°
∠1 + 4∠1 = 90°
5∠1 = 90°
∠1 = \(\frac{90°}{5}\)
∠1 = 18°
So,
∠2 = 4 ∠1
∠2 = 4 (18°)
∠2 = 72°
Now,
From the given information,
∠1 and ∠3 are vertical angles. ∠3 and ∠4 are supplementary
So,
∠1 = ∠3 = 18°
Now,
∠3 + ∠4 = 180°
∠4 = 180° – 18°
∠4 = 162°
Hence, from the above,
We can conclude that the measures of the four angles are:
∠1 = 18°, ∠2 = 72°, ∠3 = 18°, and ∠4 = 162°

KEY CONCEPT
Envision Math Common Core 7th Grade Answers Topic 8 Solve Problems Involving Geometry 37

Do You Understand?

Question 1.
Essential Question How are angles formed by intersecting lines related?
Answer:
When two lines intersect, the angles that are opposite each other are vertical angles. Recall that a linear pair is a pair of adjacent angles whose non-common sides are opposite rays. So, when two lines intersect, the angles that are on the same side of a line form a linear pair.

Question 2.
Use Structure Can vertical angles also be adjacent angles? Explain.
Answer:
No, vertical angles can never be adjacent. Adjacent angles are the ones next to each other while vertical angles are opposite from each other.

Question 3.
Reasoning Do complementary and supplementary angles also have to be adjacent angles? Explain.
Answer:
Supplementary and complementary angles do not have to be adjacent, but they can be possible. Supplementary angles are two angles whose measures sum to 180 ° and complementary are the sum have to add up to 90 °

Do You Know How?

Use the diagram below for 4-6.
Envision Math Common Core 7th Grade Answers Topic 8 Solve Problems Involving Geometry 38
Answer:
The given figure is:
Envision Math Common Core 7th Grade Answers Topic 8 Solve Problems Involving Geometry 38

Question 4.
List two pairs of adjacent angles.
Answer:
From the given figure,
We can observe that
The two pairs of adjacent angles are:
a. ∠1 and ∠2      b. ∠3 and ∠4

Question 5.
List all pairs of vertical angles.
Answer:
From the given figure,
We can observe that
The pairs of vertical angles are:
∠1 and ∠3

Question 6.
If ∠1 and ∠3 are the same measure, what is the value of x?
Answer:
From the given figure,
We can observe that
∠1 = 9x° and ∠3 = 90°
Now,
It is given that ∠1 and ∠3 are the same measure
So,
∠1 = ∠3
So,
9x° = 90°
x = \(\frac{90°}{9}\)
x = 10°
Hence, from the above,
We can conclude that the value of x is: 10°

Practice & Problem Solving

Question 7.
List each angle adjacent to ∠w.
Envision Math Common Core 7th Grade Answers Topic 8 Solve Problems Involving Geometry 39
Answer:
The given figure is:
Envision Math Common Core 7th Grade Answers Topic 8 Solve Problems Involving Geometry 39
Now,
From the given figure,
We can observe that
The angles adjacent to ∠w are: ∠x and ∠z
Hence, from the above,
We can conclude that the angles adjacent to ∠w are: ∠x and ∠z

Question 8.
List two pairs of adjacent angles.
Envision Math Common Core 7th Grade Answers Topic 8 Solve Problems Involving Geometry 40
Answer:
The given figure is:
Envision Math Common Core 7th Grade Answers Topic 8 Solve Problems Involving Geometry 40
Now,
We know that,
The “Adjacent angles” are the angles that share the common vertex and common side
Now,
From the given figure,
We can observe that
The adjacent angles are: ∠KOL and ∠NOM
Hence, from the above,
We can conclude that the two pairs of adjacent angles are: ∠KOL and ∠NOM

Question 9.
Find the value of x.
Envision Math Common Core 7th Grade Answers Topic 8 Solve Problems Involving Geometry 41
Answer:
The given figure is:
Envision Math Common Core 7th Grade Answers Topic 8 Solve Problems Involving Geometry 41
Now,
From the given figure,
We can observe that 9x° and 18° are the complementary angles
Now,
We know that,
The sum of the complementary angles is always 90°
So,
9x° + 18° = 90°
9x°= 90° – 18°
9x° = 72°
x = \(\frac{72°}{9}\)
x = 8°
Hence, from the above,
We can conclude that the value of x is: 8°

Question 10.
Find the value of x.
Envision Math Common Core 7th Grade Answers Topic 8 Solve Problems Involving Geometry 42
Answer:
The given figure is:
Envision Math Common Core 7th Grade Answers Topic 8 Solve Problems Involving Geometry 42
Now,
Fro the given figure,
We can observe that 125° and (5x + 30)° are vertical angles
Now,
We know that,
The vertical angles are always congruent
So,
(5x + 30)° = 125°
5x° = 125° – 30°
5x° = 95°
x = \(\frac{95°}{5}\)
x° = 19°
Hence, from the above,
We can conclude that the value of x is: 19°

Question 11.
∠1 and ∠2 are complementary angles. The measure of ∠1 is 42°. The measure of ∠2 is (3x)°. Find the value of x.
Envision Math Common Core 7th Grade Answers Topic 8 Solve Problems Involving Geometry 43
Answer:
It is given that
∠1 and ∠2 are complementary angles. The measure of ∠1 is 42°. The measure of ∠2 is (3x)°.
Now,
The given figure is:
Envision Math Common Core 7th Grade Answers Topic 8 Solve Problems Involving Geometry 43
Now,
We know that,
The sum of the complementary angles is always 90°
So,
∠1 + ∠2 = 90°
42°+ 3x° = 90°
3x° = 90° – 42°
3x° = 48°
x° = \(\frac{48°}{3}\)
x°= 16°
Hence, from the above,
We can conclude that the value of x is: 16°

Question 12.
Two streets form an intersection. ∠C and ∠D are supplementary angles. If the measure of ∠C is 128° and the measure of ∠D is two times the value of x, what is the value of x?
Envision Math Common Core 7th Grade Answers Topic 8 Solve Problems Involving Geometry 44
Answer:
It is given that
Two streets form an intersection. ∠C and ∠D are supplementary angles. The measure of ∠C is 128° and the measure of ∠D is two times the value of x
Now,
The given figure is:
Envision Math Common Core 7th Grade Answers Topic 8 Solve Problems Involving Geometry 44
Now,
We know that,
The sum of the supplemenatry angles is always 180°
So,
∠C + ∠D = 180°
128° + 2x° = 180°
2x° = 180° – 128°
2x° = 52°
x° = \(\frac{52°}{2}\)
x° = 26°
Hence, from the above,
We can conclude that the value of x is: 26°

Question 13.
If A and B are supplementary angles and ∠A is three times as large as ∠B, find the measures of ∠A and ∠B.
Answer:
It is given that
A and B are supplementary angles and ∠A is three times as large as ∠B
Now,
Let the value of ∠B be x°
So,
The value of ∠A will be: 3x°
Now,
We know that,
The sum of the supplementary angles is always 180°
So,
∠A + ∠B = 180°
So,
3x° + x° = 180°
4x° = 180°
x° = \(\frac{180°}{4}\)
x°= 45°
So,
∠B = 45°
∠A = 3x° = 3 (45°) = 135°
Hence, from the above,
We can conclude that the angle measures of ∠A and ∠B are 135° and 45° respectively

Question 14.
Higher-Order Thinking The measure of ∠DBE is (0.1x – 22)° and the measure of ∠CBE is (0.3x – 54)°. Find the value of x.
Envision Math Common Core 7th Grade Answers Topic 8 Solve Problems Involving Geometry 45
Answer:
It is given that
The measure of ∠DBE is (0.1x – 22)° and the measure of ∠CBE is (0.3x – 54)°
Now,
The given figure is:
Envision Math Common Core 7th Grade Answers Topic 8 Solve Problems Involving Geometry 45
Now,
From the given figure,
We can observe that ∠DBE and ∠CBE are the complementary angles
Now,
We know that,
The sum of the complementary angles is always 90°
So,
∠DBE + ∠CBE = 90°
(0.1x – 22)° + (0.3x – 54)° = 90°
0.4x° – 76° = 90°
0.4x°= 90° + 76°
0.4x° = 166°
4x° = 1,660°
x° = \(\frac{1,660°}{4}\)
x° = 415°
Hence, from the above,
We can conclude that the value of x is: 415°

Question 15.
Reasoning ∠1 and an angle that measures 50° are supplementary. Another angle that measures 50° and ∠3 is supplementary. Show that m∠1 and m∠3 are equal.
Answer:
It is give that
∠1 and an angle that measures 50° are supplementary. Another angle that measures 50° and ∠3 is supplementary
Now,
We know that,
The sum of the supplementary angles is always 180°
Now,
Let the unkown angle be ∠x
Now,
∠1 + ∠x = 180°
∠1 + 50° = 180°
∠1 = 180° – 50°
∠1 = 130°
Now,
∠x + ∠3 = 180°
∠3 + 50° = 180°
∠3 = 180° – 50°
∠3 = 130°
Hence, from the above,
We can conclude that
∠1 = ∠3

Assessment Practice
Question 16.
Using the diagram at the right, Martin incorrectly writes m∠b = 125° What mistake did Martin likely make? Find the correct measure of ∠b.
Envision Math Common Core 7th Grade Answers Topic 8 Solve Problems Involving Geometry 46
Answer:
It is given that
Martin incorrectly writes
m∠b = 125°
Now,
The given figure is:
Envision Math Common Core 7th Grade Answers Topic 8 Solve Problems Involving Geometry 46
Now,
From the give figure,
We can observe that ∠b and 55° are complementary angles
Now,
We know that,
The sum of complementary angles is always 90°
So,
Both the angles in the complementary angles must be acute angles i.e., the angle must be less than 90°
Now,
∠b + 55° = 90°
∠b = 90° – 55°
∠b = 35°
Hence, from the above,
We can conclude that
The mistake made by Martin is: Consideration of the angles as supplementary angles even though they are complementary angles
The correct measure of ∠b is: 35°

Question 17.
What is the measure, in degrees, of angle x?
Envision Math Common Core 7th Grade Answers Topic 8 Solve Problems Involving Geometry 47
Answer:
The given figure is:
Envision Math Common Core 7th Grade Answers Topic 8 Solve Problems Involving Geometry 47
Now,
From the given figure,
We can observe that
x° and 107° are adjacent angles
Now,
We know that,
The sum of the adjacent angles is always 180°
So,
x° + 107° = 180°
x° = 180° – 107°
x°= 73°
Hence, from the above,
We can conclude that the value of x is: 73°

Question 18.
What is the measure, in degrees, of the highlighted angle?
Envision Math Common Core 7th Grade Answers Topic 8 Solve Problems Involving Geometry 48
Answer:
The given figure is:
Envision Math Common Core 7th Grade Answers Topic 8 Solve Problems Involving Geometry 48
Now,
From the given figure,
We can observe that
x° and 2x° are the supplementary angles
Now,
We know that,
The sum of the supplementary angles is always 180°
So,
x° + 2x° = 180°
3x° = 180°
x° = \(\frac{180°}{3}\)
x°= 60°
So,
2x° = 2 (60°) = 120°
Hence, from the above,
We can conclude that the value of the highlighted angle is: 120°

Lesson 8.5 Solve Problems Involving Circumference of a Circle

Explore It!

The distance around a circle and the distance across a circle are related.
Envision Math Common Core 7th Grade Answers Topic 8 Solve Problems Involving Geometry 49
I can… solve problems involving radius, diameter, and circumference of circles.

A. Use string to measure the distance across each circle. How many of these lengths does it take to go completely around the circle?
Answer:
We know that,
The distance around the circle is: Circumference
The distance across the circle is: Diameter
Now,
The complete length to go completely around the circle is given as:
Circumference = πd
Hence, from the above,
We can conclude that
The total length does it take to go completely around the circle = πd
Where
d is the diameter

B. Use the string and a ruler to measure the distance across the circle and the distance around the circle. Complete the table. Round each measurement to the nearest quarter inch.
Envision Math Common Core 7th Grade Answers Topic 8 Solve Problems Involving Geometry 50
Answer:

C. What do you notice about the ratio of the distance around the circle to the distance across the circle for each circle?
Answer:
From the given table,
We can observe that the ratio of the circumference and the diameter is constant for all the circles
Hence, from the above,
We can conclude that the ratio of the distance around the circle to the distance across the circle for each circle is constant

Focus on math practices
Look for Relationships How can you estimate the distance around any circle when given the distance across the circle?
Answer:
We know that,
The distance around the circle is: Circumference
The distance across the circle is: Diameter
Now,
We know that,
The relationship between the distance around the circle and the distance across the circle is:
Distance around the circle = π × Distance across the circle
Hence, from the above,
We can conclude that
Circumference (C) = πd
Where,
d is the diameter

Essential Question
How is the circumference of a circle related to the length of its diameter?
Answer:
The relationship between the circumference of a circle and the diameter of the circle is:
Circumference (C) = πd
Where,
d is the diameter

Try It!
What is the circumference of the rim of a basketball hoop with a radius of 9 inches?
Envision Math Common Core 7th Grade Answers Topic 8 Solve Problems Involving Geometry 51
First, multiply the radius by __________ to get the diameter, __________ inches. Then, multiply the diameter by 3.14 (an approximation for π) to get a circumference of about __________ inches.
Answer:
The given figure is:
Envision Math Common Core 7th Grade Answers Topic 8 Solve Problems Involving Geometry 51
Now,
We know that,
The diameter of the rim (d) = 2 × Radius of the rim (r)
So,
d = 2 × 9
d = 18 in.
Now,
We know that,
Circumference (C) = πd
So,
C = 3.14 × 18
= 56.52 in.
Hence, from the above,
We can conclude that the circumference of the rim of a basketball hoop is about 56.52 in.

Convince Me! If the diameter is doubled, what happens to the circumference? Explain.
Answer:
We know that,
Circumference (C) = πd
So,
When the diameter is doubled,
Circumference (C) = π × 2d
Hence, from the above,
We can conclude that
If the diameter is doubled, then the circumference will also be doubled

Try It!
The circle has a circumference of 9.42 units. What is the area of the square? Use 3.14 for π. Explain how you found the answer.
Envision Math Common Core 7th Grade Answers Topic 8 Solve Problems Involving Geometry 52
Answer:
It is given that the circle has a circumference of 9.42 units
Now,
The given figure is:
Envision Math Common Core 7th Grade Answers Topic 8 Solve Problems Involving Geometry 52
Now,
From the given figure,
We can observe that
The radius of the circle = The side of the square
Now,
We know that,
The circumference of a circle (C) = 2πr
So,
2πr = 9.42
r = \(\frac{9.42}{2π}\)
r = 1.5 units
So,
The side of the square is: 1.5 units
Now,
We know that,
The area of the square = Side²
So,
The area of the square = (1.5)²
= 2.25 units²
Hence, from the above,
We can conclude that the area of the square is: 2.25 units²

KEY CONCEPT
The parts of a circle and their relationships are summarized in the diagram below.
Envision Math Common Core 7th Grade Answers Topic 8 Solve Problems Involving Geometry 53

Do You Understand?

Question 1.
Essential Question How is the circumference of a circle related to the length of its diameter?
Answer:
The relationship between the circumference of a circle and the diameter of the circle is:
Circumference (C) = πd
Where,
d is the diameter

Question 2.
Construct Arguments Are there any circles for which the relationship between the diameter and circumference cannot be represented by π? Explain.
Answer:
We know that,
The circumference of a circle (C) = πd
Where,
d is the diameter
Now,
We will find the circumference of any circle (Semi circle, Quarter circle, etc.) from the above formula only with some modifications
Hence, from the above,
We can conclude that there are not any circles for which the relationship between the diameter and circumference cannot be represented by π

Question 3.
Be Precise Can you find the exact circumference of a circle when you multiply the diameter by \(\frac{22}{7}\)? Explain.
Answer:
No, we can not find the exact circumference of a circle when you multiply the diameter by \(\frac{22}{7}\) because the diameter will not always be the multiple of 7 so that the circumference will be an integer

Do You Know How?

Question 4.
What is the circumference of a circle with a radius of 5 inches?
Answer:
The given data is:
The radius of a circle (r) = 5 inches
Now,
We know that,
The circumference of a circle (C) = 2πr
So,
C = 2 × 3.14 × 5
= 31.4 inches
Hence, from the above,
We can conclude that the circumference of the circle with the given radius is: 31.4 inches

Question 5.
What is the diameter of a circle with a circumference of 10.99 feet?
Answer:
The given data is:
The circumference of a circle (C) = 10.99 feet
Now,
We know that,
The circumference of a circle (C) = πd
So,
10.99 = 3.14 × d
d = \(\frac{10.99}{3.14}\)
d = 3.5 feet
Hence, from the above,
We can conclude that the diameter of the circle with the given circumference is: 3.5 feet

Question 6.
How many full revolutions does a car tire with a diameter of 25 inches make when the car travels one mile?
Envision Math Common Core 7th Grade Answers Topic 8 Solve Problems Involving Geometry 54
Answer:
It is given that
A car tire has a diameter of 25 inches
Now,,
We have to find the circumference of the tire and the speed of the car to find the number of revolutions
Now,
We know that,
The circumference of a circle (C) = πd
So,
C = 3.14 × 25
= 78.5 inches
So,
The circumference of the given car tire is: 78.5 inches
Now,
We know that,
1 mile = 5,280 feet
1 feet = 12 inches
So,
1 mile = 5,280 × 12
= 63,360 inches
Now,
We know that,
The number of revolutions the car tire can make = \(\frac{Speed of a car}{The circumference of a car tire}\)
= \(\frac{63,360}{78.5}\)
≅ 807 revolutions
Hence, from the above,
We can conclude that
The number of revolutions the given car tire can make with the given diameter is about 807 revolutions

Practice & Problem Solving

Question 7.
Find the circumference of the circle. Use π as part of the answer.
Envision Math Common Core 7th Grade Answers Topic 8 Solve Problems Involving Geometry 55
Answer:
The given figure is:
Envision Math Common Core 7th Grade Answers Topic 8 Solve Problems Involving Geometry 55
Now,
From the given figure,
We can observe that
The diameter of a circle (d) = 7 cm
Now,
We know that,
The circumference of a circle (C) = πd
So,
C = 7π cm
Hence, from the above,
We can conclude that the circumference of the circle with the given diameter in terms of π is: 7π cm

Question 8.
Find the circumference of the circle. Use 3.14 for π. Round to the nearest hundredth.
Envision Math Common Core 7th Grade Answers Topic 8 Solve Problems Involving Geometry 56
Answer:
The given figure is:
Envision Math Common Core 7th Grade Answers Topic 8 Solve Problems Involving Geometry 56
Now,
From the given figure,
We can observe that
The diameter of a circle (d) = 20 ft
Now,
We know that,
The circumference of a circle (C) = πd
So,
C = 3.14 × 20
= 62.80 ft
Hence, from the above,
We can conclude that the circumference of the circle with the given diameter is: 62.80 ft

Question 9.
Find the circumference of the circle. Use π as part of the answer.
Envision Math Common Core 7th Grade Answers Topic 8 Solve Problems Involving Geometry 57
Answer:
The given figure is:
Envision Math Common Core 7th Grade Answers Topic 8 Solve Problems Involving Geometry 57
Now,
From the given figure,
We can observe that
The radius of a circle (r) = 12 mi
Now,
We know that,
The circumference of a circle (C) = 2πr
So,
C = 2π × 12
= 24π mi
Hence, from the above,
We can conclude that
The circumference of the circle with the given radius in terms of π is: 24π mi

Question 10.
Find the circumference of the circle. Use 3.14 for π. Round to the nearest hundredth.
Envision Math Common Core 7th Grade Answers Topic 8 Solve Problems Involving Geometry 58
Answer:
The given figure is:
Envision Math Common Core 7th Grade Answers Topic 8 Solve Problems Involving Geometry 58
Now,
From the given figure,
We can observe that
The radius of a circle (r) = 9.5 cm
Now,
We know that,
The circumference of a circle (C) = 2πr
So,
C = 2 × 3.14 × 9.5
= 59.66 cm
Hence, from the above,
We can conclude that the circumference of the circle with the given radius is: 59.66 cm

Question 11.
Find the diameter of a circle with a circumference of 27 centimeters. Use 3.14 for π. Round to the nearest tenth.
Answer:
The given data is:
The circumference of a circle (C) = 27 cm
Now,
We know that,
The circumference of a circle (C) = πd
So,
27 = 3.14 × d
d = \(\frac{27}{3.14}\)
d = 8.59
d ≅ 9 cm
Hence, from the above,
We can conclude that the diameter of the circle with the given circumference is about 9 cm

Question 12.
The distance around a meteor crater is 9,687 feet. Find the diameter of the crater. Use \(\frac{22}{7}\) for π. Round to the nearest tenth.
Envision Math Common Core 7th Grade Answers Topic 8 Solve Problems Involving Geometry 59
Answer:
It is given that
The distance around a meteor crater is 9,687 feet
Now,
The given figure is:
Envision Math Common Core 7th Grade Answers Topic 8 Solve Problems Involving Geometry 59
Now,
From the above,
We can observe that the crater is in the form of a circle
Now,
We know that,
The distance around the circle is: Circumference
Now,
We know that,
The circumference of a circle (C) = πd
So,
9,687 = \(\frac{22}{7}\) × d
d = \(\frac{9,687 × 7}{22}\)
d = 3,082.2 feet
Hence, from the above,
We can conclude that the diameter of the crater for the given circumference of a crater is: 3,082.2 feet

Question 13.
Make Sense and Persevere The circumference of the inner circle is 44 feet. The distance between the inner circle and the outer circle is 3 feet. By how many feet is the circumference of the outer circle greater than the circumference of the inner circle? Use \(\frac{22}{7}\) for π. Round to the nearest hundredth of a foot.
Envision Math Common Core 7th Grade Answers Topic 8 Solve Problems Involving Geometry 60
Answer:
It is given that
The circumference of the inner circle is 44 feet. The distance between the inner circle and the outer circle is 3 feet
Now,
We know that,
The circumference of inner circle (c) = 2πr
So,
For inner circle:
44 = 2 × \(\frac{22}{7}\) × r
r = \(\frac{44 × 7}{22 × 2}\)
r = 7 feet
For outer circle:
The circumference of the outer circle (C) = 2π(R – r)
Where,
R is the radius of the outer circle
r is the radius of the inner circle
So,
C = 2 × \(\frac{22}{7}\) × (7 + 3)
C = 62.85 feet
So,
C – c = 62.85 – 44
= 18.85 feet
Hence, from the above,
We can conclude that the circumference of the outer circle is 18.85 feet greater than the circumference of the inner circle

Question 14.
Generalize What is the ratio of the radius to the circumference of any circle, using 3.14 for π?
Answer:
We know that,
The circumference of a circle (C) = 2πr
So,
\(\frac{C}{r}\) = 2π
\(\frac{C}{r}\) = 6.28
Hence, from the above,
We can conclude that the ratio of the radius to the circumference of any circle is: 6.28

Question 15.
What is the radius of a circle with a circumference of 26.69 centimeters?
Answer:
The given data is:
The circumference of a circle (C) = 26.69 cm
Now,
We know that,
The circumference of a circle (C) = 2πr
So,
26.69 = 2 × 3.14 × r
r = \(\frac{26.69}{6.28}\)
r = 4.25 cm
Hence, from the above,
We can conclude that the radius of the circle with the given circumference is: 4.25 cm

Question 16.
Higher Order Thinking A unicycle wheel makes five rotations. The unicycle travels 37.94 feet. Find the diameter of the wheel in inches. Use 3.14 for π. Round to the nearest tenth of an inch.
Envision Math Common Core 7th Grade Answers Topic 8 Solve Problems Involving Geometry 61
Answer:
It is given that
A unicycle wheel makes five rotations. The unicycle travels 37.94 feet
Now,
We know that,
The number of revolutions (or) rotations = \(\frac{The speed of the unicycle}{The circumference of the unicycle wheel}\)
So,
The circumference of the unicycle wheel = \(\frac{The speed of the unicycle}{The number of rotations}\)
3.14 × d = \(\frac{37.94}{5}\)
3.14 × d = 7.58
d = \(\frac{7.58}{3.14}\)
d = 2.41 feet
d = 2.41 × 12
d = 29 inches
Hence, from the above,
We can conclude that the diameter of the wheel in inches is: 29 inches

Assessment Practice
Question 17.
Camille drew the figure shown at the right. Which of the following is the best estimate of the perimeter of the figure?
Envision Math Common Core 7th Grade Answers Topic 8 Solve Problems Involving Geometry 62
A. 36 feet
B. 81 feet
C. 45 feet
D. 50 feet
Answer:

Question 18.
A cabin on a Ferris wheel has traveled one-fourth of the circumference of the wheel, a distance of 117.75 feet. What is the radius, in feet, of the Ferris wheel? Use 3.14 for π.
Answer:
It is given that
A cabin on a Ferris wheel has traveled one-fourth of the circumference of the wheel, a distance of 117.75 feet
Now,
The circumference of the wheel (C) = 4 × 117.75
= 471 feet
Now,
We know that,
The circumference of a circle (C) = 2πr
So,
471 = 2 × 3.14 × r
r = \(\frac{471}{6.28}\)
r = 75 feet
Hence, from the above,
We can conclude that the radius of the Ferris wheel is: 75 feet

Question 19.
The diagram shows a track composed of a rectangle with a semicircle on each end. The area of the rectangle is 7,200 square meters. What is the perimeter, in meters, of the track? Use 3.14 for π.
Envision Math Common Core 7th Grade Answers Topic 8 Solve Problems Involving Geometry 63
Answer:
It is given that
The diagram shows a track composed of a rectangle with a semicircle on each end. The area of the rectangle is 7,200 square meters.
Now,
The given figure is:
Envision Math Common Core 7th Grade Answers Topic 8 Solve Problems Involving Geometry 63
Now,
We know that,
The area of the rectangle (A) = Length × Width
So,
7,200 = 120 × Width
Width = \(\frac{7,200}{120}\)
Width = 60 m
So,
The radius of the semicircle (r) = \(\frac{Width of the rectangle}{2}\)
r = \(\frac{60}{2}\)
r = 30 m
Now,
We know that,
The perimeter of a rectangle = 2 (Length + Width)
The perimeter of a semicircle = r (π + 2)
Now,
The perimeter of a rectangle = 2 (120 + 60)
= 2 (180)
= 360 m
The perimeter of a semicircle = 30 (3.14 + 2)
= 154.2 m
So,
The perimeter of the track = The perimeter of a rectangle + 2 (The perimeter of a semicircle)
= 360 + 2 (154.2)
= 668.4 m
Hence, from the above,
We can conclude that the perimeter of the given track is: 668.4 m

Topic 8 MID-TOPIC CHECKPOINT

Question 1.
Vocabulary How are adjacent angles and vertical angles alike? How are they different? Lesson 8-4
Answer:
We know that,
The adjacent angles and the vertical angles have the same vertex
We know that,
The sum of the adjacent angles is 180° (or) 90°

Question 2.
On a map, 1 inch equals 150 miles. The border between the two states is 5.5 inches long on the map. What is the actual length of the border? Lesson 8-1
Answer:
It is given that
On a map, 1 inch equals 150 miles. The border between the two states is 5.5 inches long on the map.
So,
The actual length of the border – (Scale factor) × (The border between the two states on the amp)
= 150 × 5.5
= 825 miles
Hence, from the above,
We can conclude that the actual length of the border is: 825 miles

In 3 and 4, use the figure to the right.
Envision Math Common Core 7th Grade Answers Topic 8 Solve Problems Involving Geometry 64
Question 3.
What is the measure of ∠BZD? Lesson 8-4
A. 58°
B. 148°
C. 32°
D. 90°
Answer:
The given figure is:
Envision Math Common Core 7th Grade Answers Topic 8 Solve Problems Involving Geometry 64
Now,
From the given figure,
We can observe that
∠BZD = ∠BZC + ∠CZD
So,
∠BZD = 58° + 90°
∠BZD = 148°
Hence, from the above,
We can conclude that
The angle measure of ∠BZD is:

Question 4.
Find the value of x. Lesson 8-4
Answer:
The given figure is:
Envision Math Common Core 7th Grade Answers Topic 8 Solve Problems Involving Geometry 64
Now,
From the given figure,
We can observe that
(2x)° + 58° = 180°
(2x)° = 180° – 58°
(2x)° = 122°
x = \(\frac{122°}{2}\)
x = 61°
Hence,f rom the above,
We can conclude that the value of x is: 61°

Question 5.
Pierce draws a circle with a radius of 3 centimeters. Gianna draws a circle with a radius that is twice as long as the radius of Pierce’s circle. How will the circumference of Gianna’s circle compare with the circumference of Pierce’s circle? Lesson 8-5
The circumference of Gianna’s circle is ___________ times the circumference of Pierce’s circle.
Answer:
It is given that
Pierce draws a circle with a radius of 3 centimeters. Gianna draws a circle with a radius that is twice as long as the radius of Pierce’s circle
So,
The radius of Pierce’s circle is: 3 cm
So,
The radius of Gianna’s circle = 2 × (Radius of Pierce’s circle)
= 2 × 3
= 6 cm
Now,
The circumference of Pierce’s circle (C) = 2πr
= 2 × π × 3
= 6π cm
Now,
The circumference of Gianna’s circle (C) = 2πr
= 2 × π × 6
= 12π cm
Now,
Let the number of times Gianna’s circumference is a number of times as Pierce’s circumference be x
So,
x × 6π = 12π
x = \(\frac{12π}{6π}\)
x = 2
Hence, from the above,
We can conclude that
The circumference of Gianna’s circle is 2 times the circumference of Pierce’s circle.

Question 6.
Draw a triangle with one side length of 5 units and another side length of 7 units. What additional piece of information will guarantee that only one triangle can be drawn? Lessons 8-2 and 8-3
Answer:
The representation of the triangle with the given side lengths is:

Now,
The additional information that will guarantee that only one triangle can be drawn is:
The angle between the two given side lengths i.e., an included angle

Topic 8 MID-TOPIC PERFORMANCE TASK

Mrs. Thomas has two rolls of garden edging that are each 96 inches long. She wants to make two new flower beds in her backyard. Each flower bed will be bordered by one roll of edging. One flower bed will be in the shape of a quadrilateral. The other will be in the shape of a triangle.

PART A
Mrs. Thomas decides to make a scale drawing of each flower bed using a scale of 1 centimeter = 5 inches. What will be the total length of each roll of edging in her scale drawings?
Answer:
It is given that
Mrs. Thomas has two rolls of garden edging that are each 96 inches long. She wants to make two new flower beds in her backyard. Each flower bed will be bordered by one roll of edging. One flower bed will be in the shape of a quadrilateral. The other will be in the shape of a triangle.
Mrs. Thomas decides to make a scale drawing of each flower bed using a scale of 1 centimeter = 5 inches.
Now,
From the given information,
Scale factor = \(\frac{5}{1}\)
So,
The total length of each roll of edging in her scale drawings = \(\frac{The total length of each roll of edging}{Scale factor}\)
= \(\frac{96 × 2}{5}\)
= 19.2 × 2
= 38.4 inches
Hence, from the above,
We can conclude that
The total length of each roll of edging in her scale drawings is: 38.4 inches

PART B
Mrs. Thomas wants the quadrilateral flower bed to have at least two 90° angles. Draw a possible plan for this flower bed using the scale from Part A. Make sure to use a complete roll of edging in the border. Label your drawing with all the angle measures and with the scaled length of each side. Name the shape of the flower bed you drew. What will be its actual dimensions?
Answer:
It is given that
Mrs. Thomas wants the quadrilateral flower bed to have at least two 90° angles
Now,
From part (a),
The scale is: 1 cm = 5 inches
So,
The representation of the quadrilateral flower bed with the scaled measurements is:

Now,
The actual dimensions of the given quadrilateral flower bed are:
The length of the quadrilateral flower bed = (Scaled length) × (Scale factor)
= 4 × 5
= 20 inches
Since the quadrilateral bed is a square bed, all the side lengths are equal
Hence, from the above,
We can conclude that
The shape of the flower bed is: Square
The actual dimensions of the flower bed are: 20 inches, 20 inches, 20 inches, 20 inches

PART C
Mrs. Thomas began to make a drawing for the triangular flower bed. In her drawing, the length of one side of the triangle is 4.8 centimeters, the length of the second side is 6.4 centimeters, and the included angle is a right angle. Use these measures and the scale from Part A to make a completed scale drawing. Label your drawing with all the angle measures to the nearest whole degree and with the scale length of each side. What will be the actual dimensions of this flower bed?
Answer:
It is given that
Mrs. Thomas began to make a drawing for the triangular flower bed. In her drawing, the length of one side of the triangle is 4.8 centimeters, the length of the second side is 6.4 centimeters, and the included angle is a right angle
Now,
From part (a),
The scale is: 1 cm = 5 inches
Now,
The representation of the triangular flower bed with the given specifications is:

Now,
The actual dimensions of the triangular flower garden are:
The actual base length of the flower garden = (The scaled base length of the flower garden) × (Scale factor)
= 6.4 × 5
= 32.0
= 32 inches
The actual height of the flower garden = (The scaled height of the flower garden) × (scale factor)
= 4.8 × 5
= 24.0
= 24 inches
The actual hypotenuse length of the flower garden = (The scaled hypotenuse length of the flower garden) × (Scale factor)
= 8 × 5
= 40 inches
Hence, from the above,
We can conclude that
The actual dimensions of the flower garden are:
Base length: 32 inches
Height: 24 inches
Hypotenuse: 40 inches

Lesson 8.6 Solve Problems Involving Area of a Circle

Explore It!

Latoya cut a circle into 8 equal sections and arranged the pieces to form a shape resembling a parallelogram.
Envision Math Common Core 7th Grade Answers Topic 8 Solve Problems Involving Geometry 65
I can… solve problems involving the area of a circle.

A. How is the base length of the new shape related to the circumference of the circle?
Answer:
It is given that
Latoya cut a circle into 8 equal sections and arranged the pieces to form a shape resembling a parallelogram.
Now,
The given figure is:
Envision Math Common Core 7th Grade Answers Topic 8 Solve Problems Involving Geometry 65
Now,
From the given figure,
We can observe that
The base of the new figure is equal to half of the value of the circumference of the circle
Hence, from the above,
We can conclude that
The base of the new figure = \(\frac{1}{2}\) × Circumference of the circle

B. How is the height of the new shape related to the radius of the circle?
Answer:
The given figure is:
Envision Math Common Core 7th Grade Answers Topic 8 Solve Problems Involving Geometry 65
Now,
From the given figure,
We can observe that the height of the new figure is equal to the radius of the circle
Hence, from the above,
We can conclude that
The height of the new figure = Radius of the circle

C. Since this new shape was made from a circle, use the information from the diagram and the formula for the area of the parallelogram, A= bh, to discover the formula for the area of a circle.
Answer:
It is given that
The area of the parallelogram (A) = bh
Where,
b is the Base of the parallelogram
h is the height of the parallelogram
But,
It is given that
The parallelogram is made from a circle
Now,
From part (a) and (b),
A = \(\frac{1}{2}\) × The circumference of a circle × Radius of a circle
A = \(\frac{1}{2}\) × 2πr × r
A = πr × r
A = πr²
Hence, from the above,
We can conclude that teh area of the circle that is derived fro the area of the parallelogram is: πr²

Focus on math practices
Look for Relationships The formula A = bh can be used to find a good estimate for the area of the cut-out diagram. What would happen to this estimate if the circle was cut into 100 sections? 1,000 sections?
Answer:
The given figure is:
Envision Math Common Core 7th Grade Answers Topic 8 Solve Problems Involving Geometry 65
Now,
From the given figure,
We can observe that
The 8 parts of the circle made 4 parallelograms
So,
The relationship between the circles and parallelograms is:
The number of parallelograms = \(\frac{The number of parts of the circle}{2}\)
So,
For 100 pieces of the circle,
The number of parallelograms = \(\frac{100}{2}\)
= 50
For 1,000 pieces of the circle,
The number of parallelograms = \(\frac{1,000}{2}\)
= 500
Hence,
For 100 pieces of the circle,
The area of the parallelogram (A) = Base × Height [Since base = \(\frac{1}{2}\) × Circumference]
= 50r units²
For 1,000 pieces of the circle,
The area of the parallelogram (A) = Base × Height
= 500r units²

Essential Question
How can the area formula for a circle be used to solve problems?
Answer:
Area of a circle can be calculated by using the formulas:
a. Area = π × r2
where,
‘r’ is the radius.
b. Area = (π/4) × d2
where,
‘d’ is the diameter.
c. Area = C2/4π
where,
‘C’ is the circumference

Try It!
At a school play, there is a spotlight above the center of the floor that covers a lighted area with a radius of 7 feet. What is the area covered by the spotlight?
Envision Math Common Core 7th Grade Answers Topic 8 Solve Problems Involving Geometry 66
The area covered by the spotlight is about _________ square feet.
Answer:
It is given that
At a school play, there is a spotlight above the center of the floor that covers a lighted area with a radius of 7 feet
Now,
We know that,
The area of a circle (A) = πr²
So,
The area covered by the spotlight (A) = \(\frac{22}{7}\) × 7²
A = 154 feet²
Hence, from the above,
We can conclude that the area covered by the spotlight is: 154 square feet

Convince Me! If the diameter of a circle is given, how would you find the area?
Answer:
If the diameter of a circle is given, then
The area of a circle = (π/4) × d2
where,
‘d’ is the diameter.

Try It!
a. How far away can a person live from a radio station and hear its broadcast if the signal covers a circular area of 40,000 square miles? Write your answer as a whole number.
Answer:
It is given that
A person live from a radio station and hear its broadcast if the signal covers a circular area of 40,000 square miles
Now,
We have to find the diameter of a circle to find how far a person lives from a radio station
Now,
We know that,
The area of a circle (A) = πr²
So,
40,000 = 3.14 × r²
r² = \(\frac{40,000}{3.14}\)
r² = 12,738.85
r = 112.8
r ≈ 113 miles
So,
The diameter of a circle = 2 × The radius of a circle
= 2 × 113
= 226 miles
Hence, from the above,
We can conclude that the distance a person can live from a radio station and hear its broadcast is: 226 miles

b. What circular area is covered by the signal if the circumference is 754 miles?
Answer:
It is given that
The circumference of the circular signal is 754 miles
Now,
We know that,
The circumference of a circle (C) = 2πr
So,
754 = 2 × 3.14 × r
r = \(\frac{754}{6.28}\)
r = 120 miles
Now,
The area of the circular signal (A) = πr²
So,
A = 3.14 × (120)²
A = 45,216 miles²
Hence, from the above,
We can conclude that teh circular area of the given signal is: 45,216 miles²

KEY CONCEPT

You can find the area, A, of a circle using the formula A = πr2, where r is the radius.
Envision Math Common Core 7th Grade Answers Topic 8 Solve Problems Involving Geometry 67

Do You Understand?

Question 1.
Essential Question How can the area formula for a circle be used to solve problems?
Answer:
Area of a circle can be calculated by using the formulas:
a. Area = π × r2
where,
‘r’ is the radius.
b. Area = (π/4) × d2
where,
‘d’ is the diameter.
c. Area = C2/4π
where,
‘C’ is the circumference

Question 2.
Be Precise
Is an area calculation exact when you use 3.14 or \(\frac{22}{7}\) as a value for π? Explain.
Answer:
The constant (pi) is not equal to either 22/7 or 3.14; these are only approximations to . The actual value of is the ratio of the circumference of a circle to its diameter, and is a transcendental number with infinitely many non-repeating decimal places

Question 3.
Use Structure if you know the diameter of a circle, how can you find the area?
Answer:
If you know the diameter of a circle, then
The area of a circle = (π/4) × d2
where,
‘d’ is the diameter.

Do You Know How?

For 4-7, use 3.14 for π.
Question 4.
What is the area of a circle with a radius of 8 inches?
Answer:
The given data is:
The radius of a circle is 8 inches
Now,
We know that,
The area of a circle (A) = πr²
So,
A = 3.14 × 8²
= 200.96 inches²
Hence, from the above,
We can conclude that the area of the circle with the given radius is: 200.96 inches²

Question 5.
What is the radius of a circle with an area of 28.26 square feet?
Answer:
The given data is:
The area of a circle is: 28.26 square feet
Now,
We know that,
The area of a circl (A) = πr²
So,
28.26 = 3.14 × r²
r² = \(\frac{28.26}{3.14}\)
r² = 9
r = 3 feet
Hence, from the above,
We can conclude that the radius of the circle with the given area is: 3 feet

Question 6.
What is the area of a circle with a circumference of 25.12 meters?
Answer:
The given data is:
The circumference of a circle is: 25.12 meters
Now,
We know that,
The circumference of a circle (C) = 2πr
So,
25.12 = 2 × 3.14 × r
r = \(\frac{25.12}{6.28}\)
r = 4 meters
Now,
We know that,
The area of a circle (A) = πr²
So,
A = 3.14 × 4²
= 50.24 sq. meters
Hence, from the above,
We can conclude that the area of the circle with the given circmference is: 50.24 sq.meters

Question 7.
The diameter of a pizza is 12 inches. What is its area?
Envision Math Common Core 7th Grade Answers Topic 8 Solve Problems Involving Geometry 68
Answer:
It is given that
The diameter of a pizza is 12 inches.
Now,
The given figure is:
Envision Math Common Core 7th Grade Answers Topic 8 Solve Problems Involving Geometry 68
Now,
We know that,
Radius of a circle (r) = \(\frac{Diameter of a circle}{2}\)
r = \(\frac{12}{2}\)
r = 6 in.
Now,
We know that,
The area of a circle (A) = πr²
So,
A = 3.14 × 6²
= 113.04 in.²
Hence, from the above,
We can conclude that the area of the circle with the given diameter is: 113.04 in.²

Practice & Problem Solving

Question 8.
Find the area of the circle. Use 3.14 for π.
Envision Math Common Core 7th Grade Answers Topic 8 Solve Problems Involving Geometry 69
Answer:
The given figure is:
Envision Math Common Core 7th Grade Answers Topic 8 Solve Problems Involving Geometry 69
Now,
From the given figure,
We can observe that
The radius of a circle (r) = 9 ft
Now,
We know that,
The arae of a circle (A) = πr²
So,
A = 3.14 × 9²
= 254.34 ft²
Hence, from the above,
We can conclude that teh arae of the circle with the given radius is: 254.34 feet²

Question 9.
Find the area of the circle. Use 3.14 for π.
Envision Math Common Core 7th Grade Answers Topic 8 Solve Problems Involving Geometry 70
Answer:
The given figure is:
Envision Math Common Core 7th Grade Answers Topic 8 Solve Problems Involving Geometry 70
Now,
From the given figure,
We can observe that
The diameter of a circle is: 106 yd
Now,
We know that,
The radius of a circle (r) = \(\frac{The diameter of a circle}{2}\)
r = \(\frac{106}{2}\)
r = 53 yd
Now,
We know that,
The area of a circle (A) = πr²
So,
A = 3.14 × 53²
= 8,820.26 yd²
Hence, from the above,
We can conclude that the area of the circle with the given diameter is: 8,820.26 yd²

Question 10.
Jaylon created this stained-glass window. The upper two corners are quarter circles, each with a radius of 4 inches. Find the area of the window. Use 3.14 for π.
Envision Math Common Core Grade 7 Answer Key Topic 8 Solve Problems Involving Geometry 71
Answer:
It is given that
Jaylon created this stained-glass window. The upper two corners are quarter circles, each with a radius of 4 inches.
Now,
The given figure is:
Envision Math Common Core Grade 7 Answer Key Topic 8 Solve Problems Involving Geometry 71
Now,
From the given figure,
We can observe that
The stained glass window is made up of 2 quarter circles, 1 rectangle, and 1 square
Now,
We know that,
The area of a rectangle = Length × Width
The arae of a quarter circle = \(\frac{1}{4}\)πr²
The area of a square = Side × Side
Now,
The area of 2 quarter circles (A) = 2 × \(\frac{1}{4}\) × 3.14 × 4²
= 25.12 in.²
The area of the recatngle (A) = 12 × (26 – 4)
= 264 in.²
The area of the square that lies between the two quareter circles (A) = (12 – 8) × 4
= 16 in.²
So,
The area of the stained glass window = 25.12 + 264 + 16
= 305.12 in.²
Hence, from the above,
We can conclude that the area of the stained glass window is: 305.12 in.²

Question 11.
The circumference of a circle is 50.24 meters. What is the area of the circle? Use 3.14 for π.
Answer:
It is given that
The circumference of a circle is 50.24 meters
Now,
We know that,
The circumference of a circle (C) = 2πr
So,
50.24 = 2 × 3.14 × r
r = \(\frac{50.24}{6.28}\)
r = 8 meters
Now,
We know that,
The area of a circle (A) = πr²
So,
A = 3.14 × 8²
= 200.96 meters²
Hence, from the above,
We can conclude that the area of the circle for the given circumference is: 200.96 meters²

Question 12.
Higher-Order Thinking A circular flower bed is 20 meters in diameter and has a circular sidewalk around it that is 3 meters wide. Find the area of the sidewalk in square meters. Use 3.14 for π. Round to the nearest whole number.
Answer:
It is given that
A circular flower bed is 20 meters in diameter and has a circular sidewalk around it that is 3 meters wide.
Now,
The area of the sidewalk is given as:
A = The area of a flower bed – The area of a sidewalk including the flower bed
Now,
We know that,
Radius (r) = \(\frac{Diameter}{2}\)
r = \(\frac{20}{2}\)
r = 10 meters
Now,
The area of a flower bed (A) = πr²
So,
A = 3.14 × 10²
= 314 meters²
Now,
The area of a sidewalk including the flower bed (A) = 3.14 × (10 + 3)²
= 530.66 meters²
So,
The area of the sidewalk = 530.66 – 314
= 216.66 meters²
Hence, from the above,
We can conclude that the area of the sidewalk is: 216.66 meters²

Question 13.
A circular plate has a circumference of 16.3 inches. What is the area of this plate? Use 3.14 for π. Round to the nearest whole number.
Answer:
It is given that
A circular plate has a circumference of 16.3 inches
Now,
We know that,
The circumference of a circle (C) = 2πr
So,
16.3 = 2 × 3.14 × r
r = \(\frac{16.3}{6.28}\)
r = 2.59 inches
Now,
We know that,
The area of the circular plate (A) = πr²
So,
A = 3.14 × (2.59)²
= 21.06
≈21 inches²
Hence, from the above,
We can conclude that the area of the circular plate for the given circumference is: 21 inches²

Question 14.
A water sprinkler sends water out in a circular pattern. How many feet away from the sprinkler can it spread water if the area formed by the watering pattern is 379.94 square feet?
Answer:
It is given that
A water sprinkler sends water out in a circular pattern and the area formed by the watering pattern is 379.94 square feet
Now,
We have to find the diameter to find how many feet away from the sprinkler can spread the water
Now,
We know that,
The area of a circle (A) = πr²
So,
379.94 = 3.14 × r²
r² = \(\frac{379.94}{3.14}\)
r² = 121
r = 11 feet
So,
Diameter (d) = 2 × Radius
d = 2 × 11
d = 22 feet
Hence, from the above,
We can conclude that the sprinkler can spread the water 22 feet away

Question 15.
The circumference of a circular rug is 24.8 meters. What is the area of the rug? Use 3.14 for π. Round your answer to the nearest tenth.
Envision Math Common Core Grade 7 Answer Key Topic 8 Solve Problems Involving Geometry 72
Answer:
It is given that
The circumference of a circular rug is 24.8 meters
Now,
The given figure is:
Envision Math Common Core Grade 7 Answer Key Topic 8 Solve Problems Involving Geometry 72
Now,
We know that,
The circumference of a circle (C) = 2πr
So,
24.8 = 2 × 3.14 × r
r = \(\frac{24.8}{6.28}\)
r = 3.94 m
Now,
The area of the circular rug (A) = πr²
So,
A = 3.14 × (3.94)²
= 48.7 m²
Hence, from the above,
We can conclude that the area of the circular rug for the given circumference to the nearest tenth is: 48.7 m²

Question 16.
Frank wants to find the area enclosed by the figure at the right. The figure has semicircles on each side of a 40-meter-by-40-meter square. Find the area enclosed by the figure. Use 3.14 for π.
Envision Math Common Core Grade 7 Answer Key Topic 8 Solve Problems Involving Geometry 73
Answer:
It is given that
Frank wants to find the area enclosed by the figure at the right. The figure has semicircles on each side of a 40-meter-by-40-meter square.
Now,
The given figure is:
Envision Math Common Core Grade 7 Answer Key Topic 8 Solve Problems Involving Geometry 73
Now,
The area enclosed by the given figure = The area of the square + The area of the 4 semicircles
Now,
From the given figure,
We can observe that
The side of the square = 40 m
The diameter of the semi circle = 40 m
Now,
We know that,
The area of a square (A) = Side²
So,
A = 40²
= 1,600 m²
Now,
We know that,
The area of a semicircle (A) = \(\frac{1}{2}\)πr²
So,
A = \(\frac{1}{2}\) × 3.14 × \(\frac{1,600}{4}\)
= 628 m²
So,
The area enclosed bythe given figure (A) = 1,600 + 628
= 2,228 m²
Hence, from the above,
We can conclude that the area enclosed by the given figure is: 2,228 m²

Assessment Practice
Question 17.
Julia’s bedroom is 10 feet by 10 feet. She wants to place a circular rug in the corner of her room.
PART A
She places a rug with a radius of 2 feet in her room. How much of her bedroom floor, in square feet, is not covered by the rug? Use 3.14 for π. Round to the nearest tenth.
Answer:
It is given that
Julia’s bedroom is 10 feet by 10 feet. She wants to place a circular rug in the corner of her room.
So,
The diameter of the circular rug placed by Julia is: 10 feet
Now,
We know that,
Radius (r) = \(\frac{Diameter}{2}\)
r = \(\frac{10}{2}\)
r = 5 feet
So,
The space of bedroom floor that is not covered by the rug (C) = The space of the bedroom floor that is covered by the rug when the radius is 5 meters – The space of the bedroom floor that is covered by the rug when the radius is 2 meters
So,
C = 2π (5 – 2)
= 2 × 3.14 × 3
= 18.84 feet
Hence, from the above,
We can conclude that the space of bedroom floor that is not covered by the rug to the neares tenth is: 18.8 feet

PART B
Julia decides she wants a rug that covers about 50% of her floor. Which rug should she buy?
A. A rug with a radius of 5 feet
B. A rug with a diameter of 5 feet
C. A rug with a radius of 4 feet
D. A rug with a diameter of 4 feet
Answer:
It is given that Julia’s bedroom is 10 feet by 10 feet
So,
The diameter of the circular rug is: 10 feet
Now,
The rug covers the whole bedroom floor when the diameter is 10 feet
So,
When Julia decides to cover the bedroom floor only 50% with the rug,
Then, we have to find only the radius of the circular rug i.e., \(\frac{10}{2}\)
Hence, from the above,
We can conclude that option A matches the given situation

Question 18.
The circumference of a hubcap of a tire is 81.58 centimeters. Find the area, in square centimeters, of this hubcap. Use 3.14 as an approximation for π. Round your answer to the nearest whole centimeter.
Answer:
It is given that
The circumference of a hubcap of a tire is 81.58 centimeters.
Now,
We know that,
The circumference of a circle (C) = 2πr
So,
81.58 = 2 × 3.14 × r
r = \(\frac{81.58}{6.28}\)
r = 12.99 cm
Now,
The area of a circle (A) = πr²
So,
A = 3.14 × (12.99)²
= 529.8 cm²
≈ 530 cm²
Hence, from the above,
We can conclude that the area of the circular hubcap is about 530 cm²

3-Act Mathematical Modeling: Whole Lotta Dough

АСТ 1
Question 1.
After watching the video, what is the first question that comes to mind?
Answer:

Question 2.
Write the Main Question you will answer.
Answer:

Question 3.
Construct Arguments Predict an answer to this Main Question. Explain your prediction.
Envision Math Common Core Grade 7 Answer Key Topic 8 Solve Problems Involving Geometry 74
Answer:

Question 4.
On the number line below, write a number that is too small to be the answer. Write a number that is too large.
Envision Math Common Core Grade 7 Answer Key Topic 8 Solve Problems Involving Geometry 75
Answer:

Question 5.
Plot your prediction on the same number line.
Answer:

ACT 2
Question 6.
What information in this situation would be helpful to know? How would you use that information?
Envision Math Common Core Grade 7 Answer Key Topic 8 Solve Problems Involving Geometry 76
Answer:

Question 7.
Use Appropriate Tools What tools can you use to solve the problem? Explain how you would use them strategically.
Answer:

Question 8.
Model with Math Represent the situation using mathematics. Use your representation to answer the Main Question.
Answer:

Question 9.
What is your answer to the Main Question? Is it higher or lower than your prediction? Explain why.
Envision Math Common Core Grade 7 Answer Key Topic 8 Solve Problems Involving Geometry 77
Answer:

ACT 3
Question 10.
Write the answer you saw in the video.
Envision Math Common Core Grade 7 Answer Key Topic 8 Solve Problems Involving Geometry 78
Answer:

Question 11.
Reasoning Does your answer match the answer in the video? If not, what are some reasons that would explain the difference?
Answer:

Question 12.
Make Sense and Persevere Would you change your model now that you know the answer? Explain.
Envision Math Common Core Grade 7 Answer Key Topic 8 Solve Problems Involving Geometry 79
Answer:

ACT 3 Extension
Question 13.
Model with Math Explain how you used a mathematical model to represent the situation. How did the model help you answer the Main Question?
Answer:

Question 14.
Reasoning Explain why your answer to the Main Question does not involve the symbol π.
Answer:

SEQUEL
Question 15.
Use Structure If the regular pizza costs $8.99, how much do you think the big pizza costs?
Envision Math Common Core Grade 7 Answer Key Topic 8 Solve Problems Involving Geometry 80
Answer:

Lesson 8.7 Describe Cross Sections

Solve & Discuss It!

How could Mrs. Mendoza divide the ream of paper equally between two art classes? She has a paper cutter to slice the paper if needed. What will the dimensions for each sheet of paper be once she has divided the ream? How many sheets will each class receive?
Envision Math Common Core Grade 7 Answer Key Topic 8 Solve Problems Involving Geometry 81

I can… determine what the cross-section looks like when a 3D figure is sliced.
Answer:
It is given that
Mrs. Mendoza has a paper cutter to slice the paper.
Now,
The given figure is:
Envision Math Common Core Grade 7 Answer Key Topic 8 Solve Problems Involving Geometry 81
Now,
From the given figure,
We can observe that,
A ream consists of 500 sheets
We can also observe that
The dimensions of the ream are:
Length of the ream: 8\(\frac{1}{2}\) inches
Width of the ream: 11 inches
Now,
From the given figure,
We can observe that
The shape of the ream is: Rectangle
Now,
We know that,
Even we cut a ream of paper, each sheet of paper has the same dimensions as a ream of paper
So,
The dimensions of each sheet of paper after Mrs. Mendoza divided the ream into 2 equal parts are:
Length: 8\(\frac{1}{2}\) inches
Width: 11 inches
Now,
The number of sheets of paper each class will get after dividing the ream = \(\frac{500}{2}\)
= 250 sheets
Hence, from the above,
We can conclude that
The number of sheets each class will receive is: 250
The dimensions of each sheet of paper once Mrs. Mendoza divided the ream is:
Length: 8\(\frac{1}{2}\) inches
Width: 11 inches

Focus on math practices
Use Structure How would the number of sheets of paper each class receives change if Mrs. Mendoza started with 300 sheets?
Answer:
It is given that
Mrs. Mendoza has 500 sheets of paper and she divided the sheets equally between the two classes
Now,
If Mrs. Mendoza started with 300 sheets of paper, then
The number of sheets divided by Mrs. Mendoza equally between the two classes = \(\frac{300}{2}\)
= 150
Hence, from the above,
We can conclude that the number of sheets each class receives when Mrs. Mendoza started with 300 sheets is: 150

Essential Question
How do the faces of a three-dimensional figure determine the two-dimensional shapes created by slicing the figure?
Answer:
A cross-section is a new face you see when you slice through a three-dimensional figure.
Example:
If you slice a rectangular pyramid parallel to the base, you get a smaller rectangle as the cross-section.
If you slice a rectangular pyramid parallel to the apex, you get a smaller triangle as the cross-section.

Try It!
Zachary made a vertical slice that was parallel to the left and right faces of a bread roll. What shape is the cross-section, and what are its dimensions?
Envision Math Common Core Grade 7 Answer Key Topic 8 Solve Problems Involving Geometry 82
The shape of the cross section is a __________ that is __________ inches by ________ inches.
Answer:
It is given that
Zachary made a vertical slice that was parallel to the left and right faces of a bread roll
Now,
The given figure is:
Envision Math Common Core Grade 7 Answer Key Topic 8 Solve Problems Involving Geometry 82
Now,
From the given figure,
We can observe that
When Zachary made a vertical slice that was parallel to the left and right faces of a bread roll,
The shape of the cross-section Zachary can get is: Rectangle
Now,
The dimensions of the rectangle that we obtained by slicing is:
Length: 3 in.
Width: 2 in.
Hence, from the above,
We can conclude that
The shape of the cross-section is a “Rectangle” that is 3 inches × 2 inches.

Convince Me! What are the shapes of horizontal and vertical cross-sections of a rectangular prism, and how can you determine the dimensions of the cross-sections?
Answer:
The horizontal cross-section of a rectangular prism is: Rectangle
The vertical cross-section of a rectangular prism is: Rectangle
Now,
The dimensions of the cross-sections are:
a. One dimension of the cross-section will be the height of the prism.
b. Another dimension of the cross-section will be the length of the rectangle.

Try It!
Draw the cross-section that is created when a vertical plane intersects the top vertex and the shorter edge of the base of the pyramid shown. What is the area of the cross section?
Envision Math Common Core Grade 7 Answer Key Topic 8 Solve Problems Involving Geometry 83
Answer:
The given figure is:
Envision Math Common Core Grade 7 Answer Key Topic 8 Solve Problems Involving Geometry 83
Now,
From the given figure,
We can observe that
The cross-section that is created when a vertical plane intersects the top vertex and the shorter edge of the base of the pyramid is: Triangle
Now,
We know that,
The area of a triangle (A) = \(\frac{1}{2}\) × Base × Height
So,
A = \(\frac{1}{2}\) × 7 × 4
= 14 in.²
Hence, from the above,
We can conclude that the area of the cross-section is: 14 in.²

KEY CONCEPT

A cross section is the two-dimensional shape exposed when a three-dimensional figure is sliced. The shape and dimensions of a cross section in a rectangular prism are the same as the faces that are parallel to the slice.
Envision Math Common Core Grade 7 Answer Key Topic 8 Solve Problems Involving Geometry 84

Do You Understand?

Question 1.
Essential Question How do the faces of a three-dimensional figure determine the two-dimensional shapes created by slicing the figure?
Answer:
A cross-section is a new face you see when you slice through a three-dimensional figure.
Example:
If you slice a rectangular pyramid parallel to the base, you get a smaller rectangle as the cross-section.
If you slice a rectangular pyramid parallel to the apex, you get a smaller triangle as the cross-section.

Question 2.
Generalize What are the shapes of the cross-sections that are parallel or perpendicular to the bases of a right rectangular prism?
Answer:
The shape of the cross-section that is parallel to the base of a right rectangular prism is: Rectangle
The shape of the cross-section that is perpendicular to the base of a right rectangular prism is: Triangle

Question 3.
Generalize What are the shapes of the horizontal cross-sections of a right rectangular pyramid? What are the shapes of vertical cross-sections through the vertex opposite the base?
Answer:
The shape of the horizontal cross-section of a right rectangular pyramid is: Rectangle
The shape of the vertical cross-section through the vertex that is opposite to the base is: Triangle

Do You Know How?

Question 4.
The divider in a desk drawer is a cross-section that is parallel to the front of the drawer. What is its shape, and what are its dimensions?
Envision Math Common Core Grade 7 Answer Key Topic 8 Solve Problems Involving Geometry 85
Answer:
It is given that
The divider in a desk drawer is a cross-section that is parallel to the front of the drawer
Now,
The given figure is:
Envision Math Common Core Grade 7 Answer Key Topic 8 Solve Problems Involving Geometry 85
Now,
From the given figure,
We can observe that
The shape of the cross-section of the divider that is parallel in front of the door is: Rectangle
Now,
The dimensions of the cross-section of the divider are:
Length: 8 cm
Width: 5 cm
Hence, from the above,
We can conclude that
The shape of the cross-section of the divider is: Rectangle
The dimensions of the cross-section of the divider is:
Length: 8 cm
Width: 5 cm

Question 5.
Use the diagram to answer the questions.
Envision Math Common Core Grade 7 Answer Key Topic 8 Solve Problems Involving Geometry 86
a. Draw the cross-section that is formed when the pyramid is sliced vertically through its vertex and its right face, perpendicular to its base.
Answer:
The given figure is:
Envision Math Common Core Grade 7 Answer Key Topic 8 Solve Problems Involving Geometry 86
Hence,
The cross-section that is formed when the pyramid is sliced vertically through its vertex and its right face that is perpendicular to its base is:

b. What is the area of this cross section?
Answer:
From part (a),
We can observe hat
The cross-section that is formed when the pyramid is sliced vertically through its vertex and its right face that is perpendicular to its base is:

Now,
We know that,
The area of a triangle (A) = \(\frac{1}{2}\) × Base × Height
So,
The area of the given cross-section (A) = \(\frac{1}{2}\) × 9 × 10
= 45 mm²
Hence, from the above,
We can conclude that the area of the given cross-section is: 45 mm²

Question 6.
What are the dimensions of the vertical cross section shown on this right rectangular prism?
Envision Math Common Core Grade 7 Answer Key Topic 8 Solve Problems Involving Geometry 87
Answer:
The given figure is:
Envision Math Common Core Grade 7 Answer Key Topic 8 Solve Problems Involving Geometry 87
Now,
From the given figure,
We can observe that,
The vertical cross-section of the given right rectangular prism is: Rectangle
Now,
The dimensions of the vertical cross-section of the given right rectangular prism is:
Length: 5 in.
Width: 4 in.
Hence, from the above,
We can conclude that
The dimensions of the vertical cross-section shown on the given right rectangular prism is:
Length: 5 in.
Width: 4 in.

Question 7.
Be Precise Describe the cross section that is formed by a vertical plane, perpendicular to the base of the pyramid, that intersects the 9-in. edge and the top vertex of the pyramid shown.
Envision Math Common Core Grade 7 Answer Key Topic 8 Solve Problems Involving Geometry 88
Answer:
The given figure is:
Envision Math Common Core Grade 7 Answer Key Topic 8 Solve Problems Involving Geometry 88
Hence,
The cross-section that is formed by a vertical plane, perpendicular to the base of the pyramid, that intersects the 9-in. edge and the top vertex of the given pyramid is:

Question 8.
Mason is slicing butter for the meal he is preparing. Describe the vertical cross section when the knife slices through the butter, parallel to its sides.
Envision Math Common Core Grade 7 Answer Key Topic 8 Solve Problems Involving Geometry 89
Answer:
It is given that
Mason is slicing butter for the meal he is preparing
Now,
The given figure is:
Envision Math Common Core Grade 7 Answer Key Topic 8 Solve Problems Involving Geometry 89
Hence,
The vertical cross-section when the knife slices through the butter, parallel to its sides is:

Question 9.
a. Look for Relationships What are the dimensions of the vertical cross section?
Envision Math Common Core Grade 7 Answer Key Topic 8 Solve Problems Involving Geometry 90
Answer:
The given figure is:
Envision Math Common Core Grade 7 Answer Key Topic 8 Solve Problems Involving Geometry 90
Hence,
The dimensions of the vertical cross-section of the given figure is:

b. What would be the dimensions of a horizontal cross section?
Answer:
The given figure is:
Envision Math Common Core Grade 7 Answer Key Topic 8 Solve Problems Involving Geometry 90
Hence,
The horizontal cross-section of the given figure is:

Question 10.
Use the figure to the right.
a. Describe the cross section shown.
Envision Math Common Core Grade 7 Answer Key Topic 8 Solve Problems Involving Geometry 91
Answer:
The given figure is:
Envision Math Common Core Grade 7 Answer Key Topic 8 Solve Problems Involving Geometry 91
Now,
From the given figure,
We can observe that
The horizontal cross-section of the given figure is: Rectangle
The vertical cross-section of the given figure is: Rectangle
Hence,
The dimensions of the horizontal cross-section of the given figure is:
Length: 4 ft
Width: 6 ft
The dimensions of the vertical cross-section of the given figure is:
Length: 11 ft
Width: 6 ft

b. Is it possible to have a horizontal cross section with different dimensions if you had the plane intersect the prism at another height? Explain.
Answer:
Yes, it is possible to have a horizontal cross section with different dimensions if you had the plane intersect the prism at another height
Now,
The given figure is:
Envision Math Common Core Grade 7 Answer Key Topic 8 Solve Problems Involving Geometry 91
Now,
From the given figure,
We can observe that
If we make a horizontal cross-section when another plane intersects the prism at another height, then
The dimensions of the new horizontal cross-section is:
Length: 11 ft
Width: 6 ft

Question 11.
Make Sense and Persevere The base of a right rectangular pyramid has a length of 12 centimeters, a width of 6 centimeters, and a height of 14 centimeters. Describe the cross-section formed by a horizontal plane that intersects the faces of the pyramid above the base.
Answer:
It is given that
The base of a right rectangular pyramid has a length of 12 centimeters, a width of 6 centimeters, and a height of 14 centimeters
Now,
We know that,
The horizontal cross-section of a right rectangular pyramid is: Rectangle
The vertical cross-section of a right rectangular pyramid is: Triangle
So,
The dimensions of the horizontal cross-section of the given right rectangular pyramid is:
Length: 12 cm
Width: 6 cm
Hence,
The representation of the horizontal cross-section of the given right rectangular pyramid is:

Question 12.
Higher Order Thinking Luis makes blocks from a painted piece of wood with dimensions of 27 inches × 24 inches × 1.5 inches. To make 72 blocks, the wood is cut into 3-inch squares. Draw two pictures showing the horizontal cross section and the vertical cross section of each block.
Answer:
It is given that
Luis makes blocks from a painted piece of wood with dimensions of 27 inches × 24 inches × 1.5 inches. To make 72 blocks, the wood is cut into 3-inch squares.
So,
The dimensions of each block is:
Length: \(\frac{27}{3}\) = 9 inches
Width: \(\frac{24}{3}\) = 8 inches
Height: \(\frac{1.5}{3}\) = 0.5 inches
Now,
It is given that the 72 blocks are cut into 3-inch squares
So,
The figure made by the given dimensions is: Cuboid
Now,
The vertical cross-section of the given cuboid is: Rectangle
The horizontal cross-section of the given cuboid is: Rectangle
Hence,
The representation of the vertical and horizontal cross-sections of the cuboid are:

Question 13.
Make Sense and Persevere The area of the cross section shown is 52 square yards. What is the length of the unknown side of the base of the pyramid?
Envision Math Common Core Grade 7 Answer Key Topic 8 Solve Problems Involving Geometry 92
Answer:
It is given that
The area of the cross-section shown is 52 square yards.
Now,
The given figure is:
Envision Math Common Core Grade 7 Answer Key Topic 8 Solve Problems Involving Geometry 92
Now,
From the above,
We can observe that the given figure is: Right Rectangular pyramid
Now,
We know that,
The vertical cross-section of the right rectangular pyramid is: Triangle
Now,
We know that,
The area of a triangle (A) = \(\frac{1}{2}\) × Base × Height
So,
52 = \(\frac{1}{2}\) × x × 13
x = \(\frac{52 × 2}{13}\)
x = 8 yd
Hence, from the above,
We can conclude that the length of the unknown side of the base of the given pyramid is: 8 yd

Question 14.
A waiter slices a cake shaped like a square pyramid vertically through the top point.
a. Make Sense and Persevere Draw the cross section that is made by slicing the cake in this way.
Envision Math Common Core Grade 7 Answer Key Topic 8 Solve Problems Involving Geometry 93
Answer:
It is given that
A waiter slices a cake shaped like a square pyramid vertically through the top point.
Now,
The given figure is:
Envision Math Common Core Grade 7 Answer Key Topic 8 Solve Problems Involving Geometry 93
Now,
From the given figure,
We can observe that
If we made the vertical cross-section of the given square pyramid, then
We will get the vertical cross-section as a Right triangle
Hence,
The representation of the vertical cross-section of the given square pyramid is:

b. What is the area of this cross section?
Answer:
From part (a),
We can observe that
The representation of the vertical cross-section of the given square pyramid is:

Now,
We know that,
In a right triangle,
(Hypotenuse)² = (Base)² + (Side)²
So,
In the given triangle,
18² = 14² + h²
h² = 18² – 14²
h = 11.3 inches
Now,
We know that,
The area of a triangle (A) = \(\frac{1}{2}\) × Base × Height
So,
A = \(\frac{1}{2}\) × 14 × 11.3
= 79.19 yd²
Hence, from the above,
We can conclude that the area of the given cross-section is: 79.19 yd²

Question 15.
Miranda says that the triangle below represents the cross section of the rectangular pyramid shown.
Envision Math Common Core Grade 7 Answer Key Topic 8 Solve Problems Involving Geometry 94
What mistake might Miranda have made?
Answer:
It is given that
Miranda says that the triangle below represents the cross-section of the rectangular pyramid shown.
Now,
The given figure is:
Envision Math Common Core Grade 7 Answer Key Topic 8 Solve Problems Involving Geometry 94
Now,
We know that,
In a rectangular pyramid,
The vertical cross-section is: Triangle
The horizontal cross-section is: Rectangle
So,
According to the given information,
Miranda did not mention what type of cross-section she had drawn
Hence, from the above,
We can conclude that the mistake made by Miranda is:
She did not mention what type of cross-section she made

Assessment Practice
Question 16.
Estimate, to the nearest whole number, the number of vertical cross sections needed to equal the area of the base of the figure to the right. Explain how you made your estimate, and decide whether your estimate is higher or lower than the actual number.
Envision Math Common Core Grade 7 Answer Key Topic 8 Solve Problems Involving Geometry 95
Answer:
The given figure is:
Envision Math Common Core Grade 7 Answer Key Topic 8 Solve Problems Involving Geometry 95

Now,
From the given figure,
We can observe that the given 3-d shape is: Cuboid
Now,
We know that,
The vertical cross-section of a cuboid is: Rectangle
The horizontal cross-section of a cuboid is: Rectangle
Now,
From the given figure,
The dimensions of a horizontal cross-section of the given figure is:
Length: 8 cm
Width: 6 cm
The dimensions of a vertical cross-section of the given figure is:
Length: 6 cm
Width: 3 cm
Now,
We know that,
The area of a rectangle = Length × Width
So,
The area of a horizontal cross-section of the given figure is:
A = 8 × 6 = 48 cm²
The area of a vertical cross-section of the given figure is:
A = 6 × 3 = 18 cm²
Hence, from the above,
We can conclude that
The number of vertical cross-sections < The area of the base of the given figure

Lesson 8.8 Solve Problems Involving Surface Area

Solve & Discuss It!

Alaya will paint the outside of a box in three different colors. Decide how she could paint the box. What is the total area that each color will cover?
Envision Math Common Core Grade 7 Answer Key Topic 8 Solve Problems Involving Geometry 96
I can… find the area and surface area of 2-dimensional composite shapes and 3-dimensional prisms.
Answer:
It is given that
Alaya will paint the outside of a box in three different colors
Now,
The given figure is:
Envision Math Common Core Grade 7 Answer Key Topic 8 Solve Problems Involving Geometry 96
Now,
From the given figure,
We can observe that the given figure is a “Cuboid” which has 6 faces
Now,
The different ways that Alaya can paint are:
a. She can paint all the faces of the box with the same color
b. She can paint the faces of the box with alternative colors i.e., 1 face is colored with 1 color, 2nd face is colored with 1 color, etc.
Now,
We know that,
The total surface area of a cuboid (A) = 2 (lw + wh + lh)
Where,
‘l’ is the length of the cuboid
‘w’ is the width of the cuboid
‘h’ is the height of the cuboid
Now,
From the given figure,
The length of the cuboid (l) is 32 in.
The width of the cuboid (w) is 16 in.
The height of the cuboid (h) is 14 in.
So,
A = 2 (32 × 16 + 16 × 14 + 32 × 14)
= 2 (512 + 224 + 448)
= 2,368 in.²
So,
The total surface area that each color will cover = \(\frac{2,368}{6}\)
= 394.66 in.²
Hence, from the above,
We can conclude that
The different ways that Alaya can paint are:
a. She can paint all the faces of the box with the same color
b. She can paint the faces of the box with alternative colors i.e., 1 face is colored with 1 color, 2nd face is colored with 1 color, etc.
The total area covered by each color is: 394.66 in.²

Make Sense and Persevere
What do you know about the faces of a rectangular prism?
Answer:
The facts about the rectangular prism are:
a. A rectangular prism is a polyhedron with two congruent and parallel bases.
b. Because of its cross-section along the length, it is said to be a prism.
c. A rectangular prism has 6 faces, 8 vertices, and 12 edges. Its base and top are always rectangles.
d. It has 3 dimensions They are length, width, and height.

Focus on math practices
Reasoning Trista paints each pair of opposite sides of the box with the same color. How many different areas does she need to find to determine the total area covered by each color? Explain.
Answer:
The given figure is:
Envision Math Common Core Grade 7 Answer Key Topic 8 Solve Problems Involving Geometry 96
Now,
It is given that
Trista paints each pair of opposite sides of the box with the same color
So,
Each color covers two opposite pairs of rectangles.
So,
With the help of each color,
We can determine the total area of the two opposite sides of the box
Hence, from the above,
We can conclude that
she needs to find 3 different areas to determine the total area covered by each color

Essential Question
How is finding the area of composite two-dimensional figures similar to finding the surface area of three-dimensional figures?
Answer:
To find the area of something, you start by multiplying the two dimensions together. To find the area of a rectangle, you multiply the length by the width. Although the area is a two-dimensional measurement, it can also be used with three-dimensional objects.

Try It!

This diagram shows the area of a room to be carpeted. What will be the area of the new carpet?
Envision Math Common Core Grade 7 Answer Key Topic 8 Solve Problems Involving Geometry 97
A = ________ = ________ ft2 B = _______ = _______ ft2
Total area = ______ + ______
The area of the new carpet is ________ square feet
Answer:
It is given that
The below diagram shows the area of a room to be carpeted.
Now,
The given figure is:
Envision Math Common Core Grade 7 Answer Key Topic 8 Solve Problems Involving Geometry 97
Now,
From the given figure,
We can observe that
The total area of a room that is carpeted = The area of a rectangle + The area of a triangle
Now,
From the given figure,
The dimensions of the rectangle are:
Length: 6 feet
Width: 5 feet
The dimensions of the triangle are:
Height: 6 feet
Base: 4 feet
Now,
We know that,
The area of a rectangle = Length × Width
= 6 × 5
= 30 feet²
Now,
We know that,
The area of a triangle = \(\frac{1}{2}\) × Base × height
= \(\frac{1}{2}\) × 6 × 4
= 12 feet²
So,
The area of the new carpet = 30 + 12
= 42 feet²
Hence, from the above,
We can conclude that the area of the new carpet is: 42 feet²

Convince Me! How does knowing the area of familiar shapes help find the total area of a composite shape?
Answer:
A composite figure is made up of simple geometric shapes. To find the area of a composite figure or other irregular-shaped figures, divide it into simple, non-overlapping figures. Find the area of each simpler figure, and then add the areas together to find the total area of the composite figure

Try It!

Hiromi is painting the front and back of a barn. Each can of paint covers 32 square feet. How many cans of paint does Hiromi need to cover the entire front and back of the barn?
Envision Math Common Core Grade 7 Answer Key Topic 8 Solve Problems Involving Geometry 98
Answer:
It is given that
Hiromi is painting the front and back of a barn. Each can of paint covers 32 square feet.
Now,
The given figure is:
Envision Math Common Core Grade 7 Answer Key Topic 8 Solve Problems Involving Geometry 98
Now,
From the given figure,
We can observe that
The total area of the barn (A) = The area of the front + The area of the back
= 2 (The area of the front) (or) 2 (The area of the back)
Now,
The front side of the barn is made up of a rectangle and a triangle
Now,
The area of the front side of the barn = The area of the rectangle + The area of the triangle
= Length × Width + \(\frac{1}{2}\) × Base × Height
= 11 × 12 + \(\frac{1}{2}\) × 12 × (15 – 11)
= 132 + 24
= 156 ft²
So,
The total area of the barn (A) = 2 × 156
= 312 ft²
Now,
The number of cans of paint does Hiromi need to cover the entire front and back of the barn = \(\frac{The total area of the barn}{The number of square feet each can covers}\)
= \(\frac{312}{32}\)
= 9.75
≅ 10 cans of paint
Hence, from the above,
We can conclude that Hiromi needs to cover 10 cans of paint to cover the entire front and back of the barn

KEY CONCEPT

The area of a two-dimensional composite figure is the sum of the areas of all the shapes that compose it. The surface area of a three-dimensional composite figure is the sum of the areas of all its faces.
Two-dimensional composite figure
Envision Math Common Core Grade 7 Answer Key Topic 8 Solve Problems Involving Geometry 99

Three-dimensional composite figure
Envision Math Common Core Grade 7 Answer Key Topic 8 Solve Problems Involving Geometry 100
Surface area of shape A + Surface area of shape B = Surface area of composite shape

Do You Understand?

Question 1.
Essential Question How is finding the area of composite two-dimensional figures similar to finding the surface area of three-dimensional figures?
Answer:
To find the area of something, you start by multiplying the two dimensions together. To find the area of a rectangle, you multiply the length by the width. Although the area is a two-dimensional measurement, it can also be used with three-dimensional objects.

Question 2.
Make Sense and Persevere Laine wants to determine the amount of fabric needed to cover a triangular prism-shaped box. She begins by measuring the dimensions of the box. Explain her next steps.
Answer:
It is given that
Laine wants to determine the amount of fabric needed to cover a triangular prism-shaped box. She begins by measuring the dimensions of the box
Now,
The next steps she has to follow are:
Step 1:
Divide the given triangular prism into the number of parts that have many faces as the given triangular prism has.
We know that,
The triangular prism has: 5 faces
So,
The number of smaller triangles are: 5
Step 2:
Find the area of each triangle
We know that,
The area of a triangle (A) = \(\frac{1}{2}\) × Base × Height
Step 3:
Add the areas of the 5 triangles to find the area of the given triangular prism
Hence, from the above,
We can conclude that the amount of fabric needed to cover a triangular prism is its total area

Question 3.
Use Structure Explain how you would find the surface area of the figure below.
Envision Math Common Core Grade 7 Answer Key Topic 8 Solve Problems Involving Geometry 101
Answer:
The given figure is:
Envision Math Common Core Grade 7 Answer Key Topic 8 Solve Problems Involving Geometry 101
Now,
From the given figure,
We can observe that the given figure is made up of:
a. Rectangle b. Pentagon
So,
The total surface area of the given figure = Total area of the rectangle + Total area of the pentagon
Now,
From the given figure,
We can observe that
There are:
a. 5 Rectangles b. 2 Pentagons
Hence,
The total surface area of the given figure (A) = The area of 5 Rectangles + The area of 2 Pentagons

Do You Know How?

Question 4.
Paula is painting a henhouse. If a can of paint will cover 24 square feet, how many cans of paint does she need to buy? Explain the steps she might take to solve this problem.
Answer:
It is given that Paula is painting a henhouse
Now,
The representation of a henhouse is:

Now,
From the given figure,
We can observe that
The henhouse is made up of:
a. 5 Rectangles b. 4 Triangles
Now,
The steps that Paula might take to find the number of cans she needed to buy are:
Step 1:
Divide the henhouse into simple 2-d figures i.e, into rectangles and triangles
Step 2:
Find the areas of the 2-d figures
Step 3:
Find the total area of the henhouse by adding all the areas of 2-d figures
Step 4:
To find the number of cans Paula needed to buy,
The number of cans Paula needed to buy = \(\frac{The total area of the henhouse}{The area covered by 1 can of paint}\)
Hence, from the above,
We can conclude that
By following the above steps, Paula can solve her problem

Question 5.
Find the area of the composite figure. The two triangles have the same dimensions.
Envision Math Common Core Grade 7 Answer Key Topic 8 Solve Problems Involving Geometry 102
Answer:
The given figure is:
Envision Math Common Core Grade 7 Answer Key Topic 8 Solve Problems Involving Geometry 102
Now,
From the above figure,
We can observe that
The area of the composite figure (A) = The area of the 2 rectangles + The area of the 2 triangles
Now,
We know that,
The area of a rectangle (A) = Length × Width
The area of a triangle (A) = \(\frac{1}{2}\) × Base × Height
So,
A = (7.5 × 13) + (7.5 × 6) + 2 × \(\frac{1}{2}\) × 2.5 × 6
= 97.5 + 45 + 15
= 157.5 cm²
Hence, from the above,
We can conclude that the area of the composite figure is: 157.5 cm²

Question 6.
A stage block is being covered in carpet. The dimensions of the block are 2 feet by 3 feet by 6 feet. Every surface will need covering except for the surface touching the floor, which is 3 feet by 6 feet. How would you calculate the surface area that needs covering?
Answer:
It is given that
A stage block is being covered in carpet. The dimensions of the block are 2 feet by 3 feet by 6 feet. Every surface will need covering except for the surface touching the floor, which is 3 feet by 6 feet.
Now,
The surface area that needs covering = The total surface area – The surface area of the floor
Now,
From the given information,
We can say that the carpet is in the form of a rectangular prism
Now,
We know that,
The surface area of a rectangular prism (A) = 2 (lw + wh + lh)
Where,
‘l’ is the length
‘w’ is the width
‘h’ is the height
So,
The total surface area (A) = 2 (2 × 3 + 3 × 6 + 2 × 6)
= 2 (6 + 18 + 12)
= 72 feet²
Now,
From the given information,
We can observe that the floor is in the form of a rectangle
So,
The surface area of the floor (A) = 3 × 6
= 18 ft²
So,
The surface area that needs covering = The total surface area – The surface area of the floor
= 72 – 18
= 54 feet²
Hence, from the above,
We can conclude that the surface area that needs covering is: 54 feet²

Practice & Problem Solving

Leveled Practice In 7, fill in the boxes to solve.

Question 7.
Jacob is putting tiles on the sections of his yard labeled A, B, and C. What is the area of the parts that need tiles?
Envision Math Common Core Grade 7 Answer Key Topic 8 Solve Problems Involving Geometry 103
Answer:
It is given that
Jacob is putting tiles on the sections of his yard labeled A, B, and C.
Now,
From the given figure,
We can observe that the tiles are made up of parts named as:
A: Rectangle B: Rectangle C: Triangle
Now,
We know that,
The area of a rectangle (A) = Length × Width
The area of  triangle A) = \(\frac{1}{2}\) × Base × Height
So,
The area of Part A:
A = 6 × 3 = 18 m²
The area of Part B:
A = 3 × 4.5 = 13.5 m²
The area of Part C:
A = \(\frac{1}{2}\) × 3 × 3 = 4.5 m²
So,
The total area of the tiles = The area of Part A + The area of Part B + The area of Part C
= 18 + 13.5 + 4.5
= 36 m²
Hence, from the above,
We can conclude that the area of the parts that need tiles is: 36 m²

Question 8.
What is the total area of the figure?
Envision Math Common Core Grade 7 Answer Key Topic 8 Solve Problems Involving Geometry 104
Answer:
The given figure is:
Envision Math Common Core Grade 7 Answer Key Topic 8 Solve Problems Involving Geometry 104
Now,
From the given figure,
We can observe that
The area of the given figure (A) = The area of the rectangles + The area of a triangle
Now,
We know that,
The area of a rectangle (A) = Length × Width
The area of a triangle (A) = \(\frac{1}{2}\) ×Base × Height
So,
The area of the given figure (A) = 4 (4 × 9) + \(\frac{1}{2}\) × 9 × \(\sqrt{9² – 5²}\)
= 144 + 18.70
= 162.70 ft²
Hence, from the above,
We can conclude that the area of the given figure is: 162.70 ft²

Question 9.
Find the surface area of the prism.
Envision Math Common Core Grade 7 Answer Key Topic 8 Solve Problems Involving Geometry 105
Answer:
The given figure is:
Envision Math Common Core Grade 7 Answer Key Topic 8 Solve Problems Involving Geometry 105
Now,
We know that,
The surface area of a rectangular prism = 2 (lw + wh + lh)
Where,
‘l’ is the length
‘w’ is the width
‘h’ is the height
So,
The area of the given prism (A) = 2 (15 × 4 + 4 × 8 + 15 × 8)
= 2 (60 + 32 + 120)
= 424 in.²
Hence, from the above,
We can conclude that the surface area of the given prism is: 424 in.²

Question 10.
Find the surface area of the triangular prism. The base of the prism is an isosceles triangle.
Envision Math Common Core Grade 7 Answer Key Topic 8 Solve Problems Involving Geometry 106
Answer:
The given figure is:
Envision Math Common Core Grade 7 Answer Key Topic 8 Solve Problems Involving Geometry 106
Now,
From the given figure,
We can observe that
For the given Triangular prism,
There are:
a. 2 Triangles b. 3 Paralleleograms
So,
The area of the triangular prism = The total area of the triangles + The total area of the parallelograms
Now,
We know that,
The area of a triangle (A) = \(\frac{1}{2}\) × Base × height
The area of a parallelogram (A) = Base × Height
So,
The total area of the triangular prism (A) = 2 × \(\frac{1}{2}\) × 14 × 24 + 3 × 25 × 48
= 336 + 3,600
= 3,936 cm²
Hence, from the above,
We can conclude that the area of the given triangular prism is: 3,936 cm²

Question 11.
A block of wood has the shape of a triangular prism. The bases are right triangles. Find its surface area.
Envision Math Common Core Grade 7 Answer Key Topic 8 Solve Problems Involving Geometry 107
Answer:
It is given that
A block of wood has the shape of a triangular prism. The bases are right triangles
Now,
The given triangular prism is:
Envision Math Common Core Grade 7 Answer Key Topic 8 Solve Problems Involving Geometry 107
Now,
From the given triangular prism,
We can observe that
The triangular prism has:
a. 2 Right triangles b. 3 Parallelograms
Now,
We know that,
The area of a triangle (A) = \(\frac{1}{2}\) × Base × height
The area of a parallelogram (A) = Base × Height
So,
The total area of the triangular prism (A) = 2 × \(\frac{1}{2}\) × 1.5 × 2.5 + 3 × 2.5 × 15
= 3.75 + 112.5
= 116.25 in.²
Hence, from the above,
We can conclude that the area of the given triangular prism is: 116.25 in.²

Question 12.
A box has the shape of a rectangular prism. How much wrapping paper do you need to cover the box?
Envision Math Common Core Grade 7 Answer Key Topic 8 Solve Problems Involving Geometry 108
Answer:
It is given that A box has the shape of a rectangular prism
Now,
The given rectangular prism is:
Envision Math Common Core Grade 7 Answer Key Topic 8 Solve Problems Involving Geometry 108
Now,
To find the amount of wrapping paper you needed to cover the box,
We have to find the total area of the iven rectangular prism
Now,
We know that,
The surface area of a rectangular prism (A) = Length × Width × Height
So,
A = 16 × 3 × 15
= 720 in.²
Hence, from the above,
We can conclude that
The amount of wrapping paper you needed to cover the box is: 720 in.²

Question 13.
Higher-OrderThinking Find the surface area of the Rectangular Hexagonal prism. Show your work.
Envision Math Common Core Grade 7 Answer Key Topic 8 Solve Problems Involving Geometry 109
Answer:
The given rectangular hexagonal prism is:
Envision Math Common Core Grade 7 Answer Key Topic 8 Solve Problems Involving Geometry 109
Now,
From the given figure,
We can observe that
The total area of the rectangular hexagonal prism (A) = The area of the 2 cicles + The area of the 6 rectangles
Now,
We know that,
The area of a circle (A) = πr²
The area of a rectangle (A) = Length × Width
So,
A = 2 × 3.14 × (4.3)² + 6 × 14 × 5
= 116.11 + 420
= 536.11 cm²
Hence, from the above,
We can conclude that the area of the given rectangular hexagonal prism is: 536.11 cm²

Question 14.
A box has the shape of a rectangular prism with a height of 29 centimeters. If the height is increased by 0.7 centimeter, by how much does the surface area of the box increase?
Envision Math Common Core Grade 7 Answer Key Topic 8 Solve Problems Involving Geometry 110
Answer:
It is given that
A box has the shape of a rectangular prism with a height of 29 centimeters and the height is increased by 0.7 centimeters
Now,
The given rectangular prism is:
Envision Math Common Core Grade 7 Answer Key Topic 8 Solve Problems Involving Geometry 110
Now,
We know that,
The surface area of a rectangular prism (A) = Length × Width × Height
So,
A = 29 × 6.3 × 12
= 2,192.4 cm²
Now,
If the height of the given rectangular prism is increased by 0.7 cm, then
A = (29 + 0.7) × 6.3 × 12
= 2,245.32 cm²
So,
The amount of the surface area increased = 2,245.32 – 2,192.4
= 52.92 cm²
Hence, from the above,
We acn conclude that  the amount of surface area increased when the height of the box increased by 0.7 cm is: 52.92 cm²

Question 15.
The base of a prism is an equilateral triangle with an area of 73.2 square centimeters. The area of each lateral face is 104 square centimeters. Riley incorrectly claims that the surface area is 250.4 square centimeters.
a. What is the correct surface area?
Answer:
It is given that
The base of a prism is an equilateral triangle with an area of 73.2 square centimeters. The area of each lateral face is 104 square centimeters. Riley incorrectly claims that the surface area is 250.4 square centimeters.
Now,
The total surface area (A) = The area of the base of the prism + The area of each lateral face
= 73.2 + 104
= 177.2 square cm
Hence, from the above,
We can conclude that the correct surface area is: 177.2 square cm

b. What could have been Riley’s error?
Answer:
The error done by Riley is:
a. She considered the bases of a prism to be 2 and the lateral face as 1
So,
The area she got is 250.4 square cm instead of 177.2 square cm

Assessment Practice
Question 16.
The bottom part of this block is a rectangular prism. The top part is a square pyramid. How much paper, in square centimeters, is needed to cover the block completely?
Envision Math Common Core Grade 7 Answer Key Topic 8 Solve Problems Involving Geometry 111
Answer:
It is given that
The bottom part of this block is a rectangular prism. The top part is a square pyramid
Now,
The given figure is:
Envision Math Common Core Grade 7 Answer Key Topic 8 Solve Problems Involving Geometry 111
Now,
From the above,
We can observe that
The area of the given block (A) = The area of the bottom part of the block + The area of the top part of the block
= Length × Width × Height + 5 × \(\frac{1}{2}\) × Base × Height
So,
A = 3 × 4 × 4 + 5 × \(\frac{1}{2}\) × 6 × 4
= 48 + 60
= 108 cm²
Hence, from the above,
We can conclude that
The amount of paper, in square centimeters, is needed to cover the block completely is: 108 cm²

Lesson 8.9 Solve Problems Involving Volume

Solve & Discuss It!

Volunteers at a food pantry pack boxes of soup into crates. How many boxes of soup will fill each crate? Show your work.
Envision Math Common Core Grade 7 Answer Key Topic 8 Solve Problems Involving Geometry 112
I can… use the area of the base of a three-dimensional figure to find its volume.
Answer:
It is given that
Volunteers at a food pantry pack boxes of soup into crates
Now,
The given figure is:
Envision Math Common Core Grade 7 Answer Key Topic 8 Solve Problems Involving Geometry 112
Now,
From the given figure,
We can observe that the boxes and crates are in the form of a cuboid
Now,
We know that,
The volume of a cuboid = Length × Width × Height
So,
The volume of the box = 4 × 2 × 6
= 48 in.³
The volume of the crate = 12 × 18 × 12
= 2,592 in.³
So,
The number of boxes of soup that will fill each crate = \(\frac{The volume of a crate}{The volume of a box}\)
= \(\frac{2,592}{48}\)
= 54 boxes
Hence, from the above,
We can conclude that the number of boxes of soup that will fill each crate is: 54 boxes

Look for Relationships How can you layer the soup boxes to cover the bottom of the crate?
Answer:
The given figure in part (a) is:
Envision Math Common Core Grade 7 Answer Key Topic 8 Solve Problems Involving Geometry 112
Now,
From the given figure in part (a),
We can observe that
We have to layer the soup boxes in the horizontal position to cover the bottom of the crate

Focus on math practices
Reasoning A supplier donated crates to the food pantry that are 15 inches long, instead of 18 inches long. All other dimensions are the same. What is the greatest number of boxes of soup that will fit in the donated crates? How will the volume of the soup vary from the total volume of the crate?
Answer:
It is given that
A supplier donated crates to the food pantry that are 15 inches long, instead of 18 inches long. All other dimensions are the same
Now,
The given figure is:
Envision Math Common Core Grade 7 Answer Key Topic 8 Solve Problems Involving Geometry 112

Now,
From the given figure,
We can observe that the boxes and crates are in the form of a cuboid
Now,
We know that,
The volume of a cuboid = Length × Width × Height
So,
The volume of the box = 4 × 2 × 6
= 48 in.³
The volume of the crate = 12 × 15 × 12
= 2,160 in.³
So,
The number of boxes of soup that will fill each crate = \(\frac{The volume of a crate}{The volume of a box}\)
= \(\frac{2,160}{48}\)
= 45 boxes
Hence, from the above,
We can conclude that the greatest number of boxes of soup that will fit in the donated crates is: 45 boxes

Essential Question
How does the formula for the volume of a prism help you understand what volume of a prism means?
Answer:
The formula for the volume of a prism is:
V=Bh
where,
B is the base area
h is the height.
Now,
The base of the prism is a rectangle
So,
The area of a rectangle = Length × Width
Hence, from the above,
We can conclude that
The formula for the volume of a prism is:
V = Length× Width × Height

Try It!
What is the volume of the triangular prism?
Envision Math Common Core Grade 7 Answer Key Topic 8 Solve Problems Involving Geometry 113
The volume of the prism is __________ cubic centimeters.
Answer:
The given figure is:

Now,
We know that,
The volume of a triangular prism (V) = Area of a triangle × Height
Now,
We know that,
The area of a triangle (A) = \(\frac{1}{2}\) × Base of the triangle × Height of the triangle
So,
The volume of a triangular prism (V) = \(\frac{1}{2}\) × 8 × 6 × 14
= 336 cm³
Hence, from the above,
We can conclude that the volume of the given triangular prism is: 336 cm³

Convince Me! What is the shape of the base of the figure? What are its dimensions? Explain.
Answer:
The given figure is:

Now,
From the given triangular prism,
We can observe that the shape of the base of the figure is: Triangle
Now,
From the given figure,
The dimensions of the base of the figure are:
The base of the triangle: 8 cm
The height of the triangle: 6 cm
Hence, from the above,
We can conclude that the dimensions of the base of the given figure is:
Base: 8 cm
Height: 6 cm

Try It!
Amber built a custom terrarium for her plants. What is the volume of the terrarium?
Envision Math Common Core Grade 7 Answer Key Topic 8 Solve Problems Involving Geometry 114
Answer:
It is given that
Amber built a custom terrarium for her plants.
Now,
The given figure is:
Envision Math Common Core Grade 7 Answer Key Topic 8 Solve Problems Involving Geometry 114
Now,
From the given figure,
We can observe that
The terrarium is made up of two rectangular prisms
Now,
We know that,
The volume of a rectangular prism (V) = Length × Width × Height
Now,
The volume of the first rectangular prism (V) = 30 × 10 × 10
= 3,000 in.³
The volume of the second rectangular prism (V) = 40 × 10 × 12
= 4,800 in.³
So,
The volume of the custom terrarium (V) = The volume of the first rectangular prism + The volume of the second rectangular prism
= 3,000 + 4,800
= 7,800 in.³
Hence, from the above,
We can conclude that the volume of the custom terrarium that was built by Amber is: 7,800 in.³

KEY CONCEPT

You can use formulas to solve problems involving the volume of three-dimensional figures.
Find the volume of a composite figure by finding the sum of the volumes of each solid figure.
Envision Math Common Core Grade 7 Answer Key Topic 8 Solve Problems Involving Geometry 115
You can use the formula V = Bh to find the volume or unknown dimension of a solid figure.
Envision Math Common Core Grade 7 Answer Key Topic 8 Solve Problems Involving Geometry 116

Do You Understand?

Question 1.
Essential Question How does the formula for volume of a prism help you understand what volume of a prism means?
Answer:
The formula for the volume of a prism is:
V=Bh
where,
B is the base area
h is the height.
Now,
The base of the prism is a rectangle
So,
The area of a rectangle = Length × Width
Hence, from the above,
We can conclude that
The formula for the volume of a prism is:
V = Length× Width × Height

Question 2.
Look for Relationships If you know the volume of a three-dimensional figure, how can you find a missing dimension of the figure?
Answer:
We know that,
The volume of any three-dimensional figure is:
V = Length × Width × Height
Now,
If we have any missing dimension in the figure that is related to the volume, then we will first multiply the known dimensions that are present with the unknown dimension and then divide the product of the known dimensions with the volume of the three-dimensional figure

Question 3.
Make Sense and Persevere How do you find the volume of a three-dimensional figure that can be decomposed into prisms?
Answer:
We know that,
A prism is a three-dimensional figure
We know that,
The volume of a prism (V) = Length × Width × Height
So,
If we have to find the volume of a three-dimensional figure that can be decomposed into prisms, then add all the volumes of the decomposed prisms

Do You Know How?

Question 4.
An aquarium has a regular hexagonal base with side lengths of 15 centimeters. When the hexagon is divided into six equal triangles, the height of each triangle is about 13 centimeters. If the aquarium is 50 centimeters tall, what is its volume?
Answer:
It is given that
An aquarium has a regular hexagonal base with side lengths of 15 centimeters. When the hexagon is divided into six equal triangles, the height of each triangle is about 13 centimeters and the aquarium is 50 centimeters tall
Now,
From the given information,
We can observe that the aquarium is in the form of a regular hexagonal prism
Now,
We know that,
The volume of any three-dimensional figure = Base area × Height
Where,
The base area for this is the area of the hexagon
Now,
We know that,
The area of the hexagon (B) = 6 × \(\frac{1}{2}\) × base × Height
= 6 × \(\frac{1}{2}\) × 15 × 13
= 585 cm²
So,
The volume of the given aquarium (V) = 585 × 50
= 29,250 cm³
Hence, from the above,
We can conclude that the volume of the given aquarium is: 29,250 cm³

Question 5.
A cheese box is shaped like a right triangular prism. The box is 6 inches long, 4 inches tall, and has a volume of 24 cubic inches. Can a cube of cheese that is 2.5 inches on each side fit inside the box? Explain.
Envision Math Common Core Grade 7 Answer Key Topic 8 Solve Problems Involving Geometry 117
Answer:
It is given that
A cheese box is shaped like a right triangular prism. The box is 6 inches long, 4 inches tall, and has a volume of 24 cubic inches.
Now,
We know that,
The volume of a right triangular prism (V) = Base area of a right triangle × Height
Now,
We know that,
Area of a triangle (A) = \(\frac{1}{2}\) × Base × Height
So,
24 = \(\frac{1}{2}\) × 6 × 4 × h
24 = 12 × h
h = \(\frac{24}{12}\)
h = 2 inches
Now,
From the above,
We can observe that
2 inches < 2.5 inches
Hence, from the above,
We can conclude that a cube of cheese that is 2.5 inches on each side does not fit inside the box

Question 6.
Ray made a toolbox with the dimensions shown to store garden tools. What is the volume of the toolbox?
Envision Math Common Core Grade 7 Answer Key Topic 8 Solve Problems Involving Geometry 118
Answer:
It is given that
Ray made a toolbox with the dimensions shown to store garden tools.
Now,
The given figure is:
Envision Math Common Core Grade 7 Answer Key Topic 8 Solve Problems Involving Geometry 118
So,
The volume of the toolbox = (The volume of a cuboid) + (The volume of a trapezoidal prism)
Now,
We know that,
The volume of a cuboid (V) = Length × Width × Height
The volume of a trapezoidal prism (V) = (Area of a trapezoid) × (Height of the trapezoidal prism)
The area of a trapezoid (A) = [(b 1 + b 2 ) h] / 2
So,
The volume of the toolbox = (16 × 3 × 5) + [(7 + 7) × 5] ÷ 2 × 5
= (16 × 15) + \(\frac{70}{2}\) × 5
= 240 + 175
= 415 in.³
Hence, from the above,
We can conclude that
The volume of the toolbox is: 415 in.³

Practice & Problem Solving

Leveled Practice In 7-8, find the volume of each prism.

Question 7.
Envision Math Common Core Grade 7 Answer Key Topic 8 Solve Problems Involving Geometry 119
Answer:
The given figure is:

Now,
From the given figure,
We can observe that the given prism is a Right-triangular prism
So,
The volume of a right-triangular prism (V) = (Area of the right triangle) × Height
So,
The volume of the given prism is given as:

Hence, from the above,
We can conclude that
The volume of the given prism is: 41.16 m³

Question 8.
Envision Math Common Core Grade 7 Answer Key Topic 8 Solve Problems Involving Geometry 120
Answer:
The given figure is:

Now,
From the given figure,
We can observe that
The given figure is: Hexagonal prism
So,
The volume of the given prism (V) = (The area of a hexagon) × Height
Now,
We know that,
The area of a hexagon (A) = \(\frac{1}{2}\) × 6 × (Base of the triangle) × (Height of the triangle)
So,

Hence, from the above,
We can conclude that
The volume of the given prism is: 1,793.4 cm³

Question 9.
A tunnel for an amusement park ride has the shape of a regular hexagonal prism with dimensions shown. The prism has a volume of 3,572.1 cubic meters. Can two 8-meter cars connected by a 3-meter connector pass through the tunnel at the same time? Explain.
Envision Math Common Core Grade 7 Answer Key Topic 8 Solve Problems Involving Geometry 121
Answer:
It is given that
A tunnel for an amusement park ride has the shape of a regular hexagonal prism with dimensions shown. The prism has a volume of 3,572.1 cubic meters.
Now,
The given figure is:
Envision Math Common Core Grade 7 Answer Key Topic 8 Solve Problems Involving Geometry 121
Now,
From the given information,
We can say that
The volume of a regular hexagonal prism (V) = (Area of a hexagon) × (Height of a prism)
Now,
We know that,
The area of a hexagon (A) = \(\frac{1}{2}\) × 6 × (Base of the triangle) × (Height of the triangle)
So,
3,572.1 = 6 × \(\frac{1}{2}\) × 7 × 8.1 × h
3,572.1 = 170.1 × h
h = \(\frac{3,572.1}{170.1}\)
h = 21 m
Now,
The area of the tunnel = 8 × 3
= 24 m²
So,
24 > 21
Hence, from the above,
We can conclude that the two 8-meter cars connected by a 3-meter connector can’t pass through the tunnel at the same time

Question 10.
A volume of 185.5 cubic feet of concrete was used to make the section of a skateboard ramp shown. How long is the ramp?
Envision Math Common Core Grade 7 Answer Key Topic 8 Solve Problems Involving Geometry 122
Answer:
It is given that
A volume of 185.5 cubic feet of concrete was used to make the section of a skateboard ramp shown.
Now,
The given figure is:
Envision Math Common Core Grade 7 Answer Key Topic 8 Solve Problems Involving Geometry 122
Now,
We know that,
The volume of a rectangular prism (V) = Length × Width × Height
Now,
Let the length of the skateboard ramp be x ft
So,
The volume of a skateboard ramp (V) = 4 × 7 × x
185.5 = 28 × x
x = \(\frac{185.5}{28}\)
x = 6.62
x ≈ 7 ft
Hence, from the above,
We can conclude that the length of the ramp is about 7 ft

Question 11.
Make Sense and Persevere A small cube has a volume of 64 cubic feet. A larger cube has sides that are three times as long as the small cube. How long are the sides of each cube?
Envision Math Common Core Grade 7 Answer Key Topic 8 Solve Problems Involving Geometry 123
Answer:
It is given that
A small cube has a volume of 64 cubic feet. A larger cube has sides that are three times as long as the small cube
Now,
We know that,
The volume of a cube (V) = Side³
So,
For a small cube,
Side³ = 64
Side = \(\sqrt[3]{64}\)
Side = 4 feet
Now,
According to the given information,
The side of a larger cube = 3 × (The side of a small cube)
= 3 × 4
= 12 feet
Hence, from the above,
We can conclude that
The side length of a small cube is: 4 feet
The side length of a larger cube is: 12 feet

Question 12.
What is the volume of the regular hexagonal prism, to the nearest cubic centimeter?
Envision Math Common Core Grade 7 Answer Key Topic 8 Solve Problems Involving Geometry 124
Answer:
The given figure is:
Envision Math Common Core Grade 7 Answer Key Topic 8 Solve Problems Involving Geometry 124
Now,
We know that,
The volume of a regular hexagonal prism (V) = (Area of a hexagon) × (Height of the prism)
Now,
We know that,
Area of a hexagon (A) = \(\frac{1}{2}\) × 6 × (Base of the triangle) × (Height of the triangle)
So,
The volume of the given prism (V) = 6 × \(\frac{1}{2}\) × 9 × 10.4 × 24
= 6,739.2
≈ 6,740 cm³
Hence, from the above,
We can conclude that the volume of the given regular hexagonal prism is: 6,740 cm³

Question 13.
A mailbox has the dimensions shown What is the volume of the mailbox?
Envision Math Common Core Grade 7 Answer Key Topic 8 Solve Problems Involving Geometry 125
Answer:
The given figure is:
Envision Math Common Core Grade 7 Answer Key Topic 8 Solve Problems Involving Geometry 125
Now,
From the given mailbox,
We can observe that
The mailbox is the combination of a right triangular prism and a regular prism
Now,
We know that,
The volume of a right triangular prism (V) = (Area of a right triangle) × (The height of a prism)
The volume of a regular prism (V) = Length × Width × Height
Now,
We knwo that,
The area of a right triangle (A) = \(\frac{1}{2}\) × (Base of a triangle) × (Height of a triangle)
So,
The volume of the given mailbox (V) = (\(\frac{1}{2}\) × 2 × 8 × 8) + (8 × 8 × 12)
= 64 + 768
= 832 in.³
Hence, from the above,
We can conclude that the volume of the given mailbox is: 832 in.³

Question 14.
Use Structure A glass bead has the shape of a prism with a rectangular prism removed. What is the volume of the glass that forms the bead?
Envision Math Common Core Grade 7 Answer Key Topic 8 Solve Problems Involving Geometry 126
Answer:
It is given that
A glass bead has the shape of a prism with a rectangular prism removed.
Now,
The given figure is:
Envision Math Common Core Grade 7 Answer Key Topic 8 Solve Problems Involving Geometry 126
Now,
The volume of the glass that forms the bead (V) = (The volume of the glass bead that has the shape of a prism) + (The volume of a rectangular pris that is removed)
= (2 × 2 × 5) + (3 × 3 × 5)
= 20 + 45
= 65 cm³
Hence, from the above,
We can conclude that
The volume of the glass that forms the bead is: 65 cm³

Question 15.
Higher Order Thinking A cake has two layers. Each layer is a regular hexagonal prism. A slice removes one face of each prism, as shown.
Envision Math Common Core Grade 7 Answer Key Topic 8 Solve Problems Involving Geometry 127
a. What is the volume of the slice?
Answer:

b. What is the volume of the remaining cake?
Answer:

Assessment Practice
Question 16.
The area of the top of the box shown is 60 square centimeters. What is the volume, in cubic centimeters, of the box?
Envision Math Common Core Grade 7 Answer Key Topic 8 Solve Problems Involving Geometry 128
Answer:
It is given that
The area of the top of the box shown is 60 square centimeters
Now,
The given figure is:
Envision Math Common Core Grade 7 Answer Key Topic 8 Solve Problems Involving Geometry 128
So,
The volume of the given box (V) = 60 × 2 × 5
= 600 cm³
Hence, from the above,
We can conclude that
The volume of the given box is: 600 cm³

Question 17.
Which of the following freezers is the best buy in terms of dollars per cubic foot?
A. Freezer A has an interior of 1 foot by 1 foot by 5 feet and sells for $499.99.
B. Freezer B has two sections of 3 cubic feet each and sells for $629.99.
C. Freezer C has interior dimensions of 1.5 feet by 1.5 feet by 4 feet and sells for $849.99.
D. Freezer D has 3 sections of 1.5 cubic feet each and sells for $444.99.
Answer:
We now that,
To know which Freezer is best,
We have to find the volume of the given Freezer and see whether for the high volume of Freezer, the cost of the Freezer is low or not
Now,
For Freezer A,
Volume (V) = 1 × 1 × 5 = 5 ft³
The ratio in terms of dollars per cubic foot = \(\frac{499.99}{5}\)
= 100
For Freezer B,
Volume (V) = 3 + 3  = 6 ft³
The ratio in terms of dollars per cubic foot = \(\frac{629.99}{6}\)
= 105
For freezer C,
Volume (V) = 1.5 ×1.5 × 4
= 2.25×4
= 9.00
= 9 ft³
The ratio in terms of dollars per cubic foot = \(\frac{849.99}{9}\)
= 94
For Freezer D,
Volume (V) = 1.5 + 1.5 + 1.5 = 4.5 ft³
The ratio in terms of dollars per cubic foot = \(\frac{444.99}{4.5}\)
= 99
Hence, from the above,
We can conclude that
The freezers that is the best buy in terms of dollars per cubic foot are:

Topic 8 Review

Topic Essential Question
How can geometry be used to solve problems?
Answer:
You can use this field of mathematics to help you solve problems by drawing your problem and separating it out into geometric shapes. To do this, you divide your shape into smaller, common geometric shapes such as squares, rectangles, triangles, and circles and use the appropriate formulas for each

Vocabulary Review
Complete each definition, and then provide an example of each vocabulary word.
Envision Math Common Core Grade 7 Answer Key Topic 8 Solve Problems Involving Geometry 129
Envision Math Common Core Grade 7 Answer Key Topic 8 Solve Problems Involving Geometry 130
Answer:

Use Vocabulary in Writing
Shawna drew this picture of three intersecting lines. Use vocabulary terms to explain how she could determine the value of x.
Envision Math Common Core Grade 7 Answer Key Topic 8 Solve Problems Involving Geometry 131
Answer:
Shawna drew this picture of three intersecting lines
Now,
The given figure is:
Envision Math Common Core Grade 7 Answer Key Topic 8 Solve Problems Involving Geometry 131
Now,
From the given figure,
We can observe that
47° and x° are the vertical angles
Now,
We know that,
The vertical angles have the same angle measures
So,
x° = 47°
Hence, from the above,
We can conclude that the value of x is: 47°

Concepts and Skills Review

Lesson 8.1 Solve Problems involving Scale Drawings

Quick Review
In a scale drawing, the scale is a ratio that relates each drawing length to the actual length it represents. To find unknown lengths, you can use the scale to write a proportion.

Example
A blueprint of a room is drawn to a scale of 2 inches = 7 feet. The actual length of one wall is 56 feet. What is the length of this wall on the blueprint?
Answer:
Use the scale to write a proportion.
\(\frac{2 \text { in. }}{7 \mathrm{ft}}=\frac{x \text { in. }}{56 \mathrm{ft}}\)
56 feet = 7 feet x 8, so multiply 2 inches by 8
x = 2 inches x 8 = 16 inches
The length in the blueprint is 16 inches.

Practice
Use the scale drawing to answer the questions.
Envision Math Common Core Grade 7 Answer Key Topic 8 Solve Problems Involving Geometry 132

Question 1.
What is the actual area of the meeting room in square yards?
Answer:
The given figure is:
Envision Math Common Core Grade 7 Answer Key Topic 8 Solve Problems Involving Geometry 132
Now,
From the given figure,
We can observe that
Scale factor = \(\frac{3}{1}\)
So,
The actual length of the meeting room = (Scale factor) × (The length of the scale drawing)
= 3 × 4
= 12 yd
So,
The actual width of the meeting room = (Scale factor) × (The width of the scale drawing)
= 3 × 1.5
= 4.5 yd
So,
The actual area of the meeting room = (The actual length of the meeting room) × (The actual width of the meeting room)
= 12 × 4.5
= 54.0
= 54 yd²
Hence, from the above,
We can conclude that
The actual area of the meeting room is: 54 yd²

Question 2.
John decides to make a new scale drawing of the meeting room. He wants the length of the room in the new drawing to be 8 inches. What is the new scale for the drawing?
Answer:
It is given that
John decides to make a new scale drawing of the meeting room. He wants the length of the room in the new drawing to be 8 inches
Now,
From Question 1,
The actual length of the meeting room = (Scale factor) × (The length of the scale drawing)
= 3 × 4
= 12 yd
So,
\(\frac{1}{8}\) = \(\frac{12}{x}\)
x = 96
So,
The new scale for the drawing is:
8 inches = 96 yd
Divide by 8 on both sides
1 inch = 12 yd
Hence, from the above,
We can conclude that
The new scale for the drawing is:
1 inch = 12 yd

Lesson 8.2 Draw Geometric Figures

Quick Review
You can classify a quadrilateral as a trapezoid, a rectangle, a square, or a parallelogram based on its side lengths, side relationships, and angle measures.

Example
Draw a quadrilateral with exactly two perpendicular sides and one angle measuring 120°. What is the name of the figure you drew?
Answer:
Envision Math Common Core Grade 7 Answer Key Topic 8 Solve Problems Involving Geometry 133
The figure has exactly one pair of parallel sides, so it is a trapezoid.

Practice
Question 1.
Draw a quadrilateral with two pairs of parallel sides, with one side measuring 5 centimeters, one side measuring 3 centimeters, and one angle measuring 45°. What is the name of the figure you drew?
Answer:
The representation of the quadrilateral with two pairs of parallel sides, with one side measuring 5 centimeters, one side measuring 3 centimeters, and one angle measuring 45° is:

Now,
From the figure,
We can observe that
a. The side lengths of the parallel sides are the same
b. The angle measures are not 90°
Hence, from the above,
We can conclude that
The quadrilateral with the given specifications is called: parallelogram

Question 2.
What quadrilaterals can you draw that have two angles measuring 115° and two angles measuring 65°?
Answer:
It is given that the two angles measuring 115° and two angles measuring 65°
So,
The representation of the quadrilateral that has two angles measuring 115° and two angles measuring 65° is:

Hence, from the above,
We can conclude that
The quadrilateral that you can draw with the same opposite angle measures is: Parallelogram

Lesson 8.3 Draw Triangles with Given Conditions

Quick Review
When you are given certain conditions for a triangle, it may be possible to draw one triangle, more than one triangle, or no triangle.

Example
How many triangles can be drawn with side lengths of 3 inches, 5 inches, and 6 inches?
Answer:
No matter how you position the sides, the triangle has the same shape and size. There is only one way to draw a triangle with these side lengths.
Envision Math Common Core Grade 7 Answer Key Topic 8 Solve Problems Involving Geometry 134

Practice
Question 1.
Can more than one triangle be drawn with side lengths of 4 centimeters and 2 centimeters and an included angle of 50°? Explain.
Answer:
It is given that
one triangle is drawn with side lengths of 4 centimeters and 2 centimeters and an included angle of 50°
So,
The representations of the triangles with the given specifications are:

Now,
From the above triangles,
We can observe that both the representations of the triangles are the same
Hence, from the above,
We can conclude that more than one triangle can not be drawn from the given specifications

Question 2.
When given angle measures of 25°, 65°, and 90°, is it impossible to draw a triangle, possible to draw only one triangle, or possible to draw more than one triangle? Explain.
Answer:
We know that,
There is more than one triangle possible for the below cases:
a. Three different angle measures
b. Two sides and an included triangle
Now,
In this question,
Three different angle measures are given
Hence, from the above,
We can conclude that  it is impossible to draw more than one triangle

Lesson 8-4 Solve Problems Using Angle Relationships

Quick Review
Angles that have a common vertex and a common side but no common interior points are adjacent angles. Supplementary angles are angles with a sum of 180°. Complementary angles are angles with a sum of 90°. When two lines intersect, the angles that have no side in common are called vertical angles. Vertical angles are equal.

Example
List all pairs of vertical angles in this figure.
Answer:
Envision Math Common Core Grade 7 Answer Key Topic 8 Solve Problems Involving Geometry 135
There are two pairs of vertical angles:
• ∠SZR and ∠XZY
• ∠SZX and ∠RZY

Practice

Use the figure from the example.
Envision Math Common Core Grade 7 Answer Key Topic 8 Solve Problems Involving Geometry 135
Question 1.
Name a pair of complementary angles.
Answer:
The given figure is:
Envision Math Common Core Grade 7 Answer Key Topic 8 Solve Problems Involving Geometry 135
Now,
We know that,
The complementary angles are the angles that have the angle measure of 90°
Hence, from the above,
We can conclude that
The pair of Complementary angles are: ∠QZX and ∠XZY

Question 2.
The measure of ∠XZY is 55°. Which other angle has a measure of 55°? Explain.
Answer:
It is given that the angle measure of ∠XZY is: 55°
Now,
The given figure is:
Envision Math Common Core Grade 7 Answer Key Topic 8 Solve Problems Involving Geometry 135
Now,
From the given figure,
We can observe that
There are two pairs of vertical angles:
a. ∠SZR and ∠XZY
b. ∠SZX and ∠RZY
Now,
We know that,
The vertical angles are the angles that have the same angle measures
Hence, from the above,
We can conclude that
The other angles that have a measure of 55° are: ∠SZR, ∠SZX, and ∠RZY

Question 3.
Use the information from Problem 2. Find the value of n.
Answer:
The given figure is:
Envision Math Common Core Grade 7 Answer Key Topic 8 Solve Problems Involving Geometry 135
Now,
From problem 2,
We can observe that
∠SZR and ∠RZY are the adjacent angles
So,
According to the given information,
(3n + 5)° + 55° = 180°
3n° = 180° – 60°
3n° = 120°
n = \(\frac{120°}{3}\)
n = 40°
Hence, from the above,
We can conclude that the value of n is: 40°

Lesson 8.5 Solve Problems Involving Circumference of a Circle

Quick Review
The distance around a circle is called its circumference. The number a (pi) is the ratio of the circumference of any circle to its diameter. So when you know the diameter, d, of a circle, or its radius, r, you can determine its circumference, C, with the formula C = πd or C = 2πr.

Example
What is the circumference of a circle with a radius of 6 meters? Use 3.14 for π.
Answer:
C = 2πr
C = 2π(6)
C ≈ 2(3.14)(6)
C = 37.68
The circumference is about 37.68 meters.

Practice

Question 1.
The length of the minute hand of a clock is 14 inches. What is the length of the path traced by the outer tip of the minute hand in one hour? Use \(\frac{22}{7}\) for π.
Answer:
It is given that
The length of the minute hand of a clock is 14 inches
So,
The length of the path traced by the outer tip of the minute hand in one hour = 2π × (The length of the minute hand of a clock)
= 2 × \(\frac{22}{7}\) × 14
= 88 inches
Hence, from the above,
We can conclude that
The length of the path traced by the outer tip of the minute hand in one hour is: 88 inches

Question 2.
The circumference of a bicycle tire is 126.5 centimeters. What is the diameter of the tire? Use 3.14 for a. Round to the nearest tenth as needed.
Answer:
It is given that
The circumference of a bicycle tire is 126.5 centimeters
Now,
We know that,
The circumference of a bicycle tire (C) = πd
Where,
d is the diameter of the bicycle tire
So,
According to the given information,
126.5 = 3.14 × d
d = \(\frac{126.5}{3.14}\)
d = 40.28
d ≈ 40.3 centimeters
Hence, from the above,
We can conclude that
The diameter of the tire is: 40.3 centimeters

Lesson 8.6 Solve Problems involving Area of a Circle

Quick Review
The area, A, of a circle can be found using the formula A = πr2, where r is the radius. You can use 3.14 or \(\frac{22}{7}\) as an approximation for π.

Example
The diameter of the logo at the center of a basketball court is 10 feet. What is the area of the logo? Use 3.14 for π.
Answer:
The radius of a circle is half the diameter. So the radius of the logo is half of 10 feet, or 5 feet. Substitute the radius into the circle area formula.
A = πr2
A = π(5)2
A ≈ 3.14(25)
A = 78.5
The area of the logo is about 78.5 square feet.

Practice

Question 1.
Jessie wants to paint the top of the table shown. What is the approximate area that she will paint? Use 3.14 for it. Round to the nearest whole number of inches.
Envision Math Common Core Grade 7 Answer Key Topic 8 Solve Problems Involving Geometry 136
Answer:
It is given that
Jessie wants to paint the top of the table shown.
Now,
The given figure is:
Envision Math Common Core Grade 7 Answer Key Topic 8 Solve Problems Involving Geometry 136
Now,
We know that,
Radius (r) = \(\frac{Diameter}{2}\)
So,
r = \(\frac{78}{2}\)
r = 39 in
Now,
We know that,
The area of a circle (A) = πr²
So,
The area of the top of table (A) = 3.14 × 39²
= 4,775.94 in²
≈ 4,776 in²
Hence, from the above,
We can conclude that the approximate area that she will paint is: 4,776 in.²

Question 2.
What is the diameter of a circle with an area of 113.04 square centimeters? Use 3.14 for π.
Answer:
It is given that
The area of a circle is: 113.04 square centimeters
Now,
We know that,
The area of a circle (A) = πr²
So,
113.04 = 3.14 × r²
r² = \(\frac{113.04}{3.14}\)
r² = 36
r = 6 cm
Now,
We know that,
Diameter (d) = 2 × Radius (r)
So,
d = 2 × 6
d = 12 cm
Hence, from the above,
We can conclude that
The diameter of the given circle is: 12 cm

Question 3.
The distance around a circular park is 88 yards. What is the area of the park? Use \(\frac{22}{7}\) for π.
Answer:
It is given that
The distance around a circular park is 88 yards
Now,
We know that,
Circumference (C) = 2πr
So,
88 = 2 × \(\frac{22}{7}\) × r
r = \(\frac{7 × 88}{2 × 22}\)
r = 14 yards
Now,
We know that,
Area of a circle (A) = πr²
So,
The area of the circular park (A) = \(\frac{22}{7}\) × 14²
= 616 yards²
Hence, from the above,
We can conclude that the area of the given circular park is: 616 yards²

Lesson 8.7 Describe Cross Sections

Quick Review
A cross section is the two-dimensional shape exposed when a three-dimensional figure is sliced. Recognizing the shape of a cross section can help in solving some problems.

Example
Muffins are packed in two layers in a box with a piece of cardboard placed between. What shape is the cardboard and what are its dimensions?
Answer:
Envision Math Common Core Grade 7 Answer Key Topic 8 Solve Problems Involving Geometry 137
The cardboard lies on a cross section shaped like a rectangle that is 15 inches long and 10 inches wide.

Practice

Question 1.
The figure shows a vertical cross-section of a right rectangular pyramid. What shape is the cross-section and what is its area?
Envision Math Common Core Grade 7 Answer Key Topic 8 Solve Problems Involving Geometry 138
Answer:
It is given that
The figure shows a vertical cross-section of a right rectangular pyramid
Now,
The given figure is:
Envision Math Common Core Grade 7 Answer Key Topic 8 Solve Problems Involving Geometry 138
Now,
From the above figure,
We can observe that
The vertical cross-section of the given pyramid is: Right triangle
Now,
We know that,
The area of a right triangle (A) = \(\frac{1}{2}\) × (Base of the triangle) × (Height of the triangle)
So,
The area of the vertical cross-section (A) = \(\frac{1}{2}\) × 7 × 5
= 17.5 cm²
Hence, from the above,
We can conclude that
The shape of the cross-section is: Right triangle
The area of the cross-section is: 17.5 cm²

Question 2.
Zach wants to slice the pyramid along a horizontal plane that intersects the pyramid above its base. Describe the cross section that would be formed.
Answer:
It is given that
Zach wants to slice the pyramid along a horizontal plane that intersects the pyramid above its base
Now,
The given figure is:
Envision Math Common Core Grade 7 Answer Key Topic 8 Solve Problems Involving Geometry 138
Hence, from the above,
We can conclude that
The shape of the horizontal plane when the pyramid is intersected is: Rectangle

Lesson 8.8 Solve Problems involving Surface Area

Quick Review
A composite figure is the combination of two or more geometric shapes. The surface area of a two- and a three-dimensional composite figure will be the sum of the areas of all the shapes, or faces.

Example
The figure shows the plan for a kitchen countertop. What is the area of the countertop?
Envision Math Common Core Grade 7 Answer Key Topic 8 Solve Problems Involving Geometry 139
Answer:
(3 • 8) + (3 • 2) + \(\frac{1}{2}\)(3 • 3) = 24 + 6 + 4.5 = 34.5
The area of the countertop is 34.5 ft2.

Practice

Question 1.
Kara wants to paint the four outside walls of her dog’s house. She will not paint the roof or the door on the front of the house. What is the area of the surface that Kara needs to paint?
Envision Math Common Core Grade 7 Answer Key Topic 8 Solve Problems Involving Geometry 140
Answer:
It is given that
Kara wants to paint the four outside walls of her dog’s house. She will not paint the roof or the door on the front of the house.
Now,
The given figure is:
Envision Math Common Core Grade 7 Answer Key Topic 8 Solve Problems Involving Geometry 140
Now,
The area of the dog’s house that Kara needed to paint (A) = (The total area of the dog’s house) – (The area of the front of the house)
= ([\(\frac{1}{2}\) × 4 × 4] + [7 × 3]) – (\(\frac{1}{2}\) × 4 × 4)
= 29 – 8
= 21 ft²
Hence, from the above,
We can conclude that
The area of the surface that Kara needs to paint is: 21 ft²

Lesson 8.9 Solve Problems involving Volume

Quick Review
You can find the volume, V, of a prism using the formula V = Bh. In this formula, B represents the area of the base of the prism and h represents the height of the prism. Volume is measured in cubic units.

If the volume of a prism is known, you may be able to use this formula to find an unknown dimension of the prism. You also can use this formula to solve problems involving volumes of composite figures that are made up of two or more prisms.

Example
Rhonda received a package in a box shaped like a rectangular prism. What is the volume of the box?
Envision Math Common Core Grade 7 Answer Key Topic 8 Solve Problems Involving Geometry 141
Answer:
Find the area of the rectangular base.
A = 9(18) = 162 in.2
Find the volume of the prism.
V = Bh
V= 162(9)
V= 1,458 in.3

Practice

Question 1.
Holly has a gift box that is shaped like a regular hexagonal prism. What is the volume of the box?
Envision Math Common Core Grade 7 Answer Key Topic 8 Solve Problems Involving Geometry 142
Answer:
It is given that
Holly has a gift box that is shaped like a regular hexagonal prism
Now,
The given figure is:
Envision Math Common Core Grade 7 Answer Key Topic 8 Solve Problems Involving Geometry 142
Now,
We know that,
The volume of a regular hexagonal prism (V) = (Area of a hexagon) × (Height of the regular hexagonal prism)
The area of a hexagon (A) = 6 × \(\frac{1}{2}\) × (Base of a triangle}{Height of a triangle}
So,
V = 6 × \(\frac{1}{2}\) × 7.8 × 12 × 9
= 2,527.2 in.³
Hence, from the above,
We can conclude that
The volume of the given regular hexagonal prism is: 2,527.2 in.³

Question 2.
A designer is planning a trail mix box that is shaped like a rectangular prism. The front of the box must have the width and height shown. The volume of the box must be 162 cubic inches. What must be the depth, d, of the box?
Envision Math Common Core Grade 7 Answer Key Topic 8 Solve Problems Involving Geometry 143
Answer:
It is given that
A designer is planning a trail mix box that is shaped like a rectangular prism. The front of the box must have the width and height shown. The volume of the box must be 162 cubic inches.
Now,
The given figure is:
Envision Math Common Core Grade 7 Answer Key Topic 8 Solve Problems Involving Geometry 143
Now,
We know that,
The volume of a rectangular prism (V) = Length × Width × Height
So,
162 = 9 × 7.5 × d
162 = 67.5 × d
d = \(\frac{162}{67.5}\)
d = 2.4 in.
Hence, from the above,
We can conclude that
The depth of the box is 2.4 in.

Question 3.
A building that is used for storage has the dimensions shown. What is the volume of the building?
Envision Math Common Core Grade 7 Answer Key Topic 8 Solve Problems Involving Geometry 144
Answer:
It is given that
A building that is used for storage has the dimensions shown
Now,
The given figure is:
Envision Math Common Core Grade 7 Answer Key Topic 8 Solve Problems Involving Geometry 144
So,
The volume of the building (V) = (8 × 28 ×14) + (12 × 16 × 14)
= 3,136 + 2,688
= 5,824 ft³
Hence, from the above,
We can conclude that
The volume of the building is: 5,824 ft³

Topic 8 Fluency Practice

Pathfinder
Shade a path from START to FINISH. Follow the answers to the problems so that each answer is greater than the one before. You can only move up, down, right, or left.
I can… use the percent equation to solve problems.

START
Envision Math Common Core Grade 7 Answer Key Topic 8 Solve Problems Involving Geometry 145

enVision Math Common Core Grade 7 Answer Key Topic 3 Analyze And Solve Percent Problems

Practice with the help of enVision Math Common Core Grade 7 Answer Key Topic 3 Analyze and Solve Percent Problems regularly and improve your accuracy in solving questions.

enVision Math Common Core 7th Grade Answers Key Topic 3 Analyze And Solve Percent Problems

Topic 3 Essential Question

How can percent show proportional relationships between quantities and be used to solve problems?

3-ACT MATH

Envision Math Common Core 7th Grade Answer Key Topic 3 Analyze And Solve Percent Problems 1
The Smart Shopper
Why do stores and manufacturers print coupons? It seems like they lose money every time you use one. Well, some coupons are designed to steer you toward a specific brand and gain your loyalty. Stores also offer coupons to get you into the store, counting on you buying other items while you are there. If you’re clever, you can use multiple coupons. Think about this during the 3-Act Mathematical Modeling lesson.3
Envision Math Common Core 7th Grade Answer Key Topic 3 Analyze And Solve Percent Problems 2

Topic 3 enVision STEM Project

Did You Know?
One of the first popular activity trackers was a pedometer, which measures number of steps taken. Some sources trace the history of the pedometer back to Leonardo da Vinci.
Envision Math Common Core 7th Grade Answer Key Topic 3 Analyze And Solve Percent Problems 3
A pedometer called Manpo-kei (10,000 steps meter) was introduced in Japan in the mid1960s. Research led by Dr. Yoshiro Hatano indicated that 10,000 steps a day is the ideal energy output to maintain health.
Envision Math Common Core 7th Grade Answer Key Topic 3 Analyze And Solve Percent Problems 4
Today, most activity trackers are electronic devices that can sync to a computer or a smartphone.

Many activity trackers are wearable technology. Some common places to wear activity trackers are on the wrist, arm, or chest. There are even collar-mounted activity trackers for dogs.
Envision Math Common Core 7th Grade Answer Key Topic 3 Analyze And Solve Percent Problems 5
Your Task: Analyze Activity Tracker Data
Activity trackers, also called fitness trackers, have become quite popular in recent years. But is the data collected actually helpful to the user? You and your classmates will explore the types of data that an activity tracker collects, and how that data can help users reach their activity and fitness goals.
Envision Math Common Core 7th Grade Answer Key Topic 3 Analyze And Solve Percent Problems 6

Topic 3 GET READY!

Review What You Know!

Vocabulary
Choose the best term from the box. Write it on the blank.

  • percent
  • proportion
  • rate
  • ratio

Question 1.
A ________is a ratio in which the first term is compared to 100.

Answer:
A percent is a ratio in which the first term is compared to 100.

Explanation:
In the above-given question,
given that,
A percent is a ratio in which the first term is compared to 100.
for example:
38/100 = x/100.
x = 38%.
38/100 = 38.

Question 2.
A ratio that relates two quantities with different units of measure is a _________.

Answer:
A ratio that relates two quantities with different units of measure is a rate.

Explanation:
In the above-given question,
given that,
A ratio that relates two quantities with different units of measure is a rate.
for example:
34mi/2 gal.
we can convert any rate to a unit rate.

Question 3.
A statement that two ratios are equal is called a _________.

Answer:
A statement that two ratios are equal is called a proportion.

Explanation:
In the above-given question,
given that,
A statement that two ratios are equal is called a proportion.
for example:
5/10 = 1/2.

Question 4.
The relationship “3 students out of 5 students” is an example of a _________.

Answer:
The relationship “3 students out of 5 students” is an example of a ratio.

Explanation:
In the above-given question,
given that,
The relationship “3 students out of 5 students” is an example of a ratio.

Fractions, Decimals, and Percents

Write each number in two equivalent forms as a fraction, decimal, or percent.
Question 5.
0.29

Answer:
The number infraction is 29/100.
decimal = 0.29.
percent = 29%.

Explanation:
In the above-given question,
given that,
the number is 0.29.
decimal = 0.29.
percent = 29%.
fraction = 29/100.

Question 6.
35%

Answer:
The number of fraction is 35/100.
decimal = 0.35.
percent = 35%.

Explanation:
In the above-given question,
given that,
the number is 35%.
decimal = 0.35.
percent = 35%.
fraction = 35/100.

Question 7.
\(\frac{2}{5}\)

Answer:
The number infraction is 2/5.
decimal = 40.0..
percent = 0.4%.

Explanation:
In the above-given question,
given that,
the number is 2/5.
decimal = 40.0.
percent = 0.4%.
fraction = 2/5.

Proportions

Find the unknown number in each proportion.
Question 8.
\(\frac{x \text { days }}{4 \text { years }}=\frac{365.25 \text { days }}{1 \text { year }}\)

Answer:
x = 1461.

Explanation:
In the above-given question,
given that,
x/4 = 365.25.
x = 365.25 x 4.
x = 1461.

Question 9.
\(\frac{33,264 \text { feet }}{x \text { miles }}=\frac{5,280 \text { feet }}{1 \text { mile }}\)

Answer:
x = 1.569.

Explanation:
In the above-given question,
given that,
3364/x = 5280/1.
x = 5280/3364.
x = 1.569.

Question 10.
A cooking magazine shows a photo of a main dish on the front cover of 5 out of the 12 issues it publishes each year. Write and solve a proportion to determine how many times a photo of a main dish will be on the front cover during the next 5 years.

Answer:
The number of times a photo of a main dish will be on the front cover during the next 5 years = 3 times.

Explanation:
In the above-given question,
given that,
A cooking magazine shows a photo of a main dish on the front cover of 5 out of the 12 issues it publishes each year.
5/100 x 12.
0.05 x 12.
0.6.
0.6 x 5 = 3.
so the number of times a photo of the main dish will be on the front cover during the next 5 years = 3 times.

Language Development

Fill in the spider map by writing new vocabulary terms on each diagonal and related ideas.
Envision Math Common Core 7th Grade Answer Key Topic 3 Analyze And Solve Percent Problems 7

Answer:
25/100 = 0.25.
50/100 = 0.5.
75/100 = 0.75.
100/100 = 1.

Explanation:
In the above-given question,
given that,
the percent of all the terms in problem.
the four terms are 25, 50, 75, and 100.
25/100 = 0.25.
50/100 = 0.5.
75/100 = 0.75.
100/100 = 1.
Envision-Math-Common-Core-7th-Grade-Answers-Key-Topic-3-Analyze And Solve Percent Problems-1

Pick A Project

PROJECT 3A
How can you make sure you are getting the best deal?
PROJECT: SEARCH FOR BARGAINS
Envision Math Common Core 7th Grade Answer Key Topic 3 Analyze And Solve Percent Problems 8

Answer:
Yes, I am getting the best deal.

Explanation:
In the above-given question,
given that,
they kept the discount of 50% on summer sale.
50/100 = 0.5.
so  I am getting the best deal.

PROJECT 3B
What makes a song’s lyrics easy to remember?
PROJECT: WRITE AND PERFORM A CHEER OR A RAP
Envision Math Common Core 7th Grade Answer Key Topic 3 Analyze And Solve Percent Problems 9

PROJECT 3C
If you were to play basketball, what strategies would you practice?
PROJECT: ESTIMATE BASKETS
Envision Math Common Core 7th Grade Answer Key Topic 3 Analyze And Solve Percent Problems 10

PROJECT 3D
What are some elements of your favorite app?
PROJECT: DESIGN AN APP ICON
Envision Math Common Core 7th Grade Answer Key Topic 3 Analyze And Solve Percent Problems 11

Lesson 3.1 Analyze Percents of Numbers

Solve & Discuss It!
Jaime’s older brother and his three friends want to split the cost of lunch. They also want to leave a 15%-20% tip. How much should each person pay?
I can… understand, find, and analyze percents of numbers.
Envision Math Common Core 7th Grade Answer Key Topic 3 Analyze And Solve Percent Problems 12
Reasoning Which line on the receipt will you use to calculate the tip?

Answer:
The amount should each person pay = 30%

Explanation:
In the above-given question,
given that,
Jaime’s older brother and his three friends want to split the cost of lunch.
They also want to leave a 15%-20% tip.
15/100 – 20/100.
50/100 = 0.5.
20/100 = 0.2.
0.5 – 0.2 = 0.3.
30/100.
so the amount should each person pay = 30%.

Focus on math practices
Reasoning How would the amount each person pays change if the tip is determined before or after the bill is split?

Essential Question
How do percents show the relationship between quantities?

Try It!

Kita’s phone had a fully charged battery. With normal usage, her phone will last 18 hours. How much time is left on Kita’s phone battery with 12% charge remaining?
Envision Math Common Core 7th Grade Answer Key Topic 3 Analyze And Solve Percent Problems 13
Kita’s phone battery has __________ hours remaining.

Answer:
The time left on Kita’s phone battery with 12% charge remaining = 3 hours.

Explanation:
In the above-given question,
given that,
Kita’s phone had a fully charged battery.
With normal usage, her phone will last 18 hours.
21/100 = x/18.
0.21 = x/18.
x/18 x 0.21.
x = 3.78.
so the time left on Kita’s phone battery with 12% charge remaining = 3 hours.
Envision-Math-Common-Core-7th-Grade-Answers-Key-Topic-3-Analyze And Solve Percent Problems-2
Convince Me! Why is 51% of a number more than half of the number?

Try It!

a. Find 0.08% of 720.

Answer:
0.576.

Explanation:
In the above-given question,
given that,
0.08% of 720.
0.08/100 x 720.
0.0008 x 720 = 0.576.

b. Find 162.5% of 200.

Answer:
162.5.

Explanation:
In the above-given question,
given that,
162.5% of 200.
162.5/100 x 200.
1.625 x 100 = 162.5.

c. Find 0.3% of 60.

Answer:
0.18.

Explanation:
In the above-given question,
given that,
0.3% of 60.
0.3/100 x 60.
0.003 x 60 = 0.18.

KEY CONCEPT
A percent is one way to represent the relationship between two quantities, generally that of a part to the whole.
Envision Math Common Core 7th Grade Answer Key Topic 3 Analyze And Solve Percent Problems 14

Do You Understand?
Question 1.
Essential Question How do percents show the relationship between quantities?

Answer:
x = 9.

Explanation:
In the above-given question,
given that,
60/100 = x/15.
0.6 = x/15.
0.6 x 15 = x.
x = 9.

Question 2.
Reasoning How does a value that is greater than 100% of the original value or less than 1% of the original value compare to the original value?.

Answer:
The original value is greater than 1%.

Explanation:
In the above-given question,
given that,
the original value is 9%.
9/100 = 0.09.
1/100 = 0.01.
0.01 is greater than 0.09.
so the original value is greater than 1%.

Question 3.
Construct Arguments Gene stated that finding 25% of a number is the same as dividing the number by \(\frac{1}{4}\). Is Gene correct? Explain.

Answer:
Yes, Gene was correct.

Explanation:
In the above-given question,
given that,
Gene stated that finding 25% of a number is the same as dividing the number by \(\frac{1}{4}\).
25/100 = 0.25.
1/4 of 100.
so Gene was correct.

Do You Know How?
Question 4.
An 8-ounce serving of apples contains 8% of your daily vitamin C. How many ounces of apples would you need to get 100% of your daily vitamin C?

Answer:
The number of ounces of apples would you need to get 100% of your daily vitamin C = 100-ounce.

Explanation:
In the above-given question,
given that,
An 8-ounce serving of apples contains 8% of your daily vitamin C.
for 8-ounce it will get 8%.
for 100 ounces it will get 100%.
so 100 ounces of apples would you need to get 100% of your daily vitamin C.

Question 5.
Find the percent of each number.
a. 59% of 640

Answer:
377.6.

Explanation:
In the above-given question,
given that,
59% of 640.
59/100 x 640.
0.59 x 640.
377.6.

b. 0.20% of 3,542

Answer:
7.084.

Explanation:
In the above-given question,
given that,
0.20% of 3542.
0.20/100 x 3542.
0.002 x 3542.
7.084.

c. 195% of 568

Answer:
1107.6.

Explanation:
In the above-given question,
given that,
195% of 568.
195/100 x 568.
1.95 x 568.
1107.6.

d. 74% of 920

Answer:
680.8.

Explanation:
In the above-given question,
given that,
74% of 920.
74/100 x 920.
0.74 x 920.
680.8.

Question 6.
Water is 2 parts hydrogen and 1 part oxygen (H2O). For one molecule of water, each atom has the atomic mass unit, u, shown. What percent of the mass of a water molecule is hydrogen?
Envision Math Common Core 7th Grade Answer Key Topic 3 Analyze And Solve Percent Problems 15

Answer:
The mass of a water molecule in hydrogen = 2.02.

Explanation:
In the above-given question,
given that,
Water is 2 parts hydrogen and 1 part oxygen (H2O).
For one molecule of water, each atom has an atomic mass unit.
the mass of hydrogen is 1.01.
1.01 x 2 = 2.02.
so the mass of a water molecule in hydrogen is 2.02.

Practice & Problem Solving

Leveled Practice In 7-8, fill in the boxes to solve.
Question 7.
A local Little League has a total of 60 players, 80% of whom are right-handed. How many right-handed players are there?
Envision Math Common Core 7th Grade Answer Key Topic 3 Analyze And Solve Percent Problems 16

Answer:
The number of right-handed players is there = 48.

Explanation:
In the above-given question,
given that,
A local Little League has a total of 60 players,
80% of whom are right-handed.
80/100 x 60.
0.8 x 60.
48.
so the number of right-handed players is there = 48.
Envision-Math-Common-Core-7th-Grade-Answers-Key-Topic-3-Analyze And Solve Percent Problems-3

Question 8.
Sandra’s volleyball team has a total of 20 uniforms. 20% are medium-sized uniforms. How many uniforms are medium-sized?
Envision Math Common Core 7th Grade Answer Key Topic 3 Analyze And Solve Percent Problems 17

Answer:
The number of uniforms is medium-sized = 4.

Explanation:
In the above-given question,
given that,
Sandra’s volleyball team has a total of 20 uniforms.
20% are medium-sized uniforms.
20/100 = 0.20.
0.20 x 20 = 4.
so the number of uniforms are medium-sized = 4.
Envision-Math-Common-Core-7th-Grade-Answers-Key-Topic-3-Analyze And Solve Percent Problems-4

Question 9.
Meg is a veterinarian. In a given week, 50% of the 16 dogs she saw were Boxers. Steve is also a veterinarian. In the same week, 7 of the 35 dogs he saw this week were Boxers. Each wants to record the part, the whole, and the percent.
a. Does Meg need to find the part, the whole, or the percent?

Answer:
Meg needs to find the whole = 8.

Explanation:
In the above-given question,
given that,
Meg is a veterinarian. In a given week, 50% of the 16 dogs she saw were boxers.
Steve is also a veterinarian.
In the same week, 7 of the 35 dogs he saw this week were Boxers.
50/100 x 16.
0.5 x 16 = 8.
7/35 x 100.
0.2 x 100 = 20.

b. Does Steve need to find the part, the whole, or the percent?

Answer:
Steve needs to find the part.

Explanation:
In the above-given question,
given that,
Steve is also a veterinarian.
In the same week, 7 of the 35 dogs he saw this week were Boxers.
50/100 x 16.
0.5 x 16 = 8.
7/35 x 100.
0.2 x 100 = 20.
so steve needs to find the part.

Question 10.
Olivia is a stockbroker. She makes 4% of her sales in commission. Last week, she sold $7,200 worth of stocks.
a. How much commission did she make last week?

Answer:
The much commission did she make last week = $288.

Explanation:
In the above-given question,
given that,
Olivia is a stockbroker.
She makes 4% of her sales in commission.
Last week, she sold $7,200 worth of stocks.
4/100 x 7200.
0.04 x 7200 = 288.
so the much commission did she make last week = $288.

b. If she were to average that same commission each week, how much would she make in commissions in a year, treating a year as having exactly 52 weeks?

Answer:
The much would she make in commissions in a year, treating a year as having exactly 52 weeks = $3744.

Explanation:
In the above-given question,
given that,
If she were to average that same commission each week.
52/100 = 0.52.
0.52 x 7200 = 3744.
so the much she make in commission is $3744.

Question 11.
The registration fee for a used car is 0.8% of the sale price of $5,700. How much is the fee?

Answer:
The fee is $45.6.

Explanation:
In the above-given question,
given that,
The registration fee for a used car is 0.8% of the sale price of $5,700.
0.8/100 x 5700.
0.008 x 5700.
45.6.
so the fee is $45.6.

Question 12.
The total cost of an item is the price plus the sales tax.
Find the sales tax to complete the table. Then find the total cost of the item.
Envision Math Common Core 7th Grade Answer Key Topic 3 Analyze And Solve Percent Problems 18

Answer:
The total cost of the item = $1.6.

Explanation:
In the above-given question,
given that,
The total cost of an item is the price plus the sales tax.
4/100 x 40.
0.04 x 40.
1.6.
Envision-Math-Common-Core-7th-Grade-Answers-Key-Topic-3-Analyze And Solve Percent Problems-5

Question 13.
Is 700% of 5 less than 10, greater than 10 but less than 100, or greater than 100? Explain your reasoning.

Answer:
35 is greater than 10 but less than 100.

Explanation:
In the above-given question,
given that,
700/100 x 5.
7 x 5.
35.
35 is greater than 10 but less than 100.

Question 14.
Is 250% of 44 less than 100, greater than 100 but less than 150, or greater than 150? Explain your reasoning.

Answer:
110 is greater than 100 but less than 150.

Explanation:
In the above-given question,
given that,
250% of 44.
250/100 x 44.
2.5 x 44.
110.

Question 15.
The seed and skin of a typical avocado is about 30%-40% of the avocado’s weight. For an 8-ounce avocado, how many ounces of edible fruit does it have?

Answer:
The number of ounces of edible fruit does it have = 5.6 ounces.

Explanation:
In the above-given question,
given that,
The seed and skin of a typical avocado is about 30%-40% of the avocado’s weight.
30/100 – 40/100.
30 + 40 = 70.
70/100 = 0.7.
0.7 x 8 = 5.6.
so the number of ounces of edible fruit does it have = 5.6 ounces.

Question 16.
A new health drink has 130% of the recommended daily allowance (RDA) for a certain vitamin. The RDA for this vitamin is 45 mg. How many milligrams of the vitamin are in the drink?

Answer:
The milligrams of the vitamin is in the drink = 60.75 mg.

Explanation:
In the above-given question,
given that,
A new health drink has 130% of the recommended daily allowance (RDA) for a certain vitamin.
The RDA for this vitamin is 45 mg.
135/100 x 45.
1.35 x 45.
60.75.
so the milligrams of the vitamin is in the drink = 60.75 mg.

Question 17.
Make Sense and Persevere 153 is 0.9% of what number? Tell which equivalent ratios you used to find the solution.

Answer:
The equivalent ratios used to find the solution = 1.377.

Explanation:
In the above-given question,
given that,
153 is 0.9%.
153 x 0.9/100.
153 x 0.009.
1.377.
so the equivalent ratio used to find the solution is 1.377.

Question 18.
Construct Arguments Brad says that if a second number is 125% of the first number, then the first number must be 75% of the second number. Is he correct? Justify your answer.

Answer:
Yes, Brad was correct.

Explanation:
In the above-given question,
given that,
Brad says that if a second number is 125% of the first number.
the first number must be 75%of the second number.
125/100 = 1.25.
75/100 = 0.75.
so he was correct.

Question 19.
Higher-Order Thinking Mark and Joe work as jewelers. Mark has an hourly wage of $24 and gets overtime for every hour he works over 40 hours. The overtime pay rate is 150% of the normal rate. Joe makes 5% commission on all jewelry he sells. Who earns more money in a week if Mark works 60 hours and Joe sells $21,000 worth of jewelry? Explain.

Answer:
Joe earns more money than the mark.

Explanation:
In the above-given question,
given that,
Mark and Joe work as jewelers.
Mark has an hourly wage of $24 and gets overtime for every hour he works over 40 hours.
The overtime pay rate is 150% of the normal rate.
5/100 = 0.05.
40 x 60 = 2400.
so Joe earns more money than the mark.

Assessment Practice

Question 20.
Pamela and John work as tutors at two different test-prep companies. Pamela earns $20 per hour. John earns $65 per pupil. Pamela works 40 hours each week. John has 11 pupils. Who earns more money in a week? Explain.

Answer:
The more money earns in a week = Pamela.

Explanation:
In the above-given question,
given that,
Pamela and John work as tutors at two different test-prep companies.
Pamela earns $20 per hour.
John earns $65 per pupil.
Pamela works 40 hours each week.
John has 11 pupils.
40 x $20 = $800.
$65 x 11 = $715.
so pamela earns more than john.

Question 21.
An Olympic-sized pool, which holds 660,000 gallons of water, is only 63% full. The pool maintenance company adds more water, filling the pool to 90% full. How many gallons of water did they add?
A. 244,200
B. 594,000
C. 178,200
D. 415,800

Answer:
The number of gallons of water they add = 178,200 gallons.

Explanation:
In the above-given question,
given that,
An Olympic-sized pool, which holds 660,000 gallons of water, is only 63% full.
The pool maintenance company adds more water, filling the pool to 90% full.
63 – 90 = 27.
27 x 660,000 = 178,200.

Lesson 3.2 Connect Percent and Proportion

Solve & Discuss It!
A florist is making flower arrangements for a party. He uses purple and white flowers in a ratio of 3 purple flowers to 1 white flower. How many flowers will he need in order to make 30 identical arrangements?
I can… use proportions to solve percent problems.
Envision Math Common Core 7th Grade Answer Key Topic 3 Analyze And Solve Percent Problems 19

Look for Relationships
How are the number of purple flowers related to the number of white flowers?

Answer:
The ratio of purple flowers to the number of white flowers = 1: 3.

Explanation:
In the above-given question,
given that,
A florist is making flower arrangements for a party.
He uses purple and white flowers in a ratio of 3 purple flowers to 1 white flower.
the ratio is 1 : 3.
so the ratio of purple flowers to the number of white flowers = 1: 3.

Focus on math practices
Make Sense and Persevere If the florist can only buy white flowers in groups of flowers that have 3 white flowers and 2 red flowers, how many red flowers will the florist have to purchase? Explain your answer.

Answer:
The ratio of white flowers to the number of red flowers = 12.

Explanation:
In the above-given question,
given that,
If the florist can only buy white flowers in groups of flowers that have 3 white flowers and 2 red flowers.
2 + 3 = 5.
5 x 6 = 30.
2 + 2 + 2 + 2 + 2 = 12.
so the ratio of white flowers to the number of red flowers = 12.

Essential Question
How does proportional reasoning relate to percent?

Try It!

Camila makes 2 of her 5 shots attempted. Is the percent of shots she made more than, less than, or the same as Emily’s percent of shots?
Envision Math Common Core 7th Grade Answer Key Topic 3 Analyze And Solve Percent Problems 20
Camila made _______% of her shots.
Camila’s percent of the shots made is _______ Emily’s.

Answer:
Camila made 2/5% of her shots.
Camila’s percent of the sots made is 40 Emily’s.

Explanation:
In the above-given question,
given that,
Camila makes 2 of her 5 shots attempted.
2/5 = p/100.
p = 2/5 x 100.
p = 0.4 x 100.
p = 40.
Envision-Math-Common-Core-7th-Grade-Answers-Key-Topic-3-Analyze And Solve Percent Problems-6

Convince Me! A hockey goalie stops 37 out of 40 shots. What percent of attempted goals did she stop?

Answer:
The percent of attempted goals did she stop = 14.8%

Explanation:
In the above-given question,
given that,
A hockey goalie stops 37 out of 40 shots.
37/100 x 40.
0.37 x 40.
14.8 %.
so the percent of attempted goals did she stop = 14.8%.

Try It!

a. Megan’s room is expanded so the width is 150% of 3 meters. What is the new width?

Answer:
The new width = 4.5 meters.

Explanation:
In the above-given question,
Megan’s room is expanded so the width is 150% of 3 meters.
150/100 x 3.
15/10 x 3.
1.5 x 3.
4.5 meters.

b. Use the soy milk label in Example 3. What is the recommended amount of iron needed each day? Round your answer to the nearest mg.

Answer:
The recommended amount of iron needed each day =

Explanation:
In the above-given question,
given that,

KEY CONCEPT
Percent problems represent a kind of proportional relationship. You can use proportional reasoning to solve percent problems.
Envision Math Common Core 7th Grade Answer Key Topic 3 Analyze And Solve Percent Problems 21

Do You Understand?
Question 1.
Essential Question How does proportional reasoning relate to percent?

Answer:
part/whole = p/100.

Explanation:
In the above-given question,
given that,
Percent problems represent a kind of proportional relationship.
if the whole is 100%.
part is p.
part/whole = p/100.

Question 2.
Reasoning Why does one of the ratios in a percent proportion always have a denominator of 100?

Answer:
The proportion always has a denominator of 100.

Explanation:
In the above-given question,
given that,
the whole is always 100%.
so part /whole = p/100.
so the proportion always has a denominator of 100.

Question 3.
Construct Arguments The proportion \(\frac{75}{W}=\frac{150}{100}\) can be used to find the whole, w. Use the language of percent to explain whether w is less than or greater than 75.

Answer:
The weight is greater than 75.

Explanation:
In the above-given question,
given that,
The proportion \(\frac{75}{W}=\frac{150}{100}\) can be used to find the whole, w.
75/w = 150/100.
75/w = 1.5.
w = 1.5 x 75.
w = 112.5.
the weight is greater than 75.

Do You Know How?
Question 4.
Write a percent proportion for the bar diagram shown.
Envision Math Common Core 7th Grade Answer Key Topic 3 Analyze And Solve Percent Problems 22

Answer:
The percent proportion for the bar diagram is 11.56

Explanation:
In the above-given question,
given that,
68/100 = 17/p.
0.68 = 17/p.
0.68 x 17 = p.
11.56 = p.

Question 5.
Use a proportion to find each value.
a. 2% of 180
Envision Math Common Core 7th Grade Answer Key Topic 3 Analyze And Solve Percent Problems 23

Answer:
n = 3.6.

Explanation:
In the above-given question,
given that,
2% of 180.
2/100 = n/180.
0.02 = n/180.
n = 0.02 x 180.
n = 3.6.
Envision-Math-Common-Core-7th-Grade-Answers-Key-Topic-3-Analyze And Solve Percent Problems-7

b. What percent is 17 out of 40?
Envision Math Common Core 7th Grade Answer Key Topic 3 Analyze And Solve Percent Problems 24

Answer:
p = 6.8.

Explanation:
In the above-given question,
given that,
17/100 = p/40.
0.17 = p/40.
p = 0.17 x 40.
p = 6.8.
Envision-Math-Common-Core-7th-Grade-Answers-Key-Topic-3-Analyze And Solve Percent Problems-8

Question 6.
Construct Arguments Gia researches online that her car is worth $3,000. She hopes to sell it for 85% of that value, but she wants to get at least 70%. She ends up selling it for $1,800. Did she get what she wanted? Justify your answer.

Answer:
Yes, she gets what she wanted.

Explanation:
In the above-given question,
given that,
Gia researches online that her car is worth $3,000.
she hopes to sell it for 85% of that value, but she wants to get at least 70%.
She ends up selling it for $1,800.
85/100 x 3000.
0.85 x 3000.
70/100 x 1800.
0.7 x 1800.

Practice & Problem Solving

Leveled Practice In 7-8, fill in the boxes to solve.
Question 7.
The rabbit population in a certain area is 200% of last year’s population. There are 1,100 rabbits this year. How many were there last year?
\(\frac{1,100}{w}\) = \(\frac{}{}\)
There were _______ rabbits last year.

Answer:
The number of rabbits last year = 2200.

Explanation:
In the above-given question,
given that,
The rabbit population in a certain area is 200% of last year’s population.
There are 1,100 rabbits this year.
1100/w = 200/100.
1100/w = 2.
w = 2 x 1100.
w = 2200.
so the number of rabbits last year = 2200.

Question 8.
A company that makes hair-care products had 3,000 people try a new shampoo. Of the 3,000 people, 9 had a mild allergic reaction. What percent of the people had a mild allergic reaction?
\(\frac{9}{3,200}\) = \(\frac{p}{}\)
Percent = _________%

Answer:
The percent of the people who had a mild allergic reaction = 8.4%.

Explanation:
In the above-given question,
given that,
A company that makes hair-care products had 3,000 people try a new shampoo.
Of the 3,000 people, 9 had a mild allergic reaction.
9/3200 = p/3000.
0.0028 = p/3000.
p = 3000 x 0.0028.
p = 8.4%.
so the percent of the people had a mild allergic reaction = 8.4%.

Question 9.
A survey was given to people who owned a certain type of car. What percent of the people surveyed were completely satisfied with the car?
Envision Math Common Core 7th Grade Answer Key Topic 3 Analyze And Solve Percent Problems 25

Answer:
The percent of the people surveyed were completely satisfied with the car = 1100.

Explanation:
In the above-given question,
given that,
A survey was given to people who owned a certain type of car.
completely satisfied = 1100.
somewhat satisfied = 740%.
not satisfied = 160%.
so the percent of the people surveyed were completely satisfied with thw car = 1100%.

Question 10.
The Washingtons buy a studio apartment for $240,000. They pay a down payment of $60,000
a. Their down payment is what percent of the purchase price?

Answer:
The down payment is 0.4.

Explanation:
In the above-given question,
given that,
The Washingtons buy a studio apartment for $240,000.
They pay a down payment of $60,000.
240000/60000 = 0.4.

b. What percent of the purchase price would a $12,000 down payment be?

Answer:
The percent of the purchase price would be a $12000 down payment = $20.

Explanation:
In the above-given question,
given that,
The Washingtons buy a studio apartment for $240,000.
240000/12000 = 20.
so the percent of the purchase price would be a $120000 down payment = $20.

Question 11.
A restaurant customer left $3.50 as a tip. The tax on the meal was 7% and the tip was 20% of the cost including tax.
Envision Math Common Core 7th Grade Answer Key Topic 3 Analyze And Solve Percent Problems 26
a. What piece of information is not needed to compute the bill after tax and tip?

Answer:
The piece of information is not needed to compute the bill after tax and tip = $0.455.

Explanation:
In the above-given question,
given that,
A restaurant customer left $3.50 as a tip.
The tax on the meal was 7% and the tip was 20% of the cost including tax.
20% – 7% = 13%.
13/100 = 0.13.
3.50 x 0.13 = 0.455.
so the piece of information is not needed to compute the bill after tax and tip = $0.455.

b. Make Sense and Persevere What was the total bill?

Answer:

Question 12.
Reasoning What is a good estimate for 380% of 60? Explain.

Answer:
The good estimate is 228.

Explanation:
In the above-given question,
given that,
380% of 60.
380/100 x 60.
3.8 x 60 = 228.
so the good estimate is 228.

Question 13.
Critique Reasoning Marna thinks that about 35% of her mail is junk mail. She gets about twice as much regular mail as junk mail. Is she correct? Explain.

Answer:
Yes, she was correct.

Explanation:
In the above-given question,
given that,
Marna thinks that about 35% of her mail is junk mail.
She gets about twice as much regular mail as junk mail.
35% + 35%.
70%.
70/100 = 0.7.
so she was correct.

Question 14.
Hypatia has read 13 chapters of a 22-chapter book. What percent of the chapters has she read?

Answer:
The chapters she has read = 9%.

Explanation:
In the above-given question,
given that,
Hypatia has read 13 chapters of a 22-chapter book.
22 – 13 = 9.
9 percent = 9/100.
so 9% of the chapters has she read.

Question 15.
A school year has 4 quarters. What percent of a school year is 7 quarters?

Answer:
The percent of a school year = 28%.

Explanation:
In the above-given question,
given that,
A school year has 4 quarters.
4/100 = 7/q.
0.04 = 7/q.
q = 0.04 x 7.
q = 0.28.
so the percent of a school year = 28%.

Question 16.
Construct Arguments A survey found that 27% of high school students and 94% of teachers and school employees drive to school. The ratio of students to employees is about 10 to 1. Roger states that the number of students who drive to school is greater than the number of teachers and employees who drive to school. Explain how Roger’s statement could be correct.

Answer:
The ratio of students to employees is about 10 to 1.

Explanation:
In the above-given question,
given that,
A survey found that 27% of high school students and 94% of teachers and school employees.
The ratio of students to employees is about 10 to 1.
27/100 = 0.27.
94/100 = 0.94.
so the ratio of students to employees is about 10 to 1.

Question 17.
Higher Order Thinking Stefan sells Jin a bicycle for $114 and a helmet for $18. The total cost for Jin is 120% of what Stefan spent originally to buy the bike and helmet. How much did Stefan spend originally? How much money did he make by selling the bicycle and helmet to Jin?
Envision Math Common Core 7th Grade Answer Key Topic 3 Analyze And Solve Percent Problems 27

Answer:
The money did he make by selling the bicycle and helmet to Jin = $132.

Explanation:
In the above-given question,
given that,
Stefan sells Jin a bicycle for $114 and a helmet for $18.
The total cost for Jin is 120% of what Stefan spent originally to buy the bike and helmet.
$114 + $18 = $132.
so the money did he make by selling the bicycle and helmet to Jin = $132.

Assessment Practice

Question 18.
Last month Nicole spent $30. This month she spent 140% of what she spent last month. Write a proportional equation to represent the situation. How much did Nicole spend this month?

Answer:
The amount did Nicole spend this month = $42.

Explanation:
In the above-given question,
given that,
Last month Nicole spent $30.
This month she spent 140% of what she spent last month.
140/100 x $30.
1.4 x $30.
$42.
so the amount did Nicole spend this month = $42.

Question 19.
Mr. Jones, the owner of a small store buys kayak paddles for $50.00 each, and sells them for 180% of the purchase price.
PART A
A customer buys a paddle for $97.65, which includes the selling price and sales tax. What is the sales tax rate?
A 7.65%
B. 4.7%
C. 9.75%
D. 8.5%

Answer:
Option C was correct.

Explanation:
In the above-given question,
given that,
Mr. Jones, the owner of a small store buys kayak paddles for $50.00 each and sells them for 180% of the purchase price.
A customer buys a paddle for $97.65, which includes the selling price and sales tax.
180/100 = 1.8.
1.8 x 50.
90.
so option C was correct.

PART B
If Mr. Jones buys paddles for $35 instead of $50, and uses the same percent increase on price, how many paddles must Mr. Jones sell for the total paddle sales, before tax, to be at least $250? Explain your answer.

Answer:
Mr. Jones sells for the total paddle sales = $40.

Explanation:
In the above-given question,
given that,
If Mr. Jones buys paddles for $35 instead of $50 and uses the same percent increase on the price.
$250/$50.
$.5.
$35 + $5 = $40.

Lesson 3.3 Represent and Use the Percent Equation

Solve & Discuss It!
Fran is shopping for a new pair of shoes. She did some research and has narrowed the options to the two pairs she likes the most. Based on buyers’ reviews, which pair do you recommend that she buy? Explain your thinking.

I can.. represent and solve percent problems using equations.
Envision Math Common Core 7th Grade Answer Key Topic 3 Analyze And Solve Percent Problems 28

Model with Math
How can you use what you know about ratios to compare the reviews?

Focus on math practices
Model with Math Describe another situation in which you could use ratios to make a decision.

Essential Question
How are percent problems related to proportional reasoning?

Try It!

An elephant weighs 15,000 pounds on Earth and 2,500 pounds on the Moon. Assuming the weights are proportional, what percent of its weight on Earth is its weight on the Moon?
Envision Math Common Core 7th Grade Answer Key Topic 3 Analyze And Solve Percent Problems 29
The elephant’s weight on the Moon is about __________% of its weight on Earth.

Answer:
The elephant’s weight on the Moon is about 6% of its weight on Earth.

Explanation:
In the above-given question,
given that,
An elephant weighs 15,000 pounds on Earth and 2,500 pounds on the Moon.
Assuming the weights are proportional.
2500 = p x 15000.
p = 15000/2500.
p = 6.
so the elephant’s weight on the Moon is about 6% of its weight on Earth.
Envision-Math-Common-Core-7th-Grade-Answers-Key-Topic-3-Analyze And Solve Percent Problems-9

Convince Me! How does the percent describe how the weights are related?

Try It!

To make a profit, a clothing store sells board shorts at 115% of the amount they paid for them. How much did the store pay for the board shorts shown?
Envision Math Common Core 7th Grade Answer Key Topic 3 Analyze And Solve Percent Problems 30

Answer:
The store pays for the board shorts = $32.2.

Explanation:
In the above-given question,
given that,
To make a profit, a clothing store sells board shorts at 115% of the amount they paid for them.
the cost of the short is $28.
115/100 x $28.
1.15 x  $28 = $32.2.
so the store pay for the board shorts = $32.2.

KEY CONCEPT

The percent equation shows how a percent relates proportional quantities. The percent is a constant of proportionality and the equation has the same form as y = mx.
Envision Math Common Core 7th Grade Answers Topic 3 Analyze And Solve Percent Problems 31

Do You Understand?
Question 1.
Essential Question How are percent problems related to proportional reasoning?

Answer:
The percent problem related to proportional reasoning is y = mx.

Explanation:
In the above-given question,
given that,
The percent equation shows how a percent relates to proportional quantities.
The percent is a constant of proportionality and the equation has the same form as y = mx.
part/whole = percent.
part = percent x whole.

Question 2.
Reasoning A waiter at a restaurant receives $11 as a tip on a $47.20 bill. He usually receives tips that are 20% of the total bill. Is the tip amount what the waiter typically receives? Explain.

Answer:
The tip amount is the waiter typically receives = $11.64.

Explanation:
In the above-given question,
given that,
A waiter at a restaurant receives $11 as a tip on a $47.20 bill.
He usually receives tips that are 20% of the total bill.
$47.20 + $11.
$58.2.
20/100 x $58.2.
0.2 x $58.2 = $11.64.

Question 3.
Construct Arguments Sara used an equation to solve the problem below. Justify each step of her work. About 11% of people are left-handed. How many people would you expect to be left-handed in a class of 30 students?
l = 0.11 • 30
ļ = 3.3
about 3 students

Answer:
The number of people would you accept to be left-handed = 3 students.

Explanation:
In the above-given question,
given that,
11% of people are left-handed.
11/100 x 30.
0.11 x 30 = 3.3.
so the number of people would you accept to be left-handed = 3 students.

Do You Know How?
Question 4.
An auto insurance company pays 12% commission to its agents for each new insurance policy they sell. How much commission does an agent make on a $1,000 policy?

Answer:
The commission does an agent makes on a $1000 policy = $10.

Explanation:
In the above-given question,
given that,
An auto insurance company pays a 12% commission to its agents for each new insurance policy they sell.
12/100 x $1000.
0.12 x $1000.
$10.
so the commission does an agent make on a $1000 policy = $10.

Question 5.
Curt and Melanie are mixing blue and yellow paint to make seafoam green paint. Use the percent equation to find how much yellow paint they should use.
Envision Math Common Core 7th Grade Answers Topic 3 Analyze And Solve Percent Problems 32

Answer:
The quantity of yellow paint they should use = 1.5.

Explanation:
In the above-given question,
given that,
Curt and Melanie are mixing blue and yellow paint to make seafoam green paint.
70% = 70/100.
70/100 = 0.7.
30/100 = 30%.
30% = 0.3.
0.3 + 0.7 = 1.
1 x 1.5 = 1.5.
so the quantity of yellow paint they should use = 1.5.

Question 6.
Bill paid $35.99 in tax on a laptop that cost $449.99. About what percent sales tax did Bill pay?

Answer:
The percent sales tax did Bill pay = 4.8%.

Explanation:
In the above-given question,
given that,
Bill paid $35.99 in tax on a laptop that cost $449.99
$449.99 + $35.99.
$485.98.
so the percent of sales tax did Bill pay = 4.8%.

Practice & Problem Solving

Leveled Practice In 7 and 8, solve each percent problem.
Question 7.
In a survey of 500 voters, 430 said they would vote for the same candidate again. What percent of the voters would vote the same way again?
part = percent. whole
_______ = p% • ________
_______ = p%

Answer:
The percent of the votes would vote the same way again = 0.086.

Explanation:
In the above-given question,
given that,
In a survey of 500 voters, 430 said they would vote for the same candidate again.
500 = p% x 430.
p% = 430/500.
p% = 0.86.
p = 0.086.
so the percent of the votes would vote the same way again = 0.086.

Question 8.
The local newspaper has letters to the editor from 40 people. If this number represents 5% of all of the newspaper’s readers, how many readers, r, does the newspaper have?
part = percent. whole
______ = ______ • r
______ = r

Answer:
The number of readers r does the newspaper have = 0.00125.

Explanation:
In the above-given question,
given that,
The local newspaper has letters to the editor from 40 people.
If this number represents 5% of all of the newspaper’s readers.
5% = 40 x r.
5/100 = 40 x r.
0.05 = 40r.
r = 0.05/40.
r = 0.00125.
so the number of readers r does the newspaper have = 0.00125.

Question 9.
Make Sense and Persevere What percent of the 16-gigabyte hard drive shown is used for photos?
Envision Math Common Core 7th Grade Answers Topic 3 Analyze And Solve Percent Problems 33

Answer:
The percent of the 16-gigabyte hard drive shown is used for photos = 0.002075.

Explanation:
In the above-given question,
given that,
percent = part/whole.
percent = 3.32/16.
percent = 0.2075.
0.2075% = 0.002075.
so the percent of the 16-gigabyte hard drive shown is used for photos = 0.002075.

Question 10.
A shirt that normally costs $30 is on sale for $21.75. What percent of the regular price is the sale price?

Answer:
The percent of the regular price = 0.0825.

Explanation:
In the above-given question,
given that,
A shirt that normally costs $30 is on sale for $21.75.
$30 – $21.75 = $8.25.
$8.25% = 0.0825.
so the percent of the regular price = 0.0825.

Question 11.
Complete the table.
Envision Math Common Core 7th Grade Answers Topic 3 Analyze And Solve Percent Problems 34

Answer:
The commission = $30.72.

Explanation:
In the above-given question,
given that,
the sales of the earning commission = $768.
the commission rate = 4%.
4/100 = 768/c.
0.04 = 768/c.
c = 0.04 x 768.
c = $30.72.
Envision-Math-Common-Core-7th-Grade-Answers-Key-Topic-3-Analyze And Solve Percent Problems-10

Question 12.
Complete the table.
Envision Math Common Core 7th Grade Answers Topic 3 Analyze And Solve Percent Problems 35

Answer:
The sales tax = $1.7991.

Explanation:
In the above-given question,
given that,
the selling price is $39.98.
the tax rate is 4.5%.
4.5 /100 x $39.98.
0.045 x $39.98.
$1.7991.
so the sales tax = $1.7991.
Envision-Math-Common-Core-7th-Grade-Answers-Key-Topic-3-Analyze And Solve Percent Problems-11

Question 13.
A restaurant automatically charges a 20% gratuity if a party has 6 or more people. How much gratuity is added to a party of 6 on a $141 bill?

Answer:
The much gratuity is added to a party of 6 on a $141 bill = $169.2.

Explanation:
In the above-given question,
given that,
A restaurant automatically charges a 20% gratuity if a party has 6 or more people.
20/100 = 0.2.
0.2 x 6 = 1.2.
1.2 x $141 = 169.2.

Question 14.
Make Sense and Persevere A large university accepts 70% of the students who apply. Of the students the university accepts, 25% actually enroll. If 20,000 students apply, how many enroll?

Answer:
The number of students that enroll = 200 students.

Explanation:
In the above-given question,
given that,
A large university accepts 70% of the students who apply.
Of the students the university accepts, 25% actually enroll.
70 + 25 = 95.
95/100 x 20000.
0.95 x 20000.
200.
so the number of students that enroll = 200 students.

Question 15.
Model with Math There are 4,000 books in the town’s library. Of these, 2,600 are fiction. Write a percent equation that you can use to find the percent of the books that are fiction. Then solve your equation.

Answer:
The percent of the books that are fiction = 0.01538.

Explanation:
In the above-given question,
given that,
There are 4,000 books in the town’s library.
Of these, 2,600 are fiction.
4000/2600 = 1.538.
1.538% = 0.01538.

Question 16.
A salesperson earns 4% commission on furnace sales.
a. What is the commission that the salesperson earns on the sale of $33,000 worth of furnaces?

Answer:
The salesperson earns on the sale of $33000 = $1320.

Explanation:
In the above-given question,
given that,
A salesperson earns a 4% commission on furnace sales.
4/100 x $33000.
0.04 x $33000.
$1320.
so the salesperson earns on the sale of $33000 = $1320.

b. Suppose the salesperson doubles his sales of furnaces. What would be true about the commission? Explain without using any calculations.

Answer:
The salesperson doubles his sales of furnaces = $2640.

Explanation:
In the above-given question,
given that,
A salesperson earns a 4% commission on furnace sales.
4/100 x $33000.
0.04 x $33000.
$1320.
1320 x $2 = $2640.
so the salesperson doubles his sales of furnaces = $2640.

Question 17.
Heidi earns 3% commission on the jewelry she sells each week. Last week, she sold the pieces of jewelry shown.
Envision Math Common Core 7th Grade Answers Topic 3 Analyze And Solve Percent Problems 36
a. How much did she make in commission?
Answer:
She makes in commission is 21.9%.

Explanation:
In the above-given question,
given that,
Heidi earns 3% commission on the jewelry she sells each week.
$110 + $275 + $200 + $145.
$730.
3% of $730.
3/100 x 730.
0.03 x 730.
21.9.

b. Reasoning How much did the jewelry store take in from her sales? How do you know?

Answer:
The jewelry store takes in from her sales 29.2%.

Explanation:
In the above-given question,
given that,
Heidi earns 3% commission on the jewelry she sells each week.
$110 + $275 + $200 + $145.
$730.
4% of $730.
4/100 x 730.
0.04 x 730.
29.2.

Question 18.
Higher Order Thinking in a company, 60% of the workers are men. If 1,380 women work for the company, how many workers are there in all? Show two different ways that you can solve this problem.

Answer:
The number of workers in all = 828 workers.

Explanation:
In the above-given question,
given that,
In a company, 60% of the workers are men.
If 1,380 women work for the company.
60% of 1380.
60/100 x 1380.
6/10 x 1380.
0.6 x 1380.
828.
so the number of workers in all = 828 workers.

Assessment Practice

Question 19.
A salesperson starts working 40 hours per week at a job with two options for being paid. Option A is an hourly wage of $19. Option B is a commission rate of 8% on weekly sales. How much does the salesperson need to sell in a given week to earn the same amount with each option?
A. $9,500
B. $4,750
C. $760
D. $320

Answer:
The salesperson needs to sell in a given week to earn the same amount with each option = $320.

Explanation:
In the above-given question,
given that,
A salesperson starts working 40 hours per week at a job with two options for being paid.
Option A is an hourly wage of $19. Option B is a commission rate of 8% on weekly sales.
8/100 x 40.
0.08 x 40.
3.2.
so the salesperson needs to sell in a given week to earn the same amount with each option = $320.

Question 20.
At a real estate agency, an agent sold a house for $382,000. The commission rate is 5.5% for the real estate agency. The commission for the agent is 30% of the amount the real estate agency gets. How much did the agent earn in commission? Explain your answer.

Answer:
The agent earn in commission = $114600.

Explanation:
In the above-given question,
given that,
At a real estate agency, an agent sold a house for $382,000.
The commission rate is 5.5% for the real estate agency.
The commission for the agent is 30% of the amount the real estate agency gets.
30/100 x $382000.
0.3 x $382000.
114600.
so the agent earn in commission = $114600.

Topic 3 Mid-Topic Checkpoint

Question 1.
Vocabulary Explain how the percent equation relates proportional quantities. Lesson 3-3

Answer:
The percent equation shows how a percent relates to proportional quantities.
y = mx.

Explanation:
In the above-given question,
given that,
The percent equation shows how a percent relates proportional quantities.
The percent is a constant of proportionality and the equation has the same form as y = mx.
part/whole = percent.
part = percent x whole.

Question 2.
Colleen buys a movie for $20 and pays 7% sales tax. Her cousin, Brad, lives in another state. Brad buys the same movie for $22 and pays 6% sales tax. Who pays more sales tax? How much more? Lessons 3-1 and 3-3

Answer:
Brad pays more sales tax = $0.1.

Explanation:
In the above-given question,
given that,
Colleen buys a movie for $20 and pays 7% sales tax.
Her cousin, Brad, lives in another state.
Brad buys the same movie for $22 and pays 6% sales tax.
7/100 x $20.
0.07 x 20 = $1.4.
6/100 x $22.
0.06 x 22 = $1.3.
Brad pays more sales tax = $0.1.

Question 3.
Kamesh and Paolo each read 40 books in one year. Kamesh read 12 nonfiction books. Thirty-five percent of the books Paolo read were nonfiction. Who read more nonfiction books? How many more? Lesson 3-2

Answer:
Kamesh read more notifications.

Explanation:
In the above-given question,
given that,
Kamesh and Paolo each read 40 books in one year.
Kamesh read 12 nonfiction books.
Thirty-five percent of the books Paolo read were nonfiction.
35% x 40.
0.35 x 40 = $14.
35% x 12.
0.35 x 12 = $4.2.
so Kamesh read more notifications.

Question 4.
Val buys a computer for $920. If this is 115% of what the store paid for the same computer, how much did the store earn on the sale? Lesson 3-3

Answer:
The much did the store earn on the sale = $1058.

Explanation:
In the above-given question,
given that,
Val buys a computer for $920.
If this is 115% of what the store paid for the same computer.
115/100 x $920.
1.15 x $920 = $1058.
so the much did the store earn on the sale = $1058.

Question 5.
For each situation, select the percent to answer the question. Lessons 3-1, 3-2, and 3-3

Answer:
25%, 5%, and 5%

Explanation:
In the above-given question,
given that,
In an auto repair shop, 14 of the 56 cars received oil changes.
14/56 = 0.25.
0.25 x 100 = 25%.
Harry pays $3.50 sales tax on a $70 item.
In a box of 250 paperclips, 50 are red.
250/50 = 5%.

Envision Math Common Core 7th Grade Answers Topic 3 Analyze And Solve Percent Problems 37

Question 6.
Explain how you can use proportional reasoning to determine the whole if you know that 21 is 60% of the whole. Lesson 3-2

Answer:
12.6%.

Explanation:
In the above-given question,
given that,
21 is 60%.
60/100 x 21.
0.6 x 21.
12.6.
Envision-Math-Common-Core-7th-Grade-Answers-Key-Topic-3-Analyze And Solve Percent Problems-12

Topic 3 Mid-Topic Performance Task

The coach of a women’s basketball team wants each of her starting players to make at least 75% of the free throws attempted during regular season games. The table shows the statistics for the starting players after the first 15 games.
Envision Math Common Core 7th Grade Answers Topic 3 Analyze And Solve Percent Problems 38

PART A
Use the table. Find the percentage of free throws made by each player. Round to the nearest whole percent.

Answer:
The percentage of free throws made by each player = 1.5%, 1.1%, 1.4%, 1.8%, and 0.7%.

Explanation:
In the above-given question,
given that,
Wilson free throws attempted = 36/24 = 1.5.
Bartholdi free throws attempted = 42/37 = 1.1.
Jhonson free throws attempted = 22/15 = 1.4.
Garcia free throws attempted = 29/16 = 1.8.
O’Malley free throws attempted = 14/12 = 0.7.
Envision-Math-Common-Core-7th-Grade-Answers-Key-Topic-3-Analyze And Solve Percent Problems-13

PART B
Choose one of the players with a free-throw percentage less than 75%. Determine a number of free throws the player could attempt and make during the next 10 games to increase her free-throw percentage to at least 75%.
Answer:

PART C
Choose one of the players with a free-throw percentage greater than 75%. Determine the number of free throws that the player could miss during the next 10 games and still maintain an overall percentage of at least 75%.
Answer:

Lesson 3.4 Solve Percent Change and Percent Error Problems

Explain It!
Nadia lives in the town of Bayville. Quinn lives in the town of Portside. Nadia and Quinn each claim that her respective town’s population is growing more rapidly.

I can… solve problems involving percent change and percent error.
Envision Math Common Core 7th Grade Answers Topic 3 Analyze And Solve Percent Problems 39

A. Write an argument to support Nadia. Why might she argue that Bayville’s population is growing more rapidly?

Answer:
The population is growing more rapidly because they have the Historic place, LightHouse, and Beachfront.

Explanation:
In the above-given question,
given that,
Nadia lives in the town of Bayville.
Quinn lives in the town of Portside.
Nadia’s population has last year 3399.
Quinn has population has 1870.
Nadia population has this year 3702.
Quinn’s population has 1926.
Nadia has a Historic place, LightHouse, and Beachfront.
so the population is growing more rapidly.

B. Write an argument to support Quinn. Why might she argue that Portside’s population is growing more rapidly?

Answer:
The population is growing more rapidly because they have the Aquarium.

Explanation:
In the above-given question,
given that,
Nadia lives in the town of Bayville.
Quinn lives in the town of Portside.
Nadia’s population has last year 3399.
Quinn has population has 1870.
Nadia population has this year 3702.
Quinn’s population has 1926.
Quinn has a Historic place, LightHouse, and Beachfront.
so the population is growing more rapidly.

c. Whose reasoning is more logical? Explain why.

Answer:
Nadia’s reasoning is more logical.

Explanation:
In the above-given question,
given that,
Nadia lives in the town of Bayville.
Quinn lives in the town of Portside.
Nadia’s population has last year 3399.
Quinn has population has 1870.
Nadia population has this year 3702.
Quinn’s population has 1926.
the population of Nadia is very high.
so Nadia’s reasoning is more logical.

Focus on math practices
Critique Reasoning Suppose Bayville’s population is expected to grow 3% next year. Nadia says that means the population will increase by 300 people. Is Nadia’s reasoning correct? Explain.

Answer:
Yes, Nadia was correct.

Explanation:
In the above-given question,
given that,
Bayville’s population is expected to grow 3% next year.
3/100 = 0.03.
300.
so Nadia was correct.

Essential Question
How is finding percent error similar to finding percent change?

Try It!

What will be the length of the alligator next year if its length changes by the same percent as it changed this year?
percent change • length this year = change in length
Envision Math Common Core 7th Grade Answers Topic 3 Analyze And Solve Percent Problems 40
0.1875 • ________ = ________
_______ + _______ = ________
Next year, the length of the alligator will be ________ inches long.

Answer:
Next year, the length of the alligator will be 45 inches long.

Explanation:
In the above-given question,
given that,
percent change • length this year = change in length.
0.1875 x 38 = 7.125.
7.125 + 38 = 45.125.
so next year, the length of the alligator will be 45 inches long.

Convince Me! Why is the increase in the alligator’s length different from year to year, even though the percent change stayed the same?

Try It!

The specification for the length of a bolt is 4.75 inches. A machinist makes a bolt that is 4.769 inches long. What is the percent error of the bolt’s length?
Envision Math Common Core 7th Grade Answers Topic 3 Analyze And Solve Percent Problems 41

Answer:
The percent error of the bolt’s = 0.019.

Explanation:
In the above-given question,
given that,
The specification for the length of a bolt is 4.75 inches.
A machinist makes a bolt that is 4.769 inches long.
4.769 – 4.75 = 0.019.
so the percent error of the bolt’s = 0.019.

KEY CONCEPT

Percent change and percent error problems are kinds of percent problems. You can use the percent equation to solve them.
amount of change = percent change • original amount
The amount of change is the difference between original and new values
Envision Math Common Core 7th Grade Answers Topic 3 Analyze And Solve Percent Problems 42

Do You Understand?
Question 1.
Essential Question How is finding percent error similar to finding percent change?

Answer:
Yes, percent error and percent change problems are the same.

Explanation:
In the above-given question,
given that,
Percent change and percent error problems are kinds of percent problems.
amount of change = percent change • original amount.
The amount of change is the difference between original and new values.

Question 2.
Reasoning Give an example of a problem in which the percent error is greater than 20%, but less than 50%. Explain how you determined the percent error.

Answer:
The percent error is less than 20% but less than 50%.

Explanation:
In the above-given question,
given that,
percent error = percent change x original amount.
percent error = 5% x 3%.
percent error = 15%.
so the percent error is less than 20% but less than 50%.

Question 3.
Construct Arguments A store manager marked up a $10 flash drive by 20%. She then marked it down by 20%. Explain why the new price of the flash drive is not $10.

Answer:
The new price of the flash drives is $2.

Explanation:
In the above-given question,
given that,
A store manager marked up a $10 flash drive by 20%.
She then marked it down by 20%.
20/100 x 10.
0.2 x 10 = 2.
so the new price of the flash drives is $2.

Do You Know How?
Question 4.
Lita’s softball team won 8 games last month and 10 this month. What was the percent change in games the team won? Was it an increase or decrease?

Answer:
The percent change in games the team won = 80.

Explanation:
In the above-given question,
given that,
Lita’s softball team won 8 games last month and 10 this month.
percent change = percent change x original amount.
percent change = 8 x 10.
percent change = 80.

Question 5.
What is the percent change in the price of a gallon of gas, to the nearest whole percent? Is it an increase or a decrease?
Envision Math Common Core 7th Grade Answers Topic 3 Analyze And Solve Percent Problems 43

Answer:
The percent change in the price of a gallon of gas = 42.

Explanation:
In the above-given question,
given that,
3.499 last year.
3.079 this year.
3.499 – 3.079 = 0.42.
42/100 = 42.
so the percent change in the price of a gallon of gas = 42.

Question 6.
Several students measured a 25-mm-long nail and wrote the measurements shown in the table below. Whose measurement had the greatest percent error? Round to the nearest percent.
Envision Math Common Core 7th Grade Answers Topic 3 Analyze And Solve Percent Problems 44

Answer:
Tenicia has the greatest percent error.

Explanation:
In the above-given question,
given that,
Several students measured a 25-mm-long nail and wrote the measurements.
26 – 25 = 1.
25 – 23 = 2.
25 – 25 = 0.
so Tenicia had the greatest measurement.
Envision-Math-Common-Core-7th-Grade-Answers-Key-Topic-3-Analyze And Solve Percent Problems-14

Practice & Problem Solving

Leveled Practice In 7-8, use the bar diagram and fill in the boxes to solve.
Question 7.
The original quantity is 10 and the new quantity is 13. What is the percent change? Is it an increase or decrease?
Envision Math Common Core 7th Grade Answers Topic 3 Analyze And Solve Percent Problems 45
The percent decrease is _______ %.

Answer:
The percent decrease is 1.3%.

Explanation:
In the above-given question,
given that,
The original quantity is 10 and the new quantity is 13.
percent change = 10 x 13.
percent change = 130.
130/100 = 1.3.
so the percent decrease is 1.3%.
Envision-Math-Common-Core-7th-Grade-Answers-Key-Topic-3-Analyze And Solve Percent Problems-15

Question 8.
The original quantity is 5 and the new quantity is 3. What is the percent change? Is it an increase or decrease?
Envision Math Common Core 7th Grade Answers Topic 3 Analyze And Solve Percent Problems 46
The percent increase is _______%.

Answer:
The percent increase is 0.006%.

Explanation:
In the above-given question,
given that,
The original quantity is 10 and the new quantity is 13.
3 = p x 5.
percent change = 3/5.
0.6/100 = 0.006..
so the percent increase is 0.006%.
Envision-Math-Common-Core-7th-Grade-Answers-Key-Topic-3-Analyze And Solve Percent Problems-16

Question 9.
At noon, a tank contained 10 cm of water. After several hours, it contained 7 cm of water. What is the percent decrease of water in the tank?

Answer:
The percent decrease of water in the tank = 3%.

Explanation:
In the above-given question,
given that,
At noon, a tank contained 10 cm of water. After several hours, it contained 7 cm of water.
percent change = 10 – 7.
percent change = 3.
so the percent decrease of water in the tank = 3%.

Question 10.
Craig likes to collect vinyl records. Last year he had 10 records in his collection. Now he has 12 records. What is the percent increase of his collection?

Answer:
The percent increase of his collection = 2%.

Explanation:
In the above-given question,
given that,
Craig likes to collect vinyl records. Last year he had 10 records in his collection.
Now he has 12 records.
percent increase = 12 – 10.
percent increase = 2.
so the percent increase of his collection = 2%.

Question 11.
Carl bought an airline ticket. Two weeks ago, the cost of this flight was $300.
Envision Math Common Core 7th Grade Answers Topic 3 Analyze And Solve Percent Problems 47
What is the percent increase?

Answer:
The percent increase = 75%.

Explanation:
In the above-given question,
given that,
Carl bought an airline ticket. Two weeks ago, the cost of this flight was $300.
percent increase = 375 – 300.
percent increase = 75.
so the percent increase = 75%.

Question 12.
On Monday, a museum had 150 visitors. On Tuesday, it had 260 visitors.
a. Estimate the percent change in the number of visitors to the museum.

Answer:
The percent change in the number of visitors to the museum = 39000.

Explanation:
In the above-given question,
given that,
On Monday, a museum had 150 visitors. On Tuesday, it had 260 visitors.
percent change = 150 x 260.
percent change = 39000.
so the percent change in the number of visitors to the museum = 39000.

b. About how many people would have to visit the museum on Wednesday to have the same percent change from Tuesday to Wednesday as from Monday to Tuesday? Explain your answer.

Answer:
The percent change from Tuesday to Wednesday = 110.

Explanation:
In the above-given question,
given that,
On Monday, a museum had 150 visitors. On Tuesday, it had 260 visitors.
percent change = 150 – 260.
percent change = 110.
so the percent change from Tuesday to Wednesday = 110.

Question 13.
Rihanna has a container with a volume of 1.5 liters. She estimates the volume to be 2.1 liters. What is the percent error?

Answer:
The percent error = 0.6.

Explanation:
In the above-given question,
given that,
Rihanna has a container with a volume of 1.5 liters.
She estimates the volume to be 2.1 liters.
percent error is similar to the percent change.
percent change = 2.1 – 1.5.
percent change = 0.6.
so the percent error = 0.6.

Question 14.
The label on a package of bolts says each bolt has a diameter of 0.35 inch. To be in the package, the percent error of the diameter must be less than 5%. One bolt has a diameter of 0.33 inch. Should it go in the package? Why or why not?

Answer:
Yes, it would go in the package.

Explanation:
In the above-given question,
given that,
The label on a package of bolts says each bolt has a diameter of 0.35 inches.
To be in the package, the percent error of the diameter must be less than 5%. One bolt has a diameter of 0.33 inches.
percent error = 0.35 – 0.33.
percent error = 0.02.
5/100 x 0.02 = 0.05 x 0.02.
0.001.
it would go in the package.

Question 15.
A band expects to have 16 songs on their next album. The band writes and records 62.5% more songs than they expect to have in the album. During the editing process, 50% of the songs are removed. How many songs will there be in the final album?

Answer:
The number of songs will there be in the final album = 200.

Explanation:
In the above-given question,
given that,
A band expects to have 16 songs on their next album.
The band writes and records 62.5% more songs than they expect to have in the album.
During the editing process, 50% of the songs are removed.
50% – 62.5% = 12.5%.
12.5% x 16.
200.
so the number of songs will there be in the final album = 200.

Question 16.
Make Sense and Persevere in the first week of July, a record 1,060 people went to the local swimming pool. In the second week, 105 fewer people went to the pool. In the third week, 135 more people went to the pool than in the second week. In the fourth week, 136 fewer people went to the pool than in the third week.
What is the percent change in the number of people who went to the pool between the first and last weeks?

Answer:
The percent change in the number of people who went to the pool between the first and last weeks = 924.

Explanation:
In the above-given question,
given that,
Persevere in the first week of July, a record 1,060 people went to the local swimming pool.
In the second week, 105 fewer people went to the pool. In the third week, 135 more people went to the pool than in the second week.
In the fourth week, 136 fewer people went to the pool than in the third week.
percent change =1060 – 136.
percent change = 924.
so the percent change in the number of people between first and last weeks = 924.

Question 17.
Be Precise You have 20 quarters. You find 40% more quarters in your room. Then you go shopping and spend 50% of the total number of quarters.
a. Write an expression that represents the total number of quarters you take with you when you go shopping.

Answer:
The total number of quarters you take with you when you go shopping = 2 quarters.

Explanation:
In the above-given question,
given that,
You have 20 quarters. You find 40% more quarters in your room.
Then you go shopping and spend 50% of the total number of quarters.
50 – 40 = 10%.
20 x 10/100.
20 x 0.1.
2.
so the total number of quarters you take with you and go shopping = 2 quarters.

b. How much money do you have left?

Answer:
The money I have left = 18 quarters.

Explanation:
In the above-given question,
given that,
You have 20 quarters. You find 40% more quarters in your room.
Then you go shopping and spend 50% of the total number of quarters.
20 – 2 = 18.
so the money I have left = 18 quarters.

Question 18.
Higher Order Thinking The dot plot shows predictions for the winning time in a 200-meter sprint. The winner finished the race in 22.3 seconds. Find the greatest percent error for a prediction to the nearest tenth of a percent. Justify your answer.
Envision Math Common Core 7th Grade Answers Topic 3 Analyze And Solve Percent Problems 48

Answer:
The greatest percent error for a prediction to the nearest tenth of a percent = 22.2 seconds.

Explanation:
In the above-given question,
given that,
The dot plot shows predictions for the winning time in a 200-meter sprint.
The winner finished the race in 22.3 seconds.
200/100 = 0.02%.
22.3 – 0.02 = 22.2 seconds.

Assessment Practice

Question 19.
The amount of money in a savings account increases from $250 to $270 in one month. If the percent increase is the same for every month, how much money will be in the account at the end of the next month?
A. $291.60
B. $295
C. $289.60
D. $300

Answer:
The much money will be in the account at the end of the next month = 0.2.

Explanation:
In the above-given question,
given that,
The amount of money in the savings account increases from $250 to $270 in one month.
270 – 250 = 20.
20% = 20/100.
20/100 = 0.2.

Question 20.
A meteorologist predicted that there would be 1.0 inches of rainfall from a storm. Instead, there were 2.2 inches of rainfall. Which statements are true?
☐ The prediction was off by 35%.
☐ If the percent error should be less than 60%, the prediction was acceptable.
☐ The percent error of the prediction was about 55%.
☐ If the percent error should be less than 20%, the prediction was acceptable.
☐ The difference between the predicted and actual rainfall was 1.2 inches.

Answer:
The statements are true is the difference between the predicted and actual rainfall was 1.2 inches.

Explanation:
In the above-given question,
given that,
A meteorologist predicted that there would be 1.0 inches of rainfall from a storm.
Instead, there were 2.2 inches of rainfall.
2.2 – 1.0 = 1.2.
so the difference between predicted and actual rainfall = 1.2 inches.

3-ACT MATH

3-Act Mathematical Modeling:
The Smart Shopper
Envision Math Common Core 7th Grade Answers Topic 3 Analyze And Solve Percent Problems 49

АСТ 1

Question 1.
After watching the video, what is the first question that comes to mind?
Answer:

Question 2.
Write the Main Question you will answer.
Answer:

Question 3.
Construct Arguments Make a prediction to answer this Main Question. Explain how you arrived at your prediction.
Envision Math Common Core 7th Grade Answers Topic 3 Analyze And Solve Percent Problems 50
Answer:

Question 4.
On the number line below, write a number that is too small to be the answer. Write a number that is too large.
Envision Math Common Core Grade 7 Answer Key Topic 3 Analyze And Solve Percent Problems 51
Answer:

Question 5.
Plot your prediction on the same number line.
Answer:

ACT 2

Question 6.
What information in this situation would be helpful to know? How would you use that information?
Answer:

Question 7.
Use Appropriate Tools What tools can you use to solve the problem? Explain how you would use them strategically.
Answer:

Question 8.
Model with Math Represent the situation using mathematics. Use your representation to answer the Main Question.
Envision Math Common Core Grade 7 Answer Key Topic 3 Analyze And Solve Percent Problems 52
Answer:

Question 9.
What is your answer to the Main Question? Is it higher or lower than your prediction? Explain why.
Envision Math Common Core Grade 7 Answer Key Topic 3 Analyze And Solve Percent Problems 53
Answer:

ACT 3

Question 10.
Write the answer you saw in the video.
Envision Math Common Core Grade 7 Answer Key Topic 3 Analyze And Solve Percent Problems 54
Answer:

Question 11.
Reasoning Does your answer match the answer in the video? If not, what are some reasons that would explain the difference?
Answer:

Question 12.
Make Sense and Persevere Would you change your model now that you know the answer? Explain.
Envision Math Common Core Grade 7 Answer Key Topic 3 Analyze And Solve Percent Problems 55
Answer:

ACT 3

Question 13.
Model with Math Explain how you used a mathematical model to represent the situation. How did the model help you answer the Main Question?
Envision Math Common Core Grade 7 Answer Key Topic 3 Analyze And Solve Percent Problems 56
Answer:

Question 14.
Be Precise Describe how you would tell the friends to use their coupons.
Answer:

SEQUEL
Question 15.
Make Sense and Persevere Suppose the $20 coupon requires a purchase of $100 or more. How would that affect your solution?

Answer:
They affect the solution = $80.

Explanation:
In the above-given question,
given that,
Suppose the $20 coupon requires a purchase of $100 or more.
$100 – $20 = $80.
so they affect the solution = $80.

Lesson 3.5 Solve Markup and Markdown Problems

Solve & Discuss It!
Clare subscribes to an online music streaming service for a yearly fee of $96. Starting next month, there will be a 12% increase in the fee.
The ad for another music streaming service is shown below. Should Clare switch? Explain.

I can… solve problems involving percent markup and markdown.
Envision Math Common Core Grade 7 Answer Key Topic 3 Analyze And Solve Percent Problems 57

Model with Math
You can use the percent equation to determine the percent increase.

Focus on math practices
Make Sense and Persevere What is another problem-solving method you could use to check that your solution makes sense?

Essential Question
How are the concepts of percent markup and percent markdown related to the percent equation?

Try It!

What is the percent markup on a $300 phone sold for $465?
Envision Math Common Core Grade 7 Answer Key Topic 3 Analyze And Solve Percent Problems 58
markup = percent markup • cost
________ = p • 300
________ = p
The percent markup on the phone is _________%.

Answer:
The percent markup on the phone is 1.55%.

Explanation:
In the above-given question,
given that,
the percent markup on a $300 phone sold for $465.
markup = percent markup x cost.
markup = p x 300.
465/300 = p.
p = 1.55.
so the percent markup on the phone is 1.55%.
Envision-Math-Common-Core-7th-Grade-Answers-Key-Topic-3-Analyze And Solve Percent Problems-17

Convince Me! How does the percent equation help solve markup problems?

Try It!

What is the selling price for a $45 pair of shoes with a 15% markup?

Answer:
The markup = 0.003.

Explanation:
In the above-given question,
given that,
the selling price = $45.
markup = 15%.
markup = percent markup x cost.
15% = p x $45.
0.15 = p x $45.
p = 0.15/45.
p = 0.003.

Try It!

Find the percent markdown for an $80 jacket that is on sale for $48.

Answer:
The markup = $32.

Explanation:
In the above-given question,
given that,
the selling price = $45.
markup = 15%.
markdown = $80 – $48.
markdown = $32.

KEY CONCEPT
You can solve markup and markdown problems using the percent equation.
Envision Math Common Core Grade 7 Answer Key Topic 3 Analyze And Solve Percent Problems 59

Do You Understand?
Question 1.
Essential Question How are the concepts of markup and markdown related to the percent equation?

Answer:
markup and markdown are proportional to each other.

Explanation:
In the above-given question,
given that,
markup = percent markup x original value.
markdown = percent markdown x original value.
markup = new value is greater than the original value.
markdown = new value is less than the original value.

Question 2.
Reasoning What does the amount of the markup or markdown represent in the percent equation?

Answer:
The amount of the markup or markdown is proportional to each other.

Explanation:
In the above-given question,
given that,
markup = percent markup x original value.
markdown = percent markdown x original value.
so the amount of the markup or markdown is proportional to each other.

Question 3.
Generalize When an item is marked up by a certain percent and then marked down by the same percent, is the sale price equal to the price before the markup and markdown?

Answer:
markup = percent markup x original value.
markdown = percent markdown x original value.

Explanation:
In the above-given question,
given that,
When an item is marked up by a certain percent and then marked down by the same percent.
markup = percent markup x original value.
markdown = percent markdown x original value.

Do You Know How?
Question 4.
An item costs $4 before tax and $4.32 after sales tax. What is the sales tax rate?
Envision Math Common Core Grade 7 Answer Key Topic 3 Analyze And Solve Percent Problems 60

Answer:
The sales tax rate = $8.32.

Explanation:
In the above-given question,
given that,
An item costs $4 before tax and $4.32 after sales tax.
$4 + $4.32.
$8.32.
so the sales tax rate = $8.32.

Question 5.
Sheila buys two concert tickets from her friend. She pays $90 for the two tickets. She looks at the tickets and sees that each ticket has a face value of $52.50.
a. How much of a markdown did her friend give Sheila? Explain how you know.

Answer:
The percent markdown = $1.174.

Explanation:
In the above-given question,
given that,
Sheila buys two concert tickets from her friend. She pays $90 for the two tickets.
She looks at the tickets and sees that each ticket has a face value of $52.50.
markdown = percent markdown x original value.
$52.50 = percent markdown x $90.
percent markdown = $90/$52.50.
percent markdown = $1.714.

b. What was the percent markdown, rounded to the nearest whole percent?

Answer:
The percent markdown rounded to the nearest whole percent = $2.

Explanation:
In the above-given question,
given that,
Sheila buys two concert tickets from her friend. She pays $90 for the two tickets.
She looks at the tickets and sees that each ticket has a face value of $52.50.
markdown = percent markdown x original value.
$52.50 = percent markdown x $90.
percent markdown = $90/$52.50.
percent markdown = $1.714.
$2.

Question 6.
Find the sale prices.
a. $4,200 with a 35% markdown

Answer:
The sale price = $1470.

Explanation:
In the above-given question,
given that,
the sales price = percent markdown x original value.
sales price = 35% x $4200.
sales price = 0.35 x $4200.
sales price = $1470.

b. $5,000 with a 44% markdown

Answer:
The sale price = $2200.

Explanation:
In the above-given question,
given that,
the sales price = percent markdown x original value.
sales price = 44% x $5000.
sales price = 0.44 x $5000.
sales price = $2200.

Practice & Problem Solving

Leveled Practice In 7-8, fill in the boxes to solve.
Question 7.
A $300 suit is marked down by 20%. Find the sale price rounded to the nearest dollar.
markdown = percent markdown • original price
markdown = $_________
original price – markdown = sale price
$ _______ – $ ______
sale price = $ ________

Answer:
The sale price = $300.

Explanation:
In the above-given question,
given that,
A $300 suit is marked down by 20%.
sale price = original price – markdown.
sale price = $300 – 20%.
sale price = $300 – 0.2.
sale price = $299.8.

Question 8.
The selling price of an item is $650 marked up from the wholesale cost of $450. Find the percent markup from wholesale cost to selling price.
selling price – markup = wholesale cost
$ _______ – $ ______ = $ _______
markup = percent markup × wholesale cost
_______ = _______ × _______
The percent markup is about ________.

Answer:
The markup = $250.

Explanation:
In the above-given question,
given that,
The selling price of an item is $650 marked up from the wholesale cost of $450.
wholesale cost = selling price – markup.
$450 = $650 – markup.
markup = $250.

Question 9.
Karen purchased the DVD player shown in the sign on the right. Find the percent markdown rounded to the nearest percent.
Envision Math Common Core Grade 7 Answer Key Topic 3 Analyze And Solve Percent Problems 61

Answer:
The markdown = 27,048.143.

Explanation:
In the above-given question,
given that,
Karen purchased the DVD player shown in the sign on the right.
markdown = percent markdown x original value.
markdown = $175.90 x $153.77.
markdown = 27,048.143.

Question 10.
A store manager instructs his employees to mark up all items by 30%. A store clerk puts a price tag of $30 on an item that the store bought for $27. As an employee, you notice that this selling price is incorrect.
a. Find the correct selling price. Round to the nearest dollar.

Answer:
The correct selling price is $3.

Explanation:
In the above-given question,
given that,
A store manager instructs his employees to mark up all items by 30%.
A store clerk puts a price tag of $30 on an item that the store-bought for $27.
selling price = original price – mark up.
selling price = $30 – $27.
selling price = $3.
so the correct selling price = $3.

b. What was the clerk’s likely error?

Answer:
The clerk’s likely error is he said his employees to mark up all items by 30%.

Explanation:
In the above-given question,
given that,
A store clerk puts a price tag of $30 on an item that the store-bought for $27.
selling price = original price – mark up.
selling price = $30 – $27.
selling price = $3.
so the correct selling price = $3.

Question 11.
Nate has $50 to spend at the grocery store. He fills his shopping cart with items totaling $46. At checkout he will have to pay 6% sales tax on all items in the cart. Does he have enough money to buy everything in his cart? Explain.

Answer:
Enough money to buy everything in his cart = $96.

Explanation:
In the above-given question,
given that,
Nate has $50 to spend at the grocery store.
He fills his shopping cart with items totaling $46.
$50 + $46.
$96.
enough money to buy everything in his cart = $96.

Question 12.
A department store buys 300 shirts at a cost of $1,800 and sells them for $10 each. Find the percent markup rounded to the nearest percent.

Answer:
The percent markup = $180.

Explanation:
In the above-given question,
given that,
A department store buys 300 shirts at a cost of $1,800 and sells them for $10 each.
markup = percent markup x original price.
$10 = p x $1800.
p = $180.
so the percent markup rounded to the nearest percent = $180.

Question 13.
Make Sense and Persevere A computer store buys a computer system at a cost of $465.60. The selling price was first at $776, but then the store advertised a 30% markdown on the system.
a. Find the current sale price. Round to the nearest cent if necessary.

Answer:
The sales price nearest cent = $310.

Explanation:
In the above-given question,
given that,
A computer store buys a computer system at a cost of $465.60.
The selling price was first at $776, but then the store advertised a 30% markdown on the system.
sales price = original price – markdown.
sales price = $465.60 – $776.
sales price = 310.4.
so the sales price nearest cent = $310.

b. Members of the store’s loyalty club get an additional 10% off their computer purchases. How much do club members pay for the computer with their discount?

Answer:

Explanation:
In the above-given question,
given that,
Members of the store’s loyalty club get an additional 10% off their computer purchases.

Question 14.
Higher Order Thinking A sporting goods store manager was selling a kayak set for a certain price. The manager offered the markdowns shown on the right, making the one-day sale price of the kayak set $328. Find the original selling price of the kayak set.
Envision Math Common Core Grade 7 Answer Key Topic 3 Analyze And Solve Percent Problems 62

Answer:
The selling price = $312.2.

Explanation:
In the above-given question,
given that,
A sporting goods store manager was selling a kayak set for a certain price.
The manager offered the markdowns shown on the right, making the one-day sale price of the kayak set $328.
original selling price = markdown x 40%.
selling price = $328 x 0.4.
selling price = $131.2.

Assessment Practice

Question 15.
Eliza cannot decide which of two bicycles to buy. The original price of each is $380. The first is marked down by 50%. The second is marked down by 30% with an additional 20% off. Which bicycle should Eliza buy if the bicycles are the same except for the selling price? Explain your answer.

Answer:
The selling price = 0.16.

Explanation:
In the above-given question,
given that,
Eliza cannot decide which of two bicycles to buy.
The original price of each is $380.
The first is marked down by 50%.
The second is marked down by 30% with an additional 20% off.
original selling price = markdown x 20%.
selling price = 80% x 0.2.
selling price = 0.16.

Question 16.
A shoe store uses a 50% markup for all of the shoes it sells. The store also charges a 10% sales tax on all purchases. What would be the purchase price, including sales tax, of a pair of shoes that has a wholesale cost of $57?

Answer:
The purchase price, including sales tax, of a pair of shoes that has a wholesale cost = $2.28.

Explanation:
In the above-given question,
given that,
A shoe store uses a 50% markup for all of the shoes it sells.
The store also charges a 10% sales tax on all purchases.
wholesale cost = 40% x $57.
wholesale cost = 0.04 x 57.
wholesale cost = $2.28.

Lesson 3.6 Solve Simple Interest Problems

Explore It!
Gerard compares the offers at two different banks to decide where he should open a savings account.
I can… apply percent reasoning to solve simple interest problems.
Envision Math Common Core Grade 7 Answer Key Topic 3 Analyze And Solve Percent Problems 63

A. Draw a representation to show how much would be in the first savings account if Gerard’s initial deposit were d dollars.
Answer:

B. Draw a representation to show how much would be in the second savings account if Gerard’s initial deposit were d dollars.
Answer:

C. Use the two representations you drew to explain how the offers at the two banks are similar and how they are different.
Answer:

Focus on math practices
Construct Arguments Gerard’s first deposit is $500. Which bank should he choose? Explain.

Essential Question
How does simple interest show proportional reasoning and relate to the percent equation?

Try It!

Victoria has another account at the bank that pays 2\(\frac{1}{2}\)% simple interest. How much interest will she earn in 8 years on an initial deposit of $250 assuming she neither adds to nor withdraws from the account?
2\(\frac{1}{2}\)% interest expressed as a decimal is ________.
Interest after 1 year: s = _______ • $ _______
= $ _______
Interest after 8 years: $ ______ • _______
= $ ______
Victoria will earn $ _______ in interest over 8 years.

Answer:
The victoria will earn $2000 in 8 years.

Explanation:
In the above-given question,
given that,
Victoria has another account at the bank that pays 2\(\frac{1}{2}\)% simple interest.
Interest after 1 year = 1 x 250.
Interest after 8 years = 8 x 250.
2000.
so victoria will earn $2000 in 8 years.

Convince Me! Would the interest for the second year be the same if it were calculated on the total after the first year? Why or why not?

Try It!

Another company will lend Maya’s older sister $4,000. Every month, she will pay $11.88 in interest. What is the interest rate, rounded to the nearest tenth of a percent, for 1 year?

Answer:
The interest rate rounded to the nearest tenth of a percent = $3857.44.

Explanation:
In the above-given question,
given that,
Another company will lend Maya’s older sister $4,000.
Every month, she will pay $11.88 in interest.
1 year = 12 months.
$11.88 x 12 = $142.56.
4000 – $142.56 = 3857.44.
so the interest rate rounded to the nearest tenth of a percent = $3857.44.

Try It!

Katelyn’s older brother borrowed money for school. He took out a loan that charges 6% simple interest. He will end up paying $720 in interest after 6 years. How much did Katelyn’s brother borrow for school?

Answer:
Katelyn’s brother borrows for school = $120.

Explanation:
In the above-given question,
given that,
Katelyn’s older brother borrowed money for school.
He took out a loan that charges 6% simple interest.
He will end up paying $720 in interest after 6 years.
$720/6 = 120.
so Katelyn’s brother borrows for school = $120.

KEY CONCEPT
Simple interest represents a proportional relationship between the yearly interest and the principal, or initial amount. The ratio of yearly interest to principal is the interest rate.
Envision Math Common Core Grade 7 Answer Key Topic 3 Analyze And Solve Percent Problems 64

Do You Understand?
Question 1.
Essential Question How does simple interest show proportional reasoning and relate to the percent equation?

Answer:
Simple interest represents a proportional relationship between the yearly interest and the principal or initial amount.

Explanation:
In the above-given question,
given that,
Simple interest represents a proportional relationship between the yearly interest and the principal or initial amount.
the ratio of yearly interest to principal to the interest rate.

Question 2.
Reasoning If the interest earned on an account after 2 years is $15, how much would it be after 10 years? Why?

Answer:
The interest earned on an account after 10 years = $60.

Explanation:
In the above-given question,
given that,
If the interest earned on an account after 2 years is $15.
2 years = $15.
10 years = $15 x $5.
10 years = $60.
so the interest earned on an account after 10 years = $60.

Question 3.
Be Precise Angelina is deciding which bank would give her the best simple interest rate on a $300 deposit. One bank says that she will have $320 in her account if she leaves the principal for 2 years. Is this enough information for Angelina to find the interest rate? Explain.

Answer:
Yes, it is enough information for Angelina to find the interest rate.

Explanation:
In the above-given question,
given that,
Angelina is deciding which bank would give her the best simple interest rate on a $300 deposit.
One bank says that she will have $320 in her account if she leaves the principal for 2 years.
2 years = 24 months.
for 1 month = 1 rs.
do this is enough information.

Do You Know How?
Question 4.
Find the missing value in each row. Use the percent equation.
Envision Math Common Core Grade 7 Answers Topic 3 Analyze And Solve Percent Problems 65

Answer:
$15.
4 years.
$50.
1%.

Explanation:
In the above-given question,
given that,
100 x 5/100 x 3 = $15.
$500 x 4/100 x 4 = $20.
$50 x 10/100 x 7 = $35.
$200 x 1/100 x 2 = $6.
Envision-Math-Common-Core-7th-Grade-Answers-Key-Topic-3-Analyze And Solve Percent Problems-18

Question 5.
Annika’s older cousin borrowed $800 to repair her car. She will pay off the loan after 2 years by paying back the principal plus 4.5% simple interest for each year.
a. How much will she pay in interest? Show your work.

Answer:
She will pay = $72.

Explanation:
In the above-given question,
given that,
Annika’s older cousin borrowed $800 to repair her car.
She will pay off the loan after 2 years by paying back the principal plus 4.5% simple interest for each year.
800 x 2 x 4.5%.
8 x 2 x 4.5.
$72.
so she will pay = $72.

b. How much will she pay back altogether?

Answer:
She will pay back = $72.

Explanation:
In the above-given question,
given that,
Annika’s older cousin borrowed $800 to repair her car.
She will pay off the loan after 2 years by paying back the principal plus 4.5% simple interest for each year.
800 x 2 x 4.5%.
8 x 2 x 4.5.
$72.
so she will pay back = $72.

Question 6.
J.D. opened a savings account with $425. After 2 years, the total interest he earned was $10.20. What was the annual interest rate?

Answer:
The annual interest rate = $870.4.

Explanation:
In the above-given question,
given that,
J.D. opened a savings account with $425.
After 2 years, the total interest he earned was $10.20.
$10.20 + $425 = $435.20.
$435.20 x 2 = $870.4.

Practice & Problem Solving

Leveled Practice In 7-8, fill in the boxes to solve.
Question 7.
Edward deposited $6,000 into a savings account 4 years ago. The simple interest rate is 3%. How much money did Edward earn in interest?
Interest = $ ______ • ______ • ______ years
Edward earned $ _______ in interest

Answer:
Edward earned = $720.

Explanation:
In the above-given question,
given that,
Edward deposited $6,000 into a savings account 4 years ago.
The simple interest rate is 3%.
interest = $6000 x 4 x 3/100.
interest = $60 x 4 x 3.
interest = $720.
so Edward earned = $720.

Question 8.
The interest on $2,000 for 2 years is $320. What is the simple interest rate?
$ ______ = $2,000 • r • ______
$ _____ = $ _____ • r
______ = r
_______% = r

Answer:
The simple interest rate = 8%.

Explanation:
In the above-given question,
given that,
The interest on $2,000 for 2 years is $320.
$320 = $2000 x r x 2.
$320 x 2 = $640.
$640 x $2000 = 8.

Question 9.
Suppose you deposited $100 in a savings account 4 years ago with a simple interest rate of 2.2%. The interest that you earned in those 4 years is $8.80. Which of the following is true? Select all that apply.
☐ The interest rate is 0.022.
☐ The principal was $100.
☐ The interest earned is $4.
☐ The account was opened 8 years and 8 months ago

Answer:
Option B, C, and D are correct.

Explanation:
In the above-given question,
given that,
Suppose you deposited $100 in a savings account 4 years ago with a simple interest rate of 2.2%.
The interest that you earned in those 4 years is $8.80.
the principal was $100.
the interest earned is $4.
the account was opened 8 years and 8 months ago.

Question 10.
A new bank customer with $3,000 wants to open a money market account. The bank is offering a simple interest rate of 1.1%.
a. How much interest will the customer earn in 20 years?

Answer:
The interest will the customer earn in 20 years = $660.

Explanation:
In the above-given question,
given that,
A new bank customer with $3,000 wants to open a money market account.
The bank is offering a simple interest rate of 1.1%.
3000 x 11/1000 x 20.
60 x 11 = 660.
so the interest will the customer earn in 20 years = 660.

b. What will be the account balance after 20 years?

Answer:
The interest will the customer earn in 20 years = $660.

Explanation:
In the above-given question,
given that,
A new bank customer with $3,000 wants to open a money market account.
The bank is offering a simple interest rate of 1.1%.
3000 x 11/1000 x 20.
60 x 11 = 660.
so the interest will the customer earn in 20 years = 660.

Question 11.
Boden’s account has a principal of $500 and a simple interest rate of 3.3%. Complete the double number line. How much money will be in the account after 4 years, assuming Boden does not add or take out any money?
Envision Math Common Core Grade 7 Answers Topic 3 Analyze And Solve Percent Problems 66

Answer:
The money will be in the account after 4 years = $66.

Explanation:
In the above-given question,
given that,
Boden’s account has a principal of $500 and a simple interest rate of 3.3%.
$500 x 4 x 3.3.
500 x 4 x 33/1000.
20 x 3.3 = $66.
Envision-Math-Common-Core-7th-Grade-Answers-Key-Topic-3-Analyze And Solve Percent Problems-19

Question 12.
Critique Reasoning Monica deposits $100 into a savings account that pays a simple interest rate of 3.4%. Paul deposits $200 into a savings account that pays a simple interest rate of 2.2%. Monica says that she will earn more interest in one year because her interest rate is higher. Do you agree? Justify your response.

Answer:
Monica says that she will earn more interest in one year because her interest is higher.

Explanation:
In the above-given question,
given that,
Monica deposits $100 into a savings account that pays a simple interest rate of 3.4%.
Paul deposits $200 into a savings account that pays a simple interest rate of 2.2%.
100 x 3.4 =
34.
200 x 2.2 = 8.8.

Question 13.
Construct Argument Tommy earned $76.00 in interest after 5 years on a principal of $400. Jane earned $82.00 in interest after 2 years on a principal of $1,000. Which bank would you rather use, Tommy’s or Jane’s? Why?

Answer:
I will use Jane’s bank.

Explanation:
In the above-given question,
given that,
Tommy earned $76.00 in interest after 5 years on a principal of $400.
Jane earned $82.00 in interest after 2 years on a principal of $1,000.
400 x 5 x $76 = $152000.
1000 x 82 x 2 = $164000.
Jane’s bank will get the higher principal.

Question 14.
Reasoning A bank manager wants to encourage new customers to open accounts with initial deposits of at least $3,000. He has posters made for the promotion.
Envision Math Common Core Grade 7 Answers Topic 3 Analyze And Solve Percent Problems 67
a. Under the new promotion, what is the minimum amount of interest a new account would make in one year if there were no withdrawls from the account?

Answer:
There were no withdrawals from the account = $144.

Explanation:
In the above-given question,
given that,
A bank manager wants to encourage new customers to open accounts with initial deposits of at least $3,000.
simple interest of 4.8%.
4.8 x 1 x 3000.
3000 x 48/1000 x 1.
$144.

b. The manager wants to add the sentence, “Open an account with $3,000 and earn at least $120 interest each year!” to the poster. Do you agree? Explain.

Answer:
Yes, the sentence was correct.

Explanation:
In the above-given question,
given that,
The manager wants to add the sentence, “Open an account with $3,000 and earn at least $120 interest each year!.
4.8 x 1 x 3000.
3000 x 48/1000 x 1.
$144.
so 144 is greater than $120.
the sentence was correct.

Question 15.
Suppose you deposit $100 in Account A with a simple interest rate of 3.4%, and $300 in Account B with a simple interest rate of 1.8%. One year later, you get a bank statement that shows the interest for Account A is $3.40 and the interest for Account B is $540.00.
a. Which account statement is incorrect?

Answer:
Yes, the account B statement is incorrect.

Explanation:
In the above-given question,
given that,
Suppose you deposit $100 in Account A with a simple interest rate of 3.4%, and $300 in Account B with a simple interest rate of 1.8%.
One year later, you get a bank statement that shows the interest for Account A is $3.40 and the interest for Account B is $540.00.
3.4 x 100.
34/1000 x 100.
3.4 x 1.
3.4.
1.8% x 300.
18/1000 x 300.
1.8 x 3.
5.4.
so the bank statement is incorrect for Account B.

b. What may have been the bank’s error?

Answer:
The error is account B.

Explanation:
In the above-given question,
given that,
Suppose you deposit $100 in Account A with a simple interest rate of 3.4%, and $300 in Account B with a simple interest rate of 1.8%.
One year later, you get a bank statement that shows the interest for Account A is $3.40 and the interest for Account B is $540.00.
3.4 x 100.
34/1000 x 100.
3.4 x 1.
3.4.
1.8% x 300.
18/1000 x 300.
1.8 x 3.
5.4.

Question 16.
Higher Order Thinking You have two different savings accounts. For Account A, the interest earned after 18 months is $12.00. For Account B, the interest earned after 27 months is $27.00.
a. If the interest rate is 3.2% for Account A, how much is the principal?

Answer:
The principle is $6.912.

Explanation:
In the above-given question,
given that,
You have two different savings accounts.
For Account A, the interest earned after 18 months is $12.00.
For Account B, the interest earned after 27 months is $27.00.
$12 x 18 x 3.2%.
$12 x 18 x 0.032.
$12 x 0.576.
$6.912.

b. If the interest rate is 2.4% for Account B, how much is the principal?

Answer:
The principle is $17.496.

Explanation:
In the above-given question,
given that,
You have two different savings accounts.
For Account A, the interest earned after 18 months is $12.00.
For Account B, the interest earned after 27 months is $27.00.
27 x $27 x 0.024.
$27 x 0.648.
17.496.

c. Which account earned you the most interest in the first year? Explain.

Answer:
Account B has earned the most interest in the first year.

Explanation:
In the above-given question,
given that,
You have two different savings accounts.
For Account A, the interest earned after 18 months is $12.00.
For Account B, the interest earned after 27 months is $27.00.
27 x $27 x 0.024.
$27 x 0.648.
17.496.

Assessment Practice

Question 17.
A certificate of deposit with principal $150 earns 4% annual interest for 2 years. Select all the options that would earn the same amount of interest.
☐ $300 at 2% for 2 years
☐ $150 at 6% for 18 months
☐ $400 at 3% for 1 year
☐ $100 at 1% for 8 years
☐ $50 at 48% for 6 months

Answer:
Option A and option C are correct.

Explanation:
In the above-given given question,
given that,
A certificate of deposit with a principal of $150 earns 4% annual interest for 2 years.
$150 x 4% x 2.
$150 x 4/100 x 2.
$15 x 0.4 x 2.
$12.
so option A and option C are correct.

Question 18.
Dakota earned $15.75 in interest in Account A and $28.00 in interest in Account B after 21 months. If the simple interest rate is for 3.0% for Account A and 4.0% for Account B, which account had the greater principal? Explain.

Answer:
Account B had the greater principal.

Explanation:
In the above-given question,
given that,
Dakota earned $15.75 in interest in Account A and $28.00 in interest in Account B after 21 months.
If the simple interest rate is 3.0% for Account A and 4.0% for Account B, which account had the greater principal.
$15.75 x 3% x 21.
15.75 x 0.003 x 21.
0.99225.
$28 x 21 x 4%.
$28 x 21 x 0.04.
$23.52.
so account B had the greater principal.

Topic 3 Review

Topic Essential Question
How can percents show proportional relationships between quantities and be used to solve problems?

Vocabulary Review
Complete each definition and then provide an example of each word.
Envision Math Common Core Grade 7 Answers Topic 3 Analyze And Solve Percent Problems 68

Answer:
1: Markup simple interest.
2: Percent change.
3: Markup principal.
4: Vocabulary.

Explanation:
In the above-given question,
given that,
the markup simple interest is the decrease from the original price of an item to its sale price.
the accuracy of a measured or estimated value compared to its actual value is described as the percent change.
an initial amount of money that is deposited in an account is called the markup principal.
the vocabulary describes how much a quantity has changed relative to its original amount.
Envision-Math-Common-Core-7th-Grade-Answers-Key-Topic-3-Analyze And Solve Percent Problems-20

Use Vocabulary in Writing
Birute deposits $500 in a savings account with a simple interest rate of 1.3%. How could you use this information to find the interest she would earn in 4 years and determine the percent change in her savings account?

Answer:
The percent change in her savings account.

Explanation:
In the above-given question,
given that,
Birute deposits $500 in savings account with a simple interest rate of 1.3%
How could you use this information to find the interest she would earn in 4 years and determine the percent change.
$500 x 1.3% x 4.
$500 x 13/1000 x 4.
5 x 1.3 x 4.
26.
so the percent change in her savings account.

Concepts and Skills Review

Lesson 3.1 Analyze Percents of Numbers

Quick Review
You can use equivalent ratios to find the percent of a number. Remember that a percent is a ratio that relates a number to 100.

Example
Find 57% of 690.
\(\frac{57}{100}=\frac{x}{690}\)
\(\frac{57}{100} \cdot 690=\frac{x}{690} \cdot 690\)
393.3 = x

Practice
Question 1.
Find 0.8% of 1,046.

Answer:
8.368.

Explanation:
In the above-given question,
given that,
0.8% of 1046.
0.8/100 x 1046.
0.008 x 1046.
8.368.

Question 2.
Find 160% of 98.

Answer:
156.8.

Explanation:
In the above-given question,
given that,
160% of 98.
160/100 x 98.
1.6 x 98.
156.8.

Question 3.
A company charges a shipping fee that is 4.5% of the purchase price for all items it ships. What is the fee to ship an item that costs $56?

Answer:
The fee to ship an item that costs = $2.52.

Explanation:
In the above-given question,
given that,
company charges a shipping fee that is 4.5% of the purchase price for all items it ships.
$56 x 4.5%.
56 x 45/1000.
$56 x 0.045.
$2.52.

Lesson 3.2 Connect Percent and Proportion

Quick Review
You can use proportions to solve different types of percent problems. There are three values in a percent problem–the percent, the part, and the whole. If you know two of these values, you can set up a proportion to find the third value.

Example
What percent of 19 is 4.75?
The whole is 19 and the part is 4.75. Write a proportion, and solve for the percent, p.
\(\frac{\text { part }}{\text { whole }}=\frac{p}{100}\)
\(\frac{4.75}{19}=\frac{p}{100}\)
\(\frac{4.75}{19} \cdot 100=\frac{p}{100} \cdot 100\)
25 = p
So, 4.75 is 25% of 19.

Practice
Question 1.
24.94 is 29% of what number?

Answer:
The number is 7.3.

Explanation:
In the above-given question,
given that,
24.94 is 29% of what number.
24.94 / p = 29/100.
p = 0.29 x 24.94.
p = 7.3.

Question 2.
On Thursday, a restaurant serves iced tea to 35 of its 140 customers. What percent of the customers order iced tea?

Answer:
The percent of the customers who order iced tea = 25%.

Explanation:
In the above-given question,
given that,
On Thursday, a restaurant serves iced tea to 35 of its 140 customers.
35/140 = p/100.
0.25 x 100 = p.
p = 25%.

Question 3.
Liam puts $40 in savings in March and 175% of this amount in savings in April. How much does Liam put in savings in April?

Answer:

whole = 100
4,75 = 100 4,175. 100 = 70100

Lesson 3.3 Represent and Use the Percent Equation

Quick Review
You can use the percent equation to solve percent problems.
part = percent • whole
Substitute two of the three values to solve for the unknown value.

Example
Michael earns a 6% commission on each house he sells. If he sells a house for $180,000, how much does he earn in commission, c?
c = 0.06. 180,000
c = 10,800
Michael earns $10,800 in commission.

Practice
Question 1.
Sharon paid $78 sales tax on a new camera. If the sales tax rate is 6.5%, what was the cost of the camera?

Answer:

Question 2.
There are 45 students who play a woodwind instrument in the school band. Of these, 18 play the saxophone. What percent of these students play the saxophone?
Answer:

Lesson 3.4 Solve Percent Change and Percent Error Problems

Quick Review
A percent change can be an increase or a decrease. You can use an equation to find a percent change.
change = percent change • original amount
A percent error is always a nonnegative value. You can use an equation to find a percent error.
difference = percent error • actual

Example
Juan’s puppy weighed 16 pounds at the age of 2 months. The puppy weighed 60 pounds at the age of 8 months. What is the percent change in the puppy’s weight?
change in pounds = 60 – 16 = 44
44 = P • 16
\(\frac{44}{16}\) = P
2.75 = P
The puppy’s weight increased by 275%.

Practice
Question 1.
In 2014, the attendance at Jefferson School’s Fall Festival was 650. In 2015, the attendance was 575. What was the percent change in attendance from 2014 to 2015? Round to the nearest whole percent.
Answer:

Question 2.
Melissa estimated that she would read 250 pages last week. She read 290 pages. What is the percent error of Melissa’s estimate? Round to the nearest whole percent.
Answer:

Lesson 3.5 Solve Markup and Markdown Problems

Quick Review
You can use the percent equation to solve markup and markdown problems.
markup = percent markup • cost
markdown = percent markdown • selling price

Example
Bree buys a purse for $90. She sells it at her store for $135. What is the percent markup on the purse?
markup = $135 – $90 = $45
45 = P • 90
45 = P
0.5 =P The decimal 0.5 is equivalent to 50%.
The percent markup on the purse is 50%.

Practice
Question 1.
Hank buys a used car for $7,200 and plans to sell it on his used car lot. What is the price of the car after a markup of 15%?

Answer:
The price of the car after a markup of 15% =

Explanation:
In the above-given question,
given that,
Hank buys a used car for $7,200 and plans to sell it on his used car lot.
part = percent x whole.
part = 15 x 7200.
part = 15/100 x 7200.
part = 1080.
so the price of the car after a markup of 15% = 1080.

Question 2.
Nya wants to buy a sweater that had an original price of $55. The sweater is now discounted 20% and the sales tax rate is 5.5%. How much will Nya pay for the sweater?

Answer:
Nya pays for the sweater.

Explanation:
In the above-given question,
given that,
Nya wants to buy a sweater that had an original price of $55.
The sweater is now discounted 20% and the sales tax rate is 5.5%.
$55 x 20/100 x 5.5%.
55 x 0.2 x 55/1000.
55 x 0.2 x 0.055.
0.605.
so Nya pays for the sweater = 0.605.

Lesson 3.6 Solve Simple Interest Problems

Quick Review
You consider four quantities when solving problems involving simple interest.
initial amount, or principal, p
interest rate, r
time, t amount of simple interest, I

Example
Yuni loaned $400 to her brother. He will repay the loan by paying 2.5% simple interest for 3 years. How much will he pay in interest?
I = 400 • 0.025 • 3 = $30
He will pay $30 in interest.

Practice
Question 1.
Ethan put $700 into a Certificate of Deposit (CD) account that earns 1.8% interest each year. What will the interest of the CD account be after 6 years?

Answer:
The interest of the CD account is after 6 years = $75.6.

Explanation:
In the above-given question,
given that,
Ethan put $700 into a Certificate of Deposit (CD) account that earns 1.8% interest each year.
$700 x 1.8% x 6.
$700 x 1.8/100 x 6.
$700 x 0.018 x 6.
$75.6.

Question 2.
Kelly opened a bank account that earns 1.2% simple interest each year. After 7 years, Kelly will earn $126 in interest. How much did Kelly deposit when she opened the account?

Answer:
The kelly deposit when she opened the account = $10.584.

Explanation:
In the above-given question,
given that,
Kelly opened a bank account that earns 1.2% simple interest each year.
After 7 years, Kelly will earn $126 in interest.
$126 x 7 x 1.2%.
$126 x 7 x 1.2/100.
$126 x 7 x 0.012.
$126 x 0.084.
$10.584.
so the kelly deposit when she opened the account = $10.584.

Topic 3 Fluency Practice

Riddle Rearranging
Find the value of x in each unit rate. Then arrange the answers in order from least to greatest. The letters will spell out the answer to the riddle below.
I can… find unit rates with ratios of fractions.
Envision Math Common Core Grade 7 Answers Topic 3 Analyze And Solve Percent Problems 69

Answer:
0.75 xc = 0.3h, 0.4 min = 0.6 ml, and 0.25 mi = 0.625 h.

Explanation:
In the above-given question,
given that,
3/4c/1/3h = xc/1h.
0.75c/0.3h = xc/h.
0.75c x xc = 0.3h x h.
0.75xc = 0.3h.
3/5ml/2/5 min = x ml /1min.
0.6ml/0.4 min = x ml / 1 min.
0.4 x 1 min = x ml x 0.6 ml.
0.4 min = 0.6ml.
1/4mi/5/8 h = x mi / 1h.
0.25 mi/ 0.625 h = x mi / 1h.
x mi /1 h = 0.25 mi/0.625 h.

enVision Math Common Core Grade 7 Answer Key Topic 2 Analyze And Use Proportional Relationships

Practice with the help of enVision Math Common Core Grade 7 Answer Key Topic 2 Analyze and Use Proportional Relationships regularly and improve your accuracy in solving questions.

enVision Math Common Core 7th Grade Answers Key Topic 2 Analyze And Use Proportional Relationships

Topic 2 Essential Question

How can you recognize and represent proportional relationships and use them to solve problems?

3-ACT MATH

Envision Math Common Core 7th Grade Answer Key Topic 2 Analyze And Use Proportional Relationships 1
Mixin’ It Up Drinking plenty of water each day is important. Water is necessary for everything your body does. Not drinking enough water can lead to health problems. It’s even easier to drink enough water if you like the taste. There are many ways to make water more exciting. You can drink seltzer or filtered water. You can add fruit, vegetables, herbs, or flavor enhancers. You can add more or less based on what you like. Think about this during the 3-Act Mathematical Modeling lesson.
Envision Math Common Core 7th Grade Answer Key Topic 2 Analyze And Use Proportional Relationships 2

Topic 2 enVision STEM Project

Envision Math Common Core 7th Grade Answer Key Topic 2 Analyze And Use Proportional Relationships 3

Your Task: An Essential Resource
Access to fresh, clean water is important for human survival. You and your classmates will determine how much fresh water is available on Earth for people to use. You will also explore ways in which people have developed access to clean water.
Envision Math Common Core 7th Grade Answer Key Topic 2 Analyze And Use Proportional Relationships 4

Topic 2 Get Ready!

Review What You Know!

Vocabulary
Choose the best term from the box to complete each definition.

  • complex fraction
  • equivalent ratios
  • rate
  • ratio
  • terms

Question 1.
The quantities x and y in the ratio \(\frac{x}{y}\) are called _________

Answer:
The quantities x and y in the ratio \(\frac{x}{y}\) are called terms.

Explanation:
In the above-given question,
given that,
The quantities x and y in the ratio \(\frac{x}{y}\) are called terms.
for example:
x . y = x/y.

Question 2.
Envision Math Common Core 7th Grade Answer Key Topic 2 Analyze And Use Proportional Relationships 5 are an example of _________

Answer:
2dogs/3 cats and 10 dogs/15 cats are an example of Rate.

Explanation:
In the above-given question,
given that,
2 dogs/3 cats and 10 dogs/15 cats are an example of Rate.
for example:
a x c = b x b.
a x 40 = 15 x 15.
a x 40 = 225.
a = 225/40.

Question 3.
A(n) __________ is a type of ratio that has both terms expressed in different units.

Answer:
A(n) ratio is a type of ratio that has both terms expressed in different units.

Explanation:
In the above-given question,
given that,
A(n) ratio is a type of ratio that has both terms expressed in different units.
for example:
2/5 = 2 : 5

Question 4.
A(n) _________ has a fraction in its numerator, denominator, or both.

Answer:
A(n) complex fraction has a fraction in its numerator, denominator, or both.

Explanation:
In the above-given question,
given that,
A(n) complex fraction has a fraction in its numerator, denominator, or both.
for example:
3/5.
where 3 is a numerator.
5 is a denominator.

Equivalent Ratios

Complete each equivalent ratio.
Question 5.
Envision Math Common Core 7th Grade Answer Key Topic 2 Analyze And Use Proportional Relationships 6

Answer:
The equivalent ratio is 8 boys/14 girls.

Explanation:
In the above-given question,
given that,
4 boys/7 girls = 8 boys/ 14 girls.
4 x 2 = 8.
7 x 2 = 14.
Envision-Math-Common-Core-7th-Grade-Answers-Key-Topic-2-Analyze And Use Proportional Relationships-1

Question 6.
Envision Math Common Core 7th Grade Answer Key Topic 2 Analyze And Use Proportional Relationships 7

Answer:
The equivalent ratio is 4 tires/ 1 car.

Explanation:
In the above-given question,
given that,
16 tires/ 4 cars.
4 x 1 = 4.
4 x 4 = 16.
so the equivalent ratio is 4 tires/1 car.
Envision-Math-Common-Core-7th-Grade-Answers-Key-Topic-2-Analyze And Use Proportional Relationships-2

Question 7.
Envision Math Common Core 7th Grade Answer Key Topic 2 Analyze And Use Proportional Relationships 8

Answer:
The equivalent ratio is 40 correct/50 total.

Explanation:
In the above-given question,
given that,
8 correct/10 total.
8 x 5 = 40.
10 x 5 = 50.
so the equivalent ratio is 40 correct.
Envision-Math-Common-Core-7th-Grade-Answers-Key-Topic-2-Analyze And Use Proportional Relationships-3

Question 8.
Envision Math Common Core 7th Grade Answer Key Topic 2 Analyze And Use Proportional Relationships 9

Answer:
The equivalent ratio is 8 pearls/10 opals.

Explanation:
In the above-given question,
given that,
there are 16 apples and 20 opals.
16 pearls / 20 opals.
2 x 8 = 16.
2 x 10 = 20.
so the equivalent ratio is 10 opals.
Envision-Math-Common-Core-7th-Grade-Answers-Key-Topic-2-Analyze And Use Proportional Relationships-4

Question 9.
Envision Math Common Core 7th Grade Answer Key Topic 2 Analyze And Use Proportional Relationships 10

Answer:
The equivalent ratio is 8 pencils/2 erasers.

Explanation:
In the above-given question,
given that,
32 pencils / 8 erasers.
4 x 8 = 32.
4 x 2 = 8.
32/8 = 8/2.
Envision-Math-Common-Core-7th-Grade-Answers-Key-Topic-2-Analyze And Use Proportional Relationships-5

Question 10.
Envision Math Common Core 7th Grade Answer Key Topic 2 Analyze And Use Proportional Relationships 11

Answer:
The equivalent ratio is 21 balls/ 27 bats.

Explanation:
In the above-given question,
given that,
there are 7 balls and 9 bats.
7 x 3 = 21.
9 x 3 = 27.
so 21/27 is the equivalent ratio.
Envision-Math-Common-Core-7th-Grade-Answers-Key-Topic-2-Analyze And Use Proportional Relationships-6

Rates

Write each situation as a rate.
Question 11.
John travels 150 miles in 3 hours.

Answer:
The rate is 50 miles.

Explanation:
In the above-given question,
given that,
John travels 150 miles in 3 hours.
150 / 3 = 15 miles.
so the rate is 50 miles.

Question 12.
Cameron ate 5 apples in 2 days.

Answer:
The rate is 2.5 apples in a day.

Explanation:
In the above-given question,
given that,
Cameron ate 5 apples in 2 days.
5/2 = 2.5.
so the rate is 2.5 apples in a day.

Equations

Write an equation that represents the pattern in the table.
Question 13.
Envision Math Common Core 7th Grade Answer Key Topic 2 Analyze And Use Proportional Relationships 12

Answer:
The equation is y = 3x.

Explanation:
In the above-given question,
given that,
x = 4 so y = 3 x 4.
x = 5 so y = 5 x 3.
x = 6 so y = 6 x 3.
x = 7 so y = 7 x 3.
x = 8 so y = 8 x 3.

Language Development

Complete the graphic organizer to help you understand new vocabulary terms.
Envision Math Common Core 7th Grade Answer Key Topic 2 Analyze And Use Proportional Relationships 13

Answer:
When two ratios are equivalent is called proportion.
The constant of proportionality k is the constant value of the ratio of two proportional quantities y and x.
y = kx.
A relationship between two varying quantities where the ratio between the two quantities remains constant.

Explanation:
In the above-given question,
given that,
the proportion is when two ratios are equivalent.
characteristics:
when each ratio is simplified they should be equivalent.
when replacing the variable with the value the equation will balance.
example:
3/5 = a/50.
3 . 50 = a x 5.
150 = 5a.
a = 30.
non-example:
4/6 is not equal to 8/9.
example:
y = kx.
non-example:
y = 3x.
constant ratio is a graph that passes through the origin.
the constant ratio of delta y/delta x.
example:
y = 2x.
y = -2x.
non-example:
y = 2x+3.
y = x X x.

Pick A Project

PROJECT 2A
Who do you think would win a race involving different types of animals?
PROJECT: PREDICTRACE RESULTS
Envision Math Common Core 7th Grade Answer Key Topic 2 Analyze And Use Proportional Relationships 14

PROJECT 2B
What would it be like to travel to another planet?
PROJECT: CALCULATE THE WEIGHT OF YOUR PACK
Envision Math Common Core 7th Grade Answer Key Topic 2 Analyze And Use Proportional Relationships 15

PROJECT 2C
What stories can you tell?
PROJECT: WRITE A SHORT STORY
Envision Math Common Core 7th Grade Answer Key Topic 2 Analyze And Use Proportional Relationships 16

PROJECT 2D
If you could play any musical instrument, what would you play? Why?
PROJECT: PLAY MUSIC
Envision Math Common Core 7th Grade Answer Key Topic 2 Analyze And Use Proportional Relationships 17

Lesson 2.1 Connect Ratios, Rates, and Unit Rates

Explain It!
In a basketball contest, Elizabeth made 9 out of 25 free throw attempts. Alex made 8 out of 20 free throw attempts. Janie said that Elizabeth had a better free-throw record because she made more free throws than Alex.
I can… use ratio concepts and reasoning to solve multi-step problems.
Envision Math Common Core 7th Grade Answer Key Topic 2 Analyze And Use Proportional Relationships 18

A. Critique Reasoning Do you agree with Janie’s reasoning? Explain.

Answer:
Yes, I agree with Janie’s reasoning.

Explanation:
In the above-given question,
given that,
In a basketball contest, Elizabeth made 9 out of 25 free throw attempts.
Alex made 8 out of 20 free throw attempts.
Janie said that Elizabeth had a better free-throw record because she made more free throws than Alex.
9/25 = 8/20.
so I agree with Janie’s reasoning.

B. Construct Arguments Decide who had the better free-throw record. Justify your reasoning using mathematical arguments.

Answer:
Both of them had an equal record.

Explanation:
In the above-given question,
given that,
In a basketball contest, Elizabeth made 9 out of 25 free throw attempts.
Alex made 8 out of 20 free throw attempts.
Janie said that Elizabeth had a better free-throw record because she made more free throws than Alex.
9/25 = 8/20.
so both of them had an equal record.

Focus on math practices
Construct Arguments What mathematical model did you use to justify your reasoning? Are there other models you could use to represent the situation?

Answer:
Both of them had an equal record.

Explanation:
In the above-given question,
given that,
In a basketball contest, Elizabeth made 9 out of 25 free throw attempts.
Alex made 8 out of 20 free throw attempts.
Janie said that Elizabeth had a better free-throw record because she made more free throws than Alex.
9/25 = 8/20.
so both of them had an equal record.

Essential Question
How are ratios, rates, and unit rates used to solve problems?

Try It!

Jennifer is a lifeguard at the same pool. She earns $137.25 for 15 hours of lifeguarding. How much does Jennifer earn per hour?
Jennifer earns $ ________ per hour.
Envision Math Common Core 7th Grade Answer Key Topic 2 Analyze And Use Proportional Relationships 19

Convince Me! What do you notice about the models used to find how much each lifeguard earns per hour?

Answer:
Jennifer earns per hour = $9.15

Explanation:
In the above-given question,
given that,
Jennifer is a lifeguard at the same pool.
She earns $137.25 for 15 hours of lifeguarding.
$137.25/15.
$9.15.
so Jennifer earns per hour = $9.15.
Envision-Math-Common-Core-7th-Grade-Answers-Key-Topic-2-Analyze And Use Proportional Relationships-7

Try It!

A kitchen sink faucet streams 0.5 gallon of water in 10 seconds. A bathroom sink faucet streams 0.75 gallon of water in 18 seconds. Which faucet will fill a 3-gallon container faster?

Answer:
The faucet will fill a 3-gallon container faster = bathroom sink.

Explanation:
In the above-given question,
given that,
A kitchen sink faucet streams 0.5 gallons of water in 10 seconds.
A bathroom sink faucet streams 0.75 gallons of water in 18 seconds.
bathroom sink faucet streams 0.75 gallons of water in 18 seconds.
so the faucet will fill a 3-gallon container faster = bathroom sink.

KEY CONCEPT

You can use equivalent ratios and rates, including unit rates, to compare ratios and to solve problems.
Envision Math Common Core 7th Grade Answer Key Topic 2 Analyze And Use Proportional Relationships 20

Do You Understand?
Question 1.
Essential Question How are ratios, rates, and unit rates used to solve problems?

Answer:
Yes, the ratios, rates, and units are used to solve problems.

Explanation:
In the above-given question,
given that,
a rate is a ratio that compares two quantities with different units.
unit rate is a rate that has 1 as the denominator.
for example:
24 pencils cost $1.20, how much will 6 pencils cost?
$1.20/24  = $0.05/1 pencil.
5 cents per pencil.
6 pencils will cost = 6 x 5 = 30.

Question 2.
Use Structure Dorian buys 2 pounds of almonds for $21.98 and 3 pounds of dried apricots for $26.25. Which is less expensive per pound? How much less expensive?
Complete the tables of equivalent ratios to help you solve.
Envision Math Common Core 7th Grade Answer Key Topic 2 Analyze And Use Proportional Relationships 21

Answer:
The less expensive per pound = $2.135.

Explanation:
In the above-given question,
given that,
Dorian buys 2 pounds of almonds for $21.98 and 3 pounds of dried apricots for $26.25.
$21.98/2 = $10.99.
$26.25/2 = $13.125.
$13.125 – $10.99 = $2.135.
so the less expensive per pound = $2.135.
Envision-Math-Common-Core-7th-Grade-Answers-Key-Topic-2-Analyze And Use Proportional Relationships-8

Question 3.
Generalize How are unit rates and equivalent ratios related?

Answer:
The unit rates and equivalent ratios are related.

Explanation:
In the above-given question,
given that,
A unit rate is a special kind of ratio, where the second number, or the denominator, is equal to one.
for example:
unit rates are miles per gallon.
when two ratios are equivalent, the two-unit rates will always be equal.
for example:
10/4 = 2.50.

Do You Know How?
Question 4.
Krystal is comparing two Internet service plans. Plan 1 costs $34.99 per month. Plan 2 costs $134.97 every 3 months. If Krystal plans to stay with one service plan for 1 year, which should she choose? How much will she save?

Answer:
Krystal will choose plan 1. She will save $20.

Explanation:
In the above-given question,
given that,
Krystal is comparing two Internet service plans.
Plan 1 costs $34.99 per month.
Plan 2 costs $134.97 every 3 months.
$34.99 x 3 = 104.97.
$134.97 – $104.97 = $20.
so Krystal will choose plan 1.
she will save $20.

Question 5.
Pam read 126 pages of her summer reading book in 3 hours. Zack read 180 pages of his summer reading book in 4 hours. If they continue to read at the same speeds, will they both finish the 215-page book after 5 total hours of reading? Explain.

Answer:
Yes, they both will finish the 215-page book after 5 total hours of reading.

Explanation:
In the above-given question,
given that,
Pam read 126 pages of her summer reading book in 3 hours.
Zack read 180 pages of his summer reading book in 4 hours.
180 – 126 = 54.
180 + 54 = 234.
so they both will finish the 215-page book after 5 total hours of reading.

Question 6.
Nora and Eli are making homemade spring rolls for a party. Nora can make 8 spring rolls in 10 minutes. Eli can make 10 spring rolls in 12 minutes. If they each make 40 spring rolls, who will finish first?

Answer:
They both will finish at an equal time.

Explanation:
In the above-given question,
given that,
Nora and Eli are making homemade spring rolls for a party.
Nora can make 8 spring rolls in 10 minutes.
Eli can make 10 spring rolls in 12 minutes.
so both of them will finish at an equal time.

Practice & Problem Solving

Leveled Practice In 7-8, complete the tables of equivalent ratios to solve.
Question 7.
After Megan walked 5 miles, her activity tracker had Meghan’s Steps counted 9,780 steps. David’s activity tracker had counted Miles 11,928 steps after he walked 6 miles. Suppose each person’s step covers about the same distance. Who takes more steps to walk 1 mile? How many more steps?
Envision Math Common Core 7th Grade Answer Key Topic 2 Analyze And Use Proportional Relationships 22
_________ takes more steps to walk 1 mile.
_______ – ________ = _________ more steps for 1 mile

Answer:
David takes more steps to walk 1 mile = 32 steps.

Explanation:
In the above-given question,
given that,
After Megan walked 5 miles, her activity tracker had Meghan’s Steps counted 9,780 miles.
David’s activity tracker had counted Miles 11,928 steps after he walked 6 miles.
9780/5 = 1956 miles.
11928/6 = 1988 miles.
1988 – 1956 = 32.
so David takes more steps to walk 1 mile = 32 steps.
Envision-Math-Common-Core-7th-Grade-Answers-Key-Topic-2-Analyze And Use Proportional Relationships-9

Question 8.
A package of 5 pairs of insulated gloves costs $29.45. What is the cost of a single pair of gloves?
Envision Math Common Core 7th Grade Answer Key Topic 2 Analyze And Use Proportional Relationships 23
One pair of gloves costs ________.

Answer:
The single pair of gloves = $5.89.

Explanation:
In the above-given question,
given that,
A package of 5 pairs of insulated gloves costs $29.45.
$29.45/5 = $5.89.
$5.89 x 1 = $5.89.
so the single pair of gloves = $5.89.
Envision-Math-Common-Core-7th-Grade-Answers-Key-Topic-2-Analyze And Use Proportional Relationships-10

Question 9.
Which package has the lowest cost per ounce of rice?
Envision Math Common Core 7th Grade Answer Key Topic 2 Analyze And Use Proportional Relationships 24

Answer:
The lowest cost per ounce of rice = white rice.

Explanation:
In the above-given question,
given that,
the pack of white Rice is 12 ounces = $4.56.
the pack of wheat is 18 ounces = $6.30.
the pack of white rice is 7 ounces = $2.59.
so the lowest cost per ounce of rice = white rice.

Question 10.
A nursery owner buys 5 panes of glass to fix some damage to her greenhouse. The 5 panes cost $14.25. Unfortunately, she breaks 2 more panes while repairing the damage. What is the cost of another 2 panes of glass?

Answer:
The cost of another 2 panes of glass = $5.7.

Explanation:
In the above-given question,
given that,
A nursery owner buys 5 panes of glass to fix some damage to her greenhouse.
The 5 panes cost $14.25.
Unfortunately, she breaks 2 more panes while repairing the damage.
$14.25 / 5 = $2.85.
$2.85 x 2 = 5.7.
so the cost of another 2 panes of glass = $5.7.

Question 11.
Be Precise An arts academy requires there to be 3 teachers for every 75 students and 6 tutors for every 72 students. How many tutors does the academy need if it has 120 students?

Answer:
The tutors do the academy need if it has 120 students = 7 tutors.

Explanation:
In the above-given question,
given that,
An arts academy requires there to be 3 teachers for every 75 students and 6 tutors for every 72 students.
75 + 72 = 147.
3 + 6 = 9.
140/9 = 15.5.
120 / 7 = 17.1.
so the tutors does the academy need if it has 120 students = 7 tutors.

Question 12.
Make Sense and Persevere in large cities, people often take taxis to get from one place to another. What is the cost per mile of a taxi ride? How much is a 47-mile taxi ride?
Envision Math Common Core 7th Grade Answer Key Topic 2 Analyze And Use Proportional Relationships 25

Answer:
The much is a 47-mile taxi ride = 36.30 fares.

Explanation:
In the above-given question,
given that,
people often take taxis to get from one place to another.
for 36 miles in 25.20 fare.
for 47 miles in 36.30 fare.
so the much is a 47-mile taxi ride = 36.30 fares.

Question 13.
The Largo Middle School track team needs new uniforms. The students plan to sell plush toy tigers (the school mascot) for $5. The students find three companies online that sell stuffed mascots.
Envision Math Common Core 7th Grade Answer Key Topic 2 Analyze And Use Proportional Relationships 26
a. Which company has the lowest cost per tiger?

Answer:
The company that has the lowest cost per tiger = company C.

Explanation:
In the above-given question,
given that,
The Largo Middle School track team needs new uniforms.
The students plan to sell plush toy tigers (the school mascot) for 5$.
company A- 12 tigers for $33.24.
company B – 14 tigers for $44.80.
company C – 16 tigers for $41.10.
33.24/12 = 2.77.
44.80/14 = 3.2.
41.10/16 = 2.56.
so the company that has the lowest cost per tiger is company C.

b. If they use that company, how much profit will the students make for each tiger sold?

Answer:
The profit will the students make for each tiger sold = $0.84.

Explanation:
In the above-given question,
given that,
The Largo Middle School track team needs new uniforms.
The students plan to sell plush toy tigers (the school mascot) for 5$.
company A- 12 tigers for $33.24.
company B – 14 tigers for $44.80.
company C – 16 tigers for $41.10.
33.24/12 = 2.77.
44.80/14 = 3.2.
41.10/16 = 2.56.
0.21 x 4 = $0.84.
so the profit will the students make for each tiger sold = $0.84.

Question 14.
A contractor purchases 7 dozen pairs of padded work gloves for $103.32. He incorrectly calculates the unit price at $14.76 per pair.
a. What is the correct unit price?

Answer:
The correct unit price = $723.24.

Explanation:
In the above-given question,
given that,
A contractor purchases 7 dozen pairs of padded work gloves for $103.32.
He incorrectly calculates the unit price at $14.76 per pair.
$103.32 x 7 = $723.24.
so the correct unit price = $723.24.

b. Critique Reasoning What error did the contractor likely make?

Answer:
The contractor likely makes the error if he calculated the unit price wrong.

Explanation:
In the above-given question,
given that,
A contractor purchases 7 dozen pairs of padded work gloves for $103.32.
He incorrectly calculates the unit price at $14.76 per pair.
$103.32 x 7 = $723.24.
so the correct unit price = $723.24.

Question 15.
Higher Order Thinking A warehouse store sells 5.5-ounce cans of tuna in packages of 6. A package of 6 cans costs $9.24. The store also sells 6.5-ounce cans of the same tuna in packages of 3 cans for $4.68. It also sells 3.5-ounce cans in packages of 4 cans for $4.48. Which package has the lowest cost per ounce of tuna?

Answer
The package has the lowest cost per ounce of tuna = 6.5 ounces.

Explanation:
In the above-given question,
given that,
A warehouse store sells 5.5-ounce cans of tuna in packages of 6.
A package of 6 cans costs $9.24.
The store also sells 6.5-ounce cans of the same tuna in packages of 3 cans for $4.68.
It also sells 3.5-ounce cans in packages of 4 cans for $4.48.
the package that has the lowest cost per ounce of tuna = 6.5 ounces.

Assessment Practice

Question 16.
Lena is making two dishes for an event. Each batch of her mac-and-cheese recipe calls for 6 ounces of cheese and 2 tablespoons of basil. For every two pizzas, she needs 16 ounces of cheese and 5 tablespoons of basil.
PART A
Lena buys a 32-oz package of cheese. Does she have enough cheese to make 2 batches of mac-and-cheese and 3 pizzas? Explain.

Answer:
Yes, she has enough cheese to make 2 batches of mac-and-cheese and 3 pizzas.

Explanation:
In the above-given question,
given that,
Lena buys a 32-oz package of cheese.
Does she have enough cheese to make 2 batches of mac-and-cheese and 3 pizzas?
Each batch of her mac-and-cheese recipe calls for 6 ounces of cheese and 2 tablespoons of basil.
6 x 2 = 12.
3 x 2 = 6.
so she has enough cheese to make 2 batches of mac-and-cheese and 3 pizzas.

PART B
How many tablespoons of basil does she need? Explain your answer.

Answer:
The number of tablespoons of basil does she need = 6.

Explanation:
In the above-given question,
given that,
Lena buys a 32-oz package of cheese.
Does she have enough cheese to make 2 batches of mac-and-cheese and 3 pizzas?
Each batch of her mac-and-cheese recipe calls for 6 ounces of cheese and 2 tablespoons of basil.
6 x 2 = 12.
3 x 2 = 6.

Question 17.
It cost Irene $58.90 to fill her car’s gas tank with 15 gallons of gas. Select all the rates that are equivalent to $58.90 for 155 gallons of gas.
☐ $41.80 for 11 gallons of gas
☐ $28.80 for 8 gallons of gas
☐ $26.60 for 7 gallons of gas
☐ $8.55 for 2 gallons of gas
☐ $3.80 for 1 gallon of gas

Answer:
Options A, C, and E are correct.

Explanation:
In the above-given question,
given that,
It cost Irene $58.90 to fill her car’s gas tank with 15 gallons of gas.
$58.90/15 = 3.9.
$41.80/11 = 3.8.
28.80/8 = 3.6.
$26.60/7 = 3.8.
$8.55 /2 = 4.27.
$3.80/1 = 3.8.
so options A,C, and E are correct.

Lesson 2.2 Determine Unit Rates with Ratios of Fractions

Solve & Discuss It!
Allison and her classmates planted bean seeds at the same time as Yuki and her classmates in Tokyo did. Allison is video-chatting with Yuki about their class seedlings. Assume that both plants will continue to grow at the same rate. Who should expect to have the taller plant at the end of the school year?

I can… find unit rates with ratios of fractions and use them to solve problems.
Envision Math Common Core 7th Grade Answer Key Topic 2 Analyze And Use Proportional Relationships 27

Look for Relationships
How can you compare the growth rates of the seedlings?

Focus on math practices
Be Precise What must the students do before they can compare the heights of the plants?

Answer:
The height of Yuki’s class is 5.5 cm in 5 days.

Explanation:
In the above-given question,
given that,
Allison and her classmates planted bean seeds at the same time as Yuki and her classmates in Tokyo did.
the Allison class is 2.5 inches in 5 days.
so the height of Yuki’s class is 5.5 cm in 5 days.

Essential Question
Why is it useful to write a ratio of fractions as a unit rate?

Try It!

Sergio increases his target speed to 30 miles per hour. How many more miles does Sergio need to ride in hour to achieve this target speed?
Envision Math Common Core 7th Grade Answer Key Topic 2 Analyze And Use Proportional Relationships 28
Sergio must ride _______ miles in \(\frac{1}{4}\) hour to achieve this target speed, so he needs to ride an additional ______ mile per \(\frac{1}{4}\) hour.

Answer:
The more miles does Sergio needs to ride in an hour to achieve this target speed = 7.5 miles.

Explanation:
In the above-given question,
given that,
Sergio increases his target speed to 30 miles per hour.
30/4 = 7.5.
so the more does Sergio needs to ride in an hour to achieve this target speed = 7.5 miles.
Envision-Math-Common-Core-7th-Grade-Answers-Key-Topic-2-Analyze And Use Proportional Relationships-11

Convince Me! How does the unit rate describe Sergio’s cycling speed? How is the unit rate helpful in determining how much farther Sergio must cycle in a given amount of time each time he increases his target speed?

Try It!

Every other weekend, Bronwyn’s brother Daniel mows the lawn. He can mow 15,000 ft2 in \(\frac{3}{4}\) hour. Who mows the lawn in less time? Explain.

Answer:
The Bronwyn’s brother Daniel mows the lawn in less time = 20000 ft.

Explanation:
In the above-given question,
given that,
Every other weekend, Bronwyn’s brother Daniel mows the lawn.
He can mow 15,000 ft2 in \(\frac{3}{4}\) hour.
15000/0.75 = 20000.
so the Bronwyn,s brother Daniel mows the lawn in less time = 20000 ft.

Try It!

Sonoma bikes 5 miles to Paige’s house. On a map, they measure that distance as \(\frac{5}{6}\) cm. The same map shows that the mall is 3\(\frac{1}{2}\) cm from Paige’s house. What is the actual distance between Paige’s house and the mall?

Answer:
The actual distance between Paige’s house and the mall = 2.67 cm.

Explanation:
In the above-given question,
given that,
Sonoma bikes 5 miles to Paige’s house.
On a map, they measure that distance as \(\frac{5}{6}\) cm.
The same map shows that the mall is 3\(\frac{1}{2}\) cm from Paige’s house.
3(1/2) = 7/2.
7/2 = 3.5.
5/6 = 0.83.
3.5 – 0.83 = 2.67.
so the actual distance between Paige’s house and the mall = 2.67.

KEY CONCEPT
You can use what you know about equivalent ratios and operations with fractions to write a ratio of fractions as a unit rate.
Tia skateboards \(\frac{2}{3}\) mile for every \(\frac{1}{6}\) hour.
Envision Math Common Core 7th Grade Answer Key Topic 2 Analyze And Use Proportional Relationships 29
She skateboards 4 miles per hour.

Do You Understand?
Question 1.
Essential Question Why is it useful to write a ratio of fractions as a unit rate?

Answer:
The ratio of fractions as a unit rate = 4 miles per hour.

Explanation:
In the above-given question,
given that,
Tia skateboards \(\frac{2}{3}\) mile for every \(\frac{1}{6}\) hour.
2/3/1/6 = 2/3 x 6/1/1/6 x 6/1.
4/1 = 4.

Question 2.
Use Structure Jacob mixes \(\frac{1}{3}\) cup of yellow paint for every \(\frac{1}{5}\) cup of blue paint to make green paint. How many cups of yellow paint are needed for 1 cup of blue paint? Complete the table below.
Envision Math Common Core 7th Grade Answer Key Topic 2 Analyze And Use Proportional Relationships 30

Answer:
The number of cups of yellow paint is needed for 1 cup of blue paint = 3.

Explanation:
In the above-given question,
given that,
Use Structure Jacob mixes \(\frac{1}{3}\) cup of yellow paint for every \(\frac{1}{5}\) cup of blue paint to make green paint.
so the cups of yellow paint are 3.
so the number of cups of yellow paint is needed for 1 cup of blue paint = 3.
1/3 x 3.
1/5 x 1.
so the number of cups of yellow paint is needed for 1 cup of blue paint = 3.
Envision-Math-Common-Core-7th-Grade-Answers-Key-Topic-2-Analyze And Use Proportional Relationships-12

Question 3.
Construct Arguments How is making a table of equivalent ratios to find the unit rate similar to finding the unit rate by calculating with fractions? Use a specific example to explain your reasoning.

Answer:
The unit rates and equivalent ratios are related.

Explanation:
In the above-given question,
given that,
A unit rate is a special kind of ratio, where the second number, or the denominator, is equal to one.
for example:
unit rates are miles per gallon.
when two ratios are equivalent, the two-unit rates will always be equal.
for example:
10/4 = 2.50.

Do You Know How?
Question 4.
Claire boarded an airplane in Richmond, VA, and flew 414 miles directly to Charleston, SC. The total flight time was \(\frac{3}{4}\) hour. How fast did Claire’s airplane fly, in miles per hour?

Answer:
The fast did Claire’s airplane fly, in miles per hour = 552 miles.

Explanation:
In the above-given question,
given that,
Claire boarded an airplane in Richmond, VA, and flew 414 miles directly to Charleston, SC.
The total flight time was \(\frac{3}{4}\) hour.
3/4 = 0.75.
414 / 0.75 = 552.
so the fast did Claire’s airplane fly, in miles per hour = 552 miles.

Question 5.
Brad buys two packages of mushrooms. Which mushrooms cost less per pound? Explain.
Envision Math Common Core 7th Grade Answer Key Topic 2 Analyze And Use Proportional Relationships 31

Answer:
The mushrooms cost less per pound = Cremini.

Explanation:
In the above-given question,
given that,
Brad buys two packages of mushrooms.
Cremini is $11.25 for 2/3 lb.
Chanterelle is $7.99 for 1/2 lb.
$11.25 / 0.6 = 18.75.
$7.99 / 0.5 = 15.98.
so the Cremini cost less pound.

Question 6.
Jed is baking shortbread for a bake sale. The recipe calls for 1\(\frac{1}{4}\) cups of flour and \(\frac{1}{2}\) stick of butter. How many cups of flour will Jed need if he uses 3 sticks of butter?

Answer:
The number of cups of flour will Jed need if he uses 3 sticks of butter = 1.5 cups.

Explanation:
In the above-given question,
given that,
Jed is baking shortbread for a bake sale.
The recipe calls for 1\(\frac{1}{4}\) cups of flour and \(\frac{1}{2}\) stick of butter.
1(1/4) = 5/4.
5/4 = 1.25.
1/2 = 0.5.
3 x 0.5 = 1.5.
so the number of cups of flour will Jed need if he uses 3 sticks of butter = 1.5 cups.

Practice & Problem Solving

Leveled Practice In 7-10, fill in the boxes to find the unit rate.
Question 7.
Envision Math Common Core 7th Grade Answers Topic 2 Analyze And Use Proportional Relationships 32

Answer:
The 6 cups of sugar and 1 cup of butter.

Explanation:
In the above-given question,
given that,
3/4 cups of sugar and 1/8 cups of butter.
3/4/1/8.
3/4 x 8/1.
24/6 = 6/1.
so 6 cups of sugar and 1 cup of butter.
Envision-Math-Common-Core-7th-Grade-Answers-Key-Topic-2-Analyze And Use Proportional Relationships-13

Question 8.
Envision Math Common Core 7th Grade Answers Topic 2 Analyze And Use Proportional Relationships 33

Answer:
The number of miles in an hour = 1.8 miles.

Explanation:
In the above-given question,
given that,
3/5 miles in 1/3 hour.
3/5/1/3.
3/5 x 3/1.
9/5 = 1.8.
so the number of miles in an hour = 1.8 miles.
Envision-Math-Common-Core-7th-Grade-Answers-Key-Topic-2-Analyze And Use Proportional Relationships-14

Question 9.
Envision Math Common Core 7th Grade Answers Topic 2 Analyze And Use Proportional Relationships 34

Answer:
The number of miles per gallon = 21 miles.

Explanation:
In the above-given question,
given that,
7 miles in 1/3 gallons.
7/1/3 = 7/1 x 3/1.
7/3 = 2.33.
so the number of miles per gallon = 21 miles.
Envision-Math-Common-Core-7th-Grade-Answers-Key-Topic-2-Analyze And Use Proportional Relationships-15

Question 10.
Envision Math Common Core 7th Grade Answers Topic 2 Analyze And Use Proportional Relationships 35

Answer:
The number of pages in 1 minute = 6/4 pages.

Explanation:
In the above-given question,
given that,
3/4 pages in 2 minutes.
3/4/2.
3/4/2/1.
3/4 x 2/1.
6/4.
3/2.
so the number of pages in 1 minute = 3/2 pages.
Envision-Math-Common-Core-7th-Grade-Answers-Key-Topic-2-Analyze And Use Proportional Relationships-16

Question 11.
Hadley paddled a canoe \(\frac{2}{3}\) mile in \(\frac{1}{4}\) hour. How fast did Hadley paddle, in miles per hour?

Answer:
The fast did Hadley paddle, in miles per hour = 2.66 miles.

Explanation:
In the above-given question,
given that,
Hadley paddled a canoe \(\frac{2}{3}\) mile in \(\frac{1}{4}\) hour.
2/3/1/4.
2/3 x 4/1.
8/3 = 2.66.
so the fast did Hadley paddle, in miles per hour = 2.66 miles.

Question 12.
A box of cereal states that there are 90 Calories in a \(\frac{3}{4}\)cup serving. How many Calories are there in 4 cups of the cereal?

Answer:
The number of Calories is there in 4 cups of cereal = 30 calories.

Explanation:
In the above-given question,
given that,
A box of cereal states that there are 90 calories in a 3/4 cup serving.
90/3/4.
90/1 x 4/3.
360/3 = 120.
120/4 = 30.
so the number of Calories is there in 4 cups of cereal = 30 calories.

Question 13.
A robot can complete 8 tasks in \(\frac{5}{6}\) hour. Each task takes the same amount of time.
a. How long does it take the robot to complete one task?

Answer:
The long does it take the robot to complete one task = 9.6 minutes.

Explanation:
In the above-given question,
given that,
A robot can complete 8 tasks in \(\frac{5}{6}\) hour.
8/ 5/6.
8/1 x 6/5.
48/5 = 9.6.
so the long does it take the robot to complete one task = 9.6 minutes.

b. How many tasks can the robot complete in one hour?

Answer:
The tasks can complete in one hour = 576 minutes.

Explanation:
In the above-given question,
given that,
A robot can complete 8 tasks in \(\frac{5}{6}\) hour.
8/ 5/6.
8/1 x 6/5.
48/5 = 9.6.
9.6 x 60 = 576.
so the tasks can complete in one hour = 576 minutes.

Question 14.
You are running a fuel economy study. You want to find out which car can travel a greater distance on 1 gallon of gas.
Envision Math Common Core 7th Grade Answers Topic 2 Analyze And Use Proportional Relationships 36
a. What is the gas mileage, in miles per gallon, for the blue car?

Answer:
The gas mileage, in miles per gallon, for the blue car = 0.04 miles.

Explanation:
In the above-given question,
given that,
1(1/2) gallons of gas in 35(1/2).
3/2 / 71/2.
3/2 x 2/71.
6/ 142 = 0.04.
so the gas mileage, in miles per gallon, for the blue car = 0.04 miles.

b. What is the gas mileage, in miles per gallon, for the silver car?

Answer:
The gas mileage, in miles per gallon, for the silver car = 34 miles.

Explanation:
In the above-given question,
given that,
4/5 gallons of gas in 27(1/5) miles.
4/5 / 136/5.
4/5 x 5/136.
4/136 = 34.
so the gas mileage, in miles per gallon, for the silver car = 34 miles.

c. Which car could travel the greater distance on 1 gallon of gas?

Answer:
The silver color car travels a greater distance on 1 gallon of gas.

Explanation:
In the above-given question,
given that,
4/5 gallons of gas in 27(1/5) miles.
4/5 / 136/5.
4/5 x 5/136.
4/136 = 34.
so the gas mileage, in miles per gallon, for the silver car = 34 miles.

Question 15.
Henry incorrectly said the rate \(\frac{\frac{1}{5} \text { pound }}{\frac{1}{20} \text { quart }} \) can be written as the unit rate 100 pound per quart.
a. What is the correct unit rate?

Answer:
The correct unit rate = 4 pounds.

Explanation:
In the above-given question,
given that,
Henry incorrectly said the rate \(\frac{\frac{1}{5} \text { pound }}{\frac{1}{20} \text { quart }} \).
1/5 /1/20.
1/5 x 20/1.
4.
so the correct unit rate = 4 pounds.

b. Critique Reasoning What error did Henry likely make?

Answer:
The error did Henry likely make = 4 pounds.

Explanation:
In the above-given question,
given that,
Henry incorrectly said the rate \(\frac{\frac{1}{5} \text { pound }}{\frac{1}{20} \text { quart }} \).
1/5 /1/20.
1/5 x 20/1.
4.
so the correct unit rate = 4 pounds.

Question 16.
Higher Order Thinking Ari walked 2\(\frac{3}{4}\) miles at a constant speed of 2\(\frac{1}{2}\) miles per hour. Beth walked 1\(\frac{3}{4}\) miles at a constant speed of 1\(\frac{1}{4}\) miles per hour. Cindy walked for 1 hour and 21 minutes at a constant speed of 1\(\frac{1}{8}\) miles per hour. List the three people in order of the times they spent walking from least time to greatest time.

Answer:
The three people in order of the times they spent walking from least time to greatest time = Cindy, Ari, and Beth.

Explanation:
In the above-given question,
given that,
Ari walked 2\(\frac{3}{4}\) miles at a constant speed of 2\(\frac{1}{2}\) miles per hour.
Beth walked 1\(\frac{3}{4}\) miles at a constant speed of 1\(\frac{1}{4}\) miles per hour.
Cindy walked for 1 hour and 21 minutes at a constant speed of 1\(\frac{1}{8}\) miles per hour.
2(3/4) / 2(1/2) = 11/4 / 5/2.
11/4 x 2/5 = 22/20 = 11/10.
1(3/4) / 1(1/4) = 7/4 / 5/4.
7/4 x 4/5 = 28/20 = 7/5.
1/ 21 / 9/8 = 1/21 x 8/9 = 8/189 = 0.04.
so the constant speed from least to great is Cindly, Ari, and Beth.

Assessment Practice

Question 17.
A blueprint shows a house with two fences. Fence A is 1\(\frac{4}{5}\) inches long on the blueprint and is to be 1\(\frac{1}{2}\) feet long. How long is Fence B on the blueprint?
Envision Math Common Core 7th Grade Answers Topic 2 Analyze And Use Proportional Relationships 37

Answer:
The long is Fence B on the blueprint = 2 inches.

Explanation:
In the above-given question,
given that,
A blueprint shows a house with two fences.
Fence A is 1\(\frac{4}{5}\) inches long on the blueprint and is to be 1\(\frac{1}{2}\) feet long.
1(4/5) / 1(1/2).
9/5 / 3/2.
9/5 x 2/3 = 18/15 = 6/3.
so the long is Fence B on the blueprint = 2 inches.

Question 18.
Leo reads 13 pages in \(\frac{1}{3}\) hour. Use the table to find how many pages he reads in one hour.
Envision Math Common Core 7th Grade Answers Topic 2 Analyze And Use Proportional Relationships 38
Leo reads _________ pages in one hour.

Answer:
The number of pages he reads in one hour = 39 pages.

Explanation:
In the above-given question,
given that,
Leo reads 13 pages in \(\frac{1}{3}\) hour.
13 /1/3.
13/1 x 3 /1 = 39 /1.
so the number of pages he reads in one hour = 39 pages.
Envision-Math-Common-Core-7th-Grade-Answers-Key-Topic-2-Analyze And Use Proportional Relationships-17

Lesson 2.3 Understand Proportional Relationships: Equivalent Ratios

Solve & Discuss It!
Weight is a measure of force affected by gravity. The Moon’s gravity is less than Earth’s gravity, so objects weigh less on the Moon than on Earth. Using the information provided, how much do you think a cat will weigh on the Moon? Explain your reasoning.

I can… test for equivalent ratios to decide whether quantities are in a proportional relationship.
Envision Math Common Core 7th Grade Answers Topic 2 Analyze And Use Proportional Relationships 39

Make Sense and Persevere
About how much does a cat weigh on Earth?

Focus on math practices
Generalize How could you find the approximate weight of any object on the Moon? Explain your reasoning.

Essential Question
How are proportional quantities described by equivalent ratios?

Try It!

Miles records the time it takes to download a variety of file types. How is the download time related to the file size? Explain.
Envision Math Common Core 7th Grade Answers Topic 2 Analyze And Use Proportional Relationships 40
The ratios for each pair of data are ___________, so the download time and the file size are __________.

Answer:
The ratios for each pair of data are the same.
so the download time and the file size are different.

Explanation:
In the above-given question,
given that,
Miles records the time it takes to download a variety of file types.
downloaded time / file size = 25/1.25 = 20.
72/3.6 = 20.
125/6.25 = 20.
so the ratios for each pair of data are the same.
so the download time and the file size are different.
Envision-Math-Common-Core-7th-Grade-Answers-Key-Topic-2-Analyze And Use Proportional Relationships-18

Convince Me!
How can you show that two quantities have a proportional relationship?

Try It!

The table at the right shows information about regular hexagons. Is the relationship between the perimeter and the side length of the hexagons proportional? Explain.
Envision Math Common Core 7th Grade Answers Topic 2 Analyze And Use Proportional Relationships 41
Answer:

Try It!

Ginny’s favorite cookie recipe requires 1\(\frac{1}{3}\) cups of sugar to make 24 cookies. How much sugar does Ginny need to make 36 of these cookies?
Answer:

KEY CONCEPT
Two quantities x and y have a proportional relationship if all the ratios \(\frac{y}{x}\) for related pairs of x and y are equivalent.
A proportion is an equation that states that two ratios are equivalent
Envision Math Common Core 7th Grade Answers Topic 2 Analyze And Use Proportional Relationships 42
\(\frac{3}{120}\) = \(\frac{5}{200}\)

Do You Understand?
Question 1.
Essential Question How are proportional quantities described by equivalent ratios?

Answer:
Two quantities x and y have a proportional relationship if all the ratios y/x for related pairs of x and y are equivalent.

Explanation:
In the above-given question,
given that,
x = 120 and y = 3.
y/x = 120/3 = 0.025.
x = 200 and y = 5.
y/x = 200/5 = 0.025.
A proportion is an equation that states that two ratios are equivalent.

Question 2.
Look for Relationships How do you know if a relationship between two quantities is NOT proportional?

Answer:
Two quantities x and y have a proportional relationship if all the ratios y/x for related pairs of x and y are equivalent.

Explanation:
In the above-given question,
given that,
x = 120 and y = 3.
y/x = 120/3 = 0.025.
x = 200 and y = 5.
y/x = 200/5 = 0.025.
A proportion is an equation that states that two ratios are equivalent.

Question 3.
Reasoning If the ratio \(\frac{y}{x}\) is the same for all related pairs of x and y, what does that mean about the relationship between x and y?

Answer:
Two quantities x and y have a proportional relationship if all the ratios y/x for related pairs of x and y are equivalent.

Explanation:
In the above-given question,
given that,
x = 120 and y = 3.
y/x = 120/3 = 0.025.
x = 200 and y = 5.
y/x = 200/5 = 0.025.
A proportion is an equation that states that two ratios are equivalent.

Do You Know How?
Question 4.
Use the table below. Do x and y have a proportional relationship? Explain.
Envision Math Common Core 7th Grade Answers Topic 2 Analyze And Use Proportional Relationships 43

Answer:
Yes, they have a proportional relationship.

Explanation:
In the above-given question,
given that,
x = 2, 3, 5, and 8.
y = 5, 7.5, 12.5, and 18.
y/x = 5/2 = 2.5.
y/x = 7.5/3 = 2.5.
y/x = 12.5/5 = 2.5.
y/x = 18/8 = 2.25.
so x and y have a proportioal relationship.

Question 5.
Each triangle is equilateral. Is the relationship between the perimeter and the side length of the equilateral triangles proportional? Explain.
Envision Math Common Core 7th Grade Answers Topic 2 Analyze And Use Proportional Relationships 44

Answer:
Yes, the relationship between the perimeter and the side length of the equilateral triangles are proportional.

Explanation:
In the above-given question,
given that,
there are 3 triangles.
they are 1 in, 2 in, and 3 in.
so the sides of all the sides are equal in every triangle.
so the relationship between the perimeter and the side length of the equilateral triangles are proportional.

Question 6.
Is the relationship between the number of tickets sold and the number of hours proportional? If so, how many tickets were sold in 8 hours?
Envision Math Common Core 7th Grade Answers Topic 2 Analyze And Use Proportional Relationships 45

Answer:
The number of tickets was sold in 8 hours = 640 tickets.

Explanation:
In the above-given question,
given that,
the number of tickets sold in the hours.
h = 3, 5, 9.
t = 240, 400, and 720.
t/h = 240/3 = 80.
t/h =  400/5 = 80.
t/h = 720/9 = 80.
80 x 8 = 640.
so the number of tickets was sold in 8 hours = 640 tickets.

Practice & Problem Solving

Question 7.
The amount of seed a landscaper uses and the area of lawn covered have a proportional relationship. Complete the table.
Envision Math Common Core 7th Grade Answers Topic 2 Analyze And Use Proportional Relationships 46

Answer:
Yes, they have a proportional relationship.

Explanation:
In the above-given question,
given that,
the amount of seed a landscaper uses and the area of lawn covered have a proportional relationship.
the area covered is 50, 75, and 100 sq ft.
the seeds are 2, 3, and 4.
area covered/seed = 50/2 = 25/1.
75/3 = 25/1.
100/4 = 25/1.
so they both have a proportional relationship.
Envision-Math-Common-Core-7th-Grade-Answers-Key-Topic-2-Analyze And Use Proportional Relationships-19

Question 8.
Construct Arguments is the relationship between the number of slices of salami in a sandwich and the number of Calories proportional? Explain.
Envision Math Common Core 7th Grade Answers Topic 2 Analyze And Use Proportional Relationships 47

Answer:
No, they don’t have a proportional relationship.

Explanation:
In the above-given question,
given that,
the calories in a sandwich.
slices of salami in calories are given.
66/1 = 66.
96/2 = 48.
126/3 = 42.
156/4 = 39.
so they did not have a proportional relationship.

Question 9.
Look for Relationships A wholesale club sells eggs by the dozen. Does the table show a proportional relationship between the number of dozens of eggs and the cost? Explain.
Envision Math Common Core 7th Grade Answers Topic 2 Analyze And Use Proportional Relationships 48

Answer:
The number of dozens of eggs and the cost are proportional.

Explanation:
In the above-given question,
given that,
the cost of dozens of eggs.
cost in dollars is 21, 28, 35, and 49.
dozens are 6, 8, 10, and 14.
cost/dozen = 21/6 =3.5.
28/8 = 3.5
35/10 = 3.5
49/14 = 3.5.
so the number of dozens of eggs and the cost are proportional.

Question 10.
Does the table show a proportional relationship? If so, what is the value of y when x is 11?
Envision Math Common Core 7th Grade Answers Topic 2 Analyze And Use Proportional Relationships 49

Answer:
No, they don’t show a proportional relationship.

Explanation:
In the above-given question,
given that,
the values of x and y are given.
y = 64/4 = 16.
y = 125/5 = 25.
y = 216/6 = 36.
y = 1000/10 = 100.
y =
so they don’t have a proportional relationship.

Question 11.
Does the table show a proportional relationship? If so, what is the value of y when x is 10?
Envision Math Common Core 7th Grade Answers Topic 2 Analyze And Use Proportional Relationships 50

Answer:
The value of y when x is 10 is 3.2.

Explanation:
In the above-given question,
given that,
the value of y is 1(2/3), 2, 2(1/3), and 2(2/3).
the value of x = 5, 6, 7, and 8.
y/x = 1.6/5 = 0.32.
y/x = 2/6 = 0.33.
y/x = 2.3/7 = 0.32.
y/x = 2.6/8 = 0.32.
y/x = 3.2/10 = 0.32.
so the value of y when x is 10 is 3.2.

Question 12.
The height of a building is proportional to the number of floors. The figure shows the height of a building with 9 floors.
Envision Math Common Core 7th Grade Answers Topic 2 Analyze And Use Proportional Relationships 51
a. Reasoning Write the ratio of height of the building to the number of floors. Then find the unit rate, and explain what it means in this situation.

Answer:
The unit rate = 15.

Explanation:
In the above-given question,
given that,
The height of a building is proportional to the number of floors.
the number of floors = 9.
the height of the building = 135 feet.
135/9 = 15.
so the unit rate is 15.

b. How tall would the building be if it had 15 floors?

Answer:
The height of the building is 9.

Explanation:
In the above-given question,
given that,
The height of a building is proportional to the number of floors.
the number of floors = 15.
the height of the building = 135 feet.
135/15 = 9.
so the height of the building is 9.

Question 13.
Higher Order Thinking Do the two tables show the same proportional relationship between x and y? Explain how you know.
Envision Math Common Core 7th Grade Answers Topic 2 Analyze And Use Proportional Relationships 52

Answer:
Yes, both the tables have the proportional relationship.

Explanation:
In the above-given question,
given that,
the two tables show the same proportional relationship between x and y.
the values of x = 160, 500, and 1200.
the values of y = 360, 1125, and 2700.
y/x = 360/160 = 2.25.
y/x = 1125/500 = 2.25.
y/x = 2500/1200 = 2.08.
y/x = 4.5/2 = 2.25.
y/x = 11.25/5 = 2.25.
y/x = 15.75/7 = 2.25.
so both the tables have a proportional relationship.

Assessment Practice

Question 14.
The table shows the number of cell phone towers a company will build as the number of its customers increases.
Envision Math Common Core 7th Grade Answers Topic 2 Analyze And Use Proportional Relationships 53
PART A
Is the relationship between the number of towers and the number of customers proportional? Explain.

Answer:
Yes, both of them have a proportional relationship.

Explanation:
In the above-given question,
given that,
the number of cell phone towers a company will build as the number of its customer’s increases.
towers are 252, 300, 348, and 444.
customers(thousands) = 5.25, 6.25, 7.25, and 9.25.
252/5.25 = 48.
300/6.25 = 48.
348/7.25 = 48.
444/9.25 = 48.
so both of them have a proportional relationship.

PART B
If there are 576 towers, how many customers does the company have? Write a proportion you can use to solve.

Answer:
The number of customers does the company has = 56.1.

Explanation:
In the above-given question,
given that,
if there are 576 towers.
the number of customers is 10.25.
576 / 10.25 = 56.1.
so the number of customers does the company have = 56.1.

Question 15.
Select all true statements about the table at the right.
Envision Math Common Core 7th Grade Answers Topic 2 Analyze And Use Proportional Relationships 54
☐ The table shows a proportional relationship.
☐ When x is 20, y is 2.55.
☐ All the ratios \(\frac{y}{x}\) for related pairs of x and y are equivalent to 8.
☐ The unit rate of \(\frac{y}{x}\) for related pairs of x and y is \(\frac{1}{8}\).
☐ If the table continued, when x is 30, y would be 3.75.

Answer:
Options A, B, and E are correct.

Explanation:
In the above-given question,
given that,
the values of x and y are shown.
y/x = 1.5/12 = 0.125.
y/x = 2.25/18 = 0.125.
y/x = 2.75/22 = 0.125.
y/x = 3.25/26 = 0.125.
so options A, B, and E are correct.

Lesson 2.4 Describe Proportional Relationships: Constant of Proportionality

Solve & Discuss It!
Jamal can run 1 mile in 5.05 minutes. If Jamal maintains this pace during a 5-kilometer (5K) race, he expects to break the course record of 15.25 minutes. Is Jamal’s expectation reasonable? Explain.

Answer:
Yes, Jamal’s expectation is reasonable.

Explanation:
In the above-given question,
given that,
Jamal can run 1 mile in 5.05 minutes.
If Jamal maintains this pace during a 5-kilometer (5K) race, he expects to break the course record of 15.25 minutes.
15.25/5 = 3.05.
so Jamal’s expectation is reasonable.

I can… use the constant of proportionality in an equation to represent a proportional relationship
Envision Math Common Core 7th Grade Answers Topic 2 Analyze And Use Proportional Relationships 55

Be Precise How can you convert 5 kilometers to miles?

Focus on math practices
Reasoning Assuming that Jamal runs at a constant rate, how does his pace describe the time it takes him to finish a race of any length?

Essential Question
How can you represent a proportional relationship with an equation?

Answer:
Yes, Jamal’s expectation is reasonable.

Explanation:
In the above-given question,
given that,
Jamal can run 1 mile in 5.05 minutes.
If Jamal maintains this pace during a 5-kilometer (5K) race, he expects to break the course record of 15.25 minutes.
15.25/5 = 3.05.
so Jamal’s expectation is reasonable.

Try It!

Maria made two batches of fruit punch. The table at the right shows how many quarts of juice she used for each batch. Write an equation that relates the proportional quantities.
Envision Math Common Core 7th Grade Answers Topic 2 Analyze And Use Proportional Relationships 56
The constant of proportionality is __________.
An equation that represents this proportional relationship is y = ________ x.

Answer:
The constant of proportionality is 1.6.
An equation that represents this proportional relationship is y = kx.

Explanation:
In the above-given question,
given that,
Maria made two batches of fruit punch.
the grape juice contains 8 and 16.
apple juice contains 5 and 10.
y/x = 8/5 = 1.6.
y/x = 16/10 = 1.6.
an equation that represents this proportional relationship is y = kx.
Envision-Math-Common-Core-7th-Grade-Answers-Key-Topic-2-Analyze And Use Proportional Relationships-20

Convince Me! How does the equation change if the amount of grape juice is the independent variable, x, and the amount of apple juice is the dependent variable, y?

Answer:
An equation that represents this proportional relationship is y = kx.

Explanation:
In the above-given question,
given that,
Maria made two batches of fruit punch.
the grape juice contains 8 and 16.
apple juice contains 5 and 10.
y/x = 8/5 = 1.6.
y/x = 16/10 = 1.6.
an equation that represents this proportional relationship is y = kx.

Try It!

A florist sells a dozen roses for $35.40. She sells individual roses for the same unit cost. Write an equation to represent the relationship between the number of roses, x, and the total cost of the roses, y. How much would 18 roses cost?

Answer:
The cost of 18 roses is $1.96.

Explanation:
In the above-given question,
given that,
A florist sells a dozen roses for $35.40.
She sells individual roses for the same unit cost.
$35.40/18 = 1.96.
so the cost of 18 roses is $1.96.

Try It!

Balloon A is released 5 feet above the ground. Balloon B is released at ground level. Both balloons rise at a constant rate.
Envision Math Common Core 7th Grade Answers Topic 2 Analyze And Use Proportional Relationships 57
Which situation can you represent using an equation of the form y = kx? Explain.

Answer:
Balloon A does not have a proportional relationship.
Balloon B has a proportional relationship.

Explanation:
In the above-given question,
given that,
Balloon A is released 5 feet above the ground.
Balloon B is released at ground level.
y = kx.
k = y/x.
k = 9/1 = 9.
k = 13/2 = 6.5.
k = 17/3 = 5.6.
k = 4/1 = 4.
k = 8/2 = 4.
k = 12/3 = 4.

KEY CONCEPT
Two proportional quantities x and y are related by a constant multiple, or the constant of proportionality, k.
You can represent a proportional relationship using the equation y = kx.

Do You Understand?
Question 1.
Essential Question How can you represent a proportional relationship with an equation?

Answer:
Two proportional quantities x and y are related by a constant multiple, or the constant of proportionality, K.

Explanation:
In the above-given question,
given that,
Two proportional quantities x and y are related by a constant multiple, or the constant of proportionality, k.
You can represent a proportional relationship using the equation y = kx.

Question 2.
Generalize How can you use an equation to find an unknown value in a proportional relationship?

Answer:
The unknown value in a proportional relationship is y = kx.

Explanation:
In the above-given question,
given that,
Two proportional quantities x and y are related by a constant multiple, or the constant of proportionality, k.
You can represent a proportional relationship using the equation y = kx.

Question 3.
Reasoning Why does the equation y = 3x + 5 NOT represent a proportional relationship?

Answer:
The equation y = 3x + 5 not represent a proportional relationship.

Explanation:
In the above-given question,
given that,
the equation y = 3x + 5.
where k = 3.
so the equation does not represent a proportional relationship.

Do You Know How?
Question 4.
Determine whether each equation represents a proportional relationship. If it does, identify the constant of proportionality.
a. y = 0.5x – 2

Answer:
The constant of proportionality = 0.5.

Explanation:
In the above-given question,
given that,
the equation is y = 0.5x – 2.
the constant of proportionality = 0.5.

b. y = 1,000x

Answer:
The constant of proportionality = 1000.

Explanation:
In the above-given question,
given that,
the equation is y = kx.
where k = 1000.
so the constant of proportionality = 1000.

c. y = x + 1

Answer:
The constant of proportionality = x.

Explanation:
In the above-given question,
given that,
the equation is y = kx.
where k = x.
so the constant of proportionality = x.

Question 5.
The manager of a concession stand estimates that she needs 3 hot dogs for every 5 people who attend a baseball game. If 1,200 people attend the game, how many hot dogs should the manager order?

Answer:
The number of hot dogs should the manager order = 80.

Explanation:
In the above-given question,
given that,
The manager of a concession stand estimates that she needs 3 hot dogs for every 5 people who attend a baseball game.
y = kx.
y = 5 x 3.
y = 15.
1200 / 15 = 80.
so the number of hot dogs should the manager order = 80.

Question 6.
A half dozen cupcakes cost $15. What constant of proportionality relates the number of cupcakes and total cost? Write an equation that represents this relationship.

Answer:
The equation represents the relationship = $15.

Explanation:
In the above-given question,
given that,
A half dozen cupcakes cost $15.
the constant of proportionality relates to the number of cupcakes and the total cost is y = kx.
y = 15x.
so the equation represents the relationship = $15.

Practice & Problem Solving

Question 7.
What is the constant of proportionality in the equation y = 5x?

Answer:
The constant of proportionality in the equation is 5.

Explanation:
In the above-given question,
given that,
the constant of proportionality in the equation is 5.
y = 5x.

Question 8.
What is the constant of proportionality in the equation y = 0.41x?

Answer:
The constant of proportionality in the equation y = 0.41x.

Explanation:
In the above-given question,
given that,
the equation is y = 0.41x.
k = 0.41.
so the constant of proportionality in the equation y = 0.41x.

Question 9.
The equation P= 3s represents the perimeter P of an equilateral triangle with side length s. Is there a proportional relationship between the perimeter and the side length of an equilateral triangle? Explain.

Answer:
Yes, there is a proportional relationship between the perimeter and the side length of an equilateral triangle.

Explanation:
In the above-given question,
given that,
The equation P= 3s represents the perimeter P of an equilateral triangle with side length s.
the triangle has 3 sides and all the sides has equal length.
p = 3 x s.
so there is a proportional relationship between the perimeter and the side length of an equilateral triangle.

Question 10.
Model with Math In a chemical compound, there are 3 parts zinc for every 16 parts copper, by mass. A piece of the compound contains 320 grams of copper. Write and solve an equation to determine the amount of zinc in the chemical compound.

Answer:
The equation to solve the amount of zinc in the chemical compound is y = 960.

Explanation:
In the above-given question,
given that,
In a chemical compound, there are 3 parts zinc for every 16 parts copper, by mass.
A piece of the compound contains 320 grams of copper.
y = kx.
y = 3 x 320.
y = 960.
so the equation to solve the amount of zinc in the chemical compound is y = 960.

Question 11.
The weight of 3 eggs is shown. Assuming the three eggs are all the same weight, find the constant of proportionality.
Envision Math Common Core Grade 7 Answer Key Topic 2 Analyze And Use Proportional Relationships 58

Answer:
The constant of proportionality = 40.

Explanation:
In the above-given question,
given that,
The weight of 3 eggs is shown.
the constant of proportionality is:
120/3 = 40.
so the constant of proportionality = 40.

Question 12.
The height of a stack of DVD cases is proportional to the number of cases in the stack. The height of 6 DVD cases is 114 mm.
a. Write an equation that relates the height, y, of a stack of DVD cases and the number of cases, x, in the stack.

Answer:
The equation that relates the height is 19 mm.

Explanation:
In the above-given question,
given that,
The height of a stack of DVD cases is proportional to the number of cases in the stack.
The height of 6 DVD cases is 114 mm.
114/6 = 19.
so the equation that relates the height is 19.

b. What would be the height of 13 DVD cases?

Answer:
The height of 13 DVD cases = 8.769 mm.

Explanation:
In the above-given question,
given that,
The height of a stack of DVD cases is proportional to the number of cases in the stack.
The height of 13 DVD cases is 114 mm.
114/13 = 8.769.
so the height of 13 DVD cases = 8.769 mm.

Question 13.
Ann’s car can travel 228 miles on 6 gallons of gas.
a. Write an equation to represent the distance, y, in miles Ann’s car can travel on x gallons of gas.

Answer:
The equation to represent the distance is 38.

Explanation:
In the above-given question,
given that,
Ann’s car can travel 228 miles on 6 gallons of gas.
y = kx.
228 = k . 6.
k = 228/6.
k = 38.
so the equation to represent the distance is 38.

b. Ann’s car used 7 gallons of gas during a trip. How far did Ann drive?

Answer:
The far did Ann drive = 32.57.

Explanation:
In the above-given question,
given that,
Ann’s car used 7 gallons of gas during a trip.
y = kx.
228/7 = 32.57.
so the far did Ann drive = 32.57.

Question 14.
The value of a baseball player’s rookie card began to increase once the player retired in 1996. The value has increased by $2.52 each year since then.
Envision Math Common Core Grade 7 Answer Key Topic 2 Analyze And Use Proportional Relationships 59
a. How much was the baseball card worth in 1997? In 1998? In 1999?

Answer:
The baseball card was worth in 1997, 1998, and 1999 is $9.98, $12.50, and $15.02.

Explanation:
In the above-given question,
given that,
The value of a baseball player’s rookie card began to increase once the player retired in 1996.
The value has increased by $2.52 each year since then.
the value is $7.46 in 1996.
$7.46 + $2.52 = $9.98 in 1997.
$9.98 + $2.52 = $12.50 in 1998.
$12.50 + $2.52 = $15.02 in 1999.

b. Construct Arguments Why is there not a proportional relationship between the years since the player retired and the card value? Explain.

Answer:
The value has increased by $2.52 each year.

Explanation:
In the above-given question,
given that,
The value of a baseball player’s rookie card began to increase once the player retired in 1996.
The value has increased by $2.52 each year since then.
the value is $7.46 in 1996.
$7.46 + $2.52 = $9.98 in 1997.
$9.98 + $2.52 = $12.50 in 1998.
$12.50 + $2.52 = $15.02 in 1999.
so there is no proportional relationship.

Question 15.
Higher Order Thinking A car travels 2\(\frac{1}{3}\) miles in 3\(\frac{1}{2}\) minutes at a constant speed.
a. Write an equation to represent the distance the car travels, d, in miles for m minutes.

Answer:
The equation to represent the distance the car travels = 0.6.

Explanation:
In the above-given question,
given that,
A car travels 2\(\frac{1}{3}\) miles in 3\(\frac{1}{2}\) minutes at a constant speed.
2(1/3) in 3(1/2).
7/3 in 7/2.
y = kx.
d = k m.
k = d/m.
k = 7/3 / 7/2.
k = 7/3 x 2/7.
k = 14/21.
k  = 0.6.

b. Write an equation to represent the distance the car travels, d, in miles for h hours.

Answer:
The equation to represent the distance the car travels, d, in miles for h hours is k = d/m.

Explanation:
In the above-given question,
given that,
A car travels 2\(\frac{1}{3}\) miles in 3\(\frac{1}{2}\) minutes at a constant speed.
2(1/3) in 3(1/2).
7/3 in 7/2.
y = kx.
d = k m.
k = d/m.

Assessment Practice

Question 16.
For every ten sheets of stickers you buy at a craft store, the total cost increases $20.50.
Envision Math Common Core Grade 7 Answer Key Topic 2 Analyze And Use Proportional Relationships 60
An equation that relates the number of sheets purchased, x, and the total cost, y, of the stickers is y = _______x.
Use the equation you wrote to complete the table.

Answer:
The number of sheets purchased, x, and the total cost, y, of the stickers is y = kx.

Explanation:
In the above-given question,
given that,
For every ten sheets of stickers, you buy at a craft store, the total cost increases by $20.50.
y = kx.
y = k 2.
$20.50/2 = k.
k = 10.25.
$10.25/7 = 1.46.
$26.65/ 12 = 2.22.
Envision-Math-Common-Core-7th-Grade-Answers-Key-Topic-2-Analyze And Use Proportional Relationships-21

Question 17.
600,000 gallons of water pass through a given point along a river every minute. Which equation represents the amount of water, y, that passes through the point in x minutes?
A. x = 10,000y
B. y = 600,000x
C. y = 10,000x
D. y = 600,000 + x

Answer:
Option B is correct.

Explanation:
In the above-given question,
given that,
The amount of water, y, that passes through the point in x minutes.
y = kx.
where k = 600000.
y = 600000x.
so option B is correct.

Topic 2 Mid-Topic Checkpoint

Question 1.
Vocabulary How do you know if a situation represents a proportional relationship? Lesson 2-3

Answer:
Two proportional quantities x and y are related by a constant multiple, or the constant of proportionality, K.

Explanation:
In the above-given question,
given that,
Two proportional quantities x and y are related by a constant multiple, or the constant of proportionality, k.
You can represent a proportional relationship using the equation y = kx.

Question 2.
Ana runs \(\frac{3}{4}\) mile in 6 minutes. Assuming she runs at a constant rate, what is her speed, in miles per hour? Lessons 2-1 and 2-2
Envision Math Common Core Grade 7 Answer Key Topic 2 Analyze And Use Proportional Relationships 61

Answer:
The speed in miles per hour = 8.

Explanation:
In the above-given question,
given that,
Ana runs \(\frac{3}{4}\) mile in 6 minutes.
y = kx.
6 = kx 0.75.
k = 6/0.75.
k = 8.
so the speed in miles per hour = 8.
Envision-Math-Common-Core-7th-Grade-Answers-Key-Topic-2-Analyze And Use Proportional Relationships-22

Question 3.
A trail mix recipe includes granola, oats, and almonds. There are 135 Calories in a \(\frac{1}{4}\) cup serving of the trail mix. How many Calories are in 2 cups of trail mix? Lesson 2-2

Answer:
The number of calories is in 2 cups of trail mix = 1080 calories.

Explanation:
In the above-given question,
given that,
A trail mix recipe includes granola, oats, and almonds.
There are 135 Calories in a \(\frac{1}{4}\) cup serving of the trail mix.
135 / 1/4.
135 x 4 = 540.
540 x 2 =
so the number of calories are in 2 cups of trail mix = 1080 calories.

Question 4.
Does this table show a proportional relationship? If so, what is the constant of proportionality? Lessons 2-3 and 2-4
Envision Math Common Core Grade 7 Answer Key Topic 2 Analyze And Use Proportional Relationships 62
A. Yes; \(\frac{1}{5}\)
B. Yes; 5
C. Yes; 10
D. The quantities are not proportional.

Answer:
Option B is correct.

Explanation:
In the above-given question,
given that,
40/8 = 5.
50/10 = 5.
60/12 = 5.
70/14 = 5.
so option B is correct.

Question 5.
Janet, Rosi, and Tanya buy postcards from their favorite souvenir shop. Janet buys 3 postcards for $1.05. Rosi buys 5 postcards for $1.75. Tanya buys 8 postcards for $2.80. Are the cost, y, in dollars and the number of postcards, x, proportional? Explain your answer. Write an equation to represent this relationship, if possible. Lesson 2-4

Answer:
Yes, they are in a proportional relationship.

Explanation:
In the above-given question,
given that,
Janet, Rosi, and Tanya buy postcards from their favorite souvenir shop.
Janet buys 3 postcards for $1.05. Rosi buys 5 postcards for $1.75. Tanya buys 8 postcards for $2.80.
1.05/3 = 0.35.
1.75/5 = 0.35.
2.80/8 = 0.35.
so they are in a proportional relationship.

Question 6.
Four movie tickets cost $30.00. Five concert tickets cost $36.50. Does the movie or the concert cost less per ticket? How much less? Lesson 2-1

Answer:
The less per ticket = $6.5.

Explanation:
In the above-given question,
given that,
Four movie tickets cost $30.00.
Five concert tickets cost $36.50.
$36.50 – $30.00 = 6.5.
so the cost is less per ticket = $6.5.

Topic 2 Mid-Topic Performance Task

The school’s theater club is building sets that will make ordinary students look like giants. The actors need a door, a table, and a stool that will make them look almost twice as tall.
PART A
The heights of objects in the set are proportional to the actual heights of objects. Complete the table.
Envision Math Common Core Grade 7 Answer Key Topic 2 Analyze And Use Proportional Relationships 63
The constant of proportionality is _________.

Answer:

PART B
How can the stage manager use the constant of proportionality to find the dimensions of any new props the director requires?
Answer:

PART C
What should the height of a can of fruit juice used as a prop be if its actual height is 7 inches? Show your work.
Answer:

3-ACT MATH

3-Act Mathematical Modeling: Mixin’ It Up
Envision Math Common Core Grade 7 Answer Key Topic 2 Analyze And Use Proportional Relationships 64

ACT 1
Question 1.
After watching the video, what is the first question that comes to mind?

Answer:
When we are drinking first we have to shake it and we have to drink.

Explanation:
In the above-given question,
given that,
Mixin ti up.
that means when we are drinking we have to shake it and we have to use it.
either it is a syrup or any other drink.

Question 2.
Write the Main Question you will answer.

Answer:
When we are drinking first we have to shake it and we have to drink.

Explanation:
In the above-given question,
given that,
Mixin ti up.
that means when we are drinking we have to shake it and we have to use it.
either it is a syrup or any other drink.

Question 3.
Construct Arguments Predict an answer to this Main Question. Explain your prediction.
Envision Math Common Core Grade 7 Answer Key Topic 2 Analyze And Use Proportional Relationships 65

Answer:
There is one pineapple and 5 apples.

Explanation:
In the above-given question,
given that,
there are 5 apples and 1 pineapple.
so 1/5 is the unit rate.

Question 4.
On the number line below, write a number that is too small to be the answer. Write a number that is too large. Too small
Envision Math Common Core Grade 7 Answer Key Topic 2 Analyze And Use Proportional Relationships 66

Answer:
The number that is too small is 0.01.

Explanation:
In the above-given question,
given that,
the numbers on the number line are 0.01, 0.02, and …..
the number that is too small is 0.01.
the number that is too large is 1.
so the number that is too small is 0.01.
Envision-Math-Common-Core-7th-Grade-Answers-Key-Topic-2-Analyze And Use Proportional Relationships-23

Question 5.
Plot your prediction on the same number line.

Answer:
The number that is too small is 0.01.

Explanation:
In the above-given question,
given that,
the numbers on the number line are 0.01, 0.02, and …..
the number that is too small is 0.01.
the number that is too large is 1.
so the number that is too small is 0.01.

ACT 2
Question 6.
What information in this situation would be helpful to know? How would you use that information?
Envision Math Common Core Grade 7 Answer Key Topic 2 Analyze And Use Proportional Relationships 67
Answer:

Question 7.
Use Appropriate Tools What tools can you use to solve the problem? Explain how you would use them strategically.
Answer:

Question 8.
Model with Math Represent the situation using mathematics. Use your representation to answer the Main Question.
Answer:

Question 9.
What is your answer to the Main Question? Is it higher or lower than your prediction? Explain why.
Envision Math Common Core Grade 7 Answer Key Topic 2 Analyze And Use Proportional Relationships 68
Answer:

ACT 3
Question 10.
Write the answer you saw in the video.
Answer:

Question 11.
Reasoning Does your answer match the answer in the video? If not, what are some reasons that would explain the difference?
Envision Math Common Core Grade 7 Answer Key Topic 2 Analyze And Use Proportional Relationships 69
Answer:

Question 12.
Make Sense and Persevere Would you change your model now that you know the answer? Explain.
Envision Math Common Core Grade 7 Answer Key Topic 2 Analyze And Use Proportional Relationships 70
Answer:

Reflect
Question 13.
Model with Math Explain how you used a mathematical model to represent the situation. How did the model help you answer the Main Question?
Envision Math Common Core Grade 7 Answer Key Topic 2 Analyze And Use Proportional Relationships 71
Answer:

Question 14.
Critique Reasoning Choose a classmate’s model. How would you adjust that model?
Answer:

SEQUEL
Question 15.
Use Structure A classmate usually adds 6 drops to 16 ounces of water. Use your updated model to predict the number of drops she would use for the large container.
Envision Math Common Core Grade 7 Answer Key Topic 2 Analyze And Use Proportional Relationships 72
Answer:

Lesson 2.5 Graph Proportional Relationships

Explore It!
The graph shows the time it takes Jacey to print T-shirts for her school’s math club.

I can… use a graph to determine whether two quantities are proportional.
Envision Math Common Core Grade 7 Answer Key Topic 2 Analyze And Use Proportional Relationships 73

A. Use the points on the graph to complete the table. Are the quantities proportional? Explain.
Envision Math Common Core Grade 7 Answer Key Topic 2 Analyze And Use Proportional Relationships 74

Answer:
Yes, they are proportional.

Explanation:
In the bove-given question,
given that,
The graph shows the time it takes Jacey to print T-shirts for her school’s math club.
the number of t-shirts is 1 the time in minutes is 5.
2 x 5 = 10.
3 x 5 = 15.
4 x 5 = 20.
5 x 5 = 25.
so they are proportional.
Envision-Math-Common-Core-7th-Grade-Answers-Key-Topic-2-Analyze And Use Proportional Relationships-24

B. Start at (1,5). As you move from one point to the next on the graph, how does the x-coordinate change? How does the y-coordinate change?

Answer:
The x and y coordinates do not change.

Explanation:
In the above-given question,
given that,
Start at (1,5). As you move from one point to the next on the graph.
1 x 5 = 5.
so the coordinates x and y are not changed.

C. Write the points for 0 T-shirts and for 5 T-shirts as ordered pairs. Graph the points and draw the line that passes through all six points.

Answer:
The graph increases.

Explanation:
In the above-given question,
given that,
the points for 0 t-shirts and for 5 T-shirts as ordered pairs.
the point is (0,5).
so the graph increases automatically.

Focus on math practices
Reasoning Suppose that after printing 4 T-shirts it takes Jacey 4 minutes to change the ink cartridge. Would this point for 5 T-shirts lie on the line you drew in Part C? Explain.

Answer:
The point (5,15) lies on the line.

Explanation:
In the above-given question,
given that,
after printing 4 T-shirts it takes Jacey 4 minutes to change the ink cartridge.
5,15 lies on the line.

Essential Question
What does the graph of a proportional relationship look like?

Try It!

Each \(\frac{1}{4}\) cup serving of cereal has 3 grams of protein. How can you use the graph at the right to determine whether the quantities are proportional and to find how many grams of protein are in 1 cup of the cereal?
Envision Math Common Core Grade 7 Answer Key Topic 2 Analyze And Use Proportional Relationships 75

Answer:
The number of grams of protein is in 1 cup of cereal = 9 grams.

Explanation:
In the above-given question,
given that,
Each \(\frac{1}{4}\) cup serving of cereal has 3 grams of protein.
1/4 x 3.
for 1 cup of cereal is 9 grams of protein.
1 x 9 = 9.
so the number of grams of protein is in 1 cup of cereal = 9 grams.

Convince Me! How can you find the constant of proportionality from the coordinates of one point on the graph?

Try It!

Suppose the graph of Mr. Brown’s Road Trip is extended. Find the ordered pair with an x-coordinate of 7. What does this point represent in the situation?
If the graph is extended, it will pass through the point (7, _______). This means
Mr. Brown drives _______ miles in _______ hours.

Answer:

Try It!

Draw two graphs that pass through the point (2, 3), one that represents a proportional relationship and one that does not. Label your graphs as Proportional or NOT Proportional.
Envision Math Common Core Grade 7 Answer Key Topic 2 Analyze And Use Proportional Relationships 76

Answer:
The point (2,3) passes through the point.

Explanation:
In the above-given question,
given that,
the point (2,3) is a proportional relationship.
Envision-Math-Common-Core-7th-Grade-Answers-Key-Topic-2-Analyze And Use Proportional Relationships-25

KEY CONCEPT
The graph of a proportional relationship is a straight line through the origin.
Envision Math Common Core Grade 7 Answer Key Topic 2 Analyze And Use Proportional Relationships 77
This graph shows that the total cost of tickets at a county fair is proportional to the number of tickets purchased. For each point (x, y) on the line except (0, 0), \(\frac{y}{x}\) = 5, which is the constant of proportionality.

Do You Understand?
Question 1.
Essential Question What does the graph of a proportional relationship look like?

Answer:
The graph shows the proportional relationship.

Explanation:
In the above-given question,
given that,
This graph shows that the total cost of tickets at a county fair is proportional to the number of tickets purchased.
for each point (x,y) except (0,0).
y/x = 5.
so the constant of proportionality is 5.

Question 2.
Reasoning Why will the graph of every proportional relationship include the point (0, 0)?

Answer:
Yes, the graph of every proportional relationship includes the point (0,0).

Explanation:
In the above-given question,
given that,
for every graph, the point (0,0) is included.
the points are (0,0), (1,5), (2, 10), (3, 15).
so the graph of every proportional relationship includes the point (0,0).

Question 3.
Construct Arguments Makayla plotted two points, (0, 0) and (3, 33), on a coordinate grid. Noah says that she is graphing a proportional relationship. Is Noah correct? Explain.

Answer:
Yes, it is a graphing a proportional.

Explanation:
In the above-given question,
given that,
the points are (0,0) and (3, 33).
so it is a graphing a proportional.

Do You Know How?
For 4-7, use the information below.

Martin and Isabelle go bowling. Each game costs $10, and they split that cost. Martin has his own bowling shoes, but Isabelle pays $3 to rent shoes.
Question 4.
Complete the graphs below. Martin’s
Envision Math Common Core Grade 7 Answer Key Topic 2 Analyze And Use Proportional Relationships 78

Answer:
The points ($3, $10).

Explanation:
In the above-given question,
given that,
Martin and Isabelle go bowling. Each game costs $10, and they split that cost.
Martin has his own bowling shoes, but Isabelle pays $3 to rent shoes.
the point (3, 10) is not a proportional relationship.
Envision-Math-Common-Core-7th-Grade-Answers-Key-Topic-2-Analyze And Use Proportional Relationships-26

Question 5.
Which graph shows a proportional relationship? Explain why.

Answer:
The point (1,5) shows a proportional relationship.

Explanation:
In the above-given question,
given that,
points 1,5, and 2,10 are proportional to each other.
the point (1,5) and (2,10) shows a proportional relationship.

Question 6.
Choose one point on the graph of the proportional relationship and explain what this point means in terms of the situation.

Answer:
The point (1,5) shows a proportional relationship.

Explanation:
In the above-given question,
given that,
points 1,5, and 2,10 are proportional to each other.
the point (1,5) and (2,10) shows a proportional relationship.

Question 7.
What equation represents the proportional relationship?

Answer:
The point (1,5) shows a proportional relationship.

Explanation:
In the above-given question,
given that,
points 1,5, and 2,10 are proportional to each other.
the point (1,5) and (2,10) shows a proportional relationship.

Practice & Problem Solving

Question 8.
For each graph shown, tell whether it shows a proportional relationship. Explain your reasoning.
Envision Math Common Core Grade 7 Answer Key Topic 2 Analyze And Use Proportional Relationships 79

Answer:
Graph a shows the proportional relationship.

Explanation:
In the above-given question,
given that,
the graph (2,2) and (4,4) shows the proportional relationship.
so graph a shows the proportional relationship.

Question 9.
The graph shows the number of boxes a machine packages over time. Is the relationship proportional? How many boxes does the machine package in 4 minutes?
Envision Math Common Core Grade 7 Answer Key Topic 2 Analyze And Use Proportional Relationships 80

Answer:
The number of boxes does the machine package in 4 minutes = 100 boxes.

Explanation:
In the above-given question,
given that,
the graph shows the points.
the points are (0,0), (2,50), (4,100).
in 4 minutes the number of boxes that can be package is 100 boxes.
so the number of boxes does the machine package in 4 minutes = 100 boxes.

Question 10.
Does the graph show a proportional relationship? If so, use the graph to find the constant of proportionality.
Envision Math Common Core Grade 7 Answer Key Topic 2 Analyze And Use Proportional Relationships 81

Answer:
Yes, the graph shows a proportional relationship.
The constant of proportionality is 25.

Explanation:
In the above-given question,
given that,
the points (0,25), (1, 50), (2, 75), (3 , 100), (4,125), (5, 175).
so the graph shows a proportional relationship.
so the constant of proportionality is 25.

Question 11.
The graph shows a proportional relationship between the cups of flour a baker uses and the number of cookies made.
Envision Math Common Core Grade 7 Answers Topic 2 Analyze And Use Proportional Relationships 82
a. Use Structure What does the point (0, 0) represent in the situation?

Answer:
The points (0,0) represents a proportional relationship.

Explanation:
In the above-given question,
given that,
the points are (0,0), (1, 18), (2, 35), (3, 50), (4, 75) and (6, 125).
so the points (0,0) represents a proportional relationship.

b. What does the point (1, 18) represent?

Answer:
The point (1, 18) is a proportional relationship.

Explanation:
In the above-given question,
given that,
the points are (0,0), (1, 18), (2, 35), (3, 50), (4, 75) and (6, 125).
so the points (1,18) represents a proportional relationship.

Question 12.
The points (0.5, \(\frac{1}{10}\)) and (7, 1\(\frac{2}{5}\)) are on the graph of a proportional relationship.
a. What is the constant of proportionality?

Answer:
The graph is a proportional relationship.

Explanation:
In the above-given question,
given that,
The points (0.5, \(\frac{1}{10}\)) and (7, 1\(\frac{2}{5}\)).
points (0.5, 0.1) and (7, 1.4).
so the graph shows a proportional relationship.

b. Name one more point on the graph.

Answer:
Point (4,75) is a proportional relationship.

Explanation:
In the above-given question,
given that,
the points are (0.5, 0.1), (7, 1.4), and (4, 75).
the point (4, 75) is a proportional relationship.

c. Write an equation that represents the proportional relationship.

Answer:
The point (3, 50) represents the proportional relationship.

Explanation:
In the above-given question,
given that,
the points are (0.5, 0.1), (7, 1.4), and (4, 75).
the point (3, 50) represents the proportional relationship.

Question 13.
Higher Order Thinking Denmark uses the kroner as its currency. Before a trip to Denmark, Mia wants to exchange $1,700 for kroner.
Envision Math Common Core Grade 7 Answers Topic 2 Analyze And Use Proportional Relationships 83
a. Does Bank A or Bank B have the better exchange rate? Explain.

b. How many more kroner would Mia get if she exchanged her $1,700 at the bank with the better exchange rate?
Answer:

Assessment Practice

Question 14.
Does the graph at the right show a proportional relationship between x and y? Explain.
Envision Math Common Core Grade 7 Answers Topic 2 Analyze And Use Proportional Relationships 84

Answer:
No, the graph does not show a proportional relationship.

Explanation:
In the above-given question,
given that,
the points are (0,1), (2, 2), (3, 2), and (4, 5).
the point (2, 2) shows the relationship.
but the graph does not show a proportional relationship.

Question 15.
The graph at the right shows the relationship between rainfall during the growing season and the growth of a type of plant. Which statements about the graph are true?
Envision Math Common Core Grade 7 Answers Topic 2 Analyze And Use Proportional Relationships 85
☐ The point (1, 10) shows the constant of proportionality.
☐ The constant of proportionality is \(\frac{5}{7}\).
☐ The graph does not show a proportional relationship.
☐ The graph is a straight line through the origin.
☐ The point (28, 20) means the type of plant grows 20 mm when it rains 28 cm.

Answer:
The constant of proportionality is (5/7).

Explanation:
In the above-given question,
given that,
the constant of proportionality is 5/7.
2 x 5 = 10, 2 x 7 = 14.
4 x 5 = 20, and 4 x 7 = 28.
6 x 5 = 30, and 6 x 7 = 42.
so the constant of proportionality is (5/7).

Lesson 2.6 Apply Proportional Reasoning to Solve Problems

Solve & Discuss It!
Xander and Pedro are at an ice cream social. For every scoop of ice cream, Xander uses cup of fruit topping. Pedro uses one more tablespoon of fruit topping than the number of scoops. If Xander and Pedro each use the same amount of fruit topping, how many scoops of ice cream does each use?

I can… determine whether a relationship is proportional and use representations to solve problems.

Look for Relationships
There are 2 tablespoons in \(\frac{1}{8}\) cup.

Focus on math practices
Reasoning for which person, Xander or Pedro, is the relationship between the quantities of ice cream and fruit topping proportional? Explain.
Envision Math Common Core Grade 7 Answers Topic 2 Analyze And Use Proportional Relationships 86

Essential Question
How can proportional reasoning help solve a problem?

Answer:
Yes, proportional reasoning help in solving a problem.

Explanation:
In the above-given question,
given that,
There are 2 tablespoons in \(\frac{1}{8}\) cup.
2 x 1/8 = 4.
so the constant of proportionality is 4.

Try It!

After selling half their card collections, DeShawn and Stephanie each buy 9 new cards. What is the ratio of the number of cards DeShawn has to the number Stephanie has?

Answer:
The number of cards DeShawn has to the number Stephanie has = 9 : 2.

Explanation:
In the above-given question,
given that,
After selling half their card collections, DeShawn and Stephanie each buy 9 new cards.
so the ratio is 9 : 2.
so the number of cards DeShawn has to the number Stephanie has = 9 : 2.

Convince Me! Why did the ratio stay the same in the Example but change in the Try It?

Try It!

A florist makes bouquets that include 50 white flowers and 7 red flowers. If the florist orders 1,050 white flowers and 140 red flowers, there will be leftover flowers. How can the florist adjust the order so there are no leftover flowers?

Answer:
The florist adjusts the order so there are no leftover flowers = 20.

Explanation:
In the above-given question,
given that,
A florist makes bouquets that include 50 white flowers and 7 red flowers.
If the florist orders 1,050 white flowers and 140 red flowers.
1050 : 50 = 20.
140 : 7 = 20.
so there are no leftover flowers.

KEY CONCEPT
Think about how two quantities are related before you decide to use proportional reasoning to solve a problem.
Envision Math Common Core Grade 7 Answers Topic 2 Analyze And Use Proportional Relationships 87
You cannot use proportional reasoning to solve this problem because Josh’s age is not a constant multiple of Evie’s age.

Do You Understand?
Question 1.
Essential Question How can proportional reasoning help solve a problem?

Answer:
Proportional reasoning can help in solving a problem.

Explanation:
In the above-given question,
given that,
two quantities are related before you decide to use proportional reasoning to solve a problem.
for example:
Josh is 4 years older than Evie.
Josh is 3 times as old as Evie.
when Evie is 2 years old, Josh is 6 years old.

Question 2.
Use Appropriate Tools How can knowing how to represent proportional relationships in different ways be useful in solving problems?

Answer:
Proportional reasoning can help in solving a problem.

Explanation:
In the above-given question,
given that,
two quantities are related before you decide to use proportional reasoning to solve a problem.
for example:
Josh is 4 years older than Evie.
Josh is 3 times as old as Evie.
when Evie is 2 years old, Josh is 6 years old.

Question 3.
Reasoning How many ways are there to adjust two quantities so that they are in a given proportional relationship? Explain your reasoning.

Answer:
There are 2 ways to adjust two quantities so that they are in given proportional reasoning to solve a problem.

Explanation:
In the above-given question,
given that,
two quantities are related before you decide to use proportional reasoning to solve a problem.
for example:
Josh is 4 years older than Evie.
Josh is 3 times as old as Evie.
when Evie is 2 years old, Josh is 6 years old.

Do You Know How?
Question 4.
A recipe calls for 15 oz of flour for every 8 oz of milk.
a. Is the relationship between ounces of flour and ounces of milk proportional? Explain.

Answer:
No, there is no proportional relationship.

Explanation:
In the above-given question,
given that,
A recipe calls for 15 oz of flour for every 8 oz of milk.
15/8.
so there is no proportional relationship.

b. If you use 15 oz of milk, how much flour should you use?

Answer:
We can use either 3 or 5 oz of flour.

Explanation:
In the above-given question,
given that,
if we use 15 oz of milk, we can use either 3 or 5 oz of flour.
15/3 = 5.
so we can use either 3 or 5 oz of flour.

Question 5.
A food packing company makes a popular fruit cocktail. To ensure a good mixture of fruit, there are 3 cherry halves for every 8 white grapes in a jar. An inspector notices that one jar has 12 cherry halves and 20 white grapes. What can be done to fix the error?

Answer:
The error is 6.6.

Explanation:
In the above-given question,
given that,
A food packing company makes a popular fruit cocktail.
To ensure a good mixture of fruit, there are 3 cherry halves for every 8 white grapes in a jar.
An inspector notices that one jar has 12 cherry halves and 20 white grapes.
12/8 = 3/2.
20/3 = 6.6.
the error is 6.6.

Practice & Problem Solving

Multimedia In 6 and 7, determine whether you can use proportional reasoning and then solve.
Question 6.
If Hector is 8 years old and Mary is 3 years old, how old will Mary be when Hector is 16?

Answer:
Mary is 6 years old.

Explanation:
In the above-given question,
given that,
If Hector is 8 years old and Mary is 3 years old.
8 x 2 = 16.
3 x 2 = 6.
16/2 = 8.
6/2 = 3.
so Mary is 6 years old.

Question 7.
Marco needs to buy some cat food. At the nearest store, 3 bags of cat food cost $15.75. How much would Marco spend on 5 bags of cat food?

Answer:
The Marco spend of 5 bags of cat food = $26.25.

Explanation:
In the above-given question,
given that,
Marco needs to buy some cat food.
At the nearest store, 3 bags of cat food cost $15.75.
$15.75 / 3 = 5.25.
5.25 x 5 = 26.25.
so the Marco spend on 5 bags of cat food = $26.25.

Question 8.
An architect makes a model of a new house with a patio made with pavers. In the model, each paver in the patio is \(\frac{1}{3}\) in. long and a \(\frac{1}{6}\) in. wide. The actual dimensions of the pavers are shown.
Envision Math Common Core Grade 7 Answers Topic 2 Analyze And Use Proportional Relationships 88
a. What is the constant of proportionality that relates the length of a paver in the model and the length of an actual paver?

Answer:
The length of a paper in the model and the length of the actual paver = 0.03125: 0.0528.

Explanation:
In the above-given question,
given that,
An architect makes a model of a new house with a patio made with pavers.
each paver in the patio is \(\frac{1}{3}\) in. long and a \(\frac{1}{6}\) in. wide.
area of the paver = l x b.
area = 1/3 x 1/6.
area = 0.33 x 0.16.
area = 0.0528.
actual paver area = l x b.
area = 1/4 x 1/8.
area = 0.25 x 0.125.
area = 0.03125.
so the ratio is 0.03125 : 0.0528.

b. What is the constant of proportionality that relates the area of a paver in the model and the area of an actual paver? Explain your reasoning.

Answer:
The constant of proportionality is 2.

Explanation:
In the above-given question,
given that,
An architect makes a model of a new house with a patio made with pavers.
each paver in the patio is \(\frac{1}{3}\) in. long and a \(\frac{1}{6}\) in. wide.
area of the paver = l x b.
area = 1/3 x 1/6.
area = 0.33 x 0.16.
area = 0.0528.
actual paver area = l x b.
area = 1/4 x 1/8.
area = 0.25 x 0.125.
area = 0.03125.
2 x 4 = 8.
3 x 2 = 6.
so the proportionality of constant is 2.

Question 9.
Reasoning The table lists recommended amounts of food to order for 25 guests. Nathan is hosting a graduation party for 40 guests. There will also be guests stopping by for a short time. For ordering purposes, Nathan will count each of the 45 “drop-in” guests as half a guest. How much of each food item should Nathan order?
Envision Math Common Core Grade 7 Answers Topic 2 Analyze And Use Proportional Relationships 89

Answer:
The amount of each food item should Nathan order is 24 pieces of fried chicken, 3.66 pounds of Delimeats, and 1.75 pounds of Lasagna.

Explanation:
In the above-given question,
given that,
the table lists recommended amounts of food to order for 25 guests.
24 pieces of fried chicken, 3.66 pounds of Deli meats, and 1.75 pounds of Lasagna.

Question 10.
Emily and Andy each go to a hardware store to buy wire. The table shows the relationship between the cost and the length of wire.
Envision Math Common Core Grade 7 Answers Topic 2 Analyze And Use Proportional Relationships 90
a. Emily needs 24 feet of wire. How much will she spend on wire?

Answer:
The amount she will spend on wire = $5.

Explanation:
In the above-given question,
given that,
Emily needs 24 feet of wire.
580/23.2 = 25.
$25 / 5 = 5.
so the amount she will spend on wire = $5.

b. Andy needs 13 yards of wire. How much will he spend on wire?

Answer:
The amount he will spend on wire = $0.024.

Explanation:
In the above-given question,
given that,
Emily needs 24 feet of wire.
580/23.2 = 25.
1 inch = 0.027 yards.
0.027 x 12 = 0.324.
0.324/13 = 0.024.

Question 11.
Make Sense and Persevere The weights of Michael’s and Brittney’s new puppies are shown in the table and graph. Whose dog gains weight more quickly? Explain.
Envision Math Common Core Grade 7 Answers Topic 2 Analyze And Use Proportional Relationships 91

Answer:
The dog gains weight more quickly = Brittney’s puppy.

Explanation:
In the above-given question,
given that,
The weights of Michael’s and Brittney’s new puppies are shown in the table in the graph.
the weight of 155/1 = 155.
310/2 = 155.
465/3 = 155.
8.6/1 = 8.6.
17.2/2 = 8.6.
so the weight of the Brittney’s puppy weighs more.

Question 12.
Higher Order Thinking Marielle’s painting has the dimensions shown. The school asks her to paint a larger version that will hang in the cafeteria. The larger version will be twice the width and twice the height. Is the area of the original painting proportional to the area of the larger painting? If so, what is the constant of proportionality?
Envision Math Common Core Grade 7 Answers Topic 2 Analyze And Use Proportional Relationships 92

Answer:
The constant of proportionality = 2.

Explanation:
In the above-given question,
given that,
Marielle’s painting has the dimensions shown.
The larger version will be twice the width and twice the height.
area = length x width.
area = 24 x 18.
area = 432 sq inches.
so the constant of proportionality = 2.

Assessment Practice

Question 13.
The ratio of orange juice concentrate to water that Zoe used to make orange juice yesterday was 3: 7. She used 14 ounces of water in the juice yesterday. Today she wants to make twice as much orange juice.
PART A
Is the relationship between the amount of orange juice concentrate c and the amount of water w proportional? Explain.

Answer:
The amount of orange juice concentrate c and the amount of water w is proportional.

Explanation:
In the above-given question,
given that,
The ratio of orange juice concentrate to water that Zoe used to make orange juice yesterday was 3: 7.
She used 14 ounces of water in the juice yesterday.
3 : 7 = 6 : 14.
3 x 2 = 6.
7 x 2 = 14.
so the amount of orange juice concentrate c and the amount of water w is proportional.

PART B
How much orange juice concentrate does she need?
A. 6 ounces
B. 12 ounces
C. 18 ounces
D. 28 ounces

Answer:
Option A is correct.

Explanation:
In the above-given question,
given that,
The ratio of orange juice concentrate to water that Zoe used to make orange juice yesterday was 3: 7.
She used 14 ounces of water in the juice yesterday.
3 : 7 = 6 : 14.
3 x 2 = 6.
7 x 2 = 14.
so option A is correct.

Topic 2 Review

Topic Essential Question
How can you recognize and represent proportional relationships and use them to solve problems?

Vocabulary Review
Complete each definition and then provide an example of each vocabulary word.

Envision Math Common Core Grade 7 Answers Topic 2 Analyze And Use Proportional Relationships 93

Answer:
1. Proportional relationship.
2. Constant of proportionality.
3. True proportion.

Explanation:
In the above-given question,
given that,
the definitions of proportional relationship, constant of proportionality, and true proportion.
(1,5), (2, 10), (3, 15) and (4,20) are the examples of proportional relationship.
y x 12.5 = 90.
y = 90/12.5.
y = 72 is the example of constant of proportionality.
2/10 = 1/5.
Envision-Math-Common-Core-7th-Grade-Answers-Key-Topic-2-Analyze And Use Proportional Relationships-27

Use Vocabulary in Writing
A sunflower grows 8 feet in 92 days. Assuming that it grows at a constant rate, explain how you could use this information to find the number of days it took for the sunflower to grow 6 feet. Use vocabulary words in your explanation.

Answer:
The number of days it took for the sunflower to grow 6 feet = 15.3 days.

Explanation:
In the above-given question,
given that,
A sunflower grows 8 feet in 92 days.
y = kx.
6 = k . 92.
k = 92/6.
k = 15.3.
so the number of days it took for the sunflower to grow 6 feet = 15.3 days.

Concepts and Skills Review

Lessons 2.1 And 2.2 Connect Ratios, Rates, and Unit Rates Determine Unit Rates with Ratios of Fractions

Quick Review
A ratio is a relationship in which for every x units of one quantity there are y units of another quantity. A rate is a ratio that relates two quantities with different units. A unit rate relates a quantity to 1 unit of another quantity. You can use what you know about equivalent fractions and calculating with fractions to write a ratio of fractions as a unit rate.

Example
In 3 days, a typical robin can eat up to 42.6 feet of earthworms. Write a rate to relate the number of feet of earthworms to the number of days. Then find the unit rate.
\(\frac{42.6 \text { feet }}{3 \text { days }}=\frac{14.2 \text { feet }}{1 \text { day }}\)
A robin eats 14.2 feet of earthworms per day.

Practice
Question 1.
Mealworms are a healthy food for wild songbirds. Adam buys a 3.5-oz container of mealworms for $8.75. Marco buys a 3.75-oz container of mealworms for $9.75. Which container is the better deal?

Answer:
The better deal = Marco’s deal.

Explanation:
In the above-given question,
given that,
Mealworms are healthy food for wild songbirds.
Adam buys a 3.5-oz container of mealworms for $8.75.
Marco buys a 3.75-oz container of mealworms for $9.75.
8.75 / 3.5 = 2.5.
9.75 / 3.75 = 2.6.
so the Marco container is better.

Question 2.
A painter mixes 2\(\frac{1}{2}\) pints of yellow paint with 4 pints of red paint to make a certain shade of orange paint. How many pints of yellow paint should be mixed with 10 pints of red paint to make this shade of orange?

Answer:
The number of yellow paint should be mixed = 20.

Explanation:
In the above-given question,
given that,
A painter mixes 2\(\frac{1}{2}\) pints of yellow paint with 4 pints of red paint to make a certain shade of orange paint.
2(1/2) = 5/2.
5/2 = 2.5.
2.5 x 2 = 5.
10 x 2 = 20.
so the number of yellow paint should be mixed = 20.

Lesson 2.3 Understand Proportional Relationships: Equivalent Ratios

Quick Review
Two quantities x and y have a proportional relationship if the ratios for every related pair of x and y are equivalent. You can write a proportion to show that two ratios have the same value.

Example
Does the table show a proportional relationship between x and y? Explain.
Envision Math Common Core Grade 7 Answers Topic 2 Analyze And Use Proportional Relationships 94
Because every ratio is equivalent, there is a proportional relationship between x and y.

Practice

In 1 and 2, use the table below that shows information about squares.
Envision Math Common Core Grade 7 Answers Topic 2 Analyze And Use Proportional Relationships 95
Question 1.
Are the perimeter and the side length of squares proportional? Explain.

Answer:
The perimeter and the side length of squares are proportional.

Explanation:
In the above-given question,
given that,
the side lengths are 2, 4, and 6.
the perimeters are 8, 16, and 24.
2 x 4 = 8.
4 x 4 = 16.
6 x 4 = 24.
so the perimeter and the side length of squares are proportional.

Question 2.
Write and solve a proportion to find the perimeter of a square when its side length is 12.

Answer:
The perimeter is 12 cm.

Explanation:
In the above-given question,
given that,
the side length is 12.
12 x 4 = 48.
so the perimeter is 12 cm.

Lesson 2.4 Describe Proportional Relationships: Constant of Proportionality

Quick Review
The equation y = kx describes a proportional relationship between two quantities x and y, where k is the constant of proportionality.k= for any related pair of x and y except when x = 0.

Example
The table shows the wages Roger earned for the hours he worked. What equation relates the wages, w, and the number of hours, h?
Envision Math Common Core Grade 7 Answers Topic 2 Analyze And Use Proportional Relationships 96
Find the constant of proportionality, k.
\(\frac{27}{3}\) = 9 \(\frac{45}{5}\) = 9 \(\frac{54}{6}\) = 9
Write the equation in the form y = kx.
W = 9h

Practice
Question 1.
Sally is going on vacation with her family. In 2 hours they travel 90.5 miles. If they travel at the same speed, write an equation that represents how far they will travel, d, in h hours.

Answer:
The equation that represents how far they will travel = 45.25 miles.

Explanation:
In the above-given question,
given that,
Sally is going on vacation with her family.
In 2 hours they travel 90.5 miles.
90.5/2 = 45.25 miles.
so the equation that represents how far they will travel = 45.25 miles.

Question 2.
The table shows the weights of bunches of bananas and the price of each bunch. Identify the constant of proportionality. Write an equation to relate weight, w, to the price, p.
Envision Math Common Core Grade 7 Answers Topic 2 Analyze And Use Proportional Relationships 97

Answer:
The constant of proportionality is 3.

Explanation:
In the above-given question,
given that,
The table shows the weights of bunches of bananas and the price of each bunch.
the price of the bananas is $1.35, $1.71, and $2.34.
the weight of bananas is 3 pounds, 3.8 pounds, and 5.2 pounds.
1.35/3 = 0.45.
1.71/3.8 = 0.45.
2.34/5.2 = 0.45.
so the constant of proportionality = 3.

Lesson 2.5 Graph Proportional Relationships

Quick Review
The graph of a proportional relationship is a straight line through the origin. You can identify the constant of proportionality, k, from the point (1, k) or by dividing for any point (x, y) except the origin.

Example
Is the number of sunny days proportional to the number of rainy days? If so, find the constant of proportionality, and explain its meaning in this situation.
Envision Math Common Core Grade 7 Answers Topic 2 Analyze And Use Proportional Relationships 98
The graph is a straight line through the origin, so it shows a proportional relationship. The constant of proportionality is which means for every1 rainy day, there were 2 sunny days

Practice
Question 1.
Does the graph show a proportional relationship? Explain.
Envision Math Common Core Grade 7 Answers Topic 2 Analyze And Use Proportional Relationships 99

Answer:
The proportional relationship is 1 : 2.

Explanation:
In the above-given question,
given that,
the number of tickets is 4, 8, and 12.
the cost of the tickets = 4, 8, 12, and 14.
the points are (2,4), (4,8), (8, 12), and (12, 16).
2 x 2 = 4, 4 x 2 = 8, 8 x 2 = 16.
so the proportional relationship is 1:2.

Question 2.
Sketch a graph that represents a proportional relationship.

Answer:
The proportional relationship is 1 : 2.

Explanation:
In the above-given question,
given that,
the number of tickets is 4, 8, and 12.
the cost of the tickets = 4, 8, 12, and 14.
the points are (2,4), (4,8), (8, 12), and (12, 16).
2 x 2 = 4, 4 x 2 = 8, 8 x 2 = 16.
so the proportional relationship is 1:2.

Lesson 2.6 Apply Proportional Reasoning to solve Problems

Quick Review
Think about how two quantities are related before you decide to use proportional reasoning to solve a problem.

Example
Ann-Marie makes gift baskets that each contain 3 pounds of gourmet cheese with every 2 boxes of crackers. She ordered 80 pounds of cheese and 50 boxes of crackers. How can she adjust her order to make 25 gift baskets, with no leftover items?
\(\frac{3 \times 25}{2 \times 25}=\frac{75}{50}\)
Ann-Marie can order 5 fewer pounds of cheese so that her order of 75 pounds of cheese and 50 boxes of crackers will make exactly 25 gift baskets.

Practice
Decide which problems can be solved by using proportional reasoning. Select all that apply.

☐ Yani buys 4 dozen flyers for $7.25. What is the cost of 12 dozen?
☐ First-class letters cost $0.49 for the first ounce and $0.22 for each additional ounce. What is the cost of a 5-oz letter?
Envision Math Common Core Grade 7 Answers Topic 2 Analyze And Use Proportional Relationships 100
What is the value of y when x = 8?
Envision Math Common Core Grade 7 Answers Topic 2 Analyze And Use Proportional Relationships 101
What is the value of y when x = 5?

Answer:
The value of y is 15 when x = 5.

Explanation:
In the above-given question,
given that,
the values of x and y are given.
x = 1.5, 3, 5.2, 6.5, and 8.
y = 4.5, 9, 15.6, 19.5, and 24.
5 x 3 = 15.
15/3 = 5.
so the value of y is 15 when x = 5.

Topic 2 Fluency Practice

Pathfinder
Shade a path from START to FINISH. Follow the solutions from the least value to the greatest value. You can only move up, down, right, or left.
I can… add, subtract, multiply, and divide integers.
Envision Math Common Core Grade 7 Answers Topic 2 Analyze And Use Proportional Relationships 102

Answer:
3 – 24 = -21, 3 x -13 = -39, 14 x -3 = -42, 44 – 49 = -5, -48/16 = 3, -26 + 7 = -19, -7 – 27 = -34, 13 x -4 = -52, 8 – 19 = -11, -18 / -6 = 3, 3 x -5 = -15, 28/-2 = 14, -35/2 = 17.5 , 4 + – 25 = -21, -68 / 17 = -4, 23 – 46 = -23, -2 – 11 = 13, 15 + -(26) = -11, 17 – 23 = -6, 29 – 38 = -9, -12 / -3 = 4, 18 + (-32) = -14, 11 – 29 = -18, -60/12 = -5, -3 x -2 = 6.

Explanation:
In the above-given question,
given that,
the multiplication, division, addition, and subtraction of numbers.
3 – 24 = -21, 3 x -13 = -39, 14 x -3 = -42, 44 – 49 = -5, -48/16 = 3, -26 + 7 = -19, -7 – 27 = -34, 13 x -4 = -52, 8 – 19 = -11, -18 / -6 = 3, 3 x -5 = -15, 28/-2 = 14, -35/2 = 17.5 , 4 + – 25 = -21, -68 / 17 = -4, 23 – 46 = -23, -2 – 11 = 13, 15 + -(26) = -11, 17 – 23 = -6, 29 – 38 = -9, -12 / -3 = 4, 18 + (-32) = -14, 11 – 29 = -18, -60/12 = -5, -3 x -2 = 6.
Envision-Math-Common-Core-7th-Grade-Answers-Key-Topic-2-Analyze And Use Proportional Relationships-28

Envision Math Common Core Grade 6 Answer Key Topic 7 Solve Area, Surface Area, And Volume Problems

Practice with the help of enVision Math Common Core Grade 6 Answer Key Topic 7 Solve Area, Surface Area, and Volume Problems regularly and improve your accuracy in solving questions.

Envision Math Common Core 6th Grade Answers Key Topic 7 Solve Area, Surface Area, And Volume Problems

Topic 7 Essential Question

How can the areas of certain shapes be found? What are the meanings of surface area and volume and how can surface area and volume be found?

3-ACT MATH

Envision Math Common Core 6th Grade Answer Key Topic 7 Solve Area, Surface Area, And Volume Problems 1
That’s a Wrap
That’s a Wrap How you wrap a gift can say a lot about you. Some people conserve resources by reusing paper bags instead of purchasing wrapping paper. It may not look as nice at first, but you can unleash your crafty side and decorate the gift yourself.
No matter what you use to wrap a gift, you always need to consider the size of the object and how much paper you’ll need. Think about this during the 3-Act Mathematical Modeling lesson.
Envision Math Common Core 6th Grade Answer Key Topic 7 Solve Area, Surface Area, And Volume Problems 2

Topic 7 enVision STEM Project

Did You Know?
Food packaging is designed to identify products and to attract customers. Packaging also ensures food remains fresh and free from damage and contamination.
Envision Math Common Core 6th Grade Answer Key Topic 7 Solve Area, Surface Area, And Volume Problems 3

Common types of material used for packaging are paper, cardboard, glass, steel, and aluminum. The best material to use for packaging depends on the contents—for example, soup in a paper bag could be rather messy!
Envision Math Common Core 6th Grade Answer Key Topic 7 Solve Area, Surface Area, And Volume Problems 4

Manufacturers consider several factors when designing packaging, such as cost, amount of material used, and potential environmental impact. Steel food cans are the most recycled food package in the United States.
Envision Math Common Core 6th Grade Answer Key Topic 7 Solve Area, Surface Area, And Volume Problems 5

Packaging engineers consistently look for ways to innovate. Some food packaging is now selfopening, self-closing, or selfheating. Some packaging is biodegradable. Other advances include edible packaging.
Envision Math Common Core 6th Grade Answer Key Topic 7 Solve Area, Surface Area, And Volume Problems 6

Your Task: Pack It
Food packaging engineers consider many elements related to both form and function when designing packaging. How do engineers make decisions about package designs as they consider constraints, such as limited dimensions or materials? You and your classmates will use the engineering design process to explore and propose food packaging that satisfies certain criteria.
Envision Math Common Core 6th Grade Answer Key Topic 7 Solve Area, Surface Area, And Volume Problems 7

Topic 7 Get Ready!

Review What You Know!

Vocabulary
Choose the best term from the box to complete each definition.

  • area
  • parallelogram
  • perpendicular
  • polygon
  • volume

Question 1.
A triangle is an example of a three-sided ___________

Answer:
A triangle is an example of a three-sided polygon.

Explanation:
In the above-given question,
given that,
A triangle is an example of a three-sided polygon.
for example:
equilateral triangle: all the sides are equal lengths, and all the internal angles are 60 degrees.
so a triangle is an example of a three-sided polygon.

Question 2.
________ lines form a right angle.

Answer:
Perpendicular lines form a right angle.

Explanation:
In the above-given question,
given that,
perpendicular lines form a right angle.
for example:
two lines that intersect and form right angles are called perpendicular lines.

Question 3.
__________ is the number of cubic units needed to fill a solid figure.

Answer:
Volume is the number of cubic units needed to fill a solid figure.

Explanation:
In the above-given question,
given that,
Volume is the number of cubic units needed to fill a solid figure.
for example:
A plane figure is a two-dimensional figure.
so volume is the number of cubic units needed to fill a solid figure.

Question 4.
__________ as opposite sides that are parallel and the same length.

Answer:
A parallelogram as opposite sides that are parallel and the same length.

Explanation:
In the above-given question,
given that,
A parallelogram has opposite sides that are parallel and the same length.
for example:
A rhombus is a parallelogram with sides that are all equal length.

Evaluate Expressions

Find the value of each expression when a = 12, b = 3, c = 4, and d = 9.
Question 5.
ac

Answer:
ac = 48.

Explanation:
In the above-given question,
given that,
a = 12, b = 3, c = 4, and d = 9.
ac = 12 x 4.
ac = 48.

Question 6.
\(\frac{1}{2}\)b

Answer:
3/2.

Explanation:
In the above-given question,
given that,
a = 12, b = 3, c = 4, and d = 9.
(1/2)b.
1/2 x 3.
3/2.

Question 7.
abd

Answer:
abd = 324.

Explanation:
In the above-given question,
given that,
a = 12, b = 3, c = 4, and d = 9.
abd = 12 x 3 x 9.
abd = 36 x 9.
abd = 324.

Question 8.
0.5a

Answer:
6.

Explanation:
In the above-given question,
given that,
a = 12, b = 3, c = 4, and d = 9.
0.5a = 0.5 x 12.
0.5a = 6.

Question 9.
\(\frac{3}{4}\)(ad)

Answer:
81.

Explanation:
In the above-given question,
given that,
a = 12, b = 3, c = 4, and d = 9.
(3/4)ad.
3/4 x 12 x 9.
3 x 3 x 9.
9 x 9 = 81.

Question 10.
ab + cd

Answer:
ab + cd = 72.

Explanation:
In the above-given question,
given that,
a = 12, b = 3, c = 4, and d = 9.
ab + cd.
12 x 3 + 4 x 9.
36 + 36.
72.

Multiplication

Find each product.
Question 11.
3.14 × 12

Answer:
3.14 x 12 = 37.68.

Explanation:
In the above-given question,
given that,
the numbers are 3.14 and 12.
find the product.
3.14 x 12.
37.68.

Question 12.
45.8 × 5

Answer:
45.8 x 5 = 229.

Explanation:
In the above-given question,
given that,
the numbers are 45.8 and 5.
find the product.
45.8 x 5.
229.

Question 13.
0.8 × 2.7

Answer:
2.16.

Explanation:
In the above-given question,
given that,
the numbers are 0.8 and 2.7.
find the product.
0.8 x 2.7.
2.16.

Question 14.
14 × 7.25 × 2.5

Answer:
14 x 7.25 x 2.5 = 253.75.

Explanation:
In the above-given question,
given that,
the numbers are 14, 7.25, and 2.5.
find the product.
14 x 7.25 x 2.5.
101.5 x 2.5.
253.75.

Question 15.
1\(\frac{1}{2}\) × \(\frac{1}{4}\)

Answer:
0.375.

Explanation:
In the above-given question,
given that,
the numbers are 3/2 and 1/4.
find the product.
3/2 x 1/4.
1.5 x 0.25.
0.375.

Question 16.
2\(\frac{1}{3}\) × 2\(\frac{1}{4}\) × 1\(\frac{3}{8}\)

Answer:
7.115.

Explanation:
In the above-given question,
given that,
the numbers are 7/3, 9/4, and 11/8.
find the product.
7/3 x 9/4 x 11/8.
2.3 x 2.25 x 1.375.
5.175 x 1.375.
7.115.

Geometry

Question 17.
How are parallelograms and rectangles similar and how are they different?

Answer:
The main difference between them is that a parallelogram has opposite sides that are equal.

Explanation:
In the above-given question,
given that,
the main difference between them is that a parallelogram has opposite sides equal.
while in a rectangle, the opposite sides are equal with all its adjacent sides being perpendicular to each.

Question 18.
How are the formulas for the area of a rectangle and the area of a triangle alike and different? Explain.

Answer:
Area of the rectangle = l x b.
area of the triangle = 1/2 x b x h.

Explanation:
In the above-given question,
given that,
the area of the rectangle = l x b.
where l = length and b = breadth.
the area of the triangle = 1/2 x b x h.
where b = base.
h = perpendicular height.

Language Development
Use the graphic organizer to help relate new vocabulary in this topic to terms you studied in previous topics.
Envision Math Common Core 6th Grade Answer Key Topic 7 Solve Area, Surface Area, And Volume Problems 8
Envision Math Common Core 6th Grade Answer Key Topic 7 Solve Area, Surface Area, And Volume Problems 9

Pick A Project

PROJECT 7A
If you could pick the shape of a building to live in, what shape would it be?
PROJECT: COMPARE CUBE-SHAPED BUILDINGS
Envision Math Common Core 6th Grade Answer Key Topic 7 Solve Area, Surface Area, And Volume Problems 10

PROJECT 7B
What are some ways you can use number cubes?
PROJECT: DRAW NETS OF NUMBER CUBES
Envision Math Common Core 6th Grade Answer Key Topic 7 Solve Area, Surface Area, And Volume Problems 11

PROJECT 7C
How far do you think a roll of wrapping paper reaches?
PROJECT: CALCULATE THE AMOUNT OF GIFT WRAP
Envision Math Common Core 6th Grade Answer Key Topic 7 Solve Area, Surface Area, And Volume Problems 12

PROJECT 7D
What sorts of music do you like?
PROJECT: WRITE A RAP BATTLE
Envision Math Common Core 6th Grade Answer Key Topic 7 Solve Area, Surface Area, And Volume Problems 13

Lesson 7.1 Find Areas of Parallelograms and Rhombuses

Solve & Discuss It!
Sofía drew the grid below and plotted the points A, B, C, and D. Connect point A to B, B to C, C to D, and D to A. Then find the area of the shape and explain how you found it. Using the same grid, move points B and C four units to the right. Connect the points to make a new parallelogram ABCD. What is the area of this shape?

I can… use what I know about areas of rectangles to find the areas of parallelograms and rhombuses.
Envision Math Common Core 6th Grade Answer Key Topic 7 Solve Area, Surface Area, And Volume Problems 14

Look for Relationships
What relationships do you see between rectangles and parallelograms?

Focus on math practices
Generalize How can you find the area of any parallelogram?

Answer:
Area of the parallelogram = b x h.

Explanation:
In the above-given question,
given that,
the area of the parallelogram = base x height.
area of the parallelogram = b x h.

Essential Question
How can you use the area formula of a rectangle to find the area formula of a parallelogram?

Try It!

Find the area of the parallelogram.
Envision Math Common Core 6th Grade Answer Key Topic 7 Solve Area, Surface Area, And Volume Problems 15
The area of the parallelogram is ________ cm2.

Answer:
The area of the parallelogram = 31.5 sq cm.

Explanation:
In the above-given question,
given that,
area of the parallelogram = b x h.
area = 7 cm x 4.5 cm.
where b = base and h = height.
area = 31.5 sq cm.

Convince Me! Compare the area of this parallelogram to the area of a rectangle with a length of 7 cm and a width of 4.5 cm. Explain.

Answer:
Area = 15.75 sq cm.

Explanation:
In the above-given question,
given that,
area of the rectangle = 1/2( b x h).
area =1/2( 7 cm x 4.5 cm).
area = 1/2(31.5).
area = 15.75.

Try It!

a. Find the area of the rhombus.
Envision Math Common Core 6th Grade Answer Key Topic 7 Solve Area, Surface Area, And Volume Problems 16

Answer:
Area of the rhombus = 21.25.

Explanation:
In the above-given question,
given that,
area of the rhombus = pq/2.
area = 22.5 + 20/2.
area = 42.5/2.
area = 21.25.

b. The area of the parallelogram is 65 ft2. What is its height?
Envision Math Common Core 6th Grade Answer Key Topic 7 Solve Area, Surface Area, And Volume Problems 17

Answer:
The area of the parallelogram = 5 ft.

Explanation:
In the above-given question,
given that,
area = 65 sq ft.
area = b x h.
65 = 13 x h.
h = 65/13.
h = 5.
so the area of the parallelogram = 5 ft.

KEY CONCEPT
You can decompose a parallelogram and compose a rectangle to find the area of a parallelogram or a rhombus. The formula for the area of a rectangle, A = l × w, can be written as the formula A = b × h to find the area of a parallelogram or the area of a rhombus.
Envision Math Common Core 6th Grade Answer Key Topic 7 Solve Area, Surface Area, And Volume Problems 18

Do You Understand?
Question 1.
Essential Question How can you use the area formula of a rectangle to find the area formula of a parallelogram?

Answer:
The area of the rectangle = 30 sq ft.
The area of the parallelogram = 30 sq ft.

Explanation:
In the above-given question,
given that,
the area of the rectangle = l x w.
area = length x width.
for example:
area = 6 ft x 5 ft.
area = 30 sq ft.
the area of the parallelogram = b x h.
area = 6 ft x 5 ft.
area = 30 sq ft.

Question 2.
Ken combined a triangle and a trapezoid to make a parallelogram. If the area of the triangle is 12 in.2 and the area of the trapezoid is 24 in.2, what is the area of the parallelogram? Explain.

Answer:
The area of the parallelogram =

Explanation:
In the above-given question,
given that,
Ken combined a triangle and a trapezoid to make a parallelogram.
If the area of the triangle is 12 in.2and the area of the trapezoid is 24 in.2.
area of the parallelogram = b x h.
area = 12 x 24.
area = 288 sq in.
so the area of the parallelogram = 288 sq in.

Question 3.
Critique Reasoning A parallelogram is 3 meters long and 7 meters high. Liam said that the parallelogram’s area is greater than the area of a rectangle with the same dimensions. Is he correct? Explain.

Answer:
Yes, he was correct.

Explanation:
In the above-given question,
given that,
A parallelogram is 3 meters long and 7 meters high.
area of the parallelogram = b x h.
area = 7 x 3.
area = 21.
so he was correct.

Do You Know How?
In 4-6, use a formula to find the area.
Question 4.
Envision Math Common Core 6th Grade Answer Key Topic 7 Solve Area, Surface Area, And Volume Problems 19

Answer:
The area of the parallelogram = 430.

Explanation:
In the above-given question,
given that,
the area of the parallelogram = b x h.
area = 20 x 21.5.
area = 430.
so the area of the parallelogram = 430.

Question 5.
Envision Math Common Core 6th Grade Answer Key Topic 7 Solve Area, Surface Area, And Volume Problems 20

Answer:
The area of the parallelogram = 200.

Explanation:
In the above-given question,
given that,
the area of the parallelogram = b x h.
area = 16 x 12.5.
area = 200.
so the area of the parallelogram = 200.

Question 6.
Envision Math Common Core 6th Grade Answer Key Topic 7 Solve Area, Surface Area, And Volume Problems 21

Answer:
The area of the parallelogram = 120.

Explanation:
In the above-given question,
given that,
the area of the parallelogram = b x h.
area = 12 x 10.
area = 120.
so the area of the parallelogram = 120.

Question 7.
A rhombus has an area of 440 m2 and a base of 22 m. What is its height?

Answer:
The diagonal of the rhombus = 40.

Explanation:
In the above-given question,
given that,
A rhombus has an area of 440 sq m and a base of 22 m.
area of the rhombus = pq/2.
where p = diagonal and q = diagonal.
area = pq/2.
440 = px 22/2.
440 = 11p.
p = 440/11.
p = 40.

Practice & Problem Solving

Leveled Practice In 8–11, find the area of each parallelogram or rhombus.
Question 8.
Envision Math Common Core 6th Grade Answer Key Topic 7 Solve Area, Surface Area, And Volume Problems 22

Answer:
The area of the parallelogram = 12 sq yd.

Explanation:
In the above-given question,
given that,
area of the quadrilateral = b x h.
where b = base and h = height.
area = 2 x 6.
area = 12 sq yd.

Question 9.
Envision Math Common Core 6th Grade Answer Key Topic 7 Solve Area, Surface Area, And Volume Problems 23

Answer:
Area = 24 sq m.

Explanation:
In the above-given question,
given that,
area of the parallelogram = b x h.
where b = base and h = height.
area = 6 x 4.
area = 24 sq m.

Question 10.
Envision Math Common Core 6th Grade Answer Key Topic 7 Solve Area, Surface Area, And Volume Problems 24

Answer:
The area of the rhombus = 52.5.

Explanation:
In the above-given question,
given that,
area of the rhombus = pq/2.
where p = diagonal and q = diagonal.
area = pq/2.
area = 10.5 x 10 /2.
area = 52.5.

Question 11.
Envision Math Common Core 6th Grade Answer Key Topic 7 Solve Area, Surface Area, And Volume Problems 25
Answer:
Area = 20.7 sq yd.

Explanation:
In the above-given question,
given that,
area of the parallelogram = b x h.
where b = base and h = height.
area = 9 x 7/3.
area = 9 x 2.3.
area = 20.7 sq yd.

Question 12.
The area of the parallelogram is 132 in.2. What is the height of the parallelogram?
Envision Math Common Core 6th Grade Answer Key Topic 7 Solve Area, Surface Area, And Volume Problems 26

Answer:
height = 12 in.

Explanation:
In the above-given question,
given that,
area of the parallelogram = b x h.
where b = base and h = height.
132 = 11 x h.
h = 12  in.

Question 13.
The area of the rhombus is 52 m2 What is the base of the rhombus?
Envision Math Common Core 6th Grade Answer Key Topic 7 Solve Area, Surface Area, And Volume Problems 27

Answer:
The diagonal of the rhombus = 16.

Explanation:
In the above-given question,
given that,
A rhombus has an area of 52 sq m and a base of 6.5 m.
area of the rhombus = pq/2.
where p = diagonal and q = diagonal.
area = pq/2.
52 = px 6.5/2.
52 = 3.25p.
p = 52/3.25.
p = 16.

Question 14.
Micah and Jason made parallelogram-shaped stained glass windows with the same area. The height of Micah’s window is 9 inches, and its base is 10 inches. The height of Jason’s window is 6 inches. What is the base of Jason’s window?
Envision Math Common Core 6th Grade Answer Key Topic 7 Solve Area, Surface Area, And Volume Problems 28

Answer:
The base of Jason’s window = 15 in.

Explanation:
In the above-given question,
given that,
Micah and Jason made parallelogram-shaped stained glass windows with the same area.
The height of Micah’s window is 9 inches, and its base is 10 inches.
The height of Jason’s window is 6 inches.
area = b x h.
90 = b x 6.
b = 90/6.
b = 15.
so the base of Jason’s window = 15 in.

Question 15.
A rectangle has a length of 8 m and a width of 4.5 m. A parallelogram has a length of 6 m. The area of the parallelogram is twice the area of the rectangle. What is the height of the parallelogram?

Answer:
The height of the parallelogram = 3m.

Explanation:
In the above-given question,
given that,
A rectangle has a length of 8 m and a width of 4.5 m.
A parallelogram has a length of 6 m.
area of rectangle = 8 x 4.5.
area = 36.
area of parallelogram = 6 x h.
18 = 6 x h.
h = 18/6.
h = 3.

In 16 and 17, use the picture at the right.
Envision Math Common Core 6th Grade Answer Key Topic 7 Solve Area, Surface Area, And Volume Problems 29
Question 16.
Hilary made an origami dog. What is the area of the parallelogram that is highlighted in the origami figure?

Answer:
The area of the parallelogram = 9.44 sq cm.

Explanation:
In the above-given question,
given that,
area of the parallelogram = b x h.
area = 4 cm x 2.36 cm.
b = base and h = height.
area = 9.44 sq cm.

Question 17.
A type of origami paper comes in 15 cm by 15 cm square sheets. Hilary used two sheets to make the origami dog. What is the total area of the origami paper that Hilary used to make the dog?

Answer:
The total area of the origami papaer = 225 sq cm.

Explanation:
In the above-given question,
given that,
A type of origami paper comes in 15 cm by 15 cm square sheets.
Hilary used two sheets to make the origami dog.
the total area of the origami paper = 15 x 15.
area = 225 sq cm.
so the total area of the origami paper = 225 sq cm.

Question 18.
Reasoning A rectangle and a parallelogram have the same base and the same height. How are their areas related? Provide an example to justify your answer.

Answer:
area = 1/2 x bh.

Explanation:
In the above-given question,
given that,
A rectangle and a parallelogram have the same base and the same height.
area of the rectangle = 1/2 b x h.
area = half of the base x height.
area of the parallelogram = l x w.
where l = length and w = weight.

Question 19.
Soshi’s rhombus has a base of 12 in. and a height of 10 in. Jack’s rhombus has base and height measures that are double those of Soshi’s rhombus. Compare the area of Jack’s rhombus to the area of Soshi’s rhombus. Explain.

Answer:
The area of Jack’s rhombus to the area of Soshi’s rhombus = 120.

Explanation:
In the above-given question,
given that,
Soshi’s rhombus has a base of 12 in. and a height of 10 in.
Jack’s rhombus has base and height measures that are double those of Soshi’s rhombus.
area = pq/2.
area = 12 x 10/2.
area = 120/2.
area = 60.
so the area of Jack’s rhombus to the area of Soshi’s rhombus = 120.

Question 20.
Higher Order Thinking The infield of a baseball diamond is in the shape of a rhombus. An infield cover with dimensions of 85 feet by 100 feet is used to protect the field during rainy weather. Will the cover protect the entire infield? Explain.
Envision Math Common Core 6th Grade Answer Key Topic 7 Solve Area, Surface Area, And Volume Problems 30

Answer:
The area of the rhombus = 90 ft.

Explanation:
In the above-given question,
given that,
The infield of a baseball diamond is in the shape of a rhombus.
An infield cover with dimensions of 85 feet by 100 feet is used to protect the field during rainy weather.
area of the rhombus = pq/2.
area = 90 x 90/2.
area = 180/2.
area = 90.
so the area of the rhombus = 90 ft.

Assessment Practice

Question 21.
The parking space shown at the right has an area of 209 ft2. A custom truck has rectangular dimensions of 13.5 ft by 8.5 ft. Can the truck fit in the parking space? Justify your answer.
Envision Math Common Core 6th Grade Answer Key Topic 7 Solve Area, Surface Area, And Volume Problems 31

Answer:
Yes, the truck fits in the parking space.

Explanation:
In the above-given question,
given that,
The parking space shown at the right has an area of 209 ft2.
A custom truck has rectangular dimensions of 13.5 ft by 8.5 ft.
area of the parallelogram = b x h.
209 = b x 19.
b = 209/19.
b = 11.
the rectangular dimensions of 13.5 ft by 8.5 ft.
area = 13.5 x 8.5 ft.
area = 114.75.

Lesson 7.2 Solve Triangle Area Problems

Solve & Discuss It!
Connect point A to B, B to C, C to D, and D to A. Then draw a diagonal line connecting opposite vertices in the figure and find the area of each triangle formed.
I can… find the areas of triangles.
Envision Math Common Core 6th Grade Answer Key Topic 7 Solve Area, Surface Area, And Volume Problems 32

Use Structure
What relationships do you see between the area of the parallelogram and the areas of the triangles?

Focus on math practices
Generalize What is a rule for finding the area of any triangle?

Answer:
The area of the triangle = b x h/2.

Explanation:
In the above-given question,
given that,
the area of the triangle = b x h/2.
where b = base.
h = height.
so the area of the triangle = b x h/2.

Essential Question
How can you find the area of a triangle?

Try It!

Use the formula A = \(\frac{1}{2}\)bh to find the area of the triangle.
Envision Math Common Core 6th Grade Answer Key Topic 7 Solve Area, Surface Area, And Volume Problems 33
The area of the triangle is _________ cm2

Answer:
The area of the triangle = 30 sq cm.

Explanation:
In the above-given question,
given that,
area = 1/2 x b x h.
area = 1/2 x 12 x 5.
area = 1/2 x 60.
area = 30.
so the area of the triangle = 30 sq cm.

Convince Me! Two identical triangles form a parallelogram with a base of 8 inches and a height of 6 inches. What is the area of each triangle? Explain.

Try It!

Find the area of each triangle.
a.
Envision Math Common Core 6th Grade Answer Key Topic 7 Solve Area, Surface Area, And Volume Problems 34

Answer:
Area = 27 sq cm.

Explanation:
In the above-given question,
given that,
area of the triangle = 1/2 x b x h.
area = 1/2 x 9cm x 6cm.
area = 1/2 x 54.
area = 27 sq cm.

b.
Envision Math Common Core 6th Grade Answer Key Topic 7 Solve Area, Surface Area, And Volume Problems 35

Answer:
Area = 18 sq m.

Explanation:
In the above-given question,
given that,
area of the triangle = 1/2 x b x h.
area = 1/2 x 12m x 3m.
area = 1/2 x 36.
area = 18 sq m.

KEY CONCEPT
You can use the formula A = \(\frac{1}{2}\)bh to find the area of any triangle.
Envision Math Common Core 6th Grade Answer Key Topic 7 Solve Area, Surface Area, And Volume Problems 36

Do You Understand?
Question 1.
Essential Question How can you find the area of a triangle?

Answer:
The area of the triangle = 1/2bh.

Explanation:
In the above-given question,
given that,
to find the area of any triangle, we can use the formula.
area = 1/2 bh.
so the area of the triangle = 1/2bh.

Question 2.
Reasoning If you cut a rectangle into 2 identical triangles, what type of triangles will they be?

Answer:
The triangles formed are right-angled triangles.

Explanation:
In the above-given question,
given that,
the area of any triangle, we can use the formula.
area = 1/2 bh.
so the area of the triangle = 1/2bh.
the triangles formed are right-angled triangles.

Question 3.
Construct Arguments In Example 1, if the other diagonal were used to divide the parallelogram into two triangles, would the area of each of these triangles be half the area of the parallelogram? Explain.

Answer:
Yes, the area of each of these triangles is half the area of the parallelogram.

Explanation:
In the above-given question,
given that,
the area of the parallelogram = b x h.
where b = base.
h = height.
so the area of each of these triangles is half the area of the parallelogram.

Do You Know How?
In 4-6, find the area of each triangle.
Question 4.
Envision Math Common Core 6th Grade Answer Key Topic 7 Solve Area, Surface Area, And Volume Problems 37

Answer:
Area = 4 sq ft.

Explanation:
In the above-given question,
given that,
area of the triangle = 1/2 x b x h.
area = 1/2 x 2ft x 4ft.
area = 1/2 x 8.
area = 4 sq ft.

Question 5.
Envision Math Common Core 6th Grade Answer Key Topic 7 Solve Area, Surface Area, And Volume Problems 38

Answer:
Area = 7.35 sq in.

Explanation:
In the above-given question,
given that,
area of the triangle = 1/2 x b x h.
area = 1/2 x 4.2 x 3.5.
area = 1/2 x 14.7.
area = 7.35 sq in.

Question 6.
Envision Math Common Core 6th Grade Answer Key Topic 7 Solve Area, Surface Area, And Volume Problems 39

Answer:
Area = 16.25 sq cm.

Explanation:
In the above-given question,
given that,
area of the triangle = 1/2 x b x h.
area = 1/2 x 5 cm x 6.5 cm.
area = 1/2 x 32.5.
area = 16.25 sq cm.

Practice & Problem Solving

Leveled Practice In 7-12, find the area of each triangle.
Question 7.
Envision Math Common Core 6th Grade Answers Topic 7 Solve Area, Surface Area, And Volume Problems 40

Answer:
Area = 12 sq yd.

Explanation:
In the above-given question,
given that,
area of the triangle = 1/2 x b x h.
area = 1/2 x 6 yd x 4 yd.
area = 1/2 x 24 sq yd.
area = 12 sq yd.

Question 8.
Envision Math Common Core 6th Grade Answers Topic 7 Solve Area, Surface Area, And Volume Problems 41

Answer:
Area = 15 sq m.

Explanation:
In the above-given question,
given that,
area of the triangle = 1/2 x b x h.
area = 1/2 x 6m x 5m.
area = 1/2 x 30.
area = 15 sq m.

Question 9.
Envision Math Common Core 6th Grade Answers Topic 7 Solve Area, Surface Area, And Volume Problems 42

Answer:
Area = 91 sq cm.

Explanation:
In the above-given question,
given that,
area of the triangle = 1/2 x b x h.
area = 1/2 x 14 cm x 13 cm.
area = 1/2 x 182.
area = 91 sq cm.

Question 10.
Envision Math Common Core 6th Grade Answers Topic 7 Solve Area, Surface Area, And Volume Problems 43

Answer:
Area = 90 sq in.

Explanation:
In the above-given question,
given that,
area of the triangle = 1/2 x b x h.
area = 1/2 x 15 in x 12 in.
area = 1/2 x 180.
area = 90 sq in.

Question 11.
Envision Math Common Core 6th Grade Answers Topic 7 Solve Area, Surface Area, And Volume Problems 44

Answer:
Area = 54 sq cm.

Explanation:
In the above-given question,
given that,
area of the triangle = 1/2 x b x h.
area = 1/2 x 12 cm x 9 cm.
area = 1/2 x 108.
area = 54 sq cm.

Question 12.
Envision Math Common Core 6th Grade Answers Topic 7 Solve Area, Surface Area, And Volume Problems 45

Answer:
Area = 24 sq m.

Explanation:
In the above-given question,
given that,
area of the triangle = 1/2 x b x h.
area = 1/2 x 8m x 6m.
area = 1/2 x 48.
area = 24 sq m.

Question 13.
The vertices of a triangle are A(0, 0), B(3, 8), and C(9, 0). What is the area of this triangle?

Answer:
The area of this triangle = -4.

Explanation:
In the above-given question,
given that,
the vertices of a triangle are A(0,0), B(3, 8), and C(9, 0).
x1 = 0, x2 = 3, x3 = 9.
y1 = 0, y2 = 8, y3 = 0.
area = 1/2[x1(y2-y3) + x2(y3 – y1) + x3(y1 – y2)].
area = 1/2[0(8) + 3(0) + 9(-8)].
arae = 1/2(-8).
area = -4.

Question 14.
Be Precise The base of a triangle is 2 ft. The height of the triangle is 15 in. What is the area of the triangle in square inches?

Answer:
Area of the triangle = 144 sq in.

Explanation:
In the above-given question,
given that,
The base of a triangle is 2 ft.
The height of the triangle is 15 in.
1 ft = 12 in.
2 ft = 24 in.
area = b x h.
area = 1/2 x 24 x 12.
area = 1/2(288).
area = 144 sq in.

Question 15.
Reasoning Ms. Lopez drew △ ABC, with a height of 6 inches and a base of 6 inches, and △RST, with a height of 4 inches and a base of 8 inches. Which triangle has the greater area? Use an area formula to justify your answer.

Answer:
Triangle ABC has the greater area.

Explanation:
In the above-given question,
given that,
Ms. Lopez drew △ ABC, with a height of 6 inches and a base of 6 inches, and △RST, with a height of 4 inches and a base of 8 inches.
area of the trianglee ABC = 1/2 x 6  x 6.
where b = base and h = height.
area = 1/2 x 36.
area = 18 sq in.
area of the triangle RST = 1/2 x 8 x 4.
area = 1/2 x 32.
area = 16 sq in.
so the triangle ABC has the greater area.

Question 16.
The dimensions of the sail for Erica’s sailboat are shown. Find the area of the sail.
Envision Math Common Core 6th Grade Answers Topic 7 Solve Area, Surface Area, And Volume Problems 46

Answer:
Area of the sail = 67.5 sq ft.

Explanation:
In the above-given question,
given that,
area of the sail = 1/2 x b x h.
area = 1/2 x 9 x 15.
area = 1/2 x 135.
area = 67.5 sq ft.
so the area of the sail = 67.5 sq ft.

In 17 and 18, use the picture at the right.
Envision Math Common Core 6th Grade Answers Topic 7 Solve Area, Surface Area, And Volume Problems 47
Question 17.
Be Precise What is the area in square millimeters of the yellow triangle outlined on the origami figure at the right?

Answer:
Area = 2.64.

Explanation:
In the above-given question,
given that,
b = 3 cm.
h = 1.76 cm.
area = 1/2 x b x h.
area = 1/2 x 3 x 1.76.
area = 5.28/2.
area = 2.64.

Question 18.
The nose of the origami dog is a right triangle with sides that are 2 cm, 3 cm, and 3.6 cm long. What is the area of this triangle?

Answer:
Area = 3 sq cm.

Explanation:
In the above-given question,
given that,
b = 3 cm.
h = 2 cm.
area = 1/2 x b x h.
area = 1/2 x 3 x 2.
area = 6/2.
area = 3 sq cm.

Question 19.
Michael is planting a garden in the shape of a right triangle. He wants 4 tickseed plants for each square meter of area. How many 25 m tickseed plants does Michael want in the garden?
Envision Math Common Core 6th Grade Answers Topic 7 Solve Area, Surface Area, And Volume Problems 48

Answer:
The 25 m tickseed plants does Michae want in the garden = 84.

Explanation:
In the above-given question,
given that,
Michael is planting a garden in the shape of a right triangle.
He wants 4 tickseed plants for each square meter of area.
area = 1/2 x b x h.
where b = base.
h = height.
area = 1/2 x 7 x 24.
area = 168/2.
area = 84.
so the 25 m tickseed plants does Michae want in the garden = 84.

Question 20.
Higher Order Thinking If you know the area and the height of a triangle, how can you find the base?
Answer:

Assessment Practice

Question 21.
Select all expressions that represent the area of the given triangle.
Envision Math Common Core 6th Grade Answers Topic 7 Solve Area, Surface Area, And Volume Problems 49

Answer:
Options A and E are correct.

Explanation:
In the above-given question,
given that,
area = 1/2 x 16 x 12.
where b = 16 cm.
height = 12 cm.
area = 1/2 x 192.
area = 96.
so options A and E are correct.

Lesson 7.3 Find Areas of Trapezoids and Kites

Explain It!
The European basketball key was changed from a trapezoid shape to a rectangle in 2010. The diagram shows the shape of the key before 2010 outlined in blue.

I can… find areas of trapezoids and kites.
Envision Math Common Core 6th Grade Answers Topic 7 Solve Area, Surface Area, And Volume Problems 50

A. Construct Arguments Tim finds the area of the key by multiplying the base by the height. Does his strategy make sense?

Answer:
Yes.

Explanation:
In the above-given question,
given that,
area of the triangle = 1/2 x b x h.
where b = base.
h = height.
area = 1/2 x 6 x 5.8.
area = 1/2 x 34.8.
area = 17.4.

B. Use Structure How could Tim find the area of the trapezoid by decomposing it into shapes he knows? What is the area of the key?
Answer:

Focus on math practices
Use Structure How can you find the area of this kite? Explain.
Envision Math Common Core 6th Grade Answers Topic 7 Solve Area, Surface Area, And Volume Problems 51

Essential Question
How can you find the areas of trapezoids and kites?

Try It!

How would you decompose this trapezoid to find its area? Find the area of the trapezoid.
Envision Math Common Core 6th Grade Answers Topic 7 Solve Area, Surface Area, And Volume Problems 52
Answer:
The area of the trapezoid = 22.5 sq ft.

Explanation:
In the above-given question,
given that,
area of the trapezoid = 11.25 + 5 + 6.25.
area = 22.5.
so the area of the trapezoid = 22.5 sq ft.

Convince Me! How is finding the area of the trapezoid in Example 1 different from finding the area of the trapezoid in the Try It!?

Try It!

Find the area of the trapezoid and the area of the kite.
a.
Envision Math Common Core 6th Grade Answers Topic 7 Solve Area, Surface Area, And Volume Problems 53

Answer:
Area of the trapezoid = 144 sq cm.

Explanation:
In the above-given question,
given that,
area = 24 cm + 24 cm + 96.
area = 48 cm + 96.
area = 144 sq cm.

b.
Envision Math Common Core 6th Grade Answers Topic 7 Solve Area, Surface Area, And Volume Problems 54

Answer:
Area of the kite = 72 sq in.

Explanation:
In the above-given question,
given that,
each triangle = 1/2 x 24 x 3.
triangle = 1/2 x 72.
triangle = 36 sq in.
Kite = 36 + 36.
kite = 72 sq in.

KEY CONCEPT
You can find the area of a trapezoid or a kite by decomposing the shapes into rectangles and triangles.
Envision Math Common Core 6th Grade Answers Topic 7 Solve Area, Surface Area, And Volume Problems 55

Do You Understand?
Question 1.
Essential Question How can you find the areas of trapezoids and kites?

Answer:
Area of the trapezoid = area of the triangle + area of the rectangle.
area of the kite = area of the triangle.

Explanation:
In the above-given question,
given that,
area of the trapezoid = area of the triangle + area of the rectangle.
area of the kite = area of the triangle.

Question 2.
Draw a line to divide the pasture in Example 1 into two triangles. What are the measures of the bases and the heights of the two triangles?
Answer:

Question 3.
Construct Arguments In Example 3, how could you use 4 triangles to find the kite’s area?
Answer:

Do You Know How?
In 4-6, find the area of each trapezoid or kite.
Question 4.
Envision Math Common Core 6th Grade Answers Topic 7 Solve Area, Surface Area, And Volume Problems 56

Answer:
Area of the kite = 16 sq m.

Explanation:
In the above-given question,
given that,
area of the triangle = 1/2 x b x h.
where b = 2m and height = 8 m.
area = 1/2 x 2 x 8.
area = 16/2.
area = 8 sq m.
area of the kite = 8 + 8.
area = 16 sq m.

Question 5.
Envision Math Common Core 6th Grade Answers Topic 7 Solve Area, Surface Area, And Volume Problems 57

Answer:
Area of the kite = 6 sq cm.

Explanation:
In the above-given question,
given that,
area of the triangle = 1/2 x b x h.
where b = 2cm and height = 3 cm.
area = 1/2 x 2 x 3.
area = 6/2.
area = 3 sq cm.
area of the kite = 3 + 3.
area = 6 sq cm.

Question 6.
Envision Math Common Core 6th Grade Answers Topic 7 Solve Area, Surface Area, And Volume Problems 58

Answer:
Area of the kite = 90 sq ft.

Explanation:
In the above-given question,
given that,
area of the triangle = 1/2 x b x h.
where b = 15 and height = 6 ft.
area = 1/2 x 15 x 6.
area = 90/2.
area = 45 sq ft.
area of the kite = 45 + 45.
area = 90 sq ft.

Practice & Problem Solving

Leveled Practice In 7-12, find the area of each trapezoid or kite.
Question 7.
Envision Math Common Core 6th Grade Answers Topic 7 Solve Area, Surface Area, And Volume Problems 59

Answer:
Area of the trapezoid = 168 sq cm.

Explanation:
In the above-given question,
given that,
area of the trapezoid = area of each triangle + area of the rectangle.
area of the triangle = 1/2 x h x b.
area = 1/2 x 12 x 8.
area = 1/2 x 96.
area = 48.
area of the rectangle = l x w.
area = 9 x 8.
area = 72.
area of the trapezoid = 48 + 48 + 72.
area = 168 sq cm.

Question 8.
Envision Math Common Core 6th Grade Answers Topic 7 Solve Area, Surface Area, And Volume Problems 60

Answer:
Area of the trapezoid = 54 sq cm.

Explanation:
In the above-given question,
given that,
area of the trapezoid = area of each triangle + area of the rectangle.
area of the triangle = 1/2 x h x b.
area = 1/2 x 6 x 4.
area = 1/2 x 24.
area = 12.
area of the rectangle = l x w.
area = 5 x 6.
area = 30.
area of the trapezoid = 12 + 12 + 30.
area = 24 + 30 sq cm.
area = 54 sq cm.

Question 9.
Envision Math Common Core 6th Grade Answers Topic 7 Solve Area, Surface Area, And Volume Problems 61

Answer:
Area of the kite = 33 sq cm.

Explanation:
In the above-given question,
given that,
area of the kite = area of the triangle + area of the triangle.
area of the triangle = 1/2 x b x h.
where b = base and h = height.
area = 1/2 x 11 x 3.
area = 1/2 x 33.
area = 16.5.
area of the kite = 16.5 + 16.5.
area = 33.

Question 10.
Envision Math Common Core 6th Grade Answers Topic 7 Solve Area, Surface Area, And Volume Problems 62

Answer:
Area of the kite = 112 sq yd.

Explanation:
In the above-given question,
given that,
area of the kite = area of the triangle + area of the triangle.
area of the triangle = 1/2 x b x h.
where b = base and h = height.
area = 1/2 x 14 x 8.
area = 1/2 x 112.
area = 56.
area of the kite = 56 + 56.
area = 112 sq yd.

Question 11.
A sidewall of a building is shown below. What is the area of the wall?
Envision Math Common Core 6th Grade Answers Topic 7 Solve Area, Surface Area, And Volume Problems 63

Answer:
Area of the trapezoid = 1020 sq ft.

Explanation:
In the above-given question,
given that,
area of the trapezoid = area of the triangle + area of the rectangle.
area of the triangle = 1/2 x b x h.
area = 1/2 x 6 x 30.
area = 180/2.
area = 90.
area of the rectangle = l x w.
area = 28 x 30.
area = 840.
area of the trapezoid = 90 + 90 + 840.
area = 180 + 840.
area = 1020.

Question 12.
Be Precise The window has the shape of a kite. How many square meters of glass were used to make the window?
Envision Math Common Core 6th Grade Answers Topic 7 Solve Area, Surface Area, And Volume Problems 64

Answer:
The area of the Kite = 1225 sq cm.

Explanation:
In the above-given question,
given that,
area of the kite = area of the triangle.
area of the triangle = 1/2 x b x h.
area = 1/2 x 70 x 35.
area = 2450/2.
area = 1225.
area of the kite = 1225 + 1225.
area = 1225 sq cm.

Question 13.
The area of the kite is 30 m2. What is the value of x? Explain.
Envision Math Common Core 6th Grade Answers Topic 7 Solve Area, Surface Area, And Volume Problems 65

Answer:
The value of x = 6 m.

Explanation:
In the above-given question,
given that,
the area of the kite = 30 sq m.
area of the triangle = 1/2 x b x h.
30 = 1/2 x 10 x h.
30 = 5h.
h = 30/5.
h = 6.

Question 14.
Make Sense and Persevere Hunter drew two identical trapezoids and composed them to form a parallelogram. Use the area of the parallelogram to find the area of one trapezoid. Explain.
Envision Math Common Core 6th Grade Answers Topic 7 Solve Area, Surface Area, And Volume Problems 66

Answer:
The area of the trapezoid = 36 sq ft.

Explanation:
In the above-given question,
given that,
area of the parallelogram = b x h.
area = 9 x 4.
area = 36 sq ft.

Question 15.
Higher Order Thinking A craftsman wants to build this symmetrical fiddle. He needs to know the area of the face of the fiddle. How could he use the measurements shown to find the area? Use your strategy to find the area of the face of the fiddle.
Envision Math Common Core 6th Grade Answers Topic 7 Solve Area, Surface Area, And Volume Problems 67

Answer:
The area of the fiddle = 35208 sq mm.

Explanation:
In the above-given question,
given that,
area of the fiddle = 1/2 x 326 x 216.
area = 1/2 x 70416.
area = 35208.
so the area of the fiddle = 35208 sq mm.

Assessment Practice

Question 16.
Marique is making a large table in the shape of a trapezoid. She needs to calculate the area of the table. The longest side of the table is twice as long as the table’s width. Find the area of the table by decomposing the trapezoid into familiar shapes. Show your work.
Envision Math Common Core 6th Grade Answers Topic 7 Solve Area, Surface Area, And Volume Problems 68

Answer:
The area of the trapezoid = 0.9375.

Explanation:
In the above-given question,
given that,
area of the trapezoid = area of the triangle + area of the rectangle.
area of the triangle = 1/2 x b x h.
area = 1/2 x 0.5 x 1.25.
area = 1/2 x 0.625.
area = 0.3125.
area of the rectangle = l x w.
area = 0.5 x 1.25.
area = 0.625.
area = 0.3125 + 0.625.
area = 0.9375.
so the area of the trapezoid = 0.9375.

Lesson 7.4 Find Areas of Polygons

Solve & Discuss It!
Gabrielle wants to cover the floors of a room and a hallway in her dollhouse. She measured the room and hallway and sketched the floor plan below. How much felt does Gabrielle need?
I can… find the areas of polygons.
Envision Math Common Core 6th Grade Answers Topic 7 Solve Area, Surface Area, And Volume Problems 69

Answer:
Area of the polygon = 205 sq in.

Explanation:
In the above-given question,
given that,
area of the polygon = area of the triangle + area of the triangle + area of the rectangle.
area = 1/2 x 15 x 10.
area = 1/2 x 150.
area = 75.
area = 1/2 x 22 x 5.
area = 1/2 x 110.
area = 55.
area of the polygon = 75 + 55 + 75.
area = 150 + 55.
area = 205.

Make Sense and Persevere
How can you decompose the sketch into regular shapes?

Focus on math practices
Make Sense and Persevere Show another way to find the area of the sketch.
Envision Math Common Core 6th Grade Answers Topic 7 Solve Area, Surface Area, And Volume Problems 70

Essential Question
How can you find the areas of polygons?

Try It!

Shari found the area of the patio by composing the shapes as shown at the right. How is Shari’s strategy different?
Envision Math Common Core 6th Grade Answers Topic 7 Solve Area, Surface Area, And Volume Problems 71

Answer:
Area of the polygon = 30 sq m.

Explanation:
In the above-given question,
given that,
area of the polygon = area of the triangle + area of the triangle + area of the rectangle.
area = 1/2 x 3 x 2.
area = 1/2 x 6.
area = 3.
area = 1/2 x 2 x 3.
area = 1/2 x 6.
area = 3.
area of the polygon = 3 + 3 + 24.
area = 6 + 24.
area = 30.

Convince Me! How could you decompose the figure in the Try It! into two rectangles?

Try It!

Find the area of the shaded region in square units.
Envision Math Common Core 6th Grade Answers Topic 7 Solve Area, Surface Area, And Volume Problems 72
Answer:

KEY CONCEPT
There are many ways to find the area of a polygon. You can decompose or compose shapes, or you can use addition or subtraction, to calculate the area.
Use Addition
Envision Math Common Core 6th Grade Answers Topic 7 Solve Area, Surface Area, And Volume Problems 73
Blue triangle: A = \(\frac{1}{2}\)(12)(6) = 36
Green triangle: A = \(\frac{1}{2}\)(8)(6) = 24
Rectangle: A = (12)(6) = 72
36 + 24 + 72 = 132
The area of the polygon is 132 cm2.

Use Subtraction
Envision Math Common Core 6th Grade Answers Topic 7 Solve Area, Surface Area, And Volume Problems 74
Draw a rectangle around the polygon.
Rectangle: A = 8 × 5 = 40
Triangle: A = \(\frac{1}{2}\) × 8 × 2 = 8
40 – 8 = 32
The area of the polygon is 32 square units.

Do You Understand?
Question 1.
Essential Question How can you find the areas of polygons?

Answer:
The area of the polygon = area of the triangle + area of the triangle + area of the rectangle.

Explanation:
In the above-given question,
given that,
area of the polygon = area of the triangle + area of the triangle + area of the rectangle.
area of the triangle = 1/2 x b x h.
where b = base.
h = height.
area of the rectangle = l x w.
where l = length.
w = width.
so the area of the polygon = area of the triangle + area of the triangle + area of the rectangle.

Question 2.
Describe a way in which you can use subtraction to find the area of the shape in Exercise 4.
Answer:

Question 3.
Model with Math Describe how to break the floor plan in Example 3 into a trapezoid and a rectangle. Use coordinates to describe the line you can draw.
Answer:

Do You Know How?
In 4 and 5, find the area of each polygon.
Question 4.
Envision Math Common Core 6th Grade Answers Topic 7 Solve Area, Surface Area, And Volume Problems 75

Answer:
Area of the polygon = 280 sq in.

Explanation:
In the above-given question,
given that,
area of the polygon = area of the small rectangle + area of the rectangle.
area of small rectangle = 8 x 10.
area = 80 sq in.
area of the rectangle = 20 x 10.
area = 200.
area of the polygon = 80 + 200.
area = 280 sq in.

Question 5.
A polygon with vertices at (6, 2), (9,5), (12, 2), (12, -4), and (6, -4)
Envision Math Common Core 6th Grade Answers Topic 7 Solve Area, Surface Area, And Volume Problems 76
Answer:

Practice & Problem Solving

In 6-9, find the area of each polygon or shaded region.
Question 6.
Envision Math Common Core 6th Grade Answers Topic 7 Solve Area, Surface Area, And Volume Problems 77

Answer:
Area of the polygon = 200 sq ft.

Explanation:
In the above-given question,
given that,
area of the polygon = area of the small rectangle + area of the small rectangle + area of the big rectangle.
area of the small rectangle = 15 x 3.
area = 45.
area of the another small rectangle = 15 x 4.
area = 60.
area of the big rectangle = 19 x 5.
area = 95.
area of the polygon = 45 + 60 + 95.
area of the polygon = 200.

Question 7.
Envision Math Common Core 6th Grade Answers Topic 7 Solve Area, Surface Area, And Volume Problems 78

Answer:
Area of the polygon = 232 sq cm.

Explanation:
In the above-given question,
given that,
area of the polygon = area of the small rectangle + area of the small rectangle + area of the big rectangle.
area of the small rectangle = 10 x 2.
area = 20.
area of the another small rectangle = 10 x 2.
area = 20.
area of the big rectangle = 16 x 12.
area = 192.
area of the polygon = 20 + 20 + 192.
area of the polygon = 232.

Question 8.
Envision Math Common Core 6th Grade Answers Topic 7 Solve Area, Surface Area, And Volume Problems 79

Answer:
Area of the polygon = 24 sq m.

Explanation:
In the above-given question,
given that,
area of the polygon = area of the small rectangle + area of the small rectangle + area of the big rectangle.
area of the small rectangle = 2 x 2.
area = 4.
area of the another small rectangle = 2 x 2.
area = 4.
area of the big rectangle = 4 x 4.
area = 16.
area of the polygon = 16 + 4 + 4.
area of the polygon = 24.

Question 9.
Envision Math Common Core 6th Grade Answers Topic 7 Solve Area, Surface Area, And Volume Problems 80

Answer:
Area of the polygon = 76 sq cm.

Explanation:
In the above-given question,
given that,
area of the polygon = area of the small triangle + area of the small triangle + area of the big rectangle.
area of the small triangle = 1/2(4 x 3).
area = 6 .
area of the another small rectangle = 1/2(3 x 4).
area = 6.
area of the big rectangle = 8 x 8.
area = 64.
area of the polygon = 64 + 6 + 6.
area of the polygon = 76.

In 10 and 11, find the area in square units of each polygon.
Question 10.
Envision Math Common Core 6th Grade Answers Topic 7 Solve Area, Surface Area, And Volume Problems 81

Answer:
Area of the polygon = 15 sq units.

Explanation:
In the above-given question,
given that,
area of the polygon = area of the small triangle + area of the small triangle + area of the big triangle.
area of the small triangle = 1 x 3.
area = 3.
area of the another small triangle = 1 x 3.
area = 3.
area of the big rectangle = 3 x 3.
area = 9.
area of the polygon = 3 + 3 + 9.
area of the polygon = 15.

Question 11.
Envision Math Common Core 6th Grade Answers Topic 7 Solve Area, Surface Area, And Volume Problems 82

Answer:
Area of the polygon = 15 sq units.

Explanation:
In the above-given question,
given that,
area of the polygon = area of the small triangle + area of the small triangle + area of the big triangle.
area of the small triangle = 1 x 3.
area = 3.
area of the another small triangle = 1 x 3.
area = 3.
area of the big rectangle = 3 x 3.
area = 9.
area of the polygon = 3 + 3 + 9.
area of the polygon = 15.

Question 12.
Be Precise Diego is designing an exercise room. How many square feet of rubber flooring will he need to cover the floor? The product is sold in whole square yards. How many square yards should Diego buy? Explain.
Envision Math Common Core 6th Grade Answers Topic 7 Solve Area, Surface Area, And Volume Problems 83
Answer:

In 13 and 14, use the diagram at the right.
Envision Math Common Core 6th Grade Answers Topic 7 Solve Area, Surface Area, And Volume Problems 85
Question 13.
David drew this diagram of a picture frame that he is going to make. Each square represents 1 square inch. What is the area of the picture frame?
Answer:

Question 14.
Use Structure How could you find the area of the picture frame without decomposing the frame into smaller shapes?
Answer:

Question 15.
Higher Order Thinking Isabella has three rectangular cards that are 4 inches by 5 inches. How can she arrange the cards, without overlapping, to make one larger polygon with the smallest possible perimeter? How will the area of the polygon compare to the combined area of the three cards?
Envision Math Common Core 6th Grade Answers Topic 7 Solve Area, Surface Area, And Volume Problems 84
Answer:

Assessment Practice

Question 16.
Select all expressions that can be used to find the area of the given polygon.
Envision Math Common Core 6th Grade Answers Topic 7 Solve Area, Surface Area, And Volume Problems 86
☐ (2 × 5) + (6 × 4)
☐ (5 × 2) + 2 • \(\frac{1}{2}\)(3 × 4)
☐ (6 × 5) – (3 × 4)
☐ ( 6× 9) – (3 × 4)
☐ (5 × 2) + \(\frac{1}{2}\)(2 + 2 + 2) × 4]

Answer:
Options A and E are correct.

Explanation:
In the above-given question,
given that,
(2 x 5) + (6 x 4).
10 + 24.
34.
so options A and E are correct.

Question 17.
What is the area of the polygon at the right?
Envision Math Common Core 6th Grade Answers Topic 7 Solve Area, Surface Area, And Volume Problems 87
A. 86 square units
B. 78 square units
C. 70 square units
D. 68 square units

Answer:
Option C is correct.

Explanation:
In the above-given question,
given that,
-5 x 8 = -40.
-5 x 6 = -30.
-40 + (-30).
70 square units.

Topic 7 Mid-Topic Checkpoint

Question 1.
Vocabulary How many pairs of opposite sides are parallel in a trapezoid? How is this different from a parallelogram? Lesson 7-3
Answer:

Question 2.
Calculate the area of each figure at the right. Which figure has the greatest area? Lessons 7-1, 7-2, and 7-3
Envision Math Common Core 6th Grade Answers Topic 7 Solve Area, Surface Area, And Volume Problems 88
A. triangle
B. parallelogram
C. trapezoid

Answer:
Option C is correct.

Explanation:
In the above-given question,
given that,
the polygon are triangle, parallelogram, and trapezoid.
area of the triangle = l x b.
area = 6 x 8.
area = 48.
area of the parallelogram = 8.5 x 4.
area = 34.
area of trapezoid = 3 x 4 + 9 x 6.
area = 12 + 54.
area = 66.
so option C is correct.

Question 3.
An earring has the shape of a rhombus. The height is 5.2 mm and the area of the earring is 39 mm2. What is the length of each side of the earring? Lesson 7-1

Answer:
The length of each side of the earring = 7.5 mm.

Explanation:
In the above-given question,
given that,
An earring has the shape of a rhombus.
The height is 5.2 mm and the area of the earring is 39 mm2.
area = l x h.
39 = l x 5.2.
l = 39/5.2.
l = 7.5.
so the length of each side of the earring = 7.5 mm.

Question 4.
Shane wrapped the kite at the right with paper. How many square inches of paper did Shane use to cover the front and back? Lesson 7-3
Envision Math Common Core 6th Grade Answers Topic 7 Solve Area, Surface Area, And Volume Problems 89

Answer:
Area = 288 sq in.

Explanation:
In the above-given question,
given that,
Shane wrapped the kite at the right with paper.
area of the rhombus = area of each triangle + area of kite.
area of triangle = 1/2(32)(9).
area of triangle = 288/2.
area = 144.
area of kite = 144 + 144.
area = 288.

Question 5.
Explain one way to find the area of the polygon at the right. Then find the area in square units. Lesson 7-4
Envision Math Common Core 6th Grade Answers Topic 7 Solve Area, Surface Area, And Volume Problems 90
Answer:

Topic 7 Mid-Topic Performance Task

Kira hires a contractor to build a new deck in her backyard. A design for the deck is shown.
Envision Math Common Core 6th Grade Answers Topic 7 Solve Area, Surface Area, And Volume Problems 91

PART A
Kira found the area of the deck by decomposing it into two identical trapezoids. Describe another strategy you could use to find the area of the deck.

Answer:
The area of the deck = 204 sq ft.

Explanation:
In the above-given question,
given that,
area of trapezoid = area of the triangle + area of the rectangle.
area of triangle = 1/2(12)
area = 6.
area of rectangle = 16 x 12.
area = 192.
area of trapezoid = 6 + 6 + 192.
area = 12 + 192.
area = 204.

PART B
Which expression can you use to find the area of the deck? Select all that apply.
☐ (16 × 12) + 2(\(\frac{1}{2}\) × 12 × 5)
☐ (26 × 12) + 2(\(\frac{1}{2}\) × 12 × 5)
☐ (26 × 12) – 4(\(\frac{1}{2}\) × 6 × 5)
☐ (16 × 6) + 2(\(\frac{1}{2}\) × 6 × 5) + (16 × 6) + 2(\(\frac{1}{2}\) × 6 × 5)
☐ (16 × 12) + 2(\(\frac{1}{2}\) × 12 × 10)

Answer:
Option A is correct.

Explanation:
In the above-given question,
given that,
area of the deck = area of the triangle + area of the rectangle.
area = 1/2 x 12.
area = 6.
area of rectangle = 16 x 12.
area = 192.
area of trapezoid = 6 + 6 + 192.
area = 12 + 192.
area = 204.

PART C
A second design for Kira’s deck is shown at the right. Find the areas of the two deck designs. Which design will give Kira the deck with the greater area? Explain.
Envision Math Common Core 6th Grade Answers Topic 7 Solve Area, Surface Area, And Volume Problems 92

Answer:
Area = 216 sq ft.

Explanation:
In the above-given question,
given that,
A second design for Kira’s deck is shown at the right.
area of the polygon = area of the triangle + area of the rectangle.
area of triangle = 1/2(8)(6).
area = 1/2(48).
area = 24.
area of rectangle = 6 x 16.
area = 96.
area of up rectangle = 12 x 8.
area = 96.
area of polygon = 24 + 96 + 96.
area = 216 sqft.

Lesson 7.5 Represent Solid Figures Using Nets

Explore It!
Jamal is breaking down items for recycling and wonders what this box will look like when it is unfolded and flat.

I can… represent solid figures using nets.
Envision Math Common Core 6th Grade Answers Topic 7 Solve Area, Surface Area, And Volume Problems 93

A. How do the sides of the box help you think about what the unfolded box will look like?
Answer:

B. How can you use the grid to represent the unfolded box?
Envision Math Common Core 6th Grade Answers Topic 7 Solve Area, Surface Area, And Volume Problems 94
Answer:

Focus on math practices
Reasoning is there another way to represent the unfolded box on the grid? Explain.

Essential Question
How do you classify and represent solid figures?

Try It!

Classify this solid figure.
Envision Math Common Core 6th Grade Answers Topic 7 Solve Area, Surface Area, And Volume Problems 95

Answer:
The figure is Triangular pyramid.

Explanation:
In the above-given question,
given that,
the figure is triangular pyramid.
volume of triangular pyramid = 1/3 x B X H.
volume = 1/3 x 1/2 x BH.
volume = 1/6 Bh.

Convince Me! What attributes of a solid figure should you identify to classify it as a polyhedron?

Try It!

Identify the solid from its net.
Envision Math Common Core Grade 6 Answer Key Topic 7 Solve Area, Surface Area, And Volume Problems 96
Shape of polygonal faces: ___________
Shape of base(s): ___________
This is a net of a ___________.

Answer:
The figure is a polyhedron.

Explanation:
In the above-given question,
given that,
shape of polygonal faces = 3.
shape od base = 3.
this is a net of polyhedron.

KEY CONCEPT
You can use nets to represent solid figures.
Envision Math Common Core Grade 6 Answer Key Topic 7 Solve Area, Surface Area, And Volume Problems 97

Do You Understand?
Question 1.
Essential Question How do you classify and represent solid figures?

Answer:
The solid figures are prism and pyramids.

Explanation:
In the above-given question,
given that,
prism has more than one base.
pyramid has only one base.
prism does not have any face.
pyramid has its face.

Question 2.
Use Structure Explain the difference between a vertex and an edge.

Answer:
Vertex is the highest point or apex.
Edge is the outside area or a surface.

Explanation:
In the above-given question,
given that,
vertex is the highest point or top or apex.
edge is the outside area or a surface.

Question 3.
Be Precise Explain the difference between a pyramid and a prism.

Answer:
The solid figures are prism and pyramids.

Explanation:
In the above-given question,
given that,
prism has more than one base.
pyramid has only one base.
prism does not have any face.
pyramid has its face.

Question 4.
Describe the net of a triangular prism.
Answer:

Do You Know How?
In 5 and 6, classify the solid figures.
Question 5.
Envision Math Common Core Grade 6 Answer Key Topic 7 Solve Area, Surface Area, And Volume Problems 98
Answer:
The figure is Triangular pyramid.

Explanation:
In the above-given question,
given that,
the figure is triangular pymarid.
volume = 1/6 Bh.
where b = base.
h = height.
so the figure is triangular pyramid.

Question 6.
Envision Math Common Core Grade 6 Answer Key Topic 7 Solve Area, Surface Area, And Volume Problems 99

Answer:
The figure is rectangular prism.

Explanation:
In the above-given question,
given that,
the figure is a rectangular prism.
volume of prism = b x h.

In 7 and 8, identify each solid from its net.
Question 7.
Envision Math Common Core Grade 6 Answer Key Topic 7 Solve Area, Surface Area, And Volume Problems 100

Answer:
The figure is pyramid.

Explanation:
In the above-given question,
given that,
the net is pyramid.
pyramid has face, base, and vertex.
so the figure is pyramid.

Question 8.
Envision Math Common Core Grade 6 Answer Key Topic 7 Solve Area, Surface Area, And Volume Problems 101
Answer:

Question 9.
Draw a net of a rectangular prism that has a height of 2 units and bases that are 3 units long and 1 unit wide.
Envision Math Common Core Grade 6 Answer Key Topic 7 Solve Area, Surface Area, And Volume Problems 102
Answer:

Practice & Problem Solving

In 10-12, classify the solid figures.
Question 10.
Envision Math Common Core Grade 6 Answer Key Topic 7 Solve Area, Surface Area, And Volume Problems 103

Answer:
The figure is rectangular prism.

Explanation:
In the above-given question,
given that,
the figure is a rectangular prism.
volume of prism = l x w x h.
where l = length.
w = width.
h = height.

Question 11.
Envision Math Common Core Grade 6 Answer Key Topic 7 Solve Area, Surface Area, And Volume Problems 104

Answer:
The figure is Triangular pyramid.

Explanation:
In the above-given question,
given that,
the figure is triangular pymarid.
volume = 1/6 Bh.
where b = base.
h = height.
so the figure is triangular pyramid.

Question 12.
Envision Math Common Core Grade 6 Answer Key Topic 7 Solve Area, Surface Area, And Volume Problems 105

Answer:
The figure is a triangular pyramid.

Explanation:
In the above-given question,
given that,
the figure is a triangular pyramid.
volume = 1/3 x b x h.
b = base area.
h = height.

In 13-15, identify each solid from its net.
Question 13.
Envision Math Common Core Grade 6 Answer Key Topic 7 Solve Area, Surface Area, And Volume Problems 106

Answer:
The figure is pyramid.

Explanation:
In the above-given question,
given that,
the net is pyramid.
pyramid has face, base, and vertex.
so the figure is pyramid.

Question 14.
Envision Math Common Core Grade 6 Answer Key Topic 7 Solve Area, Surface Area, And Volume Problems 107

Answer:
Volume of the prism = 1/2bhl.

Explanation:
In the above-given question,
given that,
the figure is triangular prism.
volume of the prism = 1/2 x bhl.
where b = base.
h = height.
l = length.

Question 15.
Envision Math Common Core Grade 6 Answer Key Topic 7 Solve Area, Surface Area, And Volume Problems 108
Answer:
The figure is rectangular prism.

Explanation:
In the above-given question,
given that,
the figure is a rectangular prism.
volume of prism = l x w x h.
where l = length.
w = width.
h = height.

Question 16.
Ryan is going to draw a net of a rectangular prism. How many rectangles should there be in his drawing?

Answer:
The figure is rectangular prism.

Explanation:
In the above-given question,
given that,
the figure is a rectangular prism.
volume of prism = l x w x h.
where l = length.
w = width.
h = height.

Question 17.
Kayla is going to draw a net of a square pyramid. How many triangles should there be in her drawing?

Answer:
The formula of square pyramid = a x a x h/3.

Explanation:
In the above-given question,
given that,
the formula of square pyramid.
v = a x a x h/3.
where a = base edge.
h = height.

Question 18.
Make Sense and Persevere Zari folds the net below into a model of a solid figure. How many edges, faces, and vertices does the model have?
Envision Math Common Core Grade 6 Answer Key Topic 7 Solve Area, Surface Area, And Volume Problems 109

Answer:
The number of faces, edges, and vertices = 3.

Explanation:
In the above-given question,
given that,
the number of edges = 3.
the number of faces = 3.
the number of vertices = 3.

Question 19.
Critique Reasoning Tomas says that the net below can be folded to make a rectangular prism. Do you agree with Tomas? Explain.
Envision Math Common Core Grade 6 Answer Key Topic 7 Solve Area, Surface Area, And Volume Problems 110
Answer:

In 20-22, use the table at the right.
Envision Math Common Core Grade 6 Answer Key Topic 7 Solve Area, Surface Area, And Volume Problems 111
Question 20.
Look for Relationships The Swiss mathematician Leonhard Euler (OY-ler) and the French mathematician René Descartes (dã KART) both discovered a pattern in the numbers of edges, vertices, and faces of polyhedrons. Complete the table. Describe a pattern in the table.
Answer:

Question 21.
Higher Order Thinking Write an equation that relates the number of edges, E, to the number of faces, F, and vertices, V.
Answer:

Question 22.
Use the equation that you wrote in Exercise 21 to find the number of vertices of a cube, which has 12 edges and 6 faces.
Answer:

Question 23.
Corey bought the mailing tube shown at the right to mail a poster of the Gators Little League baseball team.
Envision Math Common Core Grade 6 Answer Key Topic 7 Solve Area, Surface Area, And Volume Problems 112
a. The mailing tube has the shape of which polyhedron?
Answer:

b. How many faces does the mailing tube have?
Answer:

c. When Corey bought the mailing tube, it was unfolded and looked like a net. What polygons would Corey have seen in the unfolded mailing tube?
Answer:

Assessment Practice

Question 24.
Draw a net of a square pyramid for which the base is 2 units long and the height of each triangular face is 5 units.
Envision Math Common Core Grade 6 Answer Key Topic 7 Solve Area, Surface Area, And Volume Problems 113
Answer:

3-ACT MATH

3-Act Mathematical Modeling: That’s a Wrap
Envision Math Common Core Grade 6 Answer Key Topic 7 Solve Area, Surface Area, And Volume Problems 114

ACT 1
Question 1.
After watching the video, what is the first question that comes to mind?
Answer:

Question 2.
Write the Main Question you will answer.
Envision Math Common Core Grade 6 Answer Key Topic 7 Solve Area, Surface Area, And Volume Problems 115
Answer:

Question 3.
Construct Arguments Predict an answer to this Main Question Explain your prediction.
Answer:

Question 4.
On the number line below, write a number that is too small to be the answer. Write a number that is too large.
Envision Math Common Core Grade 6 Answer Key Topic 7 Solve Area, Surface Area, And Volume Problems 116
Answer:

Question 5.
Plot your prediction on the same number line.
Answer:

ACT 2
Question 6.
What information in this situation would be helpful to know? How would you use that information?
Envision Math Common Core Grade 6 Answer Key Topic 7 Solve Area, Surface Area, And Volume Problems 117
Answer:

Question 7.
Use Appropriate Tools What tools can you use to solve the problem? Explain how you would use them strategically.
Answer:

Question 8.
Model with Math Represent the situation using mathematics. Use your representation to answer the Main Question.
Envision Math Common Core Grade 6 Answer Key Topic 7 Solve Area, Surface Area, And Volume Problems 118
Answer:

Question 9.
What is your answer to the Main Question? Is it higher or lower than your prediction? Explain why.
Answer:

ACT 3
Question 10.
Write the answer you saw in the video.
Answer:

Question 11.
Reasoning Does your answer match the answer in the video? If not, what are some reasons that would explain the difference?
Envision Math Common Core Grade 6 Answer Key Topic 7 Solve Area, Surface Area, And Volume Problems 119
Answer:

Question 12.
Make Sense and Persevere Would you change your model now that you know the answer? Explain.
Envision Math Common Core Grade 6 Answer Key Topic 7 Solve Area, Surface Area, And Volume Problems 120
Answer:

Reflect
Question 13.
Model with Math Explain how you used a mathematical model to represent the situation. How did the model help you answer the Main Question?
Answer:

Question 14.
Look for Relationships Explain how the dimensions of the gift are related to the area of its net.
Answer:

SEQUEL
Question 15.
Critique Reasoning A classmate says that if all dimensions of the gift were doubled, you would need twice as many squares. Do you agree? Justify his reasoning or explain his error.
Envision Math Common Core Grade 6 Answer Key Topic 7 Solve Area, Surface Area, And Volume Problems 121
Answer:

Lesson 7.6 Find Surface Areas of Prisms

Solve & Discuss It!
Marianne orders boxes to pack gifts. When they arrive, she finds flat pieces of cardboard as shown below. Marianne needs to cover each face of the boxes with green paper. What is the least amount of paper needed to cover each box? Explain.
I can… draw a net of a prism and use it to find the prism’s surface area
Envision Math Common Core Grade 6 Answer Key Topic 7 Solve Area, Surface Area, And Volume Problems 122

Make Sense and Persevere
What solid figure does this net represent?

Focus on math practices
Make Sense and Persevere Suppose Marianne has only one large sheet of green paper that is 15 inches by 30 inches. Is the area of this sheet of paper great enough to cover all of the faces of one box? Explain.

Essential Question
How can you find the surface area of a prism?

Try It!

Use the net and the formula to find the surface area of the prism.
SA = 2(lw) + 2(wh) + 2(lh)
Envision Math Common Core Grade 6 Answer Key Topic 7 Solve Area, Surface Area, And Volume Problems 123

Answer:
Surface area = 52 sq cm.

Explanation:
In the above-given question,
given that,
the surface area of the prism = 2(lw) + 2(wh) + 2(lh).
where l = length, w = width, and h = height.
surface area = 2(8) + 2(6) + 2(12).
surface area = 16 + 12 + 24.
surface area = 52 sq cm.

Convince Me! Why are tw, wh, and th each multiplied by 2 in the formula?
Envision Math Common Core Grade 6 Answer Key Topic 7 Solve Area, Surface Area, And Volume Problems 124

Try It!

Find the surface area of each prism.
a.
Envision Math Common Core Grade 6 Answer Key Topic 7 Solve Area, Surface Area, And Volume Problems 125

Answer:
Surface area = 52 sq cm.

Explanation:
In the above-given question,
given that,
the surface area of the prism = 2(lw) + 2(wh) + 2(lh).
where l = length, w = width, and h = height.
surface area = 2(17.64) + 2(17.64) + 2(17.64).
surface area = 35.28 + 35.28 + 35.28.
surface area = 105.84 sq cm.

b.
Envision Math Common Core Grade 6 Answer Key Topic 7 Solve Area, Surface Area, And Volume Problems 126

Answer:
Surface area = 214 sq ft.

Explanation:
In the above-given question,
given that,
the surface area of the prism = 2(lw) + 2(wh) + 2(lh).
where l = length, w = width, and h = height.
surface area = 2(42) + 2(35) + 2(30).
surface area = 84 + 70 + 60.
surface area = 214 sq ft.

KEY CONCEPT
To find the surface area of a prism, use a net or a formula.
Envision Math Common Core Grade 6 Answer Key Topic 7 Solve Area, Surface Area, And Volume Problems 127

Do You Understand?
Question 1.
Essential Question How can you find the surface area of a prism?

Answer:
Area = 216.

Explanation:
In the above-given question,
given that,
the surface area of the prism = 2(lw) + 2(wh) + 2(lh).
area = 2(10)(6) + 2(6)(3) + 2(10)(3).
area = 2(60) + 2(18) + 2(30).
area = 120 + 36 + 60.
area = 216.

Question 2.
Construct Arguments Could you use the formula for the surface area of a rectangular prism to find the surface area of a cube? Explain.
Answer:

Question 3.
Look for Relationships Which faces of a rectangular prism always have the same area?
Answer:

Question 4.
Generalize What does it mean to find surface area?
Answer:

Do You Know How?
In 5-7, find the surface area of each prism.
Question 5.
Envision Math Common Core Grade 6 Answer Key Topic 7 Solve Area, Surface Area, And Volume Problems 128

Answer:
Area = 160 sq ft.

Explanation:
In the above-given question,
given that,
the surface area of the prism = 2(lw) + 2(wh) + 2(lh).
area = 2(10)(5) + 2(5)(2) + 2(10)(2).
area = 2(50) + 2(10) + 2(20).
area = 100 + 20 + 40.
area = 160.

Question 6.
Envision Math Common Core Grade 6 Answer Key Topic 7 Solve Area, Surface Area, And Volume Problems 129

Answer:
Area = 294.

Explanation:
In the above-given question,
given that,
the surface area of the prism = 2(lw) + 2(wh) + 2(lh).
area = 2(7)(7) + 2(7)(7) + 2(7)(7).
area = 2(49) + 2(49) + 2(49).
area = 98 + 98 + 98.
area = 294.

Question 7.
Envision Math Common Core Grade 6 Answer Key Topic 7 Solve Area, Surface Area, And Volume Problems 130

Answer:
Area = 30.4 sq m.

Explanation:
In the above-given question,
given that,
the surface area of the prism = 2(lw) + 2(wh) + 2(lh).
area = 2(2)(2.8) + 2(2.8)(2) + 2(2)(2).
area = 2(5.6) + 2(5.6) + 2(4).
area = 11.2 + 11.2 + 8.
area = 30.4.

Practice & Problem Solving

In 8-13, find the surface area of each prism.
Question 8.
Envision Math Common Core Grade 6 Answer Key Topic 7 Solve Area, Surface Area, And Volume Problems 131

Answer:
Area = 384 sq in.

Explanation:
In the above-given question,
given that,
the surface area of the prism = 2(lw) + 2(wh) + 2(lh).
area = 2(8)(8) + 2(8)(8) + 2(8)(8).
area = 2(64) + 2(64) + 2(64).
area = 128 + 128 + 128.
area = 384.

Question 9.
Envision Math Common Core Grade 6 Answer Key Topic 7 Solve Area, Surface Area, And Volume Problems 132

Answer:
Area = 114 sq yd.

Explanation:
In the above-given question,
given that,
the surface area of the prism = 2(lw) + 2(wh) + 2(lh).
area = 2(3)(5) + 2(5)(6) + 2(4)(3).
area = 2(15) + 2(30) + 2(12).
area = 30 + 60 + 24.
area = 114.

Question 10.
Envision Math Common Core Grade 6 Answer Key Topic 7 Solve Area, Surface Area, And Volume Problems 133

Answer:
Area = 550 sq cm.

Explanation:
In the above-given question,
given that,
the surface area of the prism = 2(lw) + 2(wh) + 2(lh).
area = 2(10)(5) + 2(5)(15) + 2(10)(15).
area = 2(50) + 2(75) + 2(150).
area = 100 + 150 + 300.
area = 550 sq cm.

Question 11.
Envision Math Common Core Grade 6 Answer Key Topic 7 Solve Area, Surface Area, And Volume Problems 134

Answer:
Area = 73 sq m.

Explanation:
In the above-given question,
given that,
the surface area of the prism = 2(lw) + 2(wh) + 2(lh).
area = 2(7)(2) + 2(2.5)(2) + 2(7)(2.5).
area = 2(14) + 2(5) + 2(17.5).
area = 28 + 10 + 35.
area = 73 sq m.

Question 12.
Envision Math Common Core Grade 6 Answer Key Topic 7 Solve Area, Surface Area, And Volume Problems 135

Answer:
Area = 358.4 sq cm.

Explanation:
In the above-given question,
given that,
the surface area of the prism = 2(lw) + 2(wh) + 2(lh).
area = 2(8)(7.2) + 2(7.2)(8) + 2(8)(8).
area = 2(57.6) + 2(57.6) + 2(64).
area = 115.2 + 115.2 + 128.
area = 358.4 sq cm.

Question 13.
Envision Math Common Core Grade 6 Answer Key Topic 7 Solve Area, Surface Area, And Volume Problems 136

Answer:
Area = 864 sq ft.

Explanation:
In the above-given question,
given that,
the surface area of the prism = 2(lw) + 2(wh) + 2(lh).
area = 2(12)(12) + 2(12)(12) + 2(12)(12).
area = 2(144) + 2(144) + 2(144).
area = 288 + 288 + 288.
area = 864 sq ft.

Question 14.
Critique Reasoning Jacob says that the surface: area of the cube is less than 1,000 cm2. Do you agree with Jacob? Explain.
Envision Math Common Core Grade 6 Answer Key Topic 7 Solve Area, Surface Area, And Volume Problems 137

Answer:
Area = 216 sq cm.

Explanation:
In the above-given question,
given that,
the surface area of the prism = 2(lw) + 2(wh) + 2(lh).
area = 2(10)(6) + 2(6)(3) + 2(10)(3).
area = 2(60) + 2(18) + 2(30).
area = 120 + 36 + 60.
area = 216 sq cm.

Question 15.
You want to wrap a paperweight shaped like the triangular prism shown. How many square inches of wrapping paper do you need to completely cover the prism?
Envision Math Common Core Grade 6 Answer Key Topic 7 Solve Area, Surface Area, And Volume Problems 138

Answer:
Area = 128 sq in.

Explanation:
In the above-given question,
given that,
the surface area of the prism = 2(lw) + 2(wh) + 2(lh).
area = 2(4)(4) + 2(8)(4) + 2(4)(4).
area = 2(16) + 2(32) + 2(16).
area = 32 + 64 + 32.
area = 128 sq in.

Question 16.
Sasha has 2 blocks of clay shaped like the rectangular prism below. She joins them to form a rectangular prism with a length of 12 inches. What is the surface area of the larger prism?
Envision Math Common Core Grade 6 Answer Key Topic 7 Solve Area, Surface Area, And Volume Problems 139

Answer:
Area = 56 sq in.

Explanation:
In the above-given question,
given that,
the surface area of the prism = 2(lw) + 2(wh) + 2(lh).
area = 2(6)(2) + 2(2)(2) + 2(6)(2).
area = 2(12) + 2(4) + 2(12).
area = 24 + 8 + 24.
area = 56 sq in.

Question 17.
A rectangular prism has a length of 12 cm, a height of 6 cm, and a width of 4 cm. Use the formula SA = 2(lw) + 2(wh) + 2(lh) to find the surface area of the rectangular prism.

Answer:
Area = 288 sq cm.

Explanation:
In the above-given question,
given that,
the surface area of the prism = 2(lw) + 2(wh) + 2(lh).
area = 2(12)(4) + 2(4)(6) + 2(12)(6).
area = 2(48) + 2(24) + 2(72).
area = 96 + 48 + 144.
area = 288 sq cm.

In 18 and 19, use the diagram of the birdhouse.
Envision Math Common Core Grade 6 Answer Key Topic 7 Solve Area, Surface Area, And Volume Problems 140

Answer:
Area = 210 sq in.

Explanation:
In the above-given question,
given that,
the surface area of the prism = 2(lw) + 2(wh) + 2(lh).
area = 2(5)(5) + 2(5)(8) + 2(5)(8).
area = 2(25) + 2(40) + 2(40).
area = 50 + 80 + 80.
area = 210.

Question 18.
Kali wants to build this birdhouse. She bought a 24-inch by 48-inch sheet of plywood. Does Kali have enough wood to make the birdhouse? Explain.

Answer:
Yes, kali has enough wood to make the birdhouse.

Explanation:
In the above-given question,
given that,
Kali wants to build this birdhouse.
She bought a 24-inch by 48-inch sheet of plywood.
24 x 48.
1152 sq in.
so kali has enough wood to make the birdhouse.

Question 19.
Reasoning kali decides to paint the birdhouse. She has a pint of paint that covers 32.5 ft2 of surface. How can you tell that Kali has enough paint without calculating?

Answer:
Yes, kali has enough paint without calculating.

Explanation:
In the above-given question,
given that,
kali decides to paint the birdhouse.
She has a pint of paint that covers 32.5 ft2 of the surface.
24 x 48.
1152 sq in.
so kali has enough paint without calculating.

Question 20.
Use the formula SA = 2lw + 2lh + 2wh to find the surface area for a rectangular prism with a length, l, of 2.3 inches; a width, w, of 1.1 inches; and a height, h, of 3 inches.

Answer:
Area = 25.92 sq in.

Explanation:
In the above-given question,
given that,
length = 2.3 in.
width = 1.1 in.
height = 3 in.
surface area for a rectangular prism = 2lw + 2lh + 2wh.
area = 2(2.3 x 1.2) + 2(2.3 x 3) + 2(1.1 x 3).
area = 2(2.76) + 2(6.9) + 2(3.3).
area = 5.52 + 13.8 + 6.6.
area = 25.92 sq in.

Question 21.
Make Sense and Persevere Justine wants to wrap a shipping box shaped like a rectangular prism. The box is 28 inches tall and has a square base with sides that each measure 2 inches. How much paper will Justine use?

Answer:
Justin uses 232 sq in of paper.

Explanation:
In the above-given question,
given that,
Justine wants to wrap a shipping box shaped like a rectangular prism.
The box is 28 inches tall and has a square base with sides that each measure 2 inches.
area = 2lw + 2wh + 2lh.
area = 2(2 x 2) + 2(2 x 28) + 2(2 x 28).
area = 2(4) + 2(56) + 2(56).
area = 8 + 112 + 112.
area = 232 sq in.
so Justin uses 232 sq in of paper.

Question 22.
Higher Order Thinking Margaret wants to cover a footrest in the shape of a rectangular prism with cotton fabric. The footrest is 18 inches by 12 inches by 10 inches. Margaret has 1 square yard of fabric. Can she completely cover the footrest? Explain.

Answer:
Yes, she completely covers the footrest.

Explanation:
In the above-given question,
given that,
Margaret wants to cover a footrest in the shape of a rectangular prism with cotton fabric.
The footrest is 18 inches by 12 inches by 10 inches.
Margaret has 1 square yard of fabric.
area = 2lw + 2wh + 2lh.
area = 2(18 x 12) + 2(12 x 10) + 2(18 x 10).
area = 2(216) + 2(120) + 2(180).
area = 432 + 240 + 360.
area = 1032 sq in.
1 square yd = 1296 sq in.
yes, she completely covers the footrest.

Question 23.
A cube has a surface area of 486 in.2. Can the length of each side of the cube be 11 in.? Explain.

Answer:
No, the length of each side of the cube is 9 in.

Explanation:
In the above-given question,
given that,
A cube has a surface area of 486 sq.
the surface area of the cube = 6 a x a.
area = 6 x 9 x 9.
486 = 81 x 6.
no the length of each side of the cube is 9 in.

Assessment Practice

Question 24.
Using nets, find the surface area, in square feet, of a rectangular prism with a height of 2 feet, a length of 4.2 feet, and a width of 2.5 feet.

Answer:
Surface area = 47.8 sq ft.

Explanation:
In the above-given question,
given that,
rectangular prism with a height of 2 feet, a length of 4.2 feet, and a width of 2.5 feet.
surface area = 2lw + 2wh + 2lh.
area = 2(2 x 4.2) + 2(4.2 x 2.5) + 2(2 x 2.5).
area = 2(8.4) + 2(10.5) + 2(5).
area = 16.8 + 21 + 10.
area = 47.8 sq ft.

Question 25.
The surface area of a cube is 273.375 square feet. The net of the prism is shown.
Envision Math Common Core Grade 6 Answer Key Topic 7 Solve Area, Surface Area, And Volume Problems 141
What are the possible dimensions of the cube in feet?
A. 6, 6, 6
B. 6, 6.75, 6.75
C. 6.5, 6.5, 6.5
D. 6.75, 6.75, 6.75

Answer:
Option B is correct.

Explanation:
In the above-given question,
given that,
the surface area of the cube = 6a x a.
surface area = 6 x 6.75 x 6.75.
area = 6 x 45.5625.
area = 273.375 cubic feet.
so option B is correct.

Lesson 7.7 Find Surface Areas of Pyramids

Solve & Discuss It!
The fence in Marci’s front yard has decorative tops, in the shape of square pyramids, every 6 feet. Marci paints each face of each top a different color before attaching the tops to the fence. What is the total surface area that she paints on each decorative top?
I can… draw a net of a pyramid and use it to find the pyramid’s surface area.
Envision Math Common Core Grade 6 Answer Key Topic 7 Solve Area, Surface Area, And Volume Problems 142

Use Appropriate Tools
What tools can you use to help solve this problem?

Focus on math practices
Use Structure Suppose the side lengths of the square base of each decorative top are increased by 2 inches. What is the total surface area of each top?

Essential Question
How can you find the surface area of a pyramid?

Answer:
The surface area of the pyramid = A + 1/2 ps.

Explanation:
In the above-given question,
given that,
the surface area of the pyramid = A + 1/2 ps.
where A = area of the base.
p = perimeter of the base.
s = slant height.

Try It!

Find the surface area of the square pyramid. Draw a net to find the areas of the base and each face of the pyramid.
Envision Math Common Core Grade 6 Answer Key Topic 7 Solve Area, Surface Area, And Volume Problems 143

Answer:
The surface area of the square pyramid = 528 sq cm.

Explanation:
In the above-given question,
given that,
base edge = 6 cm.
height = 8 cm.
surface area of the square pyramid = axa + 2a square root of axa/4 + h x h.
area = 36 + 12 square root 36/4 + 64.
area = 48 6/2+   8.
area = 48 x 3 + 8.
area = 48 x 11.
area = 528 sq cm.

Convince Me! For the pyramid in the Try It!, what values would you use for B, n, and A in the formula SA = B + (nA)?

Try It!

Draw a net and find the surface area of the triangular pyramid. Find the area (T) of each equilateral triangle.
Envision Math Common Core Grade 6 Answer Key Topic 7 Solve Area, Surface Area, And Volume Problems 144
T = \(\frac{1}{2}\)bh
T = \(\frac{1}{2}\) _______ _______
= ________
The area of each equilateral triangle is __________ m2.

Answer:
The area of each equilateral triangle is 7 sq m.

Explanation:
In the above-given question,
given that,
the area of each equilateral triangle is 1/2 x bh.
area = 1/2 x 4 x 3.5.
area = 2 x 3.5.
area = 7.
so area of each equilateral triangle is 7 sq m.

Find the surface area (SA) of the triangular pyramid.
SA = 4T
SA = ______ × T
SA = 4 × ________
= _______
The surface area of the triangular pyramid is ________ m2

Answer:
SA = 4T.

Explanation:
In the above-given question,
given that,
the surface area of the triangular pyramid is 4T.
SA = 4T.
SA is surface area.
4T = triangular pyramid.
4T = SA.

KEY CONCEPT
You can use a net to help find the surface area of a pyramid.
Envision Math Common Core Grade 6 Answer Key Topic 7 Solve Area, Surface Area, And Volume Problems 145

Do You Understand?
Question 1.
Essential Question How can you find the surface area of a pyramid?

Answer:
Surface area = 22 sq cm.

Explanation:
In the above-given question,
given that,
the surface area of a pyramid = SxS + 1/2bh.
side = 4 x 4.
side = 16.
surface area = 16 + 1/2bh.
area = 16 + 1/2 x 4 x 3.
area = 16 + 1/2 x 12.
area = 16 + 6.
area = 22 sq cm.

Question 2.
Look for Relationships How does finding the area of one face of a triangular pyramid that is made up of equilateral triangles help you find the surface area of the triangular pyramid?
Answer:

Question 3.
Make Sense and Persevere in the formula SA = 47, for the surface area of a triangular pyramid in which the faces are equilateral triangles, what does the variable T represent?

Answer:
Surface area = 47.

Explanation:
In the above-given question,
given that,
the surface area of the equilateral triangle = SxS + 1/2 bh.
area = 4 x 4 + 1/2 x 5 x 12.
area = 16 + 5 x 6.
area = 16 + 30.
area = 47.

Do You Know How?
Question 4.
Each side of the base of a square pyramid is 4 inches and the height of each triangular face is 3 inches. Draw a net for this pyramid and find its surface area.
Envision Math Common Core Grade 6 Answers Topic 7 Solve Area, Surface Area, And Volume Problems 146

Answer:
The surface area of the pyramid = 22 sq in.

Explanation:
In the above-given question,
given that,
Each side of the base of a square pyramid is 4 inches and the height of each triangular face is 3 inches.
surface area = sxs +1/2bh.
surface area = 4 x 4 + 1/2 3 x 4.
area = 16 + 6.
area = 22 sq in.

Question 5.
The faces of this triangular pyramid are equilateral triangles. Draw a net of the pyramid and use it to find the surface area.
Envision Math Common Core Grade 6 Answers Topic 7 Solve Area, Surface Area, And Volume Problems 147

Answer:
The surface area of the pyramid = 204.4 sq ft.

Explanation:
In the above-given question,
given that,
bases = 12 ft and h = 10.4 ft.
surface area = sxs +1/2bh.
surface area = 12 x 12 + 1/2 12 x 10.4.
area = 144 +60.4.
area = 204.4 sq ft.

Practice & Problem Solving

Leveled Practice In 6 and 7, find the surface area of each pyramid. The faces of each triangular pyramid are equilateral triangles.
Question 6.
Envision Math Common Core Grade 6 Answers Topic 7 Solve Area, Surface Area, And Volume Problems 148

Answer:
The surface area is 84 sq.

Explanation:
In the above-given question,
given that,
Area of the base = 7 x 7.
area = 49.
surface area = 49 + 1/2 x 7 x 10.
area = 49 + 35.
area = 84.
so the surface area is 84 sq in.

Question 7.
Envision Math Common Core Grade 6 Answers Topic 7 Solve Area, Surface Area, And Volume Problems 149
Answer:
The surface area is 51.2 sq cm.

Explanation:
In the above-given question,
given that,
Area of the base = 6 x 6.
area = 36.
surface area = 36 + 1/2 x 6 x 5.2.
area = 36 +15.2.
area = 51.2.
so the surface area is 51.2 sq cm.

Question 8.
Complete the net at the right to find the surface area of this triangular pyramid. The faces of the pyramid are equilateral triangles.
Envision Math Common Core Grade 6 Answers Topic 7 Solve Area, Surface Area, And Volume Problems 150

Answer:
The surface area is 459.6 sq mm.

Explanation:
In the above-given question,
given that,
Area of the base = 18 x 18.
area = 324.
surface area = 324 + 1/2 x 18 x 15.6.
area = 324 + 135.6.
area = 459.6.
so the surface area is 459.6 sq mm.

Question 9.
Simone is designing a piece of artwork in the shape of a square pyramid for a hotel. She wants to cover the pyramid with decorative glass. How many square feet of glass does Simone need to cover the entire pyramid?
Envision Math Common Core Grade 6 Answers Topic 7 Solve Area, Surface Area, And Volume Problems 151

Answer:
The surface area is 35 sq ft.

Explanation:
In the above-given question,
given that,
Area of the base = 5 x 5.
area = 25.
surface area = 25 + 1/2 x 5 x 4.
area = 25 +10.
area = 35.
so the surface area is 35 sq ft.

Question 10.
Critique Reasoning Kurt says that the surface area of this triangular pyramid with faces that are equilateral triangles is 173 cm2. Do you agree with Kurt? Explain.
Envision Math Common Core Grade 6 Answers Topic 7 Solve Area, Surface Area, And Volume Problems 152

Answer:
The surface area is 570.3 sq cm.

Explanation:
In the above-given question,
given that,
Area of the base = 20 x 20.
area = 400.
surface area = 400 + 1/2 x 20 x 17.3.
area = 400 + 170.3.
area = 570.3.
so the surface area is 570.3 sq cm.

In 11 and 12, use the pyramid and net shown.
Envision Math Common Core Grade 6 Answers Topic 7 Solve Area, Surface Area, And Volume Problems 153
Question 11.
Model with Math Ken drew a square pyramid and its net to represent a doghouse that he is building. Complete the net by filling in the missing measures.

Answer:
The surface area is 16.5 sq ft.

Explanation:
In the above-given question,
given that,
Area of the base = 3 x 3.
area = 9.
surface area = 9 + 1/2 x 3 x 5.
area = 9 + 7.5.
area = 16.5.
so the surface area is 16.5 sq ft.

Question 12.
Use the net to find the amount of wood Ken needs to make the doghouse.
Answer:

Question 13.
The surface area of this square pyramid is 644 ft2. Can the value of x be 20? Explain.
Envision Math Common Core Grade 6 Answers Topic 7 Solve Area, Surface Area, And Volume Problems 154

Answer:
The surface area is 336 sq ft.

Explanation:
In the above-given question,
given that,
Area of the base = 14 x 14.
area = 196.
surface area = 196 + 1/2 x 14 x 20.
area = 196 + 140.
area = 336.
so the surface area is 336 sq ft.

Question 14.
Construct Arguments Which of these pyramids do you think has the greater surface area? Explain.

  • Square pyramid: The base is 10 cm by 10 cm and the triangular faces have a height of 8.66 cm.
  • Triangular pyramid: All the faces are equilateral triangles with a base of 10 cm and a height of 8.66 cm.
    Answer:

Question 15.
Higher Order Thinking The base of a pyramid can be any polygon. How many faces does a pentagonal pyramid have? Describe the shapes of the faces.
Answer:

Question 16.
Vocabulary What is the term used to describe a point where three or more edges of a solid figure meet?
Answer:

Assessment Practice

Question 17.
Which net represents the pyramid with the greatest surface area?
Envision Math Common Core Grade 6 Answers Topic 7 Solve Area, Surface Area, And Volume Problems 155
Answer:

Lesson 7.8 Find Volume with Fractional Edge Lengths

Solve & Discuss It!
A rectangular prism has the dimensions shown. What is the volume of this rectangular prism?

I can… find the volume of a rectangular prism with fractional edge lengths.
Envision Math Common Core Grade 6 Answers Topic 7 Solve Area, Surface Area, And Volume Problems 156

Use Structure
How might filling the rectangular prism with layers of 3-inch cubes help you find the volume?

Focus on math practices
Look for Relationships You know how to use the formula V = \(\frac{1}{2}\)wh to find the volume of a rectangular prism. How might you use the formula to find the volume of the prism above?

Essential Question
How can you find the volume of a rectangular prism with fractional edge lengths?

Try It!

Find the volume of the rectangular prism built from \(\frac{1}{2}\)-inch cubes.
Envision Math Common Core Grade 6 Answers Topic 7 Solve Area, Surface Area, And Volume Problems 157
The bottom layer has _______ cubes. The prism is _______ cubes high. There are a total of _______ cubes in the prism. Each cube has a volume of _______ in3.
Volume of prism = _______ × _______ = _______ in3.

Answer:
Volume of prism =

Explanation:
In the above-given question,
given that,
the bottom layer has 5 cubes.
volume of the rectangular prism = 2(lw) + 2(wh) + 2(lh).
volume = 2(2.5 x 2.5) + 2(2.5 x 2.5) + 2(2.5 x 2.5).
volume = 2(
volume of prism =

Convince Me! Suppose that the length of the rectangular prism in the Try It! were 3\(\frac{1}{2}\) inches instead of 2\(\frac{1}{2}\) inches. How many cubes would there be in the prism? What would be the volume of the prism?

Try It!

Find the volume of each rectangular prism.
a.
Envision Math Common Core Grade 6 Answers Topic 7 Solve Area, Surface Area, And Volume Problems 158

Answer:
The volume of rectangular prism = 61 sq cm.

Explanation:
In the above-given question,
given that,
l = 3.2 cm, h = 4.5 cm, and w = 2.8 cm.
volume of rectangular prism = 2(lw) + 2(wh) + 2(lh).
volume = 2(3.2 x 2.8) + 2(2.8 x 4.5) + 2(3.2 x 4.5).
volume = 2(3.5) + 2(12.6) + 2(14.4).
volume = 7 + 25.2 + 28.8.
volume = 61 sq cm.

b.
Envision Math Common Core Grade 6 Answers Topic 7 Solve Area, Surface Area, And Volume Problems 159
Answer:
The volume of rectangular prism = 121.5 sq ft.

Explanation:
In the above-given question,
given that,
l = 4.5 cm, h = 4.5 cm, and w = 4.5 cm.
volume of rectangular prism = 2(lw) + 2(wh) + 2(lh).
volume = 2(4.5 x 4.5) + 2(4.5 x 4.5) + 2(4.5 x 4.5).
volume = 2(20.25) + 2(20.25) + 2(20.25).
volume = 40.5 + 40.5 + 40.5.
volume = 121.5 sq ft.

KEY CONCEPT
You can find the volume of a rectangular prism with fractional edge lengths by determining the number of same-sized cubes with unit fraction edge lengths needed to completely fill the prism, then multiplying that number of cubes by the volume of each cube. You can also apply a formula.
Envision Math Common Core Grade 6 Answers Topic 7 Solve Area, Surface Area, And Volume Problems 160

Do You Understand?
Question 1.
Essential Question How can you find the volume of a rectangular prism with fractional edge lengths?

Answer:
The volume of a rectangular prism = l x w x h.

Explanation:
In the above-given question,
given that,
the volume of a rectangular prism = l x w x h.
where l = length.
w = width.
h = height.

Question 2.
How is finding the volume of a rectangular prism with fractional edge lengths similar to finding the volume of a rectangular prism with whole number edge lengths?

Answer:
The volume of a rectangular prism = l x w x h.

Explanation:
In the above-given question,
given that,
the volume of a rectangular prism = l x w x h.
where l = length.
w = width.
h = height.

Question 3.
Construct Arguments How can you use the number of \(\frac{1}{2}\)-inch cubes in a rectangular prism to find the number of unit cubes in the rectangular prism?

Answer:
The volume of a rectangular prism = 0.125 sq in.

Explanation:
In the above-given question,
given that,
the volume of a rectangular prism = l x w x h.
where l = length.
w = width.
h = height.
volume = 0.5 x 0.5 x 0.5.
volume = 0.125 sq in.

Do You Know How?
In 4 and 5, tell how many of each size of cube can fill a 1-inch cube.
Question 4.
Edge = \(\frac{1}{3}\) inch

Answer:
Volume = 0.027 cubic in.

Explanation:
In the above-given question,
given that,
edge = 1/3 in.
volume = ax ax a.
volume = 0.3 x 0.3 x 0.3.
volume = 0.027 cubic in.

Question 5.
Edge = \(\frac{1}{4}\) inch

Answer:
Volume = 0.027 cubic in.

Explanation:
In the above-given question,
given that,
edge = 1/4 in.
volume = ax ax a.
volume = 0.25 x 0.25 x 0.25.
volume = 0.75 cubic in.

In 6-9, find the volume of each rectangular prism.
Question 6.
Envision Math Common Core Grade 6 Answers Topic 7 Solve Area, Surface Area, And Volume Problems 161

Answer:
Volume = 4.5 cubic in.

Explanation:
In the above-given question,
given that,
l = 3/2 in, w = 3/2 in, and h = 3/2 in.
volume = ax ax a.
volume = 1.5 x 1.5 x 1.5.
volume = 4.5 cubic in.

Question 7.
Envision Math Common Core Grade 6 Answers Topic 7 Solve Area, Surface Area, And Volume Problems 162

Answer:
Volume = 233.1 cubic m.

Explanation:
In the above-given question,
given that,
the volume of the rectangular prism = lwh.
length = 14.8 m.
width = 4.5 m.
height = 3.5 m.
volume = 14.8 x 4.5 x 3.5.
volume = 233.1 cubic m.

Question 8.
Envision Math Common Core Grade 6 Answers Topic 7 Solve Area, Surface Area, And Volume Problems 163

Answer:
Volume = 3.75 cubic ft.

Explanation:
In the above-given question,
given that,
the volume of the rectangular prism = lwh.
length = 1.25 ft.
width = 1.25 ft.
height = 2 ft.
volume = 1.25 x 1.25 x 1.25.
volume = 3.75 cubic ft.

Question 9.
Envision Math Common Core Grade 6 Answers Topic 7 Solve Area, Surface Area, And Volume Problems 164

Answer:
Volume = 12.167 cubic cm.

Explanation:
In the above-given question,
given that,
the volume of the rectangular prism = lwh.
length = 2.3 cm.
width = 2.3 cm.
height = 2.3 cm.
volume = 2.3 x 2.3 x 2.3.
volume = 12.167 cubic cm.

Practice & Problem Solving

In 10-13, find the volume of each rectangular prism.
Question 10.
Envision Math Common Core Grade 6 Answers Topic 7 Solve Area, Surface Area, And Volume Problems 165

Answer:
Volume = 31.2 cubic m.

Explanation:
In the above-given question,
given that,
the volume of the rectangular prism = lwh.
length = 6.5 m.
width = 4 m.
height = 1.2 m.
volume = 6.5 x 4 x 1.2.
volume = 31.2 cubic m.

Question 11.
Envision Math Common Core Grade 6 Answers Topic 7 Solve Area, Surface Area, And Volume Problems 166

Answer:
Volume = 3.75 cubic yd.

Explanation:
In the above-given question,
given that,
the volume of the rectangular prism = lwh.
length = 2 yd.
width = 1.6 yd.
height = 5.3 yd.
volume = 2 x 1.6 x 5.3.
volume = 16.96 cubic yd.

Question 12.
Envision Math Common Core Grade 6 Answers Topic 7 Solve Area, Surface Area, And Volume Problems 167

Answer:
Volume = 35.937 cubic ft.

Explanation:
In the above-given question,
given that,
the volume of the rectangular prism = lwh.
length = 3.3 ft.
width = 3.3 ft.
height =  3.3 ft.
volume = 3.3 x 3.3 x 3.3.
volume = 35.937 cubic ft.

Question 13.
Envision Math Common Core Grade 6 Answers Topic 7 Solve Area, Surface Area, And Volume Problems 168

Answer:
Volume = 1.512 cubic m.

Explanation:
In the above-given question,
given that,
the volume of the rectangular prism = lwh.
length = 0.7 m.
width = 0.9 m.
height = 2.4 m.
volume = 0.7 x 0.9 x 2.4.
volume = 1.512 cubic m.

Question 14.
A clear box has the shape of a rectangular prism and is filled with sand. Find the volume of the box.
Envision Math Common Core Grade 6 Answers Topic 7 Solve Area, Surface Area, And Volume Problems 169

Answer:
Volume = 241.3 cubic in.

Explanation:
In the above-given question,
given that,
the volume of the rectangular prism = lwh.
length = 8.25 in.
width = 4.5 in.
height = 6.5 in.
volume = 8.25 x 4.5 x 6.5.
volume = 241.3 cubic in.

Question 15.
Use Structure A rectangular prism has a length of 2\(\frac{1}{2}\) yd, a width of 1\(\frac{1}{2}\) yd, and a height of 1\(\frac{1}{2}\) yd. You use cubes with fractional edge lengths of \(\frac{1}{2}\) yd to find the volume. How many cubes are tħere for each of the length, width, and height of the prism? What is the volume of the prism?

Answer:
The volume of the prism = 5.625 cubic yd.

Explanation:
In the above-given question,
given that,
A rectangular prism has a length of 2\(\frac{1}{2}\) yd, a width of 1\(\frac{1}{2}\) yd, and a height of 1\(\frac{1}{2}\) yd.
volume of the prism = l x w x h.
l = 2.5, w = 1.5, and h = 1.5.
volume = 2.5 x 1.5 x 1.5.
volume = 5.625 cubic yd.

Question 16.
A gift box has the shape of a cube. The length of each side is 10.5 cm. What is the volume of the gift box?

Answer:
The volume of the gift box = 31.5 cubic cm.

Explanation:
In the above-given question,
given that,
A gift box has the shape of a cube.
the length of each side is 10.5 cm.
volume = lx w x h.
volume = 10.5 x 10.5 x 10.5.
volume = 31.5.
so the volume of the gift box = 31.5 cubic cm.

Question 17.
A school locker has a length of 1 ft, a width of 18 in., and a height of 2\(\frac{1}{2}\) ft. What is the volume of the locker in cubic feet?
Envision Math Common Core Grade 6 Answers Topic 7 Solve Area, Surface Area, And Volume Problems 170

Answer:
The volume of rectangular prism = 45 cubic ft.

Explanation:
In the above-given question,
given that,
A school locker has a length of 1 ft, a width of 18 in, and a height of 2(1/2).
volume of the prism = l x w x h.
volume = 1 x 18 x 2.5.
volume = 45 cubic ft.

In 18 and 19, use the table.
Envision Math Common Core Grade 6 Answers Topic 7 Solve Area, Surface Area, And Volume Problems 171
Question 18.
Use Structure Sandy has two boxes with the dimensions shown. She wants to use the box with the greater volume to ship a gift to her friend. Which box should Sandy use? Explain.

Answer:
Sandy ship a box B.

Explanation:
In the above-given question,
given that,
Sandy has two boxes with the dimensions shown.
the volume of the Box A = l x w x h.
volume = 7.5 x 2 x 11.5.
volume = 172.5 cubic ft.
volume = 172.5 cubic ft.
the volume of the Box B = l x w x h.
volume = 9 x 2.5 x 8.5.
volume = 191.25 cubic ft.

Question 19.
Sandy finds a third box, box C, that has a length of 8 inches, a width of 2\(\frac{3}{4}\) inches, and a height of 10\(\frac{1}{2}\) inches. If Sandy wants to use the box with the greatest volume, should she use box C? Explain.

Answer:
Sandy will ship box C.

Explanation:
In the above-given question,
given that,
Sandy finds a third box, box C, that has a length of 8 inches, a width of 2\(\frac{3}{4}\) inches, and a height of 10\(\frac{1}{2}\) inches.
volume = l x w x h.
volume = 8 x 11/4 x 21/2.
volume = 8 x 2.75 x 10.5.
volume = 231 cubic in.
Sandy will ship box C.

Question 20.
The volume of a large crate is 84 yd3. It is 2\(\frac{2}{3}\) yd wide and 4\(\frac{2}{3}\) yd high. What is the length of the crate?

Answer:
The length of the crate = 7.023.

Explanation:
In the above-given question,
given that,
The volume of a large crate is 84 yd3.
It is 2\(\frac{2}{3}\) yd wide and 4\(\frac{2}{3}\) yd high.
v = l x w x h.
84 = l x 8/3 x 14/3.
84 = l x 2.6 x 4.6.
84 = l x 11.96.
l = 84/11.96.
l = 7.023.
so the length of the crate = 7.023 sq yds.

Question 21.
Higher Order Thinking A box covers an area of 8\(\frac{3}{4}\) in.2 when resting on its base. The volume of the box is 74\(\frac{3}{8}\) in.3. Can you find the surface area of the box? Explain.

Answer:
The surface area of the box = 74.375 cubic in.

Explanation:
In the above-given question,
given that,
A box covers an area of 8\(\frac{3}{4}\) in.2 when resting on its base.
The volume of the box is 74\(\frac{3}{8}\) in.3
The surface area of the box = 595/8.
the surface area of the box = 74.375 cubics in.

Question 22.
Make Sense and Persevere A gold bar is similar in shape to a rectangular prism. A standard mint bar is approximately 7 in. × 3\(\frac{5}{8}\) in. × 1\(\frac{3}{4}\) in. If the value of gold is $1,313 per ounce, about how much is one gold bar worth? Use the formula w≈11.15n, where w is the weight in ounces and n is the volume in cubic inches, to find the weight in ounces. Explain how you found your answer.
Envision Math Common Core Grade 6 Answers Topic 7 Solve Area, Surface Area, And Volume Problems 172
Answer:

Assessment Practice

Question 23.
Which rectangular prism with the given dimensions has the same volume as the prism shown?
Envision Math Common Core Grade 6 Answers Topic 7 Solve Area, Surface Area, And Volume Problems 173
A. 1 in., 3.5 in., 4.25 in.
B. 1.25 in., 3.5 in., 4 in.
C. 1.75 in., 3 in., 3.5 in.
D. 2 in., 2.25 in., 4 in.

Answer:
Option C is correct.

Explanation:
In the above-given question,
given that,
volume of the rectangular prism = l x w x h.
volume = 4 x 7/4 x 5/2.
volume = 4 x 1.75 x 2.5.
volume = 17.5.
so option C is correct.

Topic 7 Review

Topic Essential Question
How can the areas of certain shapes be found? What are the meanings of surface area and volume and how can surface area and volume be found?

Vocabulary Review
Write the vocabulary term that best represents each item.
Vocabulary

  • net
  • rhombus
  • trapezoid
  • vertex
  • volume

Question 1.
The measure of the space this solid figure occupies.
Envision Math Common Core Grade 6 Answers Topic 7 Solve Area, Surface Area, And Volume Problems 174

Answer:
The measure of the space this solid figure occupies is volume.

Explanation:
In the above-given question,
given that,
the measure of the space this solid figure occupies is volume.
the volume of the prism = l x w x h.
where l = length.
w = width.
h = height.

Question 2.
Envision Math Common Core Grade 6 Answers Topic 7 Solve Area, Surface Area, And Volume Problems 175

Answer:
The measure of the space this solid figure occupies is net.

Explanation:
In the above-given question,
given that,
the measure of the space this solid figure occupies is net.

Question 3.
Envision Math Common Core Grade 6 Answers Topic 7 Solve Area, Surface Area, And Volume Problems 176
Answer:
The figure is a rhombus.

Explanation:
In the above-given question,
given that,
the figure is a rhombus.
for example:
the volume of the rhombus = 12 ABH.
where a and b are diagonals of the rhombus.
h is the height of the prism.

Question 4.
Envision Math Common Core Grade 6 Answers Topic 7 Solve Area, Surface Area, And Volume Problems 177
Answer:
The figure is a trapezoid.

Explanation:
In the above-given question,
given that,
the figure is a trapezoid.
area of the trapezoid = a+b/2 h.
where a = base.
b = base.
h = height.
so the figure is a trapezoid.

Use Vocabulary in Writing
Describe how to find the area of the quadrilateral. Use vocabulary words in your explanation.
Envision Math Common Core Grade 6 Answers Topic 7 Solve Area, Surface Area, And Volume Problems 178

Answer:
Area = 44 sq cm.

Explanation:
In the above-given question,
given that,
area of the quadrilateral = 1/2 x diagonals length x (sum of the height of two triangles).
area = 1/2 x 11 x 8.
area = 44 sq cm.

Concepts and Skills Review

Lesson 7.1 Find Areas of Parallelograms and Rhombuses

Quick Review
You can use the formula A = bh to find the area of a parallelogram or a rhombus.

Example
Find the area of the parallelogram.
Envision Math Common Core Grade 6 Answers Topic 7 Solve Area, Surface Area, And Volume Problems 179
A = bh
A = 12 × 8
A = 96
The area of the parallelogram is 96 ft2.

Practice
In 1-4, find the area of each parallelogram or rhombus.
Question 1.
Envision Math Common Core Grade 6 Answers Topic 7 Solve Area, Surface Area, And Volume Problems 180

Answer:
Area = 36 sq m.

Explanation:
In the above-given question,
given that,
area of the parallelogram = bh.
where b = base and h = height.
area = 4 x 9.
area = 36 sq m.

Question 2.
Envision Math Common Core Grade 6 Answers Topic 7 Solve Area, Surface Area, And Volume Problems 181

Answer:
Area = 63.2 sq cm.

Explanation:
In the above-given question,
given that,
area of the parallelogram = bh.
where b = base and h = height.
area = 8 x 7.9.
area = 63.2 sq cm.

Question 3.
Rhombus
b = 14 in.
h = 9 in.

Answer:
Area = 126 sq in.

Explanation:
In the above-given question,
given that,
area of the parallelogram = bh.
where b = base and h = height.
area = 14 x 9.
area = 126 sq in.

Question 4.
Parallelogram
b = 12 ft
h = 8.5 ft

Answer:
Area = 102 sq ft.

Explanation:
In the above-given question,
given that,
area of the parallelogram = bh.
where b = base and h = height.
area = 12 x 8.5.
area = 102 sq ft.

Question 5.
A rhombus has an area of 375 mm2 and a base of 25 mm. What is its height?

Answer:
height = 15 sq mm.

Explanation:
In the above-given question,
given that,
area of the parallelogram = bh.
where b = base and h = height.
area = 375 sq mm.
base = 25 mm.
375 = 25h.
h = 375/25.
h = 15 sqmm.

Lesson 7.2 Solve Triangle Area Problems

Quick Review
You can use the formula A = \(\frac{1}{2}\)bh to find the area of any triangle.

Example
Find the area of the triangle.
Envision Math Common Core Grade 6 Answers Topic 7 Solve Area, Surface Area, And Volume Problems 182
A = \(\frac{1}{2}\) × (26 × 20)
A = 260 cm2

Practice
Find the area of each triangle.
Question 1.
Envision Math Common Core Grade 6 Answers Topic 7 Solve Area, Surface Area, And Volume Problems 183

Answer:
Area of the triangle = 5 sq yd.

Explanation:
In the above-given question,
given that,
area of the triangle = 1/2 bh.
area = 1/2 x 2 x 5.
area = 5.

Question 2.
Envision Math Common Core Grade 6 Answers Topic 7 Solve Area, Surface Area, And Volume Problems 184
Answer:
Area of the triangle = 24 sq ft.

Explanation:
In the above-given question,
given that,
area of the triangle = 1/2 bh.
area = 1/2 x 8 x 6.
area = 4 x 6.
area = 24 sq ft.

Question 3.
b = 12.4 cm
h = 18 cm

Answer:
Area of the triangle = 111.6 sq cm.

Explanation:
In the above-given question,
given that,
area of the triangle = 1/2 bh.
area = 1/2 x 12.4 x 18.
area = 12.4 x 9.
area = 111.6 sq cm.

Question 4.
b = 3.5 m
h = 6 m

Answer:
Area of the triangle = 10.5 sq m.

Explanation:
In the above-given question,
given that,
area of the triangle = 1/2 bh.
area = 1/2 x 3.5 x 6.
area = 3 x 3.5.
area = 10.5 sq m.

Lesson 7.3 Find Areas of Trapezoids and Kites

Quick Review
You can find the area of a trapezoid by decomposing it into a rectangle and one or more triangles. You can find the area of a kite by decomposing it into triangles.

Example
Find the area of the trapezoid and the kite.
Envision Math Common Core Grade 6 Answers Topic 7 Solve Area, Surface Area, And Volume Problems 185

Practice
Find the area of each trapezoid or kite.
Question 1.
Envision Math Common Core Grade 6 Answers Topic 7 Solve Area, Surface Area, And Volume Problems 186

Answer:
Area of the trapezoid = 32.5 sq cm.

Explanation:
In the above-given question,
given that,
base = 5 cm.
h = 11 cm.
area of the triangle = 1/2(1 x 5).
area = 5/2 = 2.5.
area of the rectangle = 5 x 6.
area = 30.
area of the trapezoid = 2.5 + 30.
area = 32.5 sq cm.

Question 2.
Envision Math Common Core Grade 6 Answers Topic 7 Solve Area, Surface Area, And Volume Problems 187
Answer:
Area of the trapezoid = 96 sq m.

Explanation:
In the above-given question,
given that,
area of the triangle = 1/2(7 x 12).
area = 42.
area of the rectangle = 1 x 12.
area = 12.
area of the trapezoid = 42 + 42 + 12.
area = 96 sq m.

Question 3.
Envision Math Common Core Grade 6 Answers Topic 7 Solve Area, Surface Area, And Volume Problems 188

Answer:
Area = 48 sq in.

Explanation:
In the above-given question,
given that,
area of the kite = 1/2[(8 x 4) x 4].
area = 1/2[12] x 4.
area = 24.
area of the kite = 24 + 24.
area = 48 sq in.

Question 4.
Envision Math Common Core Grade 6 Answers Topic 7 Solve Area, Surface Area, And Volume Problems 189

Answer:
Area of the kite = 80 sq in.

Explanation:
In the above-given question,
given that,
area of the kite = 1/2[(10 x 4) x 2].
area = 1/2[40] x 2.
area = 20 x 2.
area = 40.
area of the kite = 40 + 40.
area = 80 sq in.

Lesson 7.4 Find Areas of Polygons

Quick Review
To find the area of a polygon, you can decompose or compose shapes, then use addition or subtraction to calculate the area.

Example
Find the area of the polygon.
Envision Math Common Core Grade 6 Answers Topic 7 Solve Area, Surface Area, And Volume Problems 190
Area = (3 × 5) + (10 × 4) + (4 × 10)
= 15 + 40 + 40 = 95
The area of the polygon is 95 yd2.

Practice
Question 1.
Find the area of the polygon.
Envision Math Common Core Grade 6 Answers Topic 7 Solve Area, Surface Area, And Volume Problems 191
Answer:
Area of the polygon = -27.

Explanation:
In the above-given question,
given that,
the area of the polygon = (2 x -6) + (-3 x -5).
area of the polygon = (-12) + (-15).
area = -12 – 15.
area = -27.
so the area of the polygon = -27.

Lesson 7.5 Represent Solid Figures Using Nets

Quick Review
You can use nets to represent solid figures.

Example
Classify the solid figure and draw a net to represent it.
Envision Math Common Core Grade 6 Answers Topic 7 Solve Area, Surface Area, And Volume Problems 192
This figure has two congruent parallel bases, so it is a prism. The bases are rectangles, so it is a rectangular prism.
Net of rectangular prism:
Envision Math Common Core Grade 6 Answers Topic 7 Solve Area, Surface Area, And Volume Problems 193

Practice
In 1 and 2, classify the solid figures
Question 1.
Envision Math Common Core Grade 6 Answers Topic 7 Solve Area, Surface Area, And Volume Problems 194

Answer:
The figure name is a triangular pyramid.

Explanation:
In the above-given question,
given that,
the above figure is a triangular pyramid.
the surface area of triangular pyramid = 1/2(a x b) + 3/2(b x s).
where b = base.
s = side.

Question 2.
Envision Math Common Core Grade 6 Answers Topic 7 Solve Area, Surface Area, And Volume Problems 195

Answer:
The figure name is a triangular pyramid.

Explanation:
In the above-given question,
given that,
the above figure is a triangular pyramid.
the surface area of triangular pyramid = 1/2(a x b) + 3/2(b x s).
where b = base.
s = side.

Question 3.
Draw a net of the pyramid shown.
Envision Math Common Core Grade 6 Answers Topic 7 Solve Area, Surface Area, And Volume Problems 196
Answer:

In 4 and 5, identify each solid from its net.
Question 4.
Envision Math Common Core Grade 6 Answers Topic 7 Solve Area, Surface Area, And Volume Problems 197

Answer:
The net of the solid is a square pyramid.

Explanation:
In the above-given question,
given that,
the net of the solid is a square pyramid.
the formula of square pyramid = 1/3 x bxb x h.
where b = base, and h = height.
the figure is a square pyramid.

Question 5.
Envision Math Common Core Grade 6 Answers Topic 7 Solve Area, Surface Area, And Volume Problems 198

Answer:
The net of the solid is a rectangular pyramid.

Explanation:
In the above-given question,
given that,
the net of the solid is a rectangular pyramid.
the formula of rectangular pyramid = 1/3 x l x w x h.
where l = length, w = width, and h = height.
the figure is a rectangular pyramid.

Lesson 7.6 Find Surface Areas of Prisms

Quick Review
You can use a net or a formula to find the surface area of a prism.

Example
Find the surface area.
Envision Math Common Core Grade 6 Answers Topic 7 Solve Area, Surface Area, And Volume Problems 199

Practice
Find the surface area of each prism.
Question 1.
Envision Math Common Core Grade 6 Answers Topic 7 Solve Area, Surface Area, And Volume Problems 200

Answer:
The surface area of the rectangular prism = 126 cubic ft.

Explanation:
In the above-given question,
given that,
l = 6 ft, w = 3 ft, and h = 5 ft.
surface area of rectangular prism = 2(lw + lh + hw).
area = 2(6 x 3) + (6 x 5) + (5 x 3).
area = 2(18) + 30 + 15.
area = 2(63).
area = 126.

Question 2.
Envision Math Common Core Grade 6 Answers Topic 7 Solve Area, Surface Area, And Volume Problems 201
Answer:
The surface area of the triangular pyramid = 7168 cubic cm.

Explanation:
In the above-given question,
given that,
the surface area of the triangular prism = a x b x c x h.
where a = base side.
b = base side.
c = base side.
h = height.
surface area of the triangular prism = 8 x 16 x 8 x 7.
area = 7168 cubic cm.

Question 3.
Cube
s = 9.4 m

Answer:
Area = 530.16 sq m.

Explanation:
In the above-given question,
given that,
s = 9.4 m.
The surface area of a cube is 6a square.
area = 6 x 9.4 x 9.4.
area = 6 x 88.36.
area = 530.16 sq m.

Question 4.
Cube
s = 7 cm

Answer:
Area = 294 sq cm.

Explanation:
In the above-given question,
given that,
s = 9.4 m.
The surface area of a cube is 6a square.
area = 6 x 7 x 7.
area = 6 x 49.
area = 294 sq cm.

Question 5.
Rectangular prism
l = 12 in.
w = 7 in.
h = 3 in.

Answer:
The surface area of the rectangular prism = 282 cubics in.

Explanation:
In the above-given question,
given that,
l = 12 in, w = 7 in, and h = 3 in.
surface area of rectangular prism = 2(lw + lh + hw).
area = 2(12 x 7) + (7 x 3) + (12 x 3).
area = 2(84) + 21 + 36.
area = 2(141).
area = 282.

Question 6.
Rectangular prism
l = 5 cm
w = 6 cm
h = 7 cm

Answer:
The surface area of the rectangular prism = 214 cubics cm.

Explanation:
In the above-given question,
given that,
l = 5 cm, w = 6 cm, and h = 7 cm.
surface area of rectangular prism = 2(lw + lh + hw).
area = 2(5 x 6) + (7 x 6) + (5 x 7).
area = 2(30) + 42 + 35.
area = 2(107).
area = 214.

Lesson 7.7 Find Surface Areas of Pyramids

Quick Review
You can use a net or a formula to find the surface area of a pyramid.

Example
Find the surface area.
Area of one triangle:
Envision Math Common Core Grade 6 Answers Topic 7 Solve Area, Surface Area, And Volume Problems 202
T = \(\frac{1}{2}\) × 10 × 8.7 = 43.5
SA = 4T
= 4 × 43.5
= 174
The surface area of the pyramid is 174 m2.

Practice
In 1 and 2, find the surface area of each pyramid.
Question 1.
Envision Math Common Core Grade 6 Answers Topic 7 Solve Area, Surface Area, And Volume Problems 203

Answer:
The surface area of the pyramid = 10 sq in.

Explanation:
In the above-given question,
given that,
bases = 4 in.
height = 5 in.
surface area of the pyramid = 1/2 (p x s).
area = 1/2 (4 x 5).
area = 1/2(20).
area = 10 sq in.

Question 2.
Envision Math Common Core Grade 6 Answers Topic 7 Solve Area, Surface Area, And Volume Problems 204
Answer:
The surface area of the pyramid = 21.35 sq cm.

Explanation:
In the above-given question,
given that,
bases = 7 cm.
height = 6.1 cm.
surface area of the pyramid = 1/2 (p x s).
area = 1/2 (7 x 6.1).
area = 1/2(42.7).
area = 21.35 sq cm.

Question 3.
Each side of the base of a square pyramid is 10 ft and the height of each triangular face is 7 ft. Find the surface area of the pyramid.

Answer:
The surface area of the pyramid = 607 sq ft.

Explanation:
In the above-given question,
given that,
Each side of the base of a square pyramid is 10 ft and the height of each triangular face is 7ft.
surface area of the pyramid = axa + 2a √axa/4 + h x h.
where a = base edge.
h = height.
area = 100+ 20 √100/4 + 49.
area = 120 √25 + 49.
area = 120 x 5 + 7.
area = 600 + 7.
area = 607.

Lesson 7.8 Find Volume with Fractional Edge Lengths

Quick Review
The volume of a rectangular prism is equal to the area of the base multiplied by the height.

Example
Find the volume of the rectangular prism.
Envision Math Common Core Grade 6 Answers Topic 7 Solve Area, Surface Area, And Volume Problems 205
STEP 1 Find the number of small \(\frac{1}{4}\)-in. cubes that will fill the prism.
14 small \(\frac{1}{4}\)-in. cubes 31in. fit along the 3\(\frac{1}{2}\) in. side.
12 small \(\frac{1}{4}\)-in. cubes fit along the 3 in. side.
15 small \(\frac{1}{4}\)-in. cubes fit along the 3\(\frac{3}{4}\) in. side.
14 • 12 • 15 = 2,520 small \(\frac{1}{4}\)-in. cubes fill the prism.

STEP 2
Find the volume of each small 7-in. cube.
V = lwh = \(\frac{1}{4}\) in. • \(\frac{1}{4}\) in. • \(\frac{1}{4}\) in. = \(\frac{1}{64}\) in.3

STEP 3
Find the volume of the prism.
2,520 • \(\frac{1}{64}\) in.3 = 39\(\frac{3}{8}\) in.3
You can also use a formula. .
V = lwh = 3\(\frac{1}{2}\) in. × 3 in. × 3\(\frac{3}{4}\) in. = 39\(\frac{3}{8}\) in.3

Practice
Find the volume of each rectangular prism.
Question 1.
Envision Math Common Core Grade 6 Answers Topic 7 Solve Area, Surface Area, And Volume Problems 206

Answer:
Volume = 777.45 cubic in.

Explanation:
In the above-given question,
given that,
volume of rectangular prism = l x w x h.
where l = 5 in, w = 22/3 in, and h = 64/3 in.
v = 5 x 7.3 x 21.3.
v = 777.45 cubic in

Question 2.
Envision Math Common Core Grade 6 Answers Topic 7 Solve Area, Surface Area, And Volume Problems 207

Answer:
Volume = 234 cubic ft.

Explanation:
In the above-given question,
given that,
volume of rectangular prism = l x w x h.
where l = 8 ft, w = 4.5 ft, and h = 6.5 ft.
v = 8 x 4.5 x 6.5.
v = 234 cubic ft.

Question 3.
Envision Math Common Core Grade 6 Answers Topic 7 Solve Area, Surface Area, And Volume Problems 208

Answer:
Volume = 222.794 cubic cm.

Explanation:
In the above-given question,
given that,
volume of rectangular prism = l x w x h.
where l = 14.3 cm, w = 3.8 cm, and h = 4.1 cm.
v = 14.3 x 3.8 x 4.1.
v = 222.794 cubic cm.

Topic 7 Fluency Practice

Hidden Clue
For each ordered pair, simplify the two coordinates. Then locate and label the corresponding point on the graph. Draw line segments to connect the points in alphabetical order. Connect L to E to complete the picture, then use the completed picture to help answer the riddle below.
I can… multiply and divide multidigit decimals.
Envision Math Common Core Grade 6 Answers Topic 7 Solve Area, Surface Area, And Volume Problems 209
Envision Math Common Core Grade 6 Answers Topic 7 Solve Area, Surface Area, And Volume Problems 210

Answer:
A: (1.86, 2).
B: (2.4, 0.8).
C: (3.6, 1.25).
D: (5.4, 4.76).
E: (7.2, 8.2).
F: (13.6, 5).
G: (13.1, 8.8).
H: (11.8, 10.5).
I: (9.6, 12.7).
J: (6.4, 13.2).
K: (4.1, 13.2).
L: (1, 11.5).

Explanation:
In the above-given question,
given that,
multiply and divide the numbers.
A: (1.2 x 1.55, 2.7/1.35).
(1.86, 2).
B: (1.25 x 1.92, 0.48/0.6).
(2.4, 0.8).
C: (5.04/1.4, 0.5 x 2.5).
(3.6, 1.25).
D: (2.16 x 2.5, 11.9/2.5).
(5.4, 4.76).
E: (16.56/ 2.3, 5.125 x 1.6).
(7.2, 8.2).
F: (6.12/0.45, 6.25 x 0.8).
(13.6, 5).
G: (2.62 x 5, 12.32/1.4).
(13.1, 8.8).
H: (29.5 x 0.4, 3.675 /0.35).
(11.8, 10.5).
I: (12.48/1.3, 10.16 x 1.25)
(9.6, 12.7).
J: (20 x 0.32, 59.4/4.5).
(6.4, 13.2).
K: (7.79/1.9, 40 x 0.33).
(4.1, 13.2).
L: (25 x 0.04, 48.3/4.2).
(1, 11.5).

Envision Math Common Core Grade 6 Answer Key Topic 4 Represent And Solve Equations And Inequalities

Practice with the help of enVision Math Common Core Grade 6 Answer Key Topic 4 Represent and Solve Equations and Inequalities regularly and improve your accuracy in solving questions.

Envision Math Common Core 6th Grade Answers Key Topic 4 Represent And Solve Equations And Inequalities

?Topic Essential Question What procedures can be used to write and solve equations and inequalities?
Answer:
The three methods most commonly used to solve systems of equation are substitution, and elimination. Substitution and elimination are simple methods that can effectively solve most systems of two equations in a few straightforward steps
The steps to solve an inequality are:
a. Add the same number to both sides.
b. Subtract the same number from both sides.
c. Multiply both sides by the same positive number.
d. Divide both sides by the same positive number.
e. Multiply both sides by the same negative number and reverse the sign

3-ACT MATH

Envision Math Common Core 6th Grade Answer Key Topic 4 Represent And Solve Equations And Inequalities 1

Checking a Bag
A large plane flying across the ocean can weigh almost 1 million pounds! The heavier an airplane is, the more fuel it needs for a flight. The cost of fuel has led many airlines to add a weight restriction on luggage.
If you were to fly somewhere, what would you bring? What would you leave at home to minimize the weight of your luggage? Packing light is important, not only to avoid a fee but also to do your part to conserve fuel. Think about this during the 3-Act Mathematical Modeling lesson.
Envision Math Common Core 6th Grade Answer Key Topic 4 Represent And Solve Equations And Inequalities 2

Topic 4 EnVision STEM Project

Did You Know?
The design of a bridge depends on factors such as the distance the bridge will cover, the expected number of vehicles that will cross the bridge daily, and the geographic conditions.
Envision Math Common Core 6th Grade Answer Key Topic 4 Represent And Solve Equations And Inequalities 3

Your Task: Design a Bridge
Now that you have defined the problem, identified the criteria and constraints, and performed some data collection, it is time to focus on the solution. You and your classmates will continue to be engineers as you brainstorm solutions and develop prototypes for your bridge.
Envision Math Common Core 6th Grade Answer Key Topic 4 Represent And Solve Equations And Inequalities 4

Topic 4 Get Ready!

Review What You Know!

Vocabulary
Choose the best term from the box to complete each definition.

  • algebraic expression
  • coefficient equation
  • evaluate
  • variable

Question 1.
In 6x, x is a(n) _________ .
Answer:
We know that,
In 6x,
“6” is called as “Coefficient”
“x” is called as “Variable”
Hence, from the above,
We can conclude that
The best term that is suitable for the given statement is: Variable

Question 2.
x + 5 is an example of a(n) __________
Answer:
We know that,
The expressions are of 2 types. They are:
a. Numeric Expressions – The expression that contains only numbers and arithmetic symbols
Ex: 6 + 2 – 5
b. Algebraic expressions – The expression that contains variables, numbers,and arithmetic symbols
Ex: x + 2 – 5 + y
So,
x + 5 is an example of “Algebraic expression”
Hence, from the above,
We can conclude that
The best term that is suitable for the given statement is: Algebraic expression

Question 3.
__________ an expression to find its value.
Answer:
We know that,
“Evaluate” an expression to find its value
Hence, from the above,
We can conclude that
The best term that is suitable for the given statement is: Evaluate

Question 4.
The expressions on each side of the equal sign in a(n) __________ are equal.
Answer:
We know that,
The expressions on each side of the equal sign in an “Equation” are equal
Hence, from the above,
We can conclude that
The best term that is suitable for the given statement is: Equation

Equality

Tell whether the equation is true or false.
Question 5.
6 + 2 = 2 + 6
Answer:
The given equation is:
6 + 2 = 2 + 6
Now,
We know that,
According to the Commutative Property of Addition,
a + b = b + a
Hence, from the above,
We can conclude that
The given equation is: True

Question 6.
2.5 – 1 = 1 – 2.5
Answer:
The given equation is:
2.5 – 1 = 1 – 2.5
Now,
We know that,
According to the Commutative Property,
a – b ≠ b – a
Hence, from the above,
We can conclude that
The given equation is: False

Question 7.
\(\frac{1}{2}\) × 3= 3 × \(\frac{1}{2}\)
Answer:
The given equation is:
\(\frac{1}{2}\) × 3 = 3 × \(\frac{1}{2}\)
Now,
We know that,
According to the Commutative Property of Multiplication,
a × b = b × a
Hence, from the above,
We can conclude that
The given equation is: True

Question 8.
\(\frac{3}{4} \div 5=\frac{3}{4} \times \frac{1}{5}\)
Answer:
The given equation is:
\(\frac{3}{4}\) ÷ 5 = \(\frac{3}{4}\) × \(\frac{1}{5}\)
Now,
We know that,
When any number is present after the “Division” symbol, then it will turn into its reciprocal when it is converted into “Multiplication” and vice-versa
So,
÷ 5 will convert into × \(\frac{1}{5}\)
Hence, from the above,
We can conclude that
The given equation is: True

Question 9.
\(5 \div \frac{1}{3}=\frac{5}{3}\)
Answer:
The given equation is:
5 ÷ \(\frac{1}{3}\) = \(\frac{5}{3}\)
Now,
We know that,
When any number is present after the “Division” symbol, then it will turn into its reciprocal when it is converted into “Multiplication” and vice-versa
So,
÷ \(\frac{1}{3}\) will be converted into × 3
Hence, from the above,
We can conclude that
The given equation is: False

Question 10.
\(\frac{2}{3} \times 5=\frac{10}{15}\)
Answer:
The given equation is:
\(\frac{2}{3}\) × 5 = \(\frac{10}{15}\)
Now,
\(\frac{2}{3}\) × 5
= \(\frac{2 × 5}{3}\)
= \(\frac{10}{3}\)
Hence, from the above,
We can conclude that
The given equation is: False

Expressions

Evaluate each expression.
Question 11.
x – 2 for x = 8
Answer:
The given expression is: x – 2
Now,
For x = 8,
x – 2 = 8 – 2
= 6
Hence, from the above,
We can conclude that
The value of the given expression is: 6

Question 12.
2b for b = 9
Answer:
The given expression is: 2b
Now,
For b = 9,
2b = 2 × b
= 2 × 9
= 18
Hence, from the above,
We can conclude that
The value of the given expression is: 18

Question 13.
3\(\frac{3}{4}\) + y for y = \(\frac{5}{6}\)
Answer:
The given expression is: 3\(\frac{3}{4}\) + y
Now,
For y = \(\frac{5}{6}\),

Hence, from the above,
We can conclude that
The value of the given expression is: 4\(\frac{7}{12}\)

Question 14.
\(\frac{15}{x}\) for x = 3
Answer:
The given expression is: \(\frac{15}{x}\)
Now,
For x = 3,
\(\frac{15}{x}\) = \(\frac{15}{3}\)
= 5
Hence, from the above,
We can conclude that
The value of the given expression is: 5

Question 15.
5.6t for t = 0.7
Answer:
The given expression is: 5.6t
Now,
For t = 0.7,
5.6t = 5.6 × t
= 5.6 × 0.7
= 3.92
Hence, from the above,
We can conclude that
The value of the given expression is: 3.92

Question 16.
4x for x = \(\frac{1}{2}\)
Answer:
The given expression is: 4x
Now,
For x = \(\frac{1}{2}\),
4x = 4 × \(\frac{1}{2}\)
= \(\frac{4}{2}\)
= 2
Hence, from the above,
We can conclude that
The value of the given expression is: 2

Order of Operations

Question 17.
Explain the order in which you should compute the operations in the expression below. Then evaluate the expression.
[(33 ÷ 3) + 1] – 22
Answer:
The given expression is: [(33 ÷ 3) + 1] – 2²
Now,
We know that,
We will follow the order of operations by uisng the BODMAS rule
Where,
B – Brackets
O – Of
D – Division
M – Multiplication
A – Addition
S – Subtraction
So,
In the given expression,
The order of expressions we have to compute from the first to the last is:
a. 33 ÷ 3
b. [(33 ÷ 3) + 1]
c. 2²
d. [(33 ÷ 3) + 1] – 2²
So,
[(33 ÷ 3) + 1] – 2²
= [11 + 1] – 2²
= 12 – 4
= 8
Hence, from the above,
We can conclude that
The value of the given expression is: 8

Graphing in the Coordinate Plane

Question 18.
Describe how to plot point A(-6, 2) on a coordinate plane.
Answer:
The given point is: A (-6, 2)
Now,
When we observe the given point,
The x-coordinate of point A is negative and the y-coordinate of point A is positive
So,
For point A,
The x-coordinate is at the left side of the coordinate plane and the y-coordinate is at the top side of the coordinate plane
Hence,
The representation of A (-6, 2) in the coordinate plane is:

Language Development
Use the graphic organizer to help you understand new vocabulary terms.
Envision Math Common Core 6th Grade Answer Key Topic 4 Represent And Solve Equations And Inequalities 5
Answer:

Pick A Project

PROJECT 4A
If you were going to try a new exercise, what would it be? Why?
PROJECT: ANALYZE AN EXERCISE ROUTINE
Envision Math Common Core 6th Grade Answer Key Topic 4 Represent And Solve Equations And Inequalities 6

PROJECT 4B
What is the most interesting book you have read?
PROJECT: WRITE AND ILLUSTRATE A CHILDREN’S BOOK
Envision Math Common Core 6th Grade Answer Key Topic 4 Represent And Solve Equations And Inequalities 7

PROJECT 4C
If you were a carpenter what sorts of things would you build?
PROJECT: MAKE A MODEL OF A STAIRCASE
Envision Math Common Core 6th Grade Answer Key Topic 4 Represent And Solve Equations And Inequalities 8

PROJECT 4D
What skills would you need if you wanted to move as slowly as a snail?
PROJECT: PLAN A RACE
Envision Math Common Core 6th Grade Answer Key Topic 4 Represent And Solve Equations And Inequalities 9

Lesson 4.1 Understand Equations and Solutions

Solve & Discuss It!
Unit cubes are placed on a pan balance. There are 3 cubes on one pan and 9 cubes on the other pan. What can you do to make the pans balance?

Model with Math
A pan balance can be used to represent the relationship between two quantities. You can write an equation with a variable to show this relationship.
I can… determine if a value for a variable makes an equation true.
Envision Math Common Core 6th Grade Answer Key Topic 4 Represent And Solve Equations And Inequalities 10
Answer:
It is given that
Unit cubes are placed on a pan balance. There are 3 cubes on one pan and 9 cubes on the other pan.
Now,
Let the number of cubes on one pan that has to be added as to make the pan balance be: x
Let the number of cubes on other pan that has to be added as to make the pan balance be: y
So,
The number of cubes present on one pan = x + 3
The number of cubes present on other pan = y + 9
So,
To make the given pans balanced,
x + 3 = y + 9
Hence, from the above,
We can conclude that
The equation that makes the pans balances is:
x + 3 = y + 9

Focus on math practices
Use Structure Suppose that you added 10 cubes to the pan with 3 cubes and then added 4 cubes to the pan with 9 cubes. Would the pans balance? Write an equation to show this relationship.
Answer:
It is given that
you added 10 cubes to the pan with 3 cubes and then added 4 cubes to the pan with 9 cubes
So,
The number of cubes present on one pan = 10 + 3 = 13 cubes
The number of cubes present on other pan = 9 + 4 = 13 cubes
So,
The number of cubes present on both pans are the same
Hence, from the above,
We can conclude that
The pans are balanced

Essential Question
How can you determine whether a given number makes an equation true?
Answer:
We can check if the given solution to an expression is actually the solution the expression by replacing the variables in the equation with the given solution and solving the equation. If LHS = RHS with the given solution, the solution is correct. If not, it’s incorrect.

Try It!

Tracy received a $21.00 gift card for phone apps. She has used $9.00 of the value and wants to buy one more app from the list above to use up the balance. Complete the bar diagram and use the equation $21.00 = x + $9.00 to determine which app she should buy.
Envision Math Common Core 6th Grade Answer Key Topic 4 Represent And Solve Equations And Inequalities 11
The solution is _________, so Tracy should buy the _________ app.
Answer:
It is given that
Tracy received a $21.00 gift card for phone apps. She has used $9.00 of the value and wants to buy one more app from the list above to use up the balance
Now,
The given figure is:

Now,
Let x be the cost of the another app that Tracy wanted to buy
So,
According to the given information,
The equation that represents the used up balance is:
$9 + $x = $21
Now,
The representation of the above equation in the form of a bar diagram is:

So,
$x = $21 – $9
$x = $12
So,

Hence, from the above,
We can conclude that
Tracy should buy the “Remote Desktop” app with her balance of $12

Convince Me! What do you notice about the expression on the left side of an equation compared to the expression on the right side when a value is substituted for the variable? How do you know which value is a solution?
Answer:
When we observe the expression that is present on the left side of an equation and the expression that is present on the right side of an equation when a value is substituted for the variable,
We can say that
The left hand side of the equation = The right hand side of the equation
Now,
The solution for the equation is the value we substituted for a “Variable”

Try It!

Anthony has a total of y marbles and 4 boxes. He puts 13 marbles in each box and has none left over. Which of his friends, if any, correctly guessed how many marbles Anthony has in all? Use the equation y ÷ 4 = 13.
Substitute each guess for y and evaluate.
Envision Math Common Core 6th Grade Answer Key Topic 4 Represent And Solve Equations And Inequalities 12
Try y = 48: _______ ÷ 4 = _______
Try y = 60: _______ ÷ 4 = _______
Try y = 120: _______ ÷ 4 = _______

Of Anthony’s three friends, _______ correctly guessed the number of marbles he has in all.
No solution is given in the set of values.
Anthony has _______ marbles in all.
Answer:
It is given that
Anthony has a total of y marbles and 4 boxes. He puts 13 marbles in each box and has none left over.
Now,
The representation of the given situation in the form of a bar diagram is:

Now,
The given table is:

Now,
To find whether any friend of Anthony guesses correctly or not,
Substitute the values of marbles below

So,

Now,
The given equation that represents the number of marbles Anthony has is:
y ÷ 4 = 13
y = 13 × 4
y = 52 marbles
So,

Hence, from the above,
We can conclude that
The number of marbles Anthony has in all is: 52 marbles

KEY CONCEPT
A solution of an equation is a value for the variable that makes the equation true. Substitute values from a given set for the variable and evaluate. x – 4 = 12 x = 9, 16
Envision Math Common Core 6th Grade Answer Key Topic 4 Represent And Solve Equations And Inequalities 13

Do You Understand?
Question 1.
Essential Question How can you determine whether a given number makes an equation true?
Answer:
We can check if the given solution to an expression is actually the solution the expression by replacing the variables in the equation with the given solution and solving the equation. If LHS = RHS with the given solution, the solution is correct. If not, it’s incorrect.

Question 2.
When is an equation true?
Answer:
To make a true equation, check your math to make sure that the values on each side of the equals sign are the same. Ensure that the numerical values on both sides of the “=” sign are the same to make a true equation

Question 3.
Reasoning Ben says that n = 5 is the solution of the equation 7n = 45. How can you check whether Ben is correct?
Answer:
It is given that
Ben says that n = 5 is the solution of the equation 7n = 45
Now,
The given equation is:
7n = 45
Now,
substitute the given value of n in the given equation
So,
For the given equation,
If  LHS = RHS, then
Ben is correct
If LHS ≠ RHS, then
Ben is not correct

Question 4.
A pan balance has 3 cubes on one pan and 11 cubes on the other pan. Lucy thinks she should add 7, 8, 9, or 10 cubes to make the pans balance. How can you use the equation 3 + C = 11 to find the number of cubes Lucy should add?
Answer:
It is given that
A pan balance has 3 cubes on one pan and 11 cubes on the other pan. Lucy thinks she should add 7, 8, 9, or 10 cubes to make the pans balance.
Now,
According to the given information,
The number of cubes on one pan = 3 cubes
The number of cubes on other pan = 11 cubes
Now,
It is given that
Lucy wants to add the extra cubes with 3 cubes
So,
The number of cubes in the second pan is constant
Now,
Let the number of extra cubes on one pan to be added so that the pan should be balanced be: C
So,
3 + C = 11
So,
C = 11 – 3
C = 9 cubes
So,
Lucy should add 9 cubes to make the pans balance
Hence, from the above,
We can conclude that
Lucy should add 9 cubes on one pan to make both the pans balance

Do You Know How?
In 5-8, substitute each given value of the variable to find which, if any, is a solution of the equation.
Question 5.
d + 9 = 35; d = 16, 22, 26, 36
Answer:
The given equation is:
d + 9 = 35
Now,
For d = 16,
16 + 9 = 35
25 ≠ 35
For d = 22,
22 + 9 = 35
31 ≠ 35
For d = 26,
26 + 9 = 35
35 = 35
For d = 36,
36 + 9 = 35
45 ≠ 35
Hence, from the above,
We can conclude that
d = 26 is a solution to the given equation

Question 6.
14n = 35; n = 2, 3, 3.5, 4
Answer:
The given equation is:
14n = 35
Now,
For n = 2,
14 (2) = 35
28 ≠ 35
For n = 3,
14 (3) = 35
42 ≠ 35
For n = 3.5,
14 (3.5) = 35
49 ≠ 35
For n = 4,
14 (4) = 35
56 ≠ 35
Hence, from the above,
We can conclude that
The given values of n are not any solutions for the given equation

Question 7.
13.4 – g = 8.1; g = 4.3, 5.3, 5.5, 6.5
Answer:
The given equation is:
13.4 – g = 8.1
Now,
For g = 4.3,
13.4 – 4.3 = 8.1
9.1 ≠ 8.1
For g = 5.3,
13.4 – 5.3 = 8.1
8.1 = 8.1
For g = 5.5,
13.4 – 5.5 = 8.1
7.9 ≠ 8.1
For g = 6.5,
13.4 – 6.5 = 8.1
6.9 ≠ 8.1
Hence, from the above,
We can conclude that
g = 5.3 is a solution to the given equation

Question 8.
4 = 36 ÷ m; m = 4, 6, 8, 9
Answer:
The given equation is:
4 = 36 ÷ m
Now,
For m = 4,
4 = 36 ÷ 4
4 ≠ 9
For m = 6,
4 = 36 ÷ 6
4 ≠ 6
For m = 8,
4 = 36 ÷ 8
4 ≠ 4.5
For m = 9,
4 = 36 ÷ 9
4 = 4
Hence, from the above,
We can conclude that
m = 9 is a solution to the given equation

In 9-12, tell whether each equation is true or false for n = 8.
Question 9.
n = 54 – 36
Answer:
The given equation is:
n = 54 – 36
Now,
n = 54 – 36
= 18
So,
8 ≠ 18
Hence, from the above,
We can conclude that
The given equation is False for n = 8

Question 10.
5n = 40
Answer:
The given equation is:
5n = 40
Now,
5n = 40
n = \(\frac{40}{5}\)
= 8
So,
8 = 8
Hence, from the above,
We can conclude that
The given equation is True for n = 8

Question 11.
152 ÷ n = 21
Answer:
The given equation is:
152 ÷ n = 21
Now,
152 ÷ n = 21
n = \(\frac{152}{21}\)
= 0.7
So,
0.7 ≠ 8
Hence, from the above,
We can conclude that
The given equation is False for n = 8

Question 12.
n + 46 = 54
Answer:
The given equation is:
n + 46 = 54
Now,
n + 46 = 54
n = 54 – 46
= 8
So,
8 = 8
Hence, from the above,
We can conclude that
The given equation is True for n = 8

Practice & Problem Solving

In 13-16, tell which given value, if any, is a solution of the equation.
Question 13.
t – 2.1 = 0           t = 2.1, 2.4, 2.6, 2.8
Answer:
The given equation is:
t – 2.1 = 0
Now,
t – 2.1 = 0
Now,
Add with 2.1 on both sides
So,
t – 2.1 + 2.1 = 0 + 2.1
t = 2.1
Hence, from the above,
We can conclude that
t = 2.1 is the solution for the given equation

Question 14.
49 = 7r            r = 3, 6, 7, 9
Answer:
The given equation is:
49 = 7r
Now,
49 = 7r
Divide by 7 on both sides
So,
\(\frac{49}{7}\) = \(\frac{7r}{7}\)
7 = r
Hence, from the above,
We can conclude that
r = 7 is the solution for the given equation

Question 15.
$4.10 = $6.25 – y           y = $2.15, $2.95, $3.05, $3.15
Answer:
The given equation is:
$4.10 = $6.25 – y
Now,
$4.10 = $6.25 – y
y = $6.25 – $4.10
y = $2.15
Hence, from the above,
We can conclude that
y = $2.15 is the solution for the given equation

Question 16.
24 ÷ h = 6         h = 1, 3, 6, 8
Answer:
The given equation is:
24 ÷ h = 6
Now,
24 ÷ h = 6
h = \(\frac{24}{6}\)
h = 4
Hence, from the above,
We can conclude that
There is not any solution from the given values for the given equation

Question 17.
In the past, Marcie’s father rode his bike 108 miles in 7.5 hours. Her mother rode the same distance in 8 hours. Marcie plans to ride her bike 108 miles at a steady rate of 18 mph for y hours. Will she match her father’s or mother’s time? Use the equation 108 ÷ y = 18 to justify your answer.
Answer:
It is given that
In the past, Marcie’s father rode his bike 108 miles in 7.5 hours. Her mother rode the same distance in 8 hours. Marcie plans to ride her bike 108 miles at a steady rate of 18 mph for y hours.
Now,
We know that,
Speed = \(\frac{Distance}{Time}\)
Time = \(\frac{Distance}{Speed}\)
So,
According to the given information,
The time took for Marcie to ride her bike at a steady rate (t) = (The distance rode by Marcie) ÷ (The speed of Marcie’s bike)
= 108 ÷ 18
= 6 hours
Now,
From the given information,
The time took for Marcie’s father to ride his bike is: 7.5 hours
The time took for Marcie’s mother to ride her bike is: 8 hours
So,
6 ≠ 7.5 ≠ 8
Hence, from the above,
We can conclude that
Marcie will not match her father’s or mother’s time

Question 18.
Write if b = 6 is a solution or is not a solution of each equation.
a. 8b = 48
b. 11 – b = 6
c. b + 3 = 9
d. 54 ÷ b = 9
Answer:
The given expression sare:
a. 8b = 48
b. 11 – b = 6
c. b + 3 = 9
d. 54 ÷ b = 9
Now,
a.
For b = 6,
8 (6) = 48
48 = 48
b.
For b = 6,
11 – 6 = 6
5 ≠ 6
c.
For b = 6,
6 + 3 = 9
9 = 9
d.
For b = 6,
54 ÷ 6 = 9
9 = 9
Hence, from the above,
We can conclude that
The equations that have a solution b = 6 are:

Question 19.
A group of 4 friends is planning a fun day trip. The equations in the table represent the number of people n who can participate in each activity for $29.
Envision Math Common Core 6th Grade Answer Key Topic 4 Represent And Solve Equations And Inequalities 14
Which activity should the friends choose if they want to spend exactly $29?
Answer:
It is given that
A group of 4 friends is planning a fun day trip. The equations in the table represent the number of people n who can participate in each activity for $29.
Now,
The given table is:

Now,
For the friends to choose an activity so as they spend exactly $29,
The solutions of the variables should satisfy their own equations
Now,
a. For the Raft Trip activity:
The given equation is:
6n + 5 = 29
6n = 29 – 5
6n = 24
n = \(\frac{24}{6}\)
n = 4
Verification:
6 (4) + 5 = 29
24 + 5 = 29
29 = 29
b. For the Amusement park Trip activity:
The given equation is:
14n = 29
n = \(\frac{29}{14}\)
n = 2.07
n ≅ 2
Verification:
14 (2) = 29
28 ≠ 29
c. For the Balloon Ride activity:
30n – 40 = 29
30n = 20 + 49
30n = 69
n = \(\frac{69}{30}\)
n = \(\frac{23}{10}\)
n = 2.3
n ≅ 2
Verification:
30 (2) – 40 = 29
60 – 40 = 29
20 ≠ 29
Hence, from the above,
We can conclude that
The activity the friends should choose if they want to spend exactly $29 is: Raft Trip

Question 20.
There are 27 pennies on one pan of a pan balance and 18 pennies on the other. To make the pans balance, Hillary thinks 5 pennies should be added to the higher pan. Sean thinks 8 pennies should be added, and Rachel thinks 9 pennies should be added. Use the equation 27 = 18 + p to determine who is correct.
Answer:
It is given that
There are 27 pennies on one pan of a pan balance and 18 pennies on the other. To make the pans balance, Hillary thinks 5 pennies should be added to the higher pan. Sean thinks 8 pennies should be added, and Rachel thinks 9 pennies should be added
Now,
Let the nuber of pennies added be: p
Now,
The given equation in the problem that represents the situation is:
27 = 18 + p
So,
On the basis of the given information,
According to Hillary, p = 5
According to Sean, p = 8
According to Rachel, p = 9
Now,
For p = 5,
27 = 18 + p
27 = 18 + 5
27 ≠ 23
Now,
For p = 8,
27 = 18 + p
27 = 18 + 8
27 ≠ 26
Now,
For p = 9,
27 = 18 + p
27 = 18 + 9
27 = 27
Hence, from the above,
We can conclude that
Rachel is correct

Question 21.
Construct Arguments Gerard spent $5.12 for a drink and a sandwich. His drink cost $1.30. Did he have a ham sandwich for $3.54, a tuna sandwich for $3.82, or a turkey sandwich for $3.92? Use the equation s + 1.30 = 5.12 to justify your answer.
Answer:
It is given that
Gerard spent $5.12 for a drink and a sandwich. His drink cost $1.30
Now,
Let the cost of sandwich be: $s
So,
According to the given information,
$5.12 = s + $1.30
Subtract $1.30 on both sides
So,
$5.12 – $1.30 = s + $1.30 – $1.30
s = $3.82
Now,
According to the given information,
The cost of tuna sandwich is: $3.82
Hence, from the above,
We can conclude that
Gerard will have a tuna sandwich

Question 22.
Higher Order Thinking Write an equation that has a solution of 12. Show how you know that 12 is the solution.
Answer:
Let the equation that has a solution of 12 be:
6x = 72
Now,
We know that,
To find whether the solution is correct or not, substitute the solution in the place of the variable present in that equation and if LHS = RHS, then that solution is true for the equation
So,
The equation is: 6x = 72
Now,
Substitute 12 in place of x
So,
6 (12) = 72
Hence,
72 = 72

Question 23.
Gina’s family is driving 255 miles to visit Tallahassee. After driving for a while, they pass a sign that reads “Tallahassee: 124 miles.” Substitute the values m = 111, 121, 131, and 141 in the equation 255 – m = 124 to find the number of miles the family has already driven.
Answer:
It is given that
Gina’s family is driving 255 miles to visit Tallahassee. After driving for a while, they pass a sign that reads “Tallahassee: 124 miles.”
Now,
The given equation to represent the number of miles the Gina’s family has already driven is:
255 – m = 124 with m = 111, 121, 131, and 141
Now,
For m = 111,
255 – m = 124
255 – 111 = 124
144  ≠ 124
Now,,
For m = 121,
255 – m = 124
255 – 121 = 124
134  ≠ 124
Now,
For m = 131,
255 – m = 124
255 – 131 = 124
124  = 124
Now,
For m = 141,
255 – m = 124
255 – 141 = 124
114  ≠ 124
Hence, from the above,
We can conclude that
The number of miles the Gina’s family has already driven is: 131 miles

Question 24.
Lisa is making a quilt that uses a pattern of triangles like the one shown. Write an equation that represents the missing side length if the perimeter is 19 centimeters
Envision Math Common Core 6th Grade Answer Key Topic 4 Represent And Solve Equations And Inequalities 15
Answer:
It is given that
Lisa is making a quilt that uses a pattern of triangles like the one shown
Now,
The given figure is:
Envision Math Common Core 6th Grade Answer Key Topic 4 Represent And Solve Equations And Inequalities 15
Now,
We know that,
The “Perimeter” of a figure is the sum of all the side lengths of a figure
So,
For the given figure,
Perimeter = 5 + (3 + 3) + m + 3
19 = 5 + 6 + m + 3
19 = 14 + m
m = 19 – 14
m = 5 cm
Hence, from the above,
We can conclude that
The missing side length is: 5 cm

Question 25.
Alisa’s family planted 7 palm trees in their yard. The park down the street has 147 palm trees. Alisa guessed that the park has either 11 or 31 times as many palm trees as her yard has. Is either of Alisa’s guesses correct? Use the equation 7n = 147 to justify your answer.
Answer:
It is given that
Alisa’s family planted 7 palm trees in their yard. The park down the street has 147 palm trees. Alisa guessed that the park has either 11 or 31 times as many palm trees as her yard has.
Now,
The equation that represents the given information that is given in the problem is:
7n = 147
So,
n = \(\frac{147}{7}\)
n = 21 palm trees
But,
According to the given information,
Alisa had guessed that the park has either 11 or 31 times as many palm trees as her yard has
Hence, from the above,
We can conclude that
Alisa’s guesses are not correct

Assessment Practice

Question 26.
Trish has $26.00 to spend at a craft store. She buys fabric that costs $18.62. She also wants to buy knitting needles for $7.32, silk flowers for $7.38, or oil paints for $8.48.
Use the equation $18.62 + c = $26.00, where c is the item cost, to find the most expensive item Trish can buy Explain how you found your answer.
Answer:
It is given that
Trish has $26.00 to spend at a craft store. She buys fabric that costs $18.62. She also wants to buy knitting needles for $7.32, silk flowers for $7.38, or oil paints for $8.48.
Now,
The equation that represents the given information and is mentioned in the problem is:
$18.62 + c = $26.00
So,
c = $26.00 – $18.62
c = $7.38
So,
Trish can buy silk flowers with her balance
Hence, from the above,
We can conclude that
The most expensive item Trish can buy with the remaining amount of money is: Silk flowers

Lesson 4.2 Apply Properties of Equality

Solve & Discuss It!
Start with the equation 4 + 8 = 12 and complete each computation listed below. Do each computation individually. Which of the computations keeps the equation true? Explain.

I can… use the properties of equality to write equivalent equations.
Envision Math Common Core 6th Grade Answer Key Topic 4 Represent And Solve Equations And Inequalities 16
Answer:
The given equation is:
4 + 8 = 12
Now,
We know that,
The equation will be true only when the equations’s
LHS = RHS
Now,
The completed table with the individual computations and the result as “True” and “False” is:

Hence, from the above,
We can conclude that
The computations that keep the equation true are:
a. Add 5 to both sides of the equation
b. Subtract 4 from both sides of the equation

Reasoning
How can you determine whether an equation is true?
Answer:
From above,
The completed table is:

Now,
We know that,
According to the Properties of Equality,
In an equation,
The operation we performed with the given number on the left side of the equation will be as the same as the operation we performed with the same number on the right side of the equation
So,
From the completed table,
Only 2 computations follows the “Properties of equality”
Hence, from the above,
We can conclude that
We can determine the equation by using the “Properties of equality”

Focus on math practices
Use Structure Complete the equation 7 + ☐ = 10 – ☐ by filling in the missing numbers. Describe at least two other operations with numbers that you can do to each side of the completed equation to keep it true.
Answer:
Let the missing number on the left side and right side of the given equation be: x
Now,
We know that,
According to the Properties of Equality,
In an equation,
The operation we performed with the given number on the left side of the equation will be as the same as the operation we performed with the same number on the right side of the equation
So,
7 + x = 10 – x
x + x = 10 – 7
2x = 3
x = \(\frac{3}{2}\)
x = 1.5
Hence, from the above,
We can conclude that
The completed equation is:

Essential Question
How can you use the properties of equality to write equivalent equations?
Answer:
Just as you can add or subtract the same exact quantity on both sides of an equation, you can also multiply both sides of an equation by the same quantity to write an equivalent equation.

Try It!

If 5y = 25, which property of equality was used to keep the equation 5y – 7 = 25 – 7 equal?
Answer:
The given equation is:
5y = 25
Now,
We know that,
The “Subtraction Property of equality” states that when you subtract the same amount from both sides of an equation, the two sides of the equation stay equal
So,
According to the above property,
5y = 25 changed into 5y – 7 = 25 – 7
Hence, from the above,
We can conclude that
The “Subtraction Property of Equality” was used to keep the equation 5y – 7 = 25 – 7 equal

Convince Me! What other properties of equality could you apply to keep the equation 5y = 25 equal? Give an example of each.
The given equation is:
5y = 25
Now,
We know that,
We can use any “Property of Equality” like “Addition Property of equality”, “Subtraction Property of equality” , “Multiplication Property of equality”, and “Division Property of equality” to keep the given equation true
Hence, from the above,
We can conclude that
The other properties of equality you could apply to keep the equation 5y = 25 equal are:
a. Addition Property of equality
b. Multiplication Property of equality
c. Division Property of equality

Try It!

A. A scale balances with four blue x-blocks on one side and 36 green blocks on the other side. Complete the equation to balance the scale with only one blue x-block.
4 • x = 36
(4 • x) ÷ ☐ = 36 ÷ ☐
x = 9
Answer:
It is given that
A scale balances with four blue x-blocks on one side and 36 green blocks on the other side.
Now,
According to the given information,
To balance the equation with only one blue x-block,
Divide the both sides of the given equation by 4 so that only one blue x rock will be present
So,

Hence, from the above,
We can conclude that
The number of blue x-rocks are: 9 blue x-rocks

B. If 25 + d = 36, does 25 + d – 25 = 36 – 20? Explain.
Answer:
The given equations are: 25 + d = 36 and 25 + d – 25 = 36 – 20
Now,
We know that,
The “Subtraction Property of equality” states that when you subtract the same amount from both sides of an equation, the two sides of the equation stay equal
So,
For 25+ d = 36,
According to the given second equation,
25 + d – 25 = 36 – 25
Hence, from the above,
We can conclude that
If 25 +d = 36, then 25 + d – 25 = 36 – 25

KEY CONCEPT
You can use the properties of equality to write equivalent equations.

Addition Property of Equality
7 + 3 = 10
(7 + 3) + a = 10 + a
Add the same amount to each side to keep the equation balanced.

Subtraction Property of Equality
7 + 3 = 10
(7 + 3) – a = 10 – a
Subtract the same amount from each side to keep the equation balanced.

Multiplication Property of Equality
7 + 3 = 10
(7 + 3) × a = 10 × a
Multiply each side of the equation by the same amount to keep the equation balanced.

Division Property of Equality
7+ 3 = 10
(7 + 3) ÷ a = 10 ÷ a
Divide each side of the equation by the same non-zero amount to keep the equation balanced.

Do You Understand?
Question 1.
Essential Question How can you use the properties of equality to write equivalent equations?
Answer:
Just as you can add or subtract the same exact quantity on both sides of an equation, you can also multiply both sides of an equation by the same quantity to write an equivalent equation.

Question 2.
A pan balance shows 7 + 5 = 12. If 4 units are removed from one side, what needs to be done to the other side to keep the pans balanced?
Answer:
It is given that
A pan balance shows 7 + 5 = 12. and 4 units are removed from one side
Now,
We know that,
According to the “Subtraction Property of Equality”,
Subtract the same amount from each side to keep the equation balanced.
So,
To keeps the pans balanced, we have to remove 4 from the other side also
Hence, from the above,
We can conclude that
We have to remove 4 from other side also to keep the pans balanced

Question 3.
If one side of the equation 23 + 43 = 66 is multiplied by 3, what needs to be done to the other side of the equation to keep the sides equal?
Answer:
It is given that
One side of the equation 23 + 43 = 66 is multiplied by 3
Now,
We know that,
According to the “Multiplication Property of Equality”,
Multiply the same amount from each side to keep the equation balanced.
So,
To keeps the pans balanced, we have to multiply 3 to the other side also
Hence, from the above,
We can conclude that
We have to multiply 3 to the other side also to keep the pans balanced

Question 4.
Reasoning If one side of the equation x + 5 = 8 has 9 added to it and the other side has (4 + 5) added to it, will the equation stay equal?
Answer:
It is given that
One side of the equation x + 5 = 8 has 9 added to it and the other side has (4 + 5) added to it
Now,
We know that,
According to the “Addition Property of Equality”,
Add the same amount from each side to keep the equation balanced.
Now,
We know that,
4 + 5 = 9
So,
The equation stays equal
Hence, from the above,
We can conclude that
The equation will stay equal even we added (4 + 5) to the other side of the equation

Do You Know How?
In 5 and 6, answer yes or no and explain why or why not.
Question 5.
If 23 + 37 = 60, does 23 + 37 + 9 = 60 + 9?
Answer:
The given equations are:
23 + 37 = 60 and 23 + 37 + 9 = 60 + 9
Now,
We know that,
According to the “Addition Property of Equality”,
Add the same amount from each side to keep the equation balanced.
So,
From the above equation,
We can observe that
9 is added to both sides
Hence, from the above,
We can conclude that
The given equation is true

Question 6.
If 16 + 1 = 17, does (16 + 1) – 1 = 17 – 2?
Answer:
The given equations are:
16 + 1 = 17 and (16 + 1) – 1 = 17 – 2
Now,
We know that,
According to the “Subtraction Property of Equality”,
Subtract the same amount from each side to keep the equation balanced.
So,
From the above equation,
We can observe that
1 is subtracted from one side and 2 is subtracted from the other side
Hence, from the above,
We can conclude that
The given equation is false

Question 7.
Apply the Multiplication Property of Equality to write an equation equivalent to 7n = 28.
Answer:
The given equation is:
7n = 28
Now,
We know that,
According to the “Multiplication Property of Equality”,
Multiply the same amount from each side to keep the equation balanced.
Now,
Let the number be multiplied on both sides of the equation be 2
So,
According the Multiplication Property of Equality,
7n × 2 = 28 × 2
Hence, from the above,
We can conclude that
According to the Multiplication Property of equation,
The equation that is equivalent to the given equation is:
7n × 2 = 28 × 2

Question 8.
Critique Reasoning Tomas says that if one side of the equation 6m = 9 is divided by 2 and the other side is divided by 3, the equation will stay equal because the result will be 3m = 3. Is Tomas correct? Explain.
Answer:
It is given that
Tomas says that one side of the equation 6m = 9 is divided by 2 and the other side is divided by 3
Now,
We know that,
According to the “Division Property of Equality”,
Divide the same amount from each side to keep the equation balanced.
So,
We have to divide the both sides of the equation by either only 2 or only 2 but not with 2 on one side and with 3 on other side
Hence,f rom the above,
We can conclude that
Tomas is not correct

Practice & Problem Solving

In 9-12, tell which property of equality was used.
Question 9.
5m + 4 = 19
5m + 4 – 3 = 19 – 3
Answer:
The given equations are:
5m + 4 = 19
5m + 4 – 3 = 19 – 3
Now,
We know that,
According to the “Subtraction Property of Equality”,
Subtract the same amount from each side to keep the equation balanced.
So,
From the given equation,
We can observe that
We subtracted 3 on both sides
Hence, from the above,
We can conclude that
“Subtraction Property of Equality” was used for the given equation

Question 10.
3t = 20
3t ÷ 2 = 20 ÷ 2
Answer:
The given equations are:
3t = 20
3t ÷ 2 = 20 ÷ 2
Now,
We know that,
According to the “Division Property of Equality”,
Divide the same amount at each side to keep the equation balanced.
So,
From the given equation,
We can observe that
We divided by 2 on both sides
Hence, from the above,
We can conclude that
“Division Property of Equality” was used for the given equation

Question 11.
\(\frac{n}{6}\) = 9
\(\left(\frac{n}{6}\right)\) × 5 = 9 × 5
Answer:
The given equations are:
\(\frac{n}{6}\) = 9
\(\frac{n}{6}\) × 5 = 9 × 5
Now,
We know that,
According to the “Multiplication Property of Equality”,
Multiply the same amount at each side to keep the equation balanced.
So,
From the given equation,
We can observe that
We multiplied 5 on both sides
Hence, from the above,
We can conclude that
“Multiplication Property of Equality” was used for the given equation

Question 12.
5b – 6 = 14
(5b – 6) + 2 = 14 + 2
Answer:
The given equations are:
5b – 6 = 14
(5b – 6) + 2 = 14 + 2
Now,
We know that,
According to the “Addition Property of Equality”,
Add the same amount at each side to keep the equation balanced.
So,
From the given equation,
We can observe that
We added 2 on both sides
Hence, from the above,
We can conclude that
“Addition Property of Equality” was used for the given equation

Question 13.
If r + 9 = 42, does r + 9 – 9 = 42 + 9? Why or why not?
Answer:
The given equations are:
r + 9 = 42
r + 9 – 9 = 42 + 9
Now,
We know that,
According to the “Subtraction Property of Equality”,
Subtract the same amount from each side to keep the equation balanced.
So,
From the given equation,
We can observe that
We subtracted 9 from one side and added 9 at the other side
Hence, from the above,
We can conclude that
r + 9 = 42 is not the equivalent of r + 9 – 9 = 42 + 9

Question 14.
If 6s = 24, does 6s ÷ 6 = 24 ÷ 6? Why or why not?
Answer:
The given equations are:
6s = 24
6s ÷ 6 = 24 ÷ 6
Now,
We know that,
According to the “Division Property of Equality”,
Divide the same amount at each side to keep the equation balanced.
So,
From the given equation,
We can observe that
We divided with 6 on both sides
Hence, from the above,
We can conclude that
6s = 24 is the equivalent of 6s ÷ 6 = 24 ÷ 6

Question 15.
This scale was balanced. Find the number to add that makes the scale become balanced again. Then complete the equation to make it true.
Envision Math Common Core 6th Grade Answer Key Topic 4 Represent And Solve Equations And Inequalities 17
Answer:
It is given that
This scale was balanced
Now,
The given figure is:

Now,
From the given figure,
We can observe that
To make the scale balanced again,
We have to add some quantity to 12 on the left side of the pan
Now,
Let the some quantity that is added on the left side of the pan be x
So,
To make the scale balanced,
12 + x = 2 + 7+ 3 + 16
So,
12 + x = 28
Subtract with 12 on both sides
12 + x – 12 = 28 – 12
x = 28 – 12
x = 16
Hence, from the above,
We can conclude that
The completed equation so that the equation is true is:

Question 16.
This scale balanced with 3 green blocks on one side and 1 blue x-block on the other side. Find the number to multiply by that makes the scale balance. Then complete the equation to make it true.
Envision Math Common Core 6th Grade Answer Key Topic 4 Represent And Solve Equations And Inequalities 18
Answer:
It is given that
This scale balanced with 3 green blocks on one side and 1 blue x-block on the other side
Now,
The given figure is:

Now,
From the given information,
The equation that represents the given situation is:
1x = 3
So,
x = 3
Now,
Multiply both sides with 5
So,
5 × x = 3 × 5
5x = 15
15 = 5x
Hence, from the above,
We can conclude that
The completed equation so that the equation will be true is:

Question 17.
You start with the equation 8x = 24. Your friend changes the equation as follows.
8x = 24 ÷ 4
How can you make your friend’s equation equivalent to the original equation?
Answer:
The given equations are:
8x = 24
8x = 24 ÷ 4
Now,
From the above equation,
We can observe that
Your friend wants to change the equation by dividing the equation by 4
Now,
We know that,
According to the “Division Property of Equality”,
Divide the same amount at each side to keep the equation balanced.
So,
For the given equation,
We have to divide by 4 on both sides instead of only one side like your friend
So,
8x ÷ 4 = 24 ÷ 4
Hence, from the above,
We can conclude that
The representation of your friend’s equation equivalent to the original given equation is:
8x ÷ 4 = 24 ÷ 4

Question 18.
A scale balanced with 1 blue x-block and 20 green blocks on the left side and 40 green blocks on the right side. A student bumped into the scale and knocked some blocks off so that only 1 blue x-block and 3 green blocks remained on the left side. How many blocks do you need to remove from the right side to make the scale balance?
Answer:
It is given that
A scale balanced with 1 blue x-block and 20 green blocks on the left side and 40 green blocks on the right side. A student bumped into the scale and knocked some blocks off so that only 1 blue x-block and 3 green blocks remained on the left side
So,
According to the given information,
Now,
Before a student bumped into the scale,
The representation of the equation is:
1x blue rocks + 20 green rocks = 40 green rocks
x blue rocks + 20 green rocks = 40 green rocks
Now,
After a student bumped into the scale,
x blue rocks + 20 green rocks – 0 blue rocks – 17 green rocks = 40 green rocks – x
Where,
x is the variable to make the equation balanced
Now,
We know that,
According to the “Subtraction Property of Equality”,
Subtract the same amount from each side to keep the equation balanced.
So,
x blue rocks + 20 green rocks – 0 blue rocks – 17 green rocks = 40 green rocks – 0 blue rocks – 17 green rocks
Hence, from the above,
We can conclude that
The number of blocks you needed to remove from the right side to make the scale balance is: 17 green rocks

Question 19.
Bobbie wrote y + 6 = 15. Then she wrote (y + 6) ÷ 3 = 15. Explain why the second equation is not equivalent to the first. What can Bobbie do to make the two equations equivalent?
Answer:
It is given that
Bobbie wrote y + 6 = 15. Then she wrote (y + 6) ÷ 3 = 15
Now,
The given equations are:
y + 6 = 15
(y + 6) ÷ 3 = 15
Now,
We know that,
According to the “Division Property of Equality”,
Divide the same amount at each side to keep the equation balanced.
So,
From the given equation,
We can observe that
We have to divide by 3 on both sides
But,
According to the second equation,
It has been divided only on the left side
So,
(y + 6) ÷ 3 ≠ 15
Hence, from the above,
We can conclude that
Due to the “Division Property of Equality”,
(y + 6) ÷ 3 ≠ 15

Question 20.
Construct Arguments John wrote that 5 + 5 = 10. Then he wrote that 5 + 5 + n = 10 + n. Are the equations John wrote equivalent? Explain.
Answer:
It is given that
John wrote that 5 + 5 = 10. Then he wrote that 5 + 5 + n = 10 + n
Now,
The given equations are:
5 + 5 = 10
5 + 5 + n = 10 + n
Now,
We know that,
According to the “Addition Property of Equality”,
Add the same amount at each side to keep the equation balanced.
So,
From the given equation,
We can observe that
John added n at both sides
So,
5 + 5 = 10 is equivalent to 5 + 5 + n = 10 + n
Hence, from the above,
We can conclude that
Due to the Addition Property of Equality,
5 + 5 = 10 is equivalent to 5 + 5 + n = 10 + n

Question 21.
Reasoning Scientists often use a pan balance to measure mass when doing experiments. The equation 4 + 3 – 1 = 7 – 1 represents a scientist taking away one unit of mass from each side of a pan balance. Construct an argument to explain how the scientist knows that the pans are still in balance.
Answer:
It is given that
Scientists often use a pan balance to measure mass when doing experiments. The equation 4 + 3 – 1 = 7 – 1 represents a scientist taking away one unit of mass from each side of a pan balance.
Now,
The given expression is:
4 + 3 – 1 = 7 – 1
Now,
We know that,
According to the “Subtraction Property of Equality”,
Subtract the same amount from each side to keep the equation balanced.
So,
From the given equation,
We can observe that
1 is subtracted from both sides of the equation
So,
The pans are still in balance according to “Subtraction Property of Equality”
Hence, from the above,
We can conclude that
The pans are still in balance according to “Subtraction Property of Equality”

Question 22.
Bryce wrote the equation n – 3 = 4. Lexi used a property of equality to write an equivalent equation. Write an equation Lexi could have written. Explain how you know the equations are equivalent.
Answer:
It is given that
Bryce wrote the equation n – 3 = 4. Lexi used a property of equality to write an equivalent equation
Now,
The given equation is:
n – 3 = 4
Now,
We know that,
According to the “Addition Property of Equality”,
Add the same amount at each side to keep the equation balanced.
So,
n – 3 + 3 = 4 + 3
n = 7
So,
The equivalent equation of the given equation is: n = 7
Hence, from the above,
We can conclude that
The equation Lexi could have written is: n = 7
The equation swritten by Bryce and Lexi are equivalent by using the “Additie Property of Equality”

Question 23.
Higher Order Thinking Emil has $1 and a quarter. Jade has 5 quarters. If Emil gives Jade $1 and Jade gives Emil 4 quarters, will they each still have the same amount of money? Explain.
Envision Math Common Core 6th Grade Answer Key Topic 4 Represent And Solve Equations And Inequalities 19
Answer:
It is given that
Emil has $1 and a quarter. Jade has 5 quarters and Emil gives Jade $1 and Jade gives Emil 4 quarters
Now,
We know that,
1 Quarter = $0.25
Now,
The amount of money Emil contains = $1 + $0.25
= $1.25
The amount of money Jade contains = 5 × $0.25
= $1.25
Now,
If Emil gives Jade $1, then
The amount of money Jade contains = $1.25 + $1
= $2.25
The remaining amount of money Emil contains = $1.25 – $1
= $0.25
If Jade gives 4 Quarters to Emil, then
The amount of money Emil contains = $0.25 + 4 × $0.25
= $0.25 + $1
= $1.25
Hence, from the above,
We can conclude that
Emil and Jade each still have the same amount of money

Question 24.
Vocabulary If 7w = 49, which property of equality was used to find the equivalent equation 7w ÷ 7 = 49 ÷ 7?
Answer:
The given equations are:
7w = 49
7w ÷ 7 = 49 ÷ 7
Now,
We know that,
According to the “Division Property of Equality”,
Divide the same amount at each side to keep the equation balanced.
So,
From the given equation,
We can observe that
7 is divided on both sides
Hence, from the above,
We can conclude that
“Division Property of Equality” was used to find the equivalent equation  of the given equation 7w ÷ 7 = 49 ÷ 7

Question 25.
You start with the equation 12b = 24. What step should you take to find the quantity that equals 4b?
Answer:
The given equation is:
12b = 24
Now,
It is given that
After using the “Equality Property”, we have to make 12b as 4b
Now,
We know that,
According to the “Division Property of Equality”,
Divide the same amount at each side to keep the equation balanced.
So,
Divide the given equation by 3 on both sides
So,
\(\frac{12b}{3}\) = \(\frac{24}{3}\)
4b = 8
Hence, from the above,
We can conclude that
To find the quantity “4b”,
We have to use “Division Property of Equality” for the given equation

Assessment Practice

Question 26.
Which equation is equivalent to n + 4 = 11?.
A. (n + 4) × 2 = 11
B. (n + 4) × 2 = 11 ÷ 2
C. (n + 4) × 2 = 11 × 4
D (n + 4) × 2 = 11 × 2
Answer:
The given equation is:
n + 4 = 11
Now.
We know that,
According to the “Properties of Equality”,
The operation we are doing at the left side will be the same as the operation we are doing at the right side
Hence, from the above,
We can conclude that
The equation that is equivalent to n + 4 = 11 is:

Question 27.
Which of the equations is NOT equivalent to 8p = 12? Select all that apply.
☐ 8p ÷ 8 = 12 ÷ 8
☐ 8p ÷ 8 = 12 ÷ 12
☐ 8p + 4 = 12 + 4
☐ 8p – 2 = 12 – 2
☐ 8p × 8 = 12 × 12
Answer:
The given equation is:
8p = 12
Now.
We know that,
According to the “Properties of Equality”,
The operation we are doing at the left side will be the same as the operation we are doing at the right side
Hence, from the above,
We can conclude that
The equations that are not equivalent to 8p = 12 are:

Lesson 4.3 Write and Solve Addition and Subtraction Equations

Solve & Discuss It!
A group of students were on a school bus. How many students were on the bus before the last stop?
I can… write and solve an addition or subtraction equation.
Envision Math Common Core 6th Grade Answer Key Topic 4 Represent And Solve Equations And Inequalities 20

Use Appropriate Tools
You use a pan balance to help solve for the unknown.
Answer:
It is given that
A group of students were on a school bus
Now,
The given figure is:
Envision Math Common Core 6th Grade Answer Key Topic 4 Represent And Solve Equations And Inequalities 20
Now,
From the given figure,
We can observe that
The total number of students arrived at school = (The total number of students present on the bus before the last stop) + (The total number of students present on the bus at the last stop)
Now,
Let the total number of students present on the bus before the last stop be x
So,
25 = x + 16
x = 25 – 16
x = 9 students
Hence, from the above,
We can conclude that
The total number of students present on the bus before the last stop is: 9 students

Focus on math practices
Reasoning How does using cubes on the pan balance demonstrate the Addition and Subtraction Properties of Equality?
Answer:
In the pan balance,
The cubes represent the variables
Remember that the pans must be in a balanced position and it is represented by “=”
Ex:
In a pan balance,
At one side, 3x cubes are present
At other side, 2x cubes are present
So,
The balanced pan will be:
3x = 2x
Ex:
In a pan balance,
At one side, 10x cubes are added
At other side, 25x cubes are added and later, 6x quantities are removed
So,
The balance pan will be:
10x = 25x – 6x

Essential Question
How can you write and solve an addition or subtraction equation?
Answer:
a.
To write a proper addition equation, you have an equals sign. One side shows you the total. The other side shows you what things are being added together. You can have a variable on either side of the equation
b.
When we write a subtraction equation, we use two symbols: – and =. The minus sign (-) means one thing is being subtracted from another

Try It!

Cabrini had some markers. After she bought 12 more markers, she had 16. How many markers did Cabrini have at the start?
Let n represent the number of markers Cabrini had at the start.
Solve the addition equation.
n + 12 = 16
n + 12 ___ = 16 ____
n = _____
Cabrini had ______ markers at the start.
Answer:
It is given that
Cabrini had some markers. After she bought 12 more markers, she had 16
Now,
Let the number of markers Cabrini had before she bought more markers be: x
So,
According to the given information,
The equation will be:
x + 12 = 16
Now,
We know that,
According to the “Subtraction Property of Equality”,
Subtract the same amount from each side to keep the equation balanced.
So,
Subtract 12 from both sides in the given equation
So,

Hence, from the above,
We can conclude that
Cabrini had 4 markers at the start

Convince Me! Which property of equality is used to solve the equation n + 12 = 16? Could one of the other properties of equality have also been used? Explain.
The given equation is:
n + 12 = 16
Now,
To solve the above equation,
The property of equality we used is “Subtraction Property of Equality”
Now,
We know that,
According to the “Subtraction Property of Equality”,
Subtract the same amount from each side to keep the equation balanced.
Now,
We know that,
If we observe the above equation,
We can say that,
We can use other “Properties of Equality” like “Addition Property of Equality”, “Multiplication Property of Equality”, and “Division Property of Equality”
Hence, from the above,
We can conclude that
We used “Subtraction Property of Equality” to solve the given equation n + 12 = 16
The other properties of equality can also be used to solve the given equation n + 12 = 16

Try It!

Vivian read 14 fewer pages than she was assigned to read. She read 60 pages. Write and solve an equation to find how many pages, p, Vivian was assigned to read.
Answer:
It is given that
Vivian read 14 fewer pages than she was assigned to read. She read 60 pages.
Now,
Let the number of pages Vivian was assigned to read be: x pages
Now,
According to the given information,
The equation that represents the given situation is:
x – 14 = 60
Now,
We know that,
According to the “Addition Property of Equality”,
Add the same amount at each side to keep the equation balanced.
So,
Add 14 on both sides to the given equation
So,
x – 14 + 14 = 60 + 14
x = 74 pages
Hence, from the above,
We can conclude that
The number of pages Vivian was assigned to read is: 74 pages

KEY CONCEPT
You can use inverse relationships and the properties of equality to solve equations.
Envision Math Common Core 6th Grade Answer Key Topic 4 Represent And Solve Equations And Inequalities 21

Do You Understand?
Question 1.
Essential Question How can you write and solve an addition or subtraction equation?
Answer:
a.
To write a proper addition equation, you have an equals sign. One side shows you the total. The other side shows you what things are being added together. You can have a variable on either side of the equation
b.
When we write a subtraction equation, we use two symbols: – and =. The minus sign (-) means one thing is being subtracted from another

Question 2.
Explain how you can use the inverse relationship of addition and subtraction to solve the equation n + 7 = 25.
Answer:
The given equation is:
n + 7 = 25
Now,
We know that,
Addition and subtraction are inverse operations because one operation can “undo” the other operation
Now,
We know that,
According to the “Subtraction Property of Equality”,
Subtract the same amount from each side to keep the equation balanced.
Now,
subtract 7 from both sides
So,
n + 7 – 7 = 25 – 7
n = 18
Hence, from the above,
We can conclude that
The value of n for the given equation is: 18

Question 3.
Model with Math Clare had t seashells. After she bought 8 more seashells, she had 24 seashells. Write and solve an equation to find the number of seashells Clare started with.
Answer:
It is given that
Clare had t seashells. After she bought 8 more seashells, she had 24 seashells.
So,
The number of seashells = (The number of seashells Clare had at the start) + (The number of seashells she bought after the start)
24 = t + 8
Now,
We know that,
According to the “Subtraction Property of Equality”,
Subtract the same amount from each side to keep the equation balanced.
Now,
subtract 8 from both sides
So,
24 – 8 = t + 8 – 8
t = 16 seashells
Hence, from the above,
We can conclude that
The number of seashells Clare had at the start is: 16 seashells

Question 4.
Model with Math The outside temperature dropped 20°F from the time Arianna ate breakfast until the time she ate dinner. When she ate dinner the temperature was 35°F. Write and solve an equation to find the outside temperature t when Arianna ate breakfast.
Answer:
It is given that
The outside temperature dropped 20°F from the time Arianna ate breakfast until the time she ate dinner. When she ate dinner the temperature was 35°F
So,
The outside temperature from the time Arianna ate breakfast until the time she ate dinner = (The outside temperature when Arianna ate breakfast) + (The outside temperature when Arianna ate dinner)
20° F = x° F + 35° F
Now,
We know that,
According to the “Subtraction Property of Equality”,
Subtract the same amount from each side to keep the equation balanced.
Now,
subtract 35° F from both sides
So,
20° F – 35° F = x° F + 35° F – 35° F
x° F = – 15° F
Hence, from above,
We can conclude that
The outside temperature when Arianna ate breakfast is: -15° F

Do You Know How?
In 5-10, solve each equation.
Question 5.
24 + m = 49
Answer:
The given equation is:
24 + m = 49
Now,
Subtract with 24 on both sides
So,
24 + m – 24 = 49 – 24
m = 25
Hence, from the above,
We can conclude that
The value of m is: 25

Question 6.
12 = y – 11
Answer:
The given equation is:
12 = y – 11
Now,
Add 11 on both sides
So,
12 + 11 = y – 11 + 11
y = 23
Hence, from the above,
We can conclude that
The value of y is: 23

Question 7.
22 = 13 + a
Answer:
The givene quation is:
22 = 13 + a
Now,
Subtract 13 on both sides
So,
22 – 13 = 13 + a – 13
a = 9
Hence, from the above,
We can conclude that
The value of a is: 9

Question 8.
t – 40 = 3
Answer:
The given equation is:
t – 40 = 3
Now,
Add 40 on both sides
So,
t – 40 + 40 = 3 + 40
t = 43
Hence, from the above,
We can conclude that
The value of t is: 43

Question 9.
d + 11 = 15
Answer:
The given equation is:
d + 11 = 15
Now,
Subtract 11 on both sides
So,
d + 11 – 11 = 15 – 11
d = 4
Hence, from the above,
We can conclude that
The value of d is: 4

Question 10.
32 = s – 19
Answer:
The given equation is:
32 = s – 19
Now,
Add 19 on both sides
So,
32 + 19 = s – 19 + 19
s = 51
Hence, from the above,
We can conclude that
The value of s is: 51

Practice & Problem Solving

Leveled Practice
In 11-16, solve each equation.

Question 11.
y – 12 = 89
y – 12 + _____ = 89 + 12
y = ______
Answer:
The given equation is:
y – 12 = 89
Now,
Add 12 on both sides
So,

Hence, from the above,
We can conclude that
The value of y is: 101

Question 12.
80 + r = 160
80 + r – ______ = 160 – ______
r = ______
Answer:
The given equation is:
80 + r = 160
Now,
Subtract 80 on both sides
So,

Hence, from the above,
We can conclude that
The value of r is: 80

Question 13.
60 = x – 16
60 + ______ = x – 16 + ______
______ = x
Answer:
The given equation is:
60 = x – 16
Now,
Add 16 on both sides
So,

Hence, from the above,
We can conclude that
The value of x is: 76

Question 14.
20 = y + 12
Answer:
The givene quation is:
20 = y + 12
Now,
Subtract 12 on both sides
So,
20 – 12 = y + 12 – 12
y = 8
Hence, from the above,
We can conclude that
The value of y is: 8

Question 15.
x + 2 = 19
Answer:
The given equation is:
x + 2 = 19
Now,
Subtract 2 on both sides
So,
x + 2 – 2 = 19 – 2
x = 17
Hence, from the above,
We can conclude that
The value of x is: 17

Question 16.
z – 313 = 176
Answer:
The given equation is:
z – 313 = 176
Now,
Add 313 on both sides
So,
z – 313 + 313 = 176 + 313
z = 489
Hence, from the above,
We can conclude that
The value of z is: 489

Question 17.
You have some baseball trading cards. You give 21 baseball cards to a friend and have 9 left for yourself. How many baseball cards were in your original deck? Write and solve an equation to find t, the number of baseball cards in your original deck.
Envision Math Common Core 6th Grade Answers Topic 4 Represent And Solve Equations And Inequalities 22
Answer:
It is given that
You have some baseball trading cards. You give 21 baseball cards to a friend and have 9 left for yourself
Now,
The representation of the given information in the form of a bar diagram is:
Envision Math Common Core 6th Grade Answers Topic 4 Represent And Solve Equations And Inequalities 22
Now,
From the given bar diagram,
We can observe that
(The total number of baseball trading cards) – (The number of baseball cards you have given to your friend) = (The number of baseball cards you have left)
So,
t – 21 = 9
Now,
Add 21 on both sides
So,
t – 21 + 21 = 9 + 21
t = 30
Hence, from the above,
We can conclude that
The total number of baseball trading cards in your original deck is: 30 cards

Question 18.
Model with Math Joy added 26 new contacts to her phone list. She now has a total of 100 contacts. Let c represent how many contacts Joy had on her phone list before she updated it. Write an equation and solve for c.
Answer:
It is given that
Joy added 26 new contacts to her phone list. She now has a total of 100 contacts. Let c represent how many contacts Joy had on her phone list before she updated it
Now,
According to the given information,
The representation of the equation for the given information is:
c + 26 = 100
Now,
Subtract 26 on both sides
So,
c + 26 – 26 = 100 – 26
c = 84 contacts
Hence, from the above,
We can conclude that
The number of contacts Joy had on her phone list before she updated is: 84 contacts

Question 19.
Reasoning Jeremy bought a sandwich and a drink that cost him $7. His drink cost $1.75. Solve the equation 7 = s + 1.75 to find s, the cost of Jeremy’s sandwich.
Answer:
It is given that
Jeremy bought a sandwich and a drink that cost him $7. His drink cost $1.75
Now,
Let s be the cost of Jeremy’s sandwich
So,
According to the given information,
The total cost pf a sandwich and a drink that Jeremy bought = (The cost of Jeremy’s sandwich) + (The cost of Jeremy’s drink)
$7 = s + $1.75
Now,
subtract $1.75 on both sides
So,
$7 – $1.75 = s + $1.25 – $1.25
s = $5.25
Hence, from the baove,
We can conclude that
The cost of Jeremy’s sandwich is: $5.25

Question 20.
A triathlon is about 51 Kilometers. One participant completed two of the three legs of the race and traveled 42 Kilometers. Solve the equation 42 + d = 51 for the distance, d, of the third leg of the race.
Envision Math Common Core 6th Grade Answers Topic 4 Represent And Solve Equations And Inequalities 23
Answer:
It is given that
A triathlon is about 51 Kilometers. One participant completed two of the three legs of the race and traveled 42 Kilometers
Now,
The representation of the given information in the form of a bar diagram is:
Envision Math Common Core 6th Grade Answers Topic 4 Represent And Solve Equations And Inequalities 23

Now,
The representation of the equation that represents the given information ad as given in the problem is:
42 + d = 51
Where,
d is the Distance of the third leg
Now,
Subtract 41 from both sides
So,
42 + d – 42 = 51 – 42
d = 9 Kilometers
Hence, from the above,
We can conclude that
The distance of the third leg of the race is: 9 Kilometers

Question 21.
What operation should be used to solve the equation 153 = 9 + 45? Solve the equation.
Answer:
The given equation is:
153 = 9 + 45
Now,
When we observe the equation,
There is “Addition” operation taking place in the given equation
Now,
We have to subtract the required number
Hence, from the above,
We can conclude that
The operation we should be used to solve the given equation 153 = 9 + 45 is: Subtraction

Question 22.
Higher Order Thinking In the equation 6 + 3y = 4y + 2 the variable y represents the same value. Is y = 2, 3, 4, or 5 the solution of this equation? Explain.
Answer:
It is given that
In the equation 6 + 3y = 4y + 2 the variable y represents the same value
Now,
The given equation is:
6 + 3y = 4y + 2
Now,
For y = 2:
6 + 3y = 4y + 2
6 + 3 (2) = 4 (2) + 2
6 + 6 = 8 + 2
12 ≠ 10
For y = 3:
6 + 3y = 4y + 2
6 + 3 (3) = 4 (3) + 2
6 + 9 = 12 + 2
15 ≠ 14
For y = 4;
6 + 3y = 4y + 2
6 + 3 (4) + 4 (4) + 2
6 + 12 = 16 + 2
18 = 18
For y = 5:
6 + 3y = 4y + 2
6 + 3 (5) = 4 (5) + 2
6 + 15 = 20 + 2
21 ≠ 22
Hence, from the above,
We can conclude that
y = 4 is the solution for the given equation

Question 23.
A traffic helicopter descends to hover 477 meters above the ground. Let h be the original height of the helicopter. What is a subtraction equation that represents the problem? What was the original height of the helicopter?
Envision Math Common Core 6th Grade Answers Topic 4 Represent And Solve Equations And Inequalities 24
Answer:
It is given that
A traffic helicopter descends to hover 477 meters above the ground. Let h be the original height of the helicopter
Now,
The given figure is:
Envision Math Common Core 6th Grade Answers Topic 4 Represent And Solve Equations And Inequalities 24
Now,
From the given figure,
We can observe that
(The original height of helicopter) + 127 m = 477 m
h – 127 = 477
Now,
Add 127 on both sides
So,
h + 127 – 127 = 477 + 127
h = 604 meters
Hence, from the above,
We can conclude that
The subtraction equation that represents the given problem is:
h  – 127 = 477
The original height of the helicopter is: 604 meters

Question 24.
The drama club sold all the tickets for its annual production in three days. The club sold 143 tickets the first day and 295 tickets the second day. If the drama club sold 826 tickets, how many tickets were sold on the third day of sales? Solve the equation 438 + t = 826 for the number of tickets, t, sold on the third day of ticket sales.
Answer:
It is given that
The drama club sold all the tickets for its annual production in three days. The club sold 143 tickets the first day and 295 tickets the second day. If the drama club sold 826 tickets
So,
The total number of tickets sold by the drama club in two days = (The number of tickets sold on the first day) + (The number of tickets sold on the second day)
= 143 + 295
= 438 tickets
Now,
Let the number of tickets sold on third day be: t
So,
According to the given information,
The equation that represents the given information is:
438 + t = 826
Now,
Subtract 438 on both sides
So,
438 + t – 438 = 826 – 438
t = 388 tickets
Hence, from the above,
We can conclude that
The number of tickets sold on the third day of ticket sales is: 388 tickets

Question 25.
In a bag of mixed nuts, there are 35 almonds, 34 pecans, 32 walnuts, and p pistachios. The bag has a total of 134 nuts. Find the total number of almonds, pecans, and walnuts. Then write and solve an equation to find the number of pistachios in the bag.
Answer:
It is given that
In a bag of mixed nuts, there are 35 almonds, 34 pecans, 32 walnuts, and p pistachios. The bag has a total of 134 nuts
So,
The total number of almonds, pecans, and walnuts = 35 + 34 + 32
= 101
Now,
The total number of almonds, peacons, walnuts, and pistachios = (The total number of almond, pecans, and walnuts) + (The number of pistachios)
So,
134 = 101 + p
Now,
Subtract 101 on both sides
So,
134 – 101 = 101 + p – 101
p = 33
Hence, from the above,
We can conclude that
The number of pistachios in the bag is: 33 pistachios

Assessment Practice

Question 26.
Which equation has g = 6 as the solution?
A. g+ 2 = 10
B. 9- 1 = 10
C. 58 +9 = 60
D. 44 – g = 38
Answer:
The given equations are:
A. g+ 2 = 10
B. 9- 1 = 10
C. 58 +g = 60
D. 44 – g = 38
So,
A.
g + 2 = 10
g = 10 – 2
g = 8
B.
g – 1 = 10
g = 10 + 1
g = 11
C.
58 + g = 60
g = 60 – 58
g = 2
D.
44 – g = 38
g = 44- 38
g = 6
Hence, from the above,
We can conclude that
The equation that has g = 6 as the solution is:

Question 27.
Select all the equations that have the same solution as 36 = x + 32.
☐ 42 = 38 + x
☐ x + 15 = 19
☐ 18 = x – 2
☐ 36 = x – 32
☐ 52 – x = 46
Answer:
The given equation is:
36 = x + 32
So,
x = 36 – 32
x = 4
Now,
The given equations are:
A.
42 = 38 + x
x = 42 – 38
x = 4
B.
x + 15 = 19
x = 19 – 15
x = 4
C.
18 = x – 2
x = 18 + 2
x = 20
D.
36 = x – 32
x = 36 + 32
x = 68
E.
52 – x = 46
x = 52 – 46
x = 6
Hence, from the above,
We can conclude that
All the equations that have the same solution as 36 = x + 32 are:

Lesson 4.4 Write and Solve Multiplication and Division Equations

Solve & Discuss It!
A school group is planning a trip to New York City. There are 29 people going on the trip. They agreed to share the total cost of the trip equally. Let s equal each person’s share of the cost. What is each person’s share of the cost?

I can… write and solve a multiplication or division equation.
Envision Math Common Core 6th Grade Answers Topic 4 Represent And Solve Equations And Inequalities 25
Answer:
It is given that
A school group is planning a trip to New York City. There are 29 people going on the trip. They agreed to share the total cost of the trip equally. Let s equal each person’s share of the cost
Now,
The given table is:

Now,
From the given table,
We can observe that
The total cost is: $19,111
So,
According to the given information,
(The total number of people going to a trip) × (The share of the cost of each person) = $19,111
29 × s = $19,111
s = \(\frac{$19,111}{29}\)
s = $659
Hence, from the above,
We can conclude that
Each person’s share of the cost is: $659

Generalize
How can you use what you know about dividing lesser numbers to write equations and solve problems involving greater numbers?
Answer:
Let a smaller faction be: \(\frac{a}{b}\)
Where,
a = 1, 2, ……… n
b = 1, 2, ……… n
Now,
When we divide lesser numbers say the lesser numbers only have 2 digits, then the quotient we will also get will be a small number
Now,
When we divide larger numbers say the larger numbers have 4 digits, then the quotient we will also get will be a large number with 3 (or) 4 digits

Focus on math practices
Construct Arguments Can you use the same strategy that you used above to find each person’s share of the hotel bill? Explain.
Answer:
Yes,
We can use the same strategy that you used above to find each person’s share of the hotel bill

Essential Question
How can you write and solve a multiplication or division equation?
Answer:
The steps to solve a multiplication or division problem is:
a. Always perform the same operation to both sides of the equation.
b. When you multiply or divide, you have to multiply and divide by the entire side of the equation.
c. Try to perform addition and subtraction first to get some multiple of x by itself on one side.

Try It!

Theresia picked the same number of tomatoes each day. In 4 days she picked 52 tomatoes. How many tomatoes did Theresia pick each day? Let n represent the number of tomatoes Theresia picked each day.
4n = 52
4n ÷ 4 = 52 ______
n = _______
Theresia picked _______ tomatoes each day.
Answer:
It is given that
Theresia picked the same number of tomatoes each day. In 4 days she picked 52 tomatoes
Now,
Let n represent the number of tomatoes Theresia picked each day.
So,
4 × (The number of tomatoes Theresia picked each day) = (The number of tomatoes Theresia picked in 4 days)
4 × n = 52
Now,

Hence, from the above,
We can conclude that
Theresia picked 13 tomatoes each day

Convince Me! Which property or equality can you use to solve Theresia’s equation? Explain.
Answer:
From the above problem,
The equation is:
4n = 52
Now,
To find the value of n (or) the number of tomatoes Theresia picked each day,
We have to use “Division Property of Equality”
Now,
We know that,
According to the “Division Property of Equality”,
Divide the same amount to the equation to keep the equation balanced
Hence, from the above,
We can conclude that
We used the “Division Property of equality” to solve Theresia’s equation

Try It!

Meghann is reading a 630-page book. She reads 18 pages each day. Write and solve a division equation to find the number of days, d, it will take Meghann to finish her book.
Answer:
It is given that
Meghann is reading a 630-page book. She reads 18 pages each day
Now,
Let d be the number of days it will take Meghann to finish her book
Now,
(The number of pages Meghann reads each day) × (The number of days it will take Meghann to finish her book) = (The total number of pages in a book)
18 × d = 630
d = \(\frac{630}{18}\)
d = 35 days
Hence, from the above,
We can conclude that
It will take 35 days for Meghann to finish her 630-page book

KEY CONCEPT
You can multiply or divide both sides of an equation by the same number and it will remain balanced.
Envision Math Common Core 6th Grade Answers Topic 4 Represent And Solve Equations And Inequalities 26

Do You Understand?
Question 1.
Essential Question How can you write and solve a multiplication or division equation?
Answer:
The steps to solve a multiplication or division problem is:
a. Always perform the same operation to both sides of the equation.
b. When you multiply or divide, you have to multiply and divide by the entire side of the equation.
c. Try to perform addition and subtraction first to get some multiple of x by itself on one side.

Question 2.
Which property of equality would you use to solve the equation 8n = 16?
Answer:
The given equation is:
8n = 16
Now,
Divide by 8 to find the value of n
So,
We have to use “Division Property of Equality”
Now,
We know that,
According to the “Division Property of Equality”,
Divide the same amount at each side to keep the equation balanced.
So,
\(\frac{8n}{8}\) = \(\frac{16}{8}\)
n = 2
Hence, from the above,
We can conclude that
You would use “Division Property of Equality” to solve the given equation 8n = 16

Question 3.
Which property of equality would you use to solve the equation a ÷ 9 = 2?
Answer:
The given equation is:
a ÷ 9 = 2
Now,
Multiply with 9 to find the value of ‘a’
So,
We have to use “Multiplication Property of Equality”
Now,
We know that,
According to the “Multiplication Property of Equality”,
Multiply the same amount at each side to keep the equation balanced.
So,
(a ÷ 9) × 9 = 2 × 9
a = 18
Hence, from the above,
We can conclude that
You would use “Multiplication Property of Equality” to solve the given equation a ÷ 9 = 2

Question 4.
There are 30 students in the drama club. They are carpooling in 5 vans to perform a play. They want each van to carry an equal number of students. Let s be the number of students in each van. Write and solve a multiplication equation to find the number of students in each van.
Answer:
It is given that
There are 30 students in the drama club. They are carpooling in 5 vans to perform a play. They want each van to carry an equal number of students.
Now,
Let s be the number of students in each van
So,
(The number of vans carpooling to perform a play) × (The number of students in each van) = (The total number of students in the drama club)
5 × s = 30
5s = 80
Now,
Divide by 5 on both sides
So,
\(\frac{5s}{5}\) = \(\frac{80}{5}\)
s = 16 students
Hence, from the above,
We can conclude that
The number of students in each van is: 16 students

Do You Know How?
In 5-8, explain how to solve each equation.
Question 5.
18m = 36
Answer:
The given equation is:
18m = 36
Now,
Divide the equation by 18 on both sides
So,
\(\frac{18m}{18}\) = \(\frac{36}{18}\)
m = 2
Hence, from the above,
We can conclude that
The value of m in the given equation is: 2

Question 6.
t ÷ 3 = 10
Answer:
The given equation is:
t ÷ 3 = 10
Now,
Multiply the equation with 3 on both sides
So,
(t ÷ 3) × 3 = 10 × 3
t = 30
Hence, from the above,
We can conclude that
The value of t in the given equation is: 30

Question 7.
12 = 2y
Answer:
The given equation is:
12 = 2y
Now,
Divide the equation by 2 on both sides
So,
\(\frac{12}{2}\) = \(\frac{2y}{2}\)
6 = y
y = 6
Hence, from the above,
We can conclude that
The value of y in the given equation is: 6

Question 8.
22 = a ÷ 5
Answer:
The given equation is:
22 = a ÷ 5
Now,
Multiply the equation with 5 on both sides
So,
22 × 5 = (a ÷ 5) × 5
a = 110
Hence, from the above,
We can conclude that
The value of a in the given equation is: 110

In 9-12, solve each equation.
Question 9.
23d = 2,392
Answer:
The given equation is:
23d = 2,392
Now,
Divide the given equation by 23 on both sides
So,
\(\frac{23d}{23}\) = \(\frac{2,392}{23}\)
d = 104
Hence, from the above,
We can conclude that
The value of d in the given equation is: 104

Question 10.
74f = 6,179
Answer:
The given equation is:
74f = 6,179
Now,
Divide the given equation by 74 on both sides
So,
\(\frac{74f}{74}\) = \(\frac{6,179}{74}\)
f = 83.5
Hence, from the above,
We can conclude that
The value of f in the given equation is: 83.5

Question 11.
y ÷ 11 = 987
Answer:
The given equation is:
y ÷ 11 = 987
Now,
Multiply the given equation with 11 on both sides
So,
(y ÷ 11) × 11 = 987 × 11
y = 10,857
Hence, from the above,
We can conclude that
The value of y in the given equation is: 10,857

Question 12.
r ÷ 187 = 9
Answer:
The given equation is:
r ÷ 187 = 9
Now,
Multiply the given equation with 187 on both sides
So,
(r ÷ 187) × 187 = 9 × 187
y = 1,683
Hence, from the above,
We can conclude that
The value of r in the given equation is: 1,683

Practice & Problem Solving

In 13-16, explain how to get the variable alone in each equation.
Question 13.
8y = 56
Answer:
The given equation is:
8y = 56
Now,
Divide the given equation with 8 on both sides
so,
\(\frac{8y}{8}\) = \(\frac{56}{8}\)
y = 7
Hence, from the above,
We can conclude that
The value of y for the given equation is: 7

Question 14.
t ÷ 15 = 3
Answer:
The given equation is:
t ÷ 15 = 3
Now,
Multiply the given equation with 15 on both sides
So,
(t ÷ 15) × 15 = 3 × 15
t = 45
Hence, from the above,
We can conclude that
The value of t for the given equation is: 45

Question 15.
u ÷ 8 = 12
Answer:
The given equation is:
u ÷ 8 = 12
Now,
Multiply the given equation with 8 on both sides
So,
(u ÷ 8) × 8 = 12 × 8
u = 96
Hence, from the above,
We can conclude that
The value of u for the given equation is: 96

Question 16.
31y = 310
Answer:
The given equation is:
31y = 310
Now,
Divide the given equation with 31 on both sides
so,
\(\frac{31y}{31}\) = \(\frac{310}{31}\)
y = 10
Hence, from the above,
We can conclude that
The value of y for the given equation is: 10

In 17-20, solve each equation.
Question 17.
d ÷ 2 = 108
Answer:
The given equation is:
d ÷ 2 = 108
Now,
Multiply the given equation with 2 on both sides
So,
(d ÷ 2) × 2 = 108 × 2
d = 216
Hence, from the above,
We can conclude that
The value of d for the given equation is: 216

Question 18.
7,200 = 800s
Answer:
The given equation is:
7,200 = 800s
Now,
Divide the given equation with 800 on both sides
so,
\(\frac{7,200}{800}\) = \(\frac{800s}{800}\)
9 = s
s = 9
Hence, from the above,
We can conclude that
The value of s for the given equation is: 9

Question 19.
x ÷ 3 = 294
Answer:
The given equation is:
x ÷ 3 = 294
Now,
Multiply the given equation with 3 on both sides
So,
(x ÷ 3) × 3 = 294 × 3
d = 882
Hence, from the above,
We can conclude that
The value of x for the given equation is: 882

Question 20.
99 = 3x
Answer:
The given equation is:
99 = 3x
Now,
Divide the given equation with 3 on both sides
so,
\(\frac{99}{3}\) = \(\frac{3x}{3}\)
33 = x
x = 33
Hence, from the above,
We can conclude that
The value of x for the given equation is: 33

In 21 and 22, write a division equation and a multiplication equation to represent each problem.
Question 21.
Lolo typed 1,125 words in 15 minutes. Let w represent the number of words typed each minute. If Lolo typed the same number of words each minute, how many words did she type in 1 minute?
Answer:
It is given that
Lolo typed 1,125 words in 15 minutes. Let w represent the number of words typed each minute. and Lolo typed the same number of words each minute
So,
(The number of minutes needed to type 1,125 words) × (The number of words Lolo typed each minute) = 1,125
15 ×w = 1,125
15w = 1,125
Now,
Divide by 15 on both sides
So,
\(\frac{15w}{15}\) = \(\frac{1,125}{15}\)
w = \(\frac{1,125}{15}\)
Hence, from the above,
We can conclude that
The representation of the multiplication equation and the division equation to represent the given situation is:
a. Multiplication equation: 15w = 1,125
b. Division equation: w = \(\frac{1,125}{15}\)

Question 22.
In 12 weeks Felipe earns $4,500 doing yard work. He earns the same amount each week. Let m stand for the amount earned each week. How much does Felipe make in 1 week?
Answer:
It is given that
In 12 weeks Felipe earns $4,500 doing yard work. He earns the same amount each week. Let m stand for the amount earned each week
So,
(The number of weeks needed to earn $4,500) × (The amount earned by Felipe each week) = $4,500
12 × m = $4,500
12m = $4,500
Now,
Divide by 12 on both sides
So,
\(\frac{12m}{12}\) = \(\frac{$4,500}{12}\)
m = \(\frac{$4,500}{12}\)
Hence, from the above,
We can conclude that
The representation of the multiplication equation and the division equation to represent the given situation is:
a. Multiplication equation: 12m = $4,500
b. Division equation: m = \(\frac{$4,500}{12}\)

Question 23.
Model with Math Abel has 3,330 toothpicks. He wants to use them all to make a floor mat with 18 equal rows. Use the bar diagram to write a division equation. Then solve the equation to find how many toothpicks Abel should use in each row.
Envision Math Common Core 6th Grade Answers Topic 4 Represent And Solve Equations And Inequalities 27
Answer:
It is given that
Abel has 3,330 toothpicks. He wants to use them all to make a floor mat with 18 equal rows
Now,
The representation of the gven situation in the form of a bar diagram is:
Envision Math Common Core 6th Grade Answers Topic 4 Represent And Solve Equations And Inequalities 27
Now,
Let the number of toothpicks Abel shoud use in each row be: t
Now,
From the given bar diagram,
We can observe that
(The total number of toothpicks) ÷ (The number of tooth picks Abel should use in each row) = (The total number of rows)
\(\frac{3,330}{t}\) = 18
t = \(\frac{3,330}{18}\)
t = 185 toothpicks
Hence, from the above,
We can conclude that
The number of toothpicks Abel should use in each row is: 185 toothpicks

Question 24.
Model with Math Emily took an airplane trip. Her plane flew an equal number of miles each hour. Let m stand for the miles flown each hour. Write an equation to represent one way you can find how many miles Emily’s plane flew each hour.
Envision Math Common Core 6th Grade Answers Topic 4 Represent And Solve Equations And Inequalities 28
Answer:
It is given that
Emily took an airplane trip. Her plane flew an equal number of miles each hour. Let m stand for the miles flown each hour.
Now,
The given figure is:
Envision Math Common Core 6th Grade Answers Topic 4 Represent And Solve Equations And Inequalities 28
Now,
From the figure,
We can observe that
The total distance airplane travels is: 2,184 miles in 12 hours
So,
(The time took for airplane to reach Emily’s destination) × (The number of miles Emily’s plane flew each hour) = (The total distance an airplane travels in 12 hours)
12 × m = 2,184
12m = 2,184
Now,
Divide by 12 on both sides
So,
m = \(\frac{2,184}{12}\)
m = 182 miles
Hence, from the above,
We can conclude that
The number of miles Emily’s plane flew each hour is: 182 miles

In 25 and 26, use the triangle.
Envision Math Common Core 6th Grade Answers Topic 4 Represent And Solve Equations And Inequalities 29
Question 25.
The area of the isosceles triangle is 44 square centimeters. Use the equation \(\frac{1}{2}\)(8h) = 44 to find the height of the triangle.
Answer:
It is given that
The area of the isosceles triangle is 44 square centimeters
Now,
The given figure is:
Envision Math Common Core 6th Grade Answers Topic 4 Represent And Solve Equations And Inequalities 29
Now,
We know that,
The area of an Isosceles triangle (A) = \(\frac{1}{2}\) × Base × Height
So,
Accoring to the given information,
\(\frac{1}{2}\) × 8 × h = 44
\(\frac{8h}{2}\) = 44
8h = 44 × 2
8h = 88
h = \(\frac{88}{8}\)
h = 11 cm
Hence, from the above,
We can conclude that
The height of the given triangle is: 11 cm

Question 26.
If the perimeter of the triangle is 32 centimeters, what is the length of each of the two sides? Write and solve an equation.
Answer:
It is given that
The perimeter of the triangle is 32 centimeters
Now,
The given figure is:
Envision Math Common Core 6th Grade Answers Topic 4 Represent And Solve Equations And Inequalities 29
Now,
We know that,
In an Isosceles triangle, 2 sides lengths are the same
The “Perimeter” of any figure is the sum of all the side lengths in a given figure
Now,
Let x be the side length of the given triangle
So,
x + x + 8 = 32
2x + 8 = 32
2x = 32 – 8
2x = 24
x = \(\frac{24}{2}\)
x = 12 cm
Hence, from the above,
We can conclude that
The length of one side in the given triangle is: 12 cm
The length of the other side in the given triangle is: 12 cm

Question 27.
Kelsey and her 4 sisters spent an equal amount of time cleaning their home. Their parents added their times. They found that each of the 5 girls spent 3 hours cleaning. Let c be the total number of hours the girls spent cleaning. Write and solve a division equation to find the total number of hours the girls spent cleaning.
Answer:
It is given that
Kelsey and her 4 sisters spent an equal amount of time cleaning their home. Their parents added their times. They found that each of the 5 girls spent 3 hours cleaning. Let c be the total number of hours the girls spent cleaning.
So,
According to the given information,
(The total number of hours the girls spent cleaning) ÷ (The total number of girls) = (The time took each girl spent cleaning)
c ÷ 5 = 3
c = 3 × 5
c = 15 hours
Hence, from the above,
We can conclude that
The total number of hours the five girls spent cleaning is: 15 hours

Question 28.
Higher Order Thinking Veronica traveled 562 miles to Venice, Florida. She drove 85 miles every day. On the last day of her trip she only drove 52 miles. Write and solve an equation to find the number of days Veronica traveled. Explain each step of your problem-solving strategy.
Answer:
It is given that
Veronica traveled 562 miles to Venice, Florida. She drove 85 miles every day. On the last day of her trip she only drove 52 miles
Now,

Hence, from the above,
We can conclude that
The number of days Veronica traveled is: 7 days

Question 29.
Generalize A movie theater sells 11,550 tickets for 50 sold-out showings of the same movie. Write a division equation that you can use to find the number of people who bought tickets for each showing. Use what you know about dividing with larger numbers to solve the equation.
Envision Math Common Core 6th Grade Answers Topic 4 Represent And Solve Equations And Inequalities 30
Answer:
It is given that
A movie theater sells 11,550 tickets for 50 sold-out showings of the same movie.
Now,
Let the number of people who bought tickets for each showing be: p
So,
According to the given information,
(The total number of tickets sold by a movie theater) ÷ (The number of people who bought tickets for each showing) = (The number of showings)
11,550 ÷ p = 50
p = \(\frac{11,550}{50}\)
p = 231 people
Hence, from the above,
We can conclude that
The number of people who bought tickets for each showing is: 231 people

Assessment Practice

Question 30.
In October, Calvin’s school used 4,920 pounds of sand to protect the building against flooding during different tropical storms. One bag contains 40 pounds of sand.
Which of the following equations can be used to find how many bags of sand, b, Calvin’s school used in October?
A. 4,920b = 40
B. b ÷ 40 = 4,920
C. 40b = 4,920
D. b ÷ 4,920 = 40
Answer:
It is given that
In October, Calvin’s school used 4,920 pounds of sand to protect the building against flooding during different tropical storms. One bag contains 40 pounds of sand.
Now,
Let b be the number of bags of sand
So,
According to the given information,
(The number of bags) × (The amount of sand in each bag) = (The total amount of sand)
40b = 4,920
\(\frac{4,920}{b}\) = 40
Hence, from the above,
We can conclude that
The equation that can be used to find how many bags of sand, b, Calvin’s school used in October is:

Lesson 4.5 Write and Solve Equations with Rational Numbers

Explore It!
The cost of T-shirts for four different soccer teams are shown below.
I can… write and solve equations that involve rational numbers.
Envision Math Common Core 6th Grade Answers Topic 4 Represent And Solve Equations And Inequalities 31

A. Lorna is on Team A. Ben is on another team. They paid a total of $21.25 for both team T-shirts. Write an equation represent the cost of Ben’s shirt
Answer:
It is given that
The cost of T-shirts for four different soccer teams are shown below.
Now,
The given figure is:
Envision Math Common Core 6th Grade Answers Topic 4 Represent And Solve Equations And Inequalities 31
Now,
Let the cost of Ben’s shirt be: x
So,
According to the given information,
(The total cost of both T-shirts) = (The cost of Lorna’s shirt which is in Team A) + (The cost of Ben’s shirt)
$21.25 = $12.50 + x
So,
x = $21.25 – $12.50
x = $8.75
Hence, from the above,
We can conclude that
The cost of Ben’s shirt is: $8.75

B. Dario also plays soccer and he says that, based on the price of Ben’s T-shirt, Ben is on Team B. is Dario correct explain.
Answer:
It is given that
Dario said that based on the price of Ben’s T-shirt, Ben is on Team B
Now,
The given figure is:
Envision Math Common Core 6th Grade Answers Topic 4 Represent And Solve Equations And Inequalities 31
Now,
We know that,
From Part A,
The cost of Ben’s T-shirt is: $8.75
Now,
From the given figure,
We can observe that
The T-shirt costs $8.75 that belongs to Team C
Hence, from the above,
We can conclude that
Dario is not correct

Focus on math practices
Generalize How is solving for unknowns involving money like solving for unknowns involving whole numbers?
We know that,
The representation of money is as the same as the rational numbers
So,
Whatever operations we are performing on the whole numbers, we can perform the same operations on the unknowns involving money

Essential Question
How can you write and solve equations involving rational numbers?
Answer:
Convert the rational number to decimals, then solve it as a decimal instead, (with the optional choice of re-converting it back to rational number).

Try It!

Suppose you cut the shorter piece of fruit snack from the example above into two pieces. The longer of the two pieces is 1\(\frac{3}{8}\) feet long. Complete the bar diagram to represent the equation. Then find the length of the shorter piece.
Envision Math Common Core 6th Grade Answers Topic 4 Represent And Solve Equations And Inequalities 32
Answer:
It is given that
you cut the shorter piece of fruit snack from the example above into two pieces. The longer of the two pieces is 1\(\frac{3}{8}\) feet long.
Now,
From Example 1,
The length of the shorter piece is: 2\(\frac{1}{4}\) feet
Now,
The length of the longer piece in the given shorter piece is: 1\(\frac{3}{8}\) feet
Now,
Let the length of the shorter piece in the given shorter piece be: x
Now,
The representation of the given information in the form of a bar diagram is:

Now,
From the given diagram,
We can observe that

Hence, from the above,
We can conclude that
The shorter piece of the shorter piece is: \(\frac{7}{8}\) feet

Convince Me! How does the equation change if you know the length of the shorter piece is \(\frac{7}{8}\) foot and you want to know the length of the longer of the two pieces?
Answer:
If we know
The length of the shorter piece is: \(\frac{7}{8}\) foot
So,
The equation in the above problem will change into
\(\frac{7}{8}\) + x = 2\(\frac{1}{4}\)
Where,
x is the length of the other piece
Now,
x = 2\(\frac{1}{4}\) – \(\frac{7}{8}\)
x = 1\(\frac{3}{4}\) feet
So,
\(\frac{7}{8}\) < 1\(\frac{3}{4}\)
Hence, from the above,
We can conclude that
The length of the longer piece of the shorter piece is: 1\(\frac{3}{4}\) feet

Try It!

Solve \(\frac{5}{9}\)y = 25 for y.
Answer:
The given equation is:
\(\frac{5}{9}\)y = 25
Now,
Multiply with \(\frac{9}{5}\) on both sides
So,
\(\frac{5}{9}\) × \(\frac{9}{5}\)y = 25 × \(\frac{9}{5}\)
y = \(\frac{25 × 9}{5}\)
y = 45
Hence, from the above,
We can conclude that
The value of y for the given equation is: 45

Try It!

Molly also buys a bag of 8 apples for $3.60. Write and solve an equation to find how much Molly paid for each apple.
Answer:
It is given that
Molly also buys a bag of 8 apples for $3.60
So,
The cost of each apple = (The total cost of apples) ÷ (The total number of apples)
= \(\frac{$3.60}{8}\)
= $0.45
Hence, from the above,
We can conclude that
The cost of each apple is: $0.45

Try It!

Carmen spent $12.50 for a new notebook and a compass. The notebook cost $6.35. Write and solve an equation to find c, the cost of the compass.
Answer:
It is given that
Carmen spent $12.50 for a new notebook and a compass. The notebook cost $6.35
Now,
Let the cost of compass be c
So,
The total amount of money Carmen spent on a new notebook and a compass = (The cost of a compass) + (The cost of a new notebook)
$12.50 = c + $6.35
So,
c = $12.50 – $6.35
c = $6.15
Hence, from the above,
We can conclude that
The cost of a compass is: $6.15

KEY CONCEPT
You can solve equations using properties of equality and inverse relationships.
Envision Math Common Core 6th Grade Answers Topic 4 Represent And Solve Equations And Inequalities 33

Do You Understand?
Question 1.
Essential Question How can you write and solve equations involving rational numbers?
Answer:
Convert the rational number to decimals, then solve it as a decimal instead, (with the optional choice of re-converting it back to rational number).

Question 2.
Construct Arguments Why are inverse relationships important for solving equations?
Answer:
Mathematically, Inverse operations are opposite operations. Addition is the opposite of subtraction; division is the opposite of multiplication, and so on. Inverse operations are used to solve simple algebraic equations to more difficult equations that involve exponents, logarithms, and trigonometry

Question 3.
Critique Reasoning Johnny says that he solved the equation x – 3.5 = 7.2 by adding 3.5 to the left side of the equation. Explain whether Johnny is correct.
Answer:
It is given that
Johnny says that he solved the equation x – 3.5 = 7.2 by adding 3.5 to the left side of the equation.
Now,
The given equation is:
x – 3.5 = 7.2
Now,
We know that,
According to the “Addition Property of Equality”,
Add the same amount on both sides to make the equation balanced
So,
For the given equation,
Add 3.5 on both sides
So,
x – 3.5 + 3.5 = 7.2 – 3.5
Hence, from the above,
We can conclude that
Johnny is not correct

Question 4.
Generalize When solving an equation involving a mixed number, such as y + \(\frac{3}{4}\) = 4\(\frac{1}{2}\), what do you need to do to the mixed number?
Answer:
The given equation is:
y + \(\frac{3}{4}\) = 4\(\frac{1}{2}\)
Now,
To solve the given equation,
We have to convert the mixed number into a proper fraction
Hence, from the above,
We can conclude that
We have to convert the given mixed number into a proper fraction

Question 5.
Construct Arguments How is solving an equation with fractions like solving an equation with whole numbers? How is it different?
Answer:
Solving an equation with a fraction isn’t much different than solving an equation full of whole numbers. You group the variable terms on one side, the constants on the other, and then simplify.

Do You Know How?
In 6-14, solve each equation.
Question 6.
t – \(\frac{2}{3}\) = 25\(\frac{3}{4}\)
Answer:
The given equation is:
t – \(\frac{2}{3}\) = 25\(\frac{3}{4}\)
Now,

Hence, from the above,
We can conclude that
The value of t for the given expression is: \(\frac{317}{12}\)

Question 7.
\(\frac{f}{2}\) = \(\frac{5}{8}\)
Answer:
The given equation is:
\(\frac{f}{2}\) = \(\frac{5}{8}\)
Now,

Hence, from the above,
We can conclude that
The value of f for the given equation is: \(\frac{5}{4}\)

Question 8.
13.27 = t – 24.45
Answer:
The given equation is:
13.27 = t – 24.45
Now,

Hence, from the above,
We can conclude that
The value of t for the given equation is: \(\frac{943}{25}\)

Question 9.
r ÷ 5.5 = 18.2
Answer:
The given equation is:
r ÷ 5.5 = 18.2
Now,

Hence, from the above,
We can conclude that
The value of r for the given equation is: \(\frac{1,001}{10}\)

Question 10.
\(\frac{7}{10}=x – [latex]\frac{3}{5}\)
Answer:
The given equation is:
\(\frac{7}{10}=x-[latex]\frac{3}{5}\)
Now,

Hence, from the above,
We can conclude that
The value of x for the given equation is: \(\frac{13}{10}\)

Question 11.
1.8x = 40.14
Answer:
The given equation is:
1.8x = 40.14
Now,

Hence, from the above,
We can conclude that
The value of x for the given equation is: \(\frac{223}{10}\)

Question 12.
17.3 + v = 22.32
Answer:
The given equation is:
17.3 + v = 22.32
Now,

Hence, from the above,
We can conclude that
The value of v for the given equation is: \(\frac{251}{50}\)

Question 13.
9 = \(\frac{3}{8}\)y
Answer:
The given equation is:
9 = \(\frac{3}{8}\)y
Now,

Hence, from the above,
We can conclude that
The value of y for the given equation is: 24

Question 14.
1\(\frac{3}{4}+z=2[latex]\frac{2}{3}\)
Answer:
The given equation is:
1\(\frac{3}{4}+z=2[latex]\frac{2}{3}\)
Now,
Convert the given mixed numbers into the fraction form
So,
1\(\frac{3}{4}\) = \(\frac{7}{4}\)
2\(\frac{2}{3}\) = \(\frac{8}{3}\)
Now,

Hence, from the above,
We can conclude that
The value of y for the given equation is: \(\frac{11}{12}\)

Practice & Problem Solving

Leveled Practice
In 15-22, solve each equation.

Question 15.
w – 3.2 = 5.6
w – 3.2 + _____ = 5.6 + _____
w = _____
Answer:
The given equation is:
w – 3.2 = 5.6
Now,

Hence, from the above,
We can conclude that
The value of w for the given equation is: 8.8

Question 16.
9.6 = 1.6y
9.6 ÷ _____ = 1.6y ÷ ______
_______ = y
Answer:
The given equation is:
9.6 = 1.6y
Now,

Hence, from the above,
We can conclude that
The value of y for the given equation is: 6

Question 17.
48.55 + k = 61.77
48.55 + k – _____ = 61.77 – _____
k = _____
Answer:
The given equation is:
48.55 + k = 61.77
Now,

Hence, from the above,
We can conclude that
The value of k for the given equation is: 13.22

Question 18.
m ÷ 3.54 = 1.5
m ÷ 3.54 × _____ = 1.5 × _____
m = _____
Answer:
The given equation is:
m ÷ 3.54 = 1.5
Now,

Hence, from the above,
We can conclude that
The value of m for the given equation is: 5.31

Question 19.
\(7 \frac{1}{9}=2 \frac{4}{5}+m\)
Answer:
The given equation is:
\(7 \frac{1}{9}=2 \frac{4}{5}+m\)
Now,
Convert the given mixed numbers into fractions
So,
7\(\frac{1}{9}\) = \(\frac{64}{9}\)
2\(\frac{4}{5}\) = \(\frac{14}{5}\)
Now,

Hence, from the above,
We can conclude that
The value of m for the given equation is: \(\frac{194}{45}\)

Question 20.
\(a+3 \frac{1}{4}=5 \frac{2}{9}\)
Answer:
The given equation is:
\(a+3 \frac{1}{4}=5 \frac{2}{9}\)
Now,
Convert the given mixed numbers into the fractions
So,
3\(\frac{1}{4}\) = \(\frac{13}{4}\)
5\(\frac{2}{9}\) = \(\frac{47}{9}\)
Now,

Hence, from the above,
We can conclude that
The value of a for the given equation is: \(\frac{71}{36}\)

Question 21.
\(\frac{1}{8}\) • y=4
Answer:
The given equation is:
\(\frac{1}{8}\) • y=4
Now,

Hence, from the above,
We can conclude that
The value of y for the given equation is: 32

Question 22.
\(k-6 \frac{3}{8}=4 \frac{6}{7}\)
Answer:
The given equation is:
\(k-6 \frac{3}{8}=4 \frac{6}{7}\)
Now,
Convert the given mixed numbers into the fractions
So,
6\(\frac{3}{8}\) = \(\frac{51}{8}\)
4\(\frac{6}{7}\) = \(\frac{34}{7}\)
Now,

Hence, from the above,
We can conclude that
The value of k for the given equation is: \(\frac{629}{56}\)

Question 23.
Mr. Marlon buys these tickets for his family to visit the water park. The total cost is $210. Write and solve an equation to find the cost of each ticket.
Envision Math Common Core 6th Grade Answers Topic 4 Represent And Solve Equations And Inequalities 34
Answer:
It is given that
Mr. Marlon buys these tickets for his family to visit the water park. The total cost is $210
Now,
The given figure is:
Envision Math Common Core 6th Grade Answers Topic 4 Represent And Solve Equations And Inequalities 34
Now,
From the given figure,
We can observe that
The number of tickets Mr. Marlon bought is: 4
Now,
The cost of each ticket = (The total cost of tickets) ÷ (The total number of people)
= \(\frac{210}{4}\)
= $ 52.50
Hence, from the above,
We can conclude that
The cost of each ticket is: $52.50

Question 24.
Higher Order Thinking Without solving, tell which equation below has a greater solution. Explain.
\(\frac{5}{8}\)m = 2\(\frac{3}{4}\)
\(\frac{5}{9}\)m = 2\(\frac{3}{4}\)
Answer:
The given equations are:
\(\frac{5}{8}\)m = 2\(\frac{3}{4}\)
\(\frac{5}{9}\)m = 2\(\frac{3}{4}\)
Now,
We know that,
In the given fraction,
The numerators are the same and to find whether which fraction is greater,
We can compare by using the denominators
If the denominator is a small number when the numerators are the same then the other denominator, then the fraction that contains the smallest value of denominator is the greater fraction
Now,
From the given fractions,
When we compare the denominators,
8 < 9
Hence, from the above,
We can conclude that
The equation which is greater is:
\(\frac{5}{8}\)m = 2\(\frac{3}{4}\)

Question 25.
Make Sense and Persevere A high school track team’s long jump record is 21 feet 2\(\frac{1}{4}\) inches. This year, Tim’s best long jump is 20 feet 9\(\frac{1}{2}\) inches. If long jumps are measured to the nearest quarter inch, how much farther must Tim jump to break the record?
Answer:
It is given that
A high school track team’s long jump record is 21 feet 2\(\frac{1}{4}\) inches. This year, Tim’s best long jump is 20 feet 9\(\frac{1}{2}\) inches
Now,
We know that,
1 foot = 12 inches
So,
21 feet 2\(\frac{1}{4}\) inches = (21 × 12) + 2\(\frac{1}{4}\) inches
= 252 + 2\(\frac{1}{4}\) inches
= 252 + 2.25 inches
= 254.25 inches
So,
20 feet 9\(\frac{1}{2}\) inches = (20 × 12) + 9\(\frac{1}{2}\) inches
= 240 + 9\(\frac{1}{2}\) inches
= 240 + 9.5 inches
= 249.5 inches
So,
The distance Tim has to cross to break the record = 254.25 – 249.5
= 4.75 inches
Hence, from the above,
We can conclude that
The distance Tim has to cross to break the record is: 4.75 inches

Question 26.
Make Sense and Persevere About how many gallons of fuel does it take to move the space shuttle 3 miles from its hangar to the Vehicle Assembly Building?
Envision Math Common Core 6th Grade Answers Topic 4 Represent And Solve Equations And Inequalities 35
Answer:
The given figure is:
Envision Math Common Core 6th Grade Answers Topic 4 Represent And Solve Equations And Inequalities 35
Now,
From the given figure,
We can observe that
1 mile = 5,280 ft
It will take 1 gallon of fuel to move space shuttle 42 feet
Now,
The distance of 3 miles traveled by the space shuttle in fuel = (The number of feet required for 1 mile) × 3
= 3 × 5,280
= 15,840 feet
Now,
The number of gallons of fuel required to move the space shuttle 3 miles = (The total distance of 3 miles in feet) ÷ (The distance covered by 1 gallon of fuel)
= \(\frac{15,840}{42}\)
= 377 gallons
Hence, from the above,
We can conclude that
It will take 377 gallons of fuel to move the space shuttle 3 miles from its hangar to the Vehicle Assembly Building

Question 27.
Is the solution of b × \(\frac{5}{6}\) = 25 greater than or less than 25? How can you tell before computing?
Answer:
The given equation is:
b × \(\frac{5}{6}\) = 25
Now,
We know that,
When a multiplication is converted into a division, then the fraction after × or ÷ symbols will have to be reciprocal and vice-versa
So,
The reciprocal of \(\frac{5}{6}\) is: \(\frac{6}{5}\)
So,
b = 25 × \(\frac{6}{5}\)
Now,
We know that,
The value of \(\frac{6}{5}\) is greater than 1
So,
The solution for the given equation will also be greater than 25
Hence, from the above,
We can conclude that
The solution of b × \(\frac{5}{6}\) = 25 is greater than 25

Question 28.
What is the width of a rectangle with a length of \(\frac{3}{7}\) ft and an area of 2 ft2? Write an equation to show your work.
Answer:
It is given that
The area of a rectangle is: 2 ft²
The length of a rectangle is: \(\frac{3}{7}\) ft
Now,
Let the width of the rectangle be x
Now,
We know that,
The area of a rectangle = Length × Width
So,
2 = \(\frac{3}{7}\) × x
Now,

Hence, from the above,
We can conclude that
The width of the given rectangle is: \(\frac{14}{3}\) ft

Question 29.
Model with Math Helen is filling the pool shown for her little brother. She can carry 1\(\frac{7}{8}\) gallons of water each trip. Write and solve an equation to find how many trips Helen needs to make.
Envision Math Common Core 6th Grade Answers Topic 4 Represent And Solve Equations And Inequalities 36
Answer:
It is given that
Helen is filling the pool shown for her little brother. She can carry 1\(\frac{7}{8}\) gallons of water each trip
Now,
The given figure is:
Envision Math Common Core 6th Grade Answers Topic 4 Represent And Solve Equations And Inequalities 36
So,
The number of trips Helen needs to make = (The total amount of water held in the pool) ÷ (The amount of water Helen can carry each trip)
= 10\(\frac{1}{2}\) ÷ 1\(\frac{7}{8}\)
Now,

Hence, from the above,
We can conclude that
The number of trips Helen needs to make is about 6 trips

Question 30.
After the pool was full, Helen’s little brother and his friend splashed g gallons of water out of the pool. There are 7\(\frac{7}{8}\) gallons still left in the pool. Write and solve an equation to find how much water was splashed out of the pool.
Answer:
It is given that
After the pool was full, Helen’s little brother and his friend splashed g gallons of water out of the pool. There are 7\(\frac{7}{8}\) gallons still left in the pool
Now,
The given figure is:
Envision Math Common Core 6th Grade Answers Topic 4 Represent And Solve Equations And Inequalities 36
Now,
Let the amount of water splashed from the pool be: g gallons
So,
(The total amount of water present in the pool) – (The amount of water splashed out of the pool) = (The amount of water still left in the pool)
10\(\frac{1}{2}\) – g = 7\(\frac{7}{8}\)
g = 10\(\frac{1}{2}\) – 7\(\frac{7}{8}\)
Now,

Hence, from the above,
We can conclude that
The amount of water that splashed out from the pool is: 2\(\frac{5}{8}\) gallons

Question 31.
Grace solved the equation 2\(\frac{1}{2}\)y = \(\frac{5}{8}\). Her steps for the solution are shown in the table but are all mixed up. Write her steps in the correct order on the right side of the table.
Envision Math Common Core 6th Grade Answers Topic 4 Represent And Solve Equations And Inequalities 37
Answer:
It is given that
Grace solved the equation 2\(\frac{1}{2}\)y = \(\frac{5}{8}\). Her steps for the solution are shown in the table but are all mixed up.
Now,
The given equation is:
2\(\frac{1}{2}\)y = \(\frac{5}{8}\)
Now,

Hence,from the above,
We can conclude that
Her steps in the correct order on the right side of the table is:

Question 32.
The scientific name for the little bumps on your tongue is fungiform papillae. Each bump can contain many taste buds. The number of taste buds a person has varies. There are three general classifications of taste: supertaster, medium taster, and non-taster. Suppose a supertaster has 8,640 taste buds. Solve the equation 4.5n = 8,640 to find the number of taste buds, n, a non-taster may have.
Envision Math Common Core 6th Grade Answers Topic 4 Represent And Solve Equations And Inequalities 38
Answer:
It is given that
The scientific name for the little bumps on your tongue is fungiform papillae. Each bump can contain many taste buds. The number of taste buds a person has varies. There are three general classifications of taste: supertaster, medium taster, and non-taster. Suppose a supertaster has 8,640 taste buds
Now,
The given figure is:
Envision Math Common Core 6th Grade Answers Topic 4 Represent And Solve Equations And Inequalities 38
The given equation according to the given information is:
4.5n = 8,640
Now,

Hence, from the above,
We can conclude that
The number of taste buds a non-taster may have is: 1,920 taste buds

Question 33.
Model with Math In one study, the number of women classified as supertasters was 2.25 times the number of men classified as supertasters. Suppose 72 women were classified as supertasters. Write an equation that represents the number of men, m, who were classified as supertasters. Then solve the equation. How many men were classified as supertasters?
Answer:
It is given that
In one study, the number of women classified as supertasters was 2.25 times the number of men classified as supertasters. Suppose 72 women were classified as supertasters
Now,
From the given information,
The number of women who are supertasters = 2.25 × (The numberof men who are supertasters)
72 = 2.25 × (The number of men who are supertasters)
The number of men who are supertasters = \(\frac{72}{2.25}\)
= 32
Hence, from the above,
We can conclude that
The number of men who are classified as supertasters is: 32 men

Question 34.
Use Structure A fraction, f, multiplied by 5 equals \(\frac{1}{8}\). Write an algebraic sentence to show the equation. Then solve the equation and explain how you solved it.
Answer:
It is given that
A fraction, f, multiplied by 5 equals \(\frac{1}{8}\).
Now,
The representation of the given information in the form of an equation is:
f × 5 = \(\frac{1}{8}\)
5f = \(\frac{1}{8}\)
Now,

Hence, from the above,
We can conclude that
The value of f for the given equation is: \(\frac{1}{40}\)

Question 35.
Yelena needs to swim a total of 8 miles this week. So far, she swam 5\(\frac{3}{8}\) miles. Use the equation 5\(\frac{3}{8}\) + m = 8 to find how many more miles Yelena needs to swim.
Answer:
It is given that
Yelena needs to swim a total of 8 miles this week. So far, she swam 5\(\frac{3}{8}\) miles.
Now,
The givene quation according to the given information is:
5\(\frac{3}{8}\) + m = 8
Now,
Convert the given mixed number into a fraction
So,
\(\frac{43}{8}\) + m = 8
Now,

Hence, from the above,
We can conclude that
The number of miles Yelena needs to swim is: \(\frac{21}{8}\) miles

Question 36.
Can any equation that is written using addition be written as an equivalent equation using subtraction? Explain your reasoning and give an example containing decimals that shows your reasoning.
Answer:
Yes, any equation that is written using addition can be written as an equivalent equation using subtraction
Ex:
3.35 + y = 6.88
y = 6.88 – 3.35
y = 3.53
Now,
By using Subtraction:
Subtract any number let say 5 from both sides of 3.35 + y = 6.88
Now,
3.35 + y – 5 = 6.88 – 5
y – 1.65 = 1.88
y = 3.53

Question 37.
Critique Reasoning Oscar is 12 years old and his little sister is 6. Oscar uses a to represent his age. He says that he can use the expression a ÷ 2 to always know his sister’s age. Do you agree? Explain.
Answer:
It is given that
Oscar is 12 years old and his little sister is 6. Oscar uses a to represent his age. He says that he can use the expression a ÷ 2 to always know his sister’s age
Now,
Let the age of Oscar be: 18 years
Then,
The age of his sister will be: 8 – 6 = 12 years
So,
The relationship between the ages of Oscar and his sister is:
The age of Oscar = The age of his sister + 6
Hence, from the above,
We can conclude that
We can not agree with Oscar

Assessment Practice

Question 38.
Which value for y makes the equation 0.26y = 0.676 true?
A. y = 0.17576
B. y = 0.26
C. y = 2.6
D. y = 26
Answer:
The given equation is:
0.26y = 0.676
Now,

Hence, from the above,
We can conclude that
The value of y that makes the given equation true is:

Question 39.
Which value for x makes the equation 0.435 + x = 0.92 true?
A. x = 1.355
B. x = 0.595
C. x = 0.495
D. x = 0.485
Answer:
The given equation is:
0.435 + x = 0.92
Now,
x = 0.92 – 0.435
x = 0.485
Hence, from the above,
We can conclude that
The value of x that makes the givene quation true is:

Topic 4 Mid-Topic Checkpoint

Question 1.
Vocabulary Describe the relationship between equations and the properties of equality. Lessons 4-1 and 4-2
Answer:
The property of equality states that when we do arithmetic operations on both sides of an equation by the same number, the two sides remain equal and the property of equality is pplicable only on equations

In 2-4, write an equation for the situation. Then solve the equation.
Question 2.
A fraction f multiplied by 4 equals \(\frac{1}{2}\). Lesson 4-5
Answer:
The given situation is:
A fraction f multiplied by 4 equals \(\frac{1}{2}\)
Now,
The representation of the given situation in the form of an equation is:
4f = \(\frac{1}{2}\)
Now,
Divide the equation by 4 on both sides
So,
f = \(\frac{1}{2 × 4}\)
f = \(\frac{1}{8}\)
Hence, from the above,
We can conclude that
The value of f for the given equation is: \(\frac{1}{8}\)

Question 3.
When 832 is divided by n, the result is 16. Lesson 4-4
Answer:
The given situation is:
When 832 is divided by n, the result is 16
Now,
The representation of the given situation in the form of an equation is:
\(\frac{832}{n}\) = 16
Now,
Multiply the equation with n on both sides
So,
n = \(\frac{832}{16}\)
n = 52
Hence, from the above,
We can conclude that
The value of n for the given equation is: 52

Question 4.
When 10 is subtracted from x, the result is 6. Lesson 4-3
Answer:
The given situation is:
When 10 is subtracted from x, the result is 6.
Now,
The representation of the given situation in the form of an equation is:
x – 10 = 6
Now,
Add 10 on both sides for the equation
So,
x – 10 + 10 = 6 + 10
x = 16
Hence, from the above,
We can conclude that
The value of x for the given equation is: 16

Question 5.
Select all the equations that are equivalent to n-9 = 12. Lesson 4-2
☐ n – n – 9 = 12 – n
☐ n – 9 + 12 = 12 – 9
☐ n – 9 + 9 = 12 + 9
☐ n – 9 – n = 12 – n
☐ n – 9 + 9 = 12 – 12
Answer:
The given equation is: n – 9 = 12
Now,
We know that,
According to the “Property of Equality”,
We have to do the same arithmetic operation with the same number on both sides of the equation to keep the equation balanced
Hence, from the above,
We can conclude that
The equations that are equivalent to n – 9 = 12 are:

Question 6.
Select all the values ford that make the equation 9 = 18 ÷ d true. Lesson 4-1
☐ 2
☐ 0.5
☐ \(\frac{10}{5}\)
☐ 162
☐ \(\frac{1}{4}\)
Answer:
The given equation is:
9 = 18 ÷ d
Now,
Multiply with d on both sides fro the equation
So,
d = \(\frac{18}{9}\)
d = 2
Hence,f rom the above,
We can conclude that
The values of d that make the given equation true are:

Question 7.
The area, A, of a triangle is 15.3 square cm. Its base, b, is 4.5 cm. The formula for finding the area of a triangle is A = \(\frac{1}{2}\)bh. Write and solve an equation to find the height, h, of the triangle. Lessons 4-4 and 4-5
Envision Math Common Core 6th Grade Answers Topic 4 Represent And Solve Equations And Inequalities 39
Answer:
It is given that
The area, A, of a triangle is 15.3 square cm. Its base, b, is 4.5 cm. The formula for finding the area of a triangle is A = \(\frac{1}{2}\)bh.
Now,
The given figure is:
Envision Math Common Core 6th Grade Answers Topic 4 Represent And Solve Equations And Inequalities 39
Now,
According to the given formula in the given informatio,
15.3 = \(\frac{1}{2}\) × 4.5 × h
15.3 = 2.25 × h
h = \(\frac{15.3}{2.25}\)
h = 6.8 cm
Hence,f rom the above,
We can conclude that
The value of h of the given triangle is: 6.8 cm

Topic 4 Mid-Topic Performance Task

Ronald carved 3\(\frac{3}{8}\) feet of a totem pole. He says that the totem pole is \(\frac{3}{4}\) complete.
Envision Math Common Core 6th Grade Answers Topic 4 Represent And Solve Equations And Inequalities 40

PART A
If h represents the height, in feet, of the finished totem pole, then \(\frac{3}{4}\)h = 3\(\frac{3}{8}\) represents this situation. Which equations show the use of a reciprocal to write an equivalent equation that can be used to solve for h? Select all that apply.
Envision Math Common Core 6th Grade Answers Topic 4 Represent And Solve Equations And Inequalities 41
Answer:
It is given that
Ronald carved 3\(\frac{3}{8}\) feet of a totem pole. He says that the totem pole is \(\frac{3}{4}\) complete.
Now,
The given figure is:
Envision Math Common Core 6th Grade Answers Topic 4 Represent And Solve Equations And Inequalities 40
Now,
If h represents the height, in feet, of the finished totem pole, then
\(\frac{3}{4}\)h = 3\(\frac{3}{8}\) represents this situation
Now,
We know that,
The term “Reciprocal” is applicable only for multiplication and division
Now,
We know that,
The reciprocal of \(\frac{3}{4}\) is: \(\frac{4}{3}\)
Now,
We know that,
According to the “Property of Equality”,
The arithmetic operation we performed on the left side of th equation with a number will have to be the same to the right side of the equation also
Hence, from the above,
We can conclude that
The equations that show the use of a reciprocal to write an equivalent equation that can be used to solve for h are:

PART B
Use the equation in Part A to determine the height of the finished totem pole. Then write and solve an equation to find the height, s, of the section that has not been carved.
Answer:
From Part A,
The equation to determine the height h of the finished totem pole is:
\(\frac{3}{4}\)h = 3\(\frac{3}{8}\)
Now,
The given figure is:
Envision Math Common Core 6th Grade Answers Topic 4 Represent And Solve Equations And Inequalities 40
Now,
From the given figure,
We can observe that
The \(\frac{1}{4}\) of the section has not been carved
Now,
h = 3\(\frac{3}{8}\) × \(\frac{4}{3}\)
h = \(\frac{27}{8}\) × \(\frac{4}{3}\)
h = \(\frac{27 × 4}{8 × 3}\)
h = \(\frac{9}{2}\) ft
Now,
The height of the section that ahs not been carved (s) = \(\frac{h}{4}\)
= h × \(\frac{1}{4}\)
= \(\frac{9}{2}\) × \(\frac{1}{4}\)
= \(\frac{9}{8}\) ft
Hence, from the above,
We can conclude that
The height of the section that has not been carved is: \(\frac{9}{8}\) ft

PART C
Ronald spent $10.50 on tools and x dollars on the wood for the totem pole. His total cost for the totem pole is $19.35. The equation $10.50 + x = $19.35 represents this situation. What is the cost of the wood Ronald used?
Answer:
It is given that
Ronald spent $10.50 on tools and x dollars on the wood for the totem pole. His total cost for the totem pole is $19.35. The equation $10.50 + x = $19.35 represents this situation
Now,
The given equation that represents the given situation is:
$10.50 + x = $19.35
x = $19.35 – $10.50
x = $8.85
Hence, from the above,
We can conclude that
The cost of Wood Ronald used is: $8.85

PART D
To make the same totem pole with wood that costs y dollars, Ronald would have to spend a total of $35.19. Explain which property of equality Ronald could use to solve the equation $10.50 + y = $35.19 and why that property can be used. Then show how to use that property to solve for y.
Answer:
It is given that
To make the same totem pole with wood that costs y dollars, Ronald would have to spend a total of $35.19.
Now,
The given equation that is used to represent the given situation is:
$10.50 + y = $35.19
Now,
We know that,
According to the “Subtraction Property of Equality”,
Subtract the same amount from both sides to make the equation balanced
Now,
Subtract $10.50 from the given equation by using the “Subtraction Property of Equality”
So,
$10.50 + y – $10.50 = $35.19 – $10.50
y = $24.69
Hence, from the above,
We can conclude that
The cost of wood that is used to make the totem pole is: $24.69

Lesson 4.6 Understand and Write Inequalities

Solve & Discuss It!
The record time for the girls’ 50-meter freestyle swimming competition is 24.49 seconds. Camilla has been training and wants to break the record. What are some possible times Camilla would have to swim to break the current record?

I can… understand and write an inequality that describes a real-world situation.
Envision Math Common Core 6th Grade Answers Topic 4 Represent And Solve Equations And Inequalities 42
Answer:
It is given that
The record time for the girls’ 50-meter freestyle swimming competition is 24.49 seconds. Camilla has been training and wants to break the record”
Now,
The given table is:

Now,
From the given table,
We can observe that
To break the record,
Camilia has to swim greater than the time it is taken for the girls’ 50-meter freestyle swimming competition
So,
The possible times Camilia would have to swim to break the current record is: 26.56 seconds, 25.14 seconds, and 25.32 seconds
Hence, from the above,
We can conclude that
The possible times Camilia would have to swim to break the current record is:
a. 26.56 seconds b. 25.14 seconds c. 25.32 seconds

Reasoning
How do Camilla’s possible times compare to 24.49 seconds?
Answer:
From the above Exercise,
We know that,
Camilia wants to break the record
So,
The possible times of Camilia must be greater than 24.49 seconds
Hence, from the above,
We can conclude that
Camilia’s possible times are greater than 24.49 seconds

Focus on math practices
Be Precise Fran won a blue ribbon for growing the heaviest pumpkin. It weighed 217 pounds. What could be the weights of other pumpkins in the contest? How could you show the weights of the other pumpkins using a mathematical statement? Explain.
Answer:
It is given that
Fran won a blue ribbon for growing the heaviest pumpkin. It weighed 217 pounds.
Now,
We know that,
The heaviest weight means there is no other weight greater than that weight
Now,
From the given information,
217 pounds is the heaviest weight
So,
Other weights other than 217 pounds are lesser
Now,
Let the weight of all the other pumpkins be x
So,
According to the given information,
The weights of all the pumpkins present in the contest have lesser weight than the heaviest pumpkin
So,
The mathematical representation of the above statement is:
x < 217 pounds
Hence, from the above,
We can conclude that
The representation of the give information in the form of a mathematical statement is:
The weights of all the pumpkins present in the contest have lesser weight than the heaviest pumpkin. i.e.,
x < 217 pounds

Essential Question
How can you write an inequality to describe a situation?
Answer:
The steps to write an inequality to describe a situation are:
a. Read through the entire problem.
b. Highlight the important information and key words that you need to solve the problem.
c. Identify your variables.
d. Write the equation or inequality.
e. Solve.
f. Write your answer in a complete sentence.
g. Check or justify your answer

Try It!

Use the number line to show some of the ages of people who do not need to be accompanied by an adult. Write an inequality to represent the ages of people, n, who do not need to be accompanied by an adult.
Envision Math Common Core 6th Grade Answers Topic 4 Represent And Solve Equations And Inequalities 43
Answer:
Let the ages of the people who do not need to be accompanied by an adult be: n
Now,
The given figure is:

Now,
From the given figure,
We can observe that
The people who are less than 8 years have to be accompanied by an adult
Now,
The representation of the people who do not need to be accompanied by an adult in the given number line is:

So,
The inequality that represents ages of people ,n, who do not need to be accompanied by an adult is:
n > 8 years
Hence, from the above,
We can conclude that
The inequality that represents ages of people ,n, who do not need to be accompanied by an adult is:
n > 8 years

Convince Me! How do you know to which group an 8-year-old belongs, those who must be accompanied by an adult or those who do not need to be accompanied by an adult? Explain.
Answer:
The given figure is:

Now,
Let the number of children who must be accompanied by an adult be: x
Let the number of children who do not need to be accompanied by an adult be: y
Now,
From the given figure,
We can observe that
The children under the age of 8 must be accompanied by an adult
So,
The representation of an inequality that represents the ages of the children who must be accompanied by an adult is:
x < 8
The representation of an inequality that represents the ages of the children who need not to be accompanied by an adult is:
y > 8
Hence, from the above,
We can conclude that
The representation of an inequality that represents the ages of the children who must be accompanied by an adult is:
x < 8
The representation of an inequality that represents the ages of the children who need not to be accompanied by an adult is:
y > 8

Try It!

Write an inequality to represent each situation.
a. Harry is taller than 60 inches.
Answer:
The given situation is:
Harry is taller than 60 inches.
Now,
Let h be the height of Harry
Hence, from the above,
We can conclude that
The representation of the inequality that represents the given situation is:
h > 60 inches

b. Sherry is not 4 years old.
Answer:
The given situation is:
Sherry is not 4 years old
Now,
Let a be the age of Sherry
Hence, from the above,
We can conclude that
The representation of the inequality that represents the given situation is:
a ≠ 4 years

c. Hank has at least $7.50.
Answer:
The given situation is:
Hank has at least $7.50
Now,
Let s be the amount of money Hank has
Hence, from the above,
We can conclude that
The representation of the inequality that represents the given situation is:
s ≥ $7.50

KEY CONCEPT
Inequality symbols can be used to describe situations that have more than one possible solution.
Envision Math Common Core Grade 6 Answer Key Topic 4 Represent And Solve Equations And Inequalities 44

Do You Understand?
Question 1.
Essential Question How can you write an inequality to describe a situation?
Answer:
The steps to write an inequality to describe a situation are:
a. Read through the entire problem.
b. Highlight the important information and key words that you need to solve the problem.
c. Identify your variables.
d. Write the equation or inequality.
e. Solve.
f. Write your answer in a complete sentence.
g. Check or justify your answer

Question 2.
Generalize What is the difference between an equation with a variable and an inequality with a variable?
Answer:
An equation is a mathematical statement that shows the equal value of two expressions while an inequality is a mathematical statement that shows that an expression is lesser than or more than the other. An equation shows the equality of two variables while an inequality shows the inequality of two variables.

Question 3.
Would it be more efficient to use an inequality or to list all of the quantities less than 6? Explain.
Answer:
Without using “Inequalities”,
The qualities less than 6 are: -5, -4, -3, -2, -1, 0, 1, 2, 3, 4, and 5
By using the “Inequalities”,
The quantities less than 6 are: x < 6
Where,
x is the list of numbers that are less than 6

Question 4.
Generalize How are the symbols for greater than (>) and greater than or equal to (≥) related?
Answer:
“Greater than or equal to” and “less than or equal to” are just the applicable symbol with half an equal sign under it. For example, 4 or 3 ≥ 1 shows us a greater sign over half an equal sign, meaning that 4 or 3 are greater than or equal to 1. It works the other way, too

Do You Know How?
In 5-12, write an inequality for each situation.
Question 5.
A number, n, is greater than 22.
Answer:
The given situation is:
A number, n, is greater than 22.
Hence, from the above,
We can conclude that
The representation of the given situation in the form of an inequality is:
n > 22

Question 6.
The value, v, does not equal 2\(\frac{1}{2}\).
Answer:
The given situation is:
The value, v, does not equal 2\(\frac{1}{2}\).
Hence, from the above,
We can conclude that
The representation of the given situation in the form of an inequality is:
v ≠ 2\(\frac{1}{2}\)

Question 7.
Sally’s age, a, is at most 15.
Answer:
The given situation is:
Sally’s age, a, is at most 15.
Hence, from the above,
We can conclude that
The representation of the given situation in the form of an inequality is:
a ≤ 15

Question 8.
The width of the picture, w, is shorter than 8.5 inches.
Answer:
The given situation is:
The width of the picture, w, is shorter than 8.5 inches.
Hence, from the above,
We can conclude that
The representation of the given situation in the form of an inequality is:
w < 8.5 inches

Question 9.
Steve’s height, h, is at least 48 inches.
Answer:
The given situation is:
Steve’s height, h, is at least 48 inches.
Hence, from the above,
We can conclude that
The representation of the given situation in the form of an inequality is:
h ≥ 48 inches

Question 10.
Vera’s baby brother’s age, b, is not 24 months.
Answer:
The given situation is:
Vera’s baby brother’s age, b, is not 24 months.
Hence, from the above,
We can conclude that
The representation of the given situation in the form of an inequality is:
b ≠ 24 months

Question 11.
The number of quarters, q, in the jar is less than 75.
Answer:
The given situation is:
The number of quarters, q, in the jar is less than 75.
Hence, from the above,
We can conclude that
The representation of the given situation in the form of an inequality is:
q < 75

Question 12.
The length of the fish a fisherman catches, f, must be at least 10 inches for him to keep it.
Answer:
The given situation is:
The length of the fish a fisherman catches, f, must be at least 10 inches for him to keep it
Hence, from the above,
We can conclude that
The representation of the given situation in the form of an inequality is:
f ≥ 10 inches

Practice & Problem Solving

In 13-22, write an inequality for each situation.
Question 13.
Up to 12 people, p, can ride in the van.
Answer:
The given situation is:
Up to 12 people, p, can ride in the van.
Hence, from the above,
We can conclude that
The representation of the given situation in the form of an inequality is:
p ≤ 12 people

Question 14.
A number of days, d, of sunshine is not 28.
Answer:
The given situation is:
A number of days, d, of sunshine is not 28.
Hence, from the above,
We can conclude that
The representation of the given situation in the form of an inequality is:
d ≠ 28

Question 16.
The value, v, of the bracelet is less than $85.25.
Answer:
The given situation is:
The value, v, of the bracelet is less than $85.25.
Hence, from the above,
We can conclude that
The representation of the given situation in the form of an inequality is:
v < $85.25

Question 15.
The distance of the race, r, is farther than 6.2 miles.
Answer:
The given situation is:
The distance of the race, r, is farther than 6.2 miles.
Hence, from the above,
We can conclude that
The representation of the given situation in the form of an inequality is:
r > 6.2 miles

Question 17.
The number of people, p, that a restaurant can seat at one time is no more than 171.
Answer:
The given situation is:
The number of people, p, that a restaurant can seat at one time is no more than 171.
Hence, from the above,
We can conclude that
The representation of the given situation in the form of an inequality is:
p ≤ 171 people

Question 18.
The time, t, a customer has left on a parking meter is at least 25 minutes.
Answer:
The given situation is:
The time, t, a customer has left on a parking meter is at least 25 minutes.
Hence, from the above,
We can conclude that
The representation of the given situation in the form of an inequality is:
t ≥ 25 minutes

Question 19.
The bill, b, was less than $45.
Answer:
The given situation is:
The bill, b, was less than $45.
Hence, from the above,
We can conclude that
The representation of the given situation in the form of an inequality is:
b < $45

Question 20.
The girls live b blocks apart; they do not live 7\(\frac{1}{2}\) blocks apart.
Answer:
The given situation is:
The girls live b blocks apart; they do not live 7\(\frac{1}{2}\) blocks apart.
Hence, from the above,
We can conclude that
The representation of the given situation in the form of an inequality is:
b ≠ 7\(\frac{1}{2}\) blocks

Question 21.
The speed of the truck, s, must be no less than 34 miles per hour.
Answer:
The given situation is:
The speed of the truck, s, must be no less than 34 miles per hour.
Hence, from the above,
We can conclude that
The representation of the given situation in the form of an inequality is:
s ≤ 34 miles per hour

Question 22.
The number of baseball games, x, that Karen went to last year is more than 5.
Answer:
The given situation is:
The number of baseball games, x, that Karen went to last year is more than 5.
Hence, from the above,
We can conclude that
The representation of the given situation in the form of an inequality is:
x ≤ 5 games

Question 23.
Mia is taller than Gage. If m represents Mia’s height and g represents Gage’s height, write an inequality that shows the relationship between their heights.
Answer:
It is given that
Mia is taller than Gage and m represents Mia’s height and g represents Gage’s height
Now,
According to the given information,
The representation of the inequality for the given situation is:
m > g
Hence, from the above,
We can conclude that
The representation of the inequality for the given situation is:
m > g

Question 24.
Taryn sold gift-wrapping paper for a school fund-raiser. She sold at least 15 rolls of paper. Write an inequality to represent the amount of money, d, she earned for the fund-raiser.
Envision Math Common Core Grade 6 Answer Key Topic 4 Represent And Solve Equations And Inequalities 45
Answer:
It is given that
Taryn sold gift-wrapping paper for a school fund-raiser. She sold at least 15 rolls of paper
Now,
The given figure is:
Envision Math Common Core Grade 6 Answer Key Topic 4 Represent And Solve Equations And Inequalities 45
Now,
Let the number of rolls of paper be: x
So,
The representation of the inequality for representing the rolls of paper is:
x ≥ 15 rolls
Now,
The amount of money, d, Taryn earned for the fundraiser = (The total number of rolls) × (The cost of each roll)
= 15 × $8
= $120
So,
The representation of the inequality for representing the amount of money Taryn earned for the fundraiser is:
d ≥ $120
Hence, from the above,
We can conclude that
The representation of the inequality for representing the amount of money Taryn earned for the fundraiser is:
d ≥ $120

Question 25.
A city in New England just experienced its greatest 1-day snowfall. Write an inequality to represent a snowfall that would beat this record.
Envision Math Common Core Grade 6 Answer Key Topic 4 Represent And Solve Equations And Inequalities 46
Answer:
It is given that
A city in New England just experienced its greatest 1-day snowfall
Now,
The given figure is:
Envision Math Common Core Grade 6 Answer Key Topic 4 Represent And Solve Equations And Inequalities 46
Now,
Let the snowfall that would beat the given record be: t
From the given figure,
We can observe that
The representation of the inequality to represent a snowfall that would beat the given record is:
t > 19.7 inches of snow

Question 26.
The first bookcase, a, in a library can hold 1 less book than the second bookcase. The second bookcase holds 2,492 books. Write an inequality to represent the number of books the first bookcase can hold.
Answer:
It is given that
The first bookcase, a, in a library can hold 1 less book than the second bookcase. The second bookcase holds 2,492 books
So,
The number of books the first bookcase can hold = (The number of books the second bookcase can hold) – 1
= 2,492 – 1
= 2,491 books
Now,
Let the number of books the first bookcase can hold be: x
Let the number of books the second bookcase can hold be: y
So,
From the above,
We can observe that
x < y
Hence, from the above,
We can conclude that
The number of books the first bookcase can hold < The number of books the second bookcase can hold

Question 27.
A certain airplane must carry no more than 134 passengers during a flight. Write an inequality to represent the number of passengers, p, that would NOT be allowed during this flight.
Answer:
It is given that
A certain airplane must carry no more than 134 passengers during a flight
Now,
Let p be the number of passengers during a flight
So,
The representation of the inequality to represent the number of passengers that would not be allowed during this flight is:
p > 134 passengers
Hence, from the above,
We can conclude that
The representation of the inequality to represent the number of passengers that would not be allowed during this flight is:
p > 134 passengers

Question 28.
Higher Order Thinking To ride a certain roller coaster, a rider must be at least 42 inches tall. To represent this situation, Elias wrote h > 42 and Nina wrote h > 42. Who is correct? Explain.
Envision Math Common Core Grade 6 Answer Key Topic 4 Represent And Solve Equations And Inequalities 47
Answer:
It is given that
To ride a certain roller coaster, a rider must be at least 42 inches tall. To represent this situation, Elias wrote h > 42 and Nina wrote h > 42.
Now,
Let the height of the rider be: h inches
So,
The representation of the inequality that represents the height of a rider to ride a certain roller coaster is:
h ≥ 42
Hence, from the above,
We can conclude that
Elias is correct

Assessment Practice

Question 29.
Miguel earns extra money working two weekends with his dad. He is saving to buy a new bike that costs $140.
Heather says that Miguel needs to earn more than $6 for each hour that he works to have enough money to buy the bike. Her work is shown below. Write an inequality to explain why she is incorrect.
Envision Math Common Core Grade 6 Answer Key Topic 4 Represent And Solve Equations And Inequalities 48
$140 ÷ 23 hours > $6.00 per hour
Miguel has to earn more than $6.00 per hour.
Answer:
It is given that
Miguel earns extra money working two weekends with his dad. He is saving to buy a new bike that costs $140.
Heather says that Miguel needs to earn more than $6 for each hour that he works to have enough money to buy the bike. Her work is shown below
Now,
The given figure is:

Now,
From the given figure,
We can observe that
Miguel’s work record in Weekend 1 is: 9 hours
Miguel’s work record in Weekend 2 is: 7 hours
So,
The total number of hours in 2 Weekends for Migul = 9 + 7
= 16 hours
So,
The amount of money Migul will earn per hour = (The amount of money Migul saved to buy a new bike) ÷ (The total number of hours in 2 Weekends for Migul)
= \(\frac{$140}{16}\)
= $8.75
Hence, from the above,
We can conclude that
The inequality that represents Migul’s work is:
W > $8.75 per hour

Lesson 4.7 Solve Inequalities

Solve & Discuss It!
Henry is thinking of a number that is less than 17. What number could he be thinking of?
I can… write and represent solutions of inequalities.
Envision Math Common Core Grade 6 Answer Key Topic 4 Represent And Solve Equations And Inequalities 49
Answer:
It is given that
Henry is thinking of a number that is less than 17
Now,
Let x be the numbers less than 17
Where,
x may be an integer, decimal number (or) a whole number that is less than 17
So,
The representation of the inequality that represents the number that is less than 17 that Henry is thinking is:
x < 17
Hence, from the above,
We can conclude that
The representation of the inequality that represents the number that is less than 17 that Henry is thinking is:
x < 17

Use Appropriate Tools
How can you use a number line to show all the numbers that are less than 17?
Envision Math Common Core Grade 6 Answer Key Topic 4 Represent And Solve Equations And Inequalities 50
Answer:
We know that,
The numebrs that are 17 will be an integer, decimal number or a Whole number
Now,
The whole numbers that are less than 17 are: 0, 1, 2, 3, …….. 16
Hence, from the above,
We can conclude that
The representation of the list of all the Whole numbers that are less than 17 in the given number line is:

Focus on math practices
Reasoning Could Henry be thinking of 17? Explain.
Answer:
No, Henry should not be thinking of 17 because it is clearly mentioned that Henry is thinking of a number that is less than 17
So,
The list of the numbers that Henry may be thinking is: 0, 1, 2, 3 ….. 16

Essential Question
How can you represent the solutions of an inequality?
Answer:
The rules for Solving Inequalities are:
a. Add the same number on both sides.
b. From both sides, subtract the same number.
c. By the same positive number, multiply both sides.
d. By the same positive number, divide both sides.
e. Multiply the same negative number on both sides and reverse the sign.
A closed, or shaded, circle is used to represent the inequalities greater than or equal to (≥) or less than or equal to (≤) . The point is part of the solution. An open circle is used for greater than (>) or less than (<). The point is not part of the solution

Try It!

Graph all of the solutions of x < 8.
Envision Math Common Core Grade 6 Answer Key Topic 4 Represent And Solve Equations And Inequalities 51
To graph x < 8, draw a(n) ________ circle at 8 on the number line.
7 and 4 are two of the many possible solutions of the inequality.
Shade the solutions to the ________ of the ________ circle you drew at 8.
Answer:
The given inequality is: x < 8
Now,
The representation of the given inequality in the given number line is:

Now,


Hence, from the above,
We can conclude that
The solutions for the given inequality in the given number line is:

Convince Me! How does the graph of the inequality change when the less than sign is changed to a greater than sign? How does it stay the same?
Answer:
We know that multiplying or dividing an inequality by a negative number changes the direction of the inequality. In other words, a greater-than symbol becomes a less-than symbol, and vice versa
Now,
When the inequality has the symbol > (or) ≥, the numbers to the right of this inequality will become the solutions
When the inequality has the symbol < (or) ≤, the numbers to the left of this inequality will become the solutions

Try It!

There are no menu items on the children’s menu at the Flamingo Restaurant that cost more than $8.50. What are all the possible costs of the items on the children’s menu?
Envision Math Common Core Grade 6 Answer Key Topic 4 Represent And Solve Equations And Inequalities 52
Answer:
It is given that
There are no menu items on the children’s menu at the Flamingo Restaurant that cost more than $8.50
Now,
Let c be the cost of the menu items on the children’s menu at the Flamingo Restaurant
So,
The representation of the inequality that represents the given situation is:
c < $8.50
Hence,
The representation of all the possible costs of the items on the children’s menu on the given number line is:

Try It!

Which athletes, if any, would qualify for the finals if the length of a jump that qualifies for the finals were at least 20\(\frac{1}{2}\) feet?
Answer:
It is given that
The length of a jump that qualifies for the finals were at least 20\(\frac{1}{2}\) feet
Now,
The given table is:

Now,
Let the length of a jump be: d feet
Now,
The representation of the given situation in teh form of an inequality is:
d ≥ 20\(\frac{1}{2}\) feet
Now,
From the given table,
We can observe that
The values that are greater than 20\(\frac{1}{2}\) feet are:
20\(\frac{1}{2}\) = 20\(\frac{1}{2}\)
20\(\frac{1}{2}\) > 22\(\frac{1}{3}\)
Hence, from the above,
We can conclude that
Amir and Ryan would qualify for the finals if the length of a jump that qualifies for the finals were at least 20\(\frac{1}{2}\) feet

KEY CONCEPT
An inequality uses these symbols: <, >, ≤ , or ≥ to compare two expressions.
Envision Math Common Core Grade 6 Answer Key Topic 4 Represent And Solve Equations And Inequalities 53

Do You Understand?
Question 1.
Essential Question How can you represent the solutions of an inequality?
Answer:
The rules for Solving Inequalities are:
a. Add the same number on both sides.
b. From both sides, subtract the same number.
c. By the same positive number, multiply both sides.
d. By the same positive number, divide both sides.
e. Multiply the same negative number on both sides and reverse the sign.
A closed, or shaded, circle is used to represent the inequalities greater than or equal to (≥) or less than or equal to (≤) . The point is part of the solution. An open circle is used for greater than (>) or less than (<). The point is not part of the solution

Question 2.
In Example 1, why is 9 a solution of x > 5?
Answer:
The given equation is: x > 5
Now,
From the given equation,
We can observe that
Since x > 5,
The solutions to the given equation are all the numbers that are greater than 5
So,
The given equation will have infinitely many solutions that are greater than 5
Hence, from the above,
We can conclude that
9 is a solution of x > 5

Question 3.
Explain why 2 is NOT a solution of x > 5.
Answer:
The given equation is: x > 5
Now,
From the given equation,
We can observe that
Since x > 5,
The solutions to the given equation are all the numbers that are greater than 5
So,
The given equation will have infinitely many solutions that are greater than 5
Hence, from the above,
We can conclude that
2 is not a solution of x > 5

Question 4.
How many solutions does the inequality x > 12 have? Explain.
Answer:
The given equation is: x > 12
Now,
From the given equation,
We can observe that
Since x > 12,
The solutions to the given equation are all the numbers that are greater than 12
So,
The given equation will have infinitely many solutions that are greater than 12
Hence, from the above,
We can conclude that
x > 12 will have infinitely many solutions that are greater than 12

Question 5.
Generalize How do the graphs of the solutions of inequalities involving greater than (>) and greater than or equal to (≥) compare?
Answer:
A closed, or shaded, circle is used to represent the inequalities greater than or equal to (≥) or less than or equal to (≤) . The point is part of the solution. An open circle is used for greater than (>) or less than (<). The point is not part of the solution

Do You Know How?
In 6 and 7, write the inequality that each graph represents.
Question 6.
Envision Math Common Core Grade 6 Answer Key Topic 4 Represent And Solve Equations And Inequalities 54
Answer:
The given number line is:

Now,
From the given number line,
We can observe that
There is an open circle on 14.
So,
14 will not be included and the selected numbers are to the left side of 14 in the given number line
So,
The representation of the inequality for the given number line is:

Hence, from the above,
We can conclude that
The representation of the inequality for the given number line is:
z < 14

Question 7.
Envision Math Common Core Grade 6 Answer Key Topic 4 Represent And Solve Equations And Inequalities 55
Answer:
The given number line is:

Now,
From the given number line,
We can observe that
There is a closed circle on 18.
So,
18 will be included and the selected numbers are to the right side of 18 in the given number line
So,
The representation of the inequality for the given number line is:

Hence, from the above,
We can conclude that
The representation of the inequality for the given number line is:
d ≥ 18

In 8-11, substitute each given value of the variable to find which, if any, is a solution of the inequality.
Question 8.
w < 8         w = 4.3, 5.3, 8.3, 9
Answer:
The given inequality is: w < 8
Now,
The given list of numbers are: 4.3, 5.3, 8.3, and 9
Now,
When we observe the given list of numbers,
4.3, and 5.3 are less than 8
Hence, from the above,
We can conclude that
The solutions for the given inequality are: 4.3, 5.3

Question 9.
t > 25      t = 24, 25, 25.1, 27
Answer:
The given inequality is: t > 25
Now,
The given list of numbers are: 24, 25, 25.1, and 27
Now,
When we observe the given list of numbers,
25.1 and 27 are greater the 25
Hence, from the above,
We can conclude that
The solutions for the given inequality are: 25.1, 27

Question 10.
g ≤ 4        g = 0, 4, 5, 6
Answer:
The given inequality is: g ≤ 4
Now,
The given list of numbers are: 0, 4, 5, and 6
Now,
When we observe the given list of numbers,
0 and 4 are less than or equal to 4
Hence, from the above,
We can conclude that
The solutions for the given inequality are: 0, 4

Question 11.
y ≥ 28        y = 4, 5, 6, 7
Answer:
The given inequality is: y ≥ 8
Now,
The given list of numbers are: 4, 5, 6, and 7
Now,
When we observe the given list of numbers,
The given list of numbers are less than 28
Hence, from the above,
We can conclude that
There are no solutions for the given inequality from the given list of numbers

Practice & Problem Solving

In 12-15, write the inequality that each graph represents.
Question 12.
Envision Math Common Core Grade 6 Answer Key Topic 4 Represent And Solve Equations And Inequalities 56
Answer:
The given number line is:

Now,
From the given number line,
We can observe that
There is an open circle on 7.
So,
7 will not be included and the selected numbers are to the left side of 7 in the given number line
So,
The representation of the inequality for the given number line is:

Hence, from the above,
We can conclude that
The representation of the inequality for the given number line is:
y < 7

Question 13.
Envision Math Common Core Grade 6 Answer Key Topic 4 Represent And Solve Equations And Inequalities 57
Answer:
The given number line is:

Now,
From the given number line,
We can observe that
There is an open circle on 0.
So,
0 will not be included and the selected numbers are to the right side of 0 in the given number line
So,
The representation of the inequality for the given number line is:

Hence, from the above,
We can conclude that
The representation of the inequality for the given number line is:
b > 0

Question 14.
Envision Math Common Core Grade 6 Answer Key Topic 4 Represent And Solve Equations And Inequalities 58
Answer:
The given number line is:

Now,
From the given number line,
We can observe that
There is an open circle on 3.
So,
3 will not be included and the selected numbers are to the right side of 3 in the given number line
So,
The representation of the inequality for the given number line is:

Hence, from the above,
We can conclude that
The representation of the inequality for the given number line is:
x > 3

Question 15.
Envision Math Common Core Grade 6 Answer Key Topic 4 Represent And Solve Equations And Inequalities 59
Answer:
The given number line is:

Now,
From the given number line,
We can observe that
There is a closed circle on 5.
So,
5 will be included and the selected numbers are to the left side of 5 in the given number line
So,
The representation of the inequality for the given number line is:

Hence, from the above,
We can conclude that
The representation of the inequality for the given number line is:
t ≤ 5

In 16-19, graph each inequality on a number line.
Question 16.
h ≥ 9
Envision Math Common Core Grade 6 Answer Key Topic 4 Represent And Solve Equations And Inequalities 59
Answer:
The given inequality is: h ≥ 9
Hence, from the above,
We can conclude that
The representation of the given inequality on the number line is:

Question 17.
p < 3
Envision Math Common Core Grade 6 Answer Key Topic 4 Represent And Solve Equations And Inequalities 59
Answer:
The given inequality is: p < 3
Hence, from the above,
We can conclude that
The representation of the given inequality on the number line is:

Question 18.
t ≤ 6
Envision Math Common Core Grade 6 Answer Key Topic 4 Represent And Solve Equations And Inequalities 59
Answer:
The given inequality is: t ≤ 6
Hence, from the above,
We can conclude that
The representation of the given inequality on the number line is:

Question 19.
s > 1
Envision Math Common Core Grade 6 Answer Key Topic 4 Represent And Solve Equations And Inequalities 59
Answer:
The given inequality is: s > 1
Hence, from the above,
We can conclude that
The representation of the given inequality on the number line is:

In 20-27, name three solutions of each inequality.
Question 20.
x > 10.5
Answer:
The given inequality is: x > 10.5
Now,
We know that,
If the inequality has a “>” symbol, then the values that are greater than the given number will be the solutions for the equation
Hence, from the above,
We can conclude that
The three solutions for the given inequality are: 11, 12, and 13

Question 21.
r < 19
Answer:
The given inequality is: r < 19
Now,
We know that,
If the inequality has a “<” symbol, then the values that are lesser than the given number will be the solutions for the equation
Hence, from the above,
We can conclude that
The three solutions for the given inequality are: 15, 16, and 17

Question 22.
y ≥ 200
Answer:
The given inequality is: y ≥ 200
Now,
We know that,
If the inequality has a “≥” symbol, then the values that are greater than or equal to the given number will be the solutions for the equation
Hence, from the above,
We can conclude that
The three solutions for the given inequality are: 200, 205, and 210

Question 23.
m ≤ 82
Answer:
The given inequality is: m ≤ 82
Now,
We know that,
If the inequality has a “≤” symbol, then the values that are lesser than or equal to the given number will be the solutions for the equation
Hence, from the above,
We can conclude that
The three solutions for the given inequality are: 75, 80, and 81

Question 24.
x ≥ 12
Answer:
The given inequality is: x ≥ 12
Now,
We know that,
If the inequality has a “≥” symbol, then the values that are greater than or equal to the given number will be the solutions for the equation
Hence, from the above,
We can conclude that
The three solutions for the given inequality are: 12, 13, and 14

Question 25.
q ≤ 3.5
Answer:
The given inequality is: q ≤ 3.5
Now,
We know that,
If the inequality has a “≤” symbol, then the values that are lesster than or equal to the given number will be the solutions for the equation
Hence, from the above,
We can conclude that
The three solutions for the given inequality are: 0, 1.5, and 3.5

Question 26.
v > 35
Answer:
The given inequality is: v > 35
Now,
We know that,
If the inequality has a “>” symbol, then the values that are greater than the given number will be the solutions for the equation
Hence, from the above,
We can conclude that
The three solutions for the given inequality are: 40, 45, and 50

Question 27.
m < 2.5
Answer:
The given inequality is: m < 2.5
Now,
We know that,
If the inequality has a “<” symbol, then the values that are lesser than the given number will be the solutions for the equation
Hence, from the above,
We can conclude that
The three solutions for the given inequality are: 0, 1, and 2

Question 28.
The inequality w ≤ 1,500 describes the maximum weight in pounds, w, allowed by law in a freight elevator. Is a total weight of either 1,505 pounds or 1,600 pounds allowed in a freight elevator? Explain.
Envision Math Common Core Grade 6 Answer Key Topic 4 Represent And Solve Equations And Inequalities 61
Answer:
It is given that
The inequality w ≤ 1,500 describes the maximum weight in pounds, w, allowed by law in a freight elevator.
Now,
The given equation is: w ≤ 1,500
Now,
From the given inequality,
We can observe that
The solutions for the given inequality will be less than or equal to 1,500 but not greater than 1,500
So,
1,505 > 1,500 and 1,600 > 1,500
Hence, from the above,
We can conclude that
The total weight of either 1,505 pounds or 1,600 pounds will not be allowed in a freight elevator

Question 29.
Reasoning Graph the inequalities x > 2 and x < 2 on the same number line. What value, if any, is not a solution of either inequality? Explain.
Envision Math Common Core Grade 6 Answer Key Topic 4 Represent And Solve Equations And Inequalities 59
Answer:
The given inequalities are: x > 2 and x < 2
Now,
The representation of x > 2 on the given number line is:

The representtaion of x < 2 on the given number line is:

Now,
When we observe the number lines of x > 2 and x < 2,
We can say that
2 is not a solution for either of the number lines
Hence, from the above,
We can conclude that
2 is not a solution of either inequality

Question 30.
Model with Math Death Valley is the hottest place in the United States. The highest temperature ever recorded there was 134°F. The lowest temperature recorded there was 15°F. Write two inequalities that would describe the temperature, in degrees Fahrenheit, in Death Valley at any time since temperatures have been recorded.
Answer:
It is given that
Death Valley is the hottest place in the United States. The highest temperature ever recorded there was 134°F. The lowest temperature recorded there was 15°F
Now,
Let the highest temperature recorded in the Death Valley be: H
Let the lowest temperature recorded in the Death Valley be: L
So,
The inequality that represents the highest temperature in °F in the Death Valley is: H < 134° F
The inequality that represents the lowest temperature in °F in the Death Valley is: L > 15° F
Hence, from the above,
We can conclude that
The two inequalities that describe the temperature in the Death Vally at any time since the temperatures have been recorded is:
The inequality that represents the highest temperature in °F in the Death Valley is: H < 134° F
The inequality that represents the lowest temperature in °F in the Death Valley is: L > 15° F

Question 31.
The number line below represents the solutions of the inequality x > 7. Is 7.1 a solution? Is 7.01 a solution? Explain.
Envision Math Common Core Grade 6 Answer Key Topic 4 Represent And Solve Equations And Inequalities 62
Answer:
It is given that
The number line below represents the solutions of the inequality x > 7
Now,
The given number that represents the solution for the given inequality x > 7 is:
Envision Math Common Core Grade 6 Answer Key Topic 4 Represent And Solve Equations And Inequalities 62
Now,
From the above number line,
We can observe that
The solutions for x > 7 will be all the numbers that are greater than 7 and will be infinite
So,
7.1 > 7 and 7.01 > 7
Hence, from the above,
We can conclude that
7.1 and 7.01 are the solutions of the given inequality x > 7

Question 32.
The temperature in a greenhouse should be 65 degrees or higher. Write an inequality to describe the allowable temperature in the greenhouse.
Answer:
It is given that
The temperature in a greenhouse should be 65 degrees or higher.
Now,
Let the temperature of the greenouse be: t
So,
The inequality that describes the allowable temperature in the greenhouse is:
t ≥ 65°
Hence, from the above,
We can conclude that
The inequality that describes the allowable temperature in the greenhouse is:
t ≥ 65°

Question 33.
Higher Order Thinking Francine received a gift card to buy cell phone apps. She says that the card’s value is enough to buy any of the apps shown at the right. Let v be the dollar value of the gift card. Write an inequality that best describes the value of the gift card.
Envision Math Common Core Grade 6 Answer Key Topic 4 Represent And Solve Equations And Inequalities 63
Answer:
It is given that
Francine received a gift card to buy cell phone apps. She says that the card’s value is enough to buy any of the apps shown at the right. Let v be the dollar value of the gift card.
Now,
The given figure is:
Envision Math Common Core Grade 6 Answer Key Topic 4 Represent And Solve Equations And Inequalities 63
Now,
When we observe the given figure,
We can say that
The maximum vaue of the given gift card is: $12
So,
The costs of the given apps should be less than $12
So,
The representation of the inequality that describes the value of the gift card is:
v ≤ $12
Hence, from the above,
We can conclude that
The representation of the inequality that describes the value of the gift card is:
v ≤ $12

Question 34.
The maximum load on a small plane is 400 pounds. Let w represent the weight on the plane. Write an inequality to describe the allowable weight on the plane.
Answer:
It is given that
The maximum load on a small plane is 400 pounds. Let w represent the weight on the plane
Now,
Since the maximum load is 400 pounds, the remaining weights will be less than or equal to 400 pounds
So,
The representation of the inequalty that describes teh allowable weight on the plane is:
w ≤ 400 pounds
Hence, from the above,
We can conclude that
The representation of the inequalty that describes teh allowable weight on the plane is:
w ≤ 400 pounds

Question 35.
Jillian is thinking of a whole number that is greater than 21. What numbers, if any, make the inequality n > 21 true for n = 0, 1, 2, 3, 4, …?
Answer:
It is given that
Jillian is thinking of a whole number that is greater than 21
Now,
The given inequality is: n > 21
Now,
We know that,
If any inequality has “>” symbol, then the solutions for that inequality will be greater than thenumber and the solutions will be infinite
So,
The solutions for the given inequality will be: 22, 23, 24,……..
Hence, from the above,
We can conclude that
The numbers that make the inequality n > 21 true are: 22, 23, 24, …..

Assessment Practice

Question 36.
Select all the given values of y that make the inequality 3y < 25 true.
☐ 6.5
☐ 7
☐ 8
☐ 8.5
☐ 9
Answer:
The given inequality is: 3y < 25
Now,
Divide the given nequality be 3 on both sides
So,
\(\frac{3y}{3}\) < \(\frac{25}{3}\)
y < 8.3
Hence, from the above,
We can conclude that
All the values of y that make the inequality 3y < 25 true are:

Question 37.
Tania started a graph to show the inequality y < 3.7. Finish labeling the number line and draw the graph.
Envision Math Common Core Grade 6 Answer Key Topic 4 Represent And Solve Equations And Inequalities 64
Answer:
It is given that
Tania started a graph to show the inequality y < 3.7
Now,
The given inequality is: y < 3.7
Now,
The representation of the given number line and the solution for thegiven inequality is:

Hence, from the above,
We can conclude that
The completed number line and the solutions for y < 3.7 on the completed number line is:

3-ACT MATH

3-Act Mathematical Modeling: Checking a Bag
Envision Math Common Core Grade 6 Answer Key Topic 4 Represent And Solve Equations And Inequalities 65

АСТ 1
Question 1.
After watching the video, what is the first question that comes to mind?
Answer:

Question 2.
Write the Main Question you will answer.
Answer:

Question 3.
Construct Arguments Predict an answer to this Main Question. Explain your prediction.
Envision Math Common Core Grade 6 Answer Key Topic 4 Represent And Solve Equations And Inequalities 66
Answer:

Question 4.
On the number line below, write a number that is too small to be the answer. Write a number that is too large.
Envision Math Common Core Grade 6 Answer Key Topic 4 Represent And Solve Equations And Inequalities 67
Answer:

Question 5.
Plot your prediction on the same number line.
Answer:

ACT 2
Question 6.
What information in this situation would be helpful to know? How would you use that information?
Envision Math Common Core Grade 6 Answer Key Topic 4 Represent And Solve Equations And Inequalities 68
Answer:

Question 7.
Use Appropriate Tools What tools can you use to solve the problem? Explain how you would use them strategically.
Answer:

Question 8.
Model with Math Represent the situation using mathematics. Use your representation to answer the Main Question.
Answer:

Question 9.
What is your answer to the Main Question? is it higher or lower than your prediction? Explain why.
Envision Math Common Core Grade 6 Answer Key Topic 4 Represent And Solve Equations And Inequalities 69
Answer:

ACT 3
Question 10.
Write the answer you saw in the video.
Answer:

Question 11.
Reasoning Does your answer match the answer in the video? If not, what are some reasons that would explain the difference?
Envision Math Common Core Grade 6 Answer Key Topic 4 Represent And Solve Equations And Inequalities 70
Answer:

Question 12.
Make Sense and Persevere Would you change your model now that you know the answer? Explain.
Envision Math Common Core Grade 6 Answer Key Topic 4 Represent And Solve Equations And Inequalities 71
Answer:

Reflect
Question 13.
Model with Math Explain how you used a mathematical model to represent the situation. How did the model help you answer the Main Question?
Answer:

Question 14.
Was an equation or an inequality more useful to answer the Main Question? Explain.
Answer:

SEQUEL
Question 15.
Be Precise A different airline has a weight limit of 40 pounds for a checked bag. Explain how the answer would change for this airline.
Envision Math Common Core Grade 6 Answer Key Topic 4 Represent And Solve Equations And Inequalities 72
Answer:

Lesson 4.8 Understand Dependent and Independent Variables

Explain It!
Max is shipping a present to his grandmother.
I can… identify dependent and independent variables.
Envision Math Common Core Grade 6 Answer Key Topic 4 Represent And Solve Equations And Inequalities 73

A. What are three factors that will affect the weight of the box? What are three factors that will not affect the weight of the box?
Answer:
It is given that
Max is shipping a present to his grandmother.
Now,
The given figure is:
Envision Math Common Core Grade 6 Answer Key Topic 4 Represent And Solve Equations And Inequalities 73
Now,
From the given figure,
We can observe that
The three factors that will affect the weight of the box is:
a. Size of the box
b. The material of the box
c. Contents inside the box
The three factors that will not affect the weight of the box is:
a. The color of the box
b. The shape of the box
c. The design of the box

B. How might the size of the box and the contents of the box affect the weight of the box?
Answer:
We know that,
When we increase the size of the box, the contents of the box will also increase in the box and also the weight of the box will increase
So,
The relationship between the size of the box and the contents of the box and the weight of the box is:
Size of the box ∝ Contents of the box ∝ Weight of the box

Focus on math practices
Model with Math Describe another situation in which changing one factor results in changes to another factor.
Answer:
The example of a situation in which changing one factor results in changes to another factor is:
The distance a car travels, d, is dependent on the speed, s, at which it travels. So, speed is the independent variable and distance is the dependent variable
So,
Here,
The distance a car travels, d, is dependent on the speed, s, at which it travels. So, speed is the independent variable and distance is the dependent variable

Essential Question
What does it mean for one variable to be dependent on another variable?
Answer:
A dependent variable is a variable whose value will change depending on the value of another variable, called the independent variable. In a scientific experiment, it is the variable being tested, and therefore, it is called the dependent variable

Try It!

A baker used a certain number of cups of batter, b, to make p pancakes. Which variable, p, pancakes or b, batter is the dependent variable? Explain.
Answer:
It is given that
A baker used a certain number of cups of batter, b, to make p pancakes
Now,
We know that,
An “Independent variable” causes the dependent variable to change
A “Dependent variable” change in response to another variable
Now,
From the given situation,
We can observe that
The independent variable is: b
The dependent variable is: p
Hence, from the above,
We can conclude that
Pancakes (p) is the dependent variable

Convince Me! If the baker doubles the number of cups of batter used, b, what would you expect to happen to the number of pancakes made, p? Explain.
Answer:
It is given that
The baker doubles the number of cups of batter used, b
Now,
We know that,
An “Independent variable” causes the dependent variable to change
A “Dependent variable” change in response to another variable
So,
From the given situation,
We can observe that
If the baker doubles the number of cups of batter used, b, then the number of pancakes, p, would also be double in quantity
Hence, from the above,
We can conclude that
If the baker doubles the number of cups of batter used, b, then the number of pancakes, p, would also be double in quantity

Try It!

Jenna wants to rent a mountain bike by the week. Identify the independent variables that affect the total rental cost.
Answer:
It is given that
Jenna wants to rent a mountain bike by the week
Now,
From Example 3,
We can observe that
The dependent variables and independent present in Example 3 are:

Now,
We know that,
An “Independent variable” causes the dependent variable to change
A “Dependent variable” change in response to another variable
So,
The independent variables that affect the total rental cost for the given situation is:
a. Price per day (d)
b. Price per week (w)
c. Length of rental (l)
Hence, from the above,
We can conclude that
The independent variables that affect the total rental cost for the given situation is:
a. Price per day (d)
b. Price per week (w)
c. Length of rental (l)

KEY CONCEPT
A dependent variable changes in response to another variable, called an independent variable. An independent variable causes the change in a dependent variable. It is independent because its value is not affected by other variables.
The distance a car travels, d, is dependent on the speed, s, at which it travels. Speed is the independent variable, and distance is the dependent variable.

Do You Understand?
Question 1.
Essential Question What does it mean for one variable to be dependent on another variable?
Answer:
A dependent variable is a variable whose value will change depending on the value of another variable, called the independent variable. In a scientific experiment, it is the variable being tested, and therefore, it is called the dependent variable

Question 2.
Jake and Viola record the number of miles, m, they bike to help track the number of calories, s, they burn in an hour.
Envision Math Common Core Grade 6 Answer Key Topic 4 Represent And Solve Equations And Inequalities 74
Critique Reasoning Viola says the number of calories, c, they burn is the dependent variable. Do you agree? Explain.
Answer:
It is given that
Jake and Viola record the number of miles, m, they bike to help track the number of calories, s, they burn in an hour.
Now,
From the given situation,
We can observe that
The number of calories they burn is dependent on the number of miles
Now,
We know that,
An “Independent variable” causes the dependent variable to change
A “Dependent variable” change in response to another variable
So,
The independent variable for the given situation is: Number of miles
The dependent variable for the given situation is: Number of calories they burn
Hence, from the above,
We can conclude that
We can agree with Viola

Question 3.
Reasoning in the biking problem above, identify at least one other independent variable that could affect the dependent variable.
Answer:
The given situation from Exercise 2 is:

Now,
From the given situation,
We can observe that
The number of calories they burn is dependent on the number of miles
Now,
We know that,
An “Independent variable” causes the dependent variable to change
A “Dependent variable” change in response to another variable
So,
The independent variable for the given situation is: Number of miles
The dependent variable for the given situation is: Number of calories they burn
Hence, from the above,
We can conclude that
The independent variable that could affect the dependent variale for the given situation is: Number of miles

Do You Know How?
In 4-11, identify the independent variable and the dependent variable.
Question 4.
The amount of money, m, earned if t raffle tickets are sold
Answer:
The given situation is:
The amount of money, m, earned if t raffle tickets are sold
Now,
We know that,
An “Independent variable” causes the dependent variable to change
A “Dependent variable” change in response to another variable
So,
The independent variable for the given situation is: traffic tickets, t
The dependent variable for the given situation is: Amount of oney, m
Hence, from the above,
We can conclude that
The independent variable for the given situation is: traffic tickets, t
The dependent variable for the given situation is: Amount of money, m

Question 5.
The number of hours, h, worked and the amount of money, m, earned
Answer:
The given situation is:
The number of hours, h, worked and the amount of money, m, earned
Now,
We know that,
An “Independent variable” causes the dependent variable to change
A “Dependent variable” change in response to another variable
So,
The independent variable for the given situation is: Number of hours , h
The dependent variable for the given situation is: Amount of money, m
Hence, from the above,
We can conclude that
The independent variable for the given situation is: traffic tickets, t
The dependent variable for the given situation is: Amount of money, m

Question 6.
The number of shelves, s, in a bookcase and the number of books, b, the bookcase can hold
Answer:
The given situation is:
The number of shelves, s, in a bookcase and the number of books, b, the bookcase can hold
Now,
We know that,
An “Independent variable” causes the dependent variable to change
A “Dependent variable” change in response to another variable
So,
The independent variable for the given situation is: Number of shelves, s
The dependent variable for the given situation is: Number of books, b, the bookcase can hold
Hence, from the above,
We can conclude that
The independent variable for the given situation is: Number of shelves, s
The dependent variable for the given situation is: Number of books, b, the bookcase can hold

Question 7.
The number of pages, p, you read in your book in h hours
Answer:
The given situation is:
The number of pages, p, you read in your book in h hours
Now,
We know that,
An “Independent variable” causes the dependent variable to change
A “Dependent variable” change in response to another variable
So,
The independent variable for the given situation is: Number of pages, p
The dependent variable for the given situation is: Number of hours, h
Hence, from the above,
We can conclude that
The independent variable for the given situation is: Number of pages, p
The dependent variable for the given situation is: Number of hours, h

Question 8.
The number of gallons, g, of water a garden hose produces after running for m minutes
Answer:
The given situation is:
The number of gallons, g, of water a garden hose produces after running for m minutes
Now,
We know that,
An “Independent variable” causes the dependent variable to change
A “Dependent variable” change in response to another variable
So,
The independent variable for the given situation is: Number of gallons, g, of water
The dependent variable for the given situation is: Number of minutes, m
Hence, from the above,
We can conclude that
The independent variable for the given situation is: Number of gallons, g, of water
The dependent variable for the given situation is: Number of minutes, m

Question 9.
The number of peaches, y, a farmer harvests in x bushels
Answer:
The given situation is:
The number of peaches, y, a farmer harvests in x bushels
Now,
We know that,
An “Independent variable” causes the dependent variable to change
A “Dependent variable” change in response to another variable
So,
The independent variable for the given situation is: Number of bushels, x
The dependent variable for the given situation is: Number of peaches, y
Hence, from the above,
We can conclude that
The independent variable for the given situation is: Number of bushels, x
The dependent variable for the given situation is: Number of peaches, y

Question 10.
The number of hours, h, you spend driving at a speed of r miles per hour
Answer:
The given situation is:
The number of hours, h, you spend driving at a speed of r miles per hour
Now,
We know that,
An “Independent variable” causes the dependent variable to change
A “Dependent variable” change in response to another variable
So,
The independent variable for the given situation is: Number of miles per hour, r
The dependent variable for the given situation is: Number of hours, h
Hence, from the above,
We can conclude that
The independent variable for the given situation is: Number of miles per hour, r
The dependent variable for the given situation is: Number of hours, h

Question 11.
Name at least two independent variables that could result in a change in a monthly electric bill.
Answer:
The two independent variables that could result in a change in a monthly electric bill is:
a. Outside temperature and humidity
b. The frequency of the electrical appliances inside the house you use

Practice & Problem Solving

In 12-15, identify the independent variable and the dependent variable.
Question 12.
The pages, p, in a book and the weight, w, of the book
Answer:
The given situation is:
The pages, p, in a book and the weight, w, of the book
Now,
We know that,
An “Independent variable” causes the dependent variable to change
A “Dependent variable” change in response to another variable
So,
The independent variable for the given situation is: Number of pages, p, in a book
The dependent variable for the given situation is: Weight, w, of the book
Hence, from the above,
We can conclude that
The independent variable for the given situation is: Number of pages, p, in a book
The dependent variable for the given situation is: Weight, w, of the book

Question 13.
The number of hamburgers, h, sold and the dollar amount of sales, s, taken in
Answer:
The given situation is:
The number of hamburgers, h, sold and the dollar amount of sales, s, taken in
Now,
We know that,
An “Independent variable” causes the dependent variable to change
A “Dependent variable” change in response to another variable
So,
The independent variable for the given situation is: Number of hamburgers, h, sold
The dependent variable for the given situation is: The amount of sales, s, taken in
Hence, from the above,
We can conclude that
The independent variable for the given situation is: Number of hamburgers, h, sold
The dependent variable for the given situation is: The amount of sales, s, taken in

Question 14.
The pounds, p, of flour you buy and the number of bread loaves, b, you want to make
Answer:
The given situation is:
The pounds, p, of flour you buy and the number of bread loaves, b, you want to make
Now,
We know that,
An “Independent variable” causes the dependent variable to change
A “Dependent variable” change in response to another variable
So,
The independent variable for the given situation is: Number of pounds, p, of flour you buy
The dependent variable for the given situation is: Number of bread loaves, b, you want to make
Hence, from the above,
We can conclude that
The independent variable for the given situation is: Number of pounds, p, of flour you buy
The dependent variable for the given situation is: Number of bread loaves, b, you want to make

Question 15.
The temperature, t, of water and the number of minutes, m, the water is in the freezer
Answer:
The given situation is:
The temperature, t, of water and the number of minutes, m, the water is in the freezer
Now,
We know that,
An “Independent variable” causes the dependent variable to change
A “Dependent variable” change in response to another variable
So,
The independent variable for the given situation is: The temperature, t, of water
The dependent variable for the given situation is: Number of minutes, m, the water is in the freezer
Hence, from the above,
We can conclude that
The independent variable for the given situation is: The temperature, t, of water
The dependent variable for the given situation is: Number of minutes, m, the water is in the freezer

Question 16.
Write your own situation. Identify the independent and dependent variables.
Answer:
In an experiment in Psychology, on the impact of sleep deprivation on test performance,
a. Sleep deprivation would be the independent variable.
b. The dependent variable is the variable that is measured by the experimenter

Question 17.
Name at least two independent variables that could result in a change in the price of a basket of grapefruits.
Answer:
The independent variables that could result in a change in the price of a basket of grapefruits is:
a. The weight of grapefruits
b. The size of a basket

Question 18.
Critique Reasoning You spend c dollars for p identical pairs of pants. A friend claims that because c increases if you increase p, and p increases if you increase c, either corp could be the independent variable. Is your friend right or wrong? Explain.
Answer:
It is given that
You spend c dollars for p identical pairs of pants. A friend claims that because c increases if you increase p, and p increases if you increase c, either corp could be the independent variable.
Now,
We know that,
An “Independent variable” causes the dependent variable to change
A “Dependent variable” change in response to another variable
Now,
From the given situation,
We can observe that
The independent variable for the given situation is: Identical pairs of pants, p
The dependent variable for the given situation is: Amount of money spent, c
Hence, from the above,
We can conclude that
Your friend is wrong

Question 19.
The number of oranges in a bag and the cost of the bag of oranges are related. What is the independent variable in this relationship? Explain.
Answer:
It is given that
The number of oranges in a bag and the cost of the bag of oranges are related
Now,
We know that,
An “Independent variable” causes the dependent variable to change
A “Dependent variable” change in response to another variable
Now,
From the given situation,
We can observe that
The independent variable for the given situation is: Number of oranges
The dependent variable for the given situation is: The cost of oranges
Hence, from the above,
We can conclude that
The independent variable for the given situation is: Number of oranges

Question 20.
The dependent variable g represents the growth of a plant. What variables can represent independent variables in this situation?
Envision Math Common Core Grade 6 Answer Key Topic 4 Represent And Solve Equations And Inequalities 75
Answer:
It is given that
The dependent variable g represents the growth of a plant
Now,
We know that,
An “Independent variable” causes the dependent variable to change
A “Dependent variable” change in response to another variable
Now,
From the given figure,
We can observe that
The independent variable for the given situation is: The number of plants
Hence, from the above,
We can conclude that
The independent variable for the given situation is: The number of plants

In 21 and 22, use the table at the right.
Envision Math Common Core Grade 6 Answer Key Topic 4 Represent And Solve Equations And Inequalities 76
Question 21.
The table shows distances driven by the Williams family each day of their vacation. What is an independent variable that would affect the total distance they drove each day?
Answer:
It is given that
The table shows distances driven by the Williams family each day of their vacation
Now,
The given table is:
Envision Math Common Core Grade 6 Answer Key Topic 4 Represent And Solve Equations And Inequalities 76
Now,
From the given table,
We can observe that
The independent variable that would affect the total distance is: Number of days
Hence, from the above,
We can conclude that
The independent variable that would affect the total distance is: Number of days

Question 22.
Name at least two dependent variables that could affect the amount of money the Williams family spends on meals during their vacation.
Answer:
The given table is:
Envision Math Common Core Grade 6 Answer Key Topic 4 Represent And Solve Equations And Inequalities 76
Now,
From the given table,
We can observe that
The independent variables that could affect the amount of money the Williams family spends on meals during their vacation is:
a. The number of items present during meals
b. The number of days
Hence, from the above,
We can conclude that
The independent variables that could affect the amount of money the Williams family spends on meals during their vacation is:
a. The number of items present during meals
b. The number of days

Question 23.
The cost of a salad at a restaurant depends on many factors. List at least two independent variables that could affect the cost of a salad.
Answer:
It is given that
The cost of a salad at a restaurant depends on many factors
So,
The independent variables that could affect the cost of a salad for the given situation is:
a. The amount of a salad
b. The type of restaurant
Hence, from the above,
We can conclude that
The independent variables that could affect the cost of a salad for the given situation is:
a. The amount of a salad
b. The type of restaurant

Question 24.
Julian drove from New York to Florida. List at least two independent variables that could affect the number of days Julian took to make the trip.
Answer:
It is given that
Julian drove from New York to Florida
So,
The independent variables that could affect the number of days Julian took to make the trip is:
a. The distance between New York and Florida
b.  The number of hours she drove each day
Hence, from the above,
We can conclude that
The independent variables that could affect the number of days Julian took to make the trip is:
a. The distance between New York and Florida
b.  The number of hours she drove each day

Question 25.
The number of incorrect answers and the score on a math test are related. What is the dependent variable in this relationship? Explain.
Answer:
It is given that
The number of incorrect answers and the score on a math test are related
Now,
From the given situation,
We can observe that
The dependent variable for the given situation is: The score on a math test
Hence, from the above,
We can conclude that
The dependent variable for the given situation is: The score on a math test

Question 26.
Higher Order Thinking Write a situation in which time, t, is an independent variable Then write a situation in which time, t, is a dependent variable.
Answer:
In the study of uniform motion, “Time” is an independent variable, denoted by t and represented on the horizontal axis, while position is a dependent variable, denoted by x and represented on the vertical axis.
“Time” is usually viewed as the independent variable for the simple reason that it doesn’t depend on anything else. Time simply ticks by at the same rate wherever you are (in non-relativistic context), independent of other variables so it doesn’t make sense to express time as a dependent variable

Assessment Practice

Question 27.
Jonas is concerned about the amount of water he uses to wash his laundry. He made a table to show the number of gallons of water used by different washing machines to complete a load of laundry.
Envision Math Common Core Grade 6 Answer Key Topic 4 Represent And Solve Equations And Inequalities 77

PART A
Use variables to represent the independent and dependent quantities shown in the table.
Answer:
It is given that
Jonas is concerned about the amount of water he uses to wash his laundry. He made a table to show the number of gallons of water used by different washing machines to complete a load of laundry.
Now,
The given table is:
Envision Math Common Core Grade 6 Answer Key Topic 4 Represent And Solve Equations And Inequalities 77
Now,
From the given table,
We can observe that
The independent variable for the given situation is: Type of Washing Machine
The dependent variable for the given situation is: Gallons of Water for a Washing Machine
Hence, from the above,
We can conclude that
The independent variable for the given situation is: Type of Washing Machine
The dependent variable for the given situation is: Gallons of Water for a Washing Machine

PART B
Use variables to represent the dependent variable and the independent variable in this sentence.
Jonas records the total cost of the water he uses and the number of gallons of water he uses.
Answer:
It is given that
Jonas records the total cost of the water he uses and the number of gallons of water he uses.
Now,
From the given sentence,
We can observe that
The independent variable for the given sentence is: Gallons of water
The dependent variable for the given sentence is: The total cost of water
Hence, from the above,
We can conclude that
The independent variable for the given sentence is: Gallons of water
The dependent variable for the given sentence is: The total cost of water

Lesson 4.9 Use Patterns to Write and Solve Equations

Solve & Discuss It!
The table below shows how many candles are in different numbers of boxes. Find a pattern that explains the relationship between the values of c and b. Use words and numbers to describe the pattern. How many candles will there be in 10 boxes?

I can… use patterns to write and solve equations with variables.
Envision Math Common Core Grade 6 Answers Topic 4 Represent And Solve Equations And Inequalities 78
Answer:
It is given that
The table below shows how many candles are in different numbers of boxes.
Now,
The given table is:
Envision Math Common Core Grade 6 Answers Topic 4 Represent And Solve Equations And Inequalities 78
Now,
From the given table,
We can observe that
The pattern that explains the relationship between the number of candles and the number of boxes is:
The number of boxes is one-fourth of the number of candles
Now,
The mathematical representation for the above pattern is:
The number of boxes = \(\frac{1}{4}\) × (The number of candles)
Now,
From the above relationship,
The number of candles present in 10 boxes is:
The number of boxes = \(\frac{1}{4}\) × (The number of candles)
10 = \(\frac{1}{4}\) × (The number of candles)
The number of candles = 10 × 4
The number of candles = 40
Hence, from the above,
We can conclude that
The pattern that explains the relationship between the number of candles and the number of boxes is:
The number of boxes is one-fourth of the number of candles
The number of candles present in 10 boxes is: 40 candles

Look for Relationships
How can you get from each value in the left column to its matching value in the right column?
Envision Math Common Core Grade 6 Answers Topic 4 Represent And Solve Equations And Inequalities 79
Answer:
The given table is:
Envision Math Common Core Grade 6 Answers Topic 4 Represent And Solve Equations And Inequalities 78
Now,
From the above Problem,
The mathematical representation for the pattern between the left column and the right column is:
The number of boxes = \(\frac{1}{4}\) × (The number of candles)
So,
The number of candles = 4 × (The number of boxes)
Hence, from the above,
We can conclude that
The relationship between each value in the left column to its matching value in the right column is:
The number of candles = 4 × (The number of boxes)

Focus on math practices
Use Structure Write a rule that explains how you get from the values in the right column of the table above to the values in the left column.
Answer:
The given table is:
Envision Math Common Core Grade 6 Answers Topic 4 Represent And Solve Equations And Inequalities 78
Now,
The rule that explains the relationship between the values in the right column of the table to the values in the left column is:
The number of boxes = \(\frac{1}{4}\) × (The number of candles)

Essential Question
How can you use a pattern to write and solve an equation?
Answer:
Problems that are solved most easily by finding a pattern include those that ask students to extend a sequence of numbers or to make a prediction based on data.  Finding a Pattern is an appropriate strategy to use to solve the problem. This is a pattern that is predictable and will be continuous

Try It!

The table shows the number of yards, y, that a professional bicyclist rides in s seconds. Find a pattern that relates the variables. If the cyclist maintains this speed, how far would the cyclist ride in 8 seconds?
Envision Math Common Core Grade 6 Answers Topic 4 Represent And Solve Equations And Inequalities 80
Answer:
It is given that
The table shows the number of yards, y, that a professional bicyclist rides in s seconds.
Now,
The given table is:
Envision Math Common Core Grade 6 Answers Topic 4 Represent And Solve Equations And Inequalities 80
Now,
From the given table,
We can observe that
The relationship between the number of seconds, s, and the number of yards, y is:
12.2 × s = y
12.2s = y
Now,
The number of yards traveled by the cyclist in 8 seconds is:
12.2 × 8 = y
y = 97.6 yards
Hence, from the above,
We can conclude that
The relationship between the number of seconds, s, and the number of yards, y is:
12.2s = y
The number of yards traveled by the cyclist in 8 seconds is: 97.6 yards

Convince Me! How do you know that the equation you wrote describes the pattern in the table?
Answer:
The given table is:
Envision Math Common Core Grade 6 Answers Topic 4 Represent And Solve Equations And Inequalities 80
Now,
From the above Problem,
The relationship (or) pattern between the number of seconds, s, and the number of yards, y is:
12.2s = y
Now,
Substitute 2, 3, 5 in the place of s
So,
y = 12.2 (2) = 24.4 yards
y = 12.2 (3) = 36.6 yards
y = 12.2 (5) = 61 yards
Hence, from the above,
We can conclude that
The equation that you wrote describes the pattern in the given table because the ratio between the number of seconds and the number of yards is constant

Try It!

If Ethan continues to pay $5 per week, how many more weeks will he need to pay his mother after 12 weeks? Explain.
Answer:
From Example 2,
It is given that
Ethan owes his mother $75 and he repays his mother a set amount of $5 each week
Now,
From Example 2,
The pattern for the given situation is: $75 – $5n
Where,
n is the number of weeks
Now,
For 12 weeks,
The amount Eathan owed to his mother = $75 – $5 (12)
= $75 – $60
= $15
Now,
We know that,
Eathan has to pay a set amount of $5 each week
So,
The number of weeks more Eathan has to pay $5 for his mother = \(\frac{$15}{$5}\)
= 3 weeks
Hence, from the above,
We can conclude that
The number of more weeks will ethan need to pay for his mother after 12 weeks is: 3 weeks

KEY CONCEPT
You can use patterns in a table to write an equation that relates the independent and dependent variables.
Envision Math Common Core Grade 6 Answers Topic 4 Represent And Solve Equations And Inequalities 81

Do You Understand?
Question 1.
Essential Question How can you use a pattern to write and solve an equation?
Answer:
Problems that are solved most easily by finding a pattern include those that ask students to extend a sequence of numbers or to make a prediction based on data.  Finding a Pattern is an appropriate strategy to use to solve the problem. This is a pattern that is predictable and will be continuous

Question 2.
Make Sense and Persevere How do you find a pattern that relates the values in a table?
Answer:
To determine the rule for a table, simply look at one pair of input/output numbers and figure out what number has been added, subtracted, multiplied, or divided by it. Then, make sure that pattern matches the rest of the numbers. Once you know the rule for a table, you can finish filling in the rest of it

Question 3.
Reasoning In Example 2, what happens to the value of the dependent variable, a, the amount still owed, when the value of the independent variable, w, the number of weeks Ethan pays $5, is increased by 1?
Answer:
From Example 2,
It is given that
Ethan owes his mother $75 and he repays his mother a set amount of $5 each week
Now,
From Example 2,
The pattern for the given situation is: $75 – $5n
Where,
n is the number of weeks
Now,
It is given that
Ethan owes his mother $75 and he repays his mothe a set amount of $s each week for 13 weeks
Now,
For 13 weeks,
The amount Eathan owed to his mother = $75 – $5 (13)
= $75 – $65
= $10
Hence, from the baove,
We can conclude that
The amount Etan still owed to his mother after 13 weeks is: $10

Question 4.
Look for Relationships Use the pattern in the table below to write an equation.
Envision Math Common Core Grade 6 Answers Topic 4 Represent And Solve Equations And Inequalities 82
Answer:
The given table is:
Envision Math Common Core Grade 6 Answers Topic 4 Represent And Solve Equations And Inequalities 82
Now,
From the given table,
We can observe that
The pattern between the values of x and y is:
y = 5x + 2
Hence, from the above,
We can conclude that
The pattern in the given table to represent as an equation is:
y = 5x + 2

Do You Know How?
Question 5.
The table shows Brenda’s age, b, when Talia’s age, t, is 7, 9, and 10. Find the pattern and then write a rule and an equation that represents the pattern. Then find Brenda’s age when Talia is 12.
Envision Math Common Core Grade 6 Answers Topic 4 Represent And Solve Equations And Inequalities 83
Answer:
It is given that
The table shows Brenda’s age, b, when Talia’s age, t, is 7, 9, and 10.
Now,
The given table is:
Envision Math Common Core Grade 6 Answers Topic 4 Represent And Solve Equations And Inequalities 83
Now,
From the given table,
We can observe that
The pattern between Talia’s age and Brenda’s age is:
Brenda’s age = Talia’s age – 5
b = t – 5
So,
Brenda’s age when Talia’s age is 12 is:
b = 12 – 5
b = 7 years
Hence, from the above,
We can conclude that
The pattern between Talia’s age and Brenda’s age is:
Brenda’s age = Talia’s age – 5
Brenda’s age when Talia’s age is 12 is: 7 years

In 6 and 7, use the table below.
Envision Math Common Core Grade 6 Answers Topic 4 Represent And Solve Equations And Inequalities 84
Question 6.
Use the equation y = 2x – 7 to complete the table.
Answer:
The given table is:
Envision Math Common Core Grade 6 Answers Topic 4 Represent And Solve Equations And Inequalities 84
Now,
The given pattern (or) equation that represents the given table is:
y = 2x – 7
Now,
For x = 7,
y = 2 (7) – 7
= 14 – 7
= 7
For x = 8,
y = 2 (8) – 7
= 16 – 7
= 9
Hence, from the above,
We can conclude that
The completed table is:

Question 7.
State the rule for the pattern in words.
Answer:
The given pattern for the given table is:
y = 2x – 7
Now,
The given pattern in terms of words is:
The value of y is equal to the value of 7 less than the 2 times the value of x
Hence, from the above,
We can conclude that
The given pattern in terms of words is:
The value of y is equal to the value of 7 less than the 2 times the value of x

Practice & Problem Solving

In 8 and 9, write a rule and an equation that represents the pattern in each table.
Question 8.
Envision Math Common Core Grade 6 Answers Topic 4 Represent And Solve Equations And Inequalities 85
Answer:
The given table is:
Envision Math Common Core Grade 6 Answers Topic 4 Represent And Solve Equations And Inequalities 85
Now,
From the given table,
We can observe that
The representation of the pattern for the given table in the form of an equation is:
y = x + 32
Hence, from the above,
We can conclude that
The equation that represents the pattern in the given table is:
y = x + 32

Question 9.
Envision Math Common Core Grade 6 Answers Topic 4 Represent And Solve Equations And Inequalities 86
Answer:
The given table is:
Envision Math Common Core Grade 6 Answers Topic 4 Represent And Solve Equations And Inequalities 86
Now,
From the given table,
We can observe that
The representation of the pattern for the given table in the form of an equation is:
n = 3m
Hence, from the above,
We can conclude that
The equation that represents the pattern in the given table is:
n = 3m

In 10 and 11, write a rule and an equation that represents the pattern in each table. Then complete the table.
Question 10.
Envision Math Common Core Grade 6 Answers Topic 4 Represent And Solve Equations And Inequalities 87
Answer:
The given table is:
Envision Math Common Core Grade 6 Answers Topic 4 Represent And Solve Equations And Inequalities 87
Now,
From the given table,
We can observe that
The representation of the pattern for the given table in the form of an equation is:
k = g – 15
So,
The completed table by using the above pattern is:

Hence, from the above,
We can conclude that
The equation that represents the pattern in the given table is:
k = g – 15
The completed table by using the above pattern is:

Question 11.
Envision Math Common Core Grade 6 Answers Topic 4 Represent And Solve Equations And Inequalities 88
Answer:

The given table is:
Envision Math Common Core Grade 6 Answers Topic 4 Represent And Solve Equations And Inequalities 88
Now,
From the given table,
We can observe that
The representation of the pattern for the given table in the form of an equation is:
y = \(\frac{x}{9}\)
So,
The completed table by using the above pattern is:

Hence, from the above,
We can conclude that
The equation that represents the pattern in the given table is:
y = \(\frac{x}{9}\)
The completed table by using the above pattern is:

Question 12.
To celebrate its 125th anniversary, a company produced 125 expensive teddy bears. These “125 Karat Teddy Bears” are made of gold thread and have diamonds for eyes. The table shows the approximate cost of different numbers of these bears. Write an equation that can be used to find c, the cost of n bears.
Envision Math Common Core Grade 6 Answers Topic 4 Represent And Solve Equations And Inequalities 89
Answer:
It is given that
To celebrate its 125th anniversary, a company produced 125 expensive teddy bears. These “125 Karat Teddy Bears” are made of gold thread and have diamonds for eyes. The table shows the approximate cost of different numbers of these bears.
Now,
The given table is:
Envision Math Common Core Grade 6 Answers Topic 4 Represent And Solve Equations And Inequalities 89
Now,
From the given table,
We can observe that
The pattern between the number of bears and the cost of bears is:
The cost of n Teddy bears = 47,000 × (n Teddy Bears)
So,
c = 47,000n
Hence, from the above,
We can conclude that
The equation that can be used to find c, the cost of n bears is:
c = 47,000n

Question 13.
Andrea attends the county fair. The fair charges for admission and for each ride is shown below.
Envision Math Common Core Grade 6 Answers Topic 4 Represent And Solve Equations And Inequalities 90
a. Use the pattern in the table to the find the cost for Andrea to ride 5 rides or 8 rides. Then write an equation for the pattern.
Answer:
It is given that
Andrea attends the county fair. The fair charges for admission and for each ride is shown
Now,
The given table is:

Now,
From the given table,
We can observe that
The pattern between the number of rides and the cost of rides is:
The cost of r rides = 2.5 × (Tne number of rides) + 8
So,
c = 2.5r + 8
Now,
The completed table by using the above pattern is:

Hence, from the above,
We can conclude that
The pattern to find the cost for Andrea to ride 5 rides or 8rides is:
c = 2.5r + 8
The completed table by using the above pattern is:

b. Find the cost, c, for 12 rides.
Answer:
From Part (a),
The pattern to find the cost of r rides is:
c = 2.5r + 8
So,
The cost for 12 rides is:
c = 2.5 (12) + 8
c = $38
Hence, from the above,
We can conclude that
The cost for Andrea to ride 12 rides is: $38

In 14 and 15, write an equation that best describes the pattern in each table.
Question 14.
Envision Math Common Core Grade 6 Answers Topic 4 Represent And Solve Equations And Inequalities 91
Answer:
The given table is:
Envision Math Common Core Grade 6 Answers Topic 4 Represent And Solve Equations And Inequalities 91
Now,
From the given table,
We can observe that
The pattern between w and z is:
z = w – 2
Hence, from the above,
We can conclude that
The equation that represents the pattern in the given table is:
z = w – 2

Question 15.
Envision Math Common Core Grade 6 Answers Topic 4 Represent And Solve Equations And Inequalities 92
Answer:
The given table is:
Envision Math Common Core Grade 6 Answers Topic 4 Represent And Solve Equations And Inequalities 92
Now,
From the given table,
We can observe that
The pattern between x and y is:
y = 4x
Hence, from the above,
We can conclude that
The equation that represents the pattern in the given table is:
y = 4x

In 16-19, use the equation to complete each table.
Question 16.
t = 5d + 5
Envision Math Common Core Grade 6 Answers Topic 4 Represent And Solve Equations And Inequalities 93
Answer:
The given equation is:
t = 5d + 5
Now,
The given table that is related to the given equation is:
Envision Math Common Core Grade 6 Answers Topic 4 Represent And Solve Equations And Inequalities 93
Now,
For d = 3,
t = 5 (3) + 5
= 20
For d = 4,
t = 5 (4) + 5
= 25
So,
The completed table is:

Hence, from the above,
We can conclude that
The completed by using the given equation is:

Question 17.
y = \(\frac{1}{2}\)x – 1
Envision Math Common Core Grade 6 Answers Topic 4 Represent And Solve Equations And Inequalities 94
Answer:
The given equation is:
y = \(\frac{1}{2}\)x – 1
Now,
The given table that is related to the given equation is:
Envision Math Common Core Grade 6 Answers Topic 4 Represent And Solve Equations And Inequalities 94
Now,
For x = 8,
y = \(\frac{1}{2}\) (8) – 1
= 3
For x = 10,
y = \(\frac{1}{2}\) (10) – 1
= 4
So,
The completed table is:

Hence, from the above,
We can conclude that
The completed by using the given equation is:

Question 18.
y = 2x + 1
Envision Math Common Core Grade 6 Answers Topic 4 Represent And Solve Equations And Inequalities 95
Answer:
The given equation is:
y = 2x + 1
Now,
The given table that is related to the given equation is:
Envision Math Common Core Grade 6 Answers Topic 4 Represent And Solve Equations And Inequalities 95
Now,
For x = 2,
y = 2 (2) + 1
= 5
For x = 3,
y = 2 (3) + 1
= 7
So,
The completed table is:

Hence, from the above,
We can conclude that
The completed by using the given equation is:

Question 19.
b = \(\frac{a}{2}\) – 2
Envision Math Common Core Grade 6 Answers Topic 4 Represent And Solve Equations And Inequalities 96
Answer:
The given equation is:
b = \(\frac{a}{2}\) – 2
Now,
The given table that is related to the given equation is:
Envision Math Common Core Grade 6 Answers Topic 4 Represent And Solve Equations And Inequalities 96
Now,
For a = 17,
b = \(\frac{17}{2}\) – 2
= 6.5
For a = 14,
b = \(\frac{14}{2}\) – 2
= 5
For a = 11,
b = \(\frac{11}{2}\) – 2
= 3.5
For a = 8,
b = \(\frac{8}{2}\) – 2
= 2
For a = 5,
b = \(\frac{5}{2}\) – 2
= 0.5
So,
The completed table is:

Hence, from the above,
We can conclude that
The completed by using the given equation is:

Question 20.
Higher Order Thinking Maya wrote the equation h= d+ 22 to represent the relationship shown in the table. Is this equation correct? Explain.
Envision Math Common Core Grade 6 Answers Topic 4 Represent And Solve Equations And Inequalities 97
Answer:
It is given that
Maya wrote the equation h= d+ 22 to represent the relationship shown in the table.
Now,
The given table is:
Envision Math Common Core Grade 6 Answers Topic 4 Represent And Solve Equations And Inequalities 97
Now,
From the given table,
We can observe that
The relationship between the values of h and d is:
d = 11h
So,
h = d + 22 and d = 11h are different relationships
Hence, from the above,
We can conclude that
h = d + 22 is not correct

Assessment Practice

Question 21.
The table below shows the total cost for the number of movie tickets purchased. Write an equation that represents the relationship between these two quantities. Use the equation to find the cost of 6 tickets.
Envision Math Common Core Grade 6 Answers Topic 4 Represent And Solve Equations And Inequalities 98
Answer:
It is given that
The table below shows the total cost for the number of movie tickets purchased
Now,
The given table is:
Envision Math Common Core Grade 6 Answers Topic 4 Represent And Solve Equations And Inequalities 98
Now,
From the given table,
We can purchase that
The relationship between the number of tickets and their cost is:
Cost of n tickets (c) = 8.75 × (Number of tickets)
So,
c = 8.75n
Where,
c is teh cost of n tickets
n is the number of tickets
Now,
The cost of 6 tickets by using the above pattern is:
c = 8.75 (6)
c = $52.5
Hence, from the above,
We can conclude that
The equation that represents the relationship between the number of tickets and their cost is:
c = 8.75n
The cost of 6 tickets by using the above equation is: $52.5

Lesson 4.10 Relate Tables, Graphs, and Equations

Solve & Discuss It!
Nancy walks 4 blocks to Maria’s house. Together, they continue the walk. The walk can be described as n = m + 4, where n is the number of blocks Nancy walks and m is the number of blocks Maria walks. Describe Nancy’s house how the equation, data table, and graph reflect the walk.
I can… analyze the relationship between dependent and independent variables in tables, graphs, and equations.
Envision Math Common Core Grade 6 Answers Topic 4 Represent And Solve Equations And Inequalities 99Envision Math Common Core Grade 6 Answers Topic 4 Represent And Solve Equations And Inequalities 100
Answer:
It is given that
Nancy walks 4 blocks to Maria’s house. Together, they continue the walk. The walk can be described as n = m + 4, where n is the number of blocks Nancy walks and m is the number of blocks Maria walks.
Now,
The given equation that represents the given situation is:
n = m + 4
Where,
n is the number of blocks Nancy walks
m is the number of blocks Maria walks
Now,
From the given equation,
We can observe that
n is the output
m is the input
Now,
The given data table and the graph for the given situation is:
Envision Math Common Core Grade 6 Answers Topic 4 Represent And Solve Equations And Inequalities 100
Now,
From the given data table,
We can observe that
m is the input
n is the output
We can take any value of input and calculate the value of output
Now,
From the given graph,
We can observe that
The graph can be drawn by taking the ordered pairs (m, n) and if we observe the graph, we can say that it is a straight line without passingthrough the origin

Look for Relationships How can you use the values in one row of the data table to describe the relationship shown in the equation, data table, and graph?
Envision Math Common Core Grade 6 Answers Topic 4 Represent And Solve Equations And Inequalities 100
Answer:
From the above,
We know that,
The given equation that represents the given situation is:
n = m + 4
Where,
n is the number of blocks Nancy walks
m is the number of blocks Maria walks
Now,
From the given equation,
We can observe that
n is the output
m is the input
Now,
The given data table and the graph for the given situation is:
Envision Math Common Core Grade 6 Answers Topic 4 Represent And Solve Equations And Inequalities 100
Now,
From the given data table,
We can observe that
m is the input
n is the output
We can take any value of input and calculate the value of output
Now,
From the given graph,
We can observe that
The graph can be drawn by taking the ordered pairs (m, n) and if we observe the graph, we can say that it is a straight line without passingthrough the origin

Focus on math practices
Model with Math Draw a line through the points on the graph. What ordered pair on the line includes m = 5? Explain what that ordered pair represents.
Answer:
The given data table is:

Now,
We know that,
The representation of the ordered pair is: (Input, Output)
Now,
From the given data table,
We can observe that
The ordered pairs are: (1, ), (2, 6), (3, 7)
So,
The representation of the ordered pairs in the coordinate plane is:

Now,
From the above,
We can observe that
The equation that represents the above situation is:
n = m + 4
Now,
For m = 5,
n = 5 + 4
= 9
So,
The ordered pair when m = 5 is: (5, 9)
The above ordered pair represents that
When Maria walks 5 blocks, Nncy walks 9 blocks
Hence, from the above,
We can conclude that
The ordered pair when m = 5 is: (5, 9)
The above ordered pair represents that
When Maria walks 5 blocks, Nancy walks 9 blocks

Essential Question
How can you analyze the relationship between dependent and independent variables using tables, graphs, and equations?
Answer:
In a graph, the X-axis runs horizontally (side to side) and the Y-axis runs vertically (up and down). Typically, the independent variable will be shown on the X axis and the dependent variable will be shown on the Y axis
In a table, the independent variable will be shown in the left side or the top of the column ans the dependent variable will be shown in the right side or the bottom of the column
In an equation, the independent variable will be shown at the right side of “=” and the dependent variable will be shown at the left side of “=”

Try It!

The booster club now raises $0.41 for each pom pom they sell. Complete the table and graph. Write and solve an equation to find how many pom poms they need to sell to raise $50.
Envision Math Common Core Grade 6 Answers Topic 4 Represent And Solve Equations And Inequalities 101
Answer:
It is given that
The booster club now raises $0.41 for each pom pom they sell.
Now,
From Example 1,
We can observe that
The representation of the equation that represents the amount of money raised to the number of pom poms sold is:
r = 0.45n
Where,
r is the amount of money raised
n is the number of pom poms
Now,
According to the given situation,
The representation of the equation that represents the amount of money raised to the number of pom poms sold is:
r = 0.41n
Now,
For n = 150,
r = 0.41 (150)
= 61.5
So,
The completed table is:

The representation of the graph in the coordinate plane according to the above table is:

Now,
For r = $50,
r = 0.41n
$50 = 0.41n
n = \(\frac{50}{0.41}\)
n = 121.95
n ≅ 122
Hence, from the above,
We can conclude that
The number of pom poms they needed to sell to raise $50 must be atleast 122 pom poms

Convince Me! How does finding three values for x and y help you represent the relationship between x and y?
Answer:
We know that,
In a graph,
The x-values represent on the horizontal axis and the y-values represent on the vertical axis
Now,
The representation of x and y values in a graph gives us the information that whether the equation is linear or not

Try It!

A company makes decorations for pens. All the supplies cost $5, and the company plans to sell the decorations for $2 apiece. Analyze the relationship between the number of decorations sold and the profit by completing the table and the graph. Use the table and the graph to write and solve an equation to find the number of decorations that must be sold for the company to make a $15 profit.
Envision Math Common Core Grade 6 Answers Topic 4 Represent And Solve Equations And Inequalities 102
Answer:
It is given that
A company makes decorations for pens. All the supplies cost $5, and the company plans to sell the decorations for $2 apiece
Now,
Let the number of decorations be x
Let the profit be y
So,
The representation of the given situation in the form of an equation is:
Profit = $2 × (Number of decorations)
y = 2x
Now,
For x = 3,
y = 2 (3)
= 6
For x = 4,
y = 2 (4)
= 8
For x = 5,
y = 2 (5)
= 10
So,
The completed table for y = 2x is:

So,
The representation of the above table in the cordinate plane is:

Now,
The number of decorations that must be sold for the company to make a $15 profit is:
y = 2x
$15 = 2 (x)
x = \(\frac{15}{2}\)
x = 7.5
x ≅ 8
Hence, from the above,
We can conclude that
The number of decorations that must be sold for the company to make a profit of $15 should be about 8 decorations

KEY CONCEPT
You can analyze the relationship between independent and dependent variables in tables and graphs. You can relate tables and graphs to an equation.
Envision Math Common Core Grade 6 Answers Topic 4 Represent And Solve Equations And Inequalities 103

Do You Understand?
Question 1.
Essential Question How can you analyze the relationship between dependent and independent variables using tables, graphs, and equations?
Answer:
In a graph, the X-axis runs horizontally (side to side) and the Y-axis runs vertically (up and down). Typically, the independent variable will be shown on the X axis and the dependent variable will be shown on the Y axis
In a table, the independent variable will be shown in the left side or the top of the column ans the dependent variable will be shown in the right side or the bottom of the column
In an equation, the independent variable will be shown at the right side of “=” and the dependent variable will be shown at the left side of “=”

Question 2.
Reasoning Using the relationship in Example 1, how many pom poms must the booster club sell to raise $75 for charity? Explain.
Answer:
From Example 1,
We know that,
The relationship between the amount of money raised and the number of pom poms sold is:
r = 0.45n
Where,
r is the amount of money raised
n is the number of pom poms sold
Now,
For r = $75,
75 = 0.45n
n = \(\frac{75}{0.45}\)
n = 166.66
n ≅ 167
Hence, from the above,
We can conclude that
The number of pom poms must booster club sell to raise $75 for charity should be at least 167 pom poms

Question 3.
Construct Arguments For every 4 bananas a grocery store sells, it sells 2 apples. Mary wrote the equation 4b × 2 = a, where b= the number of bananas sold and a = the number of apples sold. Does Mary’s equation correctly represent the relationship of bananas sold to apples sold? Explain.
Answer:
It is given that
For every 4 bananas a grocery store sells, it sells 2 apples. Mary wrote the equation 4b × 2 = a, where b= the number of bananas sold and a = the number of apples sold
Now,
According to the given information,
The representation of the given information in the form of an equation is:
4 × (The number of bananas) = 2 × (The number of apples)
So,
4b = 2a
4b × 2 = a and 4b = 2a are not the same
Hence, from the above,
We can conclude that
Mary’s equation does not correctly represent the relationship of bananas sold to apples sold

Do You Know How?
In 4-6, use the equation d = 4t.
Envision Math Common Core Grade 6 Answers Topic 4 Represent And Solve Equations And Inequalities 104
Question 4.
Complete the table.
d = distance t = time
Answer:
The given equation is: d = 4t
Now,
For t = 1,
d = 4 (1) = 4
For t = 2,
d = 4 (2) = 8
For t = 3,
d = 4 (3) = 12
So,
The completed table for the given equation is:

Hence, from the above,
We can conclude that
The completed table for d = 4t is:

Question 5.
Name four ordered pairs found on the line plotted using this equation.
Answer:
The given table is:

Now,
The representation of the given table in the coordinate plane is:

Hence, from the above,
We can conclude that
The ordered pairs found on the line plotted using the equation d = 4t are:
(1, 4), (2, 8), (3, 12)

Question 6.
Describe the relationship between the variables.
Answer:
The given equation is: d = 4t
Now,
From the given equation,
We can observe that
t is the independent variable (or) input
d is the dependent variable (or) output
Now,
From Question 5,
We can observe that
As the value of t increases, the value of d also increases
Hence, from the above,
We can conclude that
The relationship between the variables d and t in the equation d = 4t is:
As the value of t increases, the value of d also increases

In 7, complete the table and graph to show the relationship between the variables in the equation d = 5 + 5t.
Question 7.
d= distance
t = time
Envision Math Common Core Grade 6 Answers Topic 4 Represent And Solve Equations And Inequalities 105
Answer:
The given equation is: d = 5 + 5t
Now,
For t = 0,
d = 5 + 0 = 5
For t = 2,
d = 5 + 5 (2)
= 5 + 10
= 15
For t = 4,
d = 5 + 5 (4)
= 5 + 20
= 25
So,
The completed table for the equation d = 5 + 5t is:

So,
The representation of the above table in the coordinate plane is:

Hence, from the above,
We can conclude that
The completed table and the graph to show the relationship between the variables in the equation d = 5 +5t are:

Practice & Problem Solving

In 8 and 9, complete the table and graph to show the relationship between the variables in each equation.
Question 8.
A rectangle is \(\frac{1}{2}\) inch longer than it is wide.
Let w = width.
Let l = length
Graph l = w + \(\frac{1}{2}\)
Envision Math Common Core Grade 6 Answers Topic 4 Represent And Solve Equations And Inequalities 106
Answer:
It is given that
A rectangle is \(\frac{1}{2}\) inch longer than it is wide.
Now,
The given equation is:
l = w + \(\frac{1}{2}\)
Now,
For w = 1,
l = 1 + \(\frac{1}{2}\)
= 1 + 0.5
= 1.5
For w = 2,
l = 2 + \(\frac{1}{2}\)
= 2 + 0.5
= 2.5
For w = 3,
l = 3 + \(\frac{1}{2}\)
= 3 + 0.5
= 3.5
So,
The completed table for the equation l = w + \(\frac{1}{2}\) is:

So,
The representation of the above table in the coordinate plane is:

Hence, from the above,
We can conclude that
The completed table and the graph to show the relationship between the variables in the equation l = w + \(\frac{1}{2}\) are:

Question 9.
The sale price is $5 less than the regular price.
Let s = the sale price.
Let r = the regular price.
Graph s = r – 5.
Envision Math Common Core Grade 6 Answers Topic 4 Represent And Solve Equations And Inequalities 107
Answer:
It is given that
The sale price is $5 less than the regular price.
Now,
The given equation is:
s = r – 5
Now,
For r = 10,
s = 10 – 5 = 5
For r = 20,
s = 20 – 5 = 15
For r = 30,
s = 30 – 5 = 25
So,
The completed table for the equation s = r – 5 is:

So,
The representation of the above table in the coordinate plane is:

Hence, from the above,
We can conclude that
The completed table and the graph to show the relationship between the variables in the equation l = w + \(\frac{1}{2}\) are:

Question 10.
The points (2, 4) and (-2, -4) are plotted on the coordinate plane using the equation y = a • x. How can you use the coordinates to find the value of a?
Answer:
It is given that
The points (2, 4) and (-2, -4) are plotted on the coordinate plane using the equation y = a • x
Now,
Compare the given points with (x, y)
Now,
The given equation is:
y = a . x
Now,
For (2, 4),
4 = a . 2
a = \(\frac{4}{2}\)
a = 2
Now,
For (-2, -4),
-4 = a . (-2)
4 = a . 2
a = \(\frac{4}{2}\)
a = 2
Hence, from the above,
We can conclude that
The value of a for the equation y = a . x is: 2

Question 11.
Without using a table or graph, identify three other points that a graph of the equation in Exercise 10 will pass through.
Answer:
From Question 10,
We know that,
The equation is: y = a . x
Now,
From Question 10,
We can observe that
The points that pss through y = a . x are: (2, 4), and (-2, -4)
Now,
For x = 3,
y = 2 . 3 = 6
For x = -3,
y = 2 . -3 = -6
For x = 4,
y = 2 . 4 = 8
Hence, from the above,
We can conclude that
The other three points that a graph of the equation y = a . x will pass through are: (3, 6), (-3, -6), and (4, 8)

Question 12.
Reasoning The Jackson family is planning a weekend vacation. They plan to rent a car from the ABC Car Rental Company. Let m represent the number of miles the family will drive. Let c represent the cost for renting a car. Write an equation that shows what the cost for renting a car will be.
Envision Math Common Core Grade 6 Answers Topic 4 Represent And Solve Equations And Inequalities 108
Answer:
It is given that
The Jackson family is planning a weekend vacation. They plan to rent a car from the ABC Car Rental Company. Let m represent the number of miles the family will drive. Let c represent the cost for renting a car.
Now,
The given figure is:
Envision Math Common Core Grade 6 Answers Topic 4 Represent And Solve Equations And Inequalities 108
Now,
According the given information,
The equation that represents the given situation is:
c = ($40 + $0.10)m
So,
c = $40.1m
Hence, from the above,
We can conclude that
The equation tht shows the cost for renting a car is:
c = $40.1m

In 13, write an equation. Complete the table and graph to solve the problem.
Envision Math Common Core Grade 6 Answers Topic 4 Represent And Solve Equations And Inequalities 109
Question 13.
A puppy weighs 1 pound. What does the puppy weigh after 4 weeks?
Answer:
It is given that
A puppy weighs 1 pound and the puppy gains \(\frac{1}{2}\) each week
Now,
Let w be the number of weeks
Let p be the weight of the puppy
Now,
The completed table for the given information is:

Now,
The representation of the above table in the coordinate plane is:

Hence, from the above,
We can conclude that
The completed table and the completed graph for the given problem is:

Question 14.

Model with Math During a movie matinee, the film projector broke. The theater manager refunded the ticket price to everyone attending. Let n represent the number of people watching the movie. Let r represent the total amount of money refunded. Write an equation to represent the amount of money refunded.
Envision Math Common Core Grade 6 Answers Topic 4 Represent And Solve Equations And Inequalities 110
Answer:
It is given that
During a movie matinee, the film projector broke. The theater manager refunded the ticket price to everyone attending. Let n represent the number of people watching the movie. Let r represent the total amount of money refunded.
Now,
The given figure is:
Envision Math Common Core Grade 6 Answers Topic 4 Represent And Solve Equations And Inequalities 110
Now,
The amount of money collecting by selling the movie tickets =  $5.00
Now,
The representation of the quation for the given information is:
The total amount of money refunded = $5 (The number of people watching the movie)
So,
r = 5n
Hence, from the above,
We can conclude that
The equation to represent the amount of money refunded is:
r = 5n

Question 15.
Higher Order Thinking Write an algebraic equation that matches the values shown in the table at the right. Explain how you solved the problem.
Envision Math Common Core Grade 6 Answers Topic 4 Represent And Solve Equations And Inequalities 111
Answer:
The given table is:
Envision Math Common Core Grade 6 Answers Topic 4 Represent And Solve Equations And Inequalities 111
Now,
From the given table,
We can observe that
The pattern that describes the relationship between x and y is:
y = 3x + 5
Hence, from the above,
We can conclude that
The algebraic equation that matches the values shown in the table is:
y = 3x + 5

Assessment Practice

Question 16.
For every hour Sonia worked, she had made 2 seashell necklaces for her gift shop.
Envision Math Common Core Grade 6 Answers Topic 4 Represent And Solve Equations And Inequalities 112

PART A
Write an equation that describes the relationship shown in the graph on the right.
Answer:
It is given that
For every hour Sonia worked, she had made 2 seashell necklaces for her gift shop.
Now,
The given graph that describes the given situation is:
Envision Math Common Core Grade 6 Answers Topic 4 Represent And Solve Equations And Inequalities 112
Now,
From the given graph,
The representation of the quation that represents the given situation is:
The number of seashell necklaces = 2 × (Time in hours)
So,
n = 2h
Hence, from the above,
We can conclude that
The equation that describes the relationship shown in the graph is:
n = 2h

PART B
Describe the relationship between the variables in the graph and the equation.
Answer:
From Part A,
The equation that describes the relationship shown in the graph is:
n = 2h
Where,
n is the number of seashell necklaces
h is the number of hours
Now,
In the graph,
The x-axis represents the number of hours, h
The y-axis represents the number of seashell necklaces, n

Topic 4 Review

Topic Essential Question
What procedures can be used to write and solve equations and inequalities?
Answer:
The three methods most commonly used to solve systems of equation are substitution, and elimination. Substitution and elimination are simple methods that can effectively solve most systems of two equations in a few straightforward steps
The steps to solve an inequality are:
a. Add the same number to both sides.
b. Subtract the same number from both sides.
c. Multiply both sides by the same positive number.
d. Divide both sides by the same positive number.
e. Multiply both sides by the same negative number and reverse the sign

Vocabulary

Review Complete each definition with a vocabulary word.
Vocabulary

  • dependent variable
  • independent variable
  • inequality
  • equation

Question 1.
In the equation y = x + 9, the variable x is the ___________
Answer:
We know that,
In the equation y = x + 9,
x is called the “Independent variable”
Hence, from the above,
We can conclude that
The best term that is suitable for the given sentence is: Independent variable

Question 2.
A(n). __________ has an infinite number of solutions.
Answer:
We know that,
An “Inequality” has infinite number of solutions
Hence, from the above,
We can conclude that
The best term that is suitable for the given sentence is: Inequality

Question 3.
In the equation y = x -9, the variable y is the _________
Answer:
We know that,
In the equation y = x – 9,
y is called the “Dependent variable”
Hence, from the above,
We can conclude that
The best term that is suitable for the given sentence is: Dependent variable

Draw a line from each equation to the property of equality it illustrates.
Envision Math Common Core Grade 6 Answers Topic 4 Represent And Solve Equations And Inequalities 113
Answer:
The equations that matched to the corresponding property of equality it illustrates is:

Use Vocabulary in Writing
Describe how to solve \(\frac{3}{7}\)n = 27. Use vocabulary words in your explanation.
Answer:
The given equation is:
\(\frac{3}{7}\)n = 27
Now,
From the given equation,
We can observe that
n is the variable
Now,
Multiply the given equation with \(\frac{7}{3}\) i.e., the reciprocal of \(\frac{3}{7}\) on both sides
So,
\(\frac{3}{7}\) × \(\frac{7}{3}\) × n = 27 × \(\frac{7}{3}\)
n = \(\frac{27 × 7}{3}\)
n = 9 × 7
n = 63
Hence, from the above,
We can conclude that
The value of n for the given equation is: 63

Concepts and Skills Review

Lesson 4.1 Understand Equations and Solutions

Quick Review
The solution of an equation makes the equation true. Substitute each of the given values into the equation for the variable to determine which value, if any, is a solution of the equation.

Example
Which value of x is a solution of the equation?
Envision Math Common Core Grade 6 Answers Topic 4 Represent And Solve Equations And Inequalities 114

Practice
Tell which value of the variable, if any, is a solution of the equation.
Question 1.
d + 9 = 25
d = 6, 14, 16, 21
Answer:
The given equation is:
d + 9 = 25
Now,
For d = 6,
6 + 9 = 25
15 ≠ 25
For d = 14,
14 + 9 = 25
23 ≠ 25
For d = 16,
16 + 9 = 25
25 = 25
For d = 21,
21 + 9 = 25
30 ≠ 25
Hence, from the above,
We can conclude that
d = 16 is a solution of the givene quation

Question 2.
C – 8 = 25
C = 17, 28, 33, 35
Answer:
The givene quation is:
c – 8 = 25
Now,
For c = 17,
17 – 8 = 25
9 ≠ 25
For c = 28,
28 – 8 = 25
20 ≠ 25
For c = 33,
33 – 8 = 25
25 = 25
For c = 35,
35 – 8 = 25
27 ≠ 25
Hence, from the above,
We can conclude that
c = 33 is a solution of the given equation

Question 3.
2y = 30
y = 10, 12, 24, 36
Answer:
The given equation is:
2y = 30
Now,
Now,
For y = 10,
2 (10) = 30
20 ≠ 30
For y = 12,
2 (12) = 30
24 ≠30
For y = 24,
2 (24) = 30
48 ≠ 30
For y = 36,
2 (36) = 30
72 ≠ 30
Hence, from the above,
We can conclude that
The given equation does not have any solution from the given values of y

Question 4.
150 ÷ h = 50
h = 2, 3, 4, 5
Answer:
The given equation is:
150 ÷ h = 50
Now,
For h = 2,
150 ÷ 2 = 50
75 ≠ 50
For h = 3,
150 ÷ 3 = 50
50 = 50
For h = 4,
150 ÷ 4 = 50
37.5 ≠ 50
For h = 5,
150 ÷ 5 = 50
30 ≠ 50
Hence, from the above,
We can conclude that
h = 3 is a solution of the given equation

Question 5.
f – 13.2 = 28.9
f= 38.7, 42.2, 45.8, 51.4
Answer:
The given equation is:
f – 13.2 = 28.9
Now,
For f = 38.7,
38.7 – 13.2 = 28.9
25.5 ≠ 28.9
For f = 42.2,
42.2 – 13.2 = 28.9
29 ≠ 28.9
For f = 45.8,
45.8 – 13.2 = 28.9
32.6 ≠ 28.9
For f = 51.4,
51.4 – 13.2 = 28.9
38.2 ≠ 28.9
Hence, from the above,
We can conclude that
There are solutions for the given equation from the given values of f

Lesson 4.2 Apply Properties of Equality

Quick Review
The properties of equality allow you to apply the same operation with the same amount to both sides of an equation.

Example
The properties of equality are illustrated in the table.
Envision Math Common Core Grade 6 Answers Topic 4 Represent And Solve Equations And Inequalities 115

Practice
Question 1.
If 6+2 = 8, does 6 + 2 + 3 = 8+ 3? Why or why not?
Answer:
The given equations are:
6 + 2 = 8
6 + 2 + 3 = 8 + 3
Now,
According to “Addition Property of Equality”,
Add the same amount of number on both sides to make the equation balanced
So,
From the given equation,
We can observe that 3 is added on both sides
Hence, from the above,
We can conclude that
The given equations are the same by using the Addition Property of Equality

Question 2.
If 8 – 1 = 7, does 8 – 1 – 2 = 7 – 3? Why or why not?
Answer:
The given equations are:
8 – 1 = 7
8- 1 – 2 = 7 – 3
Now,
According to “Subtraction Property of Equality”,
Subtract the same amount of number from both sides to make the equation balanced
So,
From the given equation,
We can observe that
2 is subtracted from one side and 3 is subtracted from other side
Hence, from the above,
We can conclude that
The given equations are not the same by using the Subtraction Property of Equality

Question 3.
If 4 + 6 = 10, does (4 + 6) × 3 = 10 × 3? Why or why not?
Answer:
The given equations are:
4 + 6 = 10
(4 + 6) × 3 = 10 × 3
Now,
According to “Multiplication Property of Equality”,
Multiply the same amount of number on both sides to make the equation balanced
So,
From the given equation,
We can observe that 3 is multiplied on both sides
Hence, from the above,
We can conclude that
The given equations are the same by using the Multiplication Property of Equality

Question 4.
If 5 + 4 = 9, does (5 + 4) ÷ 3 = 9 ÷ 4? Why or why not?
Answer:
The given equations are:
5 + 4 = 9
(5 + 4) ÷ 3 = 9 ÷ 4
Now,
According to “Division Property of Equality”,
Divide the same amount of number on both sides to make the equation balanced
So,
From the given equation,
We can observe that
3 is divided on one side and 4 is divided on the other side
Hence, from the above,
We can conclude that
The given equations are not the same by using the Division Property of Equality

Lessons 4.3 AND 4.4 Write and Solve Addition, Subtraction, Multiplication, and Division Equations

Quick Review
Use the inverse relationship of addition and subtraction or multiplication and division to solve equations. To check, substitute your answer back into the original equation.

Example
23 + y = 57
23 + y – 23 = 57 – 23a
y = 34

a – 12 = 16
a – 12 + 12 = 16 + 12
a = 28

9z = 63
9z ÷ 9 = 63 ÷ 9
z = 7

c ÷ 4 = 24
c ÷ 4 × 4 = 24 × 4
c = 96

Practice
Solve for x.
Question 1.
8x = 64
Answer:
The given equation is:
8x = 64
Now,
Divide by 8 on both sides
So,
\(\frac{8x}{8}\) = \(\frac{64}{8}\)
x = 8
Hence, from the above,
We can conclude that
The value of x for the given equation is: 8

Question 2.
x + 2 = 11
Answer:
The given equation is:
x + 2 = 11
Now,
Subtract 2 on both sides
So,
x + 2 – 2 = 11 – 2
x = 9
Hence, from the above,
We can conclude that
The value of x for the given equation is: 9

Question 3.
x ÷ 20 = 120
Answer:
The given equation is:
x ÷ 20 = 120
Now,
Multiply with 20 on both sides
So,
(x ÷ 120) × 20 = 120 × 20
x = 2,400
Hence, from the above,
We can conclude that
The value of x for the given equation is: 2,400

Question 4.
x – 17 = 13
Answer:
The given equation is:
x – 17 = 13
Now,
Add 17 on both sides
So,
x – 17 + 17 = 13 + 17
x = 30
Hence, from the above,
We can conclude that
The value of x for the given equation is: 30

Question 5.
x ÷ 12 = 2
Answer:
The given equation is:
x ÷ 12 = 2
Now,
Multiply with 12 on both sides
So,
(x ÷ 12) × 12 = 2 × 12
x = 24
Hence, from the above,
We can conclude that
The value of x for the given equation is: 24

Question 6.
8 + x = 25
Answer:
The given equation is:
8 + x = 25
Now,
Subtract 8 on both sides
So,
8 + x – 8 = 25 – 8
x = 17
Hence, from the above,
We can conclude that
The value of x for the given equation is: 17

Question 7.
7x = 77
Answer:
The givene quation is:
7x = 77
Now,
Divide by 7 on both sides
So,
\(\frac{7x}{7}\) = \(\frac{77}{7}\)
x = 11
Hence, from the above,
We can conclude that
The value of x for the given equation is: 11

Question 8.
x – 236 = 450
Answer:
The given equation is:
x – 236 = 450
Now,
Add 236 on both sides
So,
x – 236 + 236 = 450 + 236
x = 686
Hence, from the above,
We can conclude that
The value of x for the given equation is: 686

Question 9.
26 = 13x
Answer:
The given equation is:
26 = 13x
Now,
Divide by 13 on both sides
So,
\(\frac{26}{13}\) = \(\frac{13x}{13}\)
2 = x
x = 2
Hence, from the above,
We can conclude that
The value of x for the given equation is: 2

Question 10.
x + 21.9 = 27.1
Answer:
The given equation is:
x + 21.9 = 27.1
Now,
Subtract 21.9 on both sides
So,
x + 21.9 – 21.9 = 27.1 – 21.9
x = 5.2
Hence, from the above,
We can conclude that
The value of x for the given equation is: 5.2

Question 11.
2,448 ÷ 48 = x
Answer:
The given equation is:
2,448 ÷ 48 = x
Now,
x = 2,448 ÷ 48
x = 51
Hence, from the above,
We can conclude that
The value of x for the given equation is: 51

Question 12.
x + 15 = 31
Answer:
The given equation is:
x + 15 = 31
Now,
Subtract with 15 on both sides
So,
x + 15 – 15 = 31 – 15
x = 16
Hence, from the above,
We can conclude that
The value of x for the given equation is: 16

Lesson 4.5 Write and Solve Equations with Rational Numbers

Quick Review
You can use inverse relationships and properties of equality to solve each equation.

Example
Solve w + 4\(\frac{1}{3}\) = 7.
Subtract 4\(\frac{1}{3}\) from both sides.
w + 4\(\frac{1}{3}\) – 4\(\frac{1}{3}\) = 7 – 4\(\frac{1}{3}\)
w = 2\(\frac{2}{3}\)

Solve \(\frac{3}{5}\)n = \(\frac{2}{3}\)
Multiply both sides by the reciprocal of \(\frac{3}{5}\).
\(\frac{5}{3} \times \frac{3}{5} n=\frac{5}{3} \times \frac{2}{3}\)
\(n=\frac{10}{9} \text { or } 1 \frac{1}{9}\)

Practice
In 1-8, solve for x.
Question 1.
x + 3\(\frac{5}{8}\) = 7\(\frac{1}{4}\)
Answer:
The given equation is:
x + 3\(\frac{5}{8}\) = 7\(\frac{1}{4}\)
Now,
x = 7\(\frac{1}{4}\) – 3\(\frac{5}{8}\)
So,

Hence, from the above,
We can conclude that
The value of x for the given equation is: 3\(\frac{5}{8}\)

Question 2.
x – \(\frac{4}{8}\) = 4\(\frac{1}{4}\)
Answer:
The given equation is:
x – \(\frac{4}{8}\) = 4\(\frac{1}{4}\)
Now,
x = 4\(\frac{1}{4}\) + \(\frac{4}{8}\)
So,

Hence, from the above,
We can conclude that
The value of x for the given equation is: 4\(\frac{3}{4}\)

Question 3.
x ÷ 15 = 8\(\frac{1}{3}\)
Answer:
The given equation is:
x ÷ 15 = 8\(\frac{1}{3}\)
Now,
x = 8\(\frac{1}{3}\) × 15
So,

Hence, from the above,
We can conclude that
The value of x for the given equation is: 125

Question 4.
\(\frac{4}{2}\)x = 6
Answer:
The given equation is:
\(\frac{4}{2}\)x = 6
Now,
x = 6 × \(\frac{2}{4}\)
x = \(\frac{6 × 2}{4}\)
x = \(\frac{12}{4}\)
x = 3
Hence, from the above,
We can conclude that
The value of x for the given equation is: 3

Question 5.
\(\frac{x}{3}\) = 9
Answer:
The given equation is:
\(\frac{x}{3}\) = 9
Now,
x = 9 × 3
x = 27
Hence, from the above,
We can conclude that
The value of x for the given equation is: 27

Question 6.
14x = 73.5
Answer:
The given equation is:
14x = 73.5
Now,
x = \(\frac{73.5}{14}\)
x = 5.25
Hence, from the above,
We can conclude that
The value of x for the given equation is: 5.25

Question 7.
12x = 19.2
Answer:
The given equation is:
12x = 19.2
Now,
x = \(\frac{19.2}{12}\)
x = 1.6
Hence, from the above,
We can conclude that
The value of x for the given equation is: 1.6

Question 8.
17.9 – x = 12.8
Answer:
The given equation is:
17.9 – x = 12.8
Now,
x = 17.9 – 12.8
x = 5.1
Hence, from the above,
We can conclude that
The value of x for the given equation is: 5.1

Question 9.
Tomas buys a bag of 5 peaches for $3.55. Write and solve an equation to find how much money, m, Tomas paid for each peach.
Answer:
It is given that
Tomas buys a bag of 5 peaches for $3.55
Now,
The amount of money Tomas paid for each peach = (The amount of money Tomas paid for 5 peaches) ÷ (The number of peaches)
= \(\frac{$3.55}{5}\)
= $0.71
Hence, from the above,
We can conclude that
The amount of money Tomas paid for each peach is: $0.71

Question 10.
Krys has $1.54 and spends $0.76. Write and solve an equation to find how much money, m, Krys has left.
Answer:
It is given that
Krys has $1.54 and spends $0.76.
Now,
Let the amount of money Krys has left be x
So,
x = (The amount of money Krys has) – (The amount of money Krys spent)
x = $1.54 – $0.76
x = $0.78
Hence, from the above,
We can conclude that
The amount of money Krys has left is: $0.78

Lesson 4.6 Understand and Write Inequalities

Quick Review
An inequality is a mathematical sentence that contains < (less than), > (greater than), < (less than or equal to), 2 (greater than or equal to), or = (not equal to).

Example
Envision Math Common Core Grade 6 Answers Topic 4 Represent And Solve Equations And Inequalities 116

Practice
Write an inequality for each situation.
Question 1.
Up to 5 people, p, visited Mary today.
Answer:
The given situation is:
Up to 5 people, p, visited Mary today.
So,
The representation of the given situation in the form of an inequality is:
p ≤ 5
Hence, from the above,
We can conclude that
The representation of the given situation in the form of an inequality is:
p ≤ 5

Question 2.
The value, v, of the hat is less than $9.
Answer:
The given situation is:
The value, v, of the hat is less than $9.
So,
The representation of the given situation in the form of an inequality is:
v < $9
Hence, from the above,
We can conclude that
The representation of the given situation in the form of an inequality is:
v < $9

Question 3.
The number of guests, g, coming for dinner is not 8.
Answer:
The given situation is:
The number of guests, g, coming for dinner is not 8.
So,
The representation of the given situation in the form of an inequality is:
g ≠ 8
Hence, from the above,
We can conclude that
The representation of the given situation in the form of an inequality is:
g ≠ 8

Question 4.
The distance of the race, d, is at least 6 miles.
Answer:
The given situation is:
The distance of the race, d, is at least 6 miles.
So,
The representation of the given situation in the form of an inequality is:
d ≥ 6 miles
Hence, from the above,
We can conclude that
The representation of the given situation in the form of an inequality is:
d ≥ 6 miles

Question 5.
The time it takes to get to Grandma’s house, t, is longer than 2 hours.
Answer:
The given situation is:
The time it takes to get to Grandma’s house, t, is longer than 2 hours.
So,
The representation of the given situation in the form of an inequality is:
t > 2 hours
Hence, from the above,
We can conclude that
The representation of the given situation in the form of an inequality is:
t > 2 hours

Lesson 4.7 Solve Inequalities

Quick Review
To graph the solutions of an inequality on a number line, use an open circle for < or > and a closed circle for ≤ or ≥. If the values of the variable are less than the given number, shade to the left on the number line. If the values of the variable are greater than the given number, shade to the right on the number line.

Example
“Molly is less than 15 years old” is represented by the inequality x < 15. Write three ages that could represent Molly’s age.
To graph the inequality on a number line, draw an open circle at 15 and shade to the left of 15 because x is less than 15. Draw an arrow to show all numbers less than 15.
Envision Math Common Core Grade 6 Answers Topic 4 Represent And Solve Equations And Inequalities 117
There are many solutions. Molly could be 10, 12, 14, or any age less than 15 years.

Practice
Write the inequality that each graph represents.
Question 1.
Envision Math Common Core Grade 6 Answers Topic 4 Represent And Solve Equations And Inequalities 118
Answer:
The given number line is:
Envision Math Common Core Grade 6 Answers Topic 4 Represent And Solve Equations And Inequalities 118
Now,
Let the solutions of the given number line be x
Now,
From the given number line,
We can observe that
There is an open circle on 10 and the list of numbers are on the left side of 10
So,
The representation of the given number line in the form of an inequality is:
x < 10
Hence, from the above,
We can conclude that
The representation of the given number line in the form of an inequality is:
x < 10

Question 2.
Envision Math Common Core Grade 6 Answers Topic 4 Represent And Solve Equations And Inequalities 119
Answer:
The given number line is:
Envision Math Common Core Grade 6 Answers Topic 4 Represent And Solve Equations And Inequalities 119
Now,
Let the solutions of the given number line be x
Now,
From the given number line,
We can observe that
There is an open circle on 5 and the list of numbers are on the right side of 5
So,
The representation of the given number line in the form of an inequality is:
x > 5
Hence, from the above,
We can conclude that
The representation of the given number line in the form of an inequality is:
x > 5

Question 3.
Envision Math Common Core Grade 6 Answers Topic 4 Represent And Solve Equations And Inequalities 120
Answer:
The given number line is:
Envision Math Common Core Grade 6 Answers Topic 4 Represent And Solve Equations And Inequalities 120
Now,
Let the solutions of the given number line be x
Now,
From the given number line,
We can observe that
There is a closed circle on 3 and the list of numbers are on the right side of 3
So,
The representation of the given number line in the form of an inequality is:
x ≥ 3
Hence, from the above,
We can conclude that
The representation of the given number line in the form of an inequality is:
x ≥ 3

Question 4.
Envision Math Common Core Grade 6 Answers Topic 4 Represent And Solve Equations And Inequalities 121
Answer:
The given number line is:
Envision Math Common Core Grade 6 Answers Topic 4 Represent And Solve Equations And Inequalities 121
Now,
Let the solutions of the given number line be x
Now,
From the given number line,
We can observe that
There is a closed circle on 7 and the list of numbers are on the left side of 7
So,
The representation of the given number line in the form of an inequality is:
x ≤ 7
Hence, from the above,
We can conclude that
The representation of the given number line in the form of an inequality is:
x ≤ 7

Lesson 4.8 Understand Dependent and Independent Variables

Quick Review
Think about how the values of variables affect each other.
To identify the dependent variable, ask yourself which variable depends on the other.
To identify the independent variable, ask yourself which variable causes the change.

Example
The spirit squad is washing cars. The equation m=2c represents the money they make, m, for washing c cars. Identify the dependent variable and the independent variable.
The amount of money the spirit squad makes depends on the number of cars they wash. The dependent variable is m.
The number of cars washed changes the amount of money made. The independent variable is c.

Practice
Identify the dependent variable and the independent variable in each situation.
Question 1.
The distance traveled, d, and the speed, s
Answer:
The given situation is:
The distance traveled, d, and the speed, s
Now,
To identify the “Dependent variable” ask yourself which variable depends on the other
To identify the “Independent variable”, ask yourself which variable causes the change
So,
The independent variable for the given situation is: d
The dependent variable for the given situation is: s
Hence, from the above,
We can conclude that
The independent variable for the given situation is: d
The dependent variable for the given situation is: s

Question 2.
The calories, c, in a snack and the amount of the snack, a
Answer:
The given situation is:
The calories, c, in a snack and the amount of the snack, a
Now,
To identify the “Dependent variable” ask yourself which variable depends on the other
To identify the “Independent variable”, ask yourself which variable causes the change
So,
The independent variable for the given situation is: c
The dependent variable for the given situation is: a
Hence, from the above,
We can conclude that
The independent variable for the given situation is: c
The dependent variable for the given situation is: a

Question 3.
The amount of money you have spent, s, and how much money you have left, m
Answer:
The given situation is:
The amount of money you have spent, s, and how much money you have left, m
Now,
To identify the “Dependent variable” ask yourself which variable depends on the other
To identify the “Independent variable”, ask yourself which variable causes the change
So,
The independent variable for the given situation is: s
The dependent variable for the given situation is: m
Hence, from the above,
We can conclude that
The independent variable for the given situation is: s
The dependent variable for the given situation is: m

Question 4.
The number of apple slices remaining, r, and the number of apple slices eaten, e
Answer:
The given situation is:
The number of apple slices remaining, r, and the number of apple slices eaten, e
Now,
To identify the “Dependent variable” ask yourself which variable depends on the other
To identify the “Independent variable”, ask yourself which variable causes the change
So,
The independent variable for the given situation is: e
The dependent variable for the given situation is: r
Hence, from the above,
We can conclude that
The independent variable for the given situation is: e
The dependent variable for the given situation is: r

Lesson 4.9 Use Patterns to Write and Solve Equations

Quick Review
Look for patterns between two related variables to find rules and write equations.

Example
Write a rule and an equation that represents the pattern. Then complete the table.
Envision Math Common Core Grade 6 Answers Topic 4 Represent And Solve Equations And Inequalities 122
Find the rule and write an equation.
12 is 3 × 4
16 is 4 × 4
20 is 5 × 4
Rule: The value of y is 4 times the value of x.
Equation: y = 4x
Evaluate the equation for x = 6 and x = 7.
y = 4 × 6 = 24
y = 4 × 7 = 28

Practice
Question 1.
Find the pattern and then write a rule and an equation that represents the pattern. Then complete the table.
Envision Math Common Core Grade 6 Answers Topic 4 Represent And Solve Equations And Inequalities 123
Answer:
The given table is:
Envision Math Common Core Grade 6 Answers Topic 4 Represent And Solve Equations And Inequalities 123
Now,
From the given table,
We can observe that
The relationship (or) pattern between x and y is:
y = \(\frac{x}{2}\)
Now,
For x = 16,
y = \(\frac{16}{2}\)
y = 8
For x = 20,
y = \(\frac{20}{2}\)
y = 10
So,
The completed table by using the above relationship is:

Hence, from the above,
We can conclude that
The completed table by using the above relationship is:

Question 2.
Use the equation to complete the table.
y = 6x + 1
Envision Math Common Core Grade 6 Answers Topic 4 Represent And Solve Equations And Inequalities 124
Answer:
The given equation is:
y = 6x + 1
Now,
For x = 1,
y = 6 (1) + 1
= 7
For x = 2,
y = 6 (2) + 1
= 13
For x = 3,
y = 6 (3) + 1
= 19
For x = 4,
y = 6 (4) + 1
= 25
For x = 5,
y = 6 (5) + 1
= 31
So,
The completed table by uisng the above equation is:

Hence, from the above,
We can conclude that
The completed table by uisng the above equation is:

Lesson 4.10 Relate Tables, Graphs, and Equations

Quick Review
A table, equation, or graph can be used to analyze the relationship between dependent and independent variables. Ordered pairs that make an equation true can be used to graph the equation.

Example
Complete the table and graph to show the relationship between the variables in the equation t = s + 1.
A restaurant has a special that when you buy one sandwich you get a second sandwich for $1
Let s = price of one sandwich.
Let t = total price of two sandwiches.
Step 1
Make a table. Include at least three values.
Envision Math Common Core Grade 6 Answers Topic 4 Represent And Solve Equations And Inequalities 125

Step 2
Graph each ordered pair on a coordinate plane. Then draw a line through the points.
Envision Math Common Core Grade 6 Answers Topic 4 Represent And Solve Equations And Inequalities 126

Practice
Question 1.
The cross country team practices by jogging on the town’s streets. The average jogging rate is 6 miles per hour. One member jogged for 3.5 hours one weekend. How many miles did the team member jog?
a. Complete the table to relate the number of miles to the number of hours jogged.
Envision Math Common Core Grade 6 Answers Topic 4 Represent And Solve Equations And Inequalities 127
Answer:
It is given that
The cross country team practices by jogging on the town’s streets. The average jogging rate is 6 miles per hour. One member jogged for 3.5 hours one weekend.
So,
From the given situation,
We can observe that
The independent variable is: The number of hours the team member jogged
The dependent variable is: Number of miles the team member jogged
Now,
Let the dependent variable be y
Let the independent variable be y
So,
The relation between x and y from the given situation is:
y = 6x
Now,
The completed table by using the above equation is:

Hence, from the above,
We can conclude that
The completed table by using the above equation is:

b. Graph the ordered pairs on the coordinate plane.
Envision Math Common Core Grade 6 Answers Topic 4 Represent And Solve Equations And Inequalities 128
Answer:
The given table is:

Now,
From the given table,
We can observe that
The ordered pairs to draw in the coordinate plane is:
(1, 6), (2, 12), (3, 18)
Hence,
The representtaion of the given ordered pairs in the coordinate plane is:

c. Write an equation that describes the relationship. Then solve the problem.
Answer:
From part (a),
We know that,
The equation that describes the given situation is:
y = 6x
So,
The number of miles did the team member jog is:
y = 6x
= 6 (3.5)
= 21 miles
Hence, from the above,
We can conclude that
The number of miles did the team member jog is: 21 miles

Question 2.
Alex is making puppets for a show. He bought all the string needed for $125. It costs $18 for the remaining materials to make each puppet. What is the total cost to make 50 puppets?
Answer:
It is given that
Alex is making puppets for a show. He bought all the string needed for $125. It costs $18 for the remaining materials to make each puppet
So,
The total cost of the materials to make each puppet = $125 + $18
= $143
So,
The total cost to make 50 puppets = 50 × $143
= $7,150
Hence, from the above,
We can conclude that
The total cost to make 50 puppets = 50 × $143
= $7,150

Topic 4 Fluency Practice

Riddle Rearranging
Find each quotient. Then arrange the answers in order from least to greatest. The letters will spell out the answer to the riddle below.
I can… divide multi-digit numbers.
Envision Math Common Core Grade 6 Answers Topic 4 Represent And Solve Equations And Inequalities 129

enVision Math Common Core Grade 6 Answer Key Topic 1 Use Positive Rational Numbers

Practice with the help of enVision Math Common Core Grade 6 Answer Key Topic 1 Use Positive Rational Numbers regularly and improve your accuracy in solving questions.

enVision Math Common Core 6th Grade Answers Key Topic 1 Use Positive, Rational Numbers

Topic Essential Question How can you fluently add, subtract, multiply, and divide decimals? How can you multiply and divide fractions?
Answer:
The steps to add, subtract, multiply, and divide decimals are:
a. Write the numbers vertically with the decimals lined up (if the number is a whole number then the decimal goes on the right end of the number)
b. Add zeros to make sure there is the same number of digits in each number.
c. Add or subtract as normal.
d. Multiply just as you would if the numbers were all whole numbers and add at the end.
e. After that count how many decimals places the two factors have (start at the far right of each number) and then give the answer that the total number of decimal places.
f. To divide decimals, always use long division.
The process to multiply and divide fractions is:
a. Dividing two fractions is the same as multiplying the first fraction by the reciprocal of the second fraction.
b. The first step to dividing fractions is to find the reciprocal (reverse the numerator and denominator) of the second fraction.
c. Next, multiply the two numerators. Then, multiply the two denominators.

enVision STEM Project

VIDEO

Did You Know?

Engineers design equipment to make you safer.

Engineering is the application of math and science to solve problems.

Engineers solve problems by designing and building products, materials, machinery, structures, transportation vehicles, and so many other things.
Envision Math Common Core Grade 6 Answer Key Topic 1 Use Positive Rational Numbers 1
Engineers work in nearly every area from chemical and electrical engineering to biomedical and oceanographic engineering.
Envision Math Common Core Grade 6 Answer Key Topic 1 Use Positive Rational Numbers 1.1
Engineers help keep you healthy.
Envision Math Common Core Grade 6 Answer Key Topic 1 Use Positive Rational Numbers 1.2
Engineers find ways to improve and enhance performance of all kinds of products.
Envision Math Common Core Grade 6 Answer Key Topic 1 Use Positive Rational Numbers 1.3

Your Task: Improve Your School

Think like an engineer! Take a walk around the inside and the outside of your school building. Make a list of specific things or areas that need improvement. Then choose one idea and do some background research to gain an understanding of factors that might impact improvement efforts. In the next topic, you and your classmates will learn about and implement the engineering design process to propose possible ways to make the improvements.
Envision Math Common Core Grade 6 Answer Key Topic 1 Use Positive Rational Numbers 1.4

Topic 1 GET READY!

Review What You Know!

Vocabulary
Choose the best term from the box to complete each definition.

compatible numbers
decimal
divisor
estimate
quotient

Question 1.
Numbers that are easy to compute mentally are ____
Answer:
We know that,
Numbers that are easy to compute mentally are called “Compatible numbers”
Hence, from the above,
We can conclude that the best term to complete the given definition is: Compatible numbers

Question 2.
The number used to be divided is the _____
Answer:
We know that,
The number used to be divided is the “Divisor”
Hence, from the above,
We can conclude that the best term to complete the given definition is: Divisor

Question 3.
A(n) _____ is an approximate answer.
Answer:
We know that,
An “Estimate” is an approximate answer
Hence, from the above,
We can conclude that the best term to complete the given definition is: Estimate

Question 4.
The result of a division problem is a(n) ____
Answer:
We know that,
The result of a division problem is a “Quotient”
Hence, from the above,
We can conclude that the best term to complete the given definition is: Quotient

Whole Number Operations
Calculate each value.

Question 5.
Envision Math Common Core Grade 6 Answer Key Topic 1 Use Positive Rational Numbers 1.6
Answer:
The given division problem is: 348 ÷ 4
Now,
By using the Long Division method,

Hence, from the above,
We can conclude that
The value of the quotient for the given division problem is: 87

Question 6.
9,007 – 3,128
Answer:
The given subtraction problem is: 9,007 – 3,128
So,
9,007 – 3,128 = 5,879
Hence, from the above,
We can conclude that
The result for the given subtraction problem is: 5,879

Question 7.
35 × 17
Answer:
The given multiplication problem is: 35 × 17
Now,
By using the Partial Products method,
35 × 17
= (30 + 5) × (10 + 7)
= (30 × 10) + (30 × 7) + (5 × 10) + (5 × 7)
= 300 + 210 + 50 + 35
= 595
Hence, from the above,
We can conclude that
The value of product for the given multiplication problem is: 595

Question 8.
7,964 + 3,872
Answer:
The given addition problem is: 7,964 + 3,872
So,
7,964 + 3,872 = 11,836
Hence, from the above,
We can conclude that
The result for the given addition problem is: 11,836

Question 9.
Envision Math Common Core Grade 6 Answer Key Topic 1 Use Positive Rational Numbers 1.7
Answer:
The given division problem is: 4,638 ÷ 22
Now,
By using the Long Division method,

Hence, from the above,
We can conclude that
The value of quotient for the given division problem is: 210

Question 10.
181 × 42
Answer:
The given multiplication problem is: 181 × 42
Now,
By using the Partial Products method,
181 × 42
= (100 + 80 + 1) × (40 + 2)
= (100 ×40) + (100 × 2) + (80 × 40) + (80 × 2) + (1 × 40) + (1 × 2)
= 4,000 + 200 + 3,200 + 160 + 40 + 2
= 7,602
Hence, from the above,
We can conclude that
The value of the product for the given multiplication problem is: 7,602

Mixed Numbers and Fractions
Write each mixed number as a fraction. Write each fraction as a mixed number.

Question 11.
8\(\frac{1}{3}\)
Answer:
The given mixed number is: 8\(\frac{1}{3}\)
So,
To convert the given mixed number into a fraction,
8\(\frac{1}{3}\)
= \(\frac{3 × 8 + 1}{3}\)
= \(\frac{24 + 1}{3}\)
= \(\frac{25}{3}\)
Hence, from the above,
We can conclude that
The representation of the given mixed number into a fraction is: \(\frac{25}{3}\)

Question 12.
5\(\frac{3}{5}\)
Answer:
The given mixed number is: 5\(\frac{3}{5}\)
So,
To convert the given mixed number into a fraction,
5\(\frac{3}{5}\)
= \(\frac{5 × 5 + 3}{5}\)
= \(\frac{25 + 3}{5}\)
= \(\frac{28}{5}\)
Hence, from the above,
We can conclude that
The representation of the given mixed number into a fraction is: \(\frac{28}{5}\)

Question 13.
2\(\frac{5}{8}\)
Answer:
The given mixed number is: 2\(\frac{5}{8}\)
So,
To convert the given mixed number into a fraction,
2\(\frac{5}{8}\)
= \(\frac{2 × 8 + 5}{8}\)
= \(\frac{16 + 5}{8}\)
= \(\frac{21}{8}\)
Hence, from the above,
We can conclude that
The representation of the given mixed number into a fraction is: \(\frac{21}{8}\)

Question 14.
3\(\frac{4}{9}\)
Answer:
The given mixed number is: 3\(\frac{4}{9}\)
So,
To convert the given mixed number into a fraction,
3\(\frac{4}{9}\)
= \(\frac{3 × 9 + 4}{9}\)
= \(\frac{27 + 4}{9}\)
= \(\frac{31}{9}\)
Hence, from the above,
We can conclude that
The representation of the given mixed number into a fraction is: \(\frac{31}{9}\)

Question 15.
\(\frac{24}{7}\)
Answer:
The given fraction is: \(\frac{24}{7}\)
So,
To convert the given fraction into a mixed number,
Quotient\(\frac{Remainder}{Divisor}\)
So,
\(\frac{24}{7}\) = 3\(\frac{3}{7}\)
Hence, from the above,
We can conclude that
The representation of the given fraction into a mixed number is: 3\(\frac{3}{7}\)

Question 16.
\(\frac{43}{9}\)
Answer:
The given fraction is: \(\frac{43}{9}\)
So,
To convert the given fraction into a mixed number,
Quotient\(\frac{Remainder}{Divisor}\)
So,
\(\frac{43}{9}\) = 4\(\frac{7}{9}\)
Hence, from the above,
We can conclude that
The representation of the given fraction into a mixed number is: 4\(\frac{7}{9}\)

Question 17.
\(\frac{59}{8}\)
Answer:
The given fraction is: \(\frac{59}{8}\)
So,
To convert the given fraction into a mixed number,
Quotient\(\frac{Remainder}{Divisor}\)
So,
\(\frac{59}{8}\) = 7\(\frac{3}{8}\)
Hence, from the above,
We can conclude that
The representation of the given fraction into a mixed number is: 7\(\frac{3}{8}\)

Question 18.
\(\frac{32}{5}\)
Answer:
The given fraction is: \(\frac{32}{5}\)
So,
To convert the given fraction into a mixed number,
Quotient\(\frac{Remainder}{Divisor}\)
So,
\(\frac{32}{5}\) = 6\(\frac{2}{5}\)
Hence, from the above,
We can conclude that
The representation of the given fraction into a mixed number is: 6\(\frac{2}{5}\)

Verbal Expressions

Question 19.
How are the expressions “\(\frac{1}{4}\) of 12″ and “12 divided by 4” related?
Answer:
The given expressions are: \(\frac{1}{4}\) × 12 and 12 ÷ 4
So,
We can write
12 ÷ 4
= \(\frac{12}{4}\)
= 12 × \(\frac{1}{4}\)
Hence,f rom the above,
We can conclude that the given expressions 12 × \(\frac{1}{4}\) and 12 ÷ 4 are the same

Decimals

Question 20.
What decimal does this model represent? Explain.
Envision Math Common Core Grade 6 Answer Key Topic 1 Use Positive Rational Numbers 2.1
Answer:
The given model is:
Envision Math Common Core Grade 6 Answer Key Topic 1 Use Positive Rational Numbers 2.1
Now,
From the given model,
We can observe that
The total number of parts are: 100
The number of shaded parts is: 73
So,
The fraction of the shaded part in the given model = \(\frac{Shaded parts}{Total parts}\)
= \(\frac{73}{100}\)
= 0.73
Hence, from the above,
We can conclude that
The decimal number that the given model represents is: 0.73

Language Development

Fill in the boxes with terms and phrases related to Decimals and Fractions from the given bank. Include illustrations or examples.

annex zeroes (1-1)
compatible numbers (1-1)
decimal point (1-1)
estimate (1-1)
hundredths (1-1)
line up place value (1-1)
rounding (1-1)
tenths (1-1)
algorithm (1-2)
the denominator (1-3)
mixed number (1-3)
the numerator (1-3)
unit fraction (1-3)
reciprocal (1-4)
rewrite (1-4)
multistep problems (1-7)

Envision Math Common Core Grade 6 Answer Key Topic 1 Use Positive Rational Numbers 2.5

Topic 1 PICK A PROJECT

PROJECT 1A
What is the most challenging board game you have ever played?
PROJECT: MAKE YOUR OWN BOARD GAME
Envision Math Common Core Grade 6 Answer Key Topic 1 Use Positive Rational Numbers 2.8

PROJECT 1B
What is your favorite party food?
PROJECT: PLAN THE MENU FOR A SCHOOL FUNDRAISER
Envision Math Common Core Grade 6 Answer Key Topic 1 Use Positive Rational Numbers 2.9

PROJECT 1C
If you planted a garden, what would be in it?
PROJECT: DESIGN A VEGETABLE AND HERB GARDEN
Envision Math Common Core Grade 6 Answer Key Topic 1 Use Positive Rational Numbers 2.10

PROJECT 10D
How much food does a tiger eat?
PROJECT: PRESENT A PROPOSAL FOR A TIGER EXHIBIT
Envision Math Common Core Grade 6 Answer Key Topic 1 Use Positive Rational Numbers 2.11

Lesson 1.1 Fluently Add, Subtract and Multiply Decimals

ACTIVITY

Solve & Discuss It!

Maxine is making a model windmill for a science fair. She is connecting 4 cardboard tubes together vertically. Each tube is 0.28 meters in length. What is the combined measure of the connected tubes?

Use Appropriate Tools You can use decimal grids to calculate with decimals.
Envision Math Common Core Grade 6 Answer Key Topic 1 Use Positive Rational Numbers 2.12
Envision Math Common Core Grade 6 Answer Key Topic 1 Use Positive Rational Numbers 2.13
Answer:
It is given that
Maxine is making a model windmill for a science fair. She is connecting 4 cardboard tubes together vertically. Each tube is 0.28 meters in length
So,
The combined measure of the connected tubes = (The number of cardboard tubes) × (The length of each tube)
= 4 × 0.28
= 4 × (0.20 + 0.08)
= (4 × 0.20) + (4 × 0.08)
= 0.8 + 0.32
= 1.12 meters
So,
The representation of the length of 4 cardboard tubes in the decimal grid is:

Hence, from the above,
We can conclude that
The combined measure of the 4 connected tubes is: 1.12 meters

Focus on math practices Look for Relationships Suppose that Maxine made another windmill model by connecting 4 cardboard tubes that are each 2.8 meters long. What is the combined measure of this model? What relationships do you see in the factors you used here and above? Explain how this helps you solve the problem.
Answer:
It is given that
Maxine made another windmill model by connecting 4 cardboard tubes that are each 2.8 meters long
So,
The combined measure of another model made by Maxine = (The number of cardboard tubes of another model) × (The length of each tube)
= 4 × 2.8
= 4 × (2 + 0.8)
= (4 × 2) + (4 × 0.8)
= 8 + 3.2
= 11.2 meters
Now,
From the above problem,
We can observe that
The combined measure of 4 connected tubes is: 1.12 meters
So,
When we multiply 1.12 meters by 10,
We will get the combined measure of the other model made by Maxine
Hence, from the above,
We can conclude that
The relationship do you see in the factors you used in this problem and the above problem is:
(The combined measure of 4 connected tubes of the above problem) × 10 = (The combined measure of the 4 connected tubes of this problem)

? Essential Question How can you add, subtract, and multiply with decimals?
Answer:
The steps to add, subtract, multiply, and divide decimals are:
a. Write the numbers vertically with the decimals lined up (if the number is a whole number then the decimal goes on the right end of the number)
b. Add zeros to make sure there is the same number of digits in each number.
c. Add or subtract as normal.
d. Multiply just as you would if the numbers were all whole numbers and add at the end.
e. After that count how many decimals places the two factors have (start at the far right of each number) and then give the answer that the total number of decimal places.
f. To divide decimals, always use long division.

Try It!
Suppose that Martin finished the race 0.47 seconds after Kim. What was Martin’s time in the race? Use an estimate to check that your answer is reasonable.
Answer:
It is given that
Martin finished the race 0.47 seconds after Kim
Now,
We know that,
The time Kim took to complete the race is: 50.9 seconds
So,
The time took by Martin in the race = (The time took by Kim to complete the race) + (The more time taken by Martin to finish the race more than Kim)
= 50.9 + 0.47
= 50.90 + 0.47
= 51.37 seconds
Now,
Estimate:
The time took by Kim to complete the race is: 51 seconds
The more time taken by Martin to finish the race more than Kim is: 0.50 seconds
So,
The time took by Martin to complete the race = (The time took by Kim in the race) + (The more time taken by Martin to finish the race more than Kim)
= 51 + 0.50
= 51.50 seconds
Hence, from the above,
We can conclude that
The time took by Martin in the race is: 51.37 seconds
Since the estimated time and the exact time is near, your answer is reasonable

Convince Me! If Martin finished the race 0.267 seconds after Kim, you would need to add 0.267 to 50.9 to solve the problem. How is adding 0.267 to 50.9 different from adding 0.26 to 50.9?
Answer:
It is given that
If Martin finished the race 0.267 seconds after Kim, you would need to add 0.267 to 50.9 to solve the problem
So,
The time took by Martin in the race = (The time took by Kim in the race) + (The more time taken by Martin to finish the race more than Kim)
= 50.9 + 0.267
= 50.900 + 0.267
= 51.167 seconds ——(1)
So,
The time took by Martin in the race = (The time took by Kim in the race) + (The more time taken by Martin to finish the race more than Kim)
= 50.9 + 0.26
= 50.90 + 0.26
= 51.16 seconds —— (2)
So,
When we compare eq (1) and eq (2),
We can observe that
eq (1) gives the more accurate time when compared to eq (2)
Hence, from the above,
We can conclude that
The difference between the results of eq(1) and eq (2) is the accuracy and precision

Try It!
Suppose that Katie finished the race 0.13 seconds before Amy. What was Katie’s time in the race? Use an estimate to check that your answer is reasonable.
Answer:
It is given that
Katie finished the race 0.13 seconds before Amy
Now,
We know that,
The time Amy took to complete the race is: 20.7 seconds
So,
The time took by Katie in the race = (The time took by Amy to complete the race) – (The time taken by Katie to finish the race before Amy)
= 20.7 – 0.13
= 20.70 – 0.13
= 20.57 seconds
Now,
Estimate:
The time took by Amy to complete the race is: 21 seconds
The time is taken by Katie to finish the race before Amy is: 0.10 seconds
So,
The time took by Katie to complete the race = (The time took by Amy in the race) + (The time taken by Katie to finish the race before Amy)
= 21 – 0.10
= 21.00 – 0.10
= 20.90 seconds
Hence, from the above,
We can conclude that
The time took by Martin in the race is: 20.57 seconds
Since the estimated time and the exact time is near, your answer is reasonable

Try It!
How do you determine where to place the decimal point in the product?
Envision Math Common Core Grade 6 Answer Key Topic 1 Use Positive Rational Numbers 3.1
Answer:
We know that,
Decimals are multiplied as if they were whole numbers, and then the decimal point is placed in the product. To find out where the decimal point should be placed, count the number of decimal places after the decimal point in each factor.
Hence,
The representation of the decimal point in the given product is:

KEY CONCEPT
Envision Math Common Core Grade 6 Answer Key Topic 1 Use Positive Rational Numbers 3.2

Do You Understand?

Question 1.
? Essential Question How can you add, subtract, and multiply with decimals?
Answer:
The steps to add, subtract, multiply, and divide decimals are:
a. Write the numbers vertically with the decimals lined up (if the number is a whole number then the decimal goes on the right end of the number)
b. Add zeros to make sure there is the same number of digits in each number.
c. Add or subtract as normal.
d. Multiply just as you would if the numbers were all whole numbers and add at the end.
e. After that count how many decimals places the two factors have (start at the far right of each number) and then give the answer that the total number of decimal places.
f. To divide decimals, always use long division.

Question 2.
Generalize How is adding and subtracting decimals similar to and different from adding and subtracting whole numbers?
Answer:
We know that,
Adding and subtracting decimal numbers, numbers with a decimal point is just like adding and subtracting whole numbers.
The only difference is that we line up the numbers according to the decimal point.
For subtraction, the number we are subtracting goes to the bottom.

Question 3.
What can you do if a decimal product has final zeros to the right of the decimal point?
Answer:
We can drop the zeros on the right once the decimal point has been placed in the product. If the number of decimal places is greater than the number of digits in the product, you can insert zeros in front of the product.

Question 4.
Critique Reasoning Diego says that the product of 0.51 × 2.427 will have five decimal places. Is Diego correct? Explain.
Answer:
It is given that
Diego says that the product of 0.51 × 2.427 will have five decimal places
Now,
We know that,
Decimals are multiplied as if they were whole numbers, and then the decimal point is placed in the product. To find out where the decimal point should be placed, count the number of decimal places after the decimal point in each factor.
So,
In
0.51 × 2.427,
We can observe that
There are 2 decimal places in 0.51
There are 3 decimal places in 2.427
So,
In 0.51 × 2,427,
There are 5 decimal places
Hence, from the above,
We can conclude that Diego is correct

Do You Know How?

In 5-10, find each sum or difference.

Question 5.
5.9 + 2.7
Answer:
The given addition problem is: 5.9 + 2.7
So,
By using the Long Addition,

Hence, from the above,
We can conclude that the value of the given addition problem is: 8.6

Question 6.
4.01 – 2.95
Answer:
The given subtraction problem is: 4.01 – 2.95
So,
By using the Long Subtraction,

Hence, from the above,
We can conclude that the value of the given subtraction problem is: 1.06

Question 7.
6.8 – 1.45
Answer:
The given subtraction problem is: 6.8 – 1.45
So,
By using the Long Subtraction,

Hence, from the above,
We can conclude that the value of the given subtraction problem is: 5.35

Question 8.
9.62 – 0.3
Answer:
The given subtraction problem is: 9.62 – 0.3
So,
By using the Long Subtraction,

Hence, from the above,
We can conclude that the value of the given subtraction problem is: 9.32

Question 9.
2.57 + 7.706
Answer:
The given addition problem is: 2.57 + 7.706
So,
By using the Long Addition,

Hence, from the above,
We can conclude that the value of the given addition problem is: 10.276

Question 10.
15 – 6.108
Answer:
The given subtraction problem is: 15 – 6.108
So,
By using the Long Subtraction,

Hence, from the above,
We can conclude that the value of the given subtraction problem is: 8.892

In 11-16, place the decimal point in the correct location in the product.

Question 11.
4 × 0.94 = 376
Answer:

Hence, from the above,
We can conclude that
4 × 0.94 = 3.76

Question 12.
5 × 0.487 = 2435
Answer:

Hence, from the above,
We can conclude that
5 × 0.487 = 2.435

Question 13.
3.4 × 6.8 = 2312
Answer:

Hence, from the above,
We can conclude that
3.4 × 6.8 = 23.12

Question 14.
3.9 × 0.08 = 312
Answer:

Hence, from the above,
We can conclude that
3.9 × 0.08 = 0.312

Question 15.
0.9 × 0.22 = 198
Answer:

Hence, from the above,
We can conclude that
0.9 × 0.22 = 0.198

Question 16.
9 × 1.2 = 108
Answer:

Hence, from the above,
We can conclude that
9 × 1.2 = 10.8

In 17 and 18, find each product.

Question 17.
5.3 × 2.7
Answer:
The given multiplication equation is: 5.3 × 2.7
So,
By using the Long Multiplication method,

Hence, from the above,
We can conclude that
The value of the product for the given multiplication equation is: 14.31

Question 18.
8 × 4.09
Answer:
The given multiplication is: 8 × 4.09
So,
By using the Long Multiplication,

Hence, from the above,
We can conclude that
The value of the product for the given multiplication equation is: 32.72

Practice & Problem Solving

Scan for Multimedia

In 19-27, find each sum or difference.

Question 19.
2.17 – 0.8
Answer:
The given subtraction equation is: 2.17 – 0.8
So,
By using the Long subtraction,

Hence, from the above,
We can conclude that
2.17 – 0.8 = 1.37

Question 20.
4.3 + 4.16
Answer:
The given addition equation is: 4.3 + 4.16
So,
By using the Long Addition,

Hence, from the above,
We can conclude that
4.3 + 4.16 = 8.46

Question 21.
46.91 – 28.7
Answer:
The given subtraction equation is: 46.91 – 28.7
So,
By using the Long subtraction,

Hence, from the above,
We can conclude that
46.91 – 28.7 = 18.21

Question 22.
4.815 + 2.17
Answer:
The given addition equation is: 4.815 + 2.17
So,
By using the Long Addition,

Hence, from the above,
We can conclude that
4.815 + 2.17 = 6.985

Question 23.
5.1 – 0.48
Answer:
The given subtraction equation is: 5.1 – 0.48
So,
By using the Long subtraction,

Hence, from the above,
We can conclude that
5.1 – 0.48 = 4.62

Question 24.
27 + 0.185
Answer:
The given addition equation is: 27 + 0.185
So,
By using the Long Addition,

Hence, from the above,
We can conclude that
27 + 0.185 = 27.185

Question 25.
9.501 – 9.45
Answer:
The given subtraction equation is: 9.501 – 9.45
So,
By using the Long subtraction,

Hence, from the above,
We can conclude that
9.501 – 9.45 = 0.051

Question 26.
14 + 9.8
Answer:
The given addition equation is: 14 + 9.8
So,
By using the Long Addition,

Hence, from the above,
We can conclude that
14 + 9.8 = 23.8

Question 27.
12.65 + 14.24
Answer:
The given addition equation is: 12.65 + 14.24
So,
By using the Long Addition,

Hence, from the above,
We can conclude that
12.65 + 14.24 = 26.89

In 28-33, find each product.

Question 28.
7 × 0.5
Answer:
The given multiplication equation is: 7 × 0.5
So,
By using the Long Multiplication,

Hence, from the above,
We can conclude that
7 × 0.5 = 3.5

Question 29.
12 × 0.08
Answer:
The given multiplication equation is: 12 × 0.08
So,
By using the Long Multiplication,

Hence, from the above,
We can conclude that
12 × 0.08 = 0.96

Question 30.
24 × 0.17
Answer:
The given multiplication equation is: 24 × 0.17
So,
By using the Long Multiplication,

Hence, from the above,
We can conclude that
24 × 0.17 = 4.08

Question 31.
0.4 × 0.17
Answer:
The given multiplication equation is: 0.4 × 0.17
So,
By using the Long Multiplication,

Hence, from the above,
We can conclude that
0.4 × 0.17 = 0.068

Question 32.
1.9 × 0.46
Answer:
The given multiplication is: 1.9 × 0.46
So,
By using the Long Multiplication,

Hence, from the above,
We can conclude that
1.9 × 0.46 = 0.874

Question 33.
3.42 × 5.15
Answer:
The given multiplication is: 3.42 × 5.15
So,
By using the Long Multiplication,

Hence, from the above,
We can conclude that
3.42 × 5.15 = 17.6130

Question 34.
Write an equation that illustrates the following: A number with two decimal places multiplied by a number with one decimal place. The product has only two nonzero digits.
Answer:
It is given that
A number with two decimal places is multiplied by a number with one decimal place. The product has only two nonzero digits.
Now,
Let the number with two decimal places be: 2.55
Let the number with one decimal be: 1.6
So,
We have to find the product of 2.55 × 1.6
Now,
By using the Long Multiplication,

Hence, from the above,
We can conclude that
The multiplication equation with the product value that has only two non-zero digits is: 2.55 × 1.6

Question 35.
The Bright-o Shampoo Factory includes 1.078 ounces of vanilla oil in a 6.35-ounce bottle of shampoo. How much of the bottle of shampoo is NOT vanilla oil?
Answer:
It is given that
The Bright-o Shampoo Factory includes 1.078 ounces of vanilla oil in a 6.35-ounce bottle of shampoo.
So,
The total content present in Bright-o-Shampoo = (The amount of vanilla oil) + (The amount of other content)
Now,
Let the amount of other content be: x
So,
6.35 = 1.078 + x
So,
x = 6.35 – 1.078
Now,
By using the Long Subtraction,

Hence, from the above,
We can conclude that
The amount of the bottle of a shampoo that is not vanilla oil is: 5.272 ounces

In 36-38, use the graph to solve.

Envision Math Common Core Grade 6 Answer Key Topic 1 Use Positive Rational Numbers 5.1

Question 36.
The fastest speed a table tennis ball has been hit is about 13.07 times as fast as the speed for the fastest swimming. What is the speed of the table tennis ball?
Answer:
It is given that
The fastest speed a table tennis ball has been hit is about 13.07 times as fast as the speed for the fastest swimming.
Now,
The given graph is:
Envision Math Common Core Grade 6 Answer Key Topic 1 Use Positive Rational Numbers 5.1
Now,
The speed of the table tennis ball = 13.07 × (The speed for the fastest swimming)
= 13.07 × 5.35
Now,
By using the Long Multiplication,

Hence, from the above,
We can conclude that the speed of the tennis ball is about 70 miles per hour

Question 37.
Look for Relationships How fast would 1.5 times the fastest rowing speed be? Before you solve, tell the number of decimal places in your answer.
Answer:
The given graph is:
Envision Math Common Core Grade 6 Answer Key Topic 1 Use Positive Rational Numbers 5.1
Now,
From the given graph,
We can observe that
The fastest rowing speed is: 13.99 miles per hour
So,
The speed of 1.5 times the fastest rowing speed = 1.5 × 13.99
Now,
By using the Long Multiplication,

Hence, from the above,
We can conclude that
1.5 times of the fastest rowing speed is: 209.85 miles per hour

Question 38.
Which activity has a recorded speed about 7 times as fast as the fastest rowing speed?
Answer:
The given graph is:
Envision Math Common Core Grade 6 Answer Key Topic 1 Use Positive Rational Numbers 5.1
Now,
From the given graph,
We can observe that
The fastest rowing speed is: 13.99 miles per hour
Now,
7 times of the fastest rowing speed is: 7 × 13.99
Now,
By using the Long Multiplication,

So,
The fastest rowing speed is about: 98 miles per hour
Now,
From the given graph,
We can observe that
The fastest Luge is: 95.69 miles per hour
Hence, from the above,
We can conclude that
“Fastest Luge” has a recorded speed about 7 times as fast as the fastest rowing speed

Question 39.
Matthew a pennant, and a hat. He paid with a $50 bill and some money he borrowed from his friend. If Matthew got $6.01 in change from the cashier, how much did he borrow from his friend to pay for all the items?
Envision Math Common Core Grade 6 Answer Key Topic 1 Use Positive Rational Numbers 6.1
Answer:
It is given that
Matthew a pennant, and a hat. He paid with a $50 bill and some money he borrowed from his friend. If Matthew got $6.01 in change from the cashier
Now,
Let the amount Matthew borrowed from his friend be: x
So,
According to the given information,
The amount Matthew borrowed from his friend = (The amount of bill) – (The amount Matthew got in change from the cashier)
x = $50 – $6.01
Now,
By using the Long Subtraction,

Hence, from the above,
We can conclude that
The amount Matthew borrowed from his friend is: $43.99

Question 40.
Anna’s running time for a race was 23.1 seconds. Another runner’s time was 5.86 seconds faster. Find the other runner’s time.
Answer:
It is given that
Anna’s running time for a race was 23.1 seconds. Another runner’s time was 5.86 seconds faster
So,
The time took by another runner = 5.86 + (Anna’s running time)
= 5.86 + 23.1
Now,
By using the Long Addition,

Hence, from the above,
We can conclude that
The time took by another runner is: 28.96 seconds

Question 41.
Higher-Order Thinking Explain why 0.25 × 0.4 has only one decimal place in the product.
Answer:
The given multiplication equation is: 0.25 × 0.4
Now,
By using the Long Multiplication,

Now,
We know that,
In the decimal numbers,
If there are zeros on the right side, then we can neglect those zeros
So,
0.100 can become 0.1
Hence, from the above,
We can conclude that
0.25 × 0.4 has only one decimal place in the product because of the following property:
In the decimal numbers,
If there are zeros on the right side, then we can neglect those zeros

Question 42.
The wings of some hummingbirds beat 52 times per second when hovering. If a hummingbird hovers for 35.5 seconds, how many times do its wings beat?
Answer:
It is given that
The wings of some hummingbirds beat 52 times per second when hovering and a hummingbird hovers for 35.5 seconds
So,
The number of times the wings of a hummingbird beat in 35.5 seconds = (The time a hummingbird hovers) × (The number of times the wings of a hummingbird beat)
= 35.5 × 52
Now,
By using the Long Multiplication,

Hence, from the above,
We can conclude that
The number of times the wings of a hummingbird beat in 35.5 seconds is: 1,846 times

Question 43.
The students at Walden Middle School are selling tins of popcorn to raise money for new uniforms. They sold 42 tins in the first week. How much money did they make in the first week?
Envision Math Common Core Grade 6 Answer Key Topic 1 Use Positive Rational Numbers 6.10
Answer:
It is given that
The students at Walden Middle School are selling tins of popcorn to raise money for new uniforms. They sold 42 tins in the first week
Now,
The given figure is:

So,
The money did the students at Walden Middle School made in the first week = (The number of tins) × (The cost of each tin)
= 42 × $9.25
Now,
By using the Long Multiplication,

Hence, from the above,
We can conclude that
The money did the students at Walden Middle School made in the first week is: $388.50

Assessment Practice

Question 44.
Use the information in the table to solve each problem.
Envision Math Common Core Grade 6 Answer Key Topic 1 Use Positive Rational Numbers 6.3
PART A
What is the combined length in kilometers of the Bayshore Loop trail and the Rowdy Bend trail?
Answer:
The given information is:
Envision Math Common Core Grade 6 Answer Key Topic 1 Use Positive Rational Numbers 6.3
So,
The combined length of the Bayshore Loop trail and the Rowdy Bend trail = 3.2 + 4.2
Now,
By using the Long Addition,

Hence, from the above,
We can conclude that
The combined length of the Bayshore Loop trail and the Rowdy Bend trail is: 7.4 Kilometers

PART B
How many kilometers longer is the Coastal Prairie trail than the Snake Bight trail?
Answer:
The given information is:
Envision Math Common Core Grade 6 Answer Key Topic 1 Use Positive Rational Numbers 6.3
So,
The difference between the length of the Coastal Prairie trail and the Snake Bight trail = 12.1 – 2.6
Now,
By using the Long Subtraction,

Hence, from the above,
We can conclude that
The Coastal Prairie trail is 9.5 Kilometers longer than the Snake Bight trail

Lesson 1.2 Fluently Divide Whole Numbers and Decimals

Solve & Discuss It!
Some friends went to lunch and split the bill equally. If each person paid $6.75, how many people went to lunch? Use a diagram or equation to explain your thinking.
Envision Math Common Core Grade 6 Answer Key Topic 1 Use Positive Rational Numbers 6.4
Answer:
Some friends went to lunch and split the bill equally and each person paid $6.75
Now,
The given figure is:
Envision Math Common Core Grade 6 Answer Key Topic 1 Use Positive Rational Numbers 6.4

So,
The number of people that went for lunch – (The total bill) ÷ (The amount paid by each person)
= \(\frac{$27}{$6.75}\)
Now,
By using the Long Division,

Hence, from the above,
We can conclude that
The number of people that went for lunch is: 4 people

Reasoning How can you use reasoning to create a representation of the problem?
Answer:
Reasoning in maths is the process of applying logical thinking to a situation to derive the correct problem-solving strategy for a given question and using this method to develop and describe a solution. Put more simply, mathematical reasoning is the bridge between fluency and problem-solving.

Focus on math practices
Reasoning Suppose $7.00 was added to the bill for a dessert that everyone shared. How much more does each person have to pay?
Answer:
It is given that
Suppose $7.00 was added to the bill for a dessert that everyone shared
So,
The total bill = $27 + $7
= $34
So,
The amount each person has to pay = (The total bill) ÷ (The number of people that the bill was split)
= \(\frac{$34}{4}\)
Now,
By using the Long Division,

Hence, from the above,
We can conclude that
The amount that each person has to pay is about $8

? Essential Question How can you divide whole numbers and decimals?
Answer:
The steps to divide decimals by Whole Numbers are:
a. Bring down the next digit from the dividend. Continue dividing.
b. Place the decimal point in the quotient.
c. Check your answer: Multiply the divisor by the quotient to see if you get the dividend.

Scan for Multimedia

Try It!
Workers at an electronics company pack 2,610 smartphones in the boxes. Each box holds 9 smartphones. How many boxes do they fill?
Answer:
It is given that
Workers at an electronics company pack 2,610 smartphones in the boxes. Each box holds 9 smartphones
So,
The number of boxes does the workers fill = (The total number of smartphones in the box) ÷ (The number of smartphones each box holds)
= \(\frac{2,610}{9}\)
Now,
By using the Long Division,

Hence, from the above,
We can conclude that
The number of boxes does the workers fill are: 290 boxes

Convince Me! Why is the first digit of the quotient in the Try It! not in the same place as the first digit of the quotient in Example 1?
Envision Math Common Core Grade 6 Answer Key Topic 1 Use Positive Rational Numbers 6.5
Answer:
We know that,
Use place value to place the first digit. Look at the first digit. If the first digit is less than the divisor, then the first digit of the quotient will be in the hundreds place. If the first digit is greater than or equal to the divisor, then the first digit of the quotient will be in the thousands place
Hence, from the above,
We can conclude that the first digit of the quotient in the Try It! is not in the same place as the first digit of the quotient in Example 1

Try It!

Divide.

a. 65 ÷ 8
Answer:
The given division equation is: 65 ÷ 8
Now,
By using the Long Division,

Hence, from the above,
We can conclude that
65 ÷ 8 = 8.12

b. 14.4 ÷ 8
Answer:
The given division equation is: 14.4 ÷ 8
Now,
By using the Long Division,

Hence, from the above,
We can conclude that
14.4 ÷ 8 = 1.8

c. 128.8 ÷ 1.4
Answer:
The given division equation is: 128.8 ÷ 1.4
Now,
By using the Long Division,

Hence, from the above,
We can conclude that
128.8 ÷ 1.4 = 92

KEY CONCEPT
To divide by a decimal, rewrite the decimal so that you are dividing by a whole number. Multiply both the divisor and the dividend by the same power of 10. Then divide as you would with whole numbers.
Envision Math Common Core Grade 6 Answer Key Topic 1 Use Positive Rational Numbers 6.6

Do You Understand?

Question 1.
? Essential Question How can you divide whole numbers and decimals?
Answer:
The steps to divide decimals by Whole Numbers are:
a. Bring down the next digit from the dividend. Continue dividing.
b. Place the decimal point in the quotient.
c. Check your answer: Multiply the divisor by the quotient to see if you get the dividend.

Question 2.
When dividing with decimals, why is it necessary to multiply both the divisor and the dividend by the same power of 10?
Answer:
In cases like this, you can use powers of 10 to help create an easier problem to solve.
Example:
In this case, you can multiply the divisor, 0.3, by 10 to move the decimal point 1 place to the right. If you multiply the divisor by 10, then you also have to multiply the dividend by 10 to keep the quotient the same.

Question 3.
Use Structure Explain how you can decide where to place the first digit of the quotient for 6,139 ÷ 153.
Answer:
We know that,
Use place value to place the first digit. Look at the first digit. If the first digit is less than the divisor, then the first digit of the quotient will be in the hundreds place. If the first digit is greater than or equal to the divisor, then the first digit of the quotient will be in the thousands place
Hence, from the above,
We can conclude that
For 6,139 ÷ 153,
The first digit of the quotient will be in the hundred’s place

Question 4.
Use Structure How do you know where to place the decimal point in the quotient when dividing a decimal by a whole number?
Answer:
We know that,
The decimal point in the quotient is directly above the decimal point in the dividend. To divide a decimal by a whole number, we place the decimal point in the quotient above the decimal point in the dividend and then divide as usual.

Do You Know How?

Question 5.
Envision Math Common Core Grade 6 Answer Key Topic 1 Use Positive Rational Numbers 7.1
Answer:
The given division equation is: 9,853 ÷ 48
So,
By using the Long Division,

Hence, from the above,
We can conclude that
9,853 ÷ 48 = 205 R 13

In 6 and 7, divide. Record remainders.

Question 6.
2,789 ÷ 36
Answer:
The given division equation is: 2,789 ÷ 36
Now,
By using the Long Division,

Hence, from the above,
We can conclude that
2,879 ÷ 36 = 77 R 17
The remainder for the given division equation is: 17

Question 7.
Envision Math Common Core Grade 6 Answer Key Topic 1 Use Positive Rational Numbers 7.3
Answer:
The given division equation is: 153 ÷ 18
Now,
By using the Long Division,

Hence, from the above,
We can conclude that
153 ÷ 18 = 8 R 8
The remainder for the given division equation is: 8

In 8 and 9, divide. Write remainders as decimals.

Question 8.
Envision Math Common Core Grade 6 Answer Key Topic 1 Use Positive Rational Numbers 7.4
Answer:
The given division equation is: 139 ÷ 4
Now,
By using the Long Division,

Hence, from the above,
We can conclude that
139 ÷ 4 = 34 R 3
The remainder for the given division equation is: 3

Question 9.
215 ÷ 2
Answer:
The given division equation is: 215 ÷ 2
Now,
By using the Long Division,

Hence, from the above,
We can conclude that
215 ÷ 2 = 107 R 1
The remainder for the given division equation is: 1

In 10 and 11, divide.

Question 10.
Envision Math Common Core Grade 6 Answer Key Topic 1 Use Positive Rational Numbers 8.1
Answer:
The given division equation is: 34.75 ÷ 5
Now,
By using the Long Division,

Hence, from the above,
We can conclude that
34.75 ÷ 5 = 6.95

Question 11.
215.25 ÷ 5
Answer:
The given division equation is: 215.25 ÷ 5
Now,
By using the Long Division,

Hence, from the above,
We can conclude that
215.25 ÷ 5 = 43.05

In 12 and 13, divide. Annex zeros if needed to write remainders as decimals.

Question 12.
5.3 ÷ 0.2
Answer:
The given division equation is: 5.3 ÷ 0.2
Now,
By using the Long Division,

Hence, from the above,
We can conclude that
5.3 ÷ 0.2 = 26.5

Question 13.
Envision Math Common Core Grade 6 Answer Key Topic 1 Use Positive Rational Numbers 8.2
Answer:
The given division equation is: 8.9 ÷ 0.4
Now,
By using the Long Division,

Hence, from the above,
We can conclude that
8.9 ÷ 0.4 = 22.2

Practice & Problem Solving

Scan for Multimedia

Leveled Practice
In 14 and 15, divide.

Question 14.
Envision Math Common Core Grade 6 Answer Key Topic 1 Use Positive Rational Numbers 8.3
Answer:
The given division equation is: 5,841 ÷ 62
Now,
By using the Long Division,

Hence, from the above,
We can conclude that
5,841 ÷ 62 = 94 R 13

Question 15.
Envision Math Common Core Grade 6 Answer Key Topic 1 Use Positive Rational Numbers 8.4
Answer:
The given division equation is: 350 ÷ 4
Now,
By using the Long Division,

Hence, from the above,
We can conclude that
350 ÷ 4 = 87.5

In 16-19, divide. Record remainders.

Question 16.
2,593 ÷ 21
Answer:
The given division equation is: 2,593 ÷ 21
Now,
By using the Long Division,

Hence, from the above,
We can conclude that
2,593 ÷ 21 = 123 R 10
The remainder for the given division equation is: 10

Question 17.
Envision Math Common Core Grade 6 Answer Key Topic 1 Use Positive Rational Numbers 8.5
Answer:
The given division equation is: 6,927 ÷ 19
Now,
By using the Long Division,

Hence, from the above,
We can conclude that
6,927 ÷ 19 = 364 R 11
The remainder for the given division equation is: 11

Question 18.
Envision Math Common Core Grade 6 Answer Key Topic 1 Use Positive Rational Numbers 8.6
Answer:
The given division equation is: 2,483 ÷ 9
Now,
By using the Long Division,

Hence, from the above,
We can conclude that
2,483 ÷ 9 = 276 R 8
The remainder for the given division equation is: 8

Question 19.
968 ÷ 38
Answer:
The given division equation is: 968 ÷ 38
Now,
By using the Long Division,

Hence, from the above,
We can conclude that
968 ÷ 38 = 25 R 18
The remainder for the given division equation is: 25 R 18

In 20-23, divide. Write remainders as decimals.

Question 20.
Envision Math Common Core Grade 6 Answer Key Topic 1 Use Positive Rational Numbers 8.7
Answer:
The given division equation is: 56 ÷ 5
Now,
By using the Long Division,

Hence, from the above,
We can conclude that
56 ÷ 5 = 11.2

Question 21.
232 ÷ 40
Answer:
The given division equation is: 232 ÷ 40
Now,
By using the Long Division,

Hence, from the above,
We can conclude that
232 ÷ 40 = 5.8

Question 22.
44 ÷ 10
Answer:
The given division equation is: 44 ÷ 10
Now,
By using the Long Division,

Hence, from the above,
We can conclude that
44 ÷ 10 = 4.4

Question 23.
Envision Math Common Core Grade 6 Answer Key Topic 1 Use Positive Rational Numbers 9.1
Answer:
The given division equation is: 2,626 ÷ 4
Now,
By using the Long Division,

Hence, from the above,
We can conclude that
2,626 ÷ 4 = 656.5

In 24-27, divide.

Question 24.
Envision Math Common Core Grade 6 Answer Key Topic 1 Use Positive Rational Numbers 9.5
Answer:
The given division equation is: $54.18 ÷ 6
Now,
By using the Long Division,

Hence, from the above,
We can conclude that
$54.18 ÷ 6 = $9.03

Question 25.
187.2 ÷ 8
Answer:
The given division equation is: 187.2 ÷ 8
Now,
By using the Long Division,

Hence, from the above,
We can conclude that
187.2 ÷ 8 = 23.4

Question 26.
Envision Math Common Core Grade 6 Answer Key Topic 1 Use Positive Rational Numbers 9.51
Answer:
The given division equation is: 6.3 ÷ 7
Now,
By using the Long Division,

Hence, from the above,
We can conclude that
6.3 ÷ 7 = 0.9

Question 27.
137.5 ÷ 5
Answer:
The given division equation is: 137.5 ÷ 5
Now,
By using the Long Division,

Hence, from the above,
We can conclude that
137.5 ÷ 5 = 27.5

In 28-31, divide. Annex zeros if needed to write remainders as decimals.

Question 28.
6.4 ÷ 0.8
Answer:
The given division equation is: 6.4 ÷ 0.8
Now,
By using the Long Division,

Hence, from the above,
We can conclude that
6.4 ÷ 0.8 = 8

Question 29.
Envision Math Common Core Grade 6 Answer Key Topic 1 Use Positive Rational Numbers 9.6
Answer:
The given division equation is: 0.2430 ÷ 0.6
Now,
By using the Long Division,

Hence, from the above,
We can conclude that
0.2430 ÷ 0.6 = 0.405

Question 30.
52.056 ÷ 7.23
Answer:
The given division equation is: 52.056 ÷ 7.23
Now,
By using the Long Division,

Hence, from the above,
We can conclude that
52.056 ÷ 7.23 = 7.2

Question 31.
Envision Math Common Core Grade 6 Answer Key Topic 1 Use Positive Rational Numbers 10.1
Answer:
The given division equation is: 9.089 ÷ 0.745
Now,
By using the Long Division,

Hence, from the above,
We can conclude that
9.089 ÷ 0.745 = 12.2

Question 32.
Ants are one of the Thorny Devil lizard’s favorite foods. It can eat 45 ants per minute. How long would it take this lizard to eat 1,080 ants? Express your answer in minutes.
Envision Math Common Core Grade 6 Answer Key Topic 1 Use Positive Rational Numbers 10.2
Answer:
It is given that
Ants are one of the Thorny Devil lizard’s favorite foods. It can eat 45 ants per minute.
So,
The time it will take for Thorny Devil lizard to eat 1,080 ants = (The total number of ants) ÷ (The number of ants Thorny Devil lizard eat per minute)
= 1,080 ÷ 45
Now,
By using the Long Division,

Hence, from the above,
We can conclude that
The time it will take for Thorny Devil lizard to eat 1,080 ants is: 24 minutes

Question 33.
Critique Reasoning Henrieta divided 0.80 by 20 as shown. Is her work correct? If not, explain why and give a correct response.
Envision Math Common Core 6th Grade Answers Topic 1 Use Positive Rational Numbers 10.3
Answer:
It is given that
Henrieta divided 0.80 by 20 as shown
So,
The representation of the division equation Henrieta divided is: 0.80 ÷ 20
Now,
By using the Long Division,

So,
From the above,
We can observe that
0.80 ÷ 20 = 0.04
Hence, from the above,
We can conclude that Henrieta’s work is not correct

Question 34.
Which brand of fruit snacks costs less per pound? How much less?
Envision Math Common Core 6th Grade Answers Topic 1 Use Positive Rational Numbers 10.4
Answer:
The given figure is:
Envision Math Common Core 6th Grade Answers Topic 1 Use Positive Rational Numbers 10.4
Now,
The cost per pound of Brand A’s fruit snacks = (The cost of Brand A fruit snacks) ÷ (The amount of Brand A fruit snacks)
= $16.20 ÷ 15
Now,
By using the Long Division,

So,
The cost per pound of Brand A’s fruit snacks is: $1.08
Now,
The cost per pound of Brand B’s fruit snacks = (The cost of Brand B fruit snacks) ÷ (The amount of Brand B fruit snacks)
= $22.25 ÷ 25
Now,
By using the Long Division,

So,
The cost per pound of Brand B’s fruit snacks is: $0.89
So,
The difference between the cost per pound of two brands = $1.08 – $0.89
Now,
By using the Long Subtraction,

So,
The difference between the cost per pound of two brands is: $0.19
Hence, from the above,
We can conclude that
The cost per pound of Brand B’s fruit snacks is: $0.89
The difference between the cost per pound of two brands is: $0.19

Question 35.
Be Precise How many times as much does each item cost in 2010 as in 1960?
Envision Math Common Core 6th Grade Answers Topic 1 Use Positive Rational Numbers 10.5
Answer:
The given data is:

Now,
Let the number of times as much does each item cost in 2010 as in 1960 be x
So,
For the Movie Ticket,
x = $9.75 ÷ $0.75
Now,
By using the Long Division,

So,
For Regular popcorn,
x = $4.10 ÷ $0.25
Now,
By using the Long Division,

So,
For Regular Drink,
x = $3.08 ÷ $0.35
Now,
By using the Long Division,

Hence, from the above,
We can conclude that
The number of times as much does the movie ticket cost in 2010 as in 1960 is: 13 times
The number of times as much does the regular ticket cost in 2010 as in 1960 is: 16.40 times
The number of times as much does the regular drink cost in 2010 as in 1960 is: 8.80 times

Question 36.
Higher-Order Thinking Kendra has 5.5 pounds of popcorn and wants to package it equally in 50 bags. How can she use place-value reasoning to find the amount of popcorn to put in each bag?
Answer:
It is given that
Kendra has 5.5 pounds of popcorn and wants to package it equally in 50 bags.
Now,
We know that,
Use place value to place the first digit. Look at the first digit. If the first digit is less than the divisor, then the first digit of the quotient will be in the hundreds place. If the first digit is greater than or equal to the divisor, then the first digit of the quotient will be in the thousands place
So,
The amount of popcorn Kendra put in each bag = (The total number of pounds of popcorn Kendra has) ÷ (The number of bags Kendra wants to package the popcorn equally)
= 5.5 ÷ 50
Now,
By using the Long Division,

Hence, from the above,
We can conclude that
The amount of popcorn Kendra put in each bag is: 0.11 pounds

Question 37.
You and a friend are paid $38.25 for doing yard work. You worked 2.5 hours and your friend worked 2 hours. You split the money according to the amount of time each of you worked. How much is your share of the money? Explain.
Answer:
It is given that
You and a friend are paid $38.25 for doing yard work. You worked 2.5 hours and your friend worked 2 hours. You split the money according to the amount of time each of you worked
So,
The amount of money that is your share = (The amount of money you and your friend are paid for doing yard work) ÷ (The number of hours you worked)
= $38.25 ÷ 2.5
Now,
By using the Long Division,

Hence, from the above,
We can conclude that
The amount of money that is your share is: $15.30

Assessment Practice

Question 38.
What is the value of the expression 1,248 ÷ 25?
A. 49
B. 49 R9
C. 49.9
D. 49 R 23
Answer:
The given division equation is: 1,248 ÷ 25
Now,
By using the Long Division,

Hence, from the above,
We can conclude that
The value of the expression 1,248 ÷ 25 is:

Question 39.
Which expression has the same solution as 3,157 ÷ 41?
A. 1,852 ÷ 24
B. 1,928 ÷ 25
C. 2,079 ÷ 27
D. 2,184 ÷ 28
Answer:
The given division equation is: 3,157 ÷ 41
Now,
By using the Long Division,

Now,
The expressions that have the same value as the given expression by using the Long Division are:
A. 1,852 ÷ 24

B. 1,928 ÷ 25

C. 2,079 ÷ 27

D. 2,184 ÷ 28

Hence, from the above,
We can conclude that
The expressions that have the same value as the given expression are:

Lesson 1-3 Multiply Fractions

Solve & Discuss It!

ACTIVITY

The art teacher gave each student half of a sheet of paper. Then she asked the students to color one-fourth of their pieces of paper. What part of the original sheet did the students color?
Answer:
It is given that
The art teacher gave each student half of a sheet of paper. Then she asked the students to color one-fourth of their pieces of paper
So,
The part of the original sheet did the students color = (The piece of paper the art teacher gave to each student) × (The part of the paper the are teacher asked the students to color)
= \(\frac{1}{2}\) × \(\frac{1}{4}\)
= \(\frac{1 × 1}{2 × 4}\)
= \(\frac{1}{8}\) of the piece of paper
Hence, from the above,
We can conclude that
The part of the original sheet did the students color is: \(\frac{1}{8}\) of the piece of paper

A model with Math
How can you use a picture to represent the problem?
Envision Math Common Core 6th Grade Answers Topic 1 Use Positive Rational Numbers 11.1
Answer:
By representing units of measurement and other objects visually, students can begin to think about the problem mathematically. Pictures and diagrams are also good ways of describing solutions to problems; therefore they are an important part of mathematical communication.

Focus on math practices
Reasoning
Should your answer be less than or greater than 1? Explain.
Answer:
From the above problem,
We can observe that
The part of the original sheet did the students color is: \(\frac{1}{8}\) of the piece of paper
Now,
By using the Long Division,
The value of \(\frac{1}{8}\) is:

Hence, from the above,
We can conclude that
Your answer is less than 1

? Essential Question How can you multiply fractions and mixed numbers?
Answer:
The steps that are used to multiply mixed numbers and fractions are:
a. Convert the given mixed fraction to an improper fraction.
b. Write the whole number in the form of a fraction with denominator 1.
c. Multiply the numerators of both the fractions.
d. Multiply the denominators of both the fractions.
e. Combine them and simplify the fraction to its lowest form.

Try It!

Find \(\frac{1}{4}\) × \(\frac{1}{5}\) using the area model. Explain.
Envision Math Common Core 6th Grade Answers Topic 1 Use Positive Rational Numbers 11.2
Answer:
The given multiplication equation is:
\(\frac{1}{4}\) × \(\frac{1}{5}\)
Now,
By using the Area model,

So,

So,
\(\frac{1}{4}\) × \(\frac{1}{5}\)
= \(\frac{1 × 1}{4 × 5}\)
= \(\frac{1}{20}\)
Hence, from the above,
We can conclude that
\(\frac{1}{4}\) × \(\frac{1}{5}\) = \(\frac{1}{20}\)

Convince Me! Why is the product of \(\frac{1}{4}\) × \(\frac{1}{5}\) less than each factor?
Envision Math Common Core 6th Grade Answers Topic 1 Use Positive Rational Numbers 11.3
Answer:
The given multiplication equation is:
\(\frac{1}{4}\) × \(\frac{1}{5}\)
So,
\(\frac{1}{4}\) × \(\frac{1}{5}\)
= \(\frac{1}{20}\)
Now,
When we compare the denominators,
4 < 5 and 5 < 20
So,
20  > 5 > 4 —- (1)
But,
When we compare the denominators,
The same relationship of eq (1) will be reversed
So,
20 < 5 < 4
Hence, from the above,
We can conclude that
\(\frac{1}{4}\) × \(\frac{1}{5}\) is less than each factor

Try It!
Find \(\frac{3}{4}\) × \(\frac{4}{6}\) using the number line. Explain.
Envision Math Common Core 6th Grade Answers Topic 1 Use Positive Rational Numbers 11.4
Answer:
The given multiplication equation is:
\(\frac{3}{4}\) × \(\frac{4}{6}\)
So,
\(\frac{3}{4}\) × \(\frac{4}{6}\)
= \(\frac{3 × 4}{4 × 6}\)
= \(\frac{12}{24}\)
= \(\frac{3}{6}\)
So,
The representation of the product of \(\frac{3}{4}\) × \(\frac{4}{6}\) in the number line is:

Hence, from the above,
We can concldue that
\(\frac{3}{4}\) × \(\frac{4}{6}\) = \(\frac{3}{6}\)

Try It!
A clothing factory makes T-shirts. If each machine makes 3\(\frac{1}{3}\) T-shirts per hour, how many T-shirts does one machine make in 4\(\frac{1}{2}\) hours? Write and solve an equation.
Envision Math Common Core 6th Grade Answers Topic 1 Use Positive Rational Numbers 11.5
Answer:
It is given that
A clothing factory makes T-shirts. If each machine makes 3\(\frac{1}{3}\) T-shirts per hour
So,
The number of T-shirts does 1 machine makes in 4\(\frac{1}{2}\) hours = (The number of T-shirts made by each machine in a hour) × 4\(\frac{1}{2}\) hours
Now,
The representation of 3\(\frac{1}{3}\) into a fraction is: \(\frac{10}{3}\)
The representation of 4\(\frac{1}{2}\) into a fraction is: \(\frac{9}{2}\)
So,
The number of T-shirts does 1 machine makes in 4\(\frac{1}{2}\) hours = \(\frac{10}{3}\) × \(\frac{9}{2}\)
= \(\frac{10 ×9}{3 ×2}\)
= \(\frac{90}{6}\)
= 15 T-shirts
Hence, from the above,
We can conclude that
The number of T-shirts does 1 machine makes in 4\(\frac{1}{2}\) hours is: 15 T-shirts

KEY CONCEPT
You can find the product of fractions or mixed numbers.
Envision Math Common Core 6th Grade Answers Topic 1 Use Positive Rational Numbers 11.6

Do You Understand?

Question 1.
? Essential Question How can you multiply fractions and mixed numbers?
Answer:
The steps that are used to multiply mixed numbers and fractions are:
a. Convert the given mixed fraction to an improper fraction.
b. Write the whole number in the form of a fraction with denominator 1.
c. Multiply the numerators of both the fractions.
d. Multiply the denominators of both the fractions.
e. Combine them and simplify the fraction to its lowest form.

Question 2.
Reasoning Is the product of \(\frac{3}{6}\) × \(\frac{5}{4}\) equal to the product of \(\frac{3}{6}\) × \(\frac{5}{6}\)? Explain.
Answer:
The given multiplication equations are:
\(\frac{3}{6}\) × \(\frac{5}{4}\)
\(\frac{3}{6}\) × \(\frac{5}{6}\)
Now,
By using the Long Multiplication,

Now,
By using the Long Multiplication,

Hence, from the above,
We can conclude that
The product of \(\frac{3}{6}\) × \(\frac{5}{4}\)  is not equal to the product of \(\frac{3}{6}\) × \(\frac{5}{6}\)

Question 3.
Construct Arguments Why is adding \(\frac{3}{9}\) and \(\frac{6}{9}\) different from multiplying the two fractions?
Answer:

Question 4.
Tina has \(\frac{1}{2}\) of a pan of cornbread left from a dinner party. She eats \(\frac{1}{2}\) of the leftover part the next night. How much of the whole pan does Tina eat? Write and solve an equation.
Answer:
It is given that
Tina has \(\frac{1}{2}\) of a pan of cornbread left from a dinner party. She eats \(\frac{1}{2}\) of the leftover part the next night
So,
The amount of the whole pan of cornbread does Tina eat = (The amount of a pan of cornbread left from a dinner party) × (The amount of the leftover from a dinner party that Tine eat the next night)
= \(\frac{1}{2}\) × \(\frac{1}{2}\)
Now,
By using the Long Multiplication,

Hence, from the above,
We can conclude that
The amount of the whole pan of cornbread does Tina eat is: \(\frac{1}{4}\) of a pan of cornbread

Question 5.
Construct Arguments Explain how you would multiply 5 × 2\(\frac{1}{2}\).
Answer:
The given multiplication is: 5 × 2\(\frac{1}{2}\)
So,
The representation of 2\(\frac{1}{2}\) into a fraction is:

Now,
By using the Long Multiplication,

Hence, from the above,
We can conclude that
5 × 2\(\frac{1}{2}\) = \(\frac{25}{2}\)

Question 6.
In Example 1, find the fraction of a whole pan of lasagna that Tom ate if he started with \(\frac{7}{8}\) of a pan.
Answer:
It is given that
Tom has a \(\frac{7}{8}\) of a pan of lasagna left and Tom ate \(\frac{1}{3}\) of this amount
So,
The fraction of a whole pan of lasagna that Tom ate = (The amount of a pan of lasagna left) × (The amount that Tom ate from the amount of lasagna that is left)
= \(\frac{7}{8}\) × \(\frac{1}{3}\)
Now,
By using the Long Multiplication,

Hence, from the above,
We can conclude that
The fraction of a whole pan of lasagna that Tom ate is: \(\frac{7}{24}\) of a pan of lasagna

Do You Know How?

Question 7.
Find \(\frac{5}{6}\) × \(\frac{1}{2}\). Use the model to help solve.
Envision Math Common Core 6th Grade Answers Topic 1 Use Positive Rational Numbers 12.1
Answer:
The given multiplication equation is:
\(\frac{5}{6}\) × \(\frac{1}{2}\)
Now,
By using the Area model,

Now,
By using the Long Multiplication,

Hence, from the above,
We can conclude that
\(\frac{5}{6}\) × \(\frac{1}{2}\) = \(\frac{5}{12}\)

Question 8.
Find \(\frac{3}{4}\) × \(\frac{4}{9}\).
Answer:
The given multiplication equation is:
\(\frac{3}{4}\) × \(\frac{4}{9}\)
Now,
By using the Long Multiplication,

Hence, from the above,
We can conclude that
\(\frac{3}{4}\) × \(\frac{4}{9}\) = \(\frac{1}{3}\)

In 9-16, find each product.

Question 9.
\(\frac{2}{3}\) × \(\frac{1}{2}\)
Answer:
The given multiplication equation is:
\(\frac{2}{3}\) × \(\frac{1}{2}\)
Now,
By using the Long Multiplication,

Hence, from the above,
We can conclude that
\(\frac{2}{3}\) × \(\frac{1}{2}\) = \(\frac{1}{3}\)

Question 10.
\(\frac{5}{9}\) × \(\frac{1}{9}\)
Answer:
The given multiplication equation is:
\(\frac{5}{9}\) × \(\frac{1}{9}\)
Now,
By using the Long Division,

Hence, from the above,
We can conclude that
\(\frac{5}{9}\) × \(\frac{1}{9}\) = \(\frac{5}{81}\)

Question 11.
\(\frac{7}{10}\) × \(\frac{3}{4}\)
Answer:
The given multiplication equation is:
\(\frac{7}{10}\) × \(\frac{3}{4}\)
Now,
By using the Long Division,

Hence, from the above,
We can conclude that
\(\frac{7}{10}\) × \(\frac{3}{4}\) = \(\frac{21}{40}\)

Question 12.
\(\frac{1}{3}\) × \(\frac{1}{4}\)
Answer:
The given multiplication equation is:
\(\frac{1}{3}\) × \(\frac{1}{4}\)
Now,
By using the Long Multiplication,

Hence, from the above,
We can conclude that
\(\frac{1}{3}\) × \(\frac{1}{4}\) = \(\frac{1}{12}\)

Question 13.
\(\frac{5}{6}\) × \(\frac{3}{7}\)
Answer:
The given multiplication equation is:
\(\frac{5}{6}\) × \(\frac{3}{7}\)
Now,
By using the Long Division,

Hence, from the above,
We can conclude that
\(\frac{5}{6}\) × \(\frac{3}{7}\) = \(\frac{5}{14}\)

Question 14.
\(\frac{3}{5}\) × \(\frac{11}{12}\)
Answer:
The give multiplication equation is:
\(\frac{3}{5}\) × \(\frac{11}{12}\)
Now,
By using the Long Multiplication,

Hence, fro the above,
We can conclude that
\(\frac{3}{5}\) × \(\frac{11}{12}\) = \(\frac{11}{20}\)

Question 15.
\(\frac{4}{10}\) × \(\frac{2}{5}\)
Answer:
The given multiplication equation is:
\(\frac{4}{10}\) × \(\frac{2}{5}\)
Now,
By using the Long Multiplication,

Hence, from the above,
We can conclude that
\(\frac{4}{10}\) × \(\frac{2}{5}\) = \(\frac{4}{25}\)

Question 16.
\(\frac{3}{4}\) × \(\frac{2}{9}\)
Answer:
The given multiplication equation is:
\(\frac{3}{4}\) × \(\frac{2}{9}\)
Now,
By using the Long Multiplication,

Hence, from the above,
We can conclude that
\(\frac{3}{4}\) × \(\frac{2}{9}\) = \(\frac{1}{6}\)

In 17 and 18, estimate the product. Then complete the multiplication.

Question 17.
Envision Math Common Core 6th Grade Answers Topic 1 Use Positive Rational Numbers 16.1
Answer:
The given multiplication equation is:
2\(\frac{3}{4}\) × 8
Now,
The representation of 2\(\frac{3}{4}\) into a fraction is:

So,
By using the Long Multiplication,

Hence, from the above,
We can conclude that

Question 18.
Envision Math Common Core 6th Grade Answers Topic 1 Use Positive Rational Numbers 16.2
Answer:
The given multiplication equation is:
4\(\frac{1}{2}\) × 1\(\frac{1}{4}\)
Now,
The representation of 4\(\frac{1}{2}\) into a fraction is:

The representation of 1\(\frac{1}{4}\) into a fraction is:

So,
By using the Long Multiplication,

Hence, from the above,
We can conclude that

Practice & Problem Solving

Scan for Multimedia

In 19 and 20, find each product. Shade the model to help solve.

Question 19.
\(\frac{1}{3}\) × \(\frac{5}{6}\)
Envision Math Common Core 6th Grade Answers Topic 1 Use Positive Rational Numbers 16.3
Answer:
The given multiplication equation is:
\(\frac{1}{3}\) × \(\frac{5}{6}\)
Now,
The representation of the given multiplication equation in the form of a model is:

So,
By using the Long Multiplication,

Hence, from the above,
We can conclude that
\(\frac{1}{3}\) × \(\frac{5}{6}\) = \(\frac{5}{18}\)

Question 20.
\(\frac{2}{3}\) × \(\frac{1}{12}\)
Envision Math Common Core 6th Grade Answers Topic 1 Use Positive Rational Numbers 16.4
Answer:
The given multiplication equation is:
\(\frac{2}{3}\) × \(\frac{1}{12}\)
So,
The representation of the given multiplication equation in the form of a model is:

So,
By using the Long Multiplication,

Hence, from the above,
We can conclude that
\(\frac{2}{3}\) × \(\frac{1}{12}\) = \(\frac{1}{18}\)

In 21-28, find each product.

Question 21.
\(\frac{7}{8}\) × \(\frac{1}{2}\)
Answer:
The given multiplication equation is:
\(\frac{7}{8}\) × \(\frac{1}{2}\)
Now,
By using the Long Multiplication,

Hence, from the above,
We can conclude that
\(\frac{7}{8}\) × \(\frac{1}{2}\) = \(\frac{7}{16}\)

Question 22.
\(\frac{2}{5}\) × \(\frac{1}{12}\)
Answer:
The given multiplication equation is:
\(\frac{2}{5}\) × \(\frac{1}{12}\)
Now,
By using the Long Multiplication,

Hence, from the above,
We can conclude that
\(\frac{2}{5}\) × \(\frac{1}{12}\) = \(\frac{1}{30}\)

Question 23.
\(\frac{5}{7}\) × \(\frac{7}{9}\)
Answer:
The given multiplication equation is:
\(\frac{5}{7}\) × \(\frac{7}{9}\)
Now,
By using the Long Multiplication,

Hence, from the above,
We can conclude that
\(\frac{5}{7}\) × \(\frac{7}{9}\) = \(\frac{5}{9}\)

Question 24.
\(\frac{1}{2}\) × \(\frac{3}{4}\)
Answer:
The given multiplication equation is:
\(\frac{1}{2}\) × \(\frac{3}{4}\)
Now,
By using the Long Multiplication,

Hence, from the above,
We can conclude that
\(\frac{1}{2}\) × \(\frac{3}{4}\) = \(\frac{3}{8}\)

Question 25.
\(\frac{1}{4}\) × \(\frac{7}{8}\)
Answer:
The given multiplication equation is:
\(\frac{1}{4}\) × \(\frac{7}{8}\)
Now,
By using the Long Multiplication,

Hence, from the above,
We can conclude that
\(\frac{1}{4}\) × \(\frac{7}{8}\) = \(\frac{7}{32}\)

Question 26.
\(\frac{5}{6}\) × \(\frac{9}{10}\)
Answer:
The given multiplication equation is:
\(\frac{5}{6}\) × \(\frac{9}{10}\)
Now,
By using the Long Multiplication,

Hence, from the above,
We can conclude that
\(\frac{5}{6}\) × \(\frac{9}{10}\) = \(\frac{3}{4}\)

Question 27.
\(\frac{1}{4}\) × \(\frac{1}{8}\)
Answer:
The given multiplication equation is:
\(\frac{1}{4}\) × \(\frac{1}{8}\)
Now,
By using the Long Multiplication,

Hence, from the above,
We can conclude that
\(\frac{1}{4}\) × \(\frac{1}{8}\) = \(\frac{1}{32}\)

Question 28.
\(\frac{1}{3}\) × \(\frac{3}{7}\)
Answer:
The given multiplication equation is:
\(\frac{1}{3}\) × \(\frac{3}{7}\)
Now,
By using the Long Multiplication,

Hence, from the above,
We can conclude that
\(\frac{1}{3}\) × \(\frac{3}{7}\) = \(\frac{1}{7}\)

In 29-36, estimate the product. Then find each product.

Question 29.
2\(\frac{1}{6}\) × 4\(\frac{2}{9}\)
Answer:
The given multiplication equation is:
2\(\frac{1}{6}\) × 4\(\frac{2}{9}\)
Now,
By using the Long Multiplication,

Hence, from the above,
We can conclude that
2\(\frac{1}{6}\) × 4\(\frac{2}{9}\) = \(\frac{247}{27}\)

Question 30.
\(\frac{3}{4}\) × 8\(\frac{1}{2}\)
Answer:
The given multiplication equation is:
\(\frac{3}{4}\) × 8\(\frac{1}{2}\)
Now,
The representation of 8\(\frac{1}{2}\) into a fraction is:

So,
By using the Long Division,

Hence, from the above,
We can conclude that
\(\frac{3}{4}\) × 8\(\frac{1}{2}\) = \(\frac{51}{8}\)

Question 31.
1\(\frac{1}{8}\) × 3\(\frac{1}{3}\)
Answer:
The given multiplication equation is:
1\(\frac{1}{8}\) × 3\(\frac{1}{3}\)
Now,
By using the Long Multiplication,

Hence, from the above,
We can conclude that
1\(\frac{1}{8}\) × 3\(\frac{1}{3}\) = \(\frac{15}{4}\)

Question 32.
3\(\frac{1}{2}\) × \(\frac{2}{3}\)
Answer:
The given multiplication equation is:
3\(\frac{1}{2}\) × \(\frac{2}{3}\)
Now,
The representation of 3\(\frac{1}{2}\) into a fraction is:

So,
By using the Long Division,

Hence, from the above,
We can conclude that
3\(\frac{1}{2}\) × \(\frac{2}{3}\) = \(\frac{7}{3}\)

Question 33.
3\(\frac{1}{4}\) × 6
Answer:
The given multiplication equation is:
3\(\frac{1}{4}\) × 6
Now,
The representation of 3\(\frac{1}{4}\) into a fraction is:

So,
By using the Long Multiplication,

Hence, from the above,
We can conclude that
3\(\frac{1}{4}\) × 6 = \(\frac{39}{2}\)

Question 34.
5\(\frac{1}{3}\) × 3
Answer:
The given multiplication equation is:
5\(\frac{1}{3}\) × 3
Now,
The representation of 5\(\frac{1}{3}\) into a fraction is:

So,
By using the Long Multiplication,

Hence, from the above,
We can conclude that
5\(\frac{1}{3}\) × 3 = 16

Question 35.
2\(\frac{3}{8}\) × 4
Answer:
The given multiplication equation is:
2\(\frac{3}{8}\) × 4
Now,
The representation of 2\(\frac{3}{8}\) into a fraction is:

So,
By using the Long Multiplication,

Hence, from the above,
We can conclude that
2\(\frac{3}{8}\) × 4 = \(\frac{19}{2}\)

Question 36.
4\(\frac{1}{8}\) × 5\(\frac{1}{2}\)
Answer:
The given multiplication equation is:
4\(\frac{1}{8}\) × 5\(\frac{1}{2}\)
Now,
By using the Long Multiplication,

Hence, from the above,
We can conclude that
4\(\frac{1}{8}\) × 5\(\frac{1}{2}\) = \(\frac{363}{16}\)

In 37 and 38, use the diagram at the right.

Question 37.
Linda walked \(\frac{3}{4}\) of the length of the Tremont Trail before stopping for a rest. How far had Linda walked on the trail?
Envision Math Common Core 6th Grade Answers Topic 1 Use Positive Rational Numbers 20.1
Answer:
It is given that
Linda walked \(\frac{3}{4}\) of the length of the Tremont Trail before stopping for a rest
Now,
The given information is:
Envision Math Common Core 6th Grade Answers Topic 1 Use Positive Rational Numbers 20.1
So,
The distance covered by Linda on the trail = (The distance Linda walked of the Tremont trail) × (The length of the Tremont trail)
= \(\frac{3}{4}\) × 3\(\frac{1}{2}\)
Now,
The representation of 3\(\frac{1}{2}\) into a fraction is:

So,
By using the Long Multiplication,

Hence, from the above,
We can conclude that
The distance covered by Linda on the trail is: \(\frac{25}{8}\) miles

Question 38.
The city plans to extend the Wildflower Trail to make it 2\(\frac{1}{2}\) times its current length in the next 5 years. How long will the Wildflower Trail be at the end of 5 years?
Answer:
It is given that
The city plans to extend the Wildflower Trail to make it 2\(\frac{1}{2}\) times its current length in the next 5 years
Now,
The given information is:
Envision Math Common Core 6th Grade Answers Topic 1 Use Positive Rational Numbers 20.1
So,
The length of the Wildflower trail at the end of 5 years = (The extended length of the Wildflower trail at the end of 5 years) × (The current length of the Wildflower trail)
= 2\(\frac{1}{2}\) × 2\(\frac{3}{8}\)
Now,
By using the Long Multiplication,

Hence, from the above,
We can conclude that
The length of the Wildflower trail at the end of 5 years will be: \(\frac{95}{16}\) miles

Question 39.
The world’s smallest gecko is \(\frac{3}{4}\) inch long. An adult male Western Banded Gecko is 7\(\frac{1}{3}\) times as long. How long is an adult male Western Banded Gecko?
Envision Math Common Core 6th Grade Answers Topic 1 Use Positive Rational Numbers 20.2
Answer:
It is given that
The world’s smallest gecko is \(\frac{3}{4}\) inch long. An adult male Western Banded Gecko is 7\(\frac{1}{3}\) times as long
So,
The length of an adult male Western Banded Gecko = (The length of the world’s small gecko) × (The number of times an adult male western banded gecko is as larger than the world’s smallest gecko)
= \(\frac{3}{4}\) × 7\(\frac{1}{3}\)
Now,
The representation of 7\(\frac{1}{3}\) into a fraction is:

So,
By using the Long Multiplication,

Hence, from the above,
We can conclude that
The length of an adult male Western Banded Gecko is: \(\frac{11}{2}\) inches

Question 40.
Higher-Order Thinking In Ms. Barclay’s classroom, \(\frac{2}{5}\) of the students play chess. Of the students who play chess, \(\frac{5}{6}\) also play sudoku. If there are 30 students in Ms. Barclay’s class, how many play chess and sudoku?
Answer:
It is given that
In Ms. Barclay’s classroom, \(\frac{2}{5}\) of the students play chess. Of the students who play chess, \(\frac{5}{6}\) also play sudoku. If there are 30 students in Ms. Barclay’s class
So,
The number of students who play chess = (The total number of students) × (The fracti of the students that play chess)
= 30 × \(\frac{2}{5}\)
Now,
By using the Long Multiplication,

So,
The number of students who play chess are: 12 students
Now,
The number of students who play Sudoku = (The number of students who play chess) × (The fraction of the students who play Sudoku)
= 12 × \(\frac{5}{6}\)
Now,
By using the Long Multiplication,

So,
The number of students who play sudoku are: 10 students
Now,
The number of students who play chess and sudoku = (The number of students who play chess) – (The number of students who play sudoku)
= 12 – 10
= 2 students
Hence, from the above,
We can conclude that
The number of students who play chess are: 12 students
The number of students who play sudoku are: 10 students
The number of students who play chess and sudoku are: 2 students

Question 41.
The Boca Grande Causeway in Florida is about 1\(\frac{4}{9}\) times as long as the Golden Gate Bridge in San Francisco. The Golden Gate Bridge is about 9,000 feet long. About how long is the Boca Grande Causeway?
Answer:
It is given that
The Boca Grande Causeway in Florida is about 1\(\frac{4}{9}\) times as long as the Golden Gate Bridge in San Francisco. The Golden Gate Bridge is about 9,000 feet long
So,
The length of the Boca Grande causeway = (The number of times the Boca Grande causeway as long as the Golden Gate Bridge) × (The length of the Golden Gate Bridge)
= 1\(\frac{4}{9}\) × 9,000
Now,
The representation of 1\(\frac{4}{9}\) into a fraction is:

Now,
By using the Long Multiplication,

Hence, from the above,
We acn conclude that
The length of the Boca Grande causeway is: 13,000 feet

Question 42.
If \(\frac{7}{8}\) is multiplied by \(\frac{4}{5}\), will the product be greater than either of the two factors? Explain.
Answer:
The given multiplication equation is:
\(\frac{7}{8}\) × \(\frac{4}{5}\)
Now,
The representation of the given multiplication equation in the form of a decimal numbers is:
0.875 × 0.8
Now,
By using the Long Multiplication,

Now,
The representation of the value of the product in the form of a decimal number is: 0.7
Hence, from the above,
We can conclude that
The product would be smaller than either of the two factors

Question 43.
Be Precise To amend the U.S. Constitution, \(\frac{3}{4}\) of the 50 states must approve the amendment. If 35 states approve an amendment, will the Constitution be amended?
Answer:
It is given that
To amend the U.S. Constitution, \(\frac{3}{4}\) of the 50 states must approve the amendment
So,
The number of states that approved the amendment = (The total number of states) × (The number of states who approved the amendment)
= 50 × \(\frac{3}{4}\)
Now,
By using the Long Multiplication,

Now,
We can observe that about 37 states approved the amendment from the above Long Multiplication
Hence, from the above,
We can conclude that
If 35 states approve an amendment, the Constitution will be amended

Question 44.
A scientist had \(\frac{3}{4}\) of a bottle of a solution. She used \(\frac{1}{6}\) of the solution in an experiment. How much of the bottle did she use?
Envision Math Common Core 6th Grade Answers Topic 1 Use Positive Rational Numbers 22.10
Answer:
It is given that
A scientist had \(\frac{3}{4}\) of a bottle of a solution. She used \(\frac{1}{6}\) of the solution in an experiment
So,
The amount of the solution in the bottle did she used = (The total amount of solution in the bottle) × (the amount of solution did she used in an experiment)
= \(\frac{3}{4}\) × \(\frac{1}{6}\)
Now,
By using the Long Multiplication,

Hence, from the above,
We can conclude that
The amount of the solution in the bottle did she used is: \(\frac{1}{8}\) of the total amount of solution

Question 45.
In the voting for City Council Precinct 5, only \(\frac{1}{2}\) of all eligible voters cast votes. What fraction of all eligible voters voted for Shelley? Morgan? Who received the most votes?
Envision Math Common Core 6th Grade Answers Topic 1 Use Positive Rational Numbers 20.11
Answer:
It is given that
In the voting for City Council Precinct 5, only \(\frac{1}{2}\) of all eligible voters cast votes
Now,
The given table is:
Envision Math Common Core 6th Grade Answers Topic 1 Use Positive Rational Numbers 20.11
So,
The fraction of the eligible voters voted for Shelly = (The fraction of the eligible voters voted) × (The fraction of votes received by Shelly)
= \(\frac{1}{2}\) × \(\frac{3}{10}\)
Now,
By using the Long Multiplication,

So,
The fraction of the eligible voters voted for Shelly is: \(\frac{3}{20}\) votes
So,
The fraction of the eligible voters voted for Morgan = (The fraction of the eligible voters voted) × (The fraction of votes received by Morgan)
= \(\frac{1}{2}\) × \(\frac{5}{8}\)
Now,
By using the Long Multiplication,

So,
The fraction of the eligible voters voted for Morgan is: \(\frac{5}{16}\) votes
Now,
The representation of \(\frac{3}{20}\) into a decimal number is: 0.15
The representation of \(\frac{5}{16}\) into a decimal number is: 0.3125
Hence, from the above,
We can conclude that
Morgan received the most votes

Assessment Practice

Question 46.
Which of these equations is equivalent to 1\(\frac{1}{2}\) × 3\(\frac{1}{5}\) = 4\(\frac{1}{2}\)?
A. 4\(\frac{1}{2}\) ÷ 3\(\frac{1}{5}\) = 1\(\frac{1}{2}\)
B. 1\(\frac{1}{2}\) ÷ 4\(\frac{1}{2}\) = 3\(\frac{1}{5}\)
C. 1\(\frac{1}{2}\) ÷ 3\(\frac{1}{5}\) = 4\(\frac{1}{2}\)
D. 3\(\frac{1}{5}\) ÷ 4\(\frac{1}{2}\) = 1\(\frac{1}{2}\)
Answer:
The given multiplication equation is:
1\(\frac{1}{2}\) × 3\(\frac{1}{5}\) = 4\(\frac{1}{2}\)
Now,
Divide the above equation with 3\(\frac{1}{5}\)
So,
We will get
4\(\frac{1}{2}\) ÷ 3\(\frac{1}{5}\) = 1\(\frac{1}{2}\)
Hence, from the above,
We can conclude that
The equations that are equivalent to 1\(\frac{1}{2}\) × 3\(\frac{1}{5}\) = 4\(\frac{1}{2}\) are:

Question 47.
Which of these equations is equivalent to \(\frac{3}{4}\) × 8\(\frac{1}{5}\) = 6\(\frac{3}{20}\)? Select all that apply.
Envision Math Common Core 6th Grade Answers Topic 1 Use Positive Rational Numbers 26 \(\frac{3}{4}\) ÷ 8\(\frac{1}{5}\) = 6\(\frac{3}{20}\)
Envision Math Common Core 6th Grade Answers Topic 1 Use Positive Rational Numbers 26 6\(\frac{3}{20}\) ÷ \(\frac{3}{4}\) = 8\(\frac{1}{5}\)
Envision Math Common Core 6th Grade Answers Topic 1 Use Positive Rational Numbers 26 6\(\frac{3}{20}\) ÷ 8\(\frac{1}{5}\) = \(\frac{3}{4}\)
Envision Math Common Core 6th Grade Answers Topic 1 Use Positive Rational Numbers 26 \(\frac{3}{4}\) ÷ 6\(\frac{3}{20}\) = 8\(\frac{1}{5}\)
Envision Math Common Core 6th Grade Answers Topic 1 Use Positive Rational Numbers 26 8\(\frac{1}{5}\) ÷ 6\(\frac{3}{20}\) = \(\frac{3}{4}\)
Answer:
The given multiplication equation is:
\(\frac{3}{4}\) × 8\(\frac{1}{5}\) = 6\(\frac{3}{20}\)
Now,
Divide the above equation with 8\(\frac{1}{5}\)
So,
We will get
6\(\frac{3}{20}\) ÷ 8\(\frac{1}{5}\) = \(\frac{3}{4}\)
Hence, from the above,
We can conclude that
The equations that are equivalent to \(\frac{3}{4}\) × 8\(\frac{1}{5}\) = 6\(\frac{3}{20}\) are:

Topic 1 MID-TOPIC CHECKPOINT

Question 1.
Vocabulary How can you use a compatible number to estimate a quotient when dividing a decimal by a whole number? Lesson 1.2
Answer:
When dividing a decimal by a whole number, we use the following procedure:
a. Estimate the quotient.
b. Perform the division. Remember to place a zero in the quotient when the divisor is larger than the dividend.
c. Compare your estimate with your quotient to verify that the answer makes sense.

Question 2.
Keaton is building a rectangular tabletop and wants to put a metal border around the edge. The length of the tabletop is 1.83 meters and the width is 0.74 meters. Use the formula P = 2l + 2w to find the perimeter of the tabletop. Lesson 1.1
Answer:
It is given that
Keaton is building a rectangular tabletop and wants to put a metal border around the edge. The length of the tabletop is 1.83 meters and the width is 0.74 meters
So,
To find the length of the metal border around the edge,
We have to find the perimeter of the tabletop
Now,
From the give information,
We can observe that
The perimeter of the tabletop (P) = 2l + 2w
Where,
‘l’ is the length of the tabletop
‘w’ is the width of the tabletop
So,
The perimeter of the tabletop (P) = (2 × 1.83) + (2 × 0.74)
= 3.66 + 1.48
= 5.14 meters
Hence, from the above,
We can conclude that
The perimeter of thetabletop is: 5.14 meters

Question 3.
Norbert’s Nursery is having a sale. Flats of flowers are priced as marked, including tax. Jake buys 2 flats of petunias, 3 flats of daisies, and 1 flat of begonias. If he pays with a $50 bill, how much change should Jake receive? Lesson 1.1
Envision Math Common Core 6th Grade Answers Topic 1 Use Positive Rational Numbers 21.5
Answer:
It is given that
Norbert’s Nursery is having a sale. Flats of flowers are priced as marked, including tax. Jake buys 2 flats of petunias, 3 flats of daisies, and 1 flat of begonias. If he pays with a $50 bill
Now,
The given table is:
Envision Math Common Core 6th Grade Answers Topic 1 Use Positive Rational Numbers 21.5
Now,
The total price of flats of flowers Jack bought = (2 × The price of each flat of Petunia) + (3 × The price of each flat of daisies) + (1 × The price of each flat of begonias)
= (2 × $5.25) + (3 × $7.65) + (1 × $8.40)
= $10.5 + $22.95 + $8.40
= $41.85
Now,
The amount of change should Jack receive = (The total amount of money Jack gave) – (The total price of the flowers Jack bought)
= $50 – $41.85
= $8.15
Hence, from the above,
We can conclude that
The amount of change should Jack receive is: $8.15

Question 4.
Marguerite is selling space in an advertisement book for a community fund-raising event. Each \(\frac{1}{4}\) page in the book costs $15.50. What is the cost for \(\frac{3}{4}\) page? Lesson 1.1
A. $62.00
B. $46.50
C. $20.67
D. $11.63
Answer:
It is given that
Marguerite is selling space in an advertisement book for a community fund-raising event. Each \(\frac{1}{4}\) page in the book costs $15.50
So,
The cost for \(\frac{3}{4}\) page = 3 × (The cost of \(\frac{1}{4}\) page)
= 3 × $15.50
= $46.50
Hence, from the above,
We can conclude that
The cost for \(\frac{3}{4}\) page is:

Question 5.
What is the value of 170 ÷ (4 × 5)? Lesson 1.2
Answer:
The given division expression is: 170 ÷ (4 × 5)
So,
170 ÷ (4 × 5)
= 170 ÷ 20
Now,
By using the Long Division,

Hence, from the above,
We can conclude that
170 ÷ 20 = 8.5

Question 6.
Lucia walks 2\(\frac{3}{4}\) miles on Monday. On Monday, she walks 1\(\frac{1}{2}\) times farther than on Tuesday. Which equation can be used to find how far Lucia walks on Tuesday? Lesson 1-3
A. 2\(\frac{3}{4}\) × 1\(\frac{1}{2}\) = 4\(\frac{1}{8}\)
B. 2\(\frac{3}{4}\) × 1\(\frac{1}{2}\) = 4\(\frac{1}{4}\)
C. 2\(\frac{3}{4}\) ÷ 1\(\frac{1}{2}\) = 1\(\frac{5}{6}\)
D. 1\(\frac{1}{2}\) ÷ 2\(\frac{3}{4}\) = \(\frac{6}{11}\)
Answer:
It is given that
Lucia walks 2\(\frac{3}{4}\) miles on Monday. On Monday, she walks 1\(\frac{1}{2}\) times farther than on Tuesday
So,
The distance Lucia walked on Tuesday = (The distance Lucia walked on Monday) + 1\(\frac{1}{2}\)
= 2\(\frac{3}{4}\) + 1\(\frac{1}{2}\)
Now,
By using the Long Addition,

Hence, from the above,
We can conclude that
The equation that can be used to find how far Lucia walks on Tuesday is:

How well did you do on the mid-topic checkpoint? Fill in the stars. Envision Math Common Core 6th Grade Answers Topic 1 Use Positive Rational Numbers 28

Topic 1 MID-TOPIC PERFORMANCE TASK

Nyan Robotics Team received their challenge for the year and has to buy parts to build their robot for competitions.

Envision Math Common Core 6th Grade Answers Topic 1 Use Positive Rational Numbers 28.1

PART A
Team members Eric and Natalia secure a grant for $75.00 to buy beams and channels. If the team needs 3 beams and 6 channels, will the grant cover the cost? If so, how much of the grant will remain?
Answer:
It is given that
Team members Eric and Natalia secure a grant for $75.00 to buy beams and channels and the team needs 3 beams and 6 channels
Now,
The given table is;
Envision Math Common Core 6th Grade Answers Topic 1 Use Positive Rational Numbers 28.1
So,
The cost of 3 beams and 6 channels = (3 × The cost of each Beam) + (6 × The cost of each Channel)
= (3 × $5.95) + (6 × $8.50)
= $17.85 + $51
= $68.85
So,
The grant received > The cost of 3 beams and 6 channels
Now,
The amount of the grant that will remain = (The total grant granted) – (The cost of 3 beams and 6 channels)
= $75 – $68.85
= $6.15
Hence, from the above,
We can conclude that
The amount of grant that will remain is: $6.15

PART B
Team members Corinne, Kevin, and Tomas decide to share the cost of 2 motor controllers and 4 wheels equally. How much does each member need to contribute?
Answer:
It is given that
Team members Corinne, Kevin, and Tomas decide to share the cost of 2 motor controllers and 4 wheels equally
Now,
The given table is:
Envision Math Common Core 6th Grade Answers Topic 1 Use Positive Rational Numbers 28.1
So,
The total cost of 2 motor controllers and 4 wheels = (2 × The cost of each motor controller) + (4 × The cost of each wheel)
= (2 × $99.75) + (4 × $18.90)
= $199.5 + $75.6
= $275.1
Now,
The amount of money each member needs to contribute to buy 2 motor controllers and 4 wheels = $275.1 ÷ 3
Now,
By using the Long Division,

Hence, from the above,
We can conclude that
The amount of money each member needs to contribute to buy 2 motor controllers and 4 wheels is: $91.70

PART C
Nyan Robotics has a budget of $99 to buy sprockets, axles, and gears. If they spend \(\frac{2}{3}\) of the budget on sprockets, how much money from the budget remains to buy axles and gears?
Answer:
It is given that
Nyan Robotics has a budget of $99 to buy sprockets, axles, and gears and they spend \(\frac{2}{3}\) of the budget on sprockets
Now,
The given table is:
Envision Math Common Core 6th Grade Answers Topic 1 Use Positive Rational Numbers 28.1
Now,
The amount of budget remaining after they spent on sprockets to buy axles and gears = \(\frac{2}{3}\) × $99
Now,
By using the Long Multiplication,

Hence, from the above,
We can conclude that
The amount of budget remaining after they spent on sprockets to buy axles and gears is: $66

3-Act Mathematical Modeling: Stocking Up

3-ACT MATH

Stocking UP

ACT 1

Question 1.
After watching the video, what is the first question that comes to mind?
Answer:

Question 2.
Write the Main Question you will answer.
Answer:

Question 3.
Construct Arguments Predict an answer to this Main Question. Explain your prediction.
Envision Math Common Core 6th Grade Answers Topic 1 Use Positive Rational Numbers 26.1
Answer:

Question 4.
On the number line below, write a number that is too small to be the answer. Write a number that is too large.
Envision Math Common Core 6th Grade Answers Topic 1 Use Positive Rational Numbers 26.2
Answer:

Question 5.
Plot your prediction on the same number line.
Answer:

ACT 2

Question 6.
What information in this situation would be helpful to know? How would you use that information?
Envision Math Common Core 6th Grade Answers Topic 1 Use Positive Rational Numbers 26.5
Answer:

Question 7.
Use Appropriate Tools What tools can you use to solve the problem? Explain how you would use them strategically.
Answer:

Question 8.
Model with Math Represent the situation using mathematics. Use your representation to answer the Main Question.
Answer:

Question 9.
What is your answer to the Main Question? Is it higher or lower than your prediction? Explain why.
Envision Math Common Core 6th Grade Answers Topic 1 Use Positive Rational Numbers 26.6
Answer:

ACT 3

Question 10.
Write the answer you saw in the video.
Answer:

Question 11.
Reasoning Does your answer match the answer in the video? If not, what are some reasons that would explain the difference?
Envision Math Common Core 6th Grade Answers Topic 1 Use Positive Rational Numbers 26.7
Answer:

Question 12.
Make Sense and Persevere Would you change your model now that you know the answer? Explain.
Envision Math Common Core 6th Grade Answers Topic 1 Use Positive Rational Numbers 26.8
Answer:

Reflect

Question 13.
Model with Math Explain how you used a mathematical model to represent the situation. How did the model help you answer the Main Question?
Envision Math Common Core 6th Grade Answers Topic 1 Use Positive Rational Numbers 26.9
Answer:

Question 14.
Reasoning How did you represent the situation using symbols? How did you use those symbols to solve the problem?
Answer:

SEQUEL

Question 15.
Model with Math
The store purchases boxes of pasta for $0.82 and cans of sauce for $1.62. How much profit does the store make from this purchase?
Answer:

Lesson 1.4 Understand Division with Fractions

ACTIVITY

Explore It!
Students are competing in a 4-kilometer relay race. There are 10 runners.
Envision Math Common Core 6th Grade Answers Topic 1 Use Positive Rational Numbers 30.1

A. Use the number line to represent the data for the race.
Envision Math Common Core 6th Grade Answers Topic 1 Use Positive Rational Numbers 30.2
Answer:
It is given that
Students are competing in a 4-kilometer relay race. There are 10 runners.
Now,
The given data is:
Envision Math Common Core 6th Grade Answers Topic 1 Use Positive Rational Numbers 30.1
So,
From the given number line,
We can observe that
There are 10 runners and each runner runs \(\frac{2}{5}\) km and the finish line is at 4 km
Hence,
The representation of the given data in a number line is:

B. Use multiplication or division to describe your work on the number line.
Answer:
It is given that
Students are competing in a 4-kilometer relay race. There are 10 runners.
Now,
The given data is:
Envision Math Common Core 6th Grade Answers Topic 1 Use Positive Rational Numbers 30.1
Now,
Using Multiplication,
The total distance covered by 10 runners = 10 × (The distance each runner runs)
= 10 × \(\frac{2}{5}\)
Now,
By using the Long Multiplication,

So,
The total distance covered by 10 runners is: 4 km

Focus on math practices
Model with Math
Describe what a number line would look like if there were 10 runners each running; kilometer in a 5-kilometer
race.
Answer:
It is given that
There were 10 runners each running; kilometer in a 5-kilometer race
So,
The total distance covered by 10 runners = 10 × (The distance each runner runs)
= 10 × \(\frac{1}{2}\)
Now,
By using the Long Multiplication,

So,
The total distance covered by 10 runners is: 5 km
Hence,
The representation of the given information in a number line is:

? Essential Question How can you represent the division of fractions?
Answer:
Interpret a fraction as a division of the numerator by the denominator (\(\frac{a}{b}\) = a ÷ b). Solve word problems involving the division of whole numbers leading to answers in the form of fractions or mixed numbers,
Ex: By using visual fraction models or equations to represent the problem.

Try It!
A board is 6 feet long. How many \(\frac{2}{3}\)-foot-long pieces can be cut from the board? Use the number line to show your work.
Envision Math Common Core 6th Grade Answers Topic 1 Use Positive Rational Numbers 102
Answer:
It is given that
A board is 6 feet long
So,
The number of \(\frac{2}{3}\)-foot-long pieces can be cut from the board = (The length of the board) ÷ \(\frac{2}{3}\)
= 6 ÷ \(\frac{2}{3}\)
= 6 × \(\frac{3}{2}\)
Now,
By using the Long Multiplication,

Hence, from the above,
We can conclude that
The number of \(\frac{2}{3}\)-foot-long pieces that can be cut from the board are: 9 pieces

Convince Me! Why is the number of pieces that can be cut from the board greater than the number of feet in the length of the board?
Answer:

Try It!
Make a diagram to find \(\frac{2}{3}\) ÷ 4.
Answer:
The given division expression is: \(\frac{2}{3}\) ÷ 4
Hence,
The representation of the given division expression in the form of an area model is:

Hence, from the above,
We can conclude that
\(\frac{2}{3}\) ÷ 4 = \(\frac{1}{6}\)

Try It!
Use the pattern in the table above to find 8 ÷ \(\frac{3}{4}\).
Envision Math Common Core 6th Grade Answers Topic 1 Use Positive Rational Numbers 31.2
Answer:
The given division expression is: 8 ÷ \(\frac{3}{4}\)
So,
8 ÷ \(\frac{3}{4}\)
= 8 × \(\frac{4}{3}\)
Now,
By using the Long Multiplication,

Hence, from the above,
We can conclude that

KEY CONCEPT

To divide a whole number by a fraction:
Envision Math Common Core 6th Grade Answers Topic 1 Use Positive Rational Numbers 31.3
To divide a fraction by a whole number:
Envision Math Common Core 6th Grade Answers Topic 1 Use Positive Rational Numbers 31.4

Do You Understand?

Question 1.
? Essential Question How can you represent the division of fractions?
Answer:
Interpret a fraction as a division of the numerator by the denominator (\(\frac{a}{b}\) = a ÷ b). Solve word problems involving the division of whole numbers leading to answers in the form of fractions or mixed numbers,
Ex: By using visual fraction models or equations to represent the problem.

Question 2.
Reasoning Draw a diagram to represent 8 ÷ \(\frac{2}{3}\). Then write an equation to show the solution.
Answer:

Question 3.
Reasoning is 4 ÷ \(\frac{3}{2}\) the same as 4 ÷ \(\frac{2}{3}\)? Explain.
Answer:
The given division expressions are:
4 ÷ \(\frac{3}{2}\) and 4 ÷ \(\frac{2}{3}\)
So,
4 ÷ \(\frac{3}{2}\)
= 4 × \(\frac{2}{3}\)
So,
4 ÷ \(\frac{2}{3}\)
= 4 × \(\frac{3}{2}\)
Hence, from the above,
We can conclude that
4 ÷ \(\frac{3}{2}\) and 4 ÷ \(\frac{3}{2}\) are not the same

Question 4.
How can you write any nonzero whole number as a fraction?
Answer:
You can multiply the numerator and the denominator of a fraction by any nonzero whole number, as long as you multiply both by the same whole number

Question 5.
Look for Relationships How does the quotient compare to the dividend when the divisor is a fraction less than 1?
Answer:
When the divisor is less than 1, the quotient is larger than the dividend. Decreasing the divisor to \(\frac{1}{2}\) increases the quotient to \(\frac{1}{2}\). When the divisor is smaller than the dividend, the quotient is more than 1.

Question 6.
What division equation is represented by the diagram?
Envision Math Common Core 6th Grade Answers Topic 1 Use Positive Rational Numbers 33.1
Answer:
The given diagram is:
Envision Math Common Core 6th Grade Answers Topic 1 Use Positive Rational Numbers 33.1
Now,
From the given diagram,
We can observe that
The total number of equal parts are: 8
The number of shaded parts are: 6
So,
The division equation that is represented by the given diagram = (The number of shaded parts) ÷ (The total number of equal parts)
= \(\frac{6}{8}\)
= \(\frac{6 ÷ 2}{8 ÷ 2}\)
= \(\frac{3}{4}\)
Hence, from the above,
We can conclude that
The division equation that is represented by the given diagram is: \(\frac{3}{4}\)

Do You Know How?

In 7-14, find each reciprocal.

Question 7.
\(\frac{3}{5}\)
Answer:
We know that,
Two numbers whose product is “1” are called “Reciprocals” of each other. If a nonzero number is named as a fraction \(\frac{a}{b}\), then its reciprocal is \(\frac{b}{a}\)
Hence, from the above,
We can conclude that
The reciprocal of \(\frac{3}{5}\) is: \(\frac{5}{3}\)

Question 8.
\(\frac{1}{6}\)
Answer:
We know that,
Two numbers whose product is “1” are called “Reciprocals” of each other. If a nonzero number is named as a fraction \(\frac{a}{b}\), then its reciprocal is \(\frac{b}{a}\)
Hence, from the above,
We can conclude that
The reciprocal of \(\frac{1}{6}\) is: \(\frac{6}{1}\)

Question 9.
9
Answer:
We know that,
Two numbers whose product is “1” are called “Reciprocals” of each other. If a nonzero number is named as a fraction \(\frac{a}{b}\), then its reciprocal is \(\frac{b}{a}\)
Hence, from the above,
We can conclude that
The reciprocal of 9 is: \(\frac{1}{9}\)

Question 10.
\(\frac{7}{4}\)
Answer:
We know that,
Two numbers whose product is “1” are called “Reciprocals” of each other. If a nonzero number is named as a fraction \(\frac{a}{b}\), then its reciprocal is \(\frac{b}{a}\)
Hence, from the above,
We can conclude that
The reciprocal of \(\frac{7}{4}\) is: \(\frac{4}{7}\)

Question 11.
\(\frac{5}{8}\)
Answer:
We know that,
Two numbers whose product is “1” are called “Reciprocals” of each other. If a nonzero number is named as a fraction \(\frac{a}{b}\), then its reciprocal is \(\frac{b}{a}\)
Hence, from the above,
We can conclude that
The reciprocal of \(\frac{5}{8}\) is: \(\frac{8}{5}\)

Question 12.
16
Answer:
We know that,
Two numbers whose product is “1” are called “Reciprocals” of each other. If a nonzero number is named as a fraction \(\frac{a}{b}\), then its reciprocal is \(\frac{b}{a}\)
Hence, from the above,
We can conclude that
The reciprocal of 16 is: \(\frac{1}{16}\)

Question 13.
\(\frac{7}{12}\)
Answer:
We know that,
Two numbers whose product is “1” are called “Reciprocals” of each other. If a nonzero number is named as a fraction \(\frac{a}{b}\), then its reciprocal is \(\frac{b}{a}\)
Hence, from the above,
We can conclude that
The reciprocal of \(\frac{7}{12}\) is: \(\frac{12}{7}\)

Question 14.
\(\frac{11}{5}\)
Answer:
We know that,
Two numbers whose product is “1” are called “Reciprocals” of each other. If a nonzero number is named as a fraction \(\frac{a}{b}\), then its reciprocal is \(\frac{b}{a}\)
Hence, from the above,
We can conclude that
The reciprocal of \(\frac{11}{5}\) is: \(\frac{5}{11}\)

In 15-22, find each quotient.

Question 15.
6 ÷ \(\frac{2}{3}\)
Answer:
The given division equation is: 6 ÷ \(\frac{2}{3}\)
So,
By using the Long Division,

Hence, from the above,
We can conclude that
6 ÷ \(\frac{2}{3}\) = 9
The quotient for the given division equation is: 9

Question 16.
12 ÷ \(\frac{3}{8}\)
Answer:
The given division equation is: 12 ÷ \(\frac{3}{8}\)
So,
By using the Long Division,

Hence, from the above,
We can conclude that
12 ÷ \(\frac{3}{8}\) = 32
The quotient for the given division equation is: 32

Question 17.
\(\frac{1}{4}\) ÷ 3
Answer:
The given division equation is: \(\frac{1}{4}\) ÷ 3
So,
By using the Long Division,

Hence, from the above,
We can conclude that
\(\frac{1}{4}\) ÷ 3 = \(\frac{1}{12}\)
The quotient for the given division equation is: \(\frac{1}{12}\)

Question 18.
\(\frac{2}{5}\) ÷ 2
Answer:
The given division equation is: \(\frac{2}{5}\) ÷ 2
So,
By using the Long Division,

Hence, from the above,
We can conclude that
\(\frac{2}{5}\) ÷ 2 = \(\frac{1}{5}\)
The quotient for the given division equation is: \(\frac{1}{5}\)

Question 19.
2 ÷ \(\frac{1}{2}\)
Answer:
The given division equation is: 2 ÷ \(\frac{1}{2}\)
So,
By using the Long Division,

Hence, from the above,
We can conclude that
2 ÷ \(\frac{1}{2}\) = 4
The quotient for the given division equation is: 4

Question 20.
3 ÷ \(\frac{1}{4}\)
Answer:
The given division equation is: 2 ÷ \(\frac{1}{2}\)
So,
By using the Long Division,

Hence, from the above,
We can conclude that
2 ÷ \(\frac{1}{2}\) = 4
The quotient for the given division equation is: 4

Question 21.
9 ÷ \(\frac{3}{5}\)
Answer:
The given division equation is: 9 ÷ \(\frac{3}{5}\)
So,
By using the Long Division,

Hence, from the above,
We can conclude that
9 ÷ \(\frac{3}{5}\) = 15
The quotient for the given division equation is: 15

Question 22.
5 ÷ \(\frac{2}{7}\)
Answer:
The given division equation is: 5 ÷ \(\frac{2}{7}\)
So,
By using the Long Division,

Hence, from the above,
We can conclude that
5 ÷ \(\frac{2}{7}\) = \(\frac{35}{2}\)
The quotient for the given division equation is: \(\frac{35}{2}\)

Practice & Problem Solving

Scan for Multimedia

Leveled Practice In 23 and 24, complete each division sentence.

Question 23.
Envision Math Common Core 6th Grade Answers Topic 1 Use Positive Rational Numbers 40.1
The number line shows 6 wholes.
Answer:
The given number line is:

Hence, from the above,
We can conclude that
The completed division sentence is:

Question 24.
Envision Math Common Core 6th Grade Answers Topic 1 Use Positive Rational Numbers 40.2
Answer:
The given diagram is:

Hence, from the above,
We can conclude that
The completed division sentence is:

In 25 and 26, find each quotient. Draw a diagram to help.

Question 25.
\(\frac{3}{5}\) ÷ 3
Answer:
The given division equation is:
\(\frac{3}{5}\) ÷ 3
Now,
The representation of the given division equation in the form of an area model is:

Now,
By using the Long Division,

Hence, from the above,
We can conclude that
\(\frac{3}{5}\) ÷ 3 = \(\frac{1}{5}\)
The quotient for the given division equation is: \(\frac{1}{5}\)

Question 26.
2 ÷ \(\frac{2}{5}\)
Answer:
The given division equation is:
2 ÷ \(\frac{2}{5}\)
Now,
The representation of the given division equation in the form of an area model is:

Now,
By using the Long Division,

Hence, from the above,
We can conclude that
2 ÷ \(\frac{2}{5}\) = 5
The quotient for the given division equation is: 5

In 27-30, find each reciprocal.

Question 27.
\(\frac{3}{10}\)
Answer:
We know that,
Two numbers whose product is “1” are called “Reciprocals” of each other. If a nonzero number is named as a fraction \(\frac{a}{b}\), then its reciprocal is \(\frac{b}{a}\)
Hence, from the above,
We can conclude that
The reciprocal of \(\frac{3}{10}\) is: \(\frac{10}{3}\)

Question 28.
6
Answer:
We know that,
Two numbers whose product is “1” are called “Reciprocals” of each other. If a nonzero number is named as a fraction \(\frac{a}{b}\), then its reciprocal is \(\frac{b}{a}\)
Hence, from the above,
We can conclude that
The reciprocal of 6 is: \(\frac{1}{6}\)

Question 29.
\(\frac{1}{15}\)
Answer:
We know that,
Two numbers whose product is “1” are called “Reciprocals” of each other. If a nonzero number is named as a fraction \(\frac{a}{b}\), then its reciprocal is \(\frac{b}{a}\)
Hence, from the above,
We can conclude that
The reciprocal of \(\frac{1}{15}\) is: 15

Question 30.
3
Answer:
We know that,
Two numbers whose product is “1” are called “Reciprocals” of each other. If a nonzero number is named as a fraction \(\frac{a}{b}\), then its reciprocal is \(\frac{b}{a}\)
Hence, from the above,
We can conclude that
The reciprocal of 3 is: \(\frac{1}{3}\)

In 31-38, find each quotient.

Question 31.
36 ÷ \(\frac{3}{4}\)
Answer:
The given division equation is: 36 ÷ \(\frac{3}{4}\)
So,
By using the Long Division,

Hence, from the above,
We can conclude that
36 ÷ \(\frac{3}{4}\) = 48
The quotient for the given division equation is: 48

Question 32.
2 ÷ \(\frac{3}{8}\)
Answer:
The given division equation is: 2 ÷ \(\frac{3}{8}\)
So,
By using the Long Division,

Hence, from the above,
We can conclude that
2 ÷ \(\frac{3}{8}\) = \(\frac{16}{3}\)
The quotient for the given division equation is: \(\frac{16}{3}\)

Question 33.
18 ÷ \(\frac{2}{3}\)
Answer:
The given division equation is: 18 ÷ \(\frac{2}{3}\)
So,
By using the Long Division,

Hence, from the above,
We can conclude that
18 ÷ \(\frac{2}{3}\) = 27
The quotient for the given division equation is: 27

Question 34.
9 ÷ \(\frac{4}{5}\)
Answer:
The given division equation is: 9 ÷ \(\frac{4}{5}\)
So,
By using the Long Division,

Hence, from the above,
We can conclude that
9 ÷ \(\frac{4}{5}\) = \(\frac{45}{4}\)
The quotient for the given division equation is: \(\frac{45}{4}\)

Question 35.
\(\frac{1}{6}\) ÷ 2
Answer:
The given division equation is: \(\frac{1}{6}\) ÷ 2
So,
By using the Long Division,

Hence, from the above,
We can conclude that
\(\frac{1}{6}\) ÷ 2 = \(\frac{1}{12}\)
The quotient for the given division equation is: \(\frac{1}{12}\)

Question 36.
\(\frac{2}{3}\) ÷ 3
Answer:
The given division equation is: \(\frac{2}{3}\) ÷ 3
So,
By using the Long Division,

Hence, from the above,
We can conclude that
\(\frac{2}{3}\) ÷ 3 = \(\frac{2}{9}\)
The quotient for the given division equation is: \(\frac{2}{9}\)

Question 37.
\(\frac{3}{5}\) ÷ 2
Answer:
The given division equation is: \(\frac{3}{5}\) ÷ 2
So,
By using the Long Division,

Hence, from the above,
We can conclude that
\(\frac{3}{5}\) ÷ 2 = \(\frac{3}{10}\)
The quotient for the given division equation is: \(\frac{3}{10}\)

Question 38.
\(\frac{1}{4}\) ÷ 4
Answer:
The given division equation is: \(\frac{1}{4}\) ÷ 4
So,
By using the Long Division,

Hence, from the above,
We can conclude that
\(\frac{1}{4}\) ÷ 4 = \(\frac{1}{16}\)
The quotient for the given division equation is: \(\frac{1}{16}\)

Question 39.
A worker is pouring 3 quarts of liquid into \(\frac{3}{8}\)-quart containers. How many of the containers can she fill? Write and solve a division equation.
Envision Math Common Core 6th Grade Answers Topic 1 Use Positive Rational Numbers 49.1
Answer:
It is given that
A worker is pouring 3 quarts of liquid into \(\frac{3}{8}\)-quart containers
So,
The number of containers she can fill = (The number of quarts of liquid a worker poured) ÷ (The size of each container)
= 3 ÷ \(\frac{3}{8}\)
Now,
By using the Long Division,

In 40-43, use the given information.

Envision Math Common Core 6th Grade Answers Topic 1 Use Positive Rational Numbers 40.3

Question 40.
Higher-Order Thinking Without doing any calculations, how can you use the information given to tell which animal moves the fastest?
Answer:
The given information is:
Envision Math Common Core 6th Grade Answers Topic 1 Use Positive Rational Numbers 40.3
Now,
The distance covered by a snail in 1 hour = 120 ÷ \(\frac{3}{4}\)
So,
By using the Long Division,

So,
The distance covered by a snail in 1 hour is: 160 ft
Now,
The distance covered by a tortoise in 1 hour = 600 ÷ \(\frac{2}{3}\)
So,
By using the Long Division,

So,
The distance covered by a tortoise in 1 hour is: 900 ft
Now,
The distance covered by a Sloth in 1 hour = 250 ÷ \(\frac{5}{8}\)
So,
By using the Long Division,

So,
The distance covered by a Sloth in 1 hour is: 400 ft
Hence, from the above,
We can conclude that
The animal which moves the fastest is: Tortoise

Question 41.
Reasoning The quotient 250 ÷ \(\frac{5}{8}\) tells about how far a sloth may move in one hour. How far can a sloth go in 90 minutes? Justify your reasoning.
Answer:
It is given that
The quotient 250 ÷ \(\frac{5}{8}\) tells about how far a sloth may move in one hour
Now,
We know that,
1 hour = 60 minutes
1\(\frac{1}{2}\) hour = 90 minutes
So,
The representation of 1\(\frac{1}{2}\) into a fraction is:

So,
The distance covered by a Sloth in 90 minutes = 250 ÷ \(\frac{3}{2}\)
Now,
By using the Long Division,

Hence, from the above,
We can conclude that
The distance covered by a Sloth in 90 minutes is: \(\frac{500}{3}\) ft

Question 42.
The quotient 600 ÷ \(\frac{2}{3}\) tells about how far a tortoise may move in one hour. Find that distance.
Answer:
It is given that
The quotient 600 ÷ \(\frac{2}{3}\) tells about how far a tortoise may move in one hour
Now,
According to the given information,
The distance covered by a tortoise in 1 hour = 600 ÷ \(\frac{2}{3}\)
So,
By using the Long Division,

Hence, from the above,
We can conclude that
The distance covered by a tortoise in 1 hour is: 900 ft

Question 43.
Write and solve an equation to find how far a snail can go in one hour.
Answer:
The given information is:
Envision Math Common Core 6th Grade Answers Topic 1 Use Positive Rational Numbers 40.3
Now,
The distance covered by a snail in 1 hour = 120 ÷ \(\frac{3}{4}\)
So,
By using the Long Division,

Hence, from the above,
We can conclude that
The distance covered by a snail in 1 hour is: 160 ft

Question 44.
A waitress pours \(\frac{3}{4}\) gallon of orange juice equally into 5 pitchers. What fraction of a gallon of orange juice is in each pitcher? Use the rectangle to represent the problem. Then write an equation to show the solution.
Envision Math Common Core Grade 6 Answers Topic 1 Use Positive Rational Numbers 41.3
Answer:
It is given that
A waitress pours \(\frac{3}{4}\) gallon of orange juice equally into 5 pitchers.
Now,
The representation of the given information in the form of an Area model is:

Now,
Let the fraction of a gallon of orange juice that is present in each pitcher be x
So,
According to the given information,
5 ÷ x = \(\frac{3}{4}\)
So,
x = 5 ÷ \(\frac{3}{4}\)
Now,
By using the Long Division,

Hence, from the above,
We can conclude that
The fraction of a gallon of orange juice that is present in each pitcher is: \(\frac{20}{3}\) gallons

Assessment Practice

Question 45.
Select all the math statements that have the same quotient.
Envision Math Common Core Grade 6 Answers Topic 1 Use Positive Rational Numbers 54 12 ÷ \(\frac{2}{3}\)
Envision Math Common Core Grade 6 Answers Topic 1 Use Positive Rational Numbers 54 \(\frac{2}{3}\) ÷ \(\frac{1}{27}\)
Envision Math Common Core Grade 6 Answers Topic 1 Use Positive Rational Numbers 54 16 ÷ \(\frac{4}{5}\)
Envision Math Common Core Grade 6 Answers Topic 1 Use Positive Rational Numbers 54 12 ÷ \(\frac{3}{2}\)
Envision Math Common Core Grade 6 Answers Topic 1 Use Positive Rational Numbers 54 24 ÷ \(\frac{4}{3}\)
Answer:
The given math statements and their quotients are:
a. 12 ÷ \(\frac{2}{3}\)
Now,
By using the Long Division,

b. \(\frac{2}{3}\) ÷ \(\frac{1}{27}\)
Now,
By using the Long Division,

c. 16 ÷ \(\frac{4}{5}\)
Now,
By using the Long Division,

d. 12 ÷ \(\frac{3}{2}\)
Now,
By using the Long Division,

e. 24 ÷ \(\frac{4}{3}\)
Now,
By using the Long Division,

Hence, from the above,
We can conclude that
All the math statements that have the same quotient are:

Question 46.
Select all the math statements that are true.
Envision Math Common Core Grade 6 Answers Topic 1 Use Positive Rational Numbers 55.1
Answer:
All the math statements that are true are:

Lesson 1.5 Divide Fractions by Fractions

ACTIVITY

Solve & Discuss It!

A granola bar was cut into 6 equal pieces. Someone ate part of the granola bar so that \(\frac{2}{3}\) of the original bar remains. How many \(\frac{1}{6}\) parts are left? Use the picture to draw a model to represent and find \(\frac{2}{3}\) ÷ \(\frac{1}{6}\)

Envision Math Common Core Grade 6 Answers Topic 1 Use Positive Rational Numbers 55.2
Answer:
It is given that
A granola bar was cut into 6 equal pieces. Someone ate part of the granola bar so that \(\frac{2}{3}\) of the original bar remains
Now,
The given picture is:

Now,
The representation of the given information in the form of a bar diagram is:

Now,
The number of \(\frac{1}{6}\) parts that are left = (The part that is remained in the original granola bar) ÷ \(\frac{1}{6}\)
= \(\frac{2}{3}\) ÷ \(\frac{1}{6}\)
Now,
By using the Long Division,

Hence, from the above,
We can conclude that
The number of \(\frac{1}{6}\) parts that are left is: 4

A model with Math
You can model with math by dividing a whole into equal parts.
Answer:
The representation of the above problem in the form of a bar diagram is:

Focus on math practices
Use Structure How can you use multiplication to check your answer?
Answer:
You can use multiplication to check your division answer the following way:
a. Do the division problem.
b. Multiply the quotient times the divisor.
c. If there is a remainder, add it to the multiplication product.
d. Compare this answer to the dividend. They should be the same number

? Essential Question How can you divide a fraction by a fraction?
Answer:
The easiest way to divide fractions is to follow three simple steps:
a. Flip the divisor into a reciprocal.
b. Change the division sign into a multiplication sign and multiply.
c. Simplify if possible.

Try It!
Use the number line below to represent \(\frac{1}{6}\) × 3 = \(\frac{1}{2}\). Then write an equivalent division sentence.
Envision Math Common Core Grade 6 Answers Topic 1 Use Positive Rational Numbers 55.3
Answer:
The given multiplication equation is:
\(\frac{1}{6}\) × 3 = \(\frac{1}{2}\)
So,
The representation of the given division equation in the given number line is:

Now,
The representation of the given multiplication equation into division equation is:
\(\frac{1}{2}\) ÷ \(\frac{1}{6}\) = 3
Hence, from the above,
We can conclude that
The equivalent division equation for the given multiplication equation is:
\(\frac{1}{2}\) ÷ \(\frac{1}{6}\) = 3

Convince Me! How are the dividend, divisor, and quotient represented on the number line?
Answer:
In a number line,
The dividend represents the “Product”
The divisor represents the “The division of the dividend into equal parts”
The quotient represents the “The number of times the divisor divided”

Try It!
Find \(\frac{1}{4}\) ÷ \(\frac{3}{8}\). Draw an area model.
Answer:
The given division expression is: \(\frac{1}{4}\) ÷ \(\frac{3}{8}\)
Hence,
The representation of the given division expression by using the area model is:

Try It!
How wide is a rectangular strip of land with a length of \(\frac{3}{4}\) mile and an area of \(\frac{1}{2}\) square mile? Use the area formula: A = l × w.
Envision Math Common Core Grade 6 Answers Topic 1 Use Positive Rational Numbers 55.4
Answer:
It is given that
The length of a rectangular strip is \(\frac{3}{4}\) mile and an area of a rectangular strip is \(\frac{1}{2}\) square mile
Now,
It is given that
Area (A) = Length (L) × Width (W)
So,
\(\frac{1}{2}\) = \(\frac{3}{4}\) × Width (W)
Width (W) = \(\frac{1}{2}\) ÷ \(\frac{3}{4}\)
Now,
By using the Long Division,

Hence, from the above,
We can conclude that

KEY CONCEPT

To divide a fraction by a fraction, rewrite the division equation as a multiplication equation.
Envision Math Common Core Grade 6 Answers Topic 1 Use Positive Rational Numbers 56.1

Do You Understand?

Question 1.
?Essential Question How can you divide a fraction by a fraction?
Answer:
The easiest way to divide fractions is to follow three simple steps:
a. Flip the divisor into a reciprocal.
b. Change the division sign into a multiplication sign and multiply.
c. Simplify if possible.

Question 2.
Critique Reasoning To find the quotient of \(\frac{2}{5}\) ÷ \(\frac{8}{5}\), Corey rewrites the problem as \(\frac{5}{2}\) × \(\frac{8}{5}\). Explain Corey’s mistake and how to correct it.
Answer:
It is given that
To find the quotient of \(\frac{2}{5}\) ÷ \(\frac{8}{5}\), Corey rewrites the problem as \(\frac{5}{2}\) × \(\frac{8}{5}\)
Now,
The given division expression is: \(\frac{2}{5}\) ÷ \(\frac{8}{5}\)
Now,
We know that,
We have to find the reciprocal of a fraction that is present to the right side of the division symbol
So,
\(\frac{2}{5}\) ÷ \(\frac{8}{5}\)
= \(\frac{2}{5}\) × \(\frac{5}{8}\)
Hence, from the above,
We can conclude that
Corey’s mistake is: He found the reciprocal of a fraction that is present to the left side of the division symbol instead of the right side of the division symbol
The correct form of the given division expression is:
\(\frac{2}{5}\) × \(\frac{5}{8}\)

Question 3.
Reasoning is the quotient of \(\frac{3}{5}\) ÷ \(\frac{6}{7}\) greater than or less than \(\frac{3}{5}\)? Explain.
Answer:
The given division expression is: \(\frac{3}{5}\) ÷ \(\frac{6}{7}\)
Now,
The representation of the given division expression in the decimal form is:
\(\frac{3}{5}\) ÷ \(\frac{6}{7}\)
= 0.60 ÷ 0.85
Now,
By using the Long Division,

So,
The representation of the quotient value for the given divison expression in the decimal form is: 0.7
Hence, from the above,
We can conclude that
The quotient of \(\frac{3}{5}\) ÷ \(\frac{6}{7}\) is greater than \(\frac{3}{5}\)

Question 4.
How is dividing a whole number by a fraction different from dividing a fraction by a fraction?
Answer:
The steps to divide a fraction by a Whole Number are:
1. Turn the whole number into a fraction.
2. Multiply the fraction by the whole number fraction.
3. Multiply the tops together and the bottoms together
The steps to divide a fraction by a fraction are:
1. Arrange the equation
2. Keep the first fraction
3. Change the sign
4. Flip the fraction
5. Multiply the tops together and the bottoms together

Do You Know How?

In 5–7, write a division sentence to represent each model.

Question 5.
Envision Math Common Core Grade 6 Answers Topic 1 Use Positive Rational Numbers 56.2
Answer:
The given model is:
Envision Math Common Core Grade 6 Answers Topic 1 Use Positive Rational Numbers 56.2
Now,
From the given model,
We can observe that
The division sentence to represent the given model is:
\(\frac{4}{5}\) ÷ 4
Now,
By using the Long Division,

Hence, from the above,
We can conclude that
The division sentence to represent the given model is:
\(\frac{4}{5}\) ÷ 4

Question 6.
Envision Math Common Core Grade 6 Answers Topic 1 Use Positive Rational Numbers 56.3
Answer:
The given model is:
Envision Math Common Core Grade 6 Answers Topic 1 Use Positive Rational Numbers 56.3
Now,
From the given model,
We can observe that
The division sentence to represent the given model is:
\(\frac{1}{2}\) ÷ \(\frac{2}{3}\)
Now,
By using the Long Division,

Hence, from the above,
We can conclude that
The division sentence to represent the given model is:
\(\frac{1}{2}\) ÷ \(\frac{2}{3}\)

Question 7.
Envision Math Common Core Grade 6 Answers Topic 1 Use Positive Rational Numbers 56.4
Answer:
The given model is:
Envision Math Common Core Grade 6 Answers Topic 1 Use Positive Rational Numbers 56.4
Now,
From the given model,
We can observe that
The division sentence for the given model is:
\(\frac{3}{4}\) ÷ \(\frac{1}{8}\)
Now,
By using the Long Division,

Hence, from the above,
We can conclude that
The division sentence for the given model is:
\(\frac{3}{4}\) ÷ \(\frac{1}{8}\)

In 8-11, find each quotient.

Question 8.
\(\frac{3}{4}\) ÷ \(\frac{2}{3}\)
Answer:
The given division expression is:
\(\frac{3}{4}\) ÷ \(\frac{2}{3}\)
Now,
By using the Long Division,

Hence, from the above,
We can conclude that
\(\frac{3}{4}\) ÷ \(\frac{2}{3}\) = \(\frac{9}{8}\)
The quotient for the given division expression is: \(\frac{9}{8}\)

Question 9.
\(\frac{3}{12}\) ÷ \(\frac{1}{8}\)
Answer:
The given division expression is:
\(\frac{3}{12}\) ÷ \(\frac{1}{8}\)
Now,
By using the Long Division,

Hence, from the above,
We can conclude that
\(\frac{3}{12}\) ÷ \(\frac{1}{8}\) = 2
The quotient for the given division expression is: 2

Question 10.
\(\frac{1}{2}\) ÷ \(\frac{4}{5}\)
Answer:
The given division expression is:
\(\frac{1}{2}\) ÷ \(\frac{4}{5}\)
Now,
By using the Long Division,

Hence, from the above,
We can conclude that
\(\frac{1}{2}\) ÷ \(\frac{4}{5}\) = \(\frac{5}{8}\)
The quotient for the given division expression is: \(\frac{5}{8}\)

Question 11.
\(\frac{7}{10}\) ÷ \(\frac{2}{5}\)
Answer:
The given division expression is:
\(\frac{7}{10}\) ÷ \(\frac{2}{5}\)
Now,
By using the Long Division,

Hence, from the above,
We can conclude that
\(\frac{7}{10}\) ÷ \(\frac{2}{5}\) = \(\frac{7}{4}\)
The quotient for the given division expression is: \(\frac{7}{4}\)

Practice & Problem Solving

Scan for Multimedia

In 12 and 13, complete each division sentence using the models provided.

Question 12.
Envision Math Common Core Grade 6 Answers Topic 1 Use Positive Rational Numbers 56.5
Answer:
The given division expression is:
\(\frac{1}{3}\) ÷ \(\frac{1}{12}\)
Now,
The given model is:

Now,
By using the Long Division,

Hence, from the above,
We can conclude that
\(\frac{1}{3}\) ÷ \(\frac{1}{12}\) = 4

Question 13.
Envision Math Common Core Grade 6 Answers Topic 1 Use Positive Rational Numbers 56.6
Answer:
The given division expression is:
\(\frac{2}{5}\) ÷ \(\frac{1}{10}\)
Now,
The given model is:

Now,
By using the Long Division,

Hence, from the above,
We can conclude that
\(\frac{2}{5}\) ÷ \(\frac{1}{10}\) = 4

In 14-25, find each quotient.

Question 14.
\(\frac{2}{3}\) ÷ \(\frac{1}{3}\)
Answer:
The given division expression is:
\(\frac{2}{3}\) ÷ \(\frac{1}{3}\)
Now,
By using the Long Division,

Hence, from the above,
We can conclude that
\(\frac{2}{3}\) ÷ \(\frac{1}{3}\) = 2
The quotient for the given division expresion is: 2

Question 15.
\(\frac{1}{2}\) ÷ \(\frac{1}{16}\)
Answer:
The given division expression is:
\(\frac{1}{2}\) ÷ \(\frac{1}{16}\)
Now,
By using the Long Division,

Hence, from the above,
We can conclude that
\(\frac{1}{2}\) ÷ \(\frac{1}{16}\) = 8
The quotient for the given division expresion is: 8

Question 16.
\(\frac{1}{4}\) ÷ \(\frac{1}{12}\)
Answer:
The given division expression is:
\(\frac{1}{4}\) ÷ \(\frac{1}{12}\)
Now,
By using the Long Division,

Hence, from the above,
We can conclude that
\(\frac{1}{4}\) ÷ \(\frac{1}{12}\) = 3
The quotient for the given division expresion is: 3

Question 17.
\(\frac{6}{7}\) ÷ \(\frac{3}{7}\)
Answer:
The given division expression is:
\(\frac{6}{7}\) ÷ \(\frac{3}{7}\)
Now,
By using the Long Division,

Hence, from the above,
We can conclude that
\(\frac{6}{7}\) ÷ \(\frac{3}{7}\) = 2
The quotient for the given division expresion is: 2

Question 18.
\(\frac{5}{14}\) ÷ \(\frac{4}{7}\)
Answer:
The given division expression is:
\(\frac{5}{14}\) ÷ \(\frac{4}{7}\)
Now,
By using the Long Division,

Hence, from the above,
We can conclude that
\(\frac{5}{14}\) ÷ \(\frac{4}{7}\) = \(\frac{5}{8}\)
The quotient for the given division expresion is: \(\frac{5}{8}\)

Question 19.
\(\frac{5}{8}\) ÷ \(\frac{1}{2}\)
Answer:
The given division expression is:
\(\frac{5}{8}\) ÷ \(\frac{1}{2}\)
Now,
By using the Long Division,

Hence, from the above,
We can conclude that
\(\frac{5}{8}\) ÷ \(\frac{1}{2}\) = \(\frac{5}{4}\)
The quotient for the given division expresion is: \(\frac{5}{4}\)

Question 20.
\(\frac{7}{12}\) ÷ \(\frac{3}{4}\)
Answer:
The given division expression is:
\(\frac{7}{12}\) ÷ \(\frac{3}{4}\)
Now,
By using the Long Division,

Hence, from the above,
We can conclude that
\(\frac{7}{12}\) ÷ \(\frac{3}{4}\) = \(\frac{7}{9}\)
The quotient for the given division expresion is: \(\frac{7}{9}\)

Question 21.
\(\frac{2}{7}\) ÷ \(\frac{1}{2}\)
Answer:
The given division expression is:
\(\frac{2}{7}\) ÷ \(\frac{1}{2}\)
Now,
By using the Long Division,

Hence, from the above,
We can conclude that
\(\frac{2}{7}\) ÷ \(\frac{1}{2}\) = \(\frac{4}{7}\)
The quotient for the given division expresion is: \(\frac{4}{7}\)

Question 22.
\(\frac{3}{10}\) ÷ \(\frac{3}{5}\)
Answer:
The given division expression is:
\(\frac{3}{10}\) ÷ \(\frac{3}{5}\)
Now,
By using the Long Division,

Hence, from the above,
We can conclude that
\(\frac{3}{10}\) ÷ \(\frac{3}{5}\) = \(\frac{1}{2}\)
The quotient for the given division expresion is: \(\frac{1}{2}\)

Question 23.
\(\frac{7}{12}\) ÷ \(\frac{1}{8}\)
Answer:
The given division expression is:
\(\frac{7}{12}\) ÷ \(\frac{1}{8}\)
Now,
By using the Long Division,

Hence, from the above,
We can conclude that
\(\frac{7}{12}\) ÷ \(\frac{1}{8}\) = \(\frac{14}{3}\)
The quotient for the given division expresion is: \(\frac{14}{3}\)

Question 24.
\(\frac{3}{10}\) ÷ \(\frac{3}{5}\)
Answer:
The given division expression is:
\(\frac{3}{10}\) ÷ \(\frac{3}{5}\)
Now,
By using the Long Division,

Hence, from the above,
We can conclude that
\(\frac{3}{10}\) ÷ \(\frac{3}{5}\) = \(\frac{1}{2}\)
The quotient for the given division expresion is: \(\frac{1}{2}\)

Question 25.
\(\frac{2}{5}\) ÷ \(\frac{1}{8}\)
Answer:
The given division expression is:
\(\frac{2}{5}\) ÷ \(\frac{1}{8}\)
Now,
By using the Long Division,

Hence, from the above,
We can conclude that
\(\frac{2}{5}\) ÷ \(\frac{1}{8}\) = \(\frac{16}{5}\)
The quotient for the given division expresion is: \(\frac{16}{5}\)

Question 26.
Be Precise A large bag contains \(\frac{12}{15}\) pound of granola. How many \(\frac{1}{3}\) pound bags can be filled with this amount of granola? How much granola is left over?
Envision Math Common Core Grade 6 Answers Topic 1 Use Positive Rational Numbers 103
Answer:
It is given that
A large bag contains \(\frac{12}{15}\) pound of granola
Now,
Let the number of \(\frac{1}{3}\) pound bags that can be filled with the given amount of granola be: x
Now,
According to the given information,
\(\frac{1}{3}\) × x = \(\frac{12}{15}\)
x = \(\frac{12}{15}\) ÷ \(\frac{1}{3}\)
Now,
By using the Long Division,

So,
The number of \(\frac{1}{3}\) pound bags that can be filled with the given amount of granola is about 2 bags
Now,
The amount of granola that is leftover = (The number of \(\frac{1}{3}\) pound bags that can be filled with the given amount of granola) – (The total amount of granola)
= \(\frac{12}{5}\) – \(\frac{12}{15}\)
Now,
By using the Long Subtraction,

Hence, from the above,
We can conclude that
The amount of granola that is leftover is: \(\frac{8}{5}\) pounds

Question 27.
Higher-Order Thinking Find \(\frac{3}{4}\) ÷ \(\frac{2}{3}\). Then draw a picture and write an explanation describing how to get the answer.
Answer:
The given division expression is:
\(\frac{3}{4}\) ÷ \(\frac{2}{3}\)
Now,
By using the Long Division,

Hence,
The representation of the given division expression in the form of an area model is:

\(\frac{3}{4}\) ÷ \(\frac{2}{3}\) = \(\frac{9}{8}\)

Question 28.
The area of a rectangular painting is \(\frac{1}{6}\) square yard. The width is \(\frac{2}{3}\) yard. What is the length of the painting? Use the formula A = l × w.
Answer:
It is given that
The area of a rectangular painting is \(\frac{1}{6}\) square yard. The width is \(\frac{2}{3}\) yard.
Now,
Let the length of the painting be x yard
Now,
From the given information,
A = l × w
So,
\(\frac{1}{6}\) = x × \(\frac{2}{3}\)
x = \(\frac{1}{6}\) ÷ \(\frac{2}{3}\)
Now,
By using the Long Division,

Hence, from the above,
We can conclude that
The length of the painting is: \(\frac{1}{4}\) yard

Question 29.
Solve for n in the equation \(\frac{13}{16}\) ÷ \(\frac{1}{6}\) = n.
Answer:
The given division expression is:
\(\frac{13}{16}\) ÷ \(\frac{1}{6}\) = n
Where,
‘n’ is the quotient for the given division expression
Now,
By using the Long Division,

So,
\(\frac{13}{16}\) ÷ \(\frac{1}{6}\) = \(\frac{39}{8}\)
Hence, from the above,
We can conclude that
The value of ‘n’ is: \(\frac{39}{8}\)

Question 30.
Model with Math
A cafeteria uses \(\frac{1}{6}\) pound of coffee to fill a large coffee dispenser. The cafeteria has \(\frac{2}{3}\) pound of coffee to use.
a. Complete the model at the right to find how many coffee dispensers the cafeteria can fill.
b. Write a division sentence that describes the model and tells how many dispensers can be filled.
Envision Math Common Core Grade 6 Answers Topic 1 Use Positive Rational Numbers 60.1
Answer:
It is given that
A cafeteria uses \(\frac{1}{6}\) pound of coffee to fill a large coffee dispenser. The cafeteria has \(\frac{2}{3}\) pound of coffee to use.
Now,
The representation of the given information in the form of a model is:

Now,
From the given model,
We can observe that
The number of coffee dispensers the cafeteria can fill = (The number of pounds of the coffee cafeteria has to use) ÷ (The number of pounds of the coffee a cafeteria has to use)
= \(\frac{2}{3}\) ÷ \(\frac{1}{6}\)
Now,
By using the Long division,

Hence, from the above,
We can conclude that
a. The model to find how many coffee dispensers the cafeteria can fill is:

b. The division sentence that describes the model and tells how many dispensers can be filled is:
\(\frac{2}{3}\) ÷ \(\frac{1}{6}\)

Question 31.
Model with Math A full load for a small truck to haul is \(\frac{2}{3}\) ton of gravel. The truck is hauling \(\frac{1}{2}\) ton of gravel.
Envision Math Common Core Grade 6 Answers Topic 1 Use Positive Rational Numbers 60.2
a. Complete the model below to find how much of a full load the truck is hauling.
b. Write a division sentence that describes the model and tells how much of a full load the truck is hauling.
Envision Math Common Core Grade 6 Answers Topic 1 Use Positive Rational Numbers 60.3
Answer:
It is given that
A full load for a small truck to haul is \(\frac{2}{3}\) ton of gravel. The truck is hauling \(\frac{1}{2}\) ton of gravel.
Now,
The representation of the given information in the form of a model is:

Now,
From the given model,
We can observe that
The amount of a full load the truck is hauling = (The amount of a full load for a small truck to haul) ÷ (The amount of gravel a truck is hauling)
= \(\frac{2}{3}\) ÷ \(\frac{1}{2}\)
Now,
By using the Long Division,

Hence, from the above,
We can conclude that
a. The model below to find how much of a full load the truck is hauling is:

b. The division sentence describes the model and tells how much of a full load the truck is hauling is:
\(\frac{2}{3}\) ÷ \(\frac{1}{2}\)

Question 32.
Use Structure How many \(\frac{1}{4}\)-inch pieces can be cut from a piece of metal \(\frac{5}{4}\) inch long?
Answer:
It is given that
\(\frac{1}{4}\)-inch pieces can be cut from a piece of metal \(\frac{5}{4}\) inch long
So,
The number of \(\frac{1}{4}\)-inch pieces that can be cut from \(\frac{5}{4}\) inch long
= \(\frac{5}{4}\) ÷ \(\frac{1}{4}\)
Now,
By using the Long Division,

Hence, from the above,
We can conclude that
The number of \(\frac{1}{4}\)-inch pieces that can be cut from \(\frac{5}{4}\) inch long is: 5 pieces

Question 33.
Write a problem that could be solved by finding \(\frac{5}{8}\) ÷ \(\frac{2}{5}\).
Answer:
The given division expression is:
\(\frac{5}{8}\) ÷ \(\frac{2}{5}\)
Now,
By using the Long Division,

Hence, from the above,
We can conclude that
\(\frac{5}{8}\) ÷ \(\frac{2}{5}\) = \(\frac{25}{16}\)

Assessment Practice

Question 34.
Which division sentence is shown by the model at the right?
Envision Math Common Core Grade 6 Answers Topic 1 Use Positive Rational Numbers 60.30
A. \(\frac{2}{3}\) ÷ \(\frac{1}{9}\) = 6
B. \(\frac{1}{9}\) ÷ \(\frac{2}{3}\) = \(\frac{1}{6}\)
C. 6 ÷ \(\frac{1}{9}\) = 54
D. 6 ÷ \(\frac{2}{3}\) = 9
Answer:
The given model is:
Envision Math Common Core Grade 6 Answers Topic 1 Use Positive Rational Numbers 60.30
Now,
From the given model,
We can observe that
\(\frac{2}{3}\) ÷ \(\frac{1}{9}\)
Now,
By using the Long Division,

Hence, from the above,
We can conclude that
The division sentence that is shown by the model is:

Lesson 1.6 Divide Mixed Numbers

Solve & Discuss It!

A jeweler has a 5\(\frac{1}{2}\)inch strip of silver wire that she is cutting into 1\(\frac{3}{8}\)-inch pieces. How many pieces can she make?
Envision Math Common Core Grade 6 Answers Topic 1 Use Positive Rational Numbers 60.4
Answer:
It is given that
A jeweler has a 5\(\frac{1}{2}\)inch strip of silver wire that she is cutting into 1\(\frac{3}{8}\)-inch pieces
So,
The number of pieces of silver wire she can make = (The total length of the silver wire) ÷ (The length of each cutting piece of the silver wire)
= 5\(\frac{1}{2}\) ÷ 1\(\frac{3}{8}\)
Now,
By using the Long Division,

Hence, from the above,
We can conclude that
The number of pieces of silver wire she can make is: 4 pieces

Generalize How can you use what you know about solving problems with fractions to find how many pieces the jeweler can make?
Answer:
The steps to solve problems with fractions with the help of what you know is:
a. Read the problem carefully.
b. Think about what it is asking us to do.
c. Think about the information we need.
d. Solve it.
e Simplify, if necessary.
f. Think about whether our solution makes sense (in order to check it).

Focus on math practices
Generalize Explain how to use estimation to check whether your answer is reasonable.
Answer:
To use estimation, you need to read and understand the problem. There will be clues in the problem to let you know if estimation is a good option for solving that specific problem. Estimating means finding an answer that is an approximate answer

? Essential Question How can you divide with mixed numbers?
Answer:
The steps to divide the mixed numbers are:
Step 1:
Make sure everything is in fraction form (mixed numbers need to be changed to improper fractions).
Step 2:
Flip the second fraction and change the operation to multiply.
Step 3:
Multiply across the top for the new numerator and across the bottom for the new denominator.
Step 4:
Simplify (if needed).

Try It!
How many medium bumper stickers can Damon fit side by side on his car bumper? Find 37\(\frac{1}{2}\) ÷ 10\(\frac{3}{4}\).
Envision Math Common Core Grade 6 Answers Topic 1 Use Positive Rational Numbers 60.5
Answer:
The given information is:

Now,
The number of medium bumper stickers can Damon fit side by side on his car bumper = (The total length of the car bumper) ÷ (The length of the medium bumper)
= 37\(\frac{1}{2}\) ÷ 10\(\frac{3}{4}\)
Now,
By using the Long Division,

Hence, from the above,
We can conclude that

Convince Me! Why do you multiply \(\frac{75}{2}\) by \(\frac{4}{43}\) to divide 37\(\frac{1}{2}\) by 10\(\frac{3}{4}\)?
Answer:
The given division expression is:
37\(\frac{1}{2}\) ÷ 10\(\frac{3}{4}\)
Now,
The representation of the given mixed numbers into a fraction is:


So,
37\(\frac{1}{2}\) ÷ 10\(\frac{3}{4}\)
= \(\frac{75}{2}\) ÷ \(\frac{43}{4}\)
Now,
We know that,
Two numbers whose product is “1” are called “reciprocals” of each other. If a nonzero number is named as a fraction \(\frac{a}{b}\), then its reciprocal is \(\frac{b}{a}\)
So,
\(\frac{75}{2}\) ÷ \(\frac{43}{4}\)
= \(\frac{75}{2}\) × \(\frac{4}{43}\)
Hence, from the above,
We can conclude that
You can multiply \(\frac{75}{2}\) by \(\frac{4}{43}\) to divide 37\(\frac{1}{2}\) by 10\(\frac{3}{4}\) due to the property of “Reciprocals”

Try It!
Divide.

a. 20 ÷ 2\(\frac{2}{3}\)
Answer:
The given division expression is:
20 ÷ 2\(\frac{2}{3}\)
So,
The representtaion of 2\(\frac{2}{3}\) into a fraction is:

So,
20 ÷ \(\frac{8}{3}\)
Now,
By using the Long Division,

Hence, from the above,
We can conclude that
20 ÷ 2\(\frac{2}{3}\) = \(\frac{15}{2}\)

b. 12\(\frac{1}{2}\) ÷ 6
Answer:
The given division expression is:
12\(\frac{1}{2}\) ÷ 6
So,
The representation of 12\(\frac{1}{2}\) into a fraction is:

So,
\(\frac{25}{2}\) ÷ 6
Now,
By using the Long Division,

Hence, from the above,
We can conclude that
12\(\frac{1}{2}\) ÷ 6 = \(\frac{25}{12}\)

KEY CONCEPT
Envision Math Common Core Grade 6 Answers Topic 1 Use Positive Rational Numbers 60.6

Do You Understand?

Question 1.
? Essential Question How can you divide with mixed numbers?
Answer:
The steps to divide the mixed numbers are:
Step 1:
Make sure everything is in fraction form (mixed numbers need to be changed to improper fractions).
Step 2:
Flip the second fraction and change the operation to multiply.
Step 3:
Multiply across the top for the new numerator and across the bottom for the new denominator.
Step 4:
Simplify (if needed).

Question 2.
Generalize When dividing mixed numbers, why is it important to estimate the quotient first?
Answer:
You can estimate to find the approximate quotient of fractions and mixed numbers. Estimating quotients of fractions is similar to estimating products of fractions. To estimate the quotient of two fractions, you will first rewrite the problem as a multiplication problem

Question 3.
Reasoning In Example 1, how many long bumper stickers can Damon fit side by side on his car bumper? Will there be uncovered space? Explain.
Answer:
It is given that
The total length of the bumper is: 37\(\frac{1}{2}\) inches
Now,
The given information is:

Now,
From the given table,
We can observe that
The total length of the bumper = (The length of the short bumper) + (The length of the medium bumper) + (The length of the long bumper)
= 6\(\frac{1}{4}\) + 10\(\frac{3}{4}\) + 15
Now,
By using the Long Addition,

So,
The total length of the bumper = 17 + 15
= 32 inches
Now,
From the answer,
We can observe that
The given total length of the bumper > The calculated total length of the bumper
Hence, from the above,
We can conclude that there will be uncovered space in the bumper

Question 4.
What is the difference between dividing fractions less than 1 and dividing mixed numbers?
Answer:
Dividing mixed numbers is very similar to multiplying mixed numbers. You just add one step i.e., after changing the divisor into an improper fraction, you then find its reciprocal and multiply.

Do You Know How?

In 5–13, find each quotient.

Question 5.
Envision Math Common Core Grade 6 Answers Topic 1 Use Positive Rational Numbers 61.1
Answer:
The given division expression is:
2\(\frac{5}{8}\) ÷ 2\(\frac{1}{4}\)
Now,
By using the Long Division,

Hence, from the above,
We can conclude that
2\(\frac{5}{8}\) ÷ 2\(\frac{1}{4}\) = \(\frac{7}{8}\)
The quotient for the given division expression is: \(\frac{7}{6}\)

Question 6.
3 ÷ 4\(\frac{1}{2}\)
Answer:
The given division expression is:
3 ÷ 4\(\frac{1}{2}\)
Now,
The representation of 4\(\frac{1}{2}\) into a fraction is:

Now,
By using the Long Division,

Hence, from the above,
We can conclude that
3 ÷ 4\(\frac{1}{2}\) = \(\frac{2}{3}\)
The quotient for the given division expression is: \(\frac{2}{3}\)

Question 7.
18 ÷ 3\(\frac{2}{3}\)
Answer:
The given division expression is:
18 ÷ 3\(\frac{2}{3}\)
Now,
The representation of 3\(\frac{2}{3}\) into a fraction is:

Now,
By using the Long Division,

Hence, from the above,
We can conclude that
18 ÷ 3\(\frac{2}{3}\) = \(\frac{54}{11}\)
The quotient for the given division expression is: \(\frac{54}{11}\)

Question 8.
1\(\frac{2}{5}\) ÷ 7
Answer:
The given division expression is:
1\(\frac{2}{5}\) ÷ 7
Now,
The representation of 1\(\frac{2}{5}\) into a fraction is:

Now,
By using the Long Division,

Hence, from the above,
We can conclude that
1\(\frac{2}{5}\) ÷ 7 = \(\frac{1}{5}\)
The quotient for the given division expression is: \(\frac{1}{5}\)

Question 9.
5 ÷ 6\(\frac{2}{5}\)
Answer:
The given division expression is:
5 ÷ 6\(\frac{2}{5}\)
Now,
The representation of 3\(\frac{2}{3}\) into a fraction is:

Now,
By using the Long Division,

Hence, from the above,
We can conclude that
5 ÷ 6\(\frac{2}{5}\) = \(\frac{25}{32}\)
The quotient for the given division expression is: \(\frac{25}{32}\)

Question 10.
8\(\frac{1}{5}\) ÷ 3\(\frac{3}{4}\)
Answer:
The given division expression is:
8\(\frac{1}{5}\) ÷ 3\(\frac{3}{4}\)
Now,
By using the Long Division,

Hence, from the above,
We can conclude that
8\(\frac{1}{5}\) ÷ 3\(\frac{3}{4}\) = \(\frac{164}{75}\)
The quotient for the given division expression is: \(\frac{164}{75}\)

Question 11.
2\(\frac{1}{2}\) ÷ 4\(\frac{1}{10}\)
Answer:
The given division expression is:
2\(\frac{1}{2}\) ÷ 4\(\frac{1}{10}\)
Now,
By using the Long Division,

Hence, from the above,
We can conclude that
2\(\frac{1}{2}\) ÷ 4\(\frac{1}{10}\) = \(\frac{25}{41}\)
The quotient for the given division expression is: \(\frac{25}{41}\)

Question 12.
2\(\frac{2}{3}\) ÷ 6
Answer:
The given division expression is:
2\(\frac{2}{3}\) ÷ 6
Now,
The representation of 2\(\frac{2}{3}\) into a fraction is:

Now,
By using the Long Division,

Hence, from the above,
We can conclude that
2\(\frac{2}{3}\) ÷ 6 = \(\frac{4}{9}\)
The quotient for the given division expression is: \(\frac{4}{9}\)

Question 13.
6\(\frac{5}{9}\) ÷ 1\(\frac{7}{9}\)
Answer:
The given division expression is:
6\(\frac{5}{9}\) ÷ 1\(\frac{7}{9}\)
Now,
By using the Long Division,

Hence, from the above,
We can conclude that
6\(\frac{5}{9}\) ÷ 1\(\frac{7}{9}\) = \(\frac{59}{16}\)
The quotient for the given division expression is: \(\frac{59}{16}\)

Practice & Problem Solving

Scan for Multimedia

Leveled Practice In 14-25, find each quotient.

Question 14.
Envision Math Common Core Grade 6 Answers Topic 1 Use Positive Rational Numbers 62.1
Answer:
The given division expression is:
10 ÷ 2\(\frac{1}{4}\)
Now,
The representation of 2\(\frac{1}{4}\) into a fraction is:

Now,
By using the Long Division,

Hence, from the above,
We can conclude that
10 ÷ 2\(\frac{1}{4}\) = \(\frac{40}{9}\)
The quotient for the given division expression is: \(\frac{40}{9}\)

Question 15.
Envision Math Common Core Grade 6 Answers Topic 1 Use Positive Rational Numbers 67.2
Answer:
The given division expression is:
9\(\frac{1}{3}\) ÷ 6
Now,
The representation of 9\(\frac{1}{3}\) into a fraction is:

Now,
By using the Long Division,

Hence, from the above,
We can conclude that
9\(\frac{1}{3}\) ÷ 6 = \(\frac{14}{9}\)
The quotient for the given division expression is: \(\frac{14}{9}\)

Question 16.
Envision Math Common Core Grade 6 Answers Topic 1 Use Positive Rational Numbers 69.3
Answer:
The given division expression is:
1\(\frac{3}{8}\) ÷ 4\(\frac{1}{8}\)
Now,
By using the Long Division,

Hence, from the above,
We can conclude that
1\(\frac{3}{8}\) ÷ 4\(\frac{1}{8}\) = \(\frac{1}{3}\)
The quotient for the given division expression is: \(\frac{1}{3}\)

Question 17.
Envision Math Common Core Grade 6 Answers Topic 1 Use Positive Rational Numbers 69.4
Answer:
The given division expression is:
2\(\frac{2}{3}\) ÷ 8
Now,
The representation of 2\(\frac{2}{3}\) into a fraction is:

Now,
By using the Long Division,

Hence, from the above,
We can conclude that
2\(\frac{2}{3}\) ÷ 8 = \(\frac{1}{3}\)
The quotient for the given division expression is: \(\frac{1}{3}\)

Question 18.
Envision Math Common Core Grade 6 Answers Topic 1 Use Positive Rational Numbers 69.5
Answer:
The given division expression is:
4\(\frac{1}{3}\) ÷ 3\(\frac{1}{4}\)
Now,
By using the Long Division,

Hence, from the above,
We can conclude that
4\(\frac{1}{3}\) ÷ 3\(\frac{1}{4}\) = \(\frac{4}{3}\)
The quotient for the given division expression is: \(\frac{4}{3}\)

Question 19.
Envision Math Common Core Grade 6 Answers Topic 1 Use Positive Rational Numbers 69.6
Answer:
The given division expression is:
1 ÷ 8\(\frac{5}{9}\)
Now,
The representation of 8\(\frac{5}{9}\) into a fraction is:

Now,
By using the Long Division,

Hence, from the above,
We can conclude that
1 ÷ 8\(\frac{5}{9}\) = \(\frac{9}{77}\)
The quotient for the given division expression is: \(\frac{9}{77}\)

Question 20.
3\(\frac{5}{6}\) ÷ 9\(\frac{5}{6}\)
Answer:
The given division expression is:
3\(\frac{5}{6}\) ÷ 9\(\frac{5}{6}\)
Now,
By using the Long Division,

Hence, from the above,
We can conclude that
3\(\frac{5}{6}\) ÷ 9\(\frac{5}{6}\) = \(\frac{23}{59}\)
The quotient for the given division expression is: \(\frac{23}{59}\)

Question 21.
16 ÷ 2\(\frac{2}{3}\)
Answer:
The given division expression is:
16 ÷ 2\(\frac{2}{3}\)
Now,
The representation of 2\(\frac{2}{3}\) into a fraction is:

Now,
By using the Long Division,

Hence, from the above,
We can conclude that
16 ÷ 2\(\frac{2}{3}\) = 6
The quotient for the given division expression is: 6

Question 22.
2\(\frac{5}{8}\) ÷ 13
Answer:
The given division expression is:
2\(\frac{5}{8}\) ÷ 13
Now,
The representation of 2\(\frac{5}{8}\) into a fraction is:

Now,
By using the Long Division,

Hence, from the above,
We can conclude that
2\(\frac{5}{8}\) ÷ 13 = \(\frac{21}{104}\)
The quotient for the given division expression is: \(\frac{21}{104}\)

Question 23.
3\(\frac{6}{7}\) ÷ 6\(\frac{3}{4}\)
Answer:
The given division expression is:
3\(\frac{6}{7}\) ÷ 6\(\frac{3}{4}\)
Now,
By using the Long Division,

Hence, from the above,
We can conclude that
3\(\frac{6}{7}\) ÷ 6\(\frac{3}{4}\) = \(\frac{4}{7}\)
The quotient for the given division expression is: \(\frac{4}{7}\)

Question 24.
2\(\frac{1}{3}\) ÷ 1\(\frac{1}{3}\)
Answer:
The given division expression is:
2\(\frac{1}{3}\) ÷ 1\(\frac{1}{3}\)
Now,
By using the Long Division,

Hence, from the above,
We can conclude that
2\(\frac{1}{3}\) ÷ 1\(\frac{1}{3}\) = \(\frac{7}{4}\)
The quotient for the given division expression is: \(\frac{7}{4}\)

Question 25.
3\(\frac{3}{4}\) ÷ 1\(\frac{1}{2}\)
Answer:
The given division expression is:
3\(\frac{3}{4}\) ÷ 1\(\frac{1}{2}\)
Now,
By using the Long Division,

Hence, from the above,
We can conclude that
3\(\frac{3}{4}\) ÷ 1\(\frac{1}{2}\) = \(\frac{5}{2}\)
The quotient for the given division expression is: \(\frac{5}{2}\)

Question 26.
Beth is making a rope ladder. Each step of the ladder is 2\(\frac{1}{3}\), feet wide. Beth has a rope that is 21 feet long. How many steps can she make from the rope?
Envision Math Common Core Grade 6 Answers Topic 1 Use Positive Rational Numbers 69.11
Answer:
It is given that
Beth is making a rope ladder. Each step of the ladder is 2\(\frac{1}{3}\), feet wide. Beth has a rope that is 21 feet long
So,
The number of steps Berth can make from the rope = (The total length of the rope) ÷ (The length of each step of the ladder)
= 21 ÷ 2\(\frac{1}{3}\)
Now,
The representation of 2\(\frac{1}{3}\) into a fraction form is:

Now,
By using the Long Division,

Hence, from the above,
We can conclude that
The number of steps Berth can make from the rope is: 9 steps

Question 27.
The area of this rectangle is 257\(\frac{1}{4}\) in.2. Find side length w.
Envision Math Common Core Grade 6 Answers Topic 1 Use Positive Rational Numbers 69.12
Answer:
It is given that
The area of this rectangle is 257\(\frac{1}{4}\) in.2
Now,
The given figure is:
Envision Math Common Core Grade 6 Answers Topic 1 Use Positive Rational Numbers 69.12
Now,
We know that,
The area of a rectangle = Length × Width
So,
257\(\frac{1}{4}\) = 10\(\frac{1}{2}\) × w
w = 257\(\frac{1}{4}\) ÷ 10\(\frac{1}{2}\)
Now,
By using the Long Division,

Hence, from the above,
We can conclude that
The side length (w) of the given rectangle is: \(\frac{49}{2}\) in.

In 28 and 29, use the picture.

Question 28.
The larger room is twice as long as the smaller room. How long is the larger room?
Envision Math Common Core Grade 6 Answers Topic 1 Use Positive Rational Numbers 69.13
Answer:
It is given that
The larger room is twice as long as the smaller room.
Now,
The given figure is:
Envision Math Common Core Grade 6 Answers Topic 1 Use Positive Rational Numbers 69.13
So,
The length of the larger room = 2 × (The length of the smaller room)
= 2 × 20\(\frac{4}{5}\)
Now,
The representation of 20\(\frac{4}{5}\) into a fraction is:

Now,
By using the Long Multiplication,

Hence, from the above,
We can conclude that
The length of the larger room is: \(\frac{208}{5}\) feet

Question 29.
If the length of the smaller room is divided into 4 equal parts, how long is each part?
Answer:
It is given that
The length of the smaller room is divided into 4 equal parts
So,
According to the given information,
The length of each part in the smaller room = 20\(\frac{4}{5}\) ÷ 4
Now,
The representation of 20\(\frac{4}{5}\) into a fraction is:

Now,
By using the Long Division,

Hence, from the above,
We can conclude that
The length of each part in the smaller room is: [latx]\frac{26}{5}[/latex] feet

Question 30.
Make Sense and Persevere Luis has 3 pounds of ground turkey to make turkey burgers. He uses \(\frac{3}{8}\) pound per burger to make 6 burgers. How many \(\frac{1}{4}\)-pound burgers can Luis make with the remaining turkey?
Answer:
It is given that
Luis has 3 pounds of ground turkey to make turkey burgers. He uses \(\frac{3}{8}\) pound per burger to make 6 burgers
So,
The remaining amount of ground Turkey Luis has = 3 – \(\frac{3}{8}\)
Now,
By using the Long Subtraction,

So,
The remaining amount of ground turkey Luis has is: \(\frac{21}{8}\) pounds
Now,
The number of \(\frac{1}{4}\)-pound burgers can Luis make with the remaining turkey =
\(\frac{21}{8}\) ÷ \(\frac{1}{4}\)
Now,
By using the Long Division,

Hence, from the above,
We can conclude that
The number of \(\frac{1}{4}\)-pound burgers can Luis make with the remaining turkey is about 11 burgers

Question 31.
Higher Order Thinking If 9 × \(\frac{n}{5}\) = 9 ÷ \(\frac{n}{5}\), then what does n equal? Explain.
Answer:
It is given that
9 × \(\frac{n}{5}\) = 9 ÷ \(\frac{n}{5}\)
So,
According to the Property of Reciprocals,
9 × \(\frac{n}{5}\) = 9 ÷ \(\frac{n}{5}\)
9 × \(\frac{n}{5}\) = 9 × \(\frac{5}{n}\)
\(\frac{n}{5}\) = \(\frac{5}{n}\)
n × n = 5 × 5
n² = 25
n = \(\sqrt{25}\)
n = 5
Hence, from the above,
We can conclude that
The value of “n” is: 5

Question 32.
Margaret uses 1\(\frac{3}{4}\) teaspoons of key lime zest to make 12 key lime cupcakes. She wants to make 30 cupcakes. How much key lime zest will Margaret use?
Answer:
It is given that
Margaret uses 1\(\frac{3}{4}\) teaspoons of key lime zest to make 1\(\frac{1}{2}\) key lime cupcakes
Now,
Let the number of teaspoons of key lime zest to make 30 cupcakes be x
So,
1\(\frac{3}{4}\) × 12 = x × 30
Now,
The representation of 1\(\frac{3}{4}\) into a fraction is:

So,
x = \(\frac{7}{4}\) × \(\frac{12}{30}\)
Now,
By using the Long Multiplication,

Hence, from the above,
We can conclude that
The number of teaspoons of key lime zest to make 30 cupcakes is: \(\frac{7}{10}\) teaspoons of key lime zest

Question 33.
Use Structure A gem store in Fort Lauderdale received a shipment of 1\(\frac{1}{2}\) pounds of moonstone crystals. If these moonstone crystals were separated into 6 equal bags, how much would each bag weigh?
Answer:
It is given that
A gem store in Fort Lauderdale received a shipment of 1\(\frac{1}{2}\) pounds of moonstone crystals.
So,
The weight of each bag of moonstone crystals = (The total weight of moonstone crystals) ÷ (The number of bags that moonstone crystals equally divided)
= 1\(\frac{1}{2}\) ÷ 6
Now,
The representation of 1\(\frac{1}{2}\) into a fraction is:

Now,
By using the Long Division,

Hence, from the above,
We can conclude that
The weight of each bag of moonstone crystals is: \(\frac{1}{4}\) pounds

Question 34.
The owner of an aquatic store used 17\(\frac{1}{2}\) gallons of water to fill aquariums. He put 5\(\frac{5}{6}\) gallons of water in each aquarium. How many aquariums did he fill?
Answer:
It is given that
The owner of an aquatic store used 17\(\frac{1}{2}\) gallons of water to fill aquariums. He put 5\(\frac{5}{6}\) gallons of water in each aquarium.
So,
The number of aquariums did he fill = (The total amount of water used to fill aquariums) ÷ (The amount of water in each aquarium)
= 17\(\frac{1}{2}\) ÷ 5\(\frac{5}{6}\)
Now,
By using the Long Division,

Hence, from the above,
We can conclude that
The number of aquariums did he fill is: 3 aquariums

Question 35.
Write an explanation to a friend about how you would estimate 17\(\frac{1}{5}\) ÷ 3\(\frac{4}{5}\).
Answer:
The given division expression is:
17\(\frac{1}{5}\) ÷ 3\(\frac{4}{5}\)
Now,
By using the Long Division,

Hence, from the above,
We can conclude that
17\(\frac{1}{5}\) ÷ 3\(\frac{4}{5}\) = \(\frac{86}{19}\)

Assessment Practice

Question 36.
A restaurant has 15\(\frac{1}{5}\) pounds of alligator meat to make tasty alligator dishes.
PART A
Each pot of alligator stew requires 2\(\frac{3}{8}\) pounds of alligator meat. Which solution shows how many pots of alligator stew can be made?
A. 36 pots; 15\(\frac{1}{5}\) × 2\(\frac{3}{8}\)
B. \(\frac{5}{32}\) pot; 2\(\frac{3}{8}\) ÷ 15\(\frac{1}{5}\)
C. 7 pots; 15\(\frac{1}{5}\) ÷ 2\(\frac{3}{8}\)
D. 6 pots; 15\(\frac{1}{5}\) ÷ 2\(\frac{3}{8}\)
Answer:
It is given that
A restaurant has 15\(\frac{1}{5}\) pounds of alligator meat to make tasty alligator dishes. Each pot of alligator stew requires 2\(\frac{3}{8}\) pounds of alligator meat
So,
The number of pots of alligator stew can be made = 15\(\frac{1}{5}\) ÷ 2\(\frac{3}{8}\)
Now,
By using the Long Division,

So,
The number of pots of alligator stew that can be made is about 6 pots
Hence, from the above,
We can conclude that
The solution that shows how many pots of alligator stew can be made is:

PART B
The restaurant could make a smaller pot of alligator stew that uses 1\(\frac{3}{5}\) pounds of alligator meat. How many smaller pots of alligator stew can be made than the larger pots?
Answer:
It is given that
The restaurant could make a smaller pot of alligator stew that uses 1\(\frac{3}{5}\) pounds of alligator meat
So,
The number of smaller pots of stew that can be made = 15\(\frac{1}{5}\) ÷ 1\(\frac{3}{5}\)
Now,
By using the Long Division,

So,
The number of smaller pots of stew that can be made is about 10 pots
Now,
From Part (A),
We can observe that
The number of larger pots of stew that can be made is about 6 pots
So,
The number of smaller pots more than the larger pots = 10 – 6
= 4 pots
Hence, from the above,
We can conclude that
The number of smaller pots more than the larger pots are: 4 pots

Lesson 1.7 Solve Problems with Rational Numbers

ACTIVITY

Explain It!
Jenna feeds her cat twice a day. She gives her cat \(\frac{3}{4}\) can of cat food each time. Jenna is having a friend take care of her cat for 5 days. To prepare, she bought 8 cans of cat food. Did Jenna buy enough cat food?
Envision Math Common Core Grade 6 Answers Topic 1 Use Positive Rational Numbers 69.18

A. What do you need to know before you can answer the question?
Answer:
It is given that
Jenna feeds her cat twice a day. She gives her cat \(\frac{3}{4}\) can of cat food each time. Jenna is having a friend take care of her cat for 5 days. To prepare, she bought 8 cans of cat food.
Hence,
The hidden question for the given information is:
How much food will be needed for Jenna’s cat for 5 days?

B. How can you determine which operations to use to solve the problem?
Answer:
It is given that
Jenna feeds her cat twice a day. She gives her cat \(\frac{3}{4}\) can of cat food each time. Jenna is having a friend take care of her cat for 5 days. To prepare, she bought 8 cans of cat food.
So,
The operations we will use to solve the given problem is:
a.
To find the total amount of food Jenna gives her cat ——> Multiplication
b.
To find the amount of food Jenna’s cat will need for 5 days ——->Multiplication
c.
To find the amount of food Jenna’s cat will need for 8 days ——-> Multiplication
d.
To know whether Jenna bought enough food or not ——> Comparison

Focus on math practices
Reasoning To find out whether she has enough cat food, Jenna multiplies, divides, and compares. Explain how Jenna may have solved the problem.
Answer:
The steps Jenna took to solve the given problem are:
a.
To find the total amount of food Jenna gives her cat ——> Multiplication
b.
To find the amount of food Jenna’s cat will need for 5 days ——->Multiplication
c.
To find the amount of food Jenna’s cat will need for 8 days ——-> Multiplication
d.
To know whether Jenna bought enough food or not ——> Comparison

?Essential Question How can you solve problems with rational numbers?
Answer:
We know that,
A “Rational number” is a number that can be written as a fraction.
Fractions, integers, and decimals are all rational numbers.
Example:
Fraction to Decimal:
\(\frac{1}{2}\) = 0.5
Decimal to Fraction:
a. 0.15= \(\frac{3}{20}\)
b. 15÷5=3
c. 100÷5=20

Try It!
The farmer decided that he ordered enough planks for an arena that measured 115\(\frac{1}{4}\) ft by 63\(\frac{1}{2}\) ft. Is he correct? Explain.
Answer:
It is given that
The farmer decided that he ordered enough planks for an arena that measured 115\(\frac{1}{4}\) ft by 63\(\frac{1}{2}\) ft.
So,
The perimeter of the arena = 2 × (115\(\frac{1}{4}\) + 63\(\frac{1}{2}\))
Now,
By using the Long Addition,

Now,
By using the Long Multiplication,

So,
The perimeter of the arena is: \(\frac{715}{2}\) ft
Now,
The number of 8\(\frac{1}{2}\)-foot planks needed = (The perimeter of the arena) ÷ 8\(\frac{1}{2}\)
= \(\frac{715}{2}\) ÷ 8\(\frac{1}{2}\)
Now,
By using the Long Division,

So,
The number of 8\(\frac{1}{2}\)-foot planks needed is about 47 planks
But,
It is given that
A farmer needed 130 8\(\frac{1}{2}\)-foot planks
So,
47 < 130
Hence, from the above,
We can conclude that
The farmer is not correct

Convince Me! What questions do you need to answer to solve the Try It!?
Answer:
The questions you need to answer to solve the Try It is:
a. The perimeter of the arena with the given length and width of the arena
b. The number of 8\(\frac{1}{2}\)-foot planks for the perimeter we find in part (a)

Try It!
The number of runners who finish the marathon is 320. Runners donate $2.50 for each mile they run. How much money is donated? Explain.
Envision Math Common Core Grade 6 Answers Topic 1 Use Positive Rational Numbers 65.18
Answer:
It is given that
The number of runners who finish the marathon is 320. Runners donate $2.50 for each mile they run
Now,
The total number of miles run by 320 runners = (The total length of the marathon) × 320
= 26.2 × 320
Now,
By using the Long Multiplication,

So,
The total number of miles run by 320 runners is: 8,384 miles
Now,
The amount of money donated = (The total number of miles run by 320 runners) × (The amount of money donated by each runner for each mile)
= 8,384 × $2.50
Now,
By using the Long Multiplication,

So,
The amount of money donated is: $20,960
Hence, from the above,
We can conclude that

KEY CONCEPT
When solving multistep problems with fractions or decimals:
• decide the steps to use to solve the problem.
• choose the correct operations.
• identify the information you need from the problem.
• correctly use the information.
• calculate accurately.
• interpret solutions and check that the answer is reasonable.

Do You Understand?

Question 1.
?Essential Question How can you solve problems with rational numbers?
Answer:
We know that,
A “Rational number” is a number that can be written as a fraction.
Fractions, integers, and decimals are all rational numbers.
Example:
Fraction to Decimal:
\(\frac{1}{2}\) = 0.5
Decimal to Fraction:
a. 0.15= \(\frac{3}{20}\)
b. 15÷5=3
c. 100÷5=20

Question 2.
Be Precise Meghan has 5\(\frac{1}{4}\) yards of fabric. She plans to use \(\frac{2}{3}\) of the fabric to make 4 identical backpacks. To find how much fabric she will use to make the backpacks, Meghan multiplies 5\(\frac{1}{4}\) by \(\frac{2}{3}\). What else does Meghan need to do to find how much fabric she needs for each backpack?
Answer:
It is given that
Meghan has 5\(\frac{1}{4}\) yards of fabric. She plans to use \(\frac{2}{3}\) of the fabric to make 4 identical backpacks. To find how much fabric she will use to make the backpacks, Meghan multiplies 5\(\frac{1}{4}\) by \(\frac{2}{3}\)
Now,
The amount of fabric needed to make 4 identical backpacks = (The amount of fabric she planned to use to make the backpacks) × (The total length of the fabric) × (The number of identical backpacks)
= \(\frac{2}{3}\) × 5\(\frac{1}{4}\) × 4
So,
Aside From the given information,
Meghan needed to multiply the number of identical backpacks to the result of \(\frac{2}{3}\) × 5\(\frac{1}{4}\)
Hence, from the above,
We can conclude that
To find how much fabric Meghan needs for each backpack,
Meghan needed to multiply the number of identical backpacks to the result of \(\frac{2}{3}\) × 5\(\frac{1}{4}\)

Question 3.
Critique Reasoning Each side of a square patio is 10.5 feet. The patio is made up of 1.5-foot by 1.5-foot square stones. What is the number of stones on the patio? Look at the solution below. Does it include all the steps needed to solve the problem? Explain.
Envision Math Common Core Grade 6 Answers Topic 1 Use Positive Rational Numbers 69.30
Answer:
It is given that
Each side of a square patio is 10.5 feet. The patio is made up of 1.5-foot by 1.5-foot square stones
Now,
We know that,
The area of a square = Side × Side
So,
The area of a square patio = 10.5 × 10.5
So,
The area of the square stones = 1.5 × 1.5
So,
The number of stones in the patio = (The area of a square patio) ÷ (The area of the square stones)
= (10.5 × 10.5) ÷ (1.5 × 1.5)
= 110.25 ÷ 2.25
Now,
By using the Long Division,

Hence, from the above,
We can conclude that
The given solution did not include all the steps needed to solve the given problem
The number of stones in the patio is: 49 stones

Do You Know How?

Question 4.
Devon records 4 hours of reality shows on her DVR. She records comedy shows for \(\frac{3}{8}\) of that amount of time. Devon watches all the reality and comedy shows in half-hour sittings.
a. Find the number of hours of comedy shows that Devon records.
Answer:
It is given that
Devon records 4 hours of reality shows on her DVR. She records comedy shows for \(\frac{3}{8}\) of that amount of time. Devon watches all the reality and comedy shows in half-hour sittings.
So,
The number of hours of comedy shows that Devon records = \(\frac{3}{8}\) × (The amount of time Devon records the reality shows on her DVR)
= \(\frac{3}{8}\) × 4
Now,
By using the Long Multiplication,

Hence, from the above,
We can conclude that
The number of hours of comedy shows that Devon records are: \(\frac{3}{2}\) hours

b. Find the total number of hours of reality and comedy shows that Devon records.
Answer:
It is given that
Devon records 4 hours of reality shows on her DVR. She records comedy shows for \(\frac{3}{8}\) of that amount of time. Devon watches all the reality and comedy shows in half-hour sittings.
Now,
From part (a),
We can observe that
The number of hours of comedy shows that Devon records are: \(\frac{3}{2}\) hours
So,
The total number of hours of reality and comedy shows that Devon records = (The number of hours of reality shows that Devon records) + (The number of hours of comedy shows that Devon records)
= \(\frac{3}{2}\) + 4
Now,
By using the Long Addition,

Hence, from the above,
We can conclude that
The total number of hours of reality and comedy shows that Devon records are: 5\(\frac{1}{2}\) hours

c. Find the number of half-hour sittings needed to watch all the shows.
Answer:
It is given that
Devon records 4 hours of reality shows on her DVR. She records comedy shows for \(\frac{3}{8}\) of that amount of time. Devon watches all the reality and comedy shows in half-hour sittings.
Now,
From part (b),
We can observe that
The total number of hours of reality and comedy shows that Devon records are: 5\(\frac{1}{2}\) hours
So,
The number of half-hour sittings needed to watch all the shows = (The total number of hours of reality and comedy shows that Devon records) ÷ \(\frac{1}{2}\)
= 5\(\frac{1}{2}\) ÷ \(\frac{1}{2}\)
Now,
By using the Long Division,

Hence, from the above,
We can conclude that
The number of half-hour sittings needed to watch all the shows is: 11 half-hour sittings

Question 5.
An auto mechanic earns $498.75 in 35 hours during the week. His pay is $2.50 more per hour on weekends. If he works 6 hours on the weekend in addition to 35 hours during the week, how much does he earn?
a. What questions do you need to answer to solve the problem?
Answer:
It is given that
An auto mechanic earns $498.75 in 35 hours during the week. His pay is $2.50 more per hour on weekends. If he works 6 hours on the weekend in addition to 35 hours during the week
Hence,
The questions you need to answer to solve the given problem are:
a. How much does an auto mechanic earn per hour?
b. How much does an auto mechanic earn per hour on weekends?

b. How much does the auto mechanic earn? Explain.
Answer:
It is given that
An auto mechanic earns $498.75 in 35 hours during the week. His pay is $2.50 more per hour on weekends. If he works 6 hours on the weekend in addition to 35 hours during the week
So,
The amount of money does an auto mechanic earn per hour = (The amount an auto mechanic earns during the week) ÷ (Number of hours)
= $498.75 ÷ 35
Now,
By using the Long Division,

So,
The amount of money an auto mechanic earns per hour is: $14.25
Now,
The amount of money an auto mechanic earns on weekends per hour = $2.50 + $14.25
Now,
By using the Long Multiplication,

So,
The amount of money an auto mechanic earns during weekends is: $16.75
Now,
The total amount of money an auto mechanic earns = (The total amount of money an auto mechanic earns during weekdays) + (The total amount of money an auto mechanic earns on weekends)
= $498.75 + (6 × $16.75)
= $498.75 + $100.5
= $599.25
Hence, from the above,
We can conclude that
The total amount of money an auto mechanic earns is: $599.25

Practice & Problem Solving

Scan for Multimedia

In 6-8, use the picture at the right.

Question 6.
You buy 3.17 pounds of apples, 1.25 pounds of pears, and 2.56 pounds of oranges. What is your total bill rounded to the nearest cent?
Envision Math Common Core Grade 6 Answers Topic 1 Use Positive Rational Numbers 69.31
Answer:
It is given that
You buy 3.17 pounds of apples, 1.25 pounds of pears, and 2.56 pounds of oranges
So,
Your total bill rounded to the nearest cent = [(The total weight of apples) × (The cost of each apple)] + [(The total weight of pears) × (The cost of each pear)] + [(The total weight of oranges) × (The cost of each orange)]
= (3.17 × $0.99) + (1.25 × $1.19) + (2.56 × $1.09)
= $3.13 + $1.48 + 2.79
= $4.61 + $2.79
= $7.40
Hence, from the above,
We can conclude that
Your total bill rounded to the nearest cent is: $7.40

Question 7.
A student pays for 8.9 pounds of apples with a $10 bill. How much change does the student receive?
a. What do you do first to solve the problem?
Answer:
It is given that
A student pays for 8.9 pounds of apples with a $10 bill
Hence,
The hidden question you need to answer for the given problem is:
What is the total bill you have to pay for 8.9 pounds of apples for $0.99 each?

b. What do you do next?
Answer:
After completing the solution in part (a),
The thing you have to do next is:
Subtract the amount of money the student received by subtracting the value of the total bill you have to pay for 8.9 pounds of apples for $0.99 each from $10

Question 8.
A customer pays $3.27 for oranges and $4.76 for pears. How many pounds of fruit does the customer buy?
a. What do you do first to solve the problem?
Answer:
It is given that
A customer pays $3.27 for oranges and $4.76 for pears
Hence,
The hidden question you have to answer for the given problem is:
How many pounds of oranges and pears does the customer can buy?

b. What do you do next?
Answer:
After find the solution to the question present in part (a),
The thing you have to do next is:
Add the number of pounds of oranges and pears to find the total number of pounds of fruit doe the customer bought

Question 9.
Critique Reasoning Students put 2\(\frac{1}{4}\) pounds of trail mix into bags that each weigh \(\frac{3}{8}\) pound. They bring \(\frac{2}{3}\) of the bags of trail mix on a hiking trip. Can you determine how many bags of trail mix are left by completing just one step? Explain.
Answer:
It is given that
Students put 2\(\frac{1}{4}\) pounds of trail mix into bags that each weigh \(\frac{3}{8}\) pound. They bring \(\frac{2}{3}\) of the bags of trail mix on a hiking trip.
So,
The number of bags of trail mix = (The total number of pounds of trail mix) ÷ (The weight of each bag of trail mix)
= 2\(\frac{1}{4}\) ÷ \(\frac{3}{8}\)
Now,
By using the Long Division,

So,
The number of bags of trail mix is: 6 bags
Now,
The \(\frac{2}{3}\) of the bags of trail mix = 6 × \(\frac{2}{3}\)
= \(\frac{6 × 2}{3}\)
= 4 bags
So,
The number of bags of trail mix left = (The total number of bags of trail mix) – (The \(\frac{2}{3}\) of the bags of trail mix)
= 6 – 4
= 2 bags
Hence, from the above,
We can conclude that
The number of bags of trail mix that are left is: 2 bags
We can’t determine the number of bags of trail mix by completing just one step

Question 10.
Three-fifths of the T-shirts in a T-shirt shop are blue. Five-eighths of those T-shirts are on sale. One-third of the blue T-shirts that are on sale are size medium. What fraction of the shop’s T-shirts are blue T-shirts that are on sale and are size medium? Explain.
Answer:
It is given that
Three-fifths of the T-shirts in a T-shirt shop are blue. Five-eighths of those T-shirts are on sale. One-third of the blue T-shirts that are on sale are size medium.
So,
The number of blue T-shirts are: \(\frac{3}{5}\) shirts
The number of blue T-shirts that are on sale = \(\frac{3}{5}\) × \(\frac{5}{8}\)
Now,
By using the Long Multiplication,

So,
The number of blue T-shirts that are on sale is: \(\frac{3}{8}\) shirts
Now,
The number of blue T-shirts that are on sale and are medium size = \(\frac{3}{8}\) × \(\frac{1}{3}\)
Now,
By using the Long Multiplication,

So,
The number of blue shirts that are on sale and are medium size is: \(\frac{1}{8}\) shirts
Hence, from the above,
We can conclude that
The number of blue T-shirts that are on sale is: \(\frac{3}{8}\) shirts
The number of blue shirts that are on sale and are medium size is: \(\frac{1}{8}\) shirts

In 11 and 12, use the diagram.
A community garden is made up of three gardens: a vegetable garden, an herb garden, and a flower garden.

Question 11.
The area of the vegetable garden is 0.4 of the area of the community garden. What is the area of the vegetable garden?
Envision Math Common Core Grade 6 Answers Topic 1 Use Positive Rational Numbers 69.32
Answer:
It is given that
A community garden is made up of three gardens: a vegetable garden, an herb garden, and a flower garden.
The area of the vegetable garden is 0.4 of the area of the community garden.
Now,
The given figure is:
Envision Math Common Core Grade 6 Answers Topic 1 Use Positive Rational Numbers 69.32
Now,
From the given figure,
The area of the community garden = (Length of the community garden) × (Width of the community garden)
= 6.2 × 4.5
Now,
By using the Long Multiplication,

So,
The area of the community garden is: 27.9 m²
Now,
The area of the vegetable garden = 0.4 × (The area of the community garden)
= 0.4 × 27.9
= 11.16 m²
Hence, from the above,
We can conclude that
The area of the vegetable garden is: 11.16 m²

Question 12.
The area of the flower garden is 9.7 square meters greater than the herb garden. What is the area of the flower garden?
Answer:
It is given that
The area of the flower garden is 9.7 square meters greater than the herb garden.
Now,
The given figure is:
Envision Math Common Core Grade 6 Answers Topic 1 Use Positive Rational Numbers 69.32
Now,
From the given figure,
We can observe that
The area of the Herb garden = 2.2 × 1.6
= 3.52 m²
So,
The area of the flower garden = 9.7 + (The area of the flower garden)
= 9.7 + 3.52
= 13.22 m²
Hence, from the above,
We can conclude that
The area of the flower garden is: 13.22 m²

Question 13.
Reasoning At the end of a party, \(\frac{3}{4}\) cup of smoked fish dip is left. Jim divides \(\frac{4}{5}\) of the leftover smoked fish dip equally between 2 friends. How much dip does each friend get?
Answer:
It is given that
At the end of a party, \(\frac{3}{4}\) cup of smoked fish dip is left. Jim divides \(\frac{4}{5}\) of the leftover smoked fish dip equally between 2 friends.
Now,
The \(\frac{4}{5}\) of the leftover smoked fish = \(\frac{4}{5}\) × \(\frac{3}{4}\)
Now,
By using the Long Multiplication,

So,
The \(\frac{4}{5}\) of the leftover smoked fish is: \(\frac{3}{5}\)
Now,
The amount of dip each friend get = \(\frac{3}{5}\) ÷ 2
Now,
By using the Long Division,

Hence, from the above,
We can conclude that
The amount of dip each friend get is: \(\frac{3}{10}\) of the leftover smoked fish dip

Question 14.
Students are planning a 3-day hiking trip in the Everglades. The hike covers a distance of 18.5 kilometers. The students hike 0.28 of the total distance on the first day. If they split the remaining distance equally between the second and third days, how far will they hike on day 3?
Answer:
It is given that
Students are planning a 3-day hiking trip in the Everglades. The hike covers a distance of 18.5 kilometers. The students hike 0.28 of the total distance on the first day and they split the remaining distance equally between the second and third days
So,
The remaining distance of the hike after the first day = (The total distance of hike) – (The distance of hike on the first day)
= 18.5 – (0.28 × 18.5)
= 18.5 – 5.18
= 13.32 kilometers
Now,
The distance of the hike on the second and the third days which are split equally = (The remaining distance of the hike after the first day) ÷ 2
= 13.32 ÷2
= 6.66 kilometers
So,
The amount of distance the students hiked on the third day is: 6.66 kilometers
Hence, from the above,
We can conclude that
The amount of distance the students hiked on the third day is: 6.66 kilometers

Question 15.
Higher-Order Thinking Kelly buys three containers of potato salad at the deli. She brings \(\frac{4}{5}\) of the potato salad to a picnic. How many pounds of potato salad does Kelly bring to the picnic? Describe two different ways to solve the problem.
Envision Math Common Core Grade 6 Answers Topic 1 Use Positive Rational Numbers 69.33
Answer:
It is given that
Kelly buys three containers of potato salad at the deli. She brings \(\frac{4}{5}\) of the potato salad to a picnic
Now,
The given figure is:
Envision Math Common Core Grade 6 Answers Topic 1 Use Positive Rational Numbers 69.33
So,
The total weight of the containers of potato salad = 1.03 + 1.12 + 1.6
= 3.75 pounds
So,
The total weight of potato salad Kelly brought = 3 × (The total weight of the containers of potato salad)
= 3 × 3.75
= 11.25 pounds
Now,
The amount of potato salad Kelly brought to picnic = \(\frac{4}{5}\) × (The total weight of potato salad Kelly brought)
= \(\frac{4}{5}\) × 11.25
= 9 pounds
Hence, from the above,
We can conclude that
The amount of potato salad Kelly brought to the picnic is: 9 pounds

Assessment Practice

Question 16.
Students make 84\(\frac{1}{2}\) ounces of liquid soap for a craft fair. They put the soap in 6\(\frac{1}{2}\)-ounce bottles and sell each bottle for $5.50. Which expression shows how much students earn if they sell all the bottles of liquid soap?
Envision Math Common Core Grade 6 Answers Topic 1 Use Positive Rational Numbers 69.34
Answer:
It s given that
Students make 84\(\frac{1}{2}\) ounces of liquid soap for a craft fair. They put the soap in 6\(\frac{1}{2}\)-ounce bottles and sell each bottle for $5.50.
So,
The number of bottles of liquid soap = 84\(\frac{1}{2}\) ÷ 6\(\frac{1}{2}\)
Now,
By using the Long Division,

So,
The number of bottles of liquid soap is: 13 bottles
Now,
The cost of the 13 bottles of liquid soap = 13 × (The cost of each liquid soap)
= 13 × $5.50
= $71.5
Hence, from the above,
We can conclude that
The expression shows how much students earn if they sell all the bottles of liquid soap is:

Question 17.
Claire mowed 5 lawns last week. She mowed each lawn in \(\frac{7}{12}\) hour. She mowed the same lawns this week in \(\frac{5}{12}\) hour each using her new lawnmower. How many times longer was Claire’s time to mow all the lawns last week than this week?
Answer:
It is given that
Claire mowed 5 lawns last week. She mowed each lawn in \(\frac{7}{12}\) hour. She mowed the same lawns this week in \(\frac{5}{12}\) hour each using her new lawnmower
So,
The number of hours Claire mowed last week = (The number of lawns) × (The amount of time each lawn was mowed last week)
= 5 × \(\frac{7}{12}\)
Now,
By using the Long Multiplication,

So,
The number of hours Claire mowed last week is: \(\frac{35}{12}\) hours
Now,
The number of hours Claire mowed this week = (The number of lawns) × (The amount of time each lawn was mowed this week)
= 5 × \(\frac{5}{12}\)
Now,
By using the Long Multiplication,

So,
The number of hours Claire mowed this week is: \(\frac{25}{12}\) hours
Now,
The difference between the number of hours Claire mowed the last week and this week = \(\frac{35}{12}\) – \(\frac{25}{12}\)
= \(\frac{35 – 25}{12}\)
= \(\frac{10}{12}\)
= \(\frac{5}{6}\) hours
Hence, from the above,
We can conclude that
Claire’s time to mow all the lawns last week is \(\frac{5}{6}\) times longer than this week

Topic 1 REVIEW

? Topic Essential Question
How can you fluently add, subtract, multiply, and divide decimals? How can you multiply and divide fractions?
Answer:
The steps to add, subtract, multiply, and divide decimals are:
a. Write the numbers vertically with the decimals lined up (if the number is a whole number then the decimal goes on the right end of the number)
b. Add zeros to make sure there is the same number of digits in each number.
c. Add or subtract as normal.
d. Multiply just as you would if the numbers were all whole numbers and add at the end.
e. After that count how many decimals places the two factors have (start at the far right of each number) and then give the answer that the total number of decimal places.
f. To divide decimals, always use long division.
The process to multiply and divide fractions is:
a. Dividing two fractions is the same as multiplying the first fraction by the reciprocal of the second fraction.
b. The first step to dividing fractions is to find the reciprocal (reverse the numerator and denominator) of the second fraction.
c. Next, multiply the two numerators. Then, multiply the two denominators.

Vocabulary Review

Complete each definition and then provide an example of each vocabulary word.

Vocabulary

reciprocal
dividend
fraction
product

Envision Math Common Core Grade 6 Answers Topic 1 Use Positive Rational Numbers 70.
Answer:

Use Vocabulary in Writing

Explain how to use multiplication to find the value of \(\frac{1}{3}\) ÷ \(\frac{9}{5}\) Use the words multiplication, divisor, quotient, and reciprocal in your explanation.
Answer:
The given division expression is:
\(\frac{1}{3}\) ÷ \(\frac{9}{5}\)
Where,
\(\frac{1}{3}\) is the dividend
\(\frac{9}{5}\) is the divisor
Now,
The “Reciprocal” of \(\frac{9}{5}\) is: \(\frac{5}{9}\)
So,
\(\frac{1}{3}\) ÷ \(\frac{9}{5}\)
= \(\frac{1}{3}\) × \(\frac{5}{9}\)
Now,
By using the Long Multiplication,

Hence, from the above,
We can conclude that
\(\frac{1}{3}\) ÷ \(\frac{9}{5}\) = \(\frac{5}{27}\)

Concepts and Skills Review

LESSON 1.1 Fluently Add, Subtract and Multiply Decimals

Quick Review
To add or subtract decimals, line up the decimal points so that place-value positions correspond. Add or subtract as you would with whole numbers, and place the decimal point in the answer. To multiply decimals, multiply as you would with whole numbers, then place the decimal point in the product by starting at the right and counting the number of places equal to the sum of the number of decimal places in each factor.

Example
Add, subtract, or multiply.
Envision Math Common Core Grade 6 Answers Topic 1 Use Positive Rational Numbers 70.2

Practice
Add, subtract, or multiply.

Question 1.
91.2 + 89.9
Answer:
The given Addition expression is: 91.2 + 89.9
Now,
By using the Long Addition,

Hence, from the above,
We can conclude that
91.2 + 89.9 = 181.1

Question 2.
902.3 – 8.8
Answer:
The given Subtraction expression is: 902.3 – 8.8
Now,
By using the Long Subtraction,

Hence, from the above,
We can conclude that
902.3 – 8.8 = 893.5

Question 3.
5 × 98.2
Answer:
The given Multiplication Expression is: 5 × 98.2
Now,
By using the Long Multiplication,

Hence, from the above,
We can conclude that
5 × 98.2 = 491

Question 4.
4 × 0.21
Answer:
The given Multiplication Expression is: 4 × 0.21
Now,
By using the Long Multiplication,

Hence, from the above,
We can conclude that
4 × 0.21 = 0.84

Question 5.
62.99 – 10.83
Answer:
The given Subtraction Expression is: 62.99 – 10.83
Now,
By using the Long Subtraction,

Hence, from the above,
We can conclude that
62.99 – 10.83 = 52.16

Question 6.
423.22 + 98.30
Answer:
The given Addition Expression is: 423.22 + 98.30
Now,
By using the Long Addition,

Hence, from the above,
We can conclude that
423.22 + 98.30 = 521.52

Question 7.
4.4 × 6
Answer:
The given Multiplication Expression is: 4.4 × 6
Now,
By using the Long Multiplication,

Hence, from the above,
We can conclude that
4.4 × 6 = 26.4

Question 8.
7 × 21.6
Answer:
The given Multiplication Expression is: 7 × 21.6
Now,
By using the Long Multiplication,

Hence, from the above,
We can conclude that
7 × 21.6 = 151.2

Question 9.
24.52 – 9.6
Answer:
The given Subtraction Expression is: 24.52 – 9.6
Now,
By using the Long Subtraction,

Hence, from the above,
We can conclude that
24.52 – 9.6 = 14.92

Question 10.
369.45 + 32.42
Answer:
The given Addition Expression is: 369.45 + 32.42
Now,
By using the Long Addition,

Hence, from the above,
We can conclude that
369.45 + 32.42 = 401.87

Question 11.
12.5 × 163.2
Answer:
The given Multiplication Expression is: 12.5 × 163.2
Now,
By using the Long Multiplication,

Hence, from the above,
We can conclude that
12.5 × 163.2 = 2,040

Question 12.
16 × 52.3
Answer:
The given Multiplication Expression is: 16 × 52.3
Now,
By using the Long Multiplication,

Hence, from the above,
We can conclude that
16 × 52.3 = 836.8

Question 13.
121.3 + 435.7
Answer:
The given Addition Expression is: 121.3 + 435.7
Now,
By using the Long Addition,

Hence, from the above,
We can conclude that
121.3 + 435.7 = 557

Question 14.
201.7 – 104.6
Answer:
The given Subtraction Expression is: 201.7 – 104.6
Now,
By using the Long Subtraction,

Hence, from the above,
We can conclude that
201.7 – 104.6 = 97.1

LESSON 1.2 Fluently Divide Whole Numbers and Decimals

Quick Review
To divide decimals, multiply the divisor and the dividend by the same power of 10 so that the divisor is a whole number. Then use an algorithm for whole-number division.

Example
Find 2.75 ÷ 0.05.
Envision Math Common Core Grade 6 Answers Topic 1 Use Positive Rational Numbers 80.1

Practice
Divide.

Question 1.
9.6 ÷ 1.6
Answer:
The given Division Expression is: 9.6 ÷ 1.6
Now,
By using the Long Division,

Hence, from the above,
We can conclude that
9.6 ÷ 1.6 = 6

Question 2.
48.4 ÷ 0.4
Answer:
The given Division Expression is: 48.4 ÷ 0.4
Now,
By using the Long Division,

Hence, from the above,
We can conclude that
48.4 ÷ 0.4 = 121

Question 3.
13.2 ÷ 0.006
Answer:
The given Division Expression is: 13.2 ÷ 0.006
Now,
By using the Long Division,

Hence, from the above,
We can conclude that
13.2 ÷ 0.006 = 2,200

Question 4.
10.8 ÷ 0.09
Answer:
The given Division Expression is: 10.8 ÷ 0.09
Now,
By using the Long Division,

Hence, from the above,
We can conclude that
10.8 ÷ 0.09 = 120

Question 5.
45 ÷ 4.5
Answer:
The given Division expression is: 45 ÷ 4.5
Now,
By using the Long Division,

Hence, from the above,
We can conclude that
45 ÷ 4.5 = 10

Question 6.
1,008 ÷ 1.8
Answer:
The given Division Expression is: 1,008 ÷ 1.8
Now,
By using the Long Division,

Hence, from the above,
We can conclude that
1,008 ÷ 1.8 = 560

Question 7.
1.26 ÷ 0.2
Answer:
The given Division Expression is: 1.26 ÷ 0.2
Now,
By using the Long Division,

Hence, from the above,
We can conclude that
1.26 ÷ 0.2 = 6.3

Question 8.
2.24 ÷ 3.2
Answer:
The given Division Expression is: 2.24 ÷ 3.2
Now,
By using the Long Division,

Hence, from the above,
We can conclude that
2.24 ÷ 3.2 = 0.7

Question 9.
35.75 ÷ 55
Answer:
The given Division Expression is: 35.75 ÷ 55
Now,
By using the Long Division,

Hence, from the above,
We can conclude that
35.75 ÷ 55 = 0.65

Question 10.
120.4 ÷ 602
Answer:
The given Division Expression is: 120.4 ÷ 602
Now,
By using the Long Division,

Hence, from the above,
We can conclude that
120.4 ÷ 602 = 0.2

Question 11.
330 ÷ 5.5
Answer:
The given Division Expression is: 330 ÷ 5.5
Now,
By using the Long Division,

Hence, from the above,
We can conclude that
330 ÷ 5.5 = 60

Question 12.
1.08 ÷ 0.027
Answer:
The given Division Expression is: 1.08 ÷ 0.027
Now,
By using the Long Division,

Hence, from the above,
We can conclude that
1.08 ÷ 0.027 = 40

LESSON 1.3 Multiply Fractions

Quick Review
Multiply the numerators to find the numerator of the product. Multiply the denominators to find the denominator of the product.

Example
Envision Math Common Core Grade 6 Answers Topic 1 Use Positive Rational Numbers 82.1
10 of the 18 rectangles are in the overlap area. So, \(\frac{2}{3}\) × \(\frac{5}{6}\) = \(\frac{10}{8}\) or \(\frac{5}{9}\).

Practice
Find each product.

Question 1.
\(\frac{2}{3}\) × \(\frac{3}{8}\)
Answer:
The given Multiplication Expression is:
\(\frac{2}{3}\) × \(\frac{3}{8}\)
Now,
By using the Long Multiplication,

Hence, from the above,
We can conclude that
\(\frac{2}{3}\) × \(\frac{3}{8}\) = \(\frac{1}{4}\)

Question 2.
\(\frac{1}{4}\) × \(\frac{3}{5}\)
Answer:
The given Multiplication Expression is:
\(\frac{1}{4}\) × \(\frac{3}{5}\)
Now,
By using the Long Multiplication,

Hence, from the above,
We can conclude that
\(\frac{1}{4}\) × \(\frac{3}{5}\) = \(\frac{3}{20}\)

Question 3.
\(\frac{1}{6}\) × \(\frac{1}{8}\)
Answer:
The given Multiplication Expression is:
\(\frac{1}{6}\) × \(\frac{1}{8}\)
Now,
By using the Long Multiplication,

Hence, from the above,
We can conclude that
\(\frac{1}{6}\) × \(\frac{1}{8}\) = \(\frac{1}{48}\)

Question 4.
\(\frac{4}{7}\) × \(\frac{4}{7}\)
Answer:
The given Multiplication Expression is:
\(\frac{4}{7}\) × \(\frac{4}{7}\)
Now,
By using the Long Multiplication,

Hence, from the above,
We can conclude that
\(\frac{4}{7}\) × \(\frac{4}{7}\) = \(\frac{16}{49}\)

Question 5.
\(\frac{6}{7}\) × \(\frac{1}{2}\)
Answer:
The given Multiplication Expression is:
\(\frac{6}{7}\) × \(\frac{1}{2}\)
Now,
By using the Long Multiplication,

Hence, from the above,
We can conclude that
\(\frac{6}{7}\) × \(\frac{1}{2}\) = \(\frac{3}{7}\)

Question 6.
\(\frac{3}{8}\) × \(\frac{8}{3}\)
Answer:
The given Multiplication Expression is:
\(\frac{3}{8}\) × \(\frac{8}{3}\)
Now,
By using the Long Multiplication,

Hence, from the above,
We can conclude that
\(\frac{3}{8}\) × \(\frac{8}{3}\) = 1

Question 7.
\(\frac{2}{3}\) × \(\frac{1}{3}\)
Answer:
The given Multiplication Expression is:
\(\frac{2}{3}\) × \(\frac{1}{3}\)
Now,
By using the Long Multiplication,

Hence, from the above,
We can conclude that
\(\frac{2}{3}\) × \(\frac{1}{3}\) = \(\frac{2}{9}\)

Question 8.
\(\frac{7}{8}\) × \(\frac{3}{2}\)
Answer:
The given Multiplication Expression is:
\(\frac{7}{8}\) × \(\frac{3}{2}\)
Now,
By using the Long Multiplication,

Hence, from the above,
We can conclude that
\(\frac{7}{8}\) × \(\frac{3}{2}\) = \(\frac{21}{16}\)

Question 9.
2\(\frac{1}{3}\) × 4\(\frac{1}{5}\)
Answer:
The given Multiplication Expression is:
2\(\frac{1}{3}\) × 4\(\frac{1}{5}\)
Now,
By using the Long Multiplication,

Hence, from the above,
We can conclude that
2\(\frac{1}{3}\) × 4\(\frac{1}{5}\) = \(\frac{49}{5}\)

Question 10.
4\(\frac{1}{2}\) × 6\(\frac{2}{3}\)
Answer:
The given Multiplication Expression is:
4\(\frac{1}{2}\) × 6\(\frac{2}{3}\)
Now,
By using the Long Multiplication,

Hence, from the above,
We can conclude that
4\(\frac{1}{2}\) × 6\(\frac{2}{3}\) = 30

Question 11.
3\(\frac{3}{5}\) × 2\(\frac{5}{7}\)
Answer:
The given Multiplication Expression is:
3\(\frac{3}{5}\) × 2\(\frac{5}{7}\)
Now,
By using the Long Multiplication,

Hence, from the above,
We can conclude that
3\(\frac{3}{5}\) × 2\(\frac{5}{7}\) = \(\frac{342}{35}\)

Question 12.
14\(\frac{2}{7}\) × 4\(\frac{3}{10}\)
Answer:
The given Multiplication Expression is:
14\(\frac{2}{7}\) × 4\(\frac{3}{10}\)
Now,
By using the Long Multiplication,

Hence, from the above,
We can conclude that
14\(\frac{2}{7}\) × 4\(\frac{3}{10}\) = \(\frac{430}{7}\)

LESSONS 1.4 AND 1.5 Understand and Divide with Fractions

Quick Review
To divide by a fraction, use the reciprocal of the divisor to rewrite the problem as a multiplication problem.
Practice Find each quotient.

Example
Find 4 ÷ \(\frac{4}{5}\).
Envision Math Common Core 6th Grade Answer Key Topic 1 Use Positive Rational Numbers 83.2

Find \(\frac{3}{4}\) ÷ \(\frac{4}{5}\).
Envision Math Common Core 6th Grade Answer Key Topic 1 Use Positive Rational Numbers 83.3

Practice

Find each Quotient

Question 1.
7 ÷ \(\frac{1}{2}\)
Answer:
The given Division Expression is:
7 ÷ \(\frac{1}{2}\)
Now,
By using the Long Division,

Hence, from the above,
We can conclude that
7 ÷ \(\frac{1}{2}\) = 14
The quotient for the given division expression is: 14

Question 2.
6 ÷ \(\frac{2}{5}\)
Answer:
The given Division Expression is:
6 ÷ \(\frac{2}{5}\)
Now,
By using the Long Division,

Hence, from the above,
We can conclude that
6 ÷ \(\frac{2}{5}\) = 15
The quotient for the given division expression is: 15

Question 3.
2 ÷ \(\frac{1}{8}\)
Answer:
The given Division Expression is:
2 ÷ \(\frac{1}{8}\)
Now,
By using the Long Division,

Hence, from the above,
We can conclude that
2 ÷ \(\frac{1}{8}\) = 16
The quotient for the given division expression is: 16

Question 4.
8 ÷ \(\frac{4}{9}\)
Answer:
The given Division Expression is:
8 ÷ \(\frac{4}{9}\)
Now,
By using the Long Division,

Hence, from the above,
We can conclude that
8 ÷ \(\frac{4}{9}\) = 18
The quotient for the given division expression is: 18

Question 5.
\(\frac{1}{2}\) ÷ \(\frac{1}{4}\)
Answer:
The given Division Expression is:
\(\frac{1}{2}\) ÷ \(\frac{1}{4}\)
Now,
By using the Long Division,

Hence, from the above,
We can conclude that
\(\frac{1}{2}\) ÷ \(\frac{1}{4}\) = 2
The quotient for the given division expression is: 2

Question 6.
\(\frac{8}{10}\) ÷ \(\frac{1}{5}\)
Answer:
The given Division Expression is:
\(\frac{8}{10}\) ÷ \(\frac{1}{5}\)
Now,
By using the Long Division,

Hence, from the above,
We can conclude that
\(\frac{8}{10}\) ÷ \(\frac{1}{5}\) = 4
The quotient for the given division expression is: 4

Question 7.
\(\frac{5}{6}\) ÷ \(\frac{3}{8}\)
Answer:
The given Division Expression is:
\(\frac{5}{6}\) ÷ \(\frac{3}{8}\)
Now,
By using the Long Division,

Hence, from the above,
We can conclude that
\(\frac{5}{6}\) ÷ \(\frac{3}{8}\) = \(\frac{20}{9}\)
The quotient for the given division expression is: \(\frac{20}{9}\)

Question 8.
\(\frac{1}{3}\) ÷ \(\frac{1}{2}\)
Answer:
The given Division Expression is:
\(\frac{1}{3}\) ÷ \(\frac{1}{2}\)
Now,
By using the Long Division,

Hence, from the above,
we can conclude that
\(\frac{1}{3}\) ÷ \(\frac{1}{2}\) = \(\frac{2}{3}\)
The quotient for the given division expression is: \(\frac{2}{3}\)

Question 9.
5 ÷ \(\frac{5}{16}\)
Answer:
The given Division Expression is:
5 ÷ \(\frac{5}{16}\)
Now,
By using the Long Division,

Hence, from the above,
We can conclude that
5 ÷ \(\frac{5}{16}\) = 16
The quotient for the given division expression is: 16

Question 10.
\(\frac{7}{12}\) ÷ \(\frac{3}{4}\)
Answer:
The given Divisio Expression is:
\(\frac{7}{12}\) ÷ \(\frac{3}{4}\)
Now,
By using the Long Division,

Hence, from the above,
We can conclude that
\(\frac{7}{12}\) ÷ \(\frac{3}{4}\) = \(\frac{7}{9}\)
The quotient for the given division expression is: \(\frac{7}{9}\)

Question 11.
20 ÷ \(\frac{5}{6}\)
Answer;
The given Division Expression is:
20 ÷ \(\frac{5}{6}\)
Now,
By using the Long Division,

Hence, from the above,
We can conclude that
20 ÷ \(\frac{5}{6}\) = 24
The quotient for the given division expression is: 24

Question 12.
16 ÷ \(\frac{1}{4}\)
Answer:
The given Division Expression is:
16 ÷ \(\frac{1}{4}\)
Now,
By using the Long Division,

Hence, from the above,
We can conclude that
16 ÷ \(\frac{1}{4}\) = 64
The quotient for the given division expression is: 64

Question 13.
\(\frac{4}{5}\) ÷ \(\frac{1}{8}\)
Answer:
The given Division Expression is:
\(\frac{4}{5}\) ÷ \(\frac{1}{8}\)
Now,
By using the Long Division,

Hence, from the above,
We can conclude that
\(\frac{4}{5}\) ÷ \(\frac{1}{8}\) = \(\frac{32}{5}\)
The quotient for the given division expression is: \(\frac{32}{5}\)

Question 14.
5 ÷ \(\frac{1}{10}\)
Answer:
The given Division Expression is:
5 ÷ \(\frac{1}{10}\)
Now,
By using the Long Division,

Hence, from the above,
We can conclude that
5 ÷ \(\frac{1}{10}\) = 50
The quotient for the given division expression is: 50

Question 15.
\(\frac{7}{11}\) ÷ \(\frac{1}{11}\)
Answer:
The given Division Expression is:
\(\frac{7}{11}\) ÷ \(\frac{1}{11}\)
Now,
By using the Long Division,

Hence, from the above,
We can conclude that
\(\frac{7}{11}\) ÷ \(\frac{1}{11}\) = 7
The quotient for the given division expression is: 7

Question 16.
4 ÷ \(\frac{2}{8}\)
Answer:
The given Division Expression is:
4 ÷ \(\frac{2}{8}\)
Now,
By using the Long Division,

Hence, from the above,
We ca conclude that
4 ÷ \(\frac{2}{8}\) = 16
The quotient for the given division expression is: 16

LESSON 1.6 Divide Mixed Numbers

Quick Review
To divide by a mixed number, rename each mixed number as a fraction. Then use the reciprocal of the divisor to rewrite the problem as a multiplication problem.

Example
Envision Math Common Core 6th Grade Answer Key Topic 1 Use Positive Rational Numbers 88.1
Practice
Find each quotient

Question 1.
6\(\frac{3}{8}\) ÷ 4\(\frac{1}{4}\)
Answer:
The given Division Expression is:
6\(\frac{3}{8}\) ÷ 4\(\frac{1}{4}\)
Now,
By using the Long Division,

Hence, from the above,
We can conclude that
6\(\frac{3}{8}\) ÷ 4\(\frac{1}{4}\) = \(\frac{3}{2}\)
The quotient for the given division expression is: \(\frac{3}{2}\)

Question 2.
9 ÷ 2\(\frac{2}{7}\)
Answer:
The given Division Expression is:
9 ÷ 2\(\frac{2}{7}\)
Now,
By using the Long Division,

Hence, from the above,
We can conclude that
9 ÷ 2\(\frac{2}{7}\) = \(\frac{63}{16}\)
The quotient for the given division expression is: \(\frac{63}{16}\)

Question 3.
3\(\frac{3}{5}\) ÷ 1\(\frac{1}{5}\)
Answer:
The given Division Expression is:
3\(\frac{3}{5}\) ÷ 1\(\frac{1}{5}\)
Now,
By using the Long Division,

Hence, from the above,
We can conclude that
3\(\frac{3}{5}\) ÷ 1\(\frac{1}{5}\) = 3
The quotient for the given division expression is: 3

Question 4.
5\(\frac{1}{2}\) ÷ 3\(\frac{3}{8}\)
Answer:
The given Division Expression is:
5\(\frac{1}{2}\) ÷ 3\(\frac{3}{8}\)
Now,
By using the Long Division,

Hence, from the above,
We can conclude that
5\(\frac{1}{2}\) ÷ 3\(\frac{3}{8}\) = \(\frac{44}{27}\)
The quotient for the given division expression is: \(\frac{4}{27}\)

Question 5.
3\(\frac{2}{5}\) ÷1\(\frac{1}{5}\)
Answer:
The given Division Expression is:
3\(\frac{2}{5}\) ÷1\(\frac{1}{5}\)
Now,
By using the Long Division,

Hence, from the above,
We can conclude that
3\(\frac{2}{5}\) ÷1\(\frac{1}{5}\) = \(\frac{17}{6}\)
The quotient for the given division expression is: \(\frac{17}{6}\)

Question 6.
12\(\frac{1}{6}[/ltex] ÷ 3
Answer:
The given Division Expression is:
12[latex]\frac{1}{6}[/ltex] ÷ 3
Now,
By using the Long Division,

Hence, from the above,
We ca conclude that
12[latex]\frac{1}{6}[/ltex] ÷ 3 = [latex]\frac{73}{18}\)
The quotient for the given division expression is: \(\frac{73}{18}\)

Question 7.
12 ÷ 1\(\frac{1}{2}\)
Answer:
The given Division Expression is:
12 ÷ 1\(\frac{1}{2}\)
Now,
By using the Long Division,

Hence, from the above,
We can conclude that
12 ÷ 1\(\frac{1}{2}\) = 8
The quotient for the given division expression is: 8

Question 8.
3\(\frac{1}{2}\) ÷ 2\(\frac{1}{4}\)
Answer:
The given Division Expression is:
3\(\frac{1}{2}\) ÷ 2\(\frac{1}{4}\)
Now,
By using the Long Division,

Hence, from the above,
We can conclude that
3\(\frac{1}{2}\) ÷ 2\(\frac{1}{4}\) = \(\frac{14}{9}\)
The quotient for the given division expression is: \(\frac{14}{9}\)

Question 9.
8 ÷ 1\(\frac{1}{4}\)
Answer:
The given Division Expression is:
8 ÷ 1\(\frac{1}{4}\)
Now,
By using the Long Division,

Hence, from the above,
We can conclude that
8 ÷ 1\(\frac{1}{4}\) = \(\frac{32}{5}\)
The quotient for the given division expression is: \(\frac{32}{5}\)

Question 10.
10\(\frac{1}{2}\) ÷ 1\(\frac{3}{4}\)
Answer:
The given Division Expression is:
10\(\frac{1}{2}\) ÷ 1\(\frac{3}{4}\)
Now,
By using the Long Division,

Hence, from the above,
We can conclude that
10\(\frac{1}{2}\) ÷ 1\(\frac{3}{4}\) = 6
The quotient for the given division expression is: 6

Question 11.
3\(\frac{3}{4}\) ÷ 2\(\frac{1}{2}\)
Answer:
The given Division Expression is:
3\(\frac{3}{4}\) ÷ 2\(\frac{1}{2}\)
Now,
By using the Long Division,

Hence, from the above,
We can conclude that
3\(\frac{3}{4}\) ÷ 2\(\frac{1}{2}\) = \(\frac{3}{2}\)
The quotient for the given division expression is: \(\frac{3}{2}\)

Question 12.
60 ÷ 3\(\frac{1}{3}\)
Answer:
The given Division Expression is:
60 ÷ 3\(\frac{1}{3}\)
Now,
By using the Long Division,

Hence, from the above,
We can conclude that
60 ÷ 3\(\frac{1}{3}\) = 18
The quotient for the given division expression is: 18

LESSON 1.7 Solve Problems with Rational Numbers

Quick Review
When solving multistep problems:
• decide the steps to solve the problem.
• choose the correct operations.
• identify the information you need from the problem.
• correctly use the information.
• calculate accurately.
• check if the answer is reasonable.

Example
Jane’s garden is 3.4 meters by 6.5 meters. If fencing costs $2.25 per meter, how much will it cost to enclose Jane’s garden?
Step 1: Find how much fence is needed.
3.4 + 3.4 + 6.5 + 6.5 = 19.8 meters
Step 2: Multiply to find the cost.
19.8 × 2.25 = $44.55
Step 3: Estimate to check.
3 + 3 + 7 + 7 = 20 meters
20 × 2.00 = $40.00
$40 is close to $44.55, so the answer is reasonable.

Practice
Daisy has one cucumber that is 3 inches long and another cucumber that is 5 inches long. She cuts the cucumbers into \(\frac{3}{8}\)-inch-thick slices and adds them to a salad. How many \(\frac{3}{8}\)-inch-thick slices does Daisy have?

Question 1.
Write division expressions to represent the first steps in the problem.
Answer:
It is given that
Daisy has one cucumber that is 3 inches long and another cucumber that is 5 inches long. She cuts the cucumbers into \(\frac{3}{8}\)-inch-thick slices and adds them to a salad.
So,
The total length of 2 cucumbers = (The length of the first cucumber) + (The length of the second cucumber)
= 3 + 5
= 8 inches
Now,
The number of \(\frac{3}{8}\)-inch-thick slices does Daisy have = (The total length of 2 cucumbers) ÷ \(\frac{3}{8}\)
= 8 ÷ \(\frac{3}{8}\)
Hence, from the above,
We can conclude that
The division expression to represent the given information is:
The number of \(\frac{3}{8}\)-inch-thick slices do Daisy have = 8 ÷ \(\frac{3}{8}\)

Question 2.
Solve. Then explain your answer.
Answer:
From Question 1,
We can observe that
The number of \(\frac{3}{8}\)-inch-thick slices does Daisy have = (The total length of 2 cucumbers) ÷ \(\frac{3}{8}\)
= 8 ÷ \(\frac{3}{8}\)
Now,
By using the Long Division,

Hence, from the above,
We can conclude that
The number of \(\frac{3}{8}\)-inch-thick slices does Daisy have is about 21 pieces

Topic 1 Fluency Practice

Pathfinder
Shade a path from START to FINISH. Follow the solutions in which the digit in the hundredths place is greater than the digit in the tenths place. You can only move up, down, right, or left.
Envision Math Common Core 6th Grade Answer Key Topic 1 Use Positive Rational Numbers 100

enVision Math Common Core Grade 5 Answer Key Topic 14 Graph Points on the Coordinate Plane

Practice with the help of enVision Math Common Core Grade 5 Answer Key Topic 14 Graph Points on the Coordinate Plane regularly and improve your accuracy in solving questions.

enVision Math Common Core 5th Grade Answers Key Topic 14 Graph Points on the Coordinate Plane

Envision Math Common Core Grade 5 Answer Key Topic 14 Graph Points on the Coordinate Plane 1
enVision STEM Project: Earth’s Rotation
Do Research Use the Internet or other sources to find out more about Earth’s rotation. Investigate why it appears that the Sun is moving across the sky. Design a model to explain Earth’s day/night cycle. Compare Earth’s rotation to another planet’s rotation.
Write a Report: Journal Include what you found. Also in your report:
• Write a step-by-step procedure of how to use a ball and a flashlight to represent the day/night cycle.
• Explain what happens if the ball rotates slowly. What happens if the ball rotates quickly?
• Make up and solve problems for plotting points and using graphs to show relationships.

Review What You Know

A-Z Vocabulary
Choose the best term from the Word List. Write it on the blank.

• equation
• factor
• line plot
• numerical expression
• variable

Question 1.
A(n) ____ contains numbers and at least one operation.

Answer:
A(n) numerical expression contains numbers and at least one operation.

Explanation:
In the above-given question,
given that,
A(n) numerical expression contains numbers and at least one operation.
for example:
8x + 4y.
where x and y are variables.
8 and 4 are constants.

Question 2.
A letter or symbol that represents an unknown amount is a(n) ____.

Answer:
A letter or symbol that represents an unknown amount is a(n) variable.

Explanation:
In the above-given question,
given that,
A letter or symbol that represents an unknown amount is a(n) variable.
for example:
8 + n = 12.
a variable is a letter or symbol that represents a number(unknown quantity).

Question 3.
A number sentence that uses the = symbol is a(n) ____.

Answer:
A number sentence that uses the = symbol is a(n) equation.

Explanation:
In the above-given question,
given that,
A number sentence that uses the = symbol is a(n) equation.
for example:
4 + 5 = 9.
8 – 2 = 6.

Question 4.
A display that shows Xs or dots above a number line is a(n) ___.

Answer:
A display that shows Xs or dots above a number line is a(n) line plot.

Explanation:
In the above-given question,
given that,
A display that shows Xs or dots above a number line is a(n) line plot.
for example:
x < 3.

Evaluate Expressions
Evaluate each numerical expression.

Question 5.
3 × 4 × (10 – 7) ÷ 2

Answer:
3 x 4 x (10 – 7) / 2 = 12.

Explanation:
In the above-given question,
given that,
the equation is 3 x 4 x (10 – 7) / 2.
3 x 4 x (3)/3.
3 x 4 x 1.
3 x 4 = 12.
3 x 4 x (10 – 7) / 2 = 12.

Question 6.
(8 + 2) 6 – 4

Answer:
(8 + 2) 6 – 4 = 12.

Explanation:
In the above-given question,
given that,
the equation is (8 + 2) 6 – 4.
(6) 6 – 4.
(6) 2.
6 x 2 = 12.
(8 + 2) 6 – 4 = 12.

Question 7.
8 + 2 × 6 – 4

Answer:
8 + 2 x 6 – 4 = 20.

Explanation:
In the above-given question,
given that,
the equation is 8 + 2 x 6 – 4.
10 x 6 – 4.
10 x 2 = 20.
8 + 2 x 6 – 4 = 20.

Question 8.
40 ÷ 5 + 5 × (3 – 1)

Answer:
40 / 5 + 5 x (3 – 1) = 18.

Explanation:
In the above-given question,
given that,
the equation is 40 / 5 + 5 x (3 – 1).
40 / 5 + 5 x (2).
8 + 5 x 2.
8 + 10 = 18.
40 / 5 + 5 x (3 – 1) = 18.

Question 9.
15 ÷ 3 + 2 × 10

Answer:
15 / 3 + 2 x 10 =

Explanation:
In the above-given question,
given that,
the equation is 15 / 3 + 2 x 10.
5 + 2 x 10.
5 + 20.
25.
15 / 3 + 2 x 10 = 25.

Question 10.
21 × (8 – 6) ÷ 14

Answer:
21 x (8 – 6) / 14 = 3.

Explanation:
In the above-given question,
given that,
the equation is 21 x (8 – 6) / 14.
21 x ( 2) / 14.
21 x 1/7.
21 / 7 = 3.
21 x (8 – 6) / 14 = 3.

Write Expressions
Write a numerical expression for each word phrase.

Question 11.
Three less than the product of eight and six

Answer:
Three less than the product of eight and six = 45.

Explanation:
In the above-given question,
given that,
three less than the product of eight and six.
product of 6 and 8 is 48.
6 x 8 = 48.
48 – 3 = 45.

Question 12.
Thirteen more than the quotient of twenty divided by four

Answer:
Thirteen more than the quotient of twenty divided by four is 18.

Explanation:
In the above-given question,
given that,
thirteen more than the quotient of twenty divided by four is 18.
20 / 4 = 5.
5 + 13 = 18.
so thirteen more than the quotient of twenty divided by four is 18.

Question 13.
Four times the difference between seven and two

Answer:
Four times the difference between seven and two is 20.

Explanation:
In the above-given question,
given that,
Four times the difference between seven and two is 20.
7 – 2 = 5.
4 x 5 = 20.
so four times the difference between seven and two is 20.

Compare Expressions

Question 14.
Use < or > to compare 13 × (54 + 28) and 13 × 54 + 28 without calculating. Explain your reasoning.

Answer:
13 x (54 + 28) < 13 x 54 + 28.

Explanation:
In the above-given question,
given that,
the equation is 13 x (54 + 28).
13 x 82 = 1066.
13 x 54 + 28.
702 + 28 = 730.
so 13 x (54 + 28) < 13 x 54 + 28.

Pick a Project

PROJECT 14A
What does a city planner do?
Project: Plan a City
Envision Math Common Core Grade 5 Answer Key Topic 14 Graph Points on the Coordinate Plane 6.1

PROJECT 14B
What are some of the oldest childhood games?
Project: Make Your Own Game
Envision Math Common Core Grade 5 Answer Key Topic 14 Graph Points on the Coordinate Plane 6.2

PROJECT 14C
How can dogs help rescue people?
Project: Write a Story of a Missing Hiker
Envision Math Common Core Grade 5 Answer Key Topic 14 Graph Points on the Coordinate Plane 6.3

PROJECT 14D
How can artists use grids in their work?
Project: Draw a Picture Using a Grid
Envision Math Common Core Grade 5 Answer Key Topic 14 Graph Points on the Coordinate Plane 6.4

Lesson 14.1 The Coordinate System

Activity

Solve&Share
On the first grid, plot a point where two lines intersect. Name the location of the point. Plot and name another point. Work with a partner. Take turns describing the locations of the points on your first grid. Then plot the points your partner describes on your second grid. Compare your first grid with your partner’s second grid to see if they match. Use the grids below to solve this problem.
Envision Math Common Core Grade 5 Answer Key Topic 14 Graph Points on the Coordinate Plane 6.5

You can use grid paper to graph ordered pairs. Show your work!
Envision Math Common Core Grade 5 Answer Key Topic 14 Graph Points on the Coordinate Plane 6.6

Look Back! Construct Arguments Why does the order of the two numbers that name a point matter? Explain your thinking

Visual Learning Bridge

Essential Question
How Do You Name a Point on a Coordinate Grid?

A.
A map shows the locations of landmarks and has guides for finding them. In a similar way, a coordinate grid is used to graph and name the locations of points in a plane.
Envision Math Common Core Grade 5 Answer Key Topic 14 Graph Points on the Coordinate Plane 7.1

You can use ordered pairs to locate points on a coordinate grid.
Envision Math Common Core Grade 5 Answer Key Topic 14 Graph Points on the Coordinate Plane 7.2

B.
A coordinate grid has a horizontal x-axis and a vertical y-axis. The point at which the x-axis and y-axis intersect is called the origin.
Envision Math Common Core Grade 5 Answer Key Topic 14 Graph Points on the Coordinate Plane 7.3

C.
A point on the grid is named using an ordered pair of numbers. The first number, the x-coordinate, names the distance from the origin along the x-axis. The second number, the y-coordinate, names the distance from the origin along the y-axis.
Envision Math Common Core Grade 5 Answer Key Topic 14 Graph Points on the Coordinate Plane 7.4

Convince Me! Reasoning in the example above, name the ordered pair for Point B if it is 3 units to the right of Point A. Tell how you decided

Answer:
The points are (3, 3).

Explanation:
In the above-given question,
given that,
the point B if it is 3 units to the right of point A.
the point is (1, 3).
on the x-axis, the point is 1.
on the y-axis, the point is 3.
so the point is (1,3).
if it is 3 units to the right of point A.
so the points are (3, 3).

Guided Practice

Do You Understand?

Question 1.
You are graphing Point E at (0,5). Do you move to the right zero units, or up zero units? Explain.

Answer:
We will move up to zero units.

Explanation:
In the above-given question,
given that,
the point is (0, 5).
the point 0 on the x-axis.
the point 5 on the y-axis.
we have to move up zero units.
so we will move up to zero units.

Question 2.
A-Z Vocabulary What ordered pair names the origin of any coordinate grid?

Answer:
The points (0, 0) names the origin of any coordinate grid.

Explanation:
In the above-given question,
given that,
the ordered pair names the origin of any coordinate grid.
the points are (0, 0), (1, 1), (2, 2), (3, 3) and so on.
the points (0, 0) names the origin of any coordinate grid.

Question 3.
Describe how to graph Point K at (5, 4).

Answer:
Point 5 on the x-axis and 4 on the y-axis.

Explanation:
In the above-given question,
given that,
the point k at (5, 4) on the co-ordinate axis.
we have to move up zero units.
point 5 is on the x-axis.
point 4 is on the y-axis.
point 5 on the x-axis and 4 on the y-axis.

Do You Know How?

In 4 and 5, write the ordered pair for each point. Use the grid.

Question 4.
B

Envision Math Common Core Grade 5 Answer Key Topic 14 Graph Points on the Coordinate Plane 8.1

Answer:
The ordered pairs are (3, 2), (1, 4), and (5, 3).

Explanation:
In the above-given question,
given that,
the points are A, B, C, and D.
the points are (1, 4).
1 is on the x-axis.
4 is on the y-axis.
the point is (3, 2).
3 is on the x-axis.
2 is on the y-axis.
the point is (5, 3).
5 is on the x-axis.
3 is on the y-axis.
so the ordered pairs are (3, 2), (1, 4), and (5, 3).

Question 5.
A
Answer:

In 6 and 7, name the point for each ordered pair on the grid above.

Question 6.
(5, 3)

Answer:
Point 5 moves up to 3 units.

Explanation:
In the above-given question,
given that,
point 5 is on the x-axis.
point 3 is on the y-axis.
so the point is (5, 3).
point 5 moves up zero units.

Question 7.
(1, 4)

Answer:
Point 1 moves up to 4 units.

Explanation:
In the above-given question,
given that,
point 1 is on the x-axis.
point 4 is on the y-axis.
so the point is (1, 4).
point 1 moves up to 4 units.

Independent Practice

In 8-13, write the ordered pair for each point. Use the grid.

Envision Math Common Core Grade 5 Answer Key Topic 14 Graph Points on the Coordinate Plane 9.1

Question 8.
T

Answer:
The ordered pair is (0, 5).

Explanation:
In the above-given question,
given that,
point 0 is on the origin.
point 5 is on the y-axis.
the ordered pair is (0, 5).
so the ordered pair is (0, 5).

Question 9.
X

Answer:
The ordered pair is (4, 3).

Explanation:
In the above-given question,
given that,
point 4 is on the x-axis.
point 3 is on the y-axis.
the ordered pair is (4, 3).
so the ordered pair is (4, 3).

Question 10.
Y

Answer:
The ordered pair is (1, 0).

Explanation:
In the above-given question,
given that,
point 1 is on the X-axis.
point 0 is on the y-axis.
the ordered pair is (1, 0).
so the ordered pair is (1, 0).

Question 11.
W

Answer:
The ordered pair is (3, 3).

Explanation:
In the above-given question,
given that,
point 3 is on the x-axis.
point 3 is on the y-axis.
the ordered pair is (3, 3).
so the ordered pair is (3, 3).

Question 12.
Z

Answer:
The ordered pair is (1, 4).

Explanation:
In the above-given question,
given that,
point 1 is on the x-axis.
point 4 is on the y-axis.
the ordered pair is (1, 4).
so the ordered pair is (1, 4).

Question 13.
S

Answer:
The ordered pair is (5, 5).

Explanation:
In the above-given question,
given that,
point 5 is on the x-axis.
point 5 is on the y-axis.
the ordered pair is (5, 5).
so the ordered pair is (5, 5).

In 14-18, name the point for each ordered pair on the grid above.

Question 14.
(2, 2)

Answer:
The point is L.

Explanation:
In the above-given question,
given that,
the point is (2,2).
2 is on the x-axis.
2 is on the y-axis.
so the point is 2.

Question 15.
(5, 4)

Answer:
The point is Q.

Explanation:
In the above-given question,
given that,
the point is (5, 4).
5 is on the x-axis.
4 is on the y-axis.
so the point is Q.

Question 16.
(1, 5)

Answer:
The point is N.

Explanation:
In the above-given question,
given that,
the point is (1, 5).
1 is on the x-axis.
5 is on the y-axis.
so the point is N.

Question 17.
(0, 3)

Answer:
The point is M.

Explanation:
In the above-given question,
given that,
the point is (0, 3).
0 is on the x-axis.
3 is on the y-axis.
so the point is M.

Question 18.
(4, 0)

Answer:
The point is P.

Explanation:
In the above-given question,
given that,
the point is (4, 0).
4 is on the x-axis.
0 is on the y-axis.
so the point is p.

Problem Solving

Question 19.
Higher Order Thinking Describe to a friend how to find and name the ordered pair for Point R on the grid.
Envision Math Common Core Grade 5 Answer Key Topic 14 Graph Points on the Coordinate Plane 9.3

Answer:
The point is (4, 5).

Explanation:
In the above-given question,
given that,
the point is R.
4 is on the x-axis.
5 is on the y-axis.
so the point is (4, 5).

In 20-24, complete the table. List the point and ordered pair for each vertex of the pentagon at the right.

Envision Math Common Core Grade 5 Answer Key Topic 14 Graph Points on the Coordinate Plane 9.4

Envision Math Common Core Grade 5 Answer Key Topic 14 Graph Points on the Coordinate Plane 9.5

Answer:
The points are B, C, D, E, and F.

Explanation:
In the above-given question,
given that,
The point E is on the ordered pair (1, 1).
the point D is on the ordered pair (1, 2).
point C is on the ordered pair (1, 3).
point B is on the ordered pair (2, 4).
point F is on the ordered pair (3, 1).

Question 25.
Reasoning Why is the order important when naming or graphing the coordinates of a point?

Answer:
The order is important when naming or graphing the coordinates of a point.

Explanation:
In the above-given question,
given that,
the order is important because A is on the (3, 3).
B is on the (2, 4).
C is on the (1, 3).
D is on the (1, 2).
E is on the (1, 1).
F is on the (3, 1).
G is on the (5, 1).

Question 26.
How are the x-axis and the y-axis related on a coordinate grid?

Answer:
The x-axis and the y-axis are related.

Explanation:
In the above-given question,
given that,
the order is important because A is on the (3, 3).
B is on the (2, 4).
C is on the (1, 3).
D is on the (1, 2).
E is on the (1, 1).
F is on the (3, 1).
G is on the (5, 1).

Assessment Practice

Question 27.
Which of the following points is located at (4, 2)?
Envision Math Common Core Grade 5 Answer Key Topic 14 Graph Points on the Coordinate Plane 10.1
A. Point A
B. Point M
C. Point B
D. Point P

Answer:
Option M is correct.

Explanation:
In the above-given question,
given that,
the point is (4, 2).
4 is located on the x-axis.
2 is located on the y-axis.
so option M is correct.

Lesson 14.2 Graph Data Using Ordered Pairs

Activity

Sole & Share
Graph and label the point for each ordered pair below on the grid. Then connect the points with line segments to form a shape. What shape did you draw?

Use Appropriate Tools You can graph points on a coordinate grid. Show your work!
Envision Math Common Core Grade 5 Answer Key Topic 14 Graph Points on the Coordinate Plane 10.2

Answer:
The shape formed is square.

Explanation:
In the above-given question,
given that,
the ordered pairs are (2, 1), (5, 1), (5, 4), and (2, 4).
the point (2, 1) is on the A.
the point (5, 1) is on the B.
the point (5, 4) is on the C.
the point (2, 4) is on the D.
so the shape formed is square.
Envision-Math-Common-Core-5th-Grade-Answers-Key-Topic-14-Graph Points on the Coordinate Plane-1

Look Back! What tool could you use to help connect points A, B, C, and D? Explain.

Visual Learning Bridge

Essential Question How Do You Graph a Point on a Coordinate Grid?

A.
The table below shows the growth of a plant over a period of several days. Graph ordered pairs to show the plant’s growth.
Envision Math Common Core Grade 5 Answer Key Topic 14 Graph Points on the Coordinate Plane 10.4
Let x be the number of days and let y be the height of the plant in centimeters.
Envision Math Common Core Grade 5 Answer Key Topic 14 Graph Points on the Coordinate Plane 10.5
The ordered pairs are (1, 4), (3, 8), (5, 10), (7, 11), and (9, 14).

B.
Step 1
Graph the first point (1, 4).
Start at (0, 0). Move 1 unit to the right along the x-axis. Then move 4 units up.
Envision Math Common Core Grade 5 Answer Key Topic 14 Graph Points on the Coordinate Plane 10.6

C.
Step 2
Plot the rest of the ordered pairs from the table. Use a ruler to connect the points.
Envision Math Common Core Grade 5 Answer Key Topic 14 Graph Points on the Coordinate Plane 10.7

Convince Me! Reasoning Based on the data, about how tall was the plant on day 4? Day 8?

Guided Practice

Do You Understand?

Question 1.
Natalie is graphing Point T at (1,8). Should she move to the right 8 units or up 8 units? Explain.

Answer:
She moves 8 units up.

Explanation:
In the above-given question,
given that,
the point T at (1, 8).
point 1 is on the x-axis.
point 8 is on the y-axis.
so she moves 8 units up.

Question 2.
Describe how to graph the point (c, d).

Answer:

Do You Know How?

In 3-6, graph each point on the grid and label it with the appropriate letter.

Envision Math Common Core Grade 5 Answer Key Topic 14 Graph Points on the Coordinate Plane 11.1

Question 3.
E (1, 3)

Answer:
The point is E.

Explanation:
In the above-given question,
given that,
point 1 is on the x-axis.
point 3 is on the y-axis.
the ordered pair is (1, 3).
so the point is E.

Question 4.
F (4, 4)

Answer:
The point is F.

Explanation:
In the above-given question,
given that,
point 4 is on the x-axis.
point 4 is on the y-axis.
the ordered pair is (4, 4).
so the point is F.

Question 5.
G (5, 2)

Answer:
The point is G.

Explanation:
In the above-given question,
given that,
point 5 is on the x-axis.
point 2 is on the y-axis.
the ordered pair is (5, 2).
so the point is G.

Question 6.
H (0, 2)

Answer:
The point is H.

Explanation:
In the above-given question,
given that,
point 0 is on the x-axis.
point 2 is on the y-axis.
the ordered pair is (0, 2).
so the point is H.
Envision-Math-Common-Core-5th-Grade-Answers-Key-Topic-14-Graph Points on the Coordinate Plane-2

Independent Practice

In 7-18, graph and label each point on the grid at the right.

Envision Math Common Core 5th Grade Answers Topic 14 Graph Points on the Coordinate Plane 11.5

Question 7.
J (2, 6)

Answer:
The point is J.

Explanation:
In the above-given question,
given that,
point 2 is on the x-axis.
point 6 is on the y-axis.
the ordered pair is (2, 6).
so the point is J.

Question 8.
K (6, 2)

Answer:
The point is K.

Explanation:
In the above-given question,
given that,
point 6 is on the x-axis.
point 2 is on the y-axis.
the ordered pair is (6, 2).
so the point is K.

Question 9.
L (4, 5)

Answer:
The point is L.

Explanation:
In the above-given question,
given that,
point 4 is on the x-axis.
point 5 is on the y-axis.
the ordered pair is (4, 5).
so the point is L.

Question 10.
M (0, 8)

Answer:
The point is M.

Explanation:
In the above-given question,
given that,
point 0 is on the x-axis.
point 8 is on the y-axis.
the ordered pair is (0, 8).
so the point is M.

Question 11.
N (3, 9)

Answer:
The point is N.

Explanation:
In the above-given question,
given that,
point 3 is on the x-axis.
point 9 is on the y-axis.
the ordered pair is (3, 9).
so the point is N.

Question 12.
V (6, 6)

Answer:
The point is V.

Explanation:
In the above-given question,
given that,
point 6 is on the x-axis.
point 6 is on the y-axis.
the ordered pair is (6, 6).
so the point is V.

Question 13.
P (1, 4)

Answer:
The point is P.

Explanation:
In the above-given question,
given that,
point 1 is on the x-axis.
point 4 is on the y-axis.
the ordered pair is (1, 4).
so the point is P.

Question 14.
Q (5, 0)

Answer:
The point is Q.

Explanation:
In the above-given question,
given that,
point 5 is on the x-axis.
point 0 is on the y-axis.
the ordered pair is (5, 0).
so the point is Q.

Question 15.
R (7, 3)

Answer:
The point is R.

Explanation:
In the above-given question,
given that,
point 7 is on the x-axis.
point 3 is on the y-axis.
the ordered pair is (7, 3).
so the point is R.

Question 16.
S (7, 8)

Answer:
The point is S.

Explanation:
In the above-given question,
given that,
point 7 is on the x-axis.
point 8 is on the y-axis.
the ordered pair is (7, 8).
so the point is S.

Question 17.
T (8, 1)

Answer:
The point is T.

Explanation:
In the above-given question,
given that,
point 8 is on the x-axis.
point 1 is on the y-axis.
the ordered pair is (8, 1).
so the point is T.

Question 18.
U (3, 3)

Answer:
The point is E.

Explanation:
In the above-given question,
given that,
point 1 is on the x-axis.
point 3 is on the y-axis.
the ordered pair is (1, 3).
so the point is E.
Envision-Math-Common-Core-5th-Grade-Answers-Key-Topic-14-Graph Points on the Coordinate Plane-3

Problem Solving

Question 19.
Reasoning How is graphing (0, 2) different from graphing (2,0)?

Answer:
(0, 2) is on the y-axis, and (2, 0) is on the x-axis.

Explanation:
In the above-given question,
given that,
the ordered pairs are (0, 2) and (2, 0).
(0, 2) is 2 units up from the origin.
(2, 0) is 2 units right from the origin.

Question 20.
Number Sense Shane took a test that had a total of 21 items. He got about of the items correct. About how many items did he get correct?

Answer:

Question 21.
Higher Order Thinking Point C is located at (10, 3) and Point D is located at (4,3). What is the horizontal distance between the two points? Explain.

Answer:
The distance between the points is (6, 0).

Explanation:
In the above-given question,
given that,
point C is located at (10, 3).
point D is located at (4, 3).
10 – 4 = 6.
3 – 3 = 0.
so the distance between the points is (6, 0).

Question 22.
Laurel buys 3 balls of yarn. Each ball of yarn costs $4.75. She also buys 2 pairs of knitting needles. Each pair costs $5.75. She pays for her purchase with two 20-dollar bills. What is her change?

Answer:
The change is $14.25.

Explanation:
In the above-given question,
given that,
Laurel buys 3 balls of yarn.
Each ball of yarn costs $4.75.
She also buys 2 pairs of knitting needles.
Each pair costs $5.75.
3 x $4.75 = $14.25.
$5.75 x 2 = $11.5.
$14.25 + $11.5 = $25.75.
20 x 2 = 40 dollar bills.
40 – 25.75 = $14.25.

Question 23.
Graph the points below on the grid at the right.
A (2, 4) B (1, 2) C (2, 0) D (3, 0) E (4, 2) F (3, 4)
Envision Math Common Core 5th Grade Answers Topic 14 Graph Points on the Coordinate Plane 11.8

Answer:
The points are (2, 4), (1, 2), (2, 0), (3, 0), (4, 2), and (3, 4).

Explanation:
In the above-given question,
given that,
The points are (2, 4), (1, 2), (2, 0), (3, 0), (4, 2), and (3, 4).
point 2 is on the x-axis.
point 4 is on the y-axis.
point 1 is on the x-axis.
point 2 is on the y-axis.
point 2 is on the x-axis.
point 0 is on the y-axis.
point 3 is on the x-axis.
point 4 is on the x-axis.
point 2 is on the y-axis.
point 3 is on the x-axis.
point 4 is on the y-axis.
Envision-Math-Common-Core-5th-Grade-Answers-Key-Topic-14-Graph Points on the Coordinate Plane-4

Question 24.
Alejandro wants to connect the points to form a shape. What would be the most appropriate tool for him to use? Use the tool to connect the points.

Answer:
The shape formed is the hexagon.

Explanation:
In the above-given question,
given that,
Alejandro wants to connect the points to form a shape.
points are (2, 4), (1, 2), (2, 0), (3, 0), (4, 2), and (3, 4).
point 2 is on the x-axis.
point 4 is on the y-axis.
point 1 is on the x-axis.
point 2 is on the y-axis.
point 2 is on the x-axis.
point 0 is on the y-axis.
point 3 is on the x-axis.
point 4 is on the x-axis.
point 2 is on the y-axis.
point 3 is on the x-axis.
point 4 is on the y-axis.
so the shape formed is the hexagon.

Assessment Practice

Question 25.
Talia draws a map of her neighborhood on a coordinate grid. Her map shows the school at S (1,6), her house at H (4,3), and the library at L (7,2). Graph and label each location on the grid at the right.
Envision Math Common Core 5th Grade Answers Topic 14 Graph Points on the Coordinate Plane 12.1

Answer:
Points are (1, 6), (4, 3), and (7, 2).

Explanation:
In the above-given question,
given that,
Talia draws a map of her neighborhood on a coordinate grid.
Her map shows the school at S (1,6), her house at H (4,3), and the library at L (7,2).
point 1 is on the x-axis.
point 6 is on the y-axis.
point 4 is on the x-axis.
point 3 is on the y-axis.
point 7 is on the x-axis.
point 2 is on the y-axis.
Envision-Math-Common-Core-5th-Grade-Answers-Key-Topic-14-Graph Points on the Coordinate Plane-5

Lesson 14.3 Solve Problems Using Ordered Pairs

Activity

Solve&Share
The table below uses number patterns to describe changes in the width and length of a rectangle. Let x be the width and y be the length. Then plot each of the four ordered pairs in the table on the coordinate grid. What do you think the length is if the width is 5?
Envision Math Common Core 5th Grade Answers Topic 14 Graph Points on the Coordinate Plane 12.2

You can make a graph to help solve the problem. Show your work!
Envision Math Common Core 5th Grade Answers Topic 14 Graph Points on the Coordinate Plane 12.3

Look Back! Look for Relationships What pattern do the points form on your graph?

Visual Learning Bridge

Essential Question How Can You Use Ordered Pairs to Solve Problems?

A.
Both Ann and Bill earn the amount shown each week. Ann starts with no money, but Bill starts with $5. How much will Bill have when Ann has $30? Represent this situation using a table and a graph.

You know that when Ann has $0, Bill has $5.
Envision Math Common Core 5th Grade Answers Topic 14 Graph Points on the Coordinate Plane 12.4

B.
Make a table showing how much money Ann and Bill have after each week.
Envision Math Common Core 5th Grade Answers Topic 14 Graph Points on the Coordinate Plane 12.5
Let x = Ann’s earnings and y = Bill’s earnings.

C.
Plot the ordered pairs from the table. Draw a line to show the pattern. Extend your line to the point where the x-coordinate is 30. The corresponding y-coordinate is 35.
Envision Math Common Core 5th Grade Answers Topic 14 Graph Points on the Coordinate Plane 12.6
So, Bill has $35 when Ann has $30.

Convince Me! Look for Relationships What is the relationship between Bill’s earnings and Ann’s earnings?

Guided Practice

Do You Understand?

Question 1.
In the example on page 574, find another point on the line. What does this point represent?

Answer:

Question 2.
Algebra In the example on page 574, write an equation to show the relationship between Ann’s earnings and Bill’s earnings. Remember to let x = Ann’s earnings and y = Bill’s earnings.

Answer:
Let x = Ann’s earnings.
y = Bill’s earnings.

Explanation:
In the above-given question,
given that,
x + 5.
Ann has $30 and $35.
$30 + 5 = $35.

Do You Know How?

Write the missing coordinates and tell what the point represents.

Question 3.
Envision Math Common Core 5th Grade Answers Topic 14 Graph Points on the Coordinate Plane 12.8

Answer:
The missing coordinates are (5, 200).

Explanation:
In the above-given question,
given that,
the points are (20, 800), (15, 600), (10, 400).
20 – 5 = 15.
15 – 5 = 10.
10 – 5 = 5.
800- 200 = 600.
600 – 200 = 400.
400 – 200 = 200.
so the missing coordinates are (5, 200).

Independent Practice

In 4 and 5, find the missing coordinates and tell what the point represents.

Question 4.
Envision Math Common Core 5th Grade Answers Topic 14 Graph Points on the Coordinate Plane 12.9

Answer:
The missing coordinates are (900, 800).

Explanation:
In the above-given question,
given that,
the ordered pairs are (1500, 1200), (1200, 1000), (600, 600), and (300, 400).
on x-axis subtract 300.
1500 – 300 = 1200.
1200 – 300 = 900.
900 – 300 = 600.
600 – 300 = 300.
on y-axis subtract 200.
1200 – 200 = 1000.
1000 – 200 = 800.
800 – 200 = 600.
600 – 200 = 400.
so the missing coordinates are (900, 800).

Question 5.
Envision Math Common Core 5th Grade Answers Topic 14 Graph Points on the Coordinate Plane 12.10

Answer:
The missing coordinates are (200, 150).

Explanation:
In the above-given question,
given that,
the ordered pairs are (600, 550), (500, 450), (400, 350), (300, 250), and (100, 50).
subtract 100 on x-axis.
subtract 100 on y-axis.
600 – 100 = 500.
500 – 100 = 400.
400 – 100 = 300.
300 – 100 = 200.
200 – 100 = 100.
so the missing coordinates are (200, 150).

Question 6.
For Exercise 5, find two other points on the line. Then graph and label them. Describe the relationship between deer sightings and elk sightings.

Answer:

Problem Solving

In 7 and 8, use the table at the right.

Question 7.
Graph the points in the table on the grid at the right. Then draw a line through the points.
Envision Math Common Core 5th Grade Answers Topic 14 Graph Points on the Coordinate Plane 25.1

Answer:
Time(h) is on the x-axis.
pages read is on the y-axis.

Explanation:
In the above-given question,
given that,
in 1 hour number of pages read is 20.
in 2 hours number of pages read is 40.
in 3 hours number of pages read is 60.
in 4 hours number of pages read is 80.
in 5 hours number of pages read is 100.

Question 8.
Look for Relationships If the pattern continues, how many pages will have been read after 6 hours? Extend your graph to solve.
Envision Math Common Core 5th Grade Answers Topic 14 Graph Points on the Coordinate Plane 25.2

Answer:
The number of pages that will have been read after 6 hours = 120.

Explanation:
In the above-given question,
given that,
in 1 hour number of pages read is 20.
in 2 hours number of pages read is 40.
in 3 hours number of pages read is 60.
in 4 hours number of pages read is 80.
in 5 hours number of pages read is 100.
in 6 hours number of pages read is 120.

Question 9.
Higher Order Thinking Suppose you have a graph of speed that shows a lion can run four times as fast as a squirrel. Name an ordered pair that shows this relationship. What does this ordered pair represent?

Answer:
The ordered pair represents (1, 4).

Explanation:
In the above-given question,
given that,
the lion can run four times as fast as a squirrel.
x + 4y.
x represents the speed of a squirrel.
y represents the speed of a lion.
so a lion can run four times as fast as a squirrel.

Question 10.
Number Sense Candace drives a total of 48 miles each day to get to work and back home. She works 5 days a week. Her car gets 21 miles per gallon of gas. About how many gallons of gas does she need to drive to work and back home each week?!

Answer:
The number of gallons of gas does she need to drive to work = 48 gallons.

Explanation:
In the above-given question,
given that,
Candace drives a total of 48 miles each day to get to work and back home.
She works 5 days a week.
Her car gets 21 miles per gallon of gas.
48 x 5 = 240.
240 / 5 = 48.
so the number of gallons of gas does she need to drive to work.

Assessment Practice

Question 11.
What does the point (15, 4) represent on the graph at the right?
Envision Math Common Core 5th Grade Answers Topic 14 Graph Points on the Coordinate Plane 26.2
A. The ant crawled 15 meters in 19 seconds.
B. The ant crawled 15 meters in 4 seconds.
C. The ant crawled 4 meters in 19 seconds.
D. The ant crawled 4 meters in 15 seconds.

Answer:
Option B is correct.

Explanation:
In the above-given question,
given that,
point (15, 4) is in the coordinate plane.
15 is on the x-axis.
4 is on the y-axis.
the ant crawled 15 meters in 4 seconds.
so option B is correct.

Question 12.
What does the point (20, 5) represent on the graph?
A. In 20 seconds, the ant crawled 5 centimeters.
B. In 20 seconds, the ant crawled 5 meters.
C. In 5 seconds, the ant crawled 20 meters.
D. In 5 seconds, the ant crawled 15 meters.

Answer:
Option C is correct.

Explanation:
In the above-given question,
given that,
the point is (20, 5).
20 is on the x-axis.
5 is on the y-axis.
in 5 seconds the ant crawled 20 meters.
so option C is correct.

Lesson 14.4 Reasoning

Activity

Problem Solving

Solve & Share
Six clowns apply for a circus job. The specific job requires the clown to have a clown shoe size less than 15 inches and to be shorter than 5 ft 8 in. tall.
How many clowns meet the size requirements for the job? Complete the graph below to help you decide.

Envision Math Common Core 5th Grade Answers Topic 14 Graph Points on the Coordinate Plane 28.1

Thinking Habits
Be a good thinker! These questions can help you.
• What do the numbers and symbols in the problem mean?
• How are the numbers or quantities related?
• How can I represent a word problem using pictures, numbers, or equations?
Envision Math Common Core 5th Grade Answers Topic 14 Graph Points on the Coordinate Plane 28.2

Look Back! Reasoning How can you use reasoning about the completed graph to find the number of clowns that meet the requirements? Explain.

Visual Learning Bridge

Essential Question How Can You Use Reasoning to Solve Mathematical Problems?

A.
In 1705, a ship sank in the ocean at the point shown. Every year the ocean currents moved the ship 1 mile east and 2 miles north. Where was the ship located after 4 years? Where was the ship located after 10 years? Tell how you decided.
Envision Math Common Core 5th Grade Answers Topic 14 Graph Points on the Coordinate Plane 28.3

What do I need to do to solve the problem?
I need to find the ship’s location after 4 years and after 10 years.

B.
How can I use reasoning to solve this problem?
I can
• use what I know about graphing points.
• graph ordered pairs.
• look for relationships in the coordinates.
• decide if my answer makes sense.

Here’s my thinking..
Envision Math Common Core 5th Grade Answers Topic 14 Graph Points on the Coordinate Plane 29.1

C.
I will use the graph to show the location each year for 4 years. Each point is 1 mile east and 2 miles north from the previous point.
After 4 years the ship was at (8, 14).
I see a pattern. The x-coordinate increases by 1, and the y-coordinate increases by 2:
(4, 6), (5, 8), (6, 10), (7, 12), (8, 14)
Envision Math Common Core 5th Grade Answers Topic 14 Graph Points on the Coordinate Plane 29.2
I can continue the pattern for another 6 years:
(9,16), (10, 18), (11, 20), (12, 22), (13, 24), (14, 26)
After 10 years, the ship was at (14, 26).

Convince Me! Make Sense and Persevere How could you decide if your answers make sense?

Guided Practice

Tanya marked a grid in her garden. She planted a rose bush at (3, 1). She moved 2 feet east and 1 foot north and planted the second rose bush. She continued planting rose bushes so that each bush is 2 feet east and 1 foot north of the previous bush.

Envision Math Common Core 5th Grade Answers Topic 14 Graph Points on the Coordinate Plane 30.1

Question 1.
How can a coordinate grid help you reason about the problem?

Answer:
The point is (3, 1), (5, 2), and (7, 3).

Explanation:
In the above-given question,
given that,
the point is (3, 1).
She moved 2 feet east and 1 foot north and planted the second rose bush.
3 + 2 = 5.
5 + 2 = 7.
1 + 1 = 2.
2 + 1 = 3.
so the points are (3, 1), (5, 2), and (7, 3).
Envision-Math-Common-Core-5th-Grade-Answers-Key-Topic-14-Graph Points on the Coordinate Plane-6

Question 2.
Draw and label the locations of the first four bushes on the grid. Do Tanya’s 0 2 bushes lie on a straight line? How do you know?

Answer:
The bushes lie in a straight line.

Explanation:
In the above-given question,
given that,
the point is (3, 1).
She moved 2 feet east and 1 foot north and planted the second rose bush.
3 + 2 = 5.
5 + 2 = 7.
1 + 1 = 2.
2 + 1 = 3.
so the points are (3, 1), (5, 2), and (7, 3).

Question 3.
What are the locations of the fifth and ninth rose bushes?

Answer:
The locations of the fifth and ninth rose bushes are in a straight line.

Explanation:
In the above-given question,
given that,
the point is (3, 1).
She moved 2 feet east and 1 foot north and planted the second rose bush.
3 + 2 = 5.
5 + 2 = 7.
7 + 2 = 9.
1 + 1 = 2.
2 + 1 = 3.
3 + 1 = 4.
so the points are (3, 1), (5, 2), and (7, 3), (9, 4).

Independent Practice

Reasoning
A marching band uses a grid to determine the members’ positions. Juan starts at (2, 2). Every 15 seconds, he moves 4 yards east and 3 yards north.

Envision Math Common Core 5th Grade Answers Topic 14 Graph Points on the Coordinate Plane 30.11

Question 4.
How can you model this problem?

Answer:
After 15 seconds Juan moves (6, 5).

Explanation:
In the above-given question,
given that,
A marching band uses a grid to determine the members’ positions.
Juan starts at (2, 2).
Every 15 seconds, he moves 4 yards east and 3 yards north.
2 + 4 = 6.
2 + 3 = 5.
so after 15 seconds Juan moves (6, 5).

Question 5.
Draw and label the locations of Juan’s first four positions. Do the points form a pattern? How can you tell?

Answer:
Juan’s first four positions are (6, 5), (10, 8), (14, 11), and (18, 14).

Explanation:
In the above-given question,
given that,
A marching band uses a grid to determine the members’ positions.
Juan starts at (2, 2).
Every 15 seconds, he moves 4 yards east and 3 yards north.
2 + 4 = 6, 6 + 4 = 10, 10 + 4 = 14, and 14 + 4 = 18.
2 + 3 = 5, 5 + 3 = 8, 8 + 3 = 11, and 11 + 3 = 14.
so Juan’s first four positions are (6, 5), (10, 8), (14, 11), and (18, 14).

Question 6.
What will Juan’s location be after 60 seconds? 90 seconds? How does the coordinate grid help you reason about the locations?

Answer:
Juan,s location be after 60 and 90 seconds are (14, 11), and (18, 14).

Explanation:
In the above-given question,
given that,
A marching band uses a grid to determine the members’ positions.
Juan starts at (2, 2).
Every 15 seconds, he moves 4 yards east and 3 yards north.
2 + 4 = 6, 6 + 4 = 10, 10 + 4 = 14, and 14 + 4 = 18.
2 + 3 = 5, 5 + 3 = 8, 8 + 3 = 11, and 11 + 3 = 14.
so Juan,s location be after 60 and 90 seconds are (14, 11), and (18, 14).

Problem Solving

Performance Task

Rozo Robot
A toy company is testing Rozo Robot. Rozo is 18 inches tall and weighs 2 pounds. The employees of the company marked a grid on the floor and set Rozo at (2,5). They programmed Rozo to walk 3 yards east and 4 yards north each minute. What will Rozo’s location be after 7 minutes?

Envision Math Common Core 5th Grade Answers Topic 14 Graph Points on the Coordinate Plane 31.2

Question 7.
Make Sense and Persevere Do you need all of the information given in the problem to solve the problem? Describe any information that is not needed.

Answer:

Question 8.
Model with Math Label the graph and plot Rozo’s starting position. Then plot and label Rozo’s position at the end of each of the first 4 minutes.
Answer:

Question 9.
Use Appropriate Tools What tool would you choose for drawing a line segment between points on a coordinate grid? Explain your thinking.
Answer:

You can use the coordinate grid to reason about relationships between the points
Envision Math Common Core Grade 5 Answers Topic 14 Graph Points on the Coordinate Plane 35.1

Question 10.
Look for Relationships Describe the relationships between the coordinates of the points that represent Rozo’s locations.
Answer:

Question 11.
Reasoning What will Rozo’s location be after 7 minutes? Explain how you determined your answer.
Answer:

Topic 14 Fluency Practice

Activity

Point & Tally

Find a partner. Get paper and a pencil. Each partner chooses light blue or dark blue.
At the same time, Partner 1 and Partner 2 each point to one of their black numbers. Both partners find the product of the two numbers.
The partner who chose the color where the product appears gets a tally mark. Work until one partner has seven tally marks.
Envision Math Common Core Grade 5 Answers Topic 14 Graph Points on the Coordinate Plane 352

Envision Math Common Core Grade 5 Answers Topic 14 Graph Points on the Coordinate Plane 353

Topic 14 Vocabulary Review

Glossary

Understand Vocabulary

Choose the best term from the Word List. Write it on the blank.

Word List
• coordinate grid
• ordered pair
• origin
• x-axis
• x-coordinate
• y-axis
• y-coordinate

Question 1.
The point where the axes of a coordinate grid intersect is the _____

Answer:
The point where the axes of a coordinate grid intersect is the origin.

Explanation:
In the above-given question,
given that,
the point where the axes of a coordinate grid intersect is the origin.
for example:
(x, y) = (3, 2).

Question 2.
A(n) ____ names an exact location on a coordinate grid.

Answer:
A(n) coordinate grid names an exact location on a coordinate grid.

Explanation:
In the above-given question,
given that,
A(n) coordinate grid names an exact location on a coordinate grid.
for example:
(2, 3) is on the coordinate plane.
2 is on the x-axis.
3 is on the y-axis.

Question 3.
The first number of an ordered pair describes the distance from the origin along the ____

Answer:
The first number of an ordered pair describes the distance from the origin along with the ordered pair.

Explanation:
In the above-given question,
given that,
the first number of an ordered pair describes the distance from the origin along with the ordered pair.
for example:
(4, 5) 4 units on the x-axis.
5 units on the y-axis.

Question 4.
The second number of an ordered pair is the ______

Answer:
The second number of an ordered pair is the y-coordinate.

Explanation:
In the above-given question,
given that,
the second number of an ordered pair is the y-coordinate.
for example:
In an ordered pair, the first number is always the x-coordinate.
the second number is the y-coordinate.

Question 5.
A ____ is formed by two number lines that intersect at a right angle.

Answer:
A coordinate plane is formed by two number lines that intersect at a right angle.

Explanation:
In the above-given question,
given that,
A coordinate plane is formed by two number lines that intersect at a right angle.
for example:
x-axis = horizontal number line.
y-axis = vertical number line.
origin = where the number lines intersect.
the coordinates of the origin are (0, 0).

Draw a line from each lettered point in Column A to the ordered pair it represents.

Envision Math Common Core Grade 5 Answers Topic 14 Graph Points on the Coordinate Plane 341

Envision Math Common Core Grade 5 Answers Topic 14 Graph Points on the Coordinate Plane 342

Envision-Math-Common-Core-5th-Grade-Answers-Key-Topic-14-Graph Points on the Coordinate Plane-7

Use Vocabulary in Writing

Question 12.
Why is the order of the coordinates important in an ordered pair? Use terms from the Word List in your explanation.

Answer:
The order of the coordinates is important because it forms a shape, a straight line.

Explanation:
In the above-given question,
given that,
points are A(5, 2), B(1, 7), C(2, 3), D(0, 7), E(7, 1), and F(0, 6).
the points formed a shape.
so the order of the coordinates is important in an ordered pair.

Topic 14 Reteaching

Set A
pages 565-568

What ordered pair names Point A?
Start at the origin. The x-coordinate is the horizontal distance along the x-axis. The y-coordinate is the vertical distance along the y-axis.
Envision Math Common Core Grade 5 Answers Topic 14 Graph Points on the Coordinate Plane 401
Point A is at (7, 3).

Remember to first find the x-coordinate. Then find the y-coordinate. Write the coordinates in (x, y) order.

Use the grid to answer the questions.

Question 1.
Which point is located at (9, 5)?

Answer:
Point is located at (9, 5) is R.

Explanation:
In the above-given question,
given that,
point is (9, 5).
9 is on the x-axis.
5 is on the y-axis.
so the point is located at (9, 5) is R.

Question 2.
Which point is located at (2, 3)?

Answer:
Point is located at (2, 3) is K.

Explanation:
In the above-given question,
given that,
the point is (2, 3).
2 is on the x-axis.
3 is on the y-axis.
so the point is located at (2, 3) is K.

Question 3.
What ordered pair names Point T ?

Answer:
Point is located at (3, 4) is T.

Explanation:
In the above-given question,
given that,
the point is (3, 4).
3 is on the x-axis.
4 is on the y-axis.
so the point is located at (3, 4) is T.

Question 4.
What is the ordered pair for the origin?

Answer:
Point is located at (0, 7).

Explanation:
In the above-given question,
given that,
the point is (0,7).
0 is on the x-axis.
7 is on the y-axis.
so the point is located at (0, 7) is the origin.

Set B
pages 569-572, 573-576
In the table, the x-coordinate is in the left column and the y-coordinate is in the right column. Use the table to plot the ordered pairs. Then draw a line to connect the points.
Envision Math Common Core Grade 5 Answers Topic 14 Graph Points on the Coordinate Plane 402

Remember that you can use a tool, such as a ruler, to draw a line to connect the points on the graph.

Question 1.
Use the table below to plot the ordered pairs. Then complete the graph by connecting the points.
Envision Math Common Core Grade 5 Answers Topic 14 Graph Points on the Coordinate Plane 413

Answer:
The points are (2, 1), (4, 2), (6, 3), and (8, 4).

Explanation:
In the above-given question,
given that,
the points on the x-axis are 2, 4, 6, and 8.
the points on the y-axis are 1, 2, 3, and 4.
so the ordered pairs are (2, 1), (4, 2), (6, 3), and (8, 4).
Envision-Math-Common-Core-5th-Grade-Answers-Key-Topic-14-Graph Points on the Coordinate Plane-8

Question 2.
Write two ordered pairs with x-coordinates greater than 10 that are on the line.

Answer:
The two ordered pairs are (10, 5), and (12, 6).

Explanation:
In the above-given question,
given that,
the points on the x-axis are 2, 4, 6, and 8.
the points on the y-axis are 1, 2, 3, and 4.
2 + 2 = 4.
4 + 2 = 6.
6 + 2 = 8.
1 + 1 = 2.
2 + 1 = 3.
3 + 1 = 4.
4 + 1 = 5.
so the two ordered pairs are (10, 5), and (12, 6).

Set C
pages 577-580
Think about these questions to help you use reasoning to solve problems.

Thinking Habits
• What do the numbers and symbols in the problem mean?
• How are the numbers or quantities related?
• How can I represent a word problem using pictures, numbers, or equations?

Remember that you can use a graph or a table to reason about and solve word problems.
A company uses the graph to show how many packages each truck driver delivers. How many packages will one truck driver deliver in a 7-hour day?
Envision Math Common Core Grade 5 Answers Topic 14 Graph Points on the Coordinate Plane 456

Question 1.
What information can help you solve the problem?

Answer:
The number of packages each truck driver delivers in hours.

Explanation:
In the above-given question,
given that,
the number of packages each truck driver delivers in hours.
in 1 hour the number of packages is 15.
in 2 hours the number of packages is 30.
in 3 hours the number of packages is 45.
in 4 hours the number of packages is 60.

Question 2.
How can you find the number of packages a driver delivers in 3 hours?

Answer:
The number of packages a driver delivers in 3 hours is 45.

Explanation:
In the above-given question,
given that,
the number of packages each truck driver delivers in hours.
in 1 hour the number of packages is 15.
in 2 hours the number of packages is 30.
in 3 hours the number of packages is 45.
in 4 hours the number of packages is 60.
so the number of packages a driver delivers in 3 hours is 45.

Question 3.
How many packages will one truck driver deliver in a 7-hour day?

Answer:
The number of packages will one truck deliver in a 7-hour day is 105.

Explanation:
In the above-given question,
given that,
the number of packages each truck driver delivers in hours.
in 1 hour the number of packages is 15.
in 2 hours the number of packages is 30.
in 3 hours the number of packages is 45.
in 4 hours the number of packages is 60.
in 5 hours the number of packages is 75.
in 6 hours the number of packages is 90.
in 7 hours the number of packages is 105.

Question 4.
How can you find how many hours it will take for one truck driver to deliver 120 packages?

Answer:
The number of hours is 8.

Explanation:
In the above-given question,
given that,
the number of packages each truck driver delivers in hours.
in 1 hour the number of packages is 15.
in 2 hours the number of packages is 30.
in 3 hours the number of packages is 45.
in 4 hours the number of packages is 60.
in 5 hours the number of packages is 75.
in 6 hours the number of packages is 90.
in 7 hours the number of packages is 105.

Topic 14 Assessment Practice

Use the coordinate grid below to answer 1-4.

Envision Math Common Core Grade 5 Answers Topic 14 Graph Points on the Coordinate Plane 521

Question 1.
Which is the ordered pair for Point Y?
A. (4, 5)
B. (4, 9)
C. (7, 9)
D. (9, 4)

Answer:
Option B is correct.

Explanation:
In the above-given question,
given that,
the point is (4, 9).
4 is on the x-axis.
9 is on the y-axis.
so option B is correct.

Question 2.
Which point is located at (5, 2) on the coordinate grid?
A. M
B. N
C. Q
D. P

Answer:
Option B is correct.

Explanation:
In the above-given question,
given that,
the point is (5, 2).
5 is on the x-axis.
2 is on the y-axis.
so option B is correct.

Question 3.
What is the ordered pair for Point Z?

Answer:
The ordered pair for point Z is (9, 1).

Explanation:
In the above-given question,
given that,
the point is (9, 1).
9 is on the x-axis.
1 is on the y-axis.
so the ordered pair for point Z is (9, 1).

Question 4.
What is the ordered pair for Point P?

Answer:
The ordered pair for point P is (4, 5).

Explanation:
In the above-given question,
given that,
the point is (4, 5).
4 is on the x-axis.
5 is on the y-axis.
so the ordered pair for point P is (4, 5).

Question 5.
Each year, Ginny recorded the height of a tree growing in her front yard. The graph below shows her data.
Envision Math Common Core 5th Grade Answer Key Topic 14 Graph Points on the Coordinate Plane 62.2
What does the point (1, ?) represent?

Answer:
The point (1, 5) represents the height in feet.

Explanation:
In the above-given question,
given that,
Ginny recorded the height of a tree growing in her front yard.
the points on the x-axis are 0, 1, 2, 3, and 4.
the points on the y-axis are 4, 5, 6, 7, and 8.
add the 1 on the x and y-axis.
so the point (1, 5) represents the height in feet.

Question 6.
Explain how to graph the point (6, 4) on a coordinate plane.

Answer:
Point 6 is on the x-axis and 4 is on the y-axis.

Explanation:
In the above-given question,
given that,
the point is (6, 4).
point 6 is on the x-axis and 4 is on the y-axis.

Question 7.
Varsha draws a map of her neighborhood on a coordinate plane. Her map shows the park at P (3, 1), her house at H (5, 6), and the soccer field at S (2,4). Graph and label each location below.
Envision Math Common Core 5th Grade Answer Key Topic 14 Graph Points on the Coordinate Plane 70.1

Answer:
The point is P(3, 1) is the park.
the point H(5, 6) is her house.
the point S(2, 4) is the soccer field.

Explanation:
In the above-given question,
given that,
Varsha draws a map of her neighborhood on a coordinate plane.
Her map shows the park at P (3, 1), her house at H (5, 6), and the soccer field at S (2,4).
the point is p(3, 1) is the park.
the point H(5, 6) is her house.
the point S(2, 4) is the soccer field.
Envision-Math-Common-Core-5th-Grade-Answers-Key-Topic-14-Graph Points on the Coordinate Plane-9

Question 8.
Yesterday Billy earned $30 trimming hedges for Mrs. Gant. Today he will earn $10 an hour for weeding her garden. If he weeds her garden for 8 hours, how much in all will he earn working for Mrs. Gant?
Envision Math Common Core 5th Grade Answer Key Topic 14 Graph Points on the Coordinate Plane 70.2
A. $40
B. $ 80
C. $ 110
D. $ 120

Answer:
Option B is correct.

Explanation:
In the above-given question,
given that,
Billy earned $30 trimming hedges for Mrs. Gant.
Today he will earn $10 an hour for weeding her garden.
If he weeds her garden for 8 hours,
8 x $10 = $80.
so option B is correct.

Question 9.
How is graphing (0, 12) different from graphing (12, 0)?

Answer:
(0, 12) is different from (12, 0).

Explanation:
In the above-given question,
given that,
the ordered pairs are (0, 12) and (12, 0).
(0, 12) is on the y-coordinate.
(12, 0) is on the x-coordinate.
so (0, 12) is different from (12, 0).

Question 10.
What ordered pair represents the point where the x-axis and y-axis intersect? What is the name of this point?

Answer:
The ordered pair (0, 0) represents the point where the x-axis and y-axis intersect.

Explanation:
In the above-given question,
given that,
the ordered pair (0, 0) represents the origin.
the x-axis and y-axis intersect are called the origin.
so the ordered pair (0, 0) represents the point where the x-axis and y-axis intersect.

Question 11.
Three vertices of a rectangle are located at (1, 4), (1, 2), and (5, 2).
A. Graph and label each of the three vertices below.
Envision Math Common Core 5th Grade Answer Key Topic 14 Graph Points on the Coordinate Plane 75.6
B. What are the coordinates of the fourth vertex of the rectangle?

Answer:
The coordinates of the fourth vertex of the rectangle are (5, 4).

Explanation:
In the above-given question,
given that,
three vertices of a rectangle are located at (1, 4), (1, 2), and (5, 2).
the other side is (5, 4).
for the rectangle, opposite sides are equal in length.
so the coordinates of the fourth vertex of the rectangle are (5, 4).
Envision-Math-Common-Core-5th-Grade-Answers-Key-Topic-14-Graph Points on the Coordinate Plane-10

Topic 14 Performance Task

Digging for Dinosaur Bones
Omar’s mother is a paleontologist. She digs up and studies dinosaur bones. Omar is helping at the dig site.

Question 1.
The Dinosaur Bone Dig 1 grid shows the location of the tent and the triceratops skull Omar’s mother found.
Part A
What ordered pair names the location of the triceratops skull? Explain how you know.
Envision Math Common Core 5th Grade Answer Key Topic 14 Graph Points on the Coordinate Plane 80.1
Part B
omar found a leg bone at (4, 12). Graph this point on the coordinate grid and label it L. Explain how you located the point using the terms origin, x-coordinate, x-axis, y-coordinate, and y-axis.
Part C
Next, Omar dug 3 meters east and 1 meter south from the leg bone. Graph a point where Omar dug and label it A. What ordered pair names the point?

Answer:
Part A: (6, 2).
Part B: (4, 12).
Part C: (7, 13).

Explanation:
In the above-given question,
given that,
Omar found a leg bone at (4, 12).
Omar dug 3 meters east and 1 meter south from the leg bone.
4 + 3 = 7.
12 + 1 = 13.
so part C is (7, 13).

Question 2.
Omar’s mother started at the triceratops skull. She kept moving east 1 meter and north 2 meters to dig for more dinosaur bones. Complete the table and the graph to find how far north she was when she was 11 meters east of the tent.

Answer:
When she was 11 meters east of the tent, the far = 6.

Explanation:
In the above-given question,
given that,
Omar’s mother started at the triceratops skull.
She kept moving east 1 meter and north 2 meters to dig for more dinosaur bones.
(4, 12) is the ordered pair given?
4 + 1 = 5.
12 + 2 = 14.
(5, 14) is the ordered pair.
11 – 5 = 6.
so when was 11 meters east of the tent, the far is 6 meters.
Part A
Complete the table of ordered pairs.
Envision Math Common Core 5th Grade Answer Key Topic 14 Graph Points on the Coordinate Plane 85.1

Part B
Graph the points from the table in Part A on the coordinate grid in the Dinosaur Bone Dig 2 grid. Draw a line through the points. Extend the line past 11 meters east.
Part C
How far north was Omar’s mother when she was 11 meters east of the tent? Explain how to use the graph to solve and why your answer makes sense.

Answer:
The ordered pairs are (6, 2), (7, 3), ( 8, 4), and (9, 5).

Explanation:
In the above-given question,
given that,
East of the tent in meters is given.
they are 6, 7, 8, and 9 meters are given.
the first ordered pair is (6, 2) is given.
so the ordered pairs are (7, 3), (8, 4), and (9, 5).
when she was 11 meters east of the tent is (11, 7).
Envision-Math-Common-Core-5th-Grade-Answers-Key-Topic-14-Graph Points on the Coordinate Plane-11

enVision Math Common Core Grade 5 Answer Key Topic 11 Understand Volume Concepts

Practice with the help of enVision Math Common Core Grade 5 Answer Key Topic 11 Understand Volume Concepts regularly and improve your accuracy in solving questions.

enVision Math Common Core 5th Grade Answers Key Topic 11 Understand Volume Concepts

Envision Math Common Core Grade 5 Answer Key Topic 11 Understand Volume Concepts 1
Essential Questions: What is the meaning of the volume of a solid? How can the volume of a rectangular prism be found?

enVision STEM Project: Everyday Energy
Do Research Use the Internet and other sources to learn more about these five types of energy: electrical, light, mechanical, sound, and thermal. Make a table of the various types of energy you use every day. Include at least one example of how you use each type of energy.

Journal: Write a Report Include what you found. Also in your report:
• Draw a diagram of your classroom and label where and how 3 types of energy are used.
• Estimate how far your desk is from a light energy source and add this dimension to your sketch.
• Use your diagram to make up and solve problems involving measurements such as the volume of your classroom.

Review What You Know

A-Z Vocabulary

Choose the best term from the box. Write it on the blank.

• compensation
• partial products
• rectangle
• unit fraction

Question 1.
Adjusting a number to make a computation easier and balancing the adjustment by changing another number is called ____
Answer:

Adjusting a number to make a computation easier and balancing the adjustment by changing another number is called compensation

Question 2.
A fraction with a numerator of 1 is called a ____
Answer:

A fraction with a numerator of 1 is called a  unit fraction

Question 3.
A quadrilateral with 2 pairs of parallel sides that are the same length and 4 right angles is a ____
Answer:

A quadrilateral with 2 pairs of parallel sides that are the same length and 4 right angles is a rectangle

Area
Find the area of each figure.

Question 4.
Envision Math Common Core Grade 5 Answer Key Topic 11 Understand Volume Concepts 3.1
Answer:

From the figure,

Length = 10h

breadth =6h

Area of rectangle = Length x Breadth = 10h x 6h = 60h^2

 

Question 5.
Envision Math Common Core Grade 5 Answer Key Topic 11 Understand Volume Concepts 3.2
Answer:

From the figure,

Length = 12cm

breadth = 8cm

Area of rectangle = Length x Breadth = 12cm x 8cm = 96cm^2

Operations

Find each product or quotient.

Question 6.
16 × 6
Answer:

96

Question 7.
3 × 42
Answer:

126

Question 8.
216 ÷ 3
Answer:

Question 9.
128 ÷ 4
Answer:

Question 10.
(5 × 6) × 3
Answer:

(5 × 6) × 3

(30) x 3

90

Question 11.
(6 × 6) × 6
Answer:

(6 × 6) × 6

(36) x 6

=196

Question 12.
Joanie has two 12-inch-long wood pieces and two 16-inch-long wood pieces. What is the combined length of the wood pieces?
A. 28 inches
B. 32 inches
C. 56 inches
D. 192 inches
Answer:

Given,

Number of 12 inch wood pieces = 2 = 12 x 2 = 24

Number of 16 inch wood pieces = 2 = 16 x 2 = 32

combined length of the wood pieces = 24 + 32 = 56

C. 56 inches is correct

 

Finding Area

Question 13.
Niko used square tiles to make a rectangle with 2 rows and 7 tiles in each row. Explain how you can find the area of the rectangle.
Answer:

Pick a Project

PROJECT 11A
How big are skyscrapers?
Project: Build a Skyscraper with Unit Cubes
Envision Math Common Core Grade 5 Answer Key Topic 11 Understand Volume Concepts 4.1

PROJECT 11B
Why do cats climb into boxes?
Project: Design a Cat Tree
Envision Math Common Core Grade 5 Answer Key Topic 11 Understand Volume Concepts 4.2

PROJECT 11C
Why are trucks useful for transporting packages?
Project: Model a Truck’s Capacity
Envision Math Common Core Grade 5 Answer Key Topic 11 Understand Volume Concepts 4.3

3-ACT MATH PREVIEW

Math Modeling

Video

Fill ‘er Up

Envision Math Common Core Grade 5 Answer Key Topic 11 Understand Volume Concepts 4.4
Before watching the video, think:
Ice is frozen below 32°F (or 0°C), but most picnics and cookouts happen when it’s warm out. The insulated walls of a cooler help keep ice from melting, which keeps my juice nice and cold!

Lesson 11.1 Model Volume

Activity

Solve & Share
Gina is building a rectangular prism out of sugar cubes for her art class project. She started by drawing a diagram of the rectangular prism that is 4 cubes high, 4 cubes long and 2 cubes wide. How many cubes does she use to make the prism? Solve this problem any way you choose.

Envision Math Common Core Grade 5 Answer Key Topic 11 Understand Volume Concepts 4.6

Use Appropriate Tools You can use cubes to build a rectangular prism and then draw a picture to show the different faces. Show your work!

Look Back! Gina decided to change her art project and build a rectangular prism that is 3 cubes long, 4 cubes wide, and 2 cubes high. Use the picture to determine the number of cubes she used.
Envision Math Common Core Grade 5 Answer Key Topic 11 Understand Volume Concepts 4.8

Visual Learning Bridge

Essential Question How Can You Measure Space Inside a Solid Figure?

A.
Volume is the number of cubic units needed to pack a solid figure without gaps or overlaps. A cubic unit is the volume of a cube measuring 1 unit on each edge. What is the volume of this rectangular prism?
Envision Math Common Core Grade 5 Answer Key Topic 11 Understand Volume Concepts 4.9

Each cube of a solid figure is 1 cubic unit.
Envision Math Common Core Grade 5 Answer Key Topic 11 Understand Volume Concepts 4.11

B.
Use unit cubes to make a model.
Envision Math Common Core Grade 5 Answer Key Topic 11 Understand Volume Concepts 5.1
Count the number of cubes.
There are 15 unit cubes in the bottom layer. The volume of the bottom layer is 15 cubic units.

C.
There are two layers.
Envision Math Common Core Grade 5 Answer Key Topic 11 Understand Volume Concepts 5.2
Multiply the volume of the bottom layer by 2.
The volume of the prism is 2 × 15 or 30 cubic units.

Convince Me! Reasoning in the picture below, how many unit cubes does it take to make the rectangular prism below without gaps or overlaps? How many 2-cube towers does it take to make the rectangular prism?
Envision Math Common Core Grade 5 Answer Key Topic 11 Understand Volume Concepts 5.3
Envision Math Common Core Grade 5 Answer Key Topic 11 Understand Volume Concepts 5.4

Guided Practice

Do You Understand?

Question 1.
Make a model of a rectangular prism with a bottom layer that is 3 cubes long by 3 cubes wide. Make a top layer that is the same as the bottom layer. Then draw a picture of your model. What is the volume?
Answer:

Each cube of a solid figure is 1 cubic unit.

There are 9 unit cubes in the bottom layer. The volume of the bottom layer is 9 cubic units.

Multiply the volume of the bottom layer by 2.

The volume of the prism is 2 × 9 or 18 cubic units.

Question 2.
A-Z Vocabulary What is the difference between a unit cube and a cubic unit?
Answer:

A unit cube, more formally a cube of side 1, is a cube whose sides are 1 unit long. The volume of a 3-dimensional unit cube is 1 cubic unit, and its total surface area is 6 square units.

Do You Know How?

In 3 and 4, use unit cubes to make a model of each rectangular prism. Find the volume.

Question 3.
Envision Math Common Core Grade 5 Answer Key Topic 11 Understand Volume Concepts 5.40
Answer:

Each cube of a solid figure is 1 cubic unit.

There are 4 unit cubes in the bottom layer. The volume of the bottom layer is 4 cubic units.

Multiply the volume of the bottom layer by 2.

The volume of the prism is 2 × 4 or 8 cubic units.

Question 4.
Envision Math Common Core Grade 5 Answer Key Topic 11 Understand Volume Concepts 5.5
Answer:

Each cube of a solid figure is 1 cubic unit.

There are 12 unit cubes in the bottom layer. The volume of the bottom layer is 12 cubic units.

The volume of the prism is 12 × 1 or 12 cubic units.

Independent Practice

In 5-13, find the volume of each solid. Use unit cubes to help.

Question 5.
Envision Math Common Core Grade 5 Answer Key Topic 11 Understand Volume Concepts 6.1
Answer:

Each cube of a solid figure is 1 cubic unit.

There are 9 unit cubes in the bottom layer. The volume of the bottom layer is 9 cubic units.

Multiply the volume of the bottom layer by 3.

The volume of the prism is 9 × 3 or 27 cubic units.

Question 6.
Envision Math Common Core Grade 5 Answer Key Topic 11 Understand Volume Concepts 6.2
Answer:

Each cube of a solid figure is 1 cubic unit.

There are 8 unit cubes in the bottom layer. The volume of the bottom layer is 8 cubic units.

Multiply the volume of the bottom layer by 3.

The volume of the prism is 8 × 3 or 24 cubic units.

Question 7.
Envision Math Common Core Grade 5 Answer Key Topic 11 Understand Volume Concepts 6.3
Answer:

Each cube of a solid figure is 1 cubic unit.

There are 20 unit cubes in the bottom layer. The volume of the bottom layer is 20 cubic units.

Multiply the volume of the bottom layer by 2.

The volume of the prism is 20 × 2 or 40 cubic units.

Question 8.
Envision Math Common Core Grade 5 Answer Key Topic 11 Understand Volume Concepts 6.4
Answer:

Each cube of a solid figure is 1 cubic unit.

There are 8 unit cubes in the bottom layer. The volume of the bottom layer is 8 cubic units.

Multiply the volume of the bottom layer by 2.

The volume of the prism is 8 × 2 or 16 cubic units.

Question 9.
Envision Math Common Core Grade 5 Answer Key Topic 11 Understand Volume Concepts 6.5
Answer:

Each cube of a solid figure is 1 cubic unit.

There are 2 unit cubes in the bottom layer. The volume of the bottom layer is 2 cubic units.

Multiply the volume of the bottom layer by 5.

The volume of the prism is 2 × 5 or 10 cubic units.

Question 10.
Envision Math Common Core Grade 5 Answer Key Topic 11 Understand Volume Concepts 6.6
Answer:

Each cube of a solid figure is 1 cubic unit.

There are 6 unit cubes in the bottom layer. The volume of the bottom layer is 6 cubic units.

Multiply the volume of the bottom layer by 3.

The volume of the prism is 6 × 3 or 18 cubic units.

Question 11.
Envision Math Common Core Grade 5 Answer Key Topic 11 Understand Volume Concepts 6.7
Answer:

Each cube of a solid figure is 1 cubic unit.

There are 12 unit cubes in the bottom layer. The volume of the bottom layer is 12 cubic units.

Multiply the volume of the bottom layer by 3.

The volume of the prism is 12 × 3 or 36 cubic units.

Question 12.
Envision Math Common Core Grade 5 Answer Key Topic 11 Understand Volume Concepts 6.8
Answer:

Each cube of a solid figure is 1 cubic unit.

There are 12 unit cubes in the bottom layer. The volume of the bottom layer is 12 cubic units.

Multiply the volume of the bottom layer by 2.

The volume of the prism is 12 × 2 or 24 cubic units.

Question 13.
Envision Math Common Core Grade 5 Answer Key Topic 11 Understand Volume Concepts 6.9
Answer:

Each cube of a solid figure is 1 cubic unit.

There are 10 unit cubes in the bottom layer. The volume of the bottom layer is 10 cubic units.

The remaining cubes are 4 x 3 or 12 and the volume is 12 cubic units

The volume of the prism is 10 + 12 or 22 cubic units.

Problem Solving

In 14-18, use the table.

Compare the volumes of the prisms. Write >, <, or = for each Envision Math Common Core Grade 5 Answer Key Topic 11 Understand Volume Concepts 7

Envision Math Common Core Grade 5 Answer Key Topic 11 Understand Volume Concepts 6.10

Question 14.
Prism A Envision Math Common Core Grade 5 Answer Key Topic 11 Understand Volume Concepts 7 Prism B
Answer:

Prism A > Prism B

Question 15.
Prism B Envision Math Common Core Grade 5 Answer Key Topic 11 Understand Volume Concepts 7 Prism C
Answer:

Prism B = Prism C

Question 16.
Prism C Envision Math Common Core Grade 5 Answer Key Topic 11 Understand Volume Concepts 7 Prism A
Answer:

Prism C < Prism A

Question 17.
If you added another layer of unit cubes on top of Prism A, what would the volume of the new solid be in cubic units?
Answer:

Each cube of a solid figure is 1 cubic unit.

There are 8 unit cubes in the bottom layer. The volume of the bottom layer is 8 cubic units.

Multiply the volume of the bottom layer by 2.

The volume of the prism is 8 × 2 or 16 cubic units.

Question 18.
If you put Prism C on top of Prism A, what would the volume of the new solid be in cubic units?
Answer:

The volume of A is 16 cubic units

The volume of C is 6 cubic units

If you put Prism C on top of Prism A, Then the volume is 16 + 6 or 22 cubic units

Question 19.
Reasoning In an election, 471 people voted. Candidate B received \(\frac{2}{3}\) of the votes. How many votes did Candidate B receive?
Answer:

Number of votes voted = 471

Candidate B received 2/3 of the votes

Number of votes B received = 2/3 x 471 =314

Question 20.
Higher Order Thinking Ms. Kellson’s storage closet is 3 feet long, 3 feet wide, and 7 feet high. Can she fit 67 boxes that each have a volume of 1 cubic foot in her closet? Explain your answer.
Answer:

No.

Each cube of a solid figure is 1 cubic unit.

There are 9 unit cubes in the bottom layer. The volume of the bottom layer is 9 cubic units.

Multiply the volume of the bottom layer by 7.

The volume of the prism is 9 × 7 or 63 cubic units.

She can not fit 67 boxes that each have a volume of 1 cubic foot in her closet.

Assessment Practice

Question 21.
Natalie made the solid figures shown using unit cubes. Which statement about these models is true?
Envision Math Common Core Grade 5 Answer Key Topic 11 Understand Volume Concepts 8.1
A. Model X and Model Y have the same volume.
B. The volume of Model X is 9 cubic units greater than the volume of Model Y.
C. The volume of Model X is 19 cubic units greater than the volume of Model Y.
D. The volume of Model X and Model Y combined is 45 cubic units.
Answer:

C. The volume of Model X is 19 cubic units greater than the volume of Model Y.

Visual Learning Bridge

Essential Question How Can You Use a Formula to Find Stor the Volume of a Rectangular Prism?

A.
Remember that volume is the number of cubic units (units”) needed to pack a solid figure without gaps or overlaps.
Find the volume of the rectangular prism if each cubic unit represents 1 cubic foot.
Envision Math Common Core Grade 5 Answer Key Topic 11 Understand Volume Concepts 8.2

A formula is a rule that uses symbols to relate two or more quantities.
Envision Math Common Core Grade 5 Answer Key Topic 11 Understand Volume Concepts 8.3
You can find the volume of a rectangular prism by counting cubes or using a formula.

B.
If the dimensions of a rectangular prism are given as length l, width w, and height h, then use this formula to find the volume V:
Envision Math Common Core Grade 5 Answer Key Topic 11 Understand Volume Concepts 8.4
Volume = length × width × height
V = l × w × h
V = (6 × 4) × 3
V = 24 × 3
V = 72
The volume of the rectangular prism is 72 cubic feet or 72 ft3.

C.
Another formula for the volume of a rectangular prism is V = b × h, where b is the area of the base.
V = b × h.
V = 24 × 3
V = 72 ft3
Envision Math Common Core Grade 5 Answer Key Topic 11 Understand Volume Concepts 9.6

Convince Me! Reasoning Give the dimensions of a different rectangular prism that also has a volume of 72 ft3. Explain how you decided.

Guided Practice

Do You Understand?

Question 1.
In the Example on page 462, could you first multiply the width by the height? Explain.
Answer:

Question 2.
A wooden block measures 5 centimeters tall, 3 centimeters wide, and 2 centimeters long. The area of the base is 6 centimeters. Draw a rectangular prism to show the block and label it. What is the volume of the block?
Answer:

Do You Know How?

In 3 and 4, find the volume of each rectangular prism.

Question 3.
Envision Math Common Core Grade 5 Answer Key Topic 11 Understand Volume Concepts 10.1
Answer:

Volume = length × width × height
V = l × w × h
V = (9 × 2) × 4
V = 18 × 4
V = 72
The volume of the rectangular prism is 72 cubic inch or 72 in3.

Question 4.
Envision Math Common Core Grade 5 Answer Key Topic 11 Understand Volume Concepts 10.2
Answer:

The volume of a rectangular prism is V = b × h, where b is the area of the base.
V = b × h.
V = 24 × 9
V = 216 yd3

Independent Practice

In 5-10, find the volume of each rectangular prism.

Question 5.
Envision Math Common Core Grade 5 Answer Key Topic 11 Understand Volume Concepts 10.3
Answer:

Volume = length × width × height
V = l × w × h
V = (7 × 4) × 3
V = 28 × 3
V = 84

The volume of the rectangular prism is 84 cubic cm or 72 cm3.

Question 6.
Envision Math Common Core 5th Grade Answers Topic 11 Understand Volume Concepts 10.4
Answer:

Volume = length × width × height
V = l × w × h
V = (4 × 4) × 5
V = 16 × 5
V = 80
The volume of the rectangular prism is 80 cubic inch or 80 in3.

Question 7.
Envision Math Common Core 5th Grade Answers Topic 11 Understand Volume Concepts 10.5
Answer:

The volume of a rectangular prism is V = b × h, where b is the area of the base.
V = b × h.
V = 52 × 3
V = 156 m3

Question 8.
Envision Math Common Core 5th Grade Answers Topic 11 Understand Volume Concepts 10.6
Answer:

The volume of a rectangular prism is V = b × h, where b is the area of the base.
V = b × h.
V = 64 × 4
V = 256 cm3

Question 9.
Envision Math Common Core 5th Grade Answers Topic 11 Understand Volume Concepts 10.7
Answer:

Volume = length × width × height
V = l × w × h
V = (7 × 7) × 7
V = 49 × 7
V = 343
The volume of the rectangular prism is 343 cubic meter or 343 m3.

 

Question 10.
Envision Math Common Core 5th Grade Answers Topic 11 Understand Volume Concepts 10.8
Answer:

The volume of a rectangular prism is V = b × h, where b is the area of the base.
V = b × h.
V = 153 × 7
V = 1071 cm3

Problem Solving

Question 11.
The dictionary is 3 inches thick. What is the volume of the dictionary?
Envision Math Common Core 5th Grade Answers Topic 11 Understand Volume Concepts 11.1
Answer:

Volume = length × width × height
V = l × w × h
V = (7 × 3) × 9
V = 21 × 9
V = 189
The volume of the rectangular prism is 189 cubic inch or 189 in3.

Question 7.

Question 12.
Higher Order Thinking Two ovens have measurements as shown. Which oven has a greater volume? How much greater is its volume? Show your work.
Envision Math Common Core 5th Grade Answers Topic 11 Understand Volume Concepts 11.2
Answer:

The volume of a First oven is V = b × h, where b is the area of the base.
V = b × h.
V = 576 × 15
V = 8640 in3

The volume of a second oven is V = b × h, where b is the area of the base.
V = b × h.
V = 672 × 14
V = 9408 in3

The second oven has greater volume and 768 in3 greater than the first one.

Question 13.
The perimeter of an equilateral triangle is 51 feet. What is the length of one of its sides? Explain your work.
Answer:

The formula of the perimeter of an equilateral triangle is 3a, where a is the length of one side.

Given, The perimeter of an equilateral triangle is 51 feet

3a = 51

a = 51/3 = 17

Therefore, the length of one of its sides is 17 and all sides are equal.

 

Question 14.
Reasoning Harry is in line at the store. He has three items that cost $5.95, $4.25, and $1.05. Explain how Harry can add the cost of the items mentally before he pays for them.
Answer:

Given,

Harry has three items that cost $5.95, $4.25, and $1.05.

$5.95 approximately equals $6

$4.25  approximately equals $4

$1.05 approximately equals $1

Therefore, by approximation, Harry can add the cost of the items mentally before he pays for them.

$6 + $4 + $1 = $11. (approximately)

Assessment Practice

Question 15.
Choose all the Expressions that can be used to find the volume of this wooden box.
Envision Math Common Core 5th Grade Answers Topic 11 Understand Volume Concepts 11.3
Envision Math Common Core 5th Grade Answers Topic 11 Understand Volume Concepts 12 (6 × 4) < 3
Envision Math Common Core 5th Grade Answers Topic 11 Understand Volume Concepts 12 (6 × 4) + 3
Envision Math Common Core 5th Grade Answers Topic 11 Understand Volume Concepts 12 6 × 4
Envision Math Common Core 5th Grade Answers Topic 11 Understand Volume Concepts 12 6 × (4 × 3)
Envision Math Common Core 5th Grade Answers Topic 11 Understand Volume Concepts 12 24 × 3
Answer:

Envision Math Common Core 5th Grade Answers Topic 11 Understand Volume Concepts 12 6 × 4

Envision Math Common Core 5th Grade Answers Topic 11 Understand Volume Concepts 12 24 × 3

Lesson 11.3 Combine Volumes of Prisms

Activity

Solve & Share
Ariel is thinking of a three-dimensional figure that is made by combining two rectangular prisms. What is the volume of this three-dimensional figure? Solve this problem any way you choose.

Envision Math Common Core 5th Grade Answers Topic 11 Understand Volume Concepts 12.1
Use Structure You can find the volumes of the rectangular prisms that make up the solid figure. Show your work!
Envision Math Common Core 5th Grade Answers Topic 11 Understand Volume Concepts 12.2

Look Back! How did you separate the solid into simpler rectangular prisms? Write the dimensions of each of the prisms.

Visual Learning Bridge

Essential Question How Can You Find the Volume of a Solid Figure question Composed of Two Rectangular Prisms?

A.
The shape and size of a storage building are shown in the figure. The building supervisor wants to find the volume to determine how much storage space is available. What is the volume of the building?
Envision Math Common Core 5th Grade Answers Topic 11 Understand Volume Concepts 12.3
You can find the volume of this figure by finding the volume of two rectangular prisms that make up the figure.
Envision Math Common Core 5th Grade Answers Topic 11 Understand Volume Concepts 12.4

B.
The building can be separated into two rectangular prisms as shown. Identify the measurements for the length, width, and height of each prism.
Envision Math Common Core 5th Grade Answers Topic 11 Understand Volume Concepts 12.5

C.
Use the formula V = l × w × h to find the volume of each rectangular prism.
Envision Math Common Core 5th Grade Answers Topic 11 Understand Volume Concepts 12.30
Add to find the total volume.
180 + 630 = 810
The volume of the storage building is 810 cubic meters.

Convince Me! Reasoning What is another way to divide the solid above into two rectangular prisms? What are the dimensions of each prism?

Guided Practice

Do You Understand?

In 1 and 2, use the solid below. The dashed line separates it into two rectangular prisms, A and B.

Envision Math Common Core 5th Grade Answers Topic 11 Understand Volume Concepts 100

Question 1.
What are the length, width, and height of Prism A? What are the length, width, and height of Prism B?
Answer:

length, width, and height of Prism A are 4 ft, 3 ft, 12 ft respectively.

length, width, and height of Prism B are 3 ft, 2 ft, 5 ft respectively.

Question 2.
What is another way you could separate the shape into two rectangular prisms? What are each prism’s dimensions?
Answer:

Bisecting the figure horizontally, another way I could separate the shape into two rectangular prisms.

length, width, and height of Prism A are 4 ft, 3 ft, 7 ft respectively.

length, width, and height of Prism B are 6 ft, 3 ft, 5 ft respectively.

Do You Know How?

In 3 and 4, find the volume of each solid figure.

Question 3.
Envision Math Common Core 5th Grade Answers Topic 11 Understand Volume Concepts 13.1
Answer:

Volume = length × width × height
Volume of A = l × w × h
V = (5 × 7) × 8
V = 35 × 8
V = 280

The volume of the A is 280 cubic in or 280 in3.

Volume = length × width × height
Volume of B  = l × w × h
V = (10 × 7) × 2
V = 70 × 2
V = 140

The volume of the B is 140 cubic in or 140 in3.

Combined Volume  of Solid Figure is 280 in3 + 140 in= 420 in

Question 4.
Envision Math Common Core 5th Grade Answers Topic 11 Understand Volume Concepts 13.2
Answer:

Volume = length × width × height
Volume of A = l × w × h
V = (5 × 8) × 5
V = 40 × 5
V = 200

The volume of the A is 200 cubic cm or 200 cm3.

Volume = length × width × height
Volume of B  = l × w × h
V = (7 × 8) × 2
V = 56 × 2
V = 112

The volume of the B is 112 cubic cm or 112 cm3.

Combined Volume  of Solid Figure is 200 cm3 + 112 cm= 312 in3

Independent Practice

In 5-7, find the volume of each solid figure.

Question 5.
Envision Math Common Core 5th Grade Answers Topic 11 Understand Volume Concepts 13.6
Answer:

Volume = length × width × height
Volume of A = l × w × h
V = (8 × 4) × 7
V = 32 × 7
V = 224

The volume of the A is 224 cubic cm or 224 cm3.

Volume = length × width × height
Volume of B  = l × w × h
V = (6 × 4) × 3
V = 24 × 3
V = 72

The volume of the B is 72 cubic cm or 72 cm3.

Combined Volume  of Solid Figure is 224 cm3 + 72 cm= 296 cm3

Question 6.
Envision Math Common Core 5th Grade Answers Topic 11 Understand Volume Concepts 13.7
Answer:

Volume = length × width × height
Volume of A = l × w × h
V = (6 × 10) × 5
V = 60 × 5
V = 300

The volume of the A is 300 cubic ft or 300 ft3.

Volume = length × width × height
Volume of B = l × w × h
V = (6 × 2) × 4
V = 12 × 4
V = 48

The volume of the B is 48 cubic ft or 48 ft3.

Combined Volume  of Solid Figure is 300 ft3 + 48 ft= 348 ft3

Question 7.
Envision Math Common Core 5th Grade Answers Topic 11 Understand Volume Concepts 13.8
Answer:

Volume = length × width × height
Volume of A = l × w × h
V = (4 × 6) × 12
V = 24 × 12
V = 288

The volume of the A is 288 cubic m or 288 m3.

Volume = length × width × height
Volume of B  = l × w × h
V = (9 × 5) × 12
V = 45 × 12
V = 540

The volume of the B is 540 cubic m or 540 m3.

Combined Volume  of Solid Figure is 288 m3 + 540 m= 828 m3

Problem Solving

For 8-10, use the drawing of the solid figure.

Question 8.
How would you find the volume of the figure shown?
Answer:

Envision Math Common Core 5th Grade Answers Topic 11 Understand Volume Concepts 14.1

Question 9.
Algebra Write two Expressions that can be added to find the volume of the solid figure.
Answer:

Volume = length × width × height
Volume of A = l × w × h
V = (5 × 4) × 10
V = 20 × 10
V = 200

The volume of the A is 200 cubic m or 200 m3.

Volume = length × width × height
Volume of B  = l × w × h
V = (5 × 2) × 7
V = 10 × 7
V = 70

The volume of the B is 70 cubic m or 70 m3.

Question 10.
What is the volume of the solid figure?
Answer:

Combined Volume  of Solid Figure is 200 m3 + 70 m= 270 m3

 

Question 11.
Higher Order Thinking A solid figure is separated into two rectangular prisms. The volume of Rectangular Prism A is 80 cubic feet. Rectangular Prism B has a length of 6 feet and a width of 5 feet. The total volume of the solid figure is 200 cubic feet. What is the height of Rectangular Prism B? Show your work.
Answer:

Question 12.
Model with Math The Peters family will drive 615 miles to reach their vacation destination. If they drive 389 miles the first day, how many miles will they drive the second day? Complete the bar diagram to help.
Envision Math Common Core 5th Grade Answers Topic 11 Understand Volume Concepts 15.1
Answer:

Assessment Practice

Question 13.
A horizontal line separates the solid figure at the right into two rectangular prisms. Write an Expression for the volume of the solid figure.
Envision Math Common Core 5th Grade Answers Topic 11 Understand Volume Concepts 15.2
Answer:

Volume of A  = length × width × height
V = l × w × h
V = (6 × 10) × 5
V = 60 × 5
V = 300

The volume of the rectangular prism is 300 cubic in or 300 in3.

Volume of B  = length × width × height
V = l × w × h
V = (10 × 10) × 2
V = 100 × 2
V = 200

The volume of the rectangular prism is 200 cubic in or 200 in3.

Total volume =  200 in3 + 300 in= 500 in3

Lesson 11.4 Solve Word Problems Using Volume

Activity

Solve & Share
A school has two wings, each of which is a rectangular prism. The school district is planning to install air conditioning in the school and needs to know its volume. What is the volume of the school? Solve this problem any way you choose.

Model with Math Write a multiplication Expression for the volume of each wing of the building.
Envision Math Common Core 5th Grade Answers Topic 11 Understand Volume Concepts 15.3

Look Back! Write a mathematical Expression that can be used to find the total volume of the school.

Visual Learning Bridge

Essential Question How Can You Use Volume Formulas to Solve Real-World Problems?

A.
The nature center has a large bird cage called an aviary. It consists of two sections, each shaped like a rectangular prism. There needs to be 10 cubic feet of space for each bird. How many birds can the nature center have in the aviary?
Envision Math Common Core 5th Grade Answers Topic 11 Understand Volume Concepts 15.4

You can make sense of the problem by breaking it apart into simpler problems.

B.
Find the volume of each section. Use the formula V = l × w × h.
Small section:
V = 4 × 3 × 8 = 96
Large section:
V = 10 × 6 × 8 = 480
Add to find the total volume:
96 + 480 = 576
The combined volume is 576 cubic feet.

C.
Divide to find the number of birds that will fit.
Envision Math Common Core 5th Grade Answers Topic 11 Understand Volume Concepts 16.1
576 ÷ 10 = 57.6
The nature center can put 57 birds in the aviary.

Convince Me! Critique Reasoning Tom solved the problem in a different way. First, he found the total area of the floor, and then he multiplied by the height. Does Tom’s method work? Explain.

Guided Practice

Do You Understand?

Question 1.
How can you find the volume of the china cabinet?
Envision Math Common Core 5th Grade Answers Topic 11 Understand Volume Concepts 16.2

Question 2.
What is the height of the top section of the china cabinet? Explain.

Height of the top section of the china cabinet is 4 ft

Because, 7 ft – 3 ft = 4 ft

Question 3.
Find the volume of the china cabinet.
Answer:

Volume of china cabinet  = length × width × height
V = l × w × h
V = (4 × 1) × 4
V = 4 × 4
V = 16

The volume of the rectangular prism is 16 cubic ft or 16 ft3.

 

Do You Know How?

Question 4.
Find the volume of the building below.
Envision Math Common Core 5th Grade Answers Topic 11 Understand Volume Concepts 16.3
Answer:

Volume of A = length × width × height
V = l × w × h
V = (35 × 54) × 82
V = 1890 × 82
V = 154,980

The volume of the rectangular prism is 154,980 cubic ft or 154,980 ft3.

Volume of B = length × width × height
V = l × w × h
V = (40 × 30) × 50
V = 1200 × 50
V = 60000

The volume of the rectangular prism is 60000 cubic ft or 60000 ft3

Therefore, Combined volume is  154,980 ft3 + 60000 ft= 214,980 ft3

Question 5.
The nature center has a fish tank shaped like a rectangular prism that measures 6 feet long by 4 feet wide by 4 feet high. It can be stocked safely with 3 small fish in each cubic foot of water. How many small fish can safely fit in the tank?
Answer:

Volume of rectangular prism  = length × width × height
V = l × w × h
V = (6 × 4) × 4
V = 24 × 4
V = 96

The volume of the rectangular prism is 96 cubic ft or 96 ft3.

If it can be stocked safely with 3 small fish in each cubic foot of water. Then 96/3 = 12

12 fish can be stocked safely with 3 small fish in each cubic foot of water.

Independent Practice

Question 6.
Sophie built a house out of building blocks. Find the volume of the house Sophie built.
Envision Math Common Core 5th Grade Answers Topic 11 Understand Volume Concepts 16.20
Answer:

Volume of A = length × width × height
V = l × w × h
V = (3 × 5) × 10
V = 15 × 10
V = 150

The volume of the rectangular prism is 150 cubic cm or 150 cm3.

Volume of B = length × width × height
V = l × w × h
V = (20 × 24) × 20
V = 480 × 20
V = 9600

The volume of the rectangular prism is 9600 cubic cm or 9600 cm3

Therefore, Combined volume is  150 cm3 + 9600 cm= 9750 cm3

Question 7.
How many cubic inches of concrete would it take to make these stairs?
Envision Math Common Core 5th Grade Answers Topic 11 Understand Volume Concepts 16.31
Answer:

Volume of first stair = length × width × height
V = l × w × h
V = (40 × 10) × 7
V = 400 × 7
V = 2800

The volume of the first stair is 2800 cubic in or 2800 in3.

Volume of Second stair = length × width × height
V = l × w × h
V = (40 × 10) × 14
V = 400 × 14
V = 5600

The volume of the second stair is 5600 cubic in or 5600 in3

Volume of third stair = length × width × height
V = l × w × h
V = (40 × 10) × 21
V = 400 × 21
V = 8400

The volume of the rectangular prism is 8400 cubic in or 8400 in3

Therefore, Combined volume is  2800 in3 + 5600 in3 + 8400 in= 16800 in3

16800 cubic inches of concrete it takes to make these stairs

Problem Solving

Question 8.
A floor plan of Angelica’s bedroom and closet is shown at the right. The height of the bedroom is 9 feet. The height of the closet is 7 feet. What is the total volume of the bedroom and the closet?
Envision Math Common Core 5th Grade Answers Topic 11 Understand Volume Concepts 16.4
Answer:

Volume of bedroom = length × width × height
V = l × w × h
V = (14 × 12) × 9
V = 168 × 9
V = 1512

The volume of the second stair is 1512 cubic ft or 1512 ft3

Volume of  closet = length × width × height
V = l × w × h
V = (3 × 4) × 7
V = 12 × 7
V = 84

The volume of the rectangular prism is 84 cubic ft or 84 ft3

Therefore, Combined volume is  1512 ft3 + 84 ft3  = 11596 ft3

Question 9.
Critique Reasoning Does it make sense for Angelica to find the combined area of the bedroom floor and closet floor before finding the total volume? Explain your thinking.
Answer:

Yes. the total volume is sum of the bedroom and closet of Angelica room

Question 10.
Higher Order Thinking An office building surrounds a rectangular open air courtyard. What is the volume of the building? How did you find the answer?
Envision Math Common Core Grade 5 Answers Topic 11 Understand Volume Concepts 16.5
Answer:

Volume of building = length × width × height
V = l × w × h
V = (64 × 60) × 28
V = 3840 × 28
V = 107,520

The volume of the second stair is 107,520 cubic m or 107,520 m3

Volume of open air courtyard = length × width × height
V = l × w × h
V = (18 × 30) × 28
V = 540 × 28
V = 15120

The volume of the rectangular prism is 15120 cubic m or 15120 m3

Therefore, Total volume of building is  107,520 m3 – 15120 m3  = 92400 m3

Assessment Practice

Question 11.
Mrs. Bhatia’s closet consists of two sections, each shaped like a rectangular prism. She plans to buy mothballs to keep the moths away. She needs one box for every 32 cubic feet of space. How many boxes should she buy?
A. 6 boxes
B. 7 boxes
C. 8 boxes
D. 10 boxes
Envision Math Common Core Grade 5 Answers Topic 11 Understand Volume Concepts 16.6
Answer:

Volume of first section = length × width × height
V = l × w × h
V = (5 × 4) × 8
V = 20 × 8
V = 160

The volume of the first section is 160 cubic ft or 160 ft3

Volume of open second section = length × width × height
V = l × w × h
V = (3 × 2) × 8
V = 6 × 8
V = 48

The volume of the second section is 48 cubic ft or 48 ft3

Therefore, the total volume of the building is  160 ft3 + 48 ft3  = 208 ft3

If she needs one box for every 32 cubic feet of space. Then Number of boxes = 208 / 32 = 6.5

She needs 6 or 7 boxes.

 

Lesson 11.5 Use Appropriate Tools

Problem Solving

Solve & Share
A space station is being built from 24 cubic modules. The space station can be any shape but the modules must be placed together so that entire faces match up with each other. Choose a tool to create two different plans for the space station. Explain why you chose the tool you selected.

Thinking Habits
Be a good thinker! These questions can help you.
• Which tools can I use?
• Why should I use this tool to help me solve the problem?
• Is there a different tool I could use?
• Am I using the tool appropriately?
Envision Math Common Core Grade 5 Answers Topic 11 Understand Volume Concepts 17.1

Look Back! Use Appropriate Tools How did you decide which tool to use?

Visual Learning Bridge

Glossory

Essential Question How Can You Use Appropriate Tools question to Solve Volume Problems?

A.
Jeremiah needs to build a display of boxes that is 4 feet tall.
The boxes he uses are cubes that measure 1 foot on each edge. His display needs to look like a pyramid, with just one box in the top layer.
How many boxes will Jeremiah need to make his display?
Envision Math Common Core Grade 5 Answers Topic 11 Understand Volume Concepts 17.2

What do I need to do?
I need to choose an appropriate tool to solve this problem.

Here’s my thinking…
Envision Math Common Core Grade 5 Answers Topic 11 Understand Volume Concepts 17.3

B.
How can I use appropriate tools strategically to help me solve this problem?
I can
• decide which tool is appropriate.
• use cubes to solve this problem.
• use the tool correctly.

C.
I could use grid paper, but I will use cubes because building a display will make it easier to count the cubes.
Each cube represents 1 box in the display. My display will have 4 layers because it needs to be 4 feet tall, and each box is 1 foot high.
The display has 1 + 4 + 9 + 16 = 30 cubes.
Envision Math Common Core Grade 5 Answers Topic 11 Understand Volume Concepts 17.4
So, Jeremiah needs 30 boxes in all to make his display.

Convince Me! Use Appropriate Tools What tools other than cubes could you use to solve this problem? Explain.

Guided Practice

Use Appropriate Tools
A paint store manager is going to build a display with same-sized cubes. The display will look like a staircase with 5 steps. Each step in the display will be 6 cubes long. The store manager will build the staircase display with 1-foot plastic cubes.

Envision Math Common Core Grade 5 Answers Topic 11 Understand Volume Concepts 20.1

Question 1.
What tool might the manager use to be sure that there is enough space for the display? Explain.
Answer:

Question 2.
What is the volume of the display? Explain how you used tools to decide.
Answer:

Independent Practice

Use Appropriate Tools

Cindy plans to make a jewelry box shaped like a rectangular prism. She wants it to have a volume of 96 cubic inches.

Question 3.
How can you find possible dimensions of the box?
Answer:

Think about a tool you can use to help represent and solve the problem.
Envision Math Common Core Grade 5 Answers Topic 11 Understand Volume Concepts 20.3

Question 4.
What could the dimensions of that jewelry box be?
Answer:

Question 5.
Can Cindy build the box so that it is twice as wide as it is tall?
Answer:

Question 6.
Cindy has some ribbon to decorate the jewelry box. What tool might help her decide how much of the jewelry box she can decorate?
Answer:

Problem Solving
Performance Task

Flower Planters
An architect is designing flower planters for a park. Each planter consists of a border of 1-foot concrete cubes surrounding a square opening. Each concrete cube weighs 120 pounds. The diagram below shows the top view of some of the planters.
Envision Math Common Core Grade 5 Answers Topic 11 Understand Volume Concepts 21.1

Remember to think about which tools make sense for these problems.

Question 7.
Use Structure What is the total volume of a planter that has a 6 × 6 opening?
Answer:

Question 8.
Use Structure What is the total volume of a planter that has an 8 × 8 opening?
Answer:

Question 9.
Use Appropriate Tools What will be the total volume of a planter that has a 12 × 12 opening? Can you determine this by just using paper and pencil? Explain.
Answer:

Question 10.
Model with Math Each concrete cube used to make the planters costs $3.00. What is the total cost of the cubes needed for two planters with 6 × 6 openings, two with 8 × 8 openings, and two with 12 × 12 openings? Write an Expression that represents the total cost.
Answer:

Topic 11 Fluency Practice

Activity

Find a Match

Work with a partner. Point to a clue. Read the clue.
Look below the clues to find a match. Write the clue letter in the box above the match.
Find a match for every clue.

Envision Math Common Core Grade 5 Answers Topic 11 Understand Volume Concepts 50.1

Envision Math Common Core Grade 5 Answers Topic 11 Understand Volume Concepts 50.2

Answer:

Topic 11 Vocabulary Review

Word List

Glossory

Understand Vocabulary

In 1-3, choose the best term from the Word List. Write it on the blank.
• area
• cubic unit
• formula
• rectangular prism
• unit cube
• volume

Question 1.
The number of same-size unit cubes that fill a solid figure without overlaps or gaps in the figure’s ____
Answer:

The number of same-size unit cubes that fill a solid figure without overlaps or gaps in the figure’s volume

Question 2.
A solid figure with 6 rectangular faces that are not all squares is a(n) ___
Answer:

A solid figure with 6 rectangular faces that are not all squares is a rectangular prism

Question 3.
A rule that uses symbols to relate two or more quantities is a(n) ___.
Answer:

A rule that uses symbols to relate two or more quantities is a formula

Question 4.
Cross out the Expression(s) below that do NOT represent the volume of the prism.
Envision Math Common Core Grade 5 Answers Topic 11 Understand Volume Concepts 51.1
Answer:

Draw a line from each three-dimensional figure in Column A to its volume in Column B.

Envision Math Common Core Grade 5 Answers Topic 11 Understand Volume Concepts 52.1

Answer:

Question 8.
One box is 3 inches by 4 inches by 5 inches. A second box is 4 inches by 4 inches by 4 inches. Explain how to decide which box holds more.
Answer:

Volume of the first box is, V = 3 x 4 x 5 = 60 cubic inches.

Volume of the second box is, V = 4 x 4 x 4 = 64 cubic inches.

The second box holds more because it has more volume.

Topic 11 Reteaching

Set A
pages 457-460

Find the number of cubes needed to make this rectangular prism.
Envision Math Common Core Grade 5 Answers Topic 11 Understand Volume Concepts 60.1
Multiply to find the total number of cubes.
3 × 5 × 3 = 45
The volume is 45 cubic units.

Remember, you can multiply the numbers in any order!
Envision Math Common Core Grade 5 Answers Topic 11 Understand Volume Concepts 60.6

Remember that you can find the number of cubes in each layer and then multiply by the number of layers.

Find each volume. You may use cubes to help.

Question 1.
Envision Math Common Core Grade 5 Answers Topic 11 Understand Volume Concepts 60.8
Answer:

Each cube of a solid figure is 1 cubic unit.

There are 14 unit cubes in the bottom layer. The volume of the bottom layer is 14 cubic units.

The volume of the prism is 14 × 2 or 28 cubic units.

Question 2.
Envision Math Common Core Grade 5 Answers Topic 11 Understand Volume Concepts 60.9
Answer:

Each cube of a solid figure is 1 cubic unit.

There are 10 unit cubes in the bottom layer. The volume of the bottom layer is 10 cubic units.

The volume of the prism is 10 × 2 or 20 cubic units.

Question 3.
Envision Math Common Core Grade 5 Answers Topic 11 Understand Volume Concepts 60.10
Answer:

Each cube of a solid figure is 1 cubic unit.

There are 12 unit cubes in the bottom layer. The volume of the bottom layer is 12 cubic units.

The volume of the prism is 12 × 6 or 72 cubic units.

Set B
pages 461-464
Find the volume of this rectangular prism.
Envision Math Common Core Grade 5 Answers Topic 11 Understand Volume Concepts 60.11
Volume = length × width × height V=l×w×h
= 9 cm × 4 cm × 2 cm
V = 72 cubic centimeters or 72 cm3
The volume of the prism is 72 cm3.

Remember if you know the area of the base of a rectangular prism, use the formula V = b × h, where b is the area of the base.

Find each volume. You may use cubes to help.

Question 1.
Area of the base, b = 42 square meters and height = 3 meters
Answer:

The volume of a rectangular prism is V = b × h, where b is the area of the base.
V = b × h.
V = 42 × 3
V = 126 m3

Question 2.
Area of the base = 75 square inches and height = 15 inches
Answer:

The volume of a rectangular prism is V = b × h, where b is the area of the base.
V = b × h.
V = 75 × 15
V = 1125 in3

Question 3.
Envision Math Common Core Grade 5 Answers Topic 11 Understand Volume Concepts 65.1
Answer:

Volume = length × width × height
Volume = l × w × h
V = (8 × 4) × 3
V = 32 × 3
V = 96

The volume of the B is 96 cubic ft or 70 ft3.

Set C
pages 465-468, 469-472

Some solid figures can be separated into two rectangular prisms.
Envision Math Common Core Grade 5 Answers Topic 11 Understand Volume Concepts 65.2
Add the volume of each prism to find the total volume of the solid figure.
V = (4 × 4 × 5) + (10 × 4 × 5)
= 80 + 200
= 280
The volume of the solid figure is 280 cubic inches.

Remember to identify the length, width, and height of each prism, so that you can calculate the volume of each part.

Question 1.
Find the volume.
Envision Math Common Core Grade 5 Answers Topic 11 Understand Volume Concepts 70.1
Answer:

Volume = length × width × height
Volume of A = l × w × h
V = (10 × 6) × 2
V = 60 × 2
V = 120

The volume of the A is 120 cubic cm or 120 cm3.

Volume = length × width × height
Volume of B  = l × w × h
V = (2 × 6) × 3
V = 12 × 3
V = 36

The volume of the B is 36 cubic cm or 36 cm3.

Combined Volume  of Solid Figure is 120 cm3 + 36 cm= 156 cm3

Question 2.
An office building has the dimensions shown. What is the volume of the building?
Envision Math Common Core Grade 5 Answers Topic 11 Understand Volume Concepts 70.2
Answer:

Volume = length × width × height
Volume of A = l × w × h
V = (70 × 50) × 90
V = 3500 × 90
V = 315,000

The volume of the A is 315,000 cubic m or 315,000 m3.

Volume = length × width × height
Volume of B  = l × w × h
V = (60 × 50) × 20
V = 3000 × 20
V = 60000

The volume of the B is 60000 cubic m or 60000 m3.

Combined Volume  of Solid Figure is 315,000 m3 + 60000 m= 375,000 m3

Set D
pages 473-476
Think about these questions to help you use appropriate tools strategically.

Thinking Habits
Molly used 1-inch cubes to build the structure shown. She left a 3-inch by 1-inch opening in both layers of the structure.
• Which tools can I use?
• Why should I use this tool to help me solve the problem?
• Is there a different tool I could use?
• Am I using the tool appropriately?
Envision Math Common Core Grade 5 Answers Topic 11 Understand Volume Concepts 101

Remember that tools such as place-value blocks, cubes, and grid paper can help you solve problems involving volume.

Envision Math Common Core 5th Grade Answer Key Topic 11 Understand Volume Concepts 70.4

Question 1.
What tools could you use to model the problem?
Answer:

cubes

Question 2.
What is the total volume of the structure?
Answer:

Total blocks in bottom layer = 5 x 3 =15

Height =2 ; total volume is 15 x 2 = 30.

blocks left = 3 x 1 x 2 = 6

Therefore, total volume = 30 – 6 = 24 cubic units

Topic 11 Assessment Practice

Question 1.
Julio used unit cubes to make a rectangular prism. What is the volume of the prism?
Envision Math Common Core 5th Grade Answer Key Topic 11 Understand Volume Concepts 70.40
A. 18 cubic units
B. 72 cubic units
C. 54 cubic units
D. 108 cubic units
Answer:

B. 72 cubic units

Question 2.
Select the possible dimensions for a prism with each given volume.
Envision Math Common Core 5th Grade Answer Key Topic 11 Understand Volume Concepts 80.1
Answer:

Question 3.
A. A swimming pool is 50 meters long, 15 meters wide, and 3 meters deep. What is the volume of the pool?
A. 4,500 cubic meters
B. 2,250 cubic meters
C. 900 cubic meters
D. 750 cubic meters
B. After filling the pool for several minutes, the water is 1 meter deep. What is the volume of water in the pool?
Answer:

A.

Volume = length × width × depth
Volume of Swimming pool = l × w × h
V = (50 × 15) × 3
V = 750 × 3
V = 2250

The volume of the Swimming pool is 2250 cubic m or 2250 m3.

B.

Volume = length × width × depth
Volume of Swimming pool after filling the pool for several minutes, the water is 1 meter deep =
V = (50 × 15) × 4
V = 750 × 4
V = 3000

The volume of the Swimming pool is 3000 cubic m or 3000 m3.

Question 4.
A small building has the dimensions shown.
A. Write an Expression for the total volume of the building.
Envision Math Common Core 5th Grade Answer Key Topic 11 Understand Volume Concepts 80.2
B. What is the volume of the building?
Answer:

A.

Volume = length × width × height
Volume of A = l × w × h
V = (8 × 12) × 24
V = 96 × 24
V = 2304

The volume of the A is 2304 cubic ft or 2304 ft3.

Volume = length × width × height
Volume of B  = l × w × h
V = (32 × 12) × 16
V = 384 × 16
V = 6114

The volume of the B is 6114 cubic ft or 6114 ft3

B.

Combined Volume  of Solid Figure is 2304 ft3 + 6114 ft= 8418 ft3

Question 5.
A. Choose all the Expressions that could NOT be used to find the volume of the bale of hay.
Envision Math Common Core 5th Grade Answer Key Topic 11 Understand Volume Concepts 103
Envision Math Common Core 5th Grade Answer Key Topic 11 Understand Volume Concepts 81 100 × 40
Envision Math Common Core 5th Grade Answer Key Topic 11 Understand Volume Concepts 81 (100 × 40) × 20
Envision Math Common Core 5th Grade Answer Key Topic 11 Understand Volume Concepts 81 4,000 × 20
Envision Math Common Core 5th Grade Answer Key Topic 11 Understand Volume Concepts 81 (100 × 40) + 20
Envision Math Common Core 5th Grade Answer Key Topic 11 Understand Volume Concepts 81 (100 + 40) + 20
B. Another 10 cm of hay is added to the top of the bale. What is the volume of the bale of hay now?
Answer:

A.

Envision Math Common Core 5th Grade Answer Key Topic 11 Understand Volume Concepts 81 (100 × 40) × 20
Envision Math Common Core 5th Grade Answer Key Topic 11 Understand Volume Concepts 81 4,000 × 20

B.

Volume = length × width × height
Volume of bale of hay  = l × w × h
V = (100 × 40) × 30
V = 4000 × 30
V = 120000

The volume of the bale of hay is 120000 cubic cm or 120000 cm3

Question 6.
Madeline made the wooden steps shown. What is the volume of the steps?
Envision Math Common Core 5th Grade Answer Key Topic 11 Understand Volume Concepts 82
A. 72 cubic inches
B. 540 cubic inches
C. 840 cubic inches
D. 1,080 cubic inches
Answer:

Volume = length × width × height
Volume of A = l × w × h
V = (10 × 9) × 6
V = 90 × 6
V = 540

The volume of the A is 540 cubic in or 540 in3.

Volume = length × width × height
Volume of B  = l × w × h
V = (10 × 5) × 6
V = 50 × 6
V = 300

The volume of the B is 300 cubic in or 300 in3

Combined Volume  of Solid Figure is 540 in3 + 300 in= 840 in3

Question 7.
A. What is the volume of the trunk shown?
Envision Math Common Core 5th Grade Answer Key Topic 11 Understand Volume Concepts 83
B. Which equation was used to find the volume of the trunk?
A. V = b × h
B. V = l × w × h
C. V = l × w
D. V = b × b × h
Answer:

A.

The volume of a rectangular prism is V = b × h, where b is the area of the base.
V = b × h.
V = 750 × 25
V = 18750 in3

B.

Option A. V = b × h

Question 8.
For her science project, Jada wants to build a rectangular prism out of foam blocks. The prism should have a volume of 350 cubic inches and a height of 5 inches.
What does the area of the base of the prism need to be for the given volume and height? Give one pair of possible whole-number dimensions for the base.
Answer:

The volume of a rectangular prism is V = b × h, where b is the area of the base.
V = b × h.
350 = b × 5
b = 350/5 = 70

The area of the base of the prism needs to be for the given volume and height is 70 in2

Question 9.
Martin’s suitcase has a volume of 1,080 cubic inches. Lily’s suitcase measures 9 inches wide, 13 inches long, and 21 inches high. What is the combined volume of the two suitcases?
Answer:

The volume of the first suitcase is 1080 cubic in or 1080 in3.

Volume = length × width × height
Volume of Second suitcase is  = l × w × h
V = (9 × 13) × 21
V = 117 × 21
V = 2457

The volume of the second suitcase is 2457 cubic in or 2457 in3

The combined volume of both suitcases is 1080 in3 + 2457 in= 3537 in3

Question 10.
Select all the Expressions that can be used to find the volume of the box in cubic centimeters.
Envision Math Common Core 5th Grade Answer Key Topic 11 Understand Volume Concepts 104
Envision Math Common Core 5th Grade Answer Key Topic 11 Understand Volume Concepts 84 8 × 6
Envision Math Common Core 5th Grade Answer Key Topic 11 Understand Volume Concepts 84 (4 × 8) × 6
Envision Math Common Core 5th Grade Answer Key Topic 11 Understand Volume Concepts 84 32 × 6
Envision Math Common Core 5th Grade Answer Key Topic 11 Understand Volume Concepts 84 46 × 8
Envision Math Common Core 5th Grade Answer Key Topic 11 Understand Volume Concepts 84 (4 × 8) + 6
Answer:

Envision Math Common Core 5th Grade Answer Key Topic 11 Understand Volume Concepts 84 (4 × 8) × 6
Envision Math Common Core 5th Grade Answer Key Topic 11 Understand Volume Concepts 84 32 × 6

Topic 11 Performance Task

Sporting Goods
Hiroto works in a sporting goods store. 1. Hiroto stacks identical boxes of golf balls to form a rectangular prism. Each box is a cube.
Envision Math Common Core 5th Grade Answer Key Topic 11 Understand Volume Concepts 85
Part A
How many boxes are in the Golf Ball Display?

Answer:

There are 84 boxes in the Golf Ball Display
Part B
Explain how the number of boxes you found in Part A is the same as what you would find by using the formula V= l × w ×h.

Answer:

Each block has volume of 1 cubic unit.

Length = 7 boxes

Breadth = 4 boxes

Height = 3 boxes

Volume is length x breadth x height

V= l × w ×h = 7 x 4 x 3

v = 28 x 3 = 84 cubic units.

Part C
Hiroto needs to restack the boxes so the display is 2 layers high, less than 14 inches wide, and less than 30 inches long. The size of each box is shown in at the right. What is one way Hiroto can stack the boxes? Justify your answer.
Envision Math Common Core 5th Grade Answer Key Topic 11 Understand Volume Concepts 86
Part D
What is the volume of the golf ball display in cubic inches? Explain how you solved.
Answer:

Question 2.
Hiroto builds two displays using rectangular foam blocks.
Part A
What is the volume of the foam block used for the Baseball Hats and Helmets Display? Explain how to solve using the formula V = b × h.
Envision Math Common Core 5th Grade Answer Key Topic 11 Understand Volume Concepts 86.1
Part B
Hiroto used two blocks to build the Baseball Uniforms Display. What is the combined volume of the blocks? Explain how you solved.
Envision Math Common Core 5th Grade Answer Key Topic 11 Understand Volume Concepts 87
Part C
Explain how you knew which units to use for your answer to Part B.
Answer:

Envision Math Common Core Grade 5 Answer Key Topic 8 Apply Understanding of Multiplication to Multiply Fractions

Practice with the help of enVision Math Common Core Grade 5 Answer Key Topic 8 Apply Understanding of Multiplication to Multiply Fractions regularly and improve your accuracy in solving questions.

Envision Math Common Core 5th Grade Answers Key Topic 8 Apply Understanding of Multiplication to Multiply Fractions

Envision Math Common Core Grade 5 Answer Key Topic 8 Apply Understanding of Multiplication to Multiply Fractions 1
enVision STEM Project: Kitchen Chemistry
Do Research Use the Internet or other sources to learn about physical changes to substances. Look for examples of physical changes that occur in the kitchen. When you condense, freeze, melt, vaporize, or whip air into a substance, you are making physical changes to that substance.
Journal: Write a Report Include what you found. Also in your report:
• Give examples of foods that are commonly condensed, frozen, melted, vaporized, or whipped.
• Write your favorite recipe that involves making physical changes to the food.
• Make up and solve multiplication problems with fractions and mixed numbers.

Review What You Know

Vocabulary

• benchmark fractions
• mixed number
• equivalent fractions
• multiple
• factor

Choose the best term from the box. Write it on the blank.

Question 1.
To estimate the sum of two or more fractions, replace the addends with _____
Answer:

Question 2.
You can find ____ by multiplying both the numerator and the denominator of a fraction by the same nonzero number.
Answer:

Question 3.
A ____ of a number is a product of the number and any nonzero whole number.
Answer:

Multiply and Divide

Find each product or quotient.

Question 4.
108 × 2
Answer:

Question 5.
270 ÷ 30
Answer:

Question 6.
243 × 20
Answer:

Question 7.
288 ÷ 24
Answer:

Question 8.
456 × 11
Answer:

Question 9.
432 ÷ 24
Answer:

Fraction Sums and Differences
Find each answer

Question 10.
\(\frac{5}{9}\) + \(\frac{8}{9}\)
Answer:

Question 11.
2\(\frac{2}{3}\) + 5\(\frac{1}{2}\)
Answer:

Question 12.
\(\frac{11}{12}\) – \(\frac{2}{3}\)
Answer:

Question 13.
6\(\frac{7}{10}\) – 2\(\frac{3}{5}\)
Answer:

Question 14.
At the library, Herb spent \(\frac{1}{6}\) hour looking for a book, \(\frac{1}{4}\) hour reading, and \(\frac{1}{2}\) hour doing research on the computer. How many hours did Herb spend at the library?
Answer:

Common Denominators

Question 15.
Explain how you can find a common denominator for \(\frac{3}{5}\) and \(\frac{5}{8}\).
Answer:

Pick a Project

PROJECT 8A
What story does your quilt tell?
Project: Design a Quilt
Envision Math Common Core Grade 5 Answer Key Topic 8 Apply Understanding of Multiplication to Multiply Fractions 5.3

PROJECT 8B
Can you make art with just sticky notes?
Project: Create a Mosaic with Sticky Notes
Envision Math Common Core Grade 5 Answer Key Topic 8 Apply Understanding of Multiplication to Multiply Fractions 5.5

PROJECT 8C
How much calcium does your body need?
Project: Analyze Menus for Calcium-Rich Foods
Envision Math Common Core Grade 5 Answer Key Topic 8 Apply Understanding of Multiplication to Multiply Fractions 5.6

PROJECT 8D
Have you ever been in a cave?
Project: Create a Scale Model of a Cave
Envision Math Common Core Grade 5 Answer Key Topic 8 Apply Understanding of Multiplication to Multiply Fractions 5.7

Lesson 8.1 Multiply a Fraction by a Whole Number

Solve & Share
Sasha walked \(\frac{1}{2}\) mile every day for 5 days. How far did she walk? Draw a picture or use any model to help you solve the problem.

Model with Math What are some different ways you can model multiplication problems?
Envision Math Common Core Grade 5 Answer Key Topic 8 Apply Understanding of Multiplication to Multiply Fractions 5.8

Look Back! How does using a model help you multiply a fraction by a whole number?

Visual Learning Bridge

Essential Question What Are Some Ways to Multiply a on Fraction by a Whole Number?

A.
Joann wants to make 6 batches of fruit punch. How many cups of orange juice does she need?
Envision Math Common Core Grade 5 Answer Key Topic 8 Apply Understanding of Multiplication to Multiply Fractions 5.9

B.
One way to represent 6 × \(\frac{2}{3}\) is to use repeated addition.
Envision Math Common Core Grade 5 Answer Key Topic 8 Apply Understanding of Multiplication to Multiply Fractions 5.10

C.
You can think of \(\frac{2}{3}\) as 2 times \(\frac{1}{3}\).
Envision Math Common Core Grade 5 Answer Key Topic 8 Apply Understanding of Multiplication to Multiply Fractions 100
\(\frac{2}{3}\) = 2 × \(\frac{1}{3}\)
So, 6 × \(\frac{2}{3}\) = 6 × (2 × \(\frac{1}{3}\)).
Use the Associative Property.
6 × (2 × \(\frac{1}{3}\)) = (6 × 2) × \(\frac{1}{3}\)
= 12 × \(\frac{1}{3}\)
= \(\frac{12}{3}\) = 4
Joann needs 4 cups of orange juice to make 6 batches of punch.

Convince Me! Use Structure Find 10 × \(\frac{3}{5}\). Use repeated addition to check your answer. Show all of your work.

Guided Practice

Do You Understand?

Question 1.
Explain why 8 × \(\frac{3}{4}\) is the same as adding \(\frac{3}{4}\) + \(\frac{3}{4}\) + \(\frac{3}{4}\) + \(\frac{3}{4}\) + \(\frac{3}{4}\) + \(\frac{3}{4}\) + \(\frac{3}{4}\) + \(\frac{3}{4}\)
Answer:

Question 2.
Find 2 × \(\frac{3}{5}\). Shade the model to help solve.
Envision Math Common Core Grade 5 Answer Key Topic 8 Apply Understanding of Multiplication to Multiply Fractions 15.1
Answer:

Do You Know How?

Question 3.
Find 3 × \(\frac{2}{3}\) using repeated addition.
Answer:

Question 4.
Find 6 × \(\frac{3}{4}\) using the Associative Property.
Answer:

Independent Practice

Leveled Practice In 5-7, complete each equation to find the product.

Question 5.
Envision Math Common Core Grade 5 Answer Key Topic 8 Apply Understanding of Multiplication to Multiply Fractions 15.3
Answer:

Question 6.
Envision Math Common Core Grade 5 Answer Key Topic 8 Apply Understanding of Multiplication to Multiply Fractions 15.4
Answer:

Question 7.
Envision Math Common Core Grade 5 Answer Key Topic 8 Apply Understanding of Multiplication to Multiply Fractions 15.5
Answer:

In 8-15, find each product. Use models to help, if necessary.

Question 8.
35 × \(\frac{3}{5}\)
Answer:

Question 9.
7 × \(\frac{5}{12}\)
Answer:

Question 10.
9 × \(\frac{2}{3}\)
Answer:

Question 11.
300 × \(\frac{1}{2}\)
Answer:

Question 12.
64 × \(\frac{3}{8}\)
Answer:

Question 13.
900 × \(\frac{2}{3}\)
Answer:

Question 14.
84 × \(\frac{1}{4}\)
Answer:

Question 15.
42 × \(\frac{2}{7}\)
Answer:

Problem Solving

Question 16.
Higher Order Thinking Explain how you would find 36 × \(\frac{3}{4}\) mentally.
Answer:

Question 17.
Each lap around a track is \(\frac{5}{6}\) kilometer. Samantha drove around the track 24 times. How far did Samantha drive?
Answer:

Question 18.
Drake is making capes. He uses \(\frac{1}{3}\) yard of fabric for each cape he makes. What is the total amount of fabric Drake needs to make 96 capes?
Answer:

Question 19.
Bradley is making fruit salad. For each bowl of fruit salad, he needs \(\frac{3}{4}\) cup of grapes. How many cups of grapes will he use if he makes 24 bowls of fruit salad?
Answer:

Question 20.
Construct Arguments Do you think the difference 1.4 – 0.95 is less than 1 or greater than 1? Explain.
Answer:

Question 21.
Write a multiplication Expression that shows 106.
Answer:

Question 22.
The table shows the number of miles each person ran this week. Who ran more miles by the end of the week? How many more?
Envision Math Common Core Grade 5 Answer Key Topic 8 Apply Understanding of Multiplication to Multiply Fractions 20.1
Answer:

Assessment Practice

Question 23.
Select all equations that would be made true with the fraction \(\frac{3}{8}\)
Envision Math Common Core Grade 5 Answer Key Topic 8 Apply Understanding of Multiplication to Multiply Fractions 21
Answer:

Question 24.
Select all equations that would be made true with the number 56.
Envision Math Common Core Grade 5 Answer Key Topic 8 Apply Understanding of Multiplication to Multiply Fractions 22
Answer:

Lesson 8.2 Multiply a Whole Number by a Fraction

Solve & Share
Brandon has 6 eggs. He needs \(\frac{2}{3}\) of the eggs to make an omelet. How many eggs does he need?

Model with Math Would a drawing help you picture the situation?
Envision Math Common Core Grade 5 Answer Key Topic 8 Apply Understanding of Multiplication to Multiply Fractions 22.1

Look Back! Should your answer be less than or greater than 6? How do you know?

Visual Learning Bridge

Essential Question How Can You Multiply a Whole Number by a Fraction?

A.
Claudia has 8 yards of fabric. She needs \(\frac{3}{4}\) of the fabric to make a banner. How many yards of fabric does she need?
Envision Math Common Core Grade 5 Answer Key Topic 8 Apply Understanding of Multiplication to Multiply Fractions 22.3
You can use models to represent the problem.
Envision Math Common Core Grade 5 Answer Key Topic 8 Apply Understanding of Multiplication to Multiply Fractions 22.4
You need to find \(\frac{3}{4}\) of 8.

B.
Step 1
Since you are finding \(\frac{3}{4}\) of 8, divide the model into 4 equal parts.
Envision Math Common Core Grade 5 Answer Key Topic 8 Apply Understanding of Multiplication to Multiply Fractions 22.5

C.
Step 2
Since you are finding \(\frac{3}{4}\) of 8, take 3 of those parts to make 6.
Envision Math Common Core Grade 5 Answer Key Topic 8 Apply Understanding of Multiplication to Multiply Fractions 22.6
Claudia needs 6 yards of fabric to make a banner.

Convince Me! Model with Math Here is how Lydia found the product \(\frac{4}{5}\) × 10.
Envision Math Common Core Grade 5 Answer Key Topic 8 Apply Understanding of Multiplication to Multiply Fractions 22.7
Use the model at the right to show that Lydia’s answer is correct.

Another example
Find \(\frac{3}{4}\) × 2.
Divide 2 into 4 equal parts.
Envision Math Common Core Grade 5 Answer Key Topic 8 Apply Understanding of Multiplication to Multiply Fractions 22.8
Each part is \(\frac{1}{2}\). So 3 parts make \(\frac{3}{2}\).
So, \(\frac{3}{4}\) × 2 = \(\frac{3}{2}\)

Think three-fourths of 2 wholes.
Envision Math Common Core Grade 5 Answer Key Topic 8 Apply Understanding of Multiplication to Multiply Fractions 22.9

Guided Practice

Do You Understand?

Question 1.
Explain why the product of 4 × \(\frac{2}{3}\) the same as the product of \(\frac{2}{3}\) × 4.
Answer:

Question 2.
In the problem at the top of page 338, what multiplication equation could be used to find how many yards of fabric Claudia did not use?
Answer:

Do You Know How?

In 3 and 4, use the model to find each product.

Question 3.
\(\frac{2}{3}\) × 6
Envision Math Common Core Grade 5 Answer Key Topic 8 Apply Understanding of Multiplication to Multiply Fractions 25.1
Answer:

Question 4.
\(\frac{3}{8}\) × 4
Envision Math Common Core Grade 5 Answer Key Topic 8 Apply Understanding of Multiplication to Multiply Fractions 25.2
Answer:

Independent Practice

In 5-7, find each product. Draw models to help.

Question 5.
\(\frac{2}{3}\) × 15
Answer:

Question 6.
\(\frac{11}{12}\) × 6
Answer:

Question 7.
\(\frac{5}{8}\) × 16
Answer:

Problem Solving

Question 8.
Construct Arguments Janice said that when you multiply a fraction less than 1 by a nonzero whole number, the product is always less than the whole number. Do you agree? Explain.
Answer:

Question 9.
enVision STEM A scientist wants to find out how the properties of water change when salt is added to it. For every cup of water she has, she replaces of it with salt. If she has 24 cups of water, how many cups will she replace with salt?
Answer:

Question 10.
Shanna attends school for 1 week longer than \(\frac{3}{4}\) of the year. How many weeks in a year does Shanna attend school?
Answer:

There are 52 weeks in a year.
Envision Math Common Core Grade 5 Answer Key Topic 8 Apply Understanding of Multiplication to Multiply Fractions 26.1

Question 11.
Higher Order Thinking Gina has 48 stickers. \(\frac{3}{8}\) of the stickers have pictures of flowers. \(\frac{1}{8}\) of the stickers have pictures of plants. The rest of the stickers have pictures of people. How many stickers have pictures of people? Explain how you found your answer.
Answer:

Question 12.
Two paperback books cost a total of $10. How much change will Stacy get if she buys two hardcover books and two paperback books and gives the clerk three $20 bills?
Envision Math Common Core Grade 5 Answer Key Topic 8 Apply Understanding of Multiplication to Multiply Fractions 26.2
Answer:

Assessment Practice

Question 13.
Select each Expression that has a product of 12.
Envision Math Common Core Grade 5 Answer Key Topic 8 Apply Understanding of Multiplication to Multiply Fractions 26.3
Answer:

Question 14.
Select each equation that would be made true with the number 4.
Envision Math Common Core Grade 5 Answer Key Topic 8 Apply Understanding of Multiplication to Multiply Fractions 26.4
Answer:

Lesson 8.3 Multiply Fractions and Whole Numbers

Activity

Solve & Share
Julie has 10 yards of ribbon. She divides the ribbon into 3 equal pieces and uses 2 of the pieces on gifts. How much ribbon does she use? Solve this problem any way you choose.
Envision Math Common Core 5th Grade Answers Topic 8 Apply Understanding of Multiplication to Multiply Fractions 26.5

Model with Math You can use words, pictures, and equations to solve the problem. Show your work in the space above!
Envision Math Common Core 5th Grade Answers Topic 8 Apply Understanding of Multiplication to Multiply Fractions 26.6

Look Back! Should the answer be less than or greater than 5? How do you know?

Visual Learning Bridge

Essential Question How Can You Multiply Fractions question and Whole Numbers?

A.
Hal spent \(\frac{3}{4}\) hour reading each day for 7 days. How much total time did he spend reading?
Envision Math Common Core 5th Grade Answers Topic 8 Apply Understanding of Multiplication to Multiply Fractions 26.7

B.
Multiply to find the number of fourths.
Envision Math Common Core 5th Grade Answers Topic 8 Apply Understanding of Multiplication to Multiply Fractions 26.8
7 × \(\frac{3}{4}\) = 7 × 3 × \(\frac{1}{4}\)
= 21 × \(\frac{1}{4}\)
= \(\frac{21}{4}\)
Rewrite as a mixed number
\(\frac{21}{4}\) = 5\(\frac{1}{4}\)
Hal spent 5\(\frac{1}{4}\) hours reading.

To rename \(\frac{21}{4}\), divide the numerator by the denominator.
Envision Math Common Core 5th Grade Answers Topic 8 Apply Understanding of Multiplication to Multiply Fractions 27.1

Convince Me! Be Precise What are the products \(\frac{4}{9}\) × 6 and 6 × \(\frac{4}{9}\)?

Guided Practice

Do You Understand?

Question 1.
What is \(\frac{3}{4}\) of a ribbon that is 7 feet long?
Answer:

Question 2.
Explain how \(\frac{3}{4}\) × 7, 7, and 3 × \(\frac{7}{4}\) are all related.
Answer:

Do You Know How?

In 3-5, find each product. Write the product as a mixed number.

Question 3.
Envision Math Common Core 5th Grade Answers Topic 8 Apply Understanding of Multiplication to Multiply Fractions 28.1
Answer:

Question 4.
Envision Math Common Core 5th Grade Answers Topic 8 Apply Understanding of Multiplication to Multiply Fractions 28.2
Answer:

Question 5.
Envision Math Common Core 5th Grade Answers Topic 8 Apply Understanding of Multiplication to Multiply Fractions 28.3
Answer:

Independent Practice

Leveled Practice In 6-16, find each product. Write the product as a mixed number.

Remember: You can use division to rename a fraction as a mixed number.
Envision Math Common Core 5th Grade Answers Topic 8 Apply Understanding of Multiplication to Multiply Fractions 28.4

Question 6.
Envision Math Common Core 5th Grade Answers Topic 8 Apply Understanding of Multiplication to Multiply Fractions 28.5
Answer:

Question 7.
Envision Math Common Core 5th Grade Answers Topic 8 Apply Understanding of Multiplication to Multiply Fractions 28.6
Answer:

Question 8.
Envision Math Common Core 5th Grade Answers Topic 8 Apply Understanding of Multiplication to Multiply Fractions 28.7
Answer:

Question 9.
\(\frac{4}{5}\) × 500
Answer:

Question 10.
5 × \(\frac{2}{3}\)
Answer:

Question 11.
17 × \(\frac{6}{8}\)
Answer:

Question 12.
\(\frac{9}{10}\) × 25
Answer:

Question 13.
\(\frac{7}{8}\) × 320
Answer:

Question 14.
28 × \(\frac{7}{12}\)
Answer:

Question 15.
\(\frac{2}{3}\) × 1,287
Answer:

Question 16.
900 × \(\frac{2}{9}\)
Answer:

Problem Solving

Question 17.
About 0.6 of the human body is made up of water. If a person has a mass of 75 kilograms, what is the mass of the water in this person’s body?
Answer:

Question 18.
Number Sense How can you use mental math to find 25 × \(\frac{3}{10}\)?
Answer:

Question 19.
During a nature walk, Jill identified 20 species of animals and plants.
a Construct Arguments Jill said that of the species she identified were animals. Can this be correct? Explain.
b If \(\frac{3}{5}\) of the species Jill identified were animals, how many plants did Jill identify?
Answer:

Question 20.
A rectangular painting is 2 feet long and \(\frac{5}{6}\) a foot wide. What is the area of the painting?
Envision Math Common Core 5th Grade Answers Topic 8 Apply Understanding of Multiplication to Multiply Fractions 30.1
Answer:

Question 21.
Higher Order Thinking An art teacher makes a batch of purple paint by mixing \(\frac{3}{4}\) cup red paint with \(\frac{3}{4}\) cup blue paint. If she mixes 13 batches, how many cups of purple paint will she have?
Answer:

Question 22.
enVision STEM A water molecule is made up of 3 atoms. One third of the atoms are oxygen and the remaining atoms are hydrogen. If there are 114 water molecules, how many hydrogen atoms are there? Show your work.
Answer:

Question 23.
Select all that are true.
Envision Math Common Core 5th Grade Answers Topic 8 Apply Understanding of Multiplication to Multiply Fractions 30.6
Answer:

Assessment Practice

Question 24.
Select all that are true.
Envision Math Common Core 5th Grade Answers Topic 8 Apply Understanding of Multiplication to Multiply Fractions 30.2
Answer:

Lesson 8.4 Use Models to Multiply Two Fractions

Activity

Solve & Share
The art teacher gave each student half of a sheet of paper. Then she asked the students to color one fourth of their pieces of paper. What part of the original sheet did the students color? Solve this problem any way you choose.

You can draw a picture to represent the problem.
Envision Math Common Core 5th Grade Answers Topic 8 Apply Understanding of Multiplication to Multiply Fractions 30.20

Look Back! Reasoning Should your answer be less than or greater than 1? How do you know?

Visual Learning Bridge

Essential Question How Can You Use a Model to Multiply Fractions?

A.
There was \(\frac{1}{4}\) of a pan of lasagna left. Tom ate of this amount. What fraction of a whole pan of lasagna did he eat?
Envision Math Common Core 5th Grade Answers Topic 8 Apply Understanding of Multiplication to Multiply Fractions 30.3

Find \(\frac{1}{3}\) of \(\frac{1}{4}\) to solve the problem.
Envision Math Common Core 5th Grade Answers Topic 8 Apply Understanding of Multiplication to Multiply Fractions 30.4

B.
One Way
Divide one whole into fourths.
Divide \(\frac{1}{4}\) into 3 equal parts.
Divide the other \(\frac{1}{4}\)s into 3 equal parts.
Envision Math Common Core 5th Grade Answers Topic 8 Apply Understanding of Multiplication to Multiply Fractions 30.5
12 parts make one whole, so one part is \(\frac{1}{12}\).
\(\frac{1}{3}\) × \(\frac{1}{4}\) = \(\frac{1}{12}\)

C.
Another Way
Shade 1 of the 3 rows yellow to represent \(\frac{1}{3}\).
Shade 1 of the 4 columns red to represent \(\frac{1}{4}\).
The orange overlap shows the product.
Envision Math Common Core 5th Grade Answers Topic 8 Apply Understanding of Multiplication to Multiply Fractions 30.60
1 out of 12 parts are shaded orange.
\(\frac{1}{3}\) × \(\frac{1}{4}\) = \(\frac{1×1}{3×4}\)
Tom ate \(\frac{1}{12}\) of the pan of lasagna.

Convince Me! Reasoning Find \(\frac{1}{4}\) × \(\frac{1}{5}\) using the model. Explain your work.
Envision Math Common Core 5th Grade Answers Topic 8 Apply Understanding of Multiplication to Multiply Fractions 30.7

Another example

Find \(\frac{2}{3}\) × \(\frac{3}{4}\) using a number line.
\(\frac{1}{3}\) means 1 of 3 equal parts, so \(\frac{1}{3}\) of \(\frac{3}{4}\) is \(\frac{1}{4}\)
\(\frac{2}{3}\) means 2 of 3 equal parts, so \(\frac{2}{3}\) of \(\frac{3}{4}\) is 2 times \(\frac{1}{4}\).
\(\frac{2}{3}\) × \(\frac{3}{4}\) = \(\frac{2}{4}\) or \(\frac{1}{2}\)
Envision Math Common Core 5th Grade Answers Topic 8 Apply Understanding of Multiplication to Multiply Fractions 102

Guided Practice

Do You Understand?

Question 1.
Use the model in box Con page 346 to find \(\frac{2}{3}\) × \(\frac{2}{4}\).
Answer:

Question 2.
Create a story problem for \(\frac{2}{3}\) × \(\frac{2}{4}\).
Answer:

Do You Know How?

Question 3.
Find \(\frac{5}{6}\) × \(\frac{1}{2}\). Shade the model to help solve.
Envision Math Common Core 5th Grade Answers Topic 8 Apply Understanding of Multiplication to Multiply Fractions 32.1
Answer:

Question 4.
Find \(\frac{3}{4}\) of \(\frac{4}{9}\).
Answer:

Independent Practice

In 5-6, find each product. Shade the model to help solve.

Question 5.
\(\frac{1}{3}\) × \(\frac{5}{6}\)
Envision Math Common Core 5th Grade Answers Topic 8 Apply Understanding of Multiplication to Multiply Fractions 32.2
Answer:

Question 6.
\(\frac{2}{3}\) × \(\frac{1}{12}\)
Envision Math Common Core 5th Grade Answers Topic 8 Apply Understanding of Multiplication to Multiply Fractions 32.3
Answer:

In 7-14, find each product. Use models to help.

Question 7.
\(\frac{7}{8}\) × \(\frac{1}{12}\)
Answer:

Question 8.
\(\frac{2}{5}\) × \(\frac{1}{12}\)
Answer:

Question 9.
\(\frac{5}{7}\) × \(\frac{7}{9}\)
Answer:

Question 10.
\(\frac{1}{2}\) × \(\frac{3}{4}\)
Answer:

Question 11.
\(\frac{1}{4}\) × \(\frac{7}{8}\)
Answer:

Question 12.
\(\frac{5}{6}\) of \(\frac{9}{10}\)
Answer:

Question 13.
\(\frac{1}{4}\) × \(\frac{1}{8}\)
Answer:

Question 14.
\(\frac{1}{3}\) of \(\frac{3}{7}\)
Answer:

Problem Solving

Question 15.
Make Sense and Persevere Will $50 be enough to buy 6 cans of paint? Explain.
Envision Math Common Core 5th Grade Answers Topic 8 Apply Understanding of Multiplication to Multiply Fractions 33.1
Answer:

Question 16.
A scientist had \(\frac{3}{4}\) of a bottle of a solution. She used \(\frac{1}{6}\) of the solution in an Experiment. How much of the bottle did she use?
Answer:

Question 17.
Algebra What value of n makes the equation \(\frac{2}{3}\) × n = \(\frac{4}{9}\) true?
Answer:

Question 18.
Write an Expression that shows 104.
Answer:

Question 19.
A plumber charges $45 for the first hour and $30 for each additional hour. How much does he charge if it takes him 4 hours to make a repair?
Answer:

Question 20.
Higher Order Thinking If \(\frac{7}{8}\) is multiplied by \(\frac{4}{5}\) will the product be greater than either of the two factors? Explain.
Answer:

Question 21.
In the voting for City Council Precinct 5, only \(\frac{1}{2}\) of all eligible voters cast votes. What fraction of all eligible voters voted for Shelley? Morgan? Who received more votes?
Envision Math Common Core 5th Grade Answers Topic 8 Apply Understanding of Multiplication to Multiply Fractions 33.6
Answer:

Assessment Practice

Question 22.
Majid made the model to show multiplying a fraction by a fraction. Which multiplication equation does the model show?
Envision Math Common Core Grade 5 Answers Topic 8 Apply Understanding of Multiplication to Multiply Fractions 33.7
Answer:

Lesson 8.5 Multiply Two Fractions

Activity

Solve & Share
On Dan’s eReader, \(\frac{2}{3}\) of the books are fiction. Of the fiction books, \(\frac{4}{5}\) are mysteries. What fraction of the books on Dan’s eReader are mysteries? Solve this problem any way you choose.

You can model with math by writing a multiplication sentence to solve the problem.
Envision Math Common Core Grade 5 Answers Topic 8 Apply Understanding of Multiplication to Multiply Fractions 33.8

Look Back! What fraction of the books are not mysteries? Explain.

Visual Learning Bridge

Essential Question How Can You Find the Product Question of Two Fractions?

A.
Amelia takes pictures with her smartphone. Of the pictures, \(\frac{5}{6}\) are of animals. What fraction of all her pictures are of dogs?
Envision Math Common Core Grade 5 Answers Topic 8 Apply Understanding of Multiplication to Multiply Fractions 33.9

You need to find \(\frac{3}{4}\) of \(\frac{5}{6}\) to answer the question.
Envision Math Common Core Grade 5 Answers Topic 8 Apply Understanding of Multiplication to Multiply Fractions 33.10

B.
Step 1
Estimate \(\frac{3}{4}\) × \(\frac{5}{6}\).
Since both fractions are less than 1, the product will be less than 1.

C.
Step 2
Multiply the numerators. Then multiply the denominators.
\(\frac{3}{4}\) × \(\frac{5}{6}\) = \(\frac{3 \times 5}{4 \times 6}\)
Since \(\frac{15}{24}\) < 1, the answer is reasonable.
So, \(\frac{15}{24}\) or \(\frac{5}{8}\) of all Amelia’s pictures have dogs in them.

\(\frac{15}{24}\) and \(\frac{5}{8}\) are equivalent fractions.
Envision Math Common Core Grade 5 Answers Topic 8 Apply Understanding of Multiplication to Multiply Fractions 33.11

Convince Me! Model with Math \(\frac{1}{10}\) of the animal pictures on Amelia’s smartphone are of cats. Write and solve an equation to find what fraction of all her pictures have cats in them.

Guided Practice

Do You Understand?

Question 1.
Is the product \(\frac{3}{6}\) × \(\frac{5}{4}\) of equal to the product of \(\frac{3}{4}\) × \(\frac{5}{6}\)? Explain how you know.
Answer:

Question 2.
How is adding \(\frac{3}{9}\) and \(\frac{6}{9}\) different from multiplying the two fractions? Explain.
Answer:

Do You Know How?
In 3-10, find each product.

Question 3.
\(\frac{2}{3}\) × \(\frac{1}{2}\)
Answer:

Question 4.
\(\frac{5}{4}\) of \(\frac{1}{9}\)
Answer:

Question 5.
\(\frac{7}{10}\) × \(\frac{3}{4}\)
Answer:

Question 6.
\(\frac{1}{3}\) × \(\frac{1}{4}\)
Answer:

Question 7.
\(\frac{5}{6}\) of \(\frac{3}{7}\)
Answer:

Question 8.
\(\frac{3}{5}\) × \(\frac{11}{12}\)
Answer:

Question 9.
\(\frac{4}{10}\) × \(\frac{2}{5}\)
Answer:

Question 10.
\(\frac{3}{4}\) × \(\frac{2}{9}\)
Answer:

Independent Practice

In 11-30, find each product.

Question 11.
\(\frac{9}{10}\) × \(\frac{1}{2}\)
Answer:

Question 12.
\(\frac{5}{6}\) × \(\frac{1}{3}\)
Answer:

Question 13.
\(\frac{4}{7}\) of \(\frac{7}{9}\)
Answer:

Question 14.
\(\frac{3}{4}\) × \(\frac{4}{5}\)
Answer:

Question 15.
\(\frac{2}{3}\) × \(\frac{7}{8}\)
Answer:

Question 16.
\(\frac{6}{7}\) × \(\frac{3}{8}\)
Answer:

Question 17.
\(\frac{1}{3}\) of \(\frac{3}{4}\)
Answer:

Question 18.
\(\frac{6}{7}\) × \(\frac{3}{8}\)
Answer:

Question 19.
\(\frac{2}{5}\) of \(\frac{5}{12}\)
Answer:

Question 20.
\(\frac{2}{3}\) × \(\frac{4}{5}\)
Answer:

Question 21.
\(\frac{1}{2}\) × \(\frac{1}{2}\)
Answer:

Question 22.
\(\frac{1}{2}\) of \(\frac{8}{9}\)
Answer:

Question 23.
\(\frac{2}{3}\) × (\(\frac{3}{5}\) + \(\frac{1}{5}\))
Answer:

Question 24.
(\(\frac{3}{7}\) + \(\frac{2}{7}\)) × \(\frac{2}{3}\)
Answer:

Question 25.
\(\frac{1}{2}\) × (\(\frac{1}{3}\) + \(\frac{1}{3}\))
Answer:

Question 26.
(\(\frac{9}{10}\) – \(\frac{3}{10}\)) × \(\frac{1}{4}\)
Answer:

Question 27.
\(\frac{2}{3}\) × (\(\frac{3}{5}\) + \(\frac{1}{5}\))
Answer:

Question 28.
(\(\frac{8}{9}\) – \(\frac{1}{3}\)) × \(\frac{3}{4}\)
Answer:

Question 29.
(\(\frac{5}{12}\) + \(\frac{1}{6}\)) × \(\frac{5}{6}\)
Answer:

Question 30.
\(\frac{11}{12}\) × (\(\frac{3}{4}\) – \(\frac{1}{2}\))
Answer:

Problem Solving

Question 31.
Eduardo runs 6 laps around the track at Lincoln Park School. Then he runs 3\(\frac{1}{2}\) miles to get home. How far will he run in all? Show your work.
Envision Math Common Core Grade 5 Answers Topic 8 Apply Understanding of Multiplication to Multiply Fractions 40.1
Answer:

Question 32.
Be Precise To amend the U.S. Constitution, \(\frac{3}{4}\) of the 50 states must approve the amendment. If 35 states approve an amendment, will the Constitution be amended?
Answer:

Question 33.
Higher Order Thinking In Ms. Barclay’s classroom, \(\frac{2}{5}\) of the students play chess. Of the students who play chess, \(\frac{5}{6}\) also play sudoku. If there are 30 students in her class, how many play chess and sudoku?
Answer:

Question 34.
One sheet of stamps is shown at the right. Emma needs to buy 50 stamps to send out invitations for her graduation party. Will 2 sheets of stamps be enough? How do you know?
Envision Math Common Core Grade 5 Answers Topic 8 Apply Understanding of Multiplication to Multiply Fractions 40.3
Answer:

Assessment Practice

Question 35.
Choose all the Expressions that have \(\frac{3}{4}\) as a product.
Envision Math Common Core Grade 5 Answers Topic 8 Apply Understanding of Multiplication to Multiply Fractions 40.4
Answer:

Question 36.
Choose all the multiplication sentences that have \(\frac{1}{4}\) as the missing part.
Envision Math Common Core Grade 5 Answers Topic 8 Apply Understanding of Multiplication to Multiply Fractions 40.5
Answer:

Lesson 8.6 Area of a Rectangle

Activity

Solve&Share
A rectangular poster is \(\frac{1}{4}\) yard wide and \(\frac{3}{4}\) yard tall. What is its area? Solve this problem any way you choose.

You can use appropriate tools, like grid paper, to solve the problem.
Envision Math Common Core Grade 5 Answers Topic 8 Apply Understanding of Multiplication to Multiply Fractions 40.6

Look Back! Is the area of a poster that is \(\frac{3}{4}\) yard wide and \(\frac{1}{4}\) yard tall the same as the area of the poster above? Explain.

Visual Learning Bridge

Essential Question How Can You Find the Area of a Question Rectangle with Fractional Side Lengths?

A.
Jenny has a rectangular garden. What is the area of her garden?
Envision Math Common Core Grade 5 Answers Topic 8 Apply Understanding of Multiplication to Multiply Fractions 40.7
The area of a rectangle is found by multiplying the length by the width.
Envision Math Common Core Grade 5 Answers Topic 8 Apply Understanding of Multiplication to Multiply Fractions 40.8

B.
Step 1
\(\frac{1}{4}\) × \(\frac{1}{3}\) = \(\frac{1}{12}\) because 12 rectangles each \(\frac{1}{4}\) wide and \(\frac{1}{3}\) high fit in a unit square.
Envision Math Common Core Grade 5 Answers Topic 8 Apply Understanding of Multiplication to Multiply Fractions 40.9

C.
Step 2
A rectangle of width \(\frac{5}{4}\) yards and height \(\frac{2}{3}\) yard is tiled with 5 × 2 rectangles of area \(\frac{1}{12}\).
Envision Math Common Core Grade 5 Answers Topic 8 Apply Understanding of Multiplication to Multiply Fractions 40.10
so, \(\frac{5}{4}\) × \(\frac{2}{3}\) = \(\frac{5 \times 2}{4 \times 3}\) = \(\frac{10}{12}\)
The area of Jenny’s garden is \(\frac{10}{12}\) square yard.

Convince Me! Reasoning Mason has a rectangular garden that is \(\frac{2}{3}\) yard wide by \(\frac{7}{4}\) yards long. What is the area of Mason’s garden? Use a drawing to show your work.
Envision Math Common Core Grade 5 Answers Topic 8 Apply Understanding of Multiplication to Multiply Fractions 40.11

Guided Practice

Do You Understand?

Question 1.
If you do not remember the formula for finding the area of a rectangle, how can you find its area?
Answer:

Question 2.
How could you define area?
Answer:

Do You Know How?

Question 3.
Find the area of a rectangle with side lengths \(\frac{2}{3}\) foot and \(\frac{1}{2}\) foot.
Answer:

Question 4.
Find the area of a square with side lengths of \(\frac{5}{4}\) inches.
Answer:

Independent Practice

In 5-10, find each area.

Question 5.
Envision Math Common Core Grade 5 Answers Topic 8 Apply Understanding of Multiplication to Multiply Fractions 41.1
Answer:

Question 6.
Envision Math Common Core Grade 5 Answers Topic 8 Apply Understanding of Multiplication to Multiply Fractions 41.2
Answer:

Question 7.
Envision Math Common Core Grade 5 Answers Topic 8 Apply Understanding of Multiplication to Multiply Fractions 41.3
Answer:

Question 8.
Envision Math Common Core Grade 5 Answers Topic 8 Apply Understanding of Multiplication to Multiply Fractions 41.4
Answer:

Question 9.
Envision Math Common Core Grade 5 Answers Topic 8 Apply Understanding of Multiplication to Multiply Fractions 41.5
Answer:

Question 10.
Envision Math Common Core Grade 5 Answers Topic 8 Apply Understanding of Multiplication to Multiply Fractions 41.6
Answer:

Question 11.
Find the area of a rectangle with side lengths \(\frac{5}{3}\) feet and \(\frac{3}{4}\) foot.
Answer:

Question 12.
Find the area of a square with side lengths of \(\frac{3}{8}\) inch.
Answer:

Question 13.
Find the area of a rectangle with side lengths \(\frac{7}{2}\) centimeters and \(\frac{5}{4}\) centimeters.
Answer:

Problem Solving

Question 14.
Construct Arguments
Roy and Tom are working on a multiplication problem. Roy claims that \(\frac{7}{4}\) × \(\frac{3}{8}\) = \(\frac{21}{32}\). Tom claims that the correct answer is \(\frac{21}{8}\). Who is correct? Explain your
Answer:

Question 15.
Emilio needs to know how much area to clear for his son’s square sandbox. Each side of the sandbox is \(\frac{3}{4}\) yard. Find the area that the sandbox will cover.
Answer:

Question 16.
Margaret purchased a doormat measuring \(\frac{1}{2}\) yard by \(\frac{2}{3}\) yard for her back door step. If the step is square yard, will the mat fit? Explain.
Answer:

Question 17.
Each person on a Ferris wheel pays $6.50 for a ticket. There are 72 passengers. How much money is collected from all the passengers?
Answer:

Question 18.
Higher Order Thinking Kim is installing blue and white tile in her bathroom. She made a diagram of the layout showing the area of both colors. Write two Expressions that describe the area of the blue tile.
Envision Math Common Core Grade 5 Answers Topic 8 Apply Understanding of Multiplication to Multiply Fractions 42.1
Answer:

Question 19.
Wilhelmina has 8.3 ounces of peanut butter. If she makes 5 sandwiches with an equal amount of peanut butter on each, how much peanut butter does she put on each one?
Answer:

Question 20.
Irene buys a talking doll for $10.66 and some batteries for $4.22. She pays with a $20 bill. Estimate how much change she should get, to the nearest dime.
Answer:

Assessment Practice

Question 21.
Juno calculated the area of a square to be \(\frac{4}{9}\) square yard. Which shows the side length of the square?
A. \(\frac{2}{9}\) yard
B. \(\frac{4}{9}\) yard
C. \(\frac{2}{3}\) yard
D. \(\frac{8}{9}\) yard
Answer:

Question 22.
Bo calculated the area of a square to be \(\frac{25}{4}\) square inches. Which shows the side length of the square?
A. \(\frac{25}{2}\) inches
B. \(\frac{25}{8}\) inches
C. \(\frac{5}{2}\) inches
D. \(\frac{5}{4}\) inches
Answer:

Lesson 8.7 Multiply mixed Numbers

Activity

Solve & Share
Look at the ingredients needed to make Josie’s special pancakes. How much pancake mix and milk will you need if you want to double the recipe? To triple the recipe? Solve this problem any way you choose.

Generalize How can you use what you know about multiplying fractions to help you multiply mixed numbers?
Envision Math Common Core Grade 5 Answers Topic 8 Apply Understanding of Multiplication to Multiply Fractions 43.1

Look Back! What number sentence can you write using repeated addition to show how much pancake mix is needed if the recipe is tripled?

Visual Learning Bridge

Essential Question How Can You Find the Product of mixed Numbers?

A.
A clothing factory has machines that make jackets. The machines operate for 7\(\frac{1}{2}\) hours each day. How many jackets can Machine A make in one day?
Envision Math Common Core Grade 5 Answers Topic 8 Apply Understanding of Multiplication to Multiply Fractions 43.2

B.
One Way
You can use an area model to find the partial products. Then add to find the final product.
Envision Math Common Core Grade 5 Answers Topic 8 Apply Understanding of Multiplication to Multiply Fractions 43.3

C.
Another Way
You can also use an equation to find the product. Rename the mixed numbers, then multiply.
7\(\frac{1}{2}\) × 2\(\frac{3}{4}\) = \(\frac{15}{2}\) × \(\frac{11}{4}\)
= \(\frac{165}{8}\)
= 20\(\frac{5}{8}\)

Machine A makes 20 jackets each day.

Since 20 is close to the estimate of 24, the answer is reasonable.
Envision Math Common Core Grade 5 Answers Topic 8 Apply Understanding of Multiplication to Multiply Fractions 43.5

Convince Me! Model with Math How many jackets can Machine B make in one day? Write an equation to model your work.

Guided Practice

Do You Understand?

Question 1.
Explain how you would multiply 5 × 2\(\frac{1}{2}\)
Answer:

Do You Know How?

In 2 and 3, estimate the product. Then complete the multiplication.

Question 2.
Envision Math Common Core Grade 5 Answers Topic 8 Apply Understanding of Multiplication to Multiply Fractions 43.6
Answer:

Question 3.
Envision Math Common Core Grade 5 Answers Topic 8 Apply Understanding of Multiplication to Multiply Fractions 43.7
Answer:

Independent Practice

Compare your product against your estimate to check for reasonableness.
Envision Math Common Core Grade 5 Answers Topic 8 Apply Understanding of Multiplication to Multiply Fractions 43.8

In 4-9, estimate the product. Then complete the multiplication.

Question 4.
Envision Math Common Core Grade 5 Answers Topic 8 Apply Understanding of Multiplication to Multiply Fractions 43.9
Answer:

Question 5.
Envision Math Common Core Grade 5 Answers Topic 8 Apply Understanding of Multiplication to Multiply Fractions 43.10
Answer:

Question 6.
Envision Math Common Core Grade 5 Answers Topic 8 Apply Understanding of Multiplication to Multiply Fractions 43.11
Answer:

Question 7.
Envision Math Common Core Grade 5 Answers Topic 8 Apply Understanding of Multiplication to Multiply Fractions 43.12
Answer:

Question 8.
Envision Math Common Core Grade 5 Answers Topic 8 Apply Understanding of Multiplication to Multiply Fractions 43.13
Answer:

Question 9.
Envision Math Common Core Grade 5 Answers Topic 8 Apply Understanding of Multiplication to Multiply Fractions 43.14
Answer:

In 10-20, estimate the product. Then find each product.

Question 10.
2\(\frac{1}{2}\) × 4\(\frac{1}{2}\)
Answer:

Question 11.
\(\frac{3}{4}\) × 8\(\frac{1}{2}\)
Answer:

Question 12.
1\(\frac{1}{8}\) × 3\(\frac{1}{3}\)
Answer:

Question 13.
3\(\frac{1}{5}\) × \(\frac{2}{3}\)
Answer:

Question 14.
3\(\frac{1}{4}\) × 6
Answer:

Question 15.
5\(\frac{1}{3}\) × 3
Answer:

Question 16.
2\(\frac{3}{8}\) × 4
Answer:

Question 17.
4\(\frac{1}{8}\) × 5\(\frac{1}{2}\)
Answer:

Question 18.
Envision Math Common Core Grade 5 Answers Topic 8 Apply Understanding of Multiplication to Multiply Fractions 50.1
Answer:

Question 19.
Envision Math Common Core Grade 5 Answers Topic 8 Apply Understanding of Multiplication to Multiply Fractions 50.2
Answer:

Question 20.
Envision Math Common Core Grade 5 Answers Topic 8 Apply Understanding of Multiplication to Multiply Fractions 50.3
Answer:

Problem Solving

In 21-23, use the diagram at the right.

Question 21.
Make Sense and Persevere Bernie and Chloe hiked the Tremont Trail to the end and back. Then they hiked the Wildflower Trail to the end before stopping to eat lunch. How far did they hike before they ate lunch?
Envision Math Common Core 5th Grade Answer Key Topic 8 Apply Understanding of Multiplication to Multiply Fractions 50.5
Answer:

Question 22.
Higher Order Thinking In one day, Ricardo hiked 2\(\frac{1}{2}\) times as far as Bernie and Chloe hiked before they ate lunch. How far did he hike?
Answer:

Question 23.
The city plans to Extend the Wildflower Trail 2\(\frac{1}{2}\) times its current length in the next 5 years. How long will the Wildflower Trail be at the end of 5 years?
Answer:

Question 24.
How can you use multiplication to find 3\(\frac{3}{5}\) + 3\(\frac{3}{5}\) + 3\(\frac{3}{5}\) ?
Answer:

Question 25.
The world’s smallest gecko is \(\frac{3}{4}\) inch long. An adult male Western Banded Gecko is 7\(\frac{1}{3}\) times as long. How long is a Western Banded Gecko?
Answer:

Question 26.
The Akashi-Kaikyo Bridge in Japan is about 1\(\frac{5}{8}\) times as long as the Golden Gate Bridge in San Francisco. The Golden Gate Bridge is about 9,000 feet long. About how long is the Akashi-Kaikyo Bridge?
Answer:

Question 27.
Patty spent 3.5 times as much as Sandy on their shopping trip. If Sandy spent $20.50, how much did Patty spend?
Answer:

Assessment Practice

Question 28.
Choose all that are true.
Envision Math Common Core 5th Grade Answer Key Topic 8 Apply Understanding of Multiplication to Multiply Fractions 50.8
Answer:

Question 29.
Choose all that are true.
Envision Math Common Core 5th Grade Answer Key Topic 8 Apply Understanding of Multiplication to Multiply Fractions 50.9
Answer:

Lesson 8.8 Multiplication as Scaling

Solve & Share
Without multiplying, circle the problem in each set with the greatest product and underline the problem with the least product. Solve this problem any way you choose.
Envision Math Common Core 5th Grade Answer Key Topic 8 Apply Understanding of Multiplication to Multiply Fractions 50.10

How can you use what you know about multiplying fractions to help you find the problem with the greatest product?
Envision Math Common Core 5th Grade Answer Key Topic 8 Apply Understanding of Multiplication to Multiply Fractions 51.1

Look Back! Construct Arguments How is \(\frac{3}{2}\) × 2 like 1 × 2?

Visual Learning Bridge

Essential Question How Can You Use Number Sense to Question Evaluate the Size of a Product?

A.
Sue knitted scarves that are 4 feet long for herself and her friends Joe and Alan. After a month, they compared the lengths of their scarves. Some scarves had stretched and some had shrunk. The results are shown in the chart. How had the lengths of Joe’s and Alan’s scarves changed?
Envision Math Common Core 5th Grade Answer Key Topic 8 Apply Understanding of Multiplication to Multiply Fractions 51.2

Think of multiplication as scaling or resizing.
Envision Math Common Core 5th Grade Answer Key Topic 8 Apply Understanding of Multiplication to Multiply Fractions 51.3

B.
Alan’s scarf
Alan’s scarf shrank.
\(\frac{3}{4}\) × 4 < 4 Multiplying a number by a fraction less than 1 results in a product less than the given number.
Envision Math Common Core 5th Grade Answer Key Topic 8 Apply Understanding of Multiplication to Multiply Fractions 51.4

C.
Joe’s scarf
Joe’s scarf stretched.
\(\frac{3}{4}\) × 4 > 4
Multiplying a number by a fraction greater than 1 results in a product greater than the starting number.
Envision Math Common Core 5th Grade Answer Key Topic 8 Apply Understanding of Multiplication to Multiply Fractions 51.5

Convince Me! Use Structure Sue knitted a scarf for her friend June that was also 4 feet long. After a month, the length of June’s scarf could be represented by the Expression \(\frac{3}{3}\) × 4. How did the length of June’s scarf change? Explain.

Guided Practice

Do You Understand?

Question 1.
Why does multiplying a number by 3\(\frac{1}{2}\) increase its value?
Answer:

Question 2.
Which of the following are less than 8?
8 × \(\frac{9}{10}\)
8 × \(\frac{7}{6}\)
\(\frac{3}{5}\) × 8
Answer:

Do You Know How?

In 3-5, without multiplying decide which symbol belongs in the box: <, >, or =

Question 3.
Envision Math Common Core 5th Grade Answer Key Topic 8 Apply Understanding of Multiplication to Multiply Fractions 51.6
Answer:

Question 4.
Envision Math Common Core 5th Grade Answer Key Topic 8 Apply Understanding of Multiplication to Multiply Fractions 51.7
Answer:

Question 5.
Envision Math Common Core 5th Grade Answer Key Topic 8 Apply Understanding of Multiplication to Multiply Fractions 51.8
Answer:

Independent Practice

In 6-17, without multiplying, decide which symbol belongs in the box: <, >, or =.

Question 6.
2\(\frac{1}{2}\) × 1\(\frac{2}{3}\) Envision Math Common Core 5th Grade Answer Key Topic 8 Apply Understanding of Multiplication to Multiply Fractions 56 1\(\frac{2}{3}\)
Answer:

Question 7.
\(\frac{3}{5}\) × 4\(\frac{4}{5}\) Envision Math Common Core 5th Grade Answer Key Topic 8 Apply Understanding of Multiplication to Multiply Fractions 56 4\(\frac{4}{5}\)
Answer:

Question 8.
1\(\frac{2}{7}\) Envision Math Common Core 5th Grade Answer Key Topic 8 Apply Understanding of Multiplication to Multiply Fractions 56 1\(\frac{2}{7}\) × \(\frac{5}{5}\)
Answer:

Question 9.
\(\frac{1}{3}\) × 2\(\frac{2}{5}\) Envision Math Common Core 5th Grade Answer Key Topic 8 Apply Understanding of Multiplication to Multiply Fractions 562\(\frac{2}{5}\)
Answer:

Question 10.
3\(\frac{3}{5}\) Envision Math Common Core 5th Grade Answer Key Topic 8 Apply Understanding of Multiplication to Multiply Fractions 56 3\(\frac{3}{5}\) × \(\frac{2}{2}\)
Answer:

Question 11.
4\(\frac{1}{3}\) × 2\(\frac{2}{7}\) Envision Math Common Core 5th Grade Answer Key Topic 8 Apply Understanding of Multiplication to Multiply Fractions 56 2\(\frac{2}{7}\)
Answer:

Question 12.
2\(\frac{1}{5}\) × \(\frac{1}{10}\) Envision Math Common Core 5th Grade Answer Key Topic 8 Apply Understanding of Multiplication to Multiply Fractions 56 2\(\frac{1}{5}\)
Answer:

Question 13.
\(\frac{1}{2}\) × 1\(\frac{2}{5}\) Envision Math Common Core 5th Grade Answer Key Topic 8 Apply Understanding of Multiplication to Multiply Fractions 56 1\(\frac{2}{5}\)
Answer:

Question 14.
4\(\frac{3}{4}\) × 3\(\frac{1}{4}\) Envision Math Common Core 5th Grade Answer Key Topic 8 Apply Understanding of Multiplication to Multiply Fractions 56 4\(\frac{3}{4}\)
Answer:

Question 15.
1\(\frac{3}{4}\) Envision Math Common Core 5th Grade Answer Key Topic 8 Apply Understanding of Multiplication to Multiply Fractions 56 1\(\frac{1}{12}\) × 1\(\frac{3}{4}\)
Answer:

Question 16.
5\(\frac{1}{3}\) × \(\frac{5}{6}\) Envision Math Common Core 5th Grade Answer Key Topic 8 Apply Understanding of Multiplication to Multiply Fractions 56 5\(\frac{1}{3}\)
Answer:

Question 17.
\(\frac{5}{5}\) × 4\(\frac{2}{3}\) Envision Math Common Core 5th Grade Answer Key Topic 8 Apply Understanding of Multiplication to Multiply Fractions 56 4\(\frac{2}{3}\)
Answer:

In 18 and 19, without multiplying, order the following products from least to greatest.

Question 18.
Envision Math Common Core 5th Grade Answer Key Topic 8 Apply Understanding of Multiplication to Multiply Fractions 52.1
Answer:

Question 19.
Envision Math Common Core 5th Grade Answer Key Topic 8 Apply Understanding of Multiplication to Multiply Fractions 52.2
Answer:

In 20 and 21, without multiplying, order the following products from greatest to least.

Question 20.
Envision Math Common Core 5th Grade Answer Key Topic 8 Apply Understanding of Multiplication to Multiply Fractions 52.3
Answer:

Question 21.
Envision Math Common Core 5th Grade Answer Key Topic 8 Apply Understanding of Multiplication to Multiply Fractions 52.4
Answer:

Problem Solving

Question 22.
Who ran farther by the end of the week? How much farther? Use the table below that shows the distances in miles.
Envision Math Common Core 5th Grade Answer Key Topic 8 Apply Understanding of Multiplication to Multiply Fractions 52.5
Answer:

Question 23.
Be Precise Ethan took a quiz with 15 questions. If he answered of the questions correctly, how many did he get wrong?
Answer:

Question 24.
At a taffy pull, George stretched the taffy to 3 feet. Jose stretched it 1 times as far as George. Maria stretched it as far as George. Sally stretched it, as far. Who stretched it the farthest? the least?
Answer:

Question 25.
Higher Order Thinking Without multiplying, decide which symbol belongs in the box: <, >, or =. Explain how you decided
4\(\frac{3}{4}\) × 3\(\frac{1}{4}\) Envision Math Common Core 5th Grade Answer Key Topic 8 Apply Understanding of Multiplication to Multiply Fractions 58 4\(\frac{1}{2}\)
Answer:

Question 26.
Write two decimals with a product close to 6.3.
__.__ × __ .__ ≈ 6.3
≈ is a symbol that means is approximately equal to.
Envision Math Common Core 5th Grade Answer Key Topic 8 Apply Understanding of Multiplication to Multiply Fractions 58.9

Assessment Practice

Question 27.
Write each Expression in the correct answer space to show products less than 4\(\frac{1}{2}\) and those greater than 4\(\frac{1}{2}\).
Envision Math Common Core 5th Grade Answer Key Topic 8 Apply Understanding of Multiplication to Multiply Fractions 56.12
Answer:

Question 28.
Write each Expression in the correct answer space to show products less than 1\(\frac{3}{4}\) and those greater than 1\(\frac{3}{4}\).
Envision Math Common Core 5th Grade Answer Key Topic 8 Apply Understanding of Multiplication to Multiply Fractions 52.13
Answer:

Lesson 8.9 Make Sense and Persevere

Problem Solving

Solve & Share
A rectangular dog park was built with the dimensions shown. The fencing that completely surrounds the park cost $12 a yard. Each square yard of grass sod that covers the entire park cost $8. What was the total cost for the fencing and the sod? Solve this problem any way you choose.
Envision Math Common Core 5th Grade Answer Key Topic 8 Apply Understanding of Multiplication to Multiply Fractions 58.3

Thinking Habits
Be a good thinker!
These questions can help you.
• What do I need to find?
• What do I know?
• What’s my plan for solving the problem?
• What else can I try if I get stuck?
• How can I check that my solution makes sense?
Envision Math Common Core 5th Grade Answer Key Topic 8 Apply Understanding of Multiplication to Multiply Fractions 53.2

Look Back! Make Sense and Persevere Before solving the problem, how do you know that the area of the dog park must be greater than 200 square yards?

Visual Learning Bridge

Essential Question How Can You Make Sense of Problems and Persevere in Solving Them?

A.
Gwen is planning to tile the entire floor of the family room and kitchen. Tile costs $12 per square foot. What is the total cost of tiling the family room and kitchen floors?
Envision Math Common Core 5th Grade Answer Key Topic 8 Apply Understanding of Multiplication to Multiply Fractions 53.4

You can make sense of the problem by answering these questions. What do you know? What are you asked to find?
Envision Math Common Core 5th Grade Answer Key Topic 8 Apply Understanding of Multiplication to Multiply Fractions 53.40

Here’s my thinking…
Envision Math Common Core 5th Grade Answer Key Topic 8 Apply Understanding of Multiplication to Multiply Fractions 53.5

B.
How can I make sense of and solve the problem?

I can
• identify the quantities given.
• understand how the quantities are related.
• choose and implement an appropriate strategy.
• check to be sure my work and answer make sense.

C.
Find the area of the family room.
A = 17\(\frac{1}{3}\) × 13\(\frac{1}{2}\) = \(\frac{52 \times 27}{3 \times 2}\) = \(\frac{1,404}{6}\) = 234
The area of the family room is 234 square feet.
Find the area of the kitchen.
A = 12 × 10\(\frac{3}{4}\) = \(\frac{12 \times 43}{1 \times 4}\) = \(\frac{516}{4}\) = 129
The area of the kitchen is 129 square feet.
Add to find the total area. 234 + 129 = 363
Calculate the total cost. 363 × 12 = 4,356
The total cost is $4,356.

Convince Me! Make Sense and Persevere How much more does it cost to tile the family room floor than the kitchen floor? Show your work.

Guided Practice

Make Sense and Persevere
A website has a daily trivia contest. On Mondays, Wednesdays, and Fridays, you have 1\(\frac{1}{2}\) hours to submit an answer. On Tuesdays and Thursdays, you have 1\(\frac{1}{4}\) hours. On Saturdays and Sundays, you have only \(\frac{3}{4}\) of an hour. How many hours each week do you have to submit an answer?

Remember to compare your answer to your estimate.
Envision Math Common Core 5th Grade Answer Key Topic 8 Apply Understanding of Multiplication to Multiply Fractions 55.1

Question 1.
Estimate the total hours each week you have to submit an answer. Write an equation to show your work.
Answer:

Question 2.
Write an equation using multiplication and a variable to represent the problem. Then solve the equation and answer the question.
Answer:

Independent Practice

Make Sense and Persevere
Isabel is buying framing to go around the perimeter of one of her paintings. Each inch of framing costs $0.40. What is the total cost of the framing for the painting?

Envision Math Common Core 5th Grade Answer Key Topic 8 Apply Understanding of Multiplication to Multiply Fractions 55.20

Question 3.
What is the first step you need to do? What is the answer to the first step? Write an equation to show your work.
Answer:

Question 4.
What is the next step to solve the problem? What is the answer to the problem? Write an equation to show your work.
Answer:

Question 5.
How can you check that your answer makes sense?
Answer:

Problem Solving

Performance Task
Hiking Trails
The Farina family spent a week at the state park. Christine hiked the Evergreen trail twice and the Yellow River trail once. Brian hiked each of the three longest trails once. How many more miles did Brian hike than Christine?
Envision Math Common Core 5th Grade Answer Key Topic 8 Apply Understanding of Multiplication to Multiply Fractions 55.4

Question 6.
Make Sense and Persevere What do you know? What are you asked to find? What information do you not need?
Answer:

Question 7.
Make Sense and Persevere What information do you need to find before you can answer the final question?
Answer:

Read the problem carefully so you can identify what you know and what you are asked to find.
Envision Math Common Core 5th Grade Answer Key Topic 8 Apply Understanding of Multiplication to Multiply Fractions 150

Question 8.
Model with Math Write equations to represent the information in Exercise 7.
Answer:

Question 9.
Make Sense and Persevere Solve the problem.
Answer:

Question 10.
Construct Arguments Explain why your answer makes sense.
Answer:

Topic 8 Fluency Practice

Activity

Point & Tally

Work with a partner. Get paper and a pencil. Each partner chooses light blue or dark blue.
At the same time, Partner 1 and Partner 2 each point to one of their black numbers. Both partners find the product of the two numbers.
The partner who chose the color where the product appears gets a tally mark. Work until one partner has seven tally marks.
Envision Math Common Core 5th Grade Answer Key Topic 8 Apply Understanding of Multiplication to Multiply Fractions 172

Envision Math Common Core 5th Grade Answer Key Topic 8 Apply Understanding of Multiplication to Multiply Fractions 57.1

Topic 8 Vocabulary Review

Glossary

Understand Vocabulary

Choose the best term from the box. Write it on the blank.

Word List
• area model
• Associative Property of Multiplication
• benchmark fractions
• Commutative Property of Multiplication
• mixed number
• round

Question 1.
To estimate the product of two mixed numbers, ____ each factor to the nearest whole number.
Answer:

Question 2.
Using ____ can help make it easier to estimate computations.
Answer:

Question 3.
The product of two fractions can be represented by a(n) ____
Answer:

Question 4.
Another way to write the fraction \(\frac{19}{5}\) is as a ____
Answer:

True or False

Estimate each product to decide if the comparison is true or false. Write T for true or F for false.

____ Question 5.
6\(\frac{3}{5}\) × 5\(\frac{7}{8}\) < 42
Answer:

____ Question 6.
8\(\frac{2}{9}\) × 9\(\frac{1}{4}\) > 90
Answer:

____ Question 7.
\(\frac{2}{7}\) × \(\frac{5}{8}\) < 1
Answer:

___ Question 8.
5\(\frac{1}{10}\) × 3 > 15
Answer:

Use Vocabulary in Writing

Question 9.
Suppose you know the answer to \(\frac{4}{5}\) × (20 × 1\(\frac{7}{8}\)). Explain how the Commutative and Associative Properties of Multiplication can make the computation easier. Then find the answer.
Answer:

Topic 8 Reteaching

Set A
pages 333-336
Find 4 × \(\frac{2}{3}\) using a number line.
Envision Math Common Core 5th Grade Answer Key Topic 8 Apply Understanding of Multiplication to Multiply Fractions 62.1
Each jump is \(\frac{2}{3}\).
1 × \(\frac{2}{3}\) = \(\frac{2}{3}\)
2 × \(\frac{2}{3}\) = \(\frac{4}{3}\) = 1\(\frac{1}{3}\)
3 × \(\frac{2}{3}\) = \(\frac{6}{3}\) = 2
4 × \(\frac{2}{3}\) = \(\frac{8}{3}\) = 2\(\frac{2}{3}\)
So, 4 × \(\frac{2}{3}\) = 2\(\frac{2}{3}\)
This makes sense because \(\frac{2}{3}\) is less than 1, so 4 × \(\frac{2}{3}\) should be less than 4.

Remember to multiply the numerator of the fraction by the whole number.

Find each product. Use number lines, fraction strips, or drawings to help you.

Question 1.
4 × \(\frac{3}{4}\)
Answer:

Question 2.
7 × \(\frac{1}{4}\)
Answer:

Question 3.
8 × \(\frac{5}{6}\)
Answer:

Question 4.
10 × \(\frac{1}{2}\)
Answer:

Question 5.
9 × \(\frac{1}{3}\)
Answer:

Question 6.
9 × \(\frac{2}{3}\)
Answer:

Question 7.
3 × \(\frac{7}{8}\)
Answer:

Question 8.
7 × \(\frac{3}{8}\)
Answer:

Question 9.
5 × \(\frac{5}{6}\)
Answer:

Question 10.
12 × \(\frac{2}{3}\)
Answer:

Question 11.
15 × \(\frac{4}{5}\)
Answer:

Question 12.
2 × \(\frac{9}{10}\)
Answer:

Set B
pages 337-340, 341-344
Mary’s clock uses \(\frac{3}{4}\) of the batteries in the package. How many batteries does the clock need?
Envision Math Common Core 5th Grade Answer Key Topic 8 Apply Understanding of Multiplication to Multiply Fractions 63.1
Find \(\frac{3}{4}\) of 8
\(\frac{1}{4}\) of 8 is 2.
\(\frac{3}{4}\) is three times as much as \(\frac{1}{4}\).
So, \(\frac{3}{4}\) of 8 is three times as much as 2.
\(\frac{3}{4}\) of 8 is 6.
Mary’s clock needs 6 batteries.

Remember that the word of often means to multiply.

Find each product.

Question 1.
4 × \(\frac{1}{2}\)
Answer:

Question 2.
\(\frac{3}{4}\) of 16
Answer:

Question 3.
24 × \(\frac{1}{8}\)
Answer:

Question 4.
\(\frac{4}{7}\) of 28
Answer:

Question 5.
\(\frac{4}{5}\) × 37
Answer:

Question 6.
\(\frac{7}{8}\) × 219
Answer:

Question 7.
Marco weighs 80 pounds. His bones make up about \(\frac{1}{5}\) of his body weight. How much do his bones weigh?
Answer:

Question 8.
Monica bought 12 gallons of paint. She used \(\frac{2}{3}\) of the paint to paint her house. How many gallons of paint did she use?
Answer:

Question 9.
A soccer coach gives each player \(\frac{1}{2}\) liter of water at halftime. If there are 11 players, how many liters does he need?
Answer:

Set C
pages 345-348

Find \(\frac{2}{3}\) × \(\frac{5}{6}\).
A drawing can show fraction multiplication. Start with a rectangle that has 3 rows and 6 columns. There are 18 sections in all.
Envision Math Common Core 5th Grade Answer Key Topic 8 Apply Understanding of Multiplication to Multiply Fractions 65.1
For \(\frac{2}{3}\), shade 2 rows.
For \(\frac{5}{6}\), shade 5 columns
Count the sections in the overlap.
10 of the 18 squares are in the overlap area.
so, \(\frac{5}{6}\) × \(\frac{2}{3}\) = \(\frac{10}{18}\) or \(\frac{5}{9}\).

Remember to use each denominator to make the grid.

Find each product. Use models to help.

Question 1.
\(\frac{2}{3}\) × \(\frac{3}{8}\)
Envision Math Common Core 5th Grade Answer Key Topic 8 Apply Understanding of Multiplication to Multiply Fractions 65.2
Answer:

Question 2.
\(\frac{1}{4}\) × \(\frac{3}{5}\)
Envision Math Common Core 5th Grade Answer Key Topic 8 Apply Understanding of Multiplication to Multiply Fractions 65.3
Answer:

Question 3.
\(\frac{1}{6}\) × \(\frac{1}{8}\)
Answer:

Question 4.
\(\frac{4}{7}\) × \(\frac{4}{7}\)
Answer:

Set D
pages 349-352
Find \(\frac{4}{5}\) × \(\frac{3}{4}\)
Multiply the numerators to find the numerator of the product. Multiply the denominators to find the denominator of the product.
\(\frac{4}{5}\) × \(\frac{3}{4}\) = \(\frac{4 \times 3}{5 \times 4}\) = \(\frac{12}{20}\) or \(\frac{3}{5}\)

Remember to multiply the numerators together and the denominators together.

Question 1.
\(\frac{6}{7}\) × \(\frac{1}{2}\)
Answer:

Question 2.
\(\frac{3}{8}\) × \(\frac{8}{3}\)
Answer:

Question 3.
\(\frac{2}{3}\) × \(\frac{1}{3}\)
Answer:

Question 4.
\(\frac{7}{8}\) × \(\frac{3}{2}\)
Answer:

Set E
pages 353-356
Find the area of a rectangle with length \(\frac{3}{2}\) and width \(\frac{1}{3}\).
Envision Math Common Core 5th Grade Answer Key Topic 8 Apply Understanding of Multiplication to Multiply Fractions 70.1
The rectangle of length \(\frac{3}{2}\) and width \(\frac{1}{3}\) is tiled with 3 rectangles of area \(\frac{1}{2 \times 3}\)
So, the area of the rectangle is \(\frac{3}{6}\) or \(\frac{1}{2}\) square unit.

Remember that a unit square can be used to help find areas of rectangles.

Find the area of a rectangle with the given dimensions.

Question 1.
Length: \(\frac{8}{5}\) units
Width: \(\frac{3}{4}\) unit
Answer:

Question 2.
Length: \(\frac{4}{3}\) units
Width: \(\frac{7}{10}\) unit
Answer:

Question 3.
Gabriel has a square canvas that measures \(\frac{5}{4}\) feet on each side. What is the area of Gabriel’s canvas?
Answer:

Set F
pages 357-360 Find 3\(\frac{1}{2}\) × 2\(\frac{7}{8}\)
Estimate: 3\(\frac{1}{2}\) × 2\(\frac{7}{8}\) is about 4 × 3 = 12.
Rename fractions, then multiply.
\(\frac{7}{2}\) × \(\frac{23}{8}\) = \(\frac{161}{16}\) = 10\(\frac{1}{16}\) The product 10\(\frac{1}{16}\) is close to the estimate, 12.
An area model can also represent the product of mixed numbers.
A rectangular field of crops is 4\(\frac{2}{3}\) miles by 2\(\frac{3}{4}\) miles. Calculate 4\(\frac{2}{3}\) × 2\(\frac{3}{4}\) to find the area.
Estimate: 4\(\frac{2}{3}\) × 2\(\frac{3}{4}\) is about 5 × 3 = 15.
Use an area model to find the partial products.
Envision Math Common Core 5th Grade Answer Key Topic 8 Apply Understanding of Multiplication to Multiply Fractions 62.5
Add the partial products.
8 + 1\(\frac{1}{3}\) + 3 + \(\frac{1}{2}\) =
8 + 1\(\frac{1}{6}\) + 3 + \(\frac{3}{6}\) =
8 + 3 + 1\(\frac{2}{6}\) + \(\frac{3}{6}\) = 12\(\frac{5}{6}\)
So, 4\(\frac{2}{3}\) × 2\(\frac{3}{4}\) = 12\(\frac{5}{6}\)
The area of the field is 12\(\frac{5}{6}\) square miles. The product is close to the estimate of 15, so the answer is reasonable.

Remember to compare your answer with your estimate.

Estimate. Then find each product.

Question 1.
2\(\frac{1}{3}\) × 4\(\frac{1}{5}\)
Answer:

Question 2.
4\(\frac{1}{2}\) × 6\(\frac{2}{3}\)
Answer:

Question 3.
3\(\frac{3}{5}\) × 2\(\frac{5}{7}\)
Answer:

Question 4.
14\(\frac{2}{7}\) × 4\(\frac{3}{10}\)
Answer:

Use the grid. Write the missing labels and find the product.

Question 5.
6\(\frac{2}{3}\) × 3\(\frac{3}{5}\)
Envision Math Common Core 5th Grade Answer Key Topic 8 Apply Understanding of Multiplication to Multiply Fractions 71.1
Answer:

Question 6.
2\(\frac{5}{12}\) × 3\(\frac{1}{3}\)
Envision Math Common Core 5th Grade Answer Key Topic 8 Apply Understanding of Multiplication to Multiply Fractions 71.20
Answer:

Set G
pages 361-364
Multiplication can be thought of as scaling, or resizing, a fraction.
Will the product of 4\(\frac{1}{2}\) × \(\frac{3}{4}\) be greater than or less than 4\(\frac{1}{2}\)? How can you tell without multiplying?
Since \(\frac{3}{4}\) < 1, 4\(\frac{1}{2}\) × \(\frac{3}{4}\) < 4\(\frac{1}{2}\) × 1
Will the product of 4\(\frac{1}{2}\) × 2\(\frac{1}{3}\) be greater than or less than 4\(\frac{1}{2}\)? How can you tell without multiplying?
Since 2\(\frac{1}{3}\) > 1, 4\(\frac{1}{2}\) × 2\(\frac{1}{3}\) > 4\(\frac{1}{2}\) × 1.
So, 4\(\frac{1}{2}\) × 2\(\frac{1}{3}\) will be greater than 4\(\frac{1}{2}\).

Remember that a fraction is equal to 1 if the numerator and denominator are the same.

Without multiplying, decide which symbol belongs in the box: <, >, or =

Question 1.
2\(\frac{1}{10}\) × \(\frac{3}{5}\) Envision Math Common Core 5th Grade Answer Key Topic 8 Apply Understanding of Multiplication to Multiply Fractions 56 2\(\frac{1}{10}\)
Answer:

Question 2.
\(\frac{3}{4}\) × \(\frac{5}{5}\) Envision Math Common Core 5th Grade Answer Key Topic 8 Apply Understanding of Multiplication to Multiply Fractions 56 \(\frac{3}{4}\)
Answer:

Question 3.
7\(\frac{1}{2}\) × 1\(\frac{1}{6}\) Envision Math Common Core 5th Grade Answer Key Topic 8 Apply Understanding of Multiplication to Multiply Fractions 56 7\(\frac{1}{2}\)
Answer:

Question 4.
\(\frac{8}{3}\) × \(\frac{9}{10}\) Envision Math Common Core 5th Grade Answer Key Topic 8 Apply Understanding of Multiplication to Multiply Fractions 56 \(\frac{8}{3}\)
Answer:

Order each set of numbers from least to greatest.

Question 5.
3\(\frac{1}{5}\), 3\(\frac{1}{5}\) × \(\frac{9}{10}\), 3\(\frac{1}{5}\) × 1\(\frac{1}{2}\)
Answer:

Question 6.
\(\frac{2}{3}\) × \(\frac{3}{4}\), \(\frac{2}{3}\) × \(\frac{5}{4}\), \(\frac{2}{3}\)
Answer:

Question 7.
2\(\frac{1}{3}\) × \(\frac{5}{5}\), 2\(\frac{1}{3}\) × \(\frac{6}{5}\), 2\(\frac{1}{3}\) × \(\frac{1}{5}\)
Answer:

Set H
pages 365-368
Think about these questions to help you make sense and persevere in solving them.

Thinking Habits
• What do I need to find?
• What do I know?
• What’s my plan for solving the problem?
• What else can I try if I get stuck?
• How can I check that my solution makes sense?
Envision Math Common Core 5th Grade Answer Key Topic 8 Apply Understanding of Multiplication to Multiply Fractions 75.6

Remember that the problem might have more than one step. Solve. Show your work.

Question 1.
John has 1\(\frac{1}{2}\) hours of homework each day from Monday through Thursday and 2\(\frac{3}{4}\) hours over the weekend. How much homework does John have in a week?
Answer:

Question 2.
Elle is buying new flooring for her kitchen and laundry room. She knows that the area of the kitchen is 132 square feet. The laundry room is 8\(\frac{1}{3}\) feet by 6\(\frac{3}{4}\) feet. What is the total area of the two rooms?
Answer:

Topic 8 Assessment Practice

Question 1.
Which rectangle has the greater area, a rectangle with length \(\frac{1}{12}\) foot and width \(\frac{3}{4}\) foot or a rectangle with length \(\frac{1}{16}\) foot and width foot?
Answer:

Question 2.
Alberto runs 3\(\frac{1}{2}\) miles each day.
Envision Math Common Core 5th Grade Answer Key Topic 8 Apply Understanding of Multiplication to Multiply Fractions 78.1
A. Write an equation using the variable n to model how far he runs in 7 days.
B. How far does he run in 7 days?
C. Explain how to estimate how far he would run in 11 days.
Answer:

Question 3.
Is the following equation true? Explain.
\(\frac{5}{6}\) × \(\frac{8}{4}\) = \(\frac{5}{3}\)
Answer:

Question 4.
Complete the equation. Show your work.
16 × \(\frac{5}{8}\) = ?
Answer:

Question 5.
Select all the Expressions that are equal to \(\frac{4}{7}\) × 6.
Envision Math Common Core 5th Grade Answer Key Topic 8 Apply Understanding of Multiplication to Multiply Fractions 79 4 ÷ 6 × 7 = \(\frac{14}{3}\)
Envision Math Common Core 5th Grade Answer Key Topic 8 Apply Understanding of Multiplication to Multiply Fractions 79 \(\frac{6}{7}\) × 4 = \(\frac{24}{7}\)
Envision Math Common Core 5th Grade Answer Key Topic 8 Apply Understanding of Multiplication to Multiply Fractions 79 6 ÷ 4 × 7 = 10\(\frac{1}{2}\)
Envision Math Common Core 5th Grade Answer Key Topic 8 Apply Understanding of Multiplication to Multiply Fractions 79 4 × 6 ÷ 7 = 3\(\frac{3}{7}\)
Envision Math Common Core 5th Grade Answer Key Topic 8 Apply Understanding of Multiplication to Multiply Fractions 79 7 ÷ 4 × 6 = \(\frac{21}{2}\)
Answer:

Question 6.
Tracy took a test that had 24 questions. She got \(\frac{5}{6}\) of the questions correct. How many questions did she answer correctly? Write an equation to model your work.
Answer:

Question 7.
Jenna ran 2\(\frac{7}{8}\) kilometers each day for a week. How far did she run in all? Give an estimate, then find the actual amount. Show your work.
Answer:

Question 8.
Eduardo has a recipe that uses \(\frac{2}{3}\) cup of flour for each batch. If he makes 4 batches, how many cups of flour will he need? How many cups of flour will he need in total if he makes 3 more batches? Write your answers as mixed numbers. Use the number line to help.
Envision Math Common Core 5th Grade Answer Key Topic 8 Apply Understanding of Multiplication to Multiply Fractions 80
Answer:

Question 9.
Complete the equation. Explain how you got your answer.
\(\frac{6}{7}\) × \(\frac{2}{5}\) = ?
Answer:

Question 10.
Ted and his friends are rolling out clay for art class. Ted rolled out his clay until it was 2 feet long. Noah rolled out his clay
\(\frac{3}{5}\) as long as Ted’s clay. Jeannine rolled out her clay until it was 1\(\frac{1}{2}\) times as long as Ted’s clay. Miles rolled out his clay \(\frac{5}{5}\) as long as Ted’s clay.
A. Without completing the multiplication, whose clay is longer than Ted’s clay? How can you tell?
B. Without completing the multiplication, whose clay is shorter than Ted’s clay? How can you tell?
C. Whose clay is the same length as Ted’s clay? How can you tell?
Answer:

Question 11.
Find the product of \(\frac{7}{8}\) × \(\frac{9}{10}\). Then write another product with the same answer.
Answer:

Question 12.
Which of the following is equal to \(\frac{4}{7}\) × \(\frac{11}{15}\)?
a. \(\frac{4 \times 7}{11 \times 15}\) = \(\frac{28}{165}\)
b. \(\frac{4 \times 15}{7 \times 11}\) = \(\frac{60}{77}\)
c. \(\frac{4 \times 11}{7 \times 15}\) = \(\frac{44}{105}\)
d. \(\frac{7 \times 15}{4 \times 11}\) = \(\frac{105}{44}\)
Answer:

Question 13.
Members of a landscaping company built a retaining wall. They used brick to make the top \(\frac{2}{3}\) of the wall.
Envision Math Common Core 5th Grade Answer Key Topic 8 Apply Understanding of Multiplication to Multiply Fractions 82.1
A. What is the height of the brick portion of the wall? Write an equation to model your work.
B. Estimate the area of the whole retaining wall.
C. What is the area of the whole retaining wall? Write an equation to show your work. Compare your answer to your estimate to see if your answer is reasonable.
Answer:

Question 14.
Tyler’s family rented 15 DVDs last month.
Envision Math Common Core 5th Grade Answer Key Topic 8 Apply Understanding of Multiplication to Multiply Fractions 83.1
A. Of the 15 DVDs, \(\frac{1}{5}\) were comedies. How many movies were comedies? Use the model to help you.
B. Of the 15 DVDs, \(\frac{3}{5}\) were comedies. How many movies were comedies? Use the model to help you.
C. What relationship do you notice between the number of comedies and the number of documentaries?
Answer:

Question 15.
Kristen and Niko buy a canvas for their art studio.
Envision Math Common Core 5th Grade Answer Key Topic 8 Apply Understanding of Multiplication to Multiply Fractions 84.1
A. Estimate the area of their canvas. Write an equation to model your work.
B. Find the actual area of their canvas. Write your answer as a mixed number.
C. Compare your answer to your estimate to see if your answer is reasonable.
Answer:

Topic 8 Performance Task

What’s for Dinner?
Branden and Ashley are making the casserole in the Tuna Casserole Recipe card.

Question 1.
Branden is trying to decide how much tuna casserole to make.
Part A
How many cups of tuna does Branden need to make 3 times the recipe? Draw a model to show how to solve.
Envision Math Common Core 5th Grade Answer Key Topic 8 Apply Understanding of Multiplication to Multiply Fractions 85.1
Part B
How many cups of shredded cheddar cheese does Branden need to make \(\frac{2}{3}\) of the recipe? Draw a model to show your work.
Part C
How many cups of fried onion rings does Branden need to make 2\(\frac{1}{2}\) times the recipe? Show how to use a model and partial products to multiply.
Envision Math Common Core 5th Grade Answer Key Topic 8 Apply Understanding of Multiplication to Multiply Fractions 90
Part D
How many cups of cooked macaroni does Branden need to make 2\(\frac{1}{2}\) times the recipe? Show how to rename fractions, then multiply.
Answer:

Question 2.
Ashley’s casserole dish is shown in Ashley’s Dish drawing.
Envision Math Common Core 5th Grade Answer Key Topic 8 Apply Understanding of Multiplication to Multiply Fractions 90.30
Part A
What is the area of the bottom of Ashley’s dish? Complete the model to show your work.
Envision Math Common Core 5th Grade Answer Key Topic 8 Apply Understanding of Multiplication to Multiply Fractions 90.31
Part B
Branden’s casserole dish is 1\(\frac{1}{2}\) times as wide and 1\(\frac{4}{5}\) times as long as Ashley’s dish. Is Branden’s dish longer or shorter than Ashley’s dish? Explain your reasoning.
Part C
What is the area of the bottom of Branden’s dish? Show your work.
Answer: